تغطية شاملة

قام علماء الفلك بقياس تركيز المادة المظلمة منذ حوالي 6 مليارات سنة

لقد رسم علماء الفلك خريطة للمادة المظلمة على أكبر نطاق تم رصده على الإطلاق. كشفت نتائج جديدة أن الكون يتكون من شبكة كونية معقدة من المادة المظلمة والمجرات تمتد لأكثر من مليار سنة ضوئية

ملاحظة توضح أن المادة المظلمة في الكون تنقسم على شكل شبكة من المناطق الكثيفة (باللون الأبيض) والفارغة (باللون الأسود). أكبر المناطق البيضاء هي بحجم البدر في السماء. الصورة: فان ويربيك وشركاء Hymans وCFHTLenS
ملاحظة توضح أن المادة المظلمة في الكون تنقسم على شكل شبكة من المناطق الكثيفة (باللون الأبيض) والفارغة (باللون الأسود). أكبر المناطق البيضاء هي بحجم البدر في السماء. الصورة: فان ويربيك وشركاء Hymans وCFHTLenS

لقد رسم علماء الفلك خريطة للمادة المظلمة على أكبر نطاق تم رصده على الإطلاق. كشفت نتائج جديدة أن الكون يتكون من شبكة كونية معقدة من المادة المظلمة والمجرات تمتد لأكثر من مليار سنة ضوئية.

قدمت الدكتورة كاثرين هايمز والبروفيسور لودبيك فان ويربيك من جامعة كولومبيا البريطانية في فانكوفر بكندا النتائج في 9 يناير في اجتماع الاتحاد الفلكي الأمريكي الذي عقد في أوستن، تكساس.

وتوصل فريق دولي من الباحثين بقيادة فان ويربيك وهايمانز إلى هذه البيانات من خلال تحليل صور 10 ملايين مجرة ​​في أربع مناطق منفصلة من السماء. وقاموا بدراسة توزيع الضوء المنبعث من هذه المجرات، التي تنحني وكأنها تمر عبر كتل ضخمة من المادة المظلمة أثناء رحلة الضوء إلى الأرض.

المشروع المعروف باسم مسح التلسكوب الفضائي بين كندا وفرنسا وهاواي (CFHTLenS). تم تجميع هذه الصور على مدار خمس سنوات باستخدام كاميرا MegaCam ذات مجال الرؤية الواسعة بدقة 340 ميجابكسل في تلسكوب CFHT في هاواي.

وتقع المجرات المشمولة في المسح على بعد 6 مليارات سنة ضوئية في المتوسط. انبعث الضوء الذي تم التقاطه واستخدامه في الدراسة عندما كان عمر الكون 6 مليارات سنة، أو حوالي نصف عمره الحالي.

وكانت استنتاجات الدراسة مشتبه بها في السابق بناء على عمليات المحاكاة الحاسوبية، ولكن كان من الصعب التحقق من ذلك بسبب الطبيعة غير المرئية للمادة المظلمة. وهذه هي اللمحة الكبيرة للمادة المظلمة على نطاق واسع والتي أثبتت وجود الشبكة الكونية في كل الاتجاهات.

يقول الدكتور توماس كيتشنج، مركز مجموعة العمل الكونية: "لقد أتاحت لنا خريطة المادة المظلمة رسم خريطة خلفية لما يزيد عن 75% من عمر الكون اليوم، إلى وقت كان فيه مختلفًا تمامًا. ومن خلال تطور الكون عبر العصور الكونية، سيقوم الفريق الموجود في إدنبرة بالتحقيق في كيفية وصول طاقة التنشيط إلى المكان الذي تهيمن فيه على الكون اليوم.

وعلى مدى الأشهر القليلة المقبلة، سنستخدم هذه المعلومات لرسم خريطة لتطور توسع الكون ومعرفة المزيد عن المادة المظلمة، سبب توسع الكون. سوف نقوم باختبار نظريات الجاذبية لتحديد ما إذا كانت نظرية النسبية العامة لأينشتاين صحيحة أم لا. وسوف نستخدم البيانات أيضًا لتحديد خصائص النيوترينوات، وهي الجسيمات الشبحية التي تتفاعل بشكل ضعيف جدًا مع المادة.

تم دعم البحث من قبل مجلس البحوث الأوروبي، ومجلس أبحاث العلوم الطبيعية والهندسة في كندا، والمعهد الكندي للأبحاث المتقدمة والمركز الكندي للبيانات الفلكية.

لإشعار الباحثين

 

 

تعليقات 1,188

  1. آر إتش،
    1) صحيح جدًا وجميل أيضًا 🙂
    2) جميلة، ولكن لا. للحصول على جميع النقاط العشر، سيتعين عليك بذل القليل من الجهد. عين وورد اسم الملك في سفر الملوك أكثر من مرة. بالمناسبة، ذكرت شاول، هذا هو اسم مملكة الأموات، ويبدو لي أن إعطاء هذا الاسم في الكتاب المقدس لم يكن من باب المحبة.
    في الواقع، إنها مضيعة رهيبة للوقت، ولكن ليس وقتك فقط، بل وقته قبل كل شيء.

  2. غراسياس اليوبيل الكبير، غراسياس.

    ر.ح.
    لقد انتهت هذه المقالة بالفعل إلى حد كبير، إلا إذا كنت ستنتظر ربع ساعة حتى يتم تحميل التعليقات. إذا كنت ترغب في الاستمرار، فقد وجدت مقالة هادئة وهادئة على حافة الكون، والتي لن تزعج أحدا وحيث يمكننا أن ننغمس في محتوى قلوبنا. وإذا أراد شخص آخر أن يتحرك أيضاً، فأنا أقترح أن تكون القاعدة أن من يتحرك يلتزم بالأمور فقط.

    https://www.hayadan.org.il/vlt-hubble-smash-record-for-eyeing-most-distant-galaxy-2310104/#comment-333491

  3. اليوبيل,
    1) "فقال له ما اسمك فقال يعقوب". (تكوين XNUMX: XNUMX)

    2) هل أنت متأكد من الملوك؟ لأنه جاء في تكوين XNUMX: XNUMX: "ومات شملة، وملك تحته شاول من شوارع النهر". هل هذا ما قصدته؟

    إسرائيل،
    1) معذرة ولكنك لا تقرأ حقًا ما أكتب. أشعر وكأنني أتحرك ذهابًا وإيابًا وفي كل مرة ترفضني بتصريح غير رسمي "ليس الإشعاع الكهرومغناطيسي". آسف، لكن مثال السوبر نوفا لا يشير إلى الضوء أو الإشعاع الكهرومغناطيسي على الإطلاق، بل إلى ساعات درجة الحرارة نفسها.

    2) عدا عن ذلك، فإنك تتجاهل بشكل حازم الضوء "المختفي" في السيارة حسب الموديل الخاص بك.

    3) لم تفهم سؤالي "ما الذي يبرد". تقيس ساعات درجة الحرارة الخاصة بك إشعاع الخلفية من الكون الذي يخرج من الانفجار الكبير ويصبح أكثر برودة. إذا أنكرت وجود الانفجار، فقد أنكرت أساس ساعات درجة الحرارة. وإلا فماذا يقيسون؟

    4) لا يقتصر تأثير دوبلر على الأنظمة المتسارعة فحسب، بل في الأنظمة التي تبتعد عن بعضها البعض، حتى ولو بسرعة ثابتة. والدليل أنهم تحدثوا لسنوات عن الكون المتوسع وفقًا لدوبلر ومؤخرًا فقط عن الكون المتسارع. لقد وقعت في فخ حجة "الدوبلر فقط في أنظمة المسرعات".

    5) بخصوص XNUMX-XNUMX، ماذا تريد؟ أنت تقول أنهم سوف يرون نفس الشيء. أنا ويكيبيديا وكل الفيزيائيين الذين درسوا استطالة الأقمار نقول "غير صحيح، كل واحد سيرى الآخر يتباطأ". ربما لن نعرف حتى نقوم بالتجربة. فلماذا تفرض نوعًا من الاتفاق هنا؟

    يرجى النظر في النقاط وعدم تجاهلها بشكل عرضي لأنني بخلاف ذلك أشعر أنها مضيعة رهيبة للوقت بالنسبة لي.

  4. يوفال
    جنوب لوس أنجلوس إلى لوس أنجلوس مثل جنوب فرنسا (المغرب) إلى فرنسا.

    سأل تسفي
    هل تجد عيبًا في حجتي القائلة بأنه إذا استخدمنا صيغة فريدمان لربط الوقت الذي انقضى منذ الانفجار الأعظم بدرجة الحرارة، فيمكننا بناء ساعة توضح الوقت الذي انقضى منذ الانفجار فقط عن طريق قياس درجة الحرارة؟ درجة حرارة الإشعاع الكوني؟

    تم العثور على الصيغة + الآلة الحاسبة التي تربط درجة الحرارة بالوقت في
    http://hyperphysics.phy-astr.gsu.edu/hbase/astro/expand.html#c3

  5. روبي

    تعود لنفس الخطأ الذي حاولت التحذير منه والتعامل مع تشبيه البالون على أنه الشيء الحقيقي ومن هنا تصل إلى سؤال ماذا يحدث في مركز البالون.
    تشبيه البالون هو تشبيه لا دور فيه لمركز البالون في الواقع! انها غير موجودة!

    سأشرح بمزيد من التفصيل كيف توصلوا إلى فكرة الانفجار الأعظم ومن ثم ربما ستفهم إلى أي مدى لن تتمكن من السؤال عن "مركز البالون".

    بعد أن نشر أينشتاين معادلة المجال للنسبية العامة، أخذ روسي اسمه فريدمان المعادلة وحاول أن يفهم منها ما كان يحدث للكون بأكمله. لقد وضع افتراضين فقط حول الكون، أهمهما أنه موحد في كل مكان وفي جميع الاتجاهات - أي أنه افترض أنه لا توجد نقطة مميزة أو مركز ثقب أسود أو أي شيء آخر.

    ومن هذا الافتراض حصل على ثلاث مصفوفات محتملة، أي ثلاثة حلول محتملة لمعادلة أينشتاين للمجال.
    لذا، هناك ثلاثة هياكل أساسية فقط يمكن أن يمتلكها الكون* بافتراض أن الكون موحد. كل هذه الهياكل الثلاثة تتطلب انفجارًا كبيرًا. وبما أن الافتراض هو أن الكون موحد على مقاييس كبيرة جداً، فهو افتراض معقول ومقبول جداً، وبما أنه تم اكتشاف أدلة على حدوث انفجار كبير -يبدو أن نظرية فريدمان قريبة جداً من الواقع- أي، يتصرف الكون على نطاق واسع وفقًا لأحد النماذج الثلاثة التي تنبأ بها فريدمان.
    المعنى هو أنه لا توجد نقطة مميزة (مركز الكون) - لأنه على وجه التحديد من هذا الافتراض ولد الانفجار الأعظم، وبالتالي ندعي الآن أنه بسبب وجود انفجار كبير فإن الكون له مركز، إنه ببساطة نقص أساسي في الفهم.

    لكي يشرحوا للناس كيف يمكن لشيء ما أن يتوسع بدون نقطة محددة، فإنهم يقدمون مثال النقاط الموجودة على سطح البالون - لكن هذا المثال ليس هو الشيء الحقيقي حقًا ولا يمكنك التعلم منه عن الشيء الذي من المفترض أن يمثله بدقة في الأشياء التي لا توضح الواقع.

    -----------------------------
    * اعتباراً من اليوم (منذ اكتشاف التوسع المتسارع للكون عام 1998) يبدو أن الكون لا يتصرف وفقاً لكون فريدمان بل وفقاً لنموذج آخر يسمى نموذج CDM. الخطأ ليس في حل فريدمان، بل في معادلة أينشتاين للمجال، ولهذا السبب فهي مثيرة للاهتمام. على أية حال، يتضمن نموذج الكون الحالي أيضًا انفجارًا كبيرًا مثل نماذج فريدمان المختلفة، وهو أيضًا نموذج موحد على نطاق واسع بما فيه الكفاية.
    ومن المثير للاهتمام أن نلاحظ أن معادلة المجال الأصلية لأينشتاين (التي تحتوي على ما يسمى بالثابت الكوني) والتي تم تصحيحها لاحقًا لتصبح المعادلة كما حلها فريدمان، هي في الواقع المعادلة الصحيحة في النهاية.

  6. ر.ح.
    لا أعتقد أنني تجاهلت أي شيء مما قلته. السبب وراء عدم إدراج إجابة لكل تحفظاتك بسيط: كل ما تطرحه، بدءًا من المجرات المتراجعة، لمصدر راديوي مثل غرينتش، وانتهاءً بالمستعر الأعظم، يتعامل مع القياس بالوسائل الكهرومغناطيسية، وهي في الواقع الضوء على ترددات مختلفة. ولذلك فهي غير مقبولة في مناقشاتنا.

    بالنسبة لسؤالك رقم 2:

    و. لا شيء يبرد. أو إذا برد، دون الاعتماد على صيغة دالة متصلة مثل صيغة فريدمان.

    ب. لقد تم إثبات إطالة الأوقات تجريبيًا فقط للأنظمة المتسارعة. اذهب لإجراء اختبار المؤين أو الدوبلر. ستجد أنهم على أنظمة متسارعة.

    ثالث. إذا لم يكن هناك ضجة، فإن تمديد الوقت منطقي، لكنه ليس إلزاميا. وهذا تفسير واحد فقط. انظر محاضرة سسكيند.

    رابع. التناقض موجود. في حياة R.H.، لماذا لا تكمل المثال 3-3 من افتراض عدم وجود فرق بين أنظمة القصور الذاتي؟ فإذا أثبت لي أنه لا يوجد تناقض بعد قيامك بذلك، فإنك تساوي وزنك ذهباً.

    وفي الحقيقة، لا أرى فائدة كبيرة من مواصلة النقاش دون التوصل إلى اتفاق حول هذه النقطة. إذا كنت تريد الاستمرار من حيث توقفنا، فاستمر. لكن إذا كنت تنوي الاستمرار في الادعاء بأن هناك فرقًا بين الأنظمة غير المتسارعة، وأن ساعات جيل ستظهر علاقة مختلفة عن ساعات جاك، وأن الكاميرات الموجودة في نفس النقطة وفي نفس اللحظة تلتقط صورًا معاكسة، إذن الأساس فقد غاب النقاش المنطقي بيننا، إلا إذا أوضحت ما تقصده عندما تدعي ذلك. وسأظل أعتقد أن هناك تناقضا.

    هناك نقطة إيجابية - إذا تابعت محاضرات سسكيند، ليس فقط عن النسبية، ولكن أيضًا عن الكميات والتشابك وعلم الكونيات والمزيد - فقد تتفق معي على أنك لست بحاجة للذهاب إلى الجامعة بعد الآن. ضع كل ما تعلمته في جامعة كاليفورنيا في جيبي الصغير.

  7. إسرائيل،

    لقد تجاهلت جزءًا كبيرًا من ادعاءاتي/أسئلتي.

    1) السيارة - من الممكن جعل مصدر الضوء يطلق عددًا محدودًا من الفوتونات كميًا. وقد تم ذلك في تجربة القمار الشهيرة. إذا فعلنا ذلك من سيارة متحركة، فيجب أن تختفي هذه الفوتونات، أو على الأقل جزء كبير منها، وفقًا للنموذج الخاص بك. هل هذا ما يحدث؟؟؟

    2) التصادم بين الانفجار الأعظم وامتداد الزمن. لا أفهم ما يدعيه. وإذا كان هناك تعارض بين هذين الأمرين، فهناك عدة خيارات:
    و. هناك تمديد للوقت ولكن لم يكن هناك ضجة. إذن ما هو التبريد؟ كيف يحدث أن الكون كله معًا يبرد. وما هي قيمة الساعات المؤقتة الخاصة بك إذا لم يكن هناك ضجة؟
    ب. كان هناك ضجة كبيرة ولكن لم يكن هناك تمديد للوقت. لقد تم إثبات إطالة الأوقات تجريبيا ولا يخضع للنقاش.
    ثالث. لا يوجد استطالة ولا يوجد انفجار - انظر A + B
    رابع. كلاهما صحيح وليس هناك تناقض – في إسرائيل، فكر في هذا الخيار مرة أخرى.

    بالطبع يمكنك القول أنه لا يوجد تعارض بين التمدد الزمني للأجسام في حالة التسارع، ولكن لا يوجد تمدد زمني للأجسام في حالة السكون، ولكن تم إثبات ذلك واختباره أيضًا من خلال تأثير دوبلر، وهو ما ترفض الحديث عنه عن.

    أخيرًا، لقد تجاهلت تشبيهي بين الانفجار الكبير والمستعر الأعظم. هل لديك إجابة لهذا؟ ألن يكون هناك تناقض بين تمدد الزمن وكل سوبر نوفا حدث على الإطلاق؟

  8. ر.ح.
    بخصوص حجزك
    أنا لا أستبعد الانفجار الكبير، لكني لا أرى كيف يحدث مع إطالة الزمن، أليس كذلك؟ لذا أنهي النمط الثالث والثالث. سؤال: لنفترض أنك تقبل أن يرى كل من جاك وجيل نسبة 1:1. وحتى ذلك الحين، ألا ترى مشكلة في تمديد الوقت؟

    وفيما يتعلق بالبندول الباليستي، فبالطبع سيكون للسيارة ضوء. تقوم أجهزة الكشف لدينا ببساطة بكشف الفوتون بسرعة واحدة فقط، تمامًا كما تكون أعيننا حساسة للضوء بترددات معينة وتعمى عن ترددات أخرى.

    في مسألة زرن. هناك تفسير لعدم قدرتنا على تسريع البروتونات فوق سرعة معينة، لكنه ليس الاتجاه السائد، لذلك لم أتطرق إليه. من الأفضل أن ننهي الأشياء الأخرى أولاً.

    لا أعرف إذا كنتم مهتمين بإعادة فتح الموضوع، ولكن إذا كان الأمر كذلك، فلا أرى أي فائدة في الاستمرار فيه قبل أن نتوصل إلى اتفاق بشأن القضية الرئيسية، أي تمديد الوقت، وبالتالي 3-3.

    طلب واحد فقط: لقد أثنت عليك لاستيعاب التفاصيل بسرعة. لكن من فضلك، إذا واصلنا، ألا نكتشف بعد شهر أننا نتحدث عن أشياء مختلفة، كما حدث مع ساعات جيل، التي ظننت أنني أوافق على أنها تظهر نسبة مختلفة عن 1:1، بينما أنا بوضوح كتب أن هذا ليس هو الحال.

    وأود أيضًا أن أقترح عليك الاستماع إلى محاضرات سسكيند الممتازة حول هذا الموضوع، كما هو الحال في أي موضوع آخر.

    http://www.youtube.com/watch?v=BAurgxtOdxY

    لاحظ أيضًا أنه في الدقيقة 19 أثار أحد المستمعين سؤالًا مشابهًا لسؤالي، وأن سسكيند يتداخل معه نوعًا ما.

  9. روبي
    أرى أنك لم تفهم ما كتبت.

    انت كتبت:
    "تجاهل النقطة المفردة وأرجع إلى غلاف النقطة، أي بضع ثوانٍ من الانتشار من هناك وبعد ذلك سيكون لدينا إمكانية محاولة فهم ما يحدث هناك في المركز". - ادعاء غير واضح.

    و أيضا:
    "أتساءل لماذا يشير جميع الخبراء إلى نموذج البالون المتضخم للكون والذي ينبع من الملاحظات الفلكية بالطبع ومعادلات أينشتاين للنسبية الخاصة والعامة، لكن لا أحد يحاول معالجة ما يحدث في مركز البالون". - هنا أيضًا ليس من الواضح بالنسبة لي ما الذي تحاول المطالبة به. ماذا تقصد بقولك: "لا أحد يحاول الاهتمام بما يحدث في وسط البالون."؟

  10. ر.ح.
    أنا على دراية بمفهوم النقطة المفردة وقد أتيحت لي الفرصة للتساؤل عن الجرة في دراستي الهندسية في الماضي.
    ليست هذه هي القضية، أتساءل لماذا يشير جميع الخبراء إلى نموذج البالون المتضخم للكون والذي ينبع من الأرصاد الفلكية بالطبع ومعادلات النسبية الخاصة والعامة لأينشتاين ولكن لا أحد يحاول معالجة ما يحدث في مركز البالون .
    وفقًا لمعرفتنا الفيزيائية وملاحظاتنا عن المستعرات العظمى، فإننا نعلم أن التوسع الخارجي للطاقة والضوء والمادة يتطلب أيضًا انكماشًا داخليًا مطابقًا في الشدة ومعاكسًا له عمدًا.
    تجاهل النقطة المفردة وراجع غلاف النقطة، أي بضع ثوانٍ من الانتشار من هناك، ومن ثم سيكون لدينا إمكانية محاولة فهم ما يحدث هناك في المركز.

  11. روبي
    https://www.hayadan.org.il/astronomers-reach-new-frontiers-of-dark-matter-130112/#comment-333377

    "يتم تجاهل مركز النقطة المفردة" - النقطة المفردة ليس لها مركز. النقطة المفردة تمثل المركز نفسه. المشكلة هي في مركز النقطة. يمكن القول أن أي نقطة ستكون مركز الدائرة، وسيكون لمركز الدائرة أيضًا مركز. بعد كل شيء، عندما تغرز الإبرة لرسم دائرة، فإن المكان الذي تعلق فيه الإبرة (وهو مركز الدائرة - أي من هذه النقطة فصاعدًا يتم قياس قطر الدائرة) له أيضًا مكان مركز. إذا قمت برسم دائرة باستخدام الفرجار على الصفحة، فسيكون مركز مركز الدائرة عبارة عن ثقب في الصفحة نفسها، على سبيل المثال.
    في الرياضيات (وفقًا لويكيبيديا) "النقطة المفردة هي النقطة التي تكون فيها الدالة (عادةً دالة معقدة) أو المعادلة التفاضلية غير محددة بشكل جيد."
    في عالم مادي مثل عالمنا، النقطة المفردة لها معنى آخر. لقد جاء لوصف النظام الذي بدأ منه كل شيء. وفقا لنظرية الانفجار الأعظم (أو بالأحرى من المعادلات التي تنشأ عنها) يجب أن تكون نقطة المفرد أصغر من طول بلانك. أي أن كل المادة والفيزياء وكل شيء موجود خلق من شيء طوله أقل من طول بلانك. بمعنى آخر، يمكنك القول إن الجسيم الأول خُلق من شيء ما، وهو أقصر منه ووزنه أقل منه، ولا بد من وجود مكان تكون فيه الإنتروبيا في أعلى مستوياتها في اللحظة الحرجة لبداية الحدث الكبير. الانفجار (الفوضى الكاملة للنظام بسبب طبيعة سلوك الجزيئات التي تشكل النظام والتي تكون عند درجة الحرارة الأعلى).

    النقطة رقم 1 هي عملية تحدث في الفضاء والتي تخلق مركز (الدائرة) "الكرة" التي تسمى "الكون".
    بعد تطبيق مركز الدائرة، سيكون من الممكن حساب قطرها. (أو في الحالة المعنية قطر {وحجم} الكون).
    في الرياضيات ليس لهذا المركز أي معنى لأن هذا المركز (النقطة المفردة) بحكم التعريف لا يمكن قياسه (في الواقع تقرر أن نقطة المفرد هي حالة إشكالية في كل ما يتعلق بالتعريف وبالتالي لا يمكن تحديدها رياضيا {النتائج ، في المعادلات، لا تضيف إلى العالم المادي.الجسم الذي يشكل مركز الدائرة وهو الكون - غير محدد رياضيا}).
    بحسب ويكيبيديا الإنجليزية:
    "على سبيل المثال، الدالة: f(x)=1/x
    على الخط الحقيقي له تفرد عند x = 0، حيث يبدو أنه "ينفجر" إلى ±∞ ولم يتم تعريفه. الدالة g(x) = |x| (انظر القيمة المطلقة) له أيضًا تفرد عند x = 0، نظرًا لأنه غير قابل للاشتقاق هناك. وبالمثل، فإن الرسم البياني المحدد بواسطة y2 = x له أيضًا تفرد عند (0,0)، هذه المرة لأنه يحتوي على "زاوية" (ظل عمودي) عند تلك النقطة."
    لاحظ أن النقطة H هي "مجموعة مغلقة".
    أي أنه من مجموعة مغلقة تنشأ مجموعة مفتوحة (وهي "الكون") (كما في فعل التابع في نظام الفانو).
    بعد كل شيء، وفقا للانفجار، من المجموعة المغلقة وغير المحددة تم إنشاء التورم (الذي نما بشكل كبير) للمساحة نفسها، أي أن إحداثيات المساحة المادية تلقت قيمًا أكبر من 0.
    إذا فهمت هذا الحد، يمكنني الاستمرار. ومن المهم أن نفهم شيئًا آخر، وهو أن النقطة المفردة لا تختفي بعد الانفجار الأعظم. إنها لا تزال مرنة وموجودة (بفضل قوانين الديناميكا الحرارية).

  12. يوفال، إجابة لطيفة دوز بوا!

    إسرائيل،
    منذ أن طرحت موضوع الانفجار الكبير، جاء سقراط راكضًا مضطربًا:

    1) إذا استبعدت الانفجار فإنك تتجاهل الأرض تحت فكرة الساعات المؤقتة الخاصة بك. لأنه إذا لم يكن هناك فرقعة، فما هو التبريد الذي يقيسونه؟ الافتراض في قاعدة الساعات هو أن التبريد منتظم في كل مكان وفي أي مكان في الكون لأنه كان هناك انفجار. إذا لم يكن هناك، فإن ما يتم قياسه هو تقلبات درجات الحرارة المحلية وفكرة الساعة بأكملها لا قيمة لها.

    2) يقترح سقراط التجربة التالية: سيكتشف جاك وجيل مستعرًا أعظم ويضعان نفسيهما هناك على الفور. ماذا سيكون هناك سحابة غازية تزداد برودة وبرودة، عجبا وعجبا وفقا لصيغة فريدمان. سيخرج جاك وجيل ساعاتهما ويبدأان في القياس والتقاط الصور لمقارنتها بساعات السيزيوم وكل تلك الأشياء التي ناقشناها من قبل.
    ولكن بعد ذلك سيظهر السيد إسرائيل شابيرا فجأة ويقول لهم: "يا شباب، لقد تخيلتم المستعر الأعظم لأن وجوده يتعارض مع النسبية وإطالة الزمن. لا يمكن أن تكون قد حدثت!"

    ويسأل سقراط هل ترى التناقض؟

  13. آر إتش،
    في سفر صموئيل 2، الفصل 11، الآية 3، يقال أن بثشبع هي ابنة أليعام. جاء في الإصحاح 23: 34 أن أليعام هو ابن أخيتفل. إذا كان هذا هو إليام نفسه، فإن جد باتشيفا لأب هو أهيتفل. من ناحية أخرى، يدعوها كتاب أخبار الأيام بات شوا بات أميل.
    وأقترح عدم الاستمرار في تغذية نار النقاش الدؤوبة هذه. نحن فقط نسبب له الأذى ونضيع وقته. يجب عليه أن يتعمق في أفكاره مع الباحثين المحترفين في الأكاديمية وليس مع الأشخاص العاديين هنا.

  14. إسرائيل،
    وهناك شيئ اخر. وفقًا لنموذج البندول الباليستي الخاص بك، يجب ألا يكون للسيارة المتحركة أي ضوء على الإطلاق. أنت تدعي أننا لا نستطيع أن نشعر بالضوء إلا بسرعة C وأن سرعة الضوء نسبية. إذا كان الأمر كذلك، فإن الضوء الخارج من السيارة بسرعة C+100 كم/ساعة يجب أن يكون قد اختفى. بالإضافة إلى ذلك، لم تقدم أبدًا تفسيرات معقولة حول سبب استحالة بناء كاشف "يرى" الضوء الأسرع. على سبيل المثال، شاشة سميكة للغاية. البندول الثقيل في تشبيهك. لم تشرح سبب عدم رؤيتك للضوء البطيء من لغة C، وهو ما يتطلبه النموذج الخاص بك. لقد رفضته باعتباره مثيرًا للاهتمام ولكنه غير ذي صلة.

  15. إسرائيل،
    ويؤسفني أنه بعد كل الكلام الذي سكب وكل الملخصات، فإنك لا ترى جوهر الجدل بيننا إلا في النقطة التي ذكرتها.
    وجوهر الحجة في نظري هو أنك فشلت في إظهار أي تناقض جديد لم يتم مناقشته من قبل ولم يتم تقديم الحلول له. في الواقع ما أدركته في النهاية هو أنك تتحدث ببساطة عن مفارقة التوأم التي تم بحثها وإثباتها ومناقشتها مرات لا تحصى. لقد حاولت تجنب ذلك بالتلويح بيديك بأن المفارقة تنطبق فقط على الأنظمة غير المتسارعة، وهذا ليس صحيحًا تمامًا. لقد تجاهلت لسبب غير معروف البراهين المستندة إلى تأثير دوبلر على الرغم من أنها لا تتعلق فقط بالأنظمة المتسارعة، وبالنسبة للحلوى قررت تجاهل جميع الجسيمات ذات الكتلة أيضًا. لماذا؟ لأنهم لا يناسبونك.

    أفشالوم، أعتقد أن هذا يدور حول رجل محبط تعرضت أخته للاغتصاب بوحشية ورأى في العرائس الفقيرات أن والده لا يتخذ أي إجراء عقابي ضد المجرم. ثم عندما ينتقم، فهو الذي عليه أن يهرب للنجاة بحياته. وعندما عاد قوبل بالتجاهل الصارخ الذي ربما زاد من الاستياء الذي يشعر به تجاه والده لدرجة أنه اندلع في التمرد.
    اللغز: من هو جد باتشيفا؟

  16. يوفال

    "تدهشني في كل مرة مرة أخرى موهبتك في جذب الناس إلى مناظرات لا تترك في أفواههم طعما لا أكثر"

    أنا أيضًا مندهش جدًا من موهبتك في قراءة ربع الجملة واستخلاص النتائج على الفور، وعادةً ما تكون خاطئة.

    على سبيل المثال: يبدو أنك تستنتج أنني مهتم بمواصلة النقاش، ويبدو أنك تستنتج أنني أسير في اتجاه ما. وبما أنني الوحيد الذي يعرف نواياي، فأنا أستنتج أنك مخطئ. كل عادة.

    "إن ساعات درجة الحرارة هي إبداعات النظرية والخيال. من المفيد والمرغوب أن نبنيها، ولكن طالما لم يكن هناك شيء، فمن العار أن نضيع الكلمات".

    لو أنك كلفت نفسك عناء قراءة المناقشة التي دارت بيني وبين R.H.، لوجدت أنها منظمة على شكل مستويات، يجب أن يتفق الطرفان على كل منها. لقد اتفقنا على إمكانية بناء ساعات درجة الحرارة وتركيبها أينما تريد في مرحلة مبكرة جدًا. قد لا توافقون على هذا الأمر، كما هو الحال في أي قضية أخرى، لكن لا يهم المناقشة الفعلية، التي لم تكلفوا أنفسكم عناء المشاركة فيها، ولم تثروا الاعتراضات المذكورة أعلاه، وبالتالي لم تتلقوا إجابة عن السبب. هذا ممكن. (ولماذا لا بالمناسبة؟ إذا كان الوقت مرتبطًا بدرجة الحرارة باستخدام صيغة فريدمان ويمكن قياس درجة الحرارة، فلماذا لا يمكن ربطهما بساعة درجة الحرارة؟).

    "إنك تشهد أنك لا تفهم "كيف يمكن الحديث عن بداية الزمن، عما كان قبله"، ولكن بين سوء الفهم والنفي مسافة كبيرة".

    كالعادة، لم تقرأ أو لم تفهم. من نهب؟ كل ما قلته خلال المناقشة بأكملها هو أن إطالة الزمن في العلاقات لا يتوافق مع الزمن المطلق للانفجار الكبير. هذا. لأن س.ح. وخلص إلى أنني قد أنكر النسبية، وذكرت أنني شخصيًا أفضلها على الانفجار، لكنني لم أنكر أيًا منهما، لقد أشرت فقط إلى ما أسميته "التناقض المفترض".

    في صلب الموضوع: يعتقد البعض، وربما يكونون على حق، أن العدالة ليست مهمة على الطريق، في السياسة وفي الحب. أعتقد أنه من المهم في الرياضيات والفيزياء أن تكون على صواب، وأن عكس الصواب هو الخطأ.

    النقاش بيني وبين R.H. لقد علقنا في السؤال التالي: R.H. يدعي أن هناك نظامًا بالقصور الذاتي يمكننا أن نقول إنه حقًا "في حالة سكون" - في مثالنا جاك، الذي تبلغ نسبة ساعته 1:1، وأن هناك نظامًا بالقصور الذاتي "يتحرك" حقًا - في مثالنا جيل، الذي تبلغ نسبة ساعته مليار إلى واحد.

    إذا كان ر. بقبول افتراضاتي، يبدو لي أن التناقض بين افتراض إطالة الوقت والوقت المطلق للساعات المؤقتة كان واضحًا تمامًا (إذا كانت النسبة 1:1 في كلا النظامين، فإن الساعات cc والساعات المؤقتة في يظهر النظام المعطى دائمًا نفس الوقت، وبما أن الساعات المؤقتة تظهر نفس الوقت في وقت الاجتماع، فإن جميع الساعات الأربع تظهر نفس الوقت، على عكس إطالة الوقت التي تظهر فيها ساعات تشيكوسلوفاكيا مختلفة مرات).

    وبما أنني أشعر بأنني على أرض صلبة تمامًا عندما أذكر أنه لا يمكن التمييز بين أنظمة القصور الذاتي (ربما باستثناء الإشارة إلى نظام CMBR، الذي لم نشر إليه في المناقشة، ولا أينشتاين في عام 1905 عندما لم يكن يعلم حتى بوجوده) الوجود) ليس لدي خيار سوى أن أستنتج أن R.H. ولم يشر إلى أي خطأ مقبول في تلك المناقشة الطويلة.

    وفيما يتعلق بتحفظه على التناقض المفترض مع معجلات الجسيمات في تسيرن، هل يمكن أن تشرح مسألة زيادة كتلة الأجسام بسرعة حسب نموذجك؟ وفقًا لنموذج الموقع النشط، فإن الإجابة واضحة تمامًا، على الرغم من أنني لن أكون مهتمًا بفتح جبهة جديدة دون أن نتمكن من حل المشكلة الأبسط المتمثلة في تمديد الوقت.

    وفي مسألة داود وأبشالوم – في رأيي، كلاهما اكتشفا بعد فوات الأوان عدم جدوى تفكيك الأسرة على خلفية الحكم والكرامة. ربما فكر أبشالوم بهذا عندما عُلِّق بين أغصان الإلهة، وداود عندما ترك القصر طوعًا من أجل زوجاته الخائنات.

    أبشالوم، أبشالوم أبناء أبشالوم، دعني أموت اليوم في ظلكم. يا أبنائي، أبنائي أبشالوم.

  17. يوفال، يبدو لي أن الخفي أكثر من الظاهر.
    سأحاول البحث عن إجابات وإذا كان هناك أي شيء لتحديثه، فسوف أقوم بتحديث المنتدى.
    شكرا لمحاولة التوضيح

  18. روبي,
    ليس لدي الكثير لأضيفه إلى كلمات R. H. وZvi، لكني سأحاول تبسيط الأمور. يقدم المدافعون عن نظرية الانفجار الكبير صورة مفادها أن الكون بأكمله بدأ بنقطة مجهرية. والانطباع الناتج هو كما لو أن الكون عبارة عن مساحة كبيرة فارغة ظهرت فيها نقطة واحدة وبدأت في التوسع. لكن هذا الوصف ليس بالضرورة صحيحا، لأن هذه النقطة ليست نقطة واحدة في فضاء لا نهائي، بل هي في حد ذاتها الكون كله. وبما أننا لا نعرف أكوانًا أخرى يمكن مقارنتها بالكون الذي نعيش فيه، إذا صح التعبير، فإن هذا الكون الكبير بأكمله لا يزال هو نفس النقطة المجهرية. يجوز القول، بل وربما يكون صحيحاً ودقيقاً، إن الكون لا يتوسع بل ينقسم. تم تقسيم الكون النقطي إلى قسمين (على غرار تقسيم الخلية الحية في علم الأحياء)، كما تم تقسيم كل جزء أيضًا إلى قسمين ويتكرر إلى ما لا نهاية. وهكذا يبقى الكون بحجم نقطة، ولكنه من وجهة نظر مكوناته الداخلية ينمو ويتوسع دون توقف. ومن هذا الوصف (وربما تكون هناك أوصاف أخرى أيضًا) يترتب على ذلك أنه لا يوجد معنى لنقطة مفردة.

  19. روبي,

    "هذا بالضبط ما يزعجني، أن الكون ليس له مركز. إذا بدأ كل شيء من نقطة واحدة مركزة بشكل لا نهائي، فليس من المنطقي أن يخرج كل شيء (إلى غشاء بالون واحد) دون أن يدخل جزء ما إلى مركز الثقب الأسود.

    لماذا يزعجك؟ هل للثقب الأسود حركة في اتجاهين؟ ولماذا لا تكون نقطة واحدة تتزايد ولا يكون هناك تقارب داخلي؟ نوع من معكوس الثقب الأسود؟

  20. زفي، فكرة انتشار البالون الكوني معروفة وشعبية جداً، لو أخذنا الوضع الحالي ورجعنا بالزمن إلى الوراء، سنحصل على نقطة واحدة في المركز ومن هنا جاء افتراض حدوث انفجار كبير من.

    إن افتراض أن الكون موحد لا يتوافق مع نظرية الكون / بالون قابل للنفخ يعني غلاف قابل للنفخ بدون مادة داخل البالون.

    في رأيي المتواضع أن نظام المحاور البالونية الذي ينمو/يتسارع مع الزمن لا يعطي تفسيرا كافيا لسؤال لماذا لا يوجد ثقب أسود في مركز الكون (أصل المحاور).

  21. روبي ويوفال,

    الكون ليس له "مركز" لمعرفة ما يحدث فيه.

    ولدت فكرة الانفجار الكبير من أعمال فريدمان الذي كان يبحث عن حلول لمعادلات أينشتاين بناء على افتراض أن الكون متجانس ومتناحي الخواص - أي الافتراض الذي أدى إلى فكرة الانفجار الكبير الانفجار في المقام الأول هو أنه لا توجد نقطة فريدة في الكون وأن الكون بأكمله على نطاقات كبيرة بما فيه الكفاية يكون متجانسًا إلى حد ما.

    إن مسألة ما هي النقطة التي بدأ فيها كل شيء تفترض وجود نوع من الجسم الذي يتوسع في الفضاء الإقليدي، وتتساءل عما يحدث في الإحداثي حيث بدأ كل شيء - ولكن في النسبية العامة الأمر ببساطة ليس هكذا - ما الذي تم إنشاؤه في الانفجار الكبير هو، إذا صح التعبير، نظام الإحداثيات.

    والقياس المقبول لذلك هو كما يلي:
    تخيل بالونًا يستمر في التضخم، والمجرات عبارة عن نقاط مرسومة على سطحه.
    تبتعد المجرات أكثر فأكثر عن بعضها البعض بسرعة تزداد بازدياد بعدها عن بعضها البعض، ومع ذلك لا يمكنك معرفة أي نقطة تقع في المركز - كل النقاط مرسومة على البالون ومن حيث كل منها إنها ثابتة وكل الآخرين يتحركون.
    الواقع بالطبع أكثر تعقيدًا بعض الشيء من القياس، لأنه في القياس يكون الانحناء موجبًا (مجموع زوايا المثلث الموجود على سطح البالون أكبر من 180 درجة) بينما الواقع بعيد عنك يمكن أن نرى أنها مسطحة. كما أنه لا يوجد شيء واقعي يتوافق مع وجه البالون، ومع ذلك آمل أن يساهم التشبيه بشيء ما في الفهم.

  22. ر.ح.
    وهذا بالضبط ما يزعجني، وهو أن الكون ليس له مركز. إذا بدأ كل شيء من نقطة واحدة مركزة بشكل لا نهائي، فليس من المنطقي أن يخرج كل شيء (إلى غشاء بالون واحد) دون أن يدخل جزء ما داخل مركز الثقب الأسود.

  23. روبي,

    تقول نظرية الانفجار الكبير الحالية أنه لا يوجد مركز للكون وأنه من المستحيل معرفة مكان نقطة المفرد. تخيل أنك نقطة مرسومة على بالون بدون فوهة تستمر في النفخ. ما ستراه هو أن جميع النقاط الأخرى المرسومة على البالون تتحرك أكثر فأكثر بعيدًا عن بعضها البعض وعنك. لا يوجد معنى لسؤال أين بدأ التضخم أو أين كان البالون عندما كان في حد ذاته نقطة.

    والآن تخيلها بثلاثة أبعاد وهذا ما تدعيه النظرية.

  24. يوفال، ليس لدي الكثير لأتوسع فيه باستثناء أن نظرية الانفجار الأعظم لن تكتمل دون الرجوع إلى ما يحدث في مركز الكون. ليس من المنطقي بالنسبة لي أن تنتشر المادة إلى الخارج فقط دون أن تتقارب إلى الداخل نحو المركز. لم أر أي مادة حول هذا الموضوع حتى الآن. (لا أستطيع أن أصدق أنه لم يفكر أحد في هذا من قبل).
    وربما يكون هذا أيضًا موضوعًا لرسالتي في الدراسات الإضافية في الجامعة...

  25. يوفال، يمكن أن يكون التقارب الداخلي بعيدًا عن الجاذبية أيضًا بسبب القوى المتعارضة (قانون نيوتن الثاني) التي تسببت في التوسع. أفهم أنه قبل الانفجار لم يكن هناك وقت ولا قوانين فيزيائية، ولكن بعد ثوانٍ يمكننا التحدث عن القوى والسرعات/التسارع والزمن.
    نتحدث دائمًا عن المجرات التي تتوسع وتتسارع على مسافات تبلغ مليارات السنين الضوئية، لكننا نتجاهل مركز النقطة المفردة التي بدأ منها كل شيء. ماذا يحدث هناك؟ الثقب الأسود؟ ربيع الأكوان الأخرى؟ هل لديك رابط لهذا الموضوع؟

  26. روبي,
    أفترض أنه عندما تقول "تتقارب في نفس الوقت أيضًا إلى الداخل كما هو الحال في المستعرات العظمى" فإنك تقصد أن منتجات الانفجار تعود وتتقارب نتيجة لقوة الجاذبية المؤثرة بين الكتل.
    الارتباط الذي تم الحصول عليه من اسم "الانفجار الكبير" يمكن أن يكون مضللاً. ولا تتحدث النظرية عن انفجار مثل انفجار المادة المتفجرة، بل عن كيفية خلق كل المادة في الكون وخلق قوانين الفيزياء. على سبيل المثال، لم تكن المادة، في اللحظات الأولى بعد الانفجار الأعظم، هي المادة التي تعرفها الفيزياء اليوم. الجاذبية أيضًا لم تكن موجودة في ذلك الوقت ولكنها خلقت في وقت معين بعد تلك اللحظة.
    وفي ضوء ملاحظات الطيف القادمة من المجرات البعيدة واعتماد التفسير على تأثير دوبلر، يفترض معظم علماء الفيزياء أن الكون لا يتوسع فحسب، بل يتسارع أيضًا. لا تعترف الفيزياء بالتسارع دون استثمار الطاقة، ولكن بما أنه لا يوجد مصدر للطاقة مرئي، فقد سميت هذه الظاهرة "الطاقة المظلمة". لذلك، قبل أن ننتقل للبحث عن مقارنة بين الانفجار الكبير والمستعرات الأعظم، فإننا أمام ألغاز سابقة.
    إذا فهمت بشكل صحيح، فإن القياس على قشر البصل في مكانه الصحيح. وبما أن الكون لا ينتشر فقط على مساحة معينة بل على حجم، فهو ليس مجرد بالون واحد بل العديد من البالونات (أو "قشر البصل") الواحدة داخل الأخرى

  27. يوفال، يبدو لي أننا ابتعدنا كثيرًا...
    يوفال، والدي، لم أتلق ردًا على نظريتي حول قشر البصل. وباعتباري مهتما بالموضوع ومطلعا على كافة البرامج العلمية التي تتناول علم الفلك (الكون واستكشاف الأرض وغيرها)، لم أفهم لماذا تفترض نظرية الانفجار الكبير أن نفس النقطة الصغيرة التي ينتشر منها الكون فقط انتشرت إلى الخارج ولم تتقارب في نفس الوقت إلى الداخل كما هو الحال في المستعرات العظمى؟
    هل لدى أحد إجابة؟

  28. روبي، بعد إذنك، سأحاول التوسع في كلامك فيما يتعلق بالأساطير السومرية (مع الاعتذار لوالدي - إن اهتمامي بالنماذج الأسطورية المختلفة لخلق العالم ليس له علاقة بإيماني الشخصي):
    "قبل أن تكون هناك بداية للكون" والظلام (ترجمة من كلمة صفر، الإله صفر. يقدم سفر التكوين تلاعبًا بالكلمات: يتم استخدام كلمة الظلام بمعنيين: "فارغ" و"ظلام" ) فوق ثام (اللانهاية)، هذا لغز من الأساطير. النموذج الميتافيزيقي الذي استعارت منه الأساطير السومرية يتحدث عن صفر وصفر، صفر في اللانهاية (الظلام فوق الهاوية) التي شكلت الكون البدائي. أقدر أن المادة المظلمة التي نتحدث عنها اليوم هي نتيجة عملية ما وراء الرياضيات من هذا النوع (صفر مقسوم على صفر أو صفر مضروب في اللانهاية) وهي تملأ مساحة الكون بأكملها. النموذج السومري يضرب بدقة تامة في تقديري.
    كانت الأرض مكونة من الصخرتين Tehu و Bohu اللتين ابتلعهما الوحش Tehu قبل أن يتمكن تحالف الآلهة من تحييده. لقد حررهم رئيس تحالف الآلهة (مردوخ) من رحم الهاوية وخلق الأرض منهم. هذه القصة مأخوذة من نموذج ميتافيزيقي آخر، والذي لاقى رواجاً كبيراً، لكنني لست مهتماً به في الوقت الحالي.
    ما هو موجود في سفر التكوين وليس في الأساطير السومرية هو خلق النور. تم العثور على نصوص أساطير سومرية يعود تاريخها إلى الألف السادس قبل الميلاد. وقد كتبت التوراة بعدهم بآلاف السنين. إذا بدأنا من افتراض له ما يبرره إلى حد ما بأن التوراة خضعت لعملية إعادة كتابة وتحرير واسعة النطاق في أيام الملكين حزقيا ويوشيا، بينما ازدهرت فلسفة ما قبل سقراط في الجزر الأيونية، فيمكننا أن نجد توازيًا بين خلق النور والخلق. أفكار هيراقليطس: الطاقة (الضوء) والماسا (المادة المظلمة) وجهان لشيء واحد كما تدعي الفيزياء الحديثة أيضًا.

  29. أبي، من قال أنهم يستخدمون الهواتف اللاسلكية؟
    أنا لست ممثل التوراة ولكن كما تعلمون كل ما نعرفه عن الكون يتم بمساعدة الملاحظات واستخدام حواسنا المعروفة وبمساعدة الأجهزة التي تلتقط معلومات إضافية تتجاوز حواسنا مثل الكهرومغناطيسية المجالات والأطوال الموجية المختلفة للإشعاع والموجات الصوتية.
    في تقديري، هناك الكثير من المعلومات التي لا تتعرف عليها التكنولوجيا الحالية، وفي المستقبل ربما سنكشف عن جزء صغير من هذه المعلومات.
    لذا علينا أن نكون متواضعين قليلاً وألا نعتقد أننا نعرف كل شيء...

  30. روبي، وعلى نفس المنوال، كيف لنا أن نعرف أنه قبل ألفي سنة مضت كانت الهواتف اللاسلكية تستخدم؟ ولم يتم العثور على أي كابل في الأرض منذ ذلك الوقت. هذا يتعلق بمستوى إجابتك

  31. ربما الظلام فوق الهاوية؟
    ربما جيش النجوم ومساراتهم؟
    كما تعلمون، التوراة مليئة بالقرائن... فهي مرتبطة بالفعل بالسحر والتنجيم.

  32. كيف سيقول شموليك روزين عند سماع لغزك:
    ابنًا عاصيًا ومعلمًا، احتفظ برأسه حيث يمكن العثور على لسان حاييم.

  33. RH بالضبط 🙂
    يطلب أفشالوم الإذن بالذهاب إلى الخليل عبر طريق ملتوي. والده، الذي يرتدي في هذه اللحظة قبعة النبي، يعلم أن المستقبل سيحدث ويقول له وكأنه "مر" هكذا: "اذهب يا أبشالوم". وليس في هذه اللفظة بركة إلا الإذن. ولكن بحذف الحرف الأول من اسمه "اذهب بسلام" والذي يُفسر على ما يبدو على أنه بركة، تنبأ أنه سيفقد رأسه...

  34. اليوبيل,

    هل تقصد مباركة داود لأبشالوم الذي من المفترض أنه سيذبح في حبرون وهو في الواقع سوف يتمرد: "اذهب بسلام"؟

    إذا كان الأمر كذلك فلماذا يشير هذا إلى نهايته المريرة؟

  35. إسرائيل،
    كشخص يؤمن بوجود الإرادة الحرة، يجب أن أعترف أنك لا تجر أي شخص ضد إرادته. ولهذا أتعجب في كل مرة من موهبتك في جذب الناس إلى مناظرات لا تترك طعمًا في أفواههم. ويبدو لي أن ما يجذبني هو الأسئلة التي تطرحها، والتي هي في غاية الأهمية. ما يصدني (على الأقل) هو الاستنتاجات التي تتوصل إليها بطريقة ملتوية ومرهقة جدًا لأولئك الذين يدخلون معك في المناقشة (هنا أيضًا، أتحدث نيابة عن نفسي فقط).
    وعن أسئلتك حول الوقت في مشاركتك الأخيرة:
    1) ساعات درجة الحرارة هي إبداعات النظرية والخيال. إن بنائها أمر جدير بالاهتمام ومرغوب فيه، ولكن طالما لم يكن هناك شيء، فمن العار أن نضيع الكلمات.
    2) تشهد أنك لا تفهم "كيف يمكن الحديث عن بداية الزمان عما كان قبله"، ولكن بين سوء الفهم والنفي مسافة كبيرة. أنت تعيش في الزمن وتدركه، وتتعامل معه ككيان أبدي موجود وراء أي ظاهرة فيزيائية، وبالتالي لا يمكنك فهم "الزمن أو أي شيء آخر يسبق الزمن". لكن ليس من المستحيل أن تكون مرجعيتك خاطئة، والزمن ليس كيانا أبديا بل هو ظاهرة فيزيائية. ففي النهاية، لديك مثل: الكون بأكمله عبارة عن ساعة تدق؛ كل جسيم، سواء كان مادة أو مساحة فارغة، هو ساعة واحدة في هذا الكل العظيم الذي يسمى الكون؛ عندما يتكاثر جسيم من جسيم آخر (أو خلية حية من خلية حية أخرى)، فإنه يرث من الأب خاصية الساعات؛ الآن قارن هذه الساعة الكونية بساعة من نوع الأجهزة التي نستخدمها لقياس الوقت - على سبيل المثال، ساعة تعمل بطاقة نابض أو كهرباء؛ لا تبدأ هذه الساعة بالنبض إلا بعد أن يكسرها شخص ما أو يقوم بتركيب بطارية فيها. يمكن تسمية هذا الحدث التأسيسي بعدة أسماء مثل "بيريشيت" أو "مفتزجادول" 🙂 أو ما شئت. مع العلم يا أعزائي أن ساعات الحرارة لا تقيس الزمن المطلق وإنما تبدأ فقط من ذلك الحدث المحدد.
    قد بسلام وفي الطريق من فضلك حاول حل لغزي. لا مريم ولا ابنها، بل يقول البعض قريب.

  36. حرفيا: آثوس وبارتوس وأراميس.
    يجب تغيير العنوان إلى: "الفرسان الثلاثة - المدونة"
    سيطرح الفارس سقراط أسئلة بأسلوب أرسطو (من يعرف المنزل أفضل - من يعيش فيه أم من بناه؟) وسيكافئ قراءه بإجابات من الكتاب.

  37. ر.ح. غالي

    أنا لا أسحب أي شخص عن غير قصد إلى أي شيء. إذا لخصناها، فعلينا أن ننتهي، حتى لو لم نتمكن من إقناع بعضنا البعض.

    1. مفارقة التوأم – بحسب أينشتاين، من الممكن مزامنة الساعات، ولكن فقط عن طريق الاتصال بينهما، وبواسطة وسائل تكون سرعتها أقل من سرعة الضوء. ومن هنا إطالة الأوقات.

    باستخدام الساعات المؤقتة، تكون المزامنة فورية، وبالتالي يتم التخلص من إطالة الأوقات.

    2. جادلت وجادلت أيضًا. ألا تتذكرين كم مرة قلت أنني لا أفهم كيف يمكنك التحدث عن بداية الزمن؟ ما كان من قبل هل تريد مني العثور على المشاركات المحددة؟

    3. هناك العديد من هذه الآلات الحاسبة. ومن الواضح أن هذا ما تدعي النسبية. ومن المستحيل أن تثبت لي ميلاد المسيح إذا لم أقبل وجود مريم.

    4. لا يمكن أن تصل الصيغ والحسابات قبل الاتفاق على المبدأ. من هنا الثلاثاء إلى الثلاثاء.

    5. أوافق أيضًا على أن مثل هذه المناقشات أفضل من القراءة الجافة للمادة. إنهم يجبرونك على المشاركة الكاملة. لذلك، شكرًا لك أيضًا على الاستثمار الرائع.

    6. عيسو. لقد تعرض للمضايقة من قبل دوس، في رأيي. رجل لطيف وبريء يعتني بمشاكل والدته وأخيه. دعونا لا نشكو من عماليق بعد ما فعلناه بالجد.

    سأذهب إلى عرض في فيغاس لذا لن أتمكن من التعليق قريبًا. وفي هذه الأثناء فكر في الموضوع التالي:

    في بداية القرن العشرين، كانت الصناعة والنقل متطورة للغاية. عبرت القطارات القارات، وعبرت السفن بحجم تيتانيك المحيطات، واستثمرت مصالح اقتصادية هائلة في مسألة النقل برمتها. وقد عمل عشرات الآلاف من المهندسين على بناء وتحسين التدابير الحالية.

    ولذلك فمن الغريب أن الاختراع الأكثر أهمية على الإطلاق - الطائرة - ترك لاثنين من صانعي الدراجات غير المتعلمين من ولاية أوهايو، وهما ويلبر وأوربيل رايت.

    فماذا يعني هذا؟

    يعني الوعظ.

  38. إسرائيل،

    الآن أنت تسحبني عن غير قصد إلى المناقشة.

    1) "لذلك فمن الواضح أنه إذا أمكن مزامنة الساعات دون اتصال مباشر، كما اقترحت، فسيتم التخلص من إطالة الوقت." لماذا؟ في مفارقة التوأم، هل من المستحيل مزامنة الساعات على الرغم من اختلاف الأوقات لكل توأم؟ ألا يمكن أن تكون ساعات درجات الحرارة الخاصة بك أو أي تغير كوني آخر بمثابة "مرساة" للمزامنة في الأنظمة المتسارعة؟ بالطبع هو كذلك.

    2) أنت تزعم أنني أفتقد النقطة الأساسية وهي أنني لست ضد النسبية، بل أشير إلى التناقض بين النظريتين. أنا شخصياً أؤمن بالعلاقات وزمنها النسبي أكثر بكثير مما أؤمن بالانفجار الكبير والزمن المطلق الذي يتضمنه."

    هذه هي الجملة التي أعادتني إلى المناقشة. كيف يمكنك أن تدعي بعد أكثر من 1000 مشاركة أننا نتجادل حول النسبية ولم تتحفظ ولو مرة واحدة على النتائج التي تشير إلى الانفجار الأعظم، وأنك تؤمن أكثر بالنسبية وتعتقد أن الانفجار الأعظم خطأ. لقد فتحت جبهة جديدة هنا بحجم خط ماجينو.

    3) انظر الآلة الحاسبة التالية: http://www.wolframalpha.com/input/?i=time+dilation+calculator&a=*FS-_*RelativisticTimeDilationFormula.to-&f2=1+s&f=RelativisticTimeDilationFormula.to_1+s&f3=3×10%5E7+m%2Fs&f=RelativisticTimeDilationFormula.v_3x10%5E7+m%2Fs

    الذي يحسب تمديد الوقت. لاحظ أنه لا يوجد تسارع فيه، بل سرعة متجهة فقط.

    4) على أية حال، سبب خروجي من النقاش ليس أنني اقتنعت أو أنني أحاول تغيير الموضوع. أشعر أننا استنفذنا النقاش الفلسفي ويجب أن ينتقل النقاش إلى خطوط الصيغ والحسابات. وبما أنني للأسف لست على دراية كافية، وليس لدي، مرة أخرى للأسف، الوقت لدراسة معادلات النسبية بعمق، فإنني أترك الساحة لأولئك الذين هم على دراية بالموضوع.

    5) علاوة على ذلك، أشكرك على المناقشة، ونتيجة لذلك تعمقت معرفتي بالنظرية النسبية بشكل كبير، وهذه على الأقل نتيجة إيجابية واحدة لكل هذا الرعب.

    6) سأكون سعيدًا دائمًا بإعادة النظر في شخصيات الكتاب المقدس ومناقشتها

  39. روبي
    عندما قمت بالمحاكمة للقانون الثاني، كان ذلك بسبب إيماني بأنه لا يوجد شيء اسمه أشخاص سيئين. إذا أخذت شخصين، أحدهما "صالح" بكل المقاييس والآخر "سيئ" وأخبرتهما أنه سيولد لهما طفل، ويمكنهما الاختيار الآن ما إذا كان الطفل سيصبح شخصًا صالحًا عندما يكبر أو شخصًا سيئًا - كلاهما سيختار الخير.

    الخلاصة - كلنا خير، والشر نتيجة الظروف، أو القانون الثاني.

    استنتاجي فيما يتعلق بالأنظمة التي أسميها "الميكانيكية النفسية" ليست قيمًا أو أخلاقًا. فهي واقعية. توفي حوالي 60 مليون شخص في الحرب العالمية الثانية. ومنذ ذلك الحين مات أكثر من 100 مليون شخص في الحروب. بعد سقوط الكتلة الشرقية، كثر الحديث عن "نهاية التاريخ" والسلام العالمي. وبدلا من ذلك، فإننا نشهد صعود الأصولية الإسلامية. إن الأنظمة ببساطة تتغير، وهذا ما حاولت أن أعرضه في مثال إيران.

    هذا هو توقعي، المستند إلى مبادئ الميكانيكا النفسية، والذي سنكون قادرين على رؤية ما إذا كان هناك أي حقيقة فيه: في العقد الحالي، سوف يقوم أحد أعدائنا الرئيسيين، حماس أو إيران أو لبنان أو سوريا أو أي شخص آخر، بذلك. اعتدال كبير في موقفه تجاهنا، تعويضاً عن ارتفاع مستوى الكراهية لدى الأطراف الأخرى. وهذه بالمناسبة نتيجة ضرورية للقانون الأول للميكانيكا النفسية.

    ر.ح.

    لم ننتهي من مثال جاك جيل. لو انقلبت الأدوار، وكنت مكاني، أعتقد أنك ستتهمني بالتلويح بيدي ومحاولة تغيير الموضوع. لقد زعمت دائمًا أنه إذا كان الانفجار الأعظم صحيحًا، فإن هناك إطالة في الأزمنة في الأنظمة المتسارعة، لكنه ليس حقيقيًا. إذا قمنا بفصل جزرتين صغيرتين في مهدهما ووضعنا إحداهما في الثلاجة، فسوف تنضج بشكل أبطأ من شقيقتها ومعها كل نظامها المرجعي: الخضروات الأخرى في الثلاجة. ومع ذلك، فإن هذا لا يشكل "مفارقة التوائم الجزرة".

    وفقاً للنسبية، فإن التوأمين اللذين يلتقيان يكونان في زمن مختلف، وليس لأي منهما أفضلية على الآخر. لا يمكننا مزامنة ساعاتهم دون وجود اتصال مباشر بينهم، ولا يوجد شيء اسمه "الوقت الحقيقي" يمكن تحديده باستخدام الساعات المؤقتة أو ساعات المسافة المجرية. مسألة إطالة الزمن برمتها تنبع من حقيقة أننا إذا قمنا بإنشاء اتصال بين النقطة A و B باستخدام الأشعة الضوئية من أجل مزامنة الساعات، فإذا كانت النقاط في حركة نسبية، يتغير الزمن بسبب السرعة الثابتة من الضوء.

    لذلك، فمن الواضح أنه إذا أمكن مزامنة الساعات دون اتصال مباشر، كما اقترحت، فسيتم التخلص من إطالة الوقت.

    إذا كنت ترغب في مواصلة المناقشة في وقت ما في المستقبل، فسوف أصر على أن تستمر من النقطة التي توقفنا عندها: التجربة الفكرية 2-XNUMX. إن كل الإشارات إلى ما يسمى بـ "الحقائق" وما يسمى بـ "المؤسسة العلمية" لا صلة لها بالموضوع، خاصة لأنه يبدو لي أنك تكرر وتغفل النقطة الأساسية: أنني لست ضد النسبية، ولكني أشير إلى التناقض بين النسبية والحقيقة. النظريتين . أنا شخصياً أؤمن بالعلاقات وزمنها النسبي أكثر بكثير مما أؤمن بالانفجار الكبير والزمن المطلق الذي يتضمنه.

    يوفال - كلمات الاحتلال تذيب القلوب. ليس لدي مشكلة في فتح صفحة جديدة. أفتقد نوعًا ما تلك الأيام التي كنا فيها نحن الثلاثة، أنا وأنت، ر.ه.، نتناقش مع الشخصيات الأسطورية في الكتاب المقدس (ثامار، أيوب، بالاق...)

    ولكن الدير الأسود من البدء بإسرائيل!

  40. ر وإسرائيل، لقد أخطأت إليكم.
    إنني أتفهم تماما نضال إسرائيل وأتعاطف معه. أفترض أن هذه ممارسة جيدة (مجانية) بالنسبة لـ RH لمهاراته النقدية الممتازة. من وقت لآخر كنت أرسل إليك سهامًا للسخرية، لكن يجب أن أعترف بأنني تعلمت منك الكثير واستفدت كثيرًا أيضًا في موضوع نموذجي الخاص. أنا آسف على الكلمات القاسية التي ألقيت بيني وبين إسرائيل وعلى الصداقة الطيبة التي ضاعت، ولكنني أشعر بالارتياح لأنني وجدت شخصا يشبهني كثيرا من حيث الخيال الخصب والطموح والدفء. حِدّة. ومن المحتمل جدًا أن يكون لدي أيضًا الكثير لأتعلمه في العلاقات الإنسانية، لكن لسوء الحظ لست موفقًا على الإطلاق في تعلم الدروس في هذا المجال.
    إسرائيل! أنا أتفق مع رغبات ر.ه.، وليس مع السخرية على الإطلاق. أنت أحد الكتاب الأكثر موهبة الذين كنت محظوظًا بالتواصل معهم، وأنا متأكد من أنك ستحصل على عناوين إيجابية، حتى لو لم يكن بالضرورة فيما يتعلق بستوكهولم.

  41. إسرائيل،

    حسنًا، بقدر ما يهمني، يمكن تلخيص المناقشة هنا. ملخصي:

    * بعد كل المنشورات وكل الإدعاءات فشلت في إقناعي بأن هناك تناقض بين الإنفجار الكبير وإستطالة الزمن.

    * أغلب ادعاءاتك تتعلق بإطالة الزمن وهي من الركائز الثقيلة التي تقوم عليها النسبية. وقد ثبت إطالة الأوقات في مجموعة متنوعة من التجارب. ومع ذلك، فإنك ترفضها بإشارة من يدك باعتبارها "محاولات تم إجراؤها على الأنظمة المتسارعة". أفترض أنه إذا أظهر لك شخص ما تجربة في نظام غير متسارع، فسوف تدعي "هذه تجارب تم إجراؤها بالقرب من مركز الجاذبية - الأرض، الشمس، المجرة)" في رأيي، هذه مراوغات ومحاولات إحياء نظرية أن الأدلة ضدها. أعرف العديد من الأمثلة المشابهة في علم الأحياء. وفي التنقيب عن النفط، يسمى هذا "الحفر المائل".

    * كانت هناك العديد من المناقشات هنا مع الخلقيين مثل شينغوا الذي يرفض كل ادعاء يظهر التطور بادعاء مشابه لادعائك "لم يرى أحد تغيرات كلية/جزئية في البروتينات وجميع أنواع الادعاءات الأخرى التي لا أساس لها من الصحة. وفي النهاية يختتمون دائمًا بتجاهل واضح لجميع الحقائق "لم يثبتها لنا أحد". إنه ليس علمًا ولا أي باحثين. إن ادعاءاتك، في رأيي المتواضع، ليست بعيدة المنال مثل ادعاءاتهم، ولكنك تميل أيضًا إلى رفض أي شيء لا يناسبك باعتباره "ليس هذا هو الهدف، إنه نظام متسارع".

    * لقد فشلت في إقناع أنه من المحتمل أن يتحرك الضوء بسرعة غير C وأن أجهزتنا ببساطة لا تلتقط الجسيمات السريعة. لقد قدمت تشبيهًا مشكوكًا فيه للبندول الباليستي. لم تشرح لماذا لا يمكن إنتاج أجهزة كشف أفضل؟ (في تشبيهك بندول من الفولاذ منيع للكرات). لم تبين لماذا لا نرى الضوء البطيء من C. في الواقع لم تظهر أي دليل على ذلك باستثناء الادعاء بأنه قد يكون ممكنا. أبعد من ذلك عندما أظهرت لك أنه كان كذلك
    ليس هذا ما يحدث في مسرعات الجسيمات، ولكنك رفضته مرة أخرى بالحفر بشكل قطري على أساس أننا لا نتحدث عن جسيمات ذات كتلة. هل من الممكن أن يكون C صحيحًا بالنسبة للجسيمات ذات الكتلة وليس الضوء الذي تشتق منه؟

    على أية حال، ما سبق ليس محاولة لإرخاء يديك. بل واصل وتبارك وأتمنى لك التوفيق وسنلتقي في ستوكهولم.

  42. إسرائيل، فيما يتعلق بالأنظمة النفسية الميكانيكية المغلقة أو المفتوحة، تعامل الإنسان على أنه جسد مادي بلا عقل أو روح أو عقل قادر على وضع القوانين وتطبيقها. العالم يتجه نحو العولمة بحكومة مركزية تسيطر على النظام في نظام الأرض رغم كل حركات المقاومة. إن السيطرة على 7 مليار إنسان وتزويدهم بالوقود والغذاء والسرير الدافئ تتطلب تنظيماً مركزياً قوياً. الديناميكا الحرارية الاجتماعية تعادل اللاسلطوية الاجتماعية التي تحاول جلب الفوضى إلى المجال. القوة الخارجية التي تتسبب في إعادة ضبط الأرض هي كارثة عالمية مثل نيزك ضخم أو نبضة أشعة جاما سوبر نوفا أو زلزال / بركان ضخم سيحدث عصرًا جليديًا جديدًا.
    وفيما يتعلق بنظرية الانفجار الكبير، فهي النظرية السائدة لدى علماء الفلك المبنية على الأدلة التلسكوبية التي تشير إلى بساطة الكون. هناك نظريات جديدة تقول أن الكون على شكل غشاء وهناك أكوان أخرى على شكل أغشية ومن وقت لآخر يحدث اتصال بين غشاء وآخر وهذا يسبب انفجاراً عظيماً والعياذ بالله.
    أدعي أنه في الانفجار الأعظم انتشرت المادة في شكل كروي كما في النظرية السائدة ولكنها انكمشت أيضًا إلى الداخل كما في المستعر الأعظم وتسببت في خلق ثقب أسود ضخم ومن وقت لآخر ينبثق كون آخر كما في البصلة الجلد ، شخص ما يدحض نظريتي ....

  43. ر.ح.
    هذا ليس ما أقوله.
    دعني أقول لك ما أقوله بالضبط، حتى لا يكون هناك سوء فهم:

    إن إطالة الزمن في العلاقات لا تتناسب مع نظرية الانفجار الأعظم. هذا.

    حسب فهمي، فإن جميع التجارب التي اقترحتها، بما في ذلك الميونات والدوبلر، موجودة في أنظمة متسارعة. إن إطالة الأوقات في الأنظمة المتسارعة أمر منطقي، إذا تابعت المناقشة حول جوهر الوقت مع الطالب.
    لم تريني أي تجربة تثبت نظرية الانفجار الأعظم. إذا كنت تعرف أدبيات الفيزياء تحت الأرض، فهناك الكثير من المعارضة لهذه النظرية.

    على أية حال، أعتقد أنك توقفت عن محاولتك أن تثبت لي أن هناك تناقضًا في التجربة الفكرية التي اقترحتها مع جاك وجيل والساعات المؤقتة.

    يمكننا أن ننتهي هنا، إلا إذا كنت ترغب في الاستمرار. والخطوة التالية هي خطوة الكاميرات.
    على أية حال، شكرا للاستثمار. أعلم أن الأمر مرهق للغاية، لكني مازلت مقتنعًا بصحة ادعاءاتي حتى يثبت العكس.

  44. حسنًا، يا إسرائيل، لديكم حقًا طريقة علمية بحتة.
    أي نتيجة تتعارض مع نظريتك تصبح على الفور غير ذات صلة.

    يقولون لك: أطروحتك تتناقض مع مفارقة التوأم. أنت تقول "نحن لا نتحدث عن الأنظمة المتسارعة"
    يقولون لك: تم إثبات تمدد الزمن في الأنظمة غير المتسارعة من خلال تأثير دوبلر. أنت تقول "نحن لا نتحدث عن تأثير دوبلر".
    يقولون لك: المعجل الموجود في المحور يوضح لك أنه لا يمكن تجاوز سرعة الضوء ولا توجد جسيمات تختفي لأنها وصلت إلى سرعة عالية. أنت تقول "نحن لا نتحدث عن الجسيمات ذات الكتلة"

    باعتبارك شخصًا يدعي أنه يغير إحدى النظريات الفيزيائية الأكثر رسوخًا ويدعي أن الموقع موجود بالفعل ضد كل الإجماع، فأنت بحاجة إلى إظهار شيء عام وواسع النطاق. لا يمكنك تقليصها وتقليصها إلى حالة خاصة لم يتم اختبارها بعد، ومن ناحية أخرى تقول إن أي مثال مضاد ليس له صلة بالموضوع.

    ففي النهاية، إذا كانت نظريتك صحيحة، فستكون صحيحة سواء في الأنظمة المتسارعة أو في تأثير دوبلر وفي المحور وفي تأثير الفراشة في الحوض الشرقي للمنخفض الشمالي. باختصار، كانت عامة وجميلة.

    أتمنى أن تكون على الأقل قد علمت ابنتك المتخصصة في الكيمياء الحيوية العلوم بشكل أفضل من ذلك.

  45. إسرائيل،

    باختصار، بعد كل هذه المناقشة المطولة، يمكن تبسيط ادعاءك بأنك تتحدى بالفعل إطالة الوقت. حجتك صالحة في فهمي حتى بدون ساعات درجة الحرارة. ما تدعيه، وصححني إن كنت مخطئا، أنه إذا تحرك جسمان بدون تسارع فسوف يشاهدان نفس الوقت. وهذا الادعاء يتناقض مع الادعاء بإطالة مدة العلاقات.

    للأسف ليس لدي المعرفة الكافية في الفيزياء، ما يمكنني فعله هو البحث قليلاً. وبالبحث السريع وجدت الموقع التالي:
    http://math.ucr.edu/home/baez/physics/Relativity/SR/experiments.html#Ives-Stilwell
    الذي يجلب البراهين التجريبية للنظرية النسبية. ووفقا للقسم 4 هناك دليل جيد على إطالة الأوقات بتأثير دوبلر (ولاحظ أن هذا ليس في أنظمة المعجل).

  46. ر.ح.

    "سوف تبدو أشياء مختلفة."

    دعونا نتفحص هذا الأمر لحظة، فهذه نقطة طرحها الآخرون مرات عديدة وهنا تكمن المشكلة في رأيي.

    دعونا نتصرف كالمعتاد. لدينا سيارة، على سطحها ساعة كبيرة وكاميرا ذات دقة عالية. تمر أمامها سيارة أخرى بنفس الجهاز. عندما يمرون ببعضهم البعض، يتسبب تقريبهم في تنشيط الكاميرات، ويقومون بالتقاط الساعتين معًا.

    1. عند السرعة النسبية 100 م/ث، هل لديك أي شك في أن كلا الكاميرتين ستشهدان نفس الصورة، ومن هذا المنطلق سيكون من الممكن معرفة الوقت الذي تظهر فيه كل ساعة؟

    2. 1000 م/ث؟

    3. 10,000، 100,000 وهكذا حتى سرعة الضوء تقريبًا؟ في أي نقطة سوف ترى كل كاميرا عكس رفيقتها؟

    علاوة على ذلك، ليست هناك حاجة للوصول إلى سرعات عالية على الإطلاق. من الناحية النظرية، مع قيادة السيارات لفترة كافية، يمكن أن يصل فارق التوقيت إلى عدة دقائق حتى عند السرعات المنخفضة. هل يمكنك تخيل موقف تمر فيه سيارتان ببعضهما البعض بسرعة نسبية 2 كم/ساعة، وتهتز الكاميرات، ثم عندما تفتح الفيلم يتبين أن الصور تظهر صورة مقلوبة وكل لقطة تظهر الساعة الأخرى كما هي بطيء؟

    ودعونا لا نذهب بعيدا. في مثال جاك وجيل الأصلي، الذي قرأته على الرابط، يتفق كلاهما على أن وقت جيل متأخر بثانيتين. لا يوجد أي ادعاء من جيل بأن ساعة جاك هي في الخلف.

    أخبرني إذا كنت لا تزال تعتقد أن أي كاميرا يمكنها تصوير عقارب الساعة العكسية على أنها بطيئة.

    ولا تهتم بالأشباح. وظيفته هي الجلوس في الكمين والانتظار لمعرفة ما إذا كان يمكنه أن يقول شيئًا ضدي. أرني تعليقًا واحدًا منه في الشهر الماضي لا علاقة له بي. الجميع وهوسهم.

  47. ر.ح. ورفاعي م.، أرجوا أن يتوقفا عن هذه الإهانات التي لا تضيف احتراما لأحد.
    أنظر مثلا ماذا حدث لي مع إسرائيل. يجب أن تتذكر دائمًا أن كل عصا لها طرفان.

  48. رونان ه.
    "وهي تعرف أيضًا عن المواقع ذات X المتعددة التي تنظر إليها؟" - والا ؟ لو تكلمنا عن الأخطاء الفرويدية.. (ربما كنت تريد أن تكتب شيئًا آخر، أليس كذلك؟ د:)

  49. يوفال
    أتمنى أن تكون على حق. إذا كان الأمر كذلك، سنكون جميعا محظوظين.

    رونان...عفوا آسف، خطأ فرويدي. قصدت "R.H.":

    بين، جلي.

  50. ماذا حدث؟ كاسبر؟ هل عدت من الجرم السماوي؟ أو تمتد إلى الصفر المطلق؟
    هل تسمح أمي باللعب على الكمبيوتر في مثل هذه الساعات؟

  51. إسرائيل

    قد تقول: "لا تنس أن لدينا كاميرات عالية الدقة في كلا السفينتين الفضائيتين. من الصعب الجدال مع الصور. ما يراه جاك هو ما تراه جيل."

    لا يعني ذلك أنني أفهم كيف يمكن أن يكون الأمر، لكن بالتأكيد لا! وفي ضوء ما كتب عن تمديد المدة:

    عندما يكون هناك راصدان في حركة موحدة نسبية ولا يتأثران بأي كتلة جاذبية، فإن وجهة نظر كل منهما ستكون أن ساعة الآخر (المتحركة) تدق بمعدل أبطأ من الساعة المحلية.

    سوف تبدو أشياء مختلفة. الحقيقة هي أنها تجربة يجب القيام بها! سفينتان فضائيتان مع ساعات على الكاميرا الجانبية لبعضهما البعض.

    بالمناسبة، ليس هناك أي صلة بالمليار الثاني، في رأيي حتى من بعيد ستكون نفس القصة

  52. اليوبيل.
    وكما كان متوقعا، لم تثر نقطة واحدة تؤيد ادعاءك وكأنني أتهرب من أمر ما أو أحاول التضليل.
    لذا سألخص تعليقك الأخير كمحاولة تافهة أخرى لخفض إنتروبيا الخاص بك حتى تزيد إنتروبيتي.

    وأيضًا، عندما تطرح نماذج أو افتراءات، حاول أن تدعمها بشيء آخر غير غباء قرائك.

    دعونا نختتم الأمر هنا. ليست هناك حاجة حقا للرد عليك. كل ما يضيف - يطرح.

  53. ميشال شابيرا!
    اتهامات، تشهير، اعتذارات... هل أنت متأكد أنك اخترت المهنة الصحيحة؟ تقوم بمراجعة قانون القانون الثاني باحترافية لا مثيل لها. أشعر بالأسف عليك يا أخي، لأنك بدلاً من أن تجعل جندياً بين جدران المحكمة، فإنك تضيع وقتك (ووقت من يأخذونك على محمل الجد) هباءً.
    انت شخص طيب ملح الأرض. موهوب جدا ذكي للغاية. يجب على الأشخاص مثلك أن يفعلوا ذلك لمنزلهم في وقت ما، وقبل ساعة واحدة يكون ذلك جيدًا.

  54. ر.ح.
    ربما ينبغي علينا حقا ترتيب الأمور.

    و. يوافق.
    ب. لا اوافق.
    ثالث. ومن هنا التناقض.

    نكمل. "لقد أسرعت جيل في الماضي" وكذلك فعل جاك. يمكن الوصول إلى نظام جاك-جيل بالطريقة التالية: جاك وجيل معًا ظهرًا لظهر في الحقل التجريبي في تيزنبي. كلاهما في راحة. في مرحلة ما، يقوم كلاهما بتشغيل المحركات ويبتعدان عن بعضهما البعض. وبعد دقيقة لكليهما، أوقف كلاهما المحركات وانتظرا لمدة ساعة، حتى تبقى مسافة كبيرة بينهما. لذلك يعكسون اتجاه الرحلة ويشغلون المحركات مرة أخرى لمدة دقيقتين، ثم يتوقفون عن العمل.

    ما يحدث هو أن كل واحد منهما يتحرك الآن بالنسبة للآخر، وبحسب النسبية فإن زمن كل منهما يمر بشكل أبطأ بالنسبة للآخر. ومع ذلك، لاحظ أن النظام متماثل تمامًا: لا يوجد شيء مثل جاك "يستريح" وجيل "يتحرك" على الرغم من أن هذا هو ما يُرى من وجهة نظر كل منهما.

    في هذه الحالة تلقينا بيانات المشكلة. لقد افترضت أن جيل "تحركت" لكن جيل كانت تستريح بالنسبة لها. وكذلك جاك. كلاهما حاليا في وضع الخمول. كلاهما يستريحان.

    نكمل:

    لا يوجد وضع A=B=C=D في العالم.
    (لنقل: درجة الحرارة A وC، وB وD السيزيوم)

    يرى جاك أ = ب = ج و د بطيئين

    ترى جيل أن A = C = D وأن B بطيء.

    لا أحد في العالم يرى أ=ب=ج=د.

    لا تنس أن لدينا كاميرات عالية الدقة على كلتا المركبتين الفضائيتين. من الصعب الجدال مع الصور. ما يراه جاك هو ما تراه جيل. يجب أن نتفق على هذه النقطة قبل أن نواصل:

    هل توافق على أنه في لحظة اللقاء، في جزء من مليار من الثانية التي يمر فيها جاك وجيل ببعضهما البعض، وتلتقط الكاميرات الموجودة على كلتا المركبتين الفضائيتين الساعات الأربع جميعها، سيريان صورًا متطابقة؟ أو تعتقد أن الصور ستظهر أوقاتًا مختلفة على الساعات. (على سبيل المثال: أن كاميرا جاك ستظهر أن ساعات جيل أبطأ من ساعاته، وأن كاميرا جيل ستظهر أن ساعات جاك أبطأ من ساعاتها).

    يمكننا أن نستمر إذا توصلنا إلى اتفاق بشأن هذه النقطة. (وهذا ليس واضحًا تمامًا بالمناسبة. على ما أذكر، كانت هذه هي المرحلة التي تم فيها التعبير عن الخلاف من قبل الفيزيائيين العاديين. لا بأس، لقد تمكنا من التغلب عليه، لكنني أريد أن نتوصل إلى اتفاق بيننا).

  55. يوفال
    أنتم تتهمون بالاتهامات والافتراءات. لقد أوضحت لك أنه في النقاط التي طرحتها، إما أنك لم تكلف نفسك عناء القراءة حتى النهاية أو أنك ببساطة لم تفهم.
    ينبغي عليك أن تفعل أحد الأمرين:
    1. أرني أماكن أخرى أهرب فيها، حسب رأيك، أو اشرح لي السبب الذي يجعلك تعتقد أنني تهربت من الأماكن التي قمت بتربيتها.
    2. اعتذر وتوقف.

    وإذا جعلتك ترغب في التحدث معي - لماذا لا تفعل ذلك وتتجاهل ردودي تمامًا؟

    روبي.

    في انتظار الرد الذي أتمنى أن يلقي الضوء على ما كتبته.

  56. شكرا روبي ،
    إن التسارع للأمام والتباطؤ في طريق العودة هما الحل المقبول لمفارقة التوأم. ولكن يبدو لي أن إسرائيل تحاول تأكيد شيء ما بشأن أوضاع أخرى لم يتمكن أحد هنا حتى الآن من فهمها. مثل المثقف اليهودي العادي، يسعى بلا كلل لتقديم صورته للعالم، وعلى طول الطريق يجذب الكثير من النار.
    منذ السبي البابلي، بل وحتى قبل ذلك، كان اليهود دائمًا أقلية في العالم. ينتقل السكان من حرب إلى حرب، وفي الحروب تكون الأقليات هي التي تتأذى أكثر من غيرها. والسؤال ليس ما هو سبب مضايقة العالم بل ما سر بقاء هذه الأقلية بالذات.

  57. وتحدثت عناوين الصحف التي أعلنت افتتاح المحاكمة عن "محاكمة كل العصور".
    أمطر معظم الكتّاب القانون الثاني بالنار والكبريت مستخدمين تعابير قاسية للغاية. وكانت "المتغطرسة" و"الفاسدة" و"القاسية" من بين العبارات المعتدلة. ووصفته إحدى الصحف ببساطة بأنه "وحشي" وأضافت: "في بلد ديمقراطي، الشعب هو الذي يضع القوانين. لا أفهم لماذا يجب أن نستمر في دفع الضرائب وتمويل نظام يدعو قانونه الأساسي إلى حل كل النظام والانضباط".
    وطالبت منظمات محافظة بإلغاء القانون الثاني فورا من جميع الكتب المدرسية. لقد اقتبسوا الكثير من كلمات البروفيسور بولتزمانسكي، وعلى وجه الخصوص، غضبوا من التأكيد على أنه "لا يمكن لأي نظام أن يقلل من إنتروبيا النظام الخاص به إلا إذا زاد من إنتروبيا الأنظمة الأخرى" التي عرفوها بـ "الداروينية المطلقة!". هل هذه هي صورة العالم الذي نريد أن نربي أولادنا فيه؟". سأل المتحدث باسمهم بطريقة ساخنة، مضيفًا: "إنه أسوأ من التطور".
    وسمعت هنا وهناك أصوات مترددة تقول إن القانون شر لا بد منه، وأن هناك حقيقة في الادعاء بأنه هو بالفعل مصدر الحياة. وتساءلنا كيف يمكن لإشعاع الشمس أن يصل إلينا بدون القانون الثاني؟ وكيف ستعمل جميع الآلات؟ وآلية الحياة نفسها؟
    وسرعان ما تم إسكات هذه الأصوات من قبل العلماء الغاضبين الذين أوضحوا بإسهاب أنه يمكن بسهولة تنظيم قانون أكثر إيجابية، والذي من شأنه أن يحافظ على النظام الكوني دون الحاجة إلى التفكك المستمر.
    وكان الأكثر واقعية أولئك الذين قالوا إنه سواء تمت إدانته أو تبرئته، فإن القانون حقيقة قائمة، ويجب علينا أن نكيف حياتنا معه وألا نحاول تغييره. وكما أحدث اكتشاف القانون على يد كلوسيوس في القرن التاسع عشر ثورة في علم الديناميكا الحرارية والفيزياء بشكل عام، فيجب علينا أن نتقبل حقيقة القانون في الحياة اليومية ونتصرف وفقًا لذلك.
    تحدث الاقتصاديون عن "القانون الثاني في الاقتصاد" - ألا يتوازى تدفق رأس المال والأصول في العالم مع تدفق الحرارة في النظام الديناميكي الحراري؟ ألا توجد قوانين للحفاظ على الثروة والفقر في العالم تشبه في الأساس القانون الأول للديناميكا الحرارية وتشرح لماذا ينطوي الرخاء في مناطق جغرافية معينة دائمًا على الاكتئاب في مناطق أخرى؟ لا شيء، فمفهوما "الثروة" و"الفقر" في حد ذاتهما نسبيان، والفرق بين "الغني" و"الأكثر ثراء" وحده هو الذي سيحدد اتجاه تدفق رأس المال، كما هو الحال في حالة معينة من التراكم، فقط درجة الحرارة الفرق يحدد ما هو "الساخن" وما هو "البارد" وفي أي اتجاه سوف تتدفق الطاقة؟
    وإذا كانت قوانين الاقتصاد تشبه في الأساس قوانين الديناميكا الحرارية - أليس من المناسب الاعتراف بوجود "الإنتروبيا الاقتصادية" - الاعتراف بأن أي تحسن في نظام اقتصادي ما سيؤدي حتما إلى تدهور نظام آخر؟
    تحدث علماء الاجتماع عن "الانتروبيا الاجتماعية". أشار علماء الأحياء في مجال "إنتروبيا الجهاز المناعي" وأخصائيي التغذية إلى الحقيقة المعروفة وهي أنه على الرغم من كل الجهود المبذولة في الأنظمة الغذائية وصالات الألعاب الرياضية، فإن السكان يزدادون سمنة، كما أن الغالبية العظمى من الأنظمة الغذائية تنتهي في نهاية المطاف بالعودة إلى الوزن الأصلي مع القليل من الوزن. إضافية، تمامًا كما يحدث لدرجة الحرارة في نظام ديناميكي حراري مغلق ونشط.
    كانت المجموعة التي أطلقت على نفسها اسم "مجموعة الميكانيكا النفسية" مثيرة للاهتمام بشكل خاص. ادعى مؤسسها، الذي عرف فقط بلقبه "رجل الميكانيكا النفسية"، أن الأنظمة البشرية التي تضم عددًا كبيرًا من الأفراد، تتصرف تقريبًا مثل الأنظمة الديناميكية الحرارية. وكما لا توجد إمكانية عملية لحساب سلوك جزيء واحد، بل حساب سلوك ملايين عديدة من الجزيئات فقط، فإن الميكانيكا النفسية لا تتعامل مع الأفراد ولكن مع مجموعات كبيرة مثل البلدان أو القارات.
    تم تمويل المجموعة، التي تزعم أنها حصلت على أحدث المعدات واستخدمت الأساليب الإحصائية الأكثر تقدمًا، من مصدر خاص مجهول واجتذبت عقولًا كانت تعتبر رائعة بشكل خاص. كان الرمز عبارة عن صورة ثلاثية الأبعاد للكرة، مع وجود الحرف الصغير i في المنتصف. وكان السؤال الرئيسي الذي واجهته هو: إذا كانت الأنظمة البشرية، كما زعمت، تتصرف تقريبًا مثل الأنظمة الديناميكية الحرارية، ألا يستلزم هذا وجود مبدأ مشابه لقانون الإنتروبيا للمجتمعات البشرية؟ إجابة مثيرة للقلق: احتمال يميل إلى واحد لأن النظام أكبر وأكثر عشوائية، أو ببساطة: نعم. ومن هنا يأتي السؤال المصيري: هل تستطيع الأرض، وهي نظام نفسي ميكانيكي مغلق، أن تقلل من إجمالي إنتروبيا النفسية الميكانيكية؟ إجابة مخيبة للآمال ومحبطة: ليس من تلقاء نفسه. يستبعد "القانون الثاني للميكانيكا النفسية" إمكانية انخفاض الإنتروبيا في نظام ميكانيكي نفسي مغلق، ما لم تزيد إنتروبيا نظام آخر.
    يمكنك أن تقول مرحباً بالسلام والأخوة العالميين. إن أي ازدهار ووفرة في مجموعة واحدة سيأتي دائمًا على حساب مجموعة أخرى. لن تأتي ساعة الدلو من تلقاء نفسها، ولن يفيد إذا طلع القمر في البيت السابع واصطف المشتري والمريخ كرجل واحد. بدون مصدر خارجي للطاقة النفسية الميكانيكية، فإن أي نظام مغلق، سواء كان دولة أو معتقدًا دينيًا أو وضعًا اقتصاديًا، وخاضعًا للضغط النفسي الميكانيكي، محكوم عليه بالميل المستمر إلى التوسع كوسيلة لتقليل الإنتروبيا المحلية الخاصة به - وهذا في غياب الفضاء النفسي الميكانيكي الحر على حساب نظام نفسي ميكانيكي آخر.

  58. اليوبيل.
    لدي انطباع متزايد بأنه لسبب ما، ربما بسبب عسر القراءة، ربما تقرأ نصف جملة فقط وتتجاهل الباقي.

    إذن هذا ما كتبته عن الجذر i. يرجى القراءة حتى النهاية.

    "وفيما يتعلق بـ 1- ماذا عن i، جذر 1-؟ هل لديه أيضا جذر؟

    وبشكل عام، ما هي المضامين الفلسفية المتعلقة بالإيمان بالألف الأول؟ هل يمكننا أيضًا أن نؤمن بالله ناقصًا واحدًا؟ وهذا منطقي جدًا في رأيي، لأننا في الماضي كنا نؤمن بآلهة كثيرة، وحصرنا الأمر في إله واحد، والملحدون يؤمنون بالصفر، أليست هذه هي الخطوة المنطقية المطلوبة يا دوكي؟".

    سؤال: هل تنوي جديا الإجابة على الأسئلة:
    "وبشكل عام، ما هي المضامين الفلسفية المتعلقة بالإيمان باللغة الأولى؟ هل يمكننا أيضًا أن نؤمن بالله ناقصًا واحدًا؟»

    ألا ترى أن الأمر كله قيل على سبيل المزاح؟ لم أقصد على الإطلاق أن يعلق أحد على ذلك؟ كان الأمر برمته "كما لو" استمرارًا منطقيًا للأسئلة "الجوهرية" المتعلقة بدور مصير 1- في حياتنا؟

    فلنكمل "سرعة الضوء هي ببساطة نسبة إلى المصدر".

    كان رد فعلي الأول هو:

    "حتى أنه من الممكن التخلي عن مسألة الأمر الواقع، والتوجه مباشرة إلى العلم. لقد شرحت لك الإشكالية الموجودة في تفسيرك لتجربة MM. (أعتقد أنك قصدت تفسير لورنتز، الانكماش).

    سؤال: لماذا لا تأخذ بالتفسير الأبسط، وهو أن سرعة الضوء بسيطة بالنسبة لمصدر الضوء؟ وهذا من شأنه أن يفسر نتائج تجربة M-M بسهولة، أليس كذلك؟

    وجاء ذلك بعد أن نظرت إلى النموذج الخاص بك. لأنه، في رأيي، لا يمكن استخلاص السؤال المركزي من النموذج: لماذا تكون سرعة الضوء واحدة لكل جهاز قياس؟ ولكن فقط السؤال أعلاه، وهو شيء مختلف، سألتك نفس السؤال.

    لقد طلبت مني أن أشرح لك، وهذا ما فعلته. تشرح نظرية الانبعاث نتائج تجربة M-M بشكل جيد. (هل هذا ما قصدته؟ ليس من الواضح لك لماذا تشرح النظرية MM؟ أليس مكتوبًا بشكل صريح في الرابط بعد أن اضطررت إلى التمرير خلال المفهوم؟ وما هو الاحتجاج الكبير لأنني لم أرفق الصريح رابط لك، ألا تعلم أن مثل هذه الروابط تؤخر الردود حتى تتقادم؟) واصلت متابعة ما بدا لك (كما فهمت) واضحا تماما الذي قصدته: أن أشرح لماذا سرعة الضوء هي السبب؟ نفسه لكل كييل؟ ولكن هذا ليس ما قلته. اذهب إلى الموضوع.

    نفس الشيء عندما قلت أن الزمن والقانون الثاني هما نفس الشيء. إنه مثل القول بأن السواد وتسوس الأسنان هما نفس الشيء. هذا صحيح بالنسبة لشخص طبيب أسنان، لكنه ليس صحيحًا من الناحية النظرية.

    ورأيك فيما يتعلق بـ "توسعاتي" حول موضوع الهوية هو بالضبط هذا - رأيك. ليس لها أولوية في نظري على رأيي، أو رأي أي شخص آخر. وكعادتك تنطق جملة عامة دون الإشارة إلى مضمون الأشياء إطلاقا.

  59. إسرائيل،

    الآن أنا في حيرة من أمري. دعونا نرجع ونتذكر الجدل الذي دار بيننا.
    ولتذكيرك، القصة الطويلة والمرهقة بأكملها بدأت بادعاءك أن السفن الفضائية التي تتسارع وهيجان يمكنها تنسيق هجوم بناءً على ساعات درجة الحرارة (التي اتفقت معها) ثم ادعيت أن هذا يؤدي إلى تناقض مع الانفجار الأعظم ( الذي ادعيت أنه لا يوجد تناقض)
    و. يوافق ؟
    ومن هنا أتينا إلى جاك وجيل لتبسيط القصة. إذا تسارع جيل،
    ب. هل توافق على أنه سيكون هناك اختلاف في الساعات؟
    ثالث. إذا كان الأمر كذلك، فهل سيكون هناك تناقض مع النسبية أو الانفجار؟

    إذا لم تسرع جيل ولكنها قادت بسرعة أثناء السكون (من الواضح أنها أسرعت ذات مرة للوصول إلى هناك ولكننا سنترك هذه التفاصيل الدقيقة خلفنا) فأنت على حق، ستظهر الساعتان كما هي وسيصبح وقت جاك أبطأ عينيها بينما وقتها سيمضي أبطأ في عيني ج وبعد ذلك ينشأ تناقض فيما تسميه أ = ب = ج = د، ولكن ليس هناك تناقض هنا إلا أنه مبدأ تطويل مرات.

    لا يوجد وضع A=B=C=D في العالم.
    (لنقل: درجة الحرارة A وC، وB وD السيزيوم)

    يرى جاك أ = ب = ج و د بطيئين

    يرى جيل أن A = C = D وأن B بطيء.

    لا أحد في العالم يرى أ=ب=ج=د.

    وهذا هو جوهر تمديد الوقت. تماما كما في ويكيبيديا التي نقلت منها:
    وجهة نظر الراصد الآخر ستكون أن الساعة المحلية مرة أخرى (هذه المرة الساعة الأخرى) صحيحة وأن الساعة البعيدة المتحركة هي البطيئة. من منظور محلي، يبدو أن الوقت المسجل بواسطة الساعات الساكنة بالنسبة للإطار المرجعي المحلي (والبعيد عن أي كتلة جاذبية) يمر دائمًا بنفس المعدل

    عندما يكون هناك راصدان في حركة موحدة نسبية ولا يتأثران بأي كتلة جاذبية، فإن وجهة نظر كل منهما ستكون أن ساعة الآخر (المتحركة) تدق بمعدل أبطأ من الساعة المحلية. كلما زادت السرعة النسبية، زاد حجم تمدد الزمن

    إذن ما هو السر؟

  60. وأنا أتفق مع يوفال في أن تشبيه إسرائيل بالعرب كنظام ديناميكي حراري مغلق هو تشبيه مفرط في التبسيط.
    وأشبه دولة إسرائيل أكثر بجزيرة صغيرة في بحر عاصف وفاض، تحاول أن تغطيها أمواجه بنجاحات مؤقتة.
    إن الموقع الاستراتيجي للبلاد وحق الأوائل في أن يؤمن الشعب اليهودي بإله واحد بكل ما يعنيه ذلك يزعج الكثير من الناس، المسلمين في الحاضر والمسيحيين في الماضي بشكل رئيسي.
    أما فيما يتعلق بمفارقة التوأم فمن الواضح أن أحدهما في حالة سكون أو بسرعة ثابتة بينما الآخر يتسارع وعند عودته يتباطأ ليلتقي بأخيه الأكبر.

  61. إسرائيل،
    لقد سألت فاقبل (يذكرني بالأحرف الأولى من اسم BZK. وأتساءل لماذا 😀 ):
    أ) سألت ما هو جذر ط. لقد أظهرت لك طريقة لحساب ذلك ولم أتلق أي شكر، أو على النقيض من ذلك، لم أحصل على أي ركلة، فقط تجاهل تام. طلبت منك التعليق ولم ترد لقد طلبت منك مرتين أخريين، وأنت تجاهلتني.
    ب) قلت/سألت/زعمت: "لماذا لا تأخذ بالتفسير الأبسط، وهو أن سرعة الضوء بسيطة بالنسبة إلى مصدر الضوء؟ وهذا سوف يفسر نتائج تجربة M-M بسهولة، أليس كذلك؟ لقد طلبت منك إثبات ذلك فتهربت منه لفترة طويلة حتى بعد فترة طويلة توصلت إلى شيء لا علاقة له بالموضوع.
    وهذه مجرد أمثلة.
    وما فعلتموه الآن، عندما اخترتم تقديم سياسة الشرق الأوسط على أنها "ديناميكا حرارية"، هو تكرار النهج الذي اتبعتموه طوال المناقشات التي تجريونها، بافتراض تعميم واسع للظاهرة مع إهمال الأهمية. من التفاصيل الأساسية. ورغم أنه صحيح أن حروب دولة إسرائيل كانت «دائمًا» (كما لو كانت الدولة موجودة بالفعل أيام الانفجار الكبير) ضد المسلمين فقط، ولا يهم أين جلس مطلق النار في الخدمة، لكن هذه ما هي إلا أمثلة قليلة لظاهرة أوسع بكثير بدأت قبل وقت طويل من إنشاء الدولة وشملت أكثر من زمن لم يكن العالم يعرف فيه بعد إسلام محمد ولا حتى الديانة المنتظرية.

  62. روبي,

    "فكر بإيجابية"؟ إذا كان هذا هو حالك عندما تفكر بشكل إيجابي (الأقسام 1-4) فماذا يحدث عندما تفكر بشكل سلبي؟ القيامة؟ يوم القيامة ؟ يأجوج و ماغوغ؟

  63. ر.ح.
    "كيف ج = د كيف؟
    بعد كل شيء، هل كتبت عدة مرات على لوحة المفاتيح أن نسبة الجل في الساعة هي 1:1000,000؟

    إذن كيف أصبحا متساويين فجأة؟

    في رأيي، كل قصة من قصص جاك-جيل هي ببساطة نسخة من مفارقة التوأم الشهيرة.
    في هذه المفارقة، يتقدم أحد التوأم في السن والآخر لا يتقدم في السن.

    المرات الوحيدة التي كتبت فيها أن 1=1، وأن نسبة ساعات جيل كانت مختلفة عن XNUMX:XNUMX، كانت لإظهار أن مثل هذا الافتراض يقودنا إلى التناقض.

    مرة أخرى: لا يوجد فرق بين نظامي جاك وجيل. إذا كانت النسبة بالنسبة لجاك هي 1:1، فهي أيضًا بالنسبة لجيل. وأي افتراض آخر يؤدي إلى التناقض.

    قد تفترض أن نظام جاك "يستريح" حقًا وأن نظام جيل "يتحرك حقًا". ومع ذلك، لا يوجد أي دعم لهذا في بيانات المشكلة الأصلية. أي منطق تستخدمه لتوضيح أن نسبة ساعة جيل تختلف عن 1:1 يعتبر صالحًا تمامًا لنظام جاك.

    وفيما يتعلق بمفارقة التوأم - كما تعلمون، هذا نظام متسارع، والغرض من التغيير يختلف عن نظام القصور الذاتي. كما أن كل التأكيدات التي وجدتها لإطالة الأزمنة في التجارب (تجربة الطائرة، موانيم، تسيرن) تتناول فقط الأنظمة المتسارعة. أحب أن أسمع عن التجارب في أنظمة القصور الذاتي التي يتم فيها التعبير عن موضوع تمديد الوقت.

    اليوبيل.

    "صفة التخفي التي نسبتها إليك."

    وبما أنني أعتقد أنني أجيب على كل سؤال في المواضيع التي أثرتها بالتفصيل، سأكون ممتناً لو تفضلتم بتنويري في تلك الحالات التي تهربت فيها وخدعت ونثرت ستائر الدخان وغيرها من التعبيرات الملونة.

    إذا سألت، سأفعل ذلك بكل سرور فيما يتعلق بإجاباتك.

    من ناحية أخرى، إذا أثبتت، بعد الاطلاع على الموضوع، أنك لن تجد ولو ذرّة من الأدلة على الاتهامات غير المسؤولة التي تتهمني بها منذ الصباح، فمن المناسب أن تعتذر وتتوقف.

    روبي.

    فيما يتعلق بإيران، دعونا نرى ما إذا كان ما تعلمناه في هذا المقال يمكن أن يساعدنا في رؤية جوانب أخرى من القضية:

    الوضع السياسي حتى مارس 2012:

    الحدود الساخنة:

    لبنان
    غزة.

    حدود هادئة:

    مصر.
    الأردن.
    سوريا.

    الأعداء الحقيقيون: (الذين يطلقون النار)

    الفلسطينيين.
    لبناني.

    الأعداء في القوة (أولئك الذين يهددون)

    إيران
    ديك رومى.

    الجلوس في صمت نسبي:
    مصر. (اتفاقية سلام)
    أردنيون. (اتفاقية سلام).
    سوريون (حدود هادئة لسنوات عديدة).

    الوضع قبل 40 سنة:

    الحدود الساخنة:
    مصر.
    الأردن.
    سوريا.

    حدود هادئة:
    لبنان.
    غزة.

    الأعداء الفعليون (أولئك الذين يطلقون النار)

    مصر.
    الأردن.
    سوريا.

    الجلوس في صمت نسبي:
    الفلسطينيين.
    لبناني.

    الصداقات التي تكاد تكون حلفاء:
    إيران.
    ديك رومى.

    إذن ما الذي سيتغير؟

    ببساطة، قام جميع الأعداء والأعداء الأقل بتغيير أسمائهم وأدوارهم.

    خطر:

    سيكون هناك دائمًا بعض العرب أو المسلمين يطلقون النار علينا. كان اسمه كوكجي، ثم غير اسمه إلى عبد الله، ثم ناصر، والسادات، وعرفات، ونصر الله، وياسين، والآن هنية، وغدا أحمدي نجاد. لكن في الخلاصة، تحت كل كومة الملاعق، هو نفس العربي (أو المسلم).

    فلماذا؟
    هذا هو المكان الذي تأتي فيه هذه المقالة لمساعدتنا. هناك الفيزياء، والفيزياء لها قوانين، والفيزياء لها فرع الديناميكا الحرارية، والديناميكا الحرارية لها القانون الثاني، والقانون الثاني يقول أنه في النظام المغلق لا يمكن للإنتروبيا أن تتناقص من تلقاء نفسها، وأننا والمسلمين في هذه البيئة نظام مغلق تماما.

    ولذلك إذا صنعت السلام مع مصر - فسوف يطلق عليك لبنان النار. وإذا ضربت لبنان، ستطلق عليك غزة النار. وإذا ضربتم لبنان وغزة - فإيران التي لم تلحق بها ضررا قط، والتي لم تغتصبوا أرضها، والتي ليس لكم عليها شيء، والتي ستسعدون أن تكونوا صديقتها (كما ستكونون معها). سائر العرب والمسلمين) -
    سوف تكون فجأة هي التي تنقض عليك.

    بسيط جدا: الديناميكا الحرارية.

  64. إيران ستهاجم إسرائيل في أي حالة هجوم أمريكي مشترك أم لا.
    وستكون القوة مختلفة، وحججها لمهاجمة إسرائيل لن تكون صالحة أيضاً رغم أنها لن تهمنا.
    يجب على الإيرانيين أن يخافوا من وضع قد "تصاب فيه إسرائيل بالجنون" نتيجة للأضرار "غير المعقولة" التي ستسببها.

  65. شكرا روبي، أشعر بنفس الطريقة التي تشعر بها. ولكنني أخشى أيضاً أن أي هجوم من جانب الولايات المتحدة أو حلف شمال الأطلسي سوف يعطي إيران سبباً كافياً لمهاجمة إسرائيل حتى لو لم تتدخل.

  66. فيما يتعلق بإيران:
    1. يمكن لإيران شراء قنابل نووية من كوريا الشمالية وإدخالها إلى إيران باستخدام غواصة أو جوية أو أي وسيلة نقل برية أخرى. إن الحجة حول خطورة الاحتفاظ بالقنبلة لا تصمد هنا.
    2. القدرة على تطوير قنبلة موجودة بالفعل، ولن يؤدي أي قصف إلى القضاء عليها بشكل نهائي، بل قد يؤدي فقط إلى تأخيرها.
    3. هدف إسرائيل والغرب هو الضغط على إيران ونبذها وإحداث عملية تغيير الحكومة
    4. اهدأ، إسرائيل لن تقوم بالقصف الأول وتترك للولايات المتحدة وحلف شمال الأطلسي أن ينفذه
    وعلى النقيض من القصف في العراق، الذي كان معزولا نسبيا، فإن إيران حليفة لحزب الله وحماس وآخرين، وبالتالي فإن الرد على إسرائيل سيكون مؤلما للغاية.
    فكر بإيجابية

  67. وهذا ما أحاول أن أفعله مع إسرائيل، لكنه لا ينجح.
    ربما ينبغي لنا أن نسافر إلى المريخ من أجل ذلك

  68. R.H.، أرجو أن تتقبل اعتذاري. لم أقصد العبث بميولك السياسية. ببساطة، بما أننا اتفقنا على أن دولة إسرائيل لن تكون قادرة على التصرف دون صداقتها الكبيرة، ويكون لدى المرء انطباع بأن الرئيس الحالي ليس في عجلة من أمره للوصول إلى أي مكان...

  69. R.H.، نوعية كلماتك مقبولة بالنسبة لي. دعونا نواجه الأمر، إذا كانت إسرائيل مهتمة بشن هجوم عسكري، فلن تكون قادرة على التحرك بمفردها. كل شيء آخر هو تكهنات. هل صوتت لصالح ميت رومني؟

  70. إسرائيل! مرة أخرى تخترع اتصالات حيث لا يوجد شيء.
    لم أطعن في صحة النظرية النسبية، بل في صحة الاستنتاجات التي قد تتوصل إليها أنت وR.H. عندما لا تكون لديكم طريقة لاختبارها عمليا.
    عبارة "يُظهر لك المكان الذي تتبول منه" عبارة عن مزيج من عبارة معروفة مع ميزة التخفي التي نسبتها إليك. لقد اعتدت بالفعل على عدم الفهم، وهذا يشمل محاولاتي للمزاح.
    لا أستطيع التعليق على تعليقك بشأن الفتيات في حانات جلاسكو، فأنا لا أزور تلك الأماكن. ويمكن تطبيق تعليق Point بسهولة على ما تقوم بتشغيله هنا أيضًا.
    مرة أخرى، تمكنت من جرني إلى جدالات غبية. كيف تفعل ذلك؟ ما هي خدعتك كتاب من فضلك، كتاب.

  71. اليوبيل,

    فكيف تمنعهم من الحصول على قنبلة دون هجوم؟ ربما سنطلب واحدة لطيفة جدًا جدًا جدًا؟ ربما سنرسل بار رفائيلي؟ أو كاتلين رايتر لتغني لهم؟

    هل يمكننا البناء أو الاعتماد على معارضة كيكيون لإسقاط النظام قبل يوم القيامة؟ فهل من خطوة دبلوماسية أو عقوبات تقنعهم؟

    هل تذكرون كور تموز 1981؟ ثم وقف شمعون بيريز أيضا وقال: الويل لنا إذا هاجمنا. لقد هاجمنا وكنا محظوظين لأنه لولا ذلك لكان من الممكن أن نحصل على صاروخ سكود ذري ​​في عام 1981 بدلاً من تلك التي سقطت برأس خرساني.

    والأكثر من ذلك، لا أعتقد أن إسرائيل قادرة وحدها على تدمير البرنامج النووي الإيراني، ويجب أن يكون الهجوم دولياً أو أميركياً على الأقل.

  72. آر إتش،
    أفترض أنه إذا تمكنت إيران من إسقاط قنبلة ذرية على تل أبيب، فإنها لن تطالب بالانسحاب إلى حدود عام 1948، بل بعودة كل اليهود إلى بلدانهم الأصلية (بل والأفضل تدميرهم، كما أوصى القرآن). والقضاء على دولة إسرائيل والكيان الصهيوني بشكل عام.
    ولابد من اتخاذ كل التدابير اللازمة لمنع إيران من الحصول على القنبلة النووية، ولكن الحرب لن تساعد، بل ستضر فقط. وإذا هاجمت إسرائيل إيران، فإن النظام الإيراني سيحصل على الشرعية الكاملة في نظر المعارضة الإيرانية للحصول على القنبلة النووية. في رأيي أن مهاجمة إيران لن تكون إلا مكلفة ولن تكون مفيدة على الإطلاق.

  73. يوفال
    وافعل ذلك من خلال التجارب الفكرية - التي تطعن في صحتها بالقول: "هذا جدل عقيم لأنه يتناول تجارب فكرية ليس لدينا وسيلة لإثباتها أو دحضها".

    وكلماتك: "لقد استنتجت "رغما عن إرادتك" من نهج غير تجريبي تماما (وأيضا غير عقلاني، بالمناسبة)."

    ويمكن الاستدلال عليه بأشياء قلتها سابقًا مثل: "أظهر لك بصبر لا نهاية له أنني أفتقد المكان الذي تتبول منه".

    يكفي أن تلعب دور طفلها الصغير. لقد أوضحت لك Point بالفعل في الماضي أن نماذجك تعتمد بشكل أساسي على درجة جهل قرائك. وهذا يشمل أيضًا نماذج العدوان. ومثل العارضات الأخريات، فهو لا يتجاوز عتبة الفتيات في حانات جلاسكو. يمكن أن يترك انطباعًا عليهم، وليس علينا.

  74. اليوبيل,

    وماذا ستفعل كرئيس للوزراء يوم يقدم الإيرانيون قنبلة ذرية ويعلنون أنه إذا لم تعد إسرائيل بحلول نهاية الشهر المقبل إلى حدود 1948 فسوف يسقطونها على تل أبيب؟ ماذا بعد؟ دعونا نلعب البوكر ونرى ما إذا كانوا يخادعون أم أنهم جادون؟
    هل يمكنك أن تأخذ مثل هذا الرهان؟ وإذا استسلمنا؟ هل ستكون هذه نهاية المتطلبات؟

    ضد المبتز هناك طريقة واحدة فقط.

  75. لم أفهم العلاقة بالقنبلة الإيرانية. ولكن بما أنك سألت، فلك رأيي: القنبلة النووية، بعد هيروشيما وناكازاكي، هي أداة ردع تعطي لصاحبها هيبة واحتراماً بين الأمم. وعندما تصبح إيران على هذا النحو، سيكون لها نطاق سياسي أكبر مما هي عليه اليوم.
    الإنسانية تنتقل من حرب إلى حرب. تحتاج كل دولة تقريباً إلى الحرب من وقت لآخر، وإيران ليست استثناءً. أما دولة إسرائيل، من ناحية أخرى، فليس لديها ما تكسبه من الحرب. لديها الكثير لتخسره.
    الحرب مع إيران لن تكون نووية، لكن الإيرانيين يعرفون الكثير من الحيل ويجب على إسرائيل ألا تستهين بها.
    إنهم ينتظرون أن تهاجم إسرائيل حتى يكون لديهم ذريعة لضرب إسرائيل دون أن يتهموا بأنهم معتديون. إنهم يستفزون إسرائيل للهجوم. إنهم يفعلون ذلك من خلال تصريحات تهديدية، واستفزازات مثل الهجمات غير الاحترافية على السفارات الإسرائيلية في العالم، ودعم الأنظمة التي تكره إسرائيل، وتدريب وحدات عسكرية في دول معادية لإسرائيل، وغير ذلك. بل إنهم الآن يقدمون إسرائيل على أنها من يخطط بالتأكيد للهجوم أولاً، مما يمهد الطريق لتبرير الحرب بمبادرة منهم. تذكروا كيف اندلعت حرب لبنان الثانية: حزب الله، التابع للإيرانيين، خلق استفزازًا لجر دولة إسرائيل إلى إجراء عقابي صغير تطور إلى حرب أصبح من الواضح خلالها أنه مدرب جيدًا ومجهز بشكل مناسب أسلحة من الدرجة الأولى. أتذكر مخبأً مهجوراً لجيش الدفاع الإسرائيلي في لبنان حيث اختبأ جنود حزب الله وانتظروا بهدوء لعدة أسابيع ولم يكن الجيش الإسرائيلي مريباً على الإطلاق.
    إذا كنتم ستسألون في رأيي، فيجب على دولة إسرائيل ألا تعطي الإيرانيين أدنى عذر لبدء الحرب. ويتعين عليها أن تدلي ببيان عام شامل مفاده أنها تحترم الشعب الإيراني وتعتقد أنه يسعى إلى السلام وأنها لن تهاجمه تحت أي ظرف من الظروف. وفي الوقت نفسه، يجب أن تكشف أمام أعين العالم كل الأدلة التي تمتلكها المخابرات الإسرائيلية (وهناك الكثير بالتأكيد) حول النوايا العدوانية للنظام الحالي في إيران ليس فقط ضد إسرائيل ولكن أيضًا ضد الدول الأخرى. تشجيع توسيع العقوبات الاقتصادية الدولية.

  76. إسرائيل،

    1) كيف C=D كيف؟
    بعد كل شيء، هل كتبت عدة مرات على لوحة المفاتيح أن نسبة الجل في الساعة هي 1:1000,000؟

    إذن كيف أصبحا متساويين فجأة؟

    2) في رأيي، قصتك مع جاك-جيل هي ببساطة نسخة من مفارقة التوأم الشهيرة.
    في هذه المفارقة، يتقدم أحد التوأم في السن والآخر لا يتقدم في السن.
    ولا تقتصر المفارقة على التسارع.
    وقد تمت دراسة هذه المفارقة ودراستها بشكل مكثف وسكبت فيها ملايين الكلمات والإجماع على أنها ليست مفارقة ولا تناقض مع توسع الأزمنة. علاوة على ذلك، فقد تم إثبات ذلك تجريبيًا في الطائرات وفي الجسيمات الموجودة في المسرع. إذن قصة جاك وجيل لا تشكل تناقضا بين العلاقة مع الانفجار، ولا يوجد مفارقة هنا.

  77. رجل.
    اذهب إلى الموقع
    http://hyperphysics.phy-astr.gsu.edu/hbase/astro/expand.html#c3
    هناك صيغة هناك - صيغة فريدمان لاعتماد الوقت على درجة الحرارة.
    توجد أيضًا آلة حاسبة يمكنك استخدامها لتحويل الوقت إلى درجة حرارة والعكس صحيح.

    إذا لم يكن هناك اتصال مباشر بين الأوقات التي تنتجها درجة الحرارة والوقت اليومي الطبيعي، فلن نتمكن من الحديث عن كون عمره 13.7 مليار سنة، لأنه سيتم تفسيره على أنه شيء مختلف بالنسبة للجميع. وبالتالي فإن السنة الحرارية تساوي السنة العادية.

  78. ر.ح.
    لقد كان طريقا مختصرا.
    الجملة الأصلية كانت: إذا كان a=b، c=d، وa=c، فإن a=b=c=d.
    أ - جاك ساعة درجة الحرارة.
    ب - الساعة 3:00 مساءً جاك.

    ج – ساعة تيمبي جيل .
    د – ساعة جيل .

    يظهر A وB دائمًا نفس الوقت (نظام القصور الذاتي).
    يظهر C وD دائمًا نفس الوقت (نظام القصور الذاتي).

    يظهر A وC نفس الوقت في وقت الاجتماع (الساعات المؤقتة).
    وبالتالي: أ=ب=ج=د.

    وليس هناك تمديد للوقت.

    خاص.

    ما كتبته في الرابط الذي أضفته هو:

    "المشكلة هي:

    1. كل من جاك وجيل في حالة راحة بالنسبة لهما. وأنظمتها ليست متسارعة، وتسمى في لغة النسبية "بالقصور الذاتي". ولو أمكن التفريق بينهما بعملية عد الدورات مثلا لكان هذا مناقضا للمسلمة الأولى للنسبية."

    وهذا دليل على نفي عدم وجود مثل هذا الوضع.
    ولذلك فإن نسبة ساعات جيل يجب أن تكون نفس نسبة ساعات جاك.

    إذا لم تكن قد قرأت الملخص بعد، يمكنك التبديل إلى مقال آخر مضى عليه عامين، حتى يتم التحميل بسرعة.

    يوفال - اخترع أينشتاين كل النسبية دون مغادرة المكتب، وتجارب فكرية. هل هذا يعني أنه بإمكاننا التوقف عن الخوف من القنبلة الإيرانية؟

  79. حسنا حسنا حسنا

    هذه المناقشة أصبحت أكثر وأكثر قمعية. كل من وقت تحميل الصفحة والحظر الدائم للتعليقات.

    آفي بيليزوفسكي،

    أقترح عليك أن تفعل وردية من الكلمات المحظورة الخاصة بك. لا معنى لهم. انظر تعليقي الأخير. أنت تتسبب في تعقيم المناقشات.

  80. إسرائيل،

    كيف أ=ب و ج=د إذن أ=ب=ج=د؟

    نسيت ما كتبته في:
    https://www.hayadan.org.il/astronomers-reach-new-frontiers-of-dark-matter-130112/#comment-331692

    ". إذا قارن جاك سوفير عدد الدورات بين الساعتين (ومن الممكن بالتأكيد ترتيب أن تظهر ساعات السيزيوم ودرجة الحرارة الوقت في دورات الساعة) فسوف يرى نسبة 2:1. ليس عليه في الواقع الانتظار طوال الوقت حتى اللقاء. وفي أي فترة زمنية محددة، ستكون النسبة هي نفسها.

    2. إذا قامت جيل بالعد، فستحصل على نسبة مثلاً 1,000,000,000:1"

    فكيف أ=ب و ج=د إذن أ=ب=ج=د ؟؟؟؟؟

  81. إسرائيل!
    يمكن استنتاج الكثير من الأمور، اعتماداً على الافتراضات الأولية التي تضاف إلى الصيدلية. لقد استنتجت "ضد إرادتك" من نهج غير تجريبي تمامًا (وأيضًا غير عقلاني، بالمناسبة).
    إنه نقاش عقيم لأنه يتناول تجارب فكرية ليس لدينا وسيلة لإثباتها أو دحضها. أتذكر بالضبط تلك المناقشات، حول النظرية النسبية ومفارقة التوأم، التي دارت في الصف السابع مع اثنين من زملائي (كان ذلك أثناء التدريب العام على العرض، أسلوب كوريا الشمالية، عشية عيد الاستقلال في استاد كوريا الشمالية). الجامعة العبرية في جفعات رام عام 1966، عندما كنا نجلس معظم الوقت على سطح مرتفع وننتظر التعليمات) وحتى ذلك الحين توصلت إلى نفس الاستنتاجات التي أتمسك بها حتى يومنا هذا.
    لقد لاحظت أن الناس يميلون إلى الانجرار إلى مثل هذه الحجج. وفي غياب تعريف أكثر دقة لأسباب هذه الحاجة الفكرية المستهلكة للوقت، فإنها تسمى "الإرادة الحرة". مسألة ما إذا كانت هناك إرادة حرة أم لا تنتمي إلى محاكمة أمام هيئة محلفين. قبل أسابيع قليلة خطر ببالي أن أقترح عليك أن تصبح محاميًا جنائيًا لأنه يبدو لي أن لديك كل المواهب المطلوبة للنجاح بشكل جيد في هذا المجال، وفي هذا الرأي أنا أزداد قوة كل يوم.

  82. إلى المعنيين،
    ومن المثير للاهتمام قراءة الموضوع.
    هل من المؤكد أن درجة الحرارة ليست كمية نسبية؟

  83. ر.ح.
    منذ الأنفلونزا، أذهب إلى الفراش مبكراً. يمكنك رؤية بداية مناقشتنا في
    https://www.hayadan.org.il/astronomers-reach-new-frontiers-of-dark-matter-130112/#comment-331534
    لاحظ أنني لم أقبل أبدًا الافتراض بأن ساعات جيل ستظهر أوقاتًا مختلفة. هذا نظام في حالة سكون، حيث تعرض كلتا الساعتين نفس الوقت، تمامًا مثل ساعات جاك. وإلا فإنه سيكون من الممكن التمييز بين أنظمة القصور الذاتي، خلافا للمسلمة أ. وصحيح أن هذا لا يتفق مع النسبية، ومن هنا جاءت المناقشة.

    رابع. نظرت إلى الرابط الخاص بك. التوأم الأصغر يستدير. لا يمكنك الالتفاف دون الإسراع (أو التباطؤ - لقد كنا أنت).

    مساء الخير.

  84. أرني عندما وافقت على B.
    بعد كل شيء، أحاول دائمًا أن أوضح لك أن ساعات جيل يجب أن تظهر نفس الوقت الذي تظهر فيه ساعات جاك. تذكر: إذا كانت A=B وC=D إذن A=B=C=D؟
    يشير C و D إلى ساعات جيل.

  85. إسرائيل،

    أما بخصوص الاقتباس فقد أجبت عليك أعلاه. ولكن دعونا ندعو سقراط مرة أخرى

    و. هل تظهر ساعتا جاك نفس الوقت؟ أعتقد أننا اتفقنا بالفعل على ذلك، نعم، أليس كذلك؟ لتذكيرك 1:1

    ب. هل هناك فرق بين ساعات جيل؟ لقد اتفقنا بالفعل على أن هناك فرقًا، أليس كذلك؟ لتذكيرك 9^1:10

    ثالث. إذا أجبت بنعم على كلا السؤالين، فكيف يمكن أن تدعي في الإجابة 6 "أدعي أن الوقت نفسه قد مر لكليهما، سواء في ساعات تشيكوسلوفاكيا أو في الساعات المؤقتة"؟ ألا ترى أنك تناقض نفسك؟

    رابع. كيف في مفارقة التوأم أن يكبر أحد التوأمين (ويسمى جاك) بينما يظل الآخر الذي ذهب بعيدًا (ويسمى جيل في قصتنا) صغيرًا إذا كان الوقت هو نفسه؟ ولا تخبرني بأنظمة التسريع، اقرأ الاقتباسات التي أحضرتها لك أعلاه من مدخل Twin Paradox والتي توضح أنه ليس شرطًا ضروريًا.

    ال. هذا ليس سؤالا بل اقتراحا. تحقق من إجاباتك على السؤالين (أ) و (ب). ليس من الممكن أن تكون إجابتك بـ "لا" لأننا اتفقنا عليها بالفعل وكانت في الواقع بلاغية كما كانت عادة سقراط القديم، فهل من الممكن أنك لم تقرأ بعناية وأجبت بشكل عرضي؟

    و. أما فيما يتعلق بحجتك مع يوفال، فهذا صحيح وغير صحيح. صحيح أنني بريء من أي خطأ، وليس صحيحاً أنك جرتني رغماً عني.

  86. ر.ح.

    إذا لم نغير أي شيء في البيانات الأصلية، فإن سقراط يجيب.

    و. لا.
    بدون.
    ثالث. نعم.
    د. لا. (يبدو لي أنك عكست الترتيب. مع جاك، يمر الوقت بسرعة. ومعه، مر مليار سنة.)
    ال. لا. (كما هو مذكور أعلاه، عكس).
    و. أنا لا أدعي ذلك. أزعم أن الوقت نفسه قد مر بالنسبة لكليهما، سواء على الساعات أو على الساعات المؤقتة. هذا هو بالضبط التناقض مع نظرية الانفجار - موضوع مناقشتنا.
    ز. إذا كان تلميذه أفلاطون فليقتل نفسه.

    والآن على سؤالي: هل تقبل، وفقًا للنظرية النسبية، أن ساعة جاك تتحرك ببطء بالنسبة إلى الساعات في نظام جيل، تمامًا كما تتحرك ساعة جيل ببطء بالنسبة إلى الساعات في نظام جاك؟

    أجبت: "بالطبع لا!"

    كيف يتم تكديس هذا مع:

    عندما يكون هناك راصدان في حركة موحدة نسبية ولا يتأثران بأي كتلة جاذبية، فإن وجهة نظر كل منهما ستكون أن ساعة الآخر (المتحركة) تدق بمعدل أبطأ من الساعة المحلية.

    ألا يتناسب جاك وجيل مع التعريف هنا بشكل سريع؟

    وفيما يتعلق يوفال:

    لاحظ تعليقه:

    "ربما ستظل قادرًا على انتزاع إجابة مني لا أستطيع ضمان صحتها، لكنها ستكون كافية لجرني إلى جدال عقيم لا نهاية له كما فعلت مع صديقنا الذي هو بريء من أي مخالفة".

    ومن هنا يمكن أن نستنتج أنني قمت بجرك ضد إرادتك، مثل طفل أسير، إلى بعض الحجج الفارغة التي لم تكن مهتمًا بها ولم تكن مهتمًا بها.

    وبما أنني في هذا الأمر تجريبي ولا أخوض في التكهنات، فسأسألك مباشرة: هل هذا صحيح؟ هل قمت بجرك إلى جدال ما ضد إرادتك؟ والأهم من ذلك، هل تريد أن ننتهي الآن؟

  87. إسرائيل،

    وأخيرا، خرجت رسائلي من صحافة النظام نظيفة وواضحة. لذا يرجى التمرير لأعلى إلى مشاركة روبي والرجوع إليها.

    يوفال وإسرائيل الجميلان والممتعان،
    فهل انتقلت إلى مرحلة الغناء؟ إندهشت من حبك للنساء، هل وصلت إليه بعد؟ هل قمت بالفعل بتنظيم رحلة إلى بروكباك؟

  88. وكيف يستنتج تجريبي مثلك أن هذه حجة فارغة؟ ولماذا تعتقد أنه تم جره؟ هل تابعت الموضوع أم أنك قرأت تعليقاتك فقط؟

  89. إسرائيل، شيئًا فشيئًا تكشف وجهي. حقا قاسية من الأنانية القاسية.
    اختبأت خلف لافتة "التجريبي" وبهذا كنت آمل أن تفهم سبب عدم إجابتي. لكنك، أيها العنيد في العالم، ربما لا تزال قادرًا على انتزاع إجابة مني لا أستطيع أن أضمنها، لكنها ستكون كافية لجرني إلى نقاش عقيم لا نهاية له كما فعلت مع صديقنا البريء من أي خطأ. .
    بالمناسبة قاسية، يتطلب الأمر معرفة واحدة

  90. يوفال
    لأن هنا تنظيف الرأس ينفد،
    آه كم سيكون الغد جميلاً
    أوه كم هو وسيم
    وما هو سعيد

    ولماذا على سؤالي
    لم تجب أيها القاسي

  91. إسرائيل،

    وبقدر ما أعرف، فإن مفارقة ديلا توين صالحة أيضًا في الأنظمة غير المتسارعة. حتى أنني ذهبت للتحقق من ذلك بالنسبة لك. انظر هنا: http://en.wikipedia.org/wiki/Twin_paradox

    وخاصة تحت الأقسام:
    مثال محدد وحل المفارقة في النسبية الخاصة

    على أي حال. لقد ضلنا قليلا. ما هو بالضبط ادعاءك؟ هل ستظهر نفس ساعة السيزيوم الخاصة بجيل وجاك؟ لقد وافقت على عدم القيام بذلك. أعني أن جاك أصبح أكبر سنًا من جيل، فماذا تريد مني؟

  92. ♫ لذا، في الوضع الحالي، ليس كل شيء على ما يرام ♪
    ♫ فقط ما الذي يحتاجه الإنسان، إن لم يكن الحفاظ على رأس نظيف ♪
    إذا قمت باستدعاء الترانتيلا التي لا نهاية لها على أصوات الكمان (غير المضبوط جدًا) لأسرع عازف كمان في الكون "ينظف الرأس"، فأنت تشير إلى فولاذ شديد الصلابة. الصلب الماسي. المجد للصبر والمثابرة 8) (شيء إيجابي آخر تعلمته منك 🙂 )

  93. ر.ح. شاركنا المقال، وألقِ عظمة للأشرار الأسطوريين السحريين.

    مفارقة التوأم هي الأنظمة المتسارعة. نحن حاليا في أنظمة القصور الذاتي. هل أنتم موافقون على ما ورد في البيان، دون الإضافة التي لا تتعلق بشؤوننا حاليا، والتي قد نصل إليها لاحقا؟ هل ينطبق ذلك على أنظمة جاك وجيل؟

  94. اليوبيل,

    أنا أيضًا تجريبي. في الواقع هذه هي وظيفتي. العلم بالنسبة لي هو العلاج. بين الرسوم البيانية والمقال الذي أكتبه، ألقي نظرة وأصفي ذهني قليلاً.

  95. وعلى العكس من ذلك، بالطبع، كيف يمكن أن يشيخ التوأم الموجود على الأرض فقط ويبقى التوأم الموجود على تيس شابًا إذا كان وقت كل منهما يمر بشكل أبطأ بالنسبة للآخر؟

  96. إسرائيل،

    لماذا لا تنظر إلى الأسفل قليلاً في الصفحة التي نسخت منها في البيان:

    إن تباطؤ الزمن من شأنه أن يتيح للركاب في مركبة سريعة الحركة السفر أبعد إلى المستقبل بينما يتقدمون في السن قليلاً، حيث أن سرعتهم الكبيرة تؤدي إلى إبطاء معدل مرور الوقت على متن الطائرة. أي أن ساعة السفينة (وبحسب النسبية أي إنسان يسافر معها) تُظهر وقتًا منقضيًا أقل من ساعات المراقبين على الأرض. للحصول على سرعات عالية بما فيه الكفاية يكون التأثير مثيرًا.

    هل تعرف مفارقة التوأم؟ كيف يتم أن يكون عمر التوأم الطائر فقط؟

  97. ر.ح. وأنا أتفق معك، والسؤال هو ما يدعي يوفال.

    قبل أن نواصل ماذا عن العبارة التالية:

    عندما يكون هناك راصدان في حركة موحدة نسبية ولا يتأثران بأي كتلة جاذبية، فإن وجهة نظر كل منهما ستكون أن ساعة الآخر (المتحركة) تدق بمعدل أبطأ من الساعة المحلية. كلما زادت السرعة النسبية، زاد حجم تمدد الزمن.

  98. وكالعادة الرد في انتظار التأكيد. هل لدى أي شخص قائمة بالكلمات "المحظورة"؟

    إسرائيل،
    ما هو السؤال؟ ومن الواضح أن هذا هو الحال. الساعة الحرارية عبارة عن مسبار أو مقياس حرارة يخرج من المركبة الفضائية إلى الفضاء ويقيس درجة الحرارة. ما في الفضاء هو ما سيراه الحرير البني والكاميرات التي تلتقطه.

  99. الذهاب لذلك بالمناسبة، الصيغة تسمى صيغة فريدمان. سؤال: إذا قلنا أن هناك سفينتين فضائيتين مزودتين بساعتين مؤقتتين وتمران ببعضهما البعض بسرعة عالية، فهل الكاميرات عالية الدقة الموجودة في السفن الفضائية والتي تلتقط الساعات الحرارية في كلتا السفينتين الفضائيتين ستظهر نفس الوقت على كلتا الساعتين الحراريتين؟

  100. إسرائيل،

    أنت تفتقد تمامًا طراز السيارة وتشابه سفينة الفضاء. إذا أصررت، فلا توجد مشكلة في إنشاء دالة المسافة كدالة ذات قيمة واحدة (1-1):
    نقطتان على مسافة كيلومتر من بعضهما البعض. يتحرك جاك ببطء شديد في خط مستقيم بين النقاط ويكمل المسافة في ساعة واحدة.
    يسافر جيل بسرعة عالية في خطوط متعرجة بين النقاط ويكمل المسافة في ساعة واحدة.
    يحتوي كلاهما على عداد المسافات ونظام تحديد المواقع العالمي (GPS) الذي ينتج رسمًا بيانيًا للمسافة بمرور الوقت.

    يسأل سقراط:
    1) هل سيكون الرسم البياني الذي يظهره عداد المسافات الخاص بجاك خطيًا؟
    2) هل سيكون الرسم البياني الذي يعرضه نظام تحديد المواقع العالمي (GPS) خطيًا؟
    3) هل سيكون الرسم البياني لجيل من عداد المسافات على العجلة هو المسافة الخطية مقابل الزمن؟
    4) هل سيكون الرسم البياني الذي ينتجه نظام تحديد المواقع العالمي (GPS) خطيًا؟ وأؤكد على عدم التكرار أو التراجع، الخطي.

    ليس عليك أن تجيب. الجواب هو نعم لجميع الأسئلة. لكن لا تقلق ستكون هناك أسئلة أيضًا.

    إذن ماذا لدينا هنا؟ نظام محلي في السيارة يقيس المسافة (يشبه تماماً ساعة السيزيوم) ونظام مرجعي خارجي، أي نظام تحديد المواقع العالمي (يشبه تماماً ساعة درجة الحرارة). والعجيب والعجب في الأنظمة الخارجية (GPS والساعة المؤقتة) المسافة في نموذج السيارة والوقت في نموذج سفينة الفضاء هي نفسها بين جاك وجيل،
    و. يسأل سقراط: موافقة؟

    العجيبة والعجيبة 2، في الأنظمة المحلية، أي عداد المسافات في السيارة أو ساعة السيزيوم في المركبة الفضائية، هناك فرق كبير بين جاك وجيل.
    ب. سقراط: موافقة؟

    التشبيه واضح. ولا بد من النظر إلى الزمن باعتباره بعدا ومن ثم يصبح القياس على الأبعاد الثلاثة "العادية" واضحا.

    على سؤالك: ". هل تقبل، وفقًا للنظرية النسبية، أن ساعة جاك تتحرك ببطء بالنسبة إلى الساعات في نظام جيل، تمامًا كما تتحرك ساعة جيل ببطء بالنسبة إلى الساعات في نظام جاك؟

    الجواب: بالطبع لا! دعونا ننظر إلى الأرقام. على ساعة درجة الحرارة، انخفضت درجة الحرارة لكل منهما من 6000 كلفن إلى 2.73.
    ثالث. سقراط: موافقة؟

    في المقابل، كانت ساعة السيزيوم الخاصة بجاك تحتوي على اضمحلال X الذي يتوافق مع ساعة واحدة.
    رابع. سقراط: موافقة؟
    في حين أن جيل كان لديه X^10 من الاضمحلال الذي يتوافق مع مليار سنة.
    ال. سقراط: موافقة؟

    لذا في الختام سقراط، قبل أن يتناول كأس السم يصرخ، لا أستطيع أن أموت حتى أفهم!
    و. كيف يمكن ذلك يدعي شابيرا أن ساعة جاك يمكن أن تتحرك بشكل أبطأ من ساعة جيل؟

    ز. هل ستتركه يقتل نفسه بسلام؟

  101. تحسب هذه الأدوات الوقت الذي انقضى منذ الانفجار الكبير وفقًا لصيغة التبريد (التي تعتمد على أي افتراضات حول معدل توسع الكون) وتقوم بتركيب بيانات درجة حرارة إشعاع الخلفية الكونية في اللحظة التي يتم اختبارها. إذا بدا لك أن اسم "ساعة درجة الحرارة" يجسد هذه الوظيفة، فابحث عنها.

  102. وفيما يتعلق بالجدل حول من يسافر بسرعة الضوء تقريبًا ومن يستريح، عندي لك الحل، بعد الرحلة سيجتمعون ويرون من كبر ومن بقي شابًا...

  103. ر.ح. في مثال السيارة المزودة بنظام تحديد المواقع العالمي (GPS)، من الممكن تمامًا أن ترى نفس السيارة في نفس النقطة مسافات مختلفة، وهو أمر غير ممكن فيما يتعلق بالعلاقة بين درجة الحرارة والوقت.

  104. ر.ح.
    صفقة عظيمة، أسئلة سقراطية. أعتقد أننا التقينا تقريبًا.

    أولاً: الأجوبة على أسئلتك:

    1. نعم.
    2. رقم أ-ف.
    ز نعم.
    3. لا
    4. لا.
    5. نعم. تقيس ساعات درجة الحرارة درجة حرارة فريدة تعتمد على الوقت وفقًا لصيغة مستمرة لا يمكن تكرار نتيجتها لمعلمات مختلفة، ما يسمى بالدالة 1-1. ولكل وقت معين في الدالة، ترتبط درجة حرارة فريدة فقط. في حالة السيارات المجهزة بنظام تحديد المواقع العالمي (GPS)، فمن الممكن تمامًا أن ترى سيارتان في نفس النقطة مسافات مختلفة. في رأيي، يمكنك التخلي بهدوء عن السيارات المزودة بنظام تحديد المواقع العالمي (GPS) والذهاب مباشرة إلى دراجة التمرين: سوف تظهر لك مسافة كبيرة، دون تحريك ملليمتر واحد. باختصار - لا أرى أي صلة بكتبنا.

    6. نعم.

    أعتقد أننا جمعنا ما يكفي لاختزال كل شيء في سؤال واحد:

    1. هل تقبل، وفقًا للنظرية النسبية، أن ساعة جاك تتحرك ببطء بالنسبة إلى الساعات في نظام جيل بنفس القدر من البطء الذي تتحرك به ساعة جيل بالنسبة إلى الساعات في نظام جاك؟

    اليوبيل.

    لقد فهمت أنك تقول أنه من الممكن نظريًا بناء ساعة تُظهر الوقت العالمي المطلق فقط عن طريق قياس درجة الحرارة.

  105. إسرائيل،
    إذا لم يتم تعريف بروتوكول الاتصال بشكل صحيح، فمن المحتمل أن تكون هناك فجوة بين ما يقوله المرء وما يفهمه الآخر. قلت: "بافتراض أن درجة حرارة إشعاع الخلفية الكونية هي نفسها في كل مكان في الكون، فلدينا ساعة زمنية عالمية مطلقة." أخبرني بما تفهمه وسأرى إن كان بإمكاني النفي أو التأكيد.

  106. يوفال
    أنت تكتب "بافتراض أن درجة حرارة إشعاع الخلفية الكونية هي نفسها في كل مكان في الكون، إذن لدينا ساعة زمنية عالمية مطلقة ("جرينتش مجرة، وفقًا لتعريفك)" هل تقصد ذلك؟

  107. حسنًا، إجاباتي تنتظر الموافقة. بالمناسبة، خرج الوجه الصوتي عن طريق الخطأ من ثمانية + قوسين. من الآن فصاعدا نحن نحسب فقط بالنقاط.

    لذا، في هذه الأثناء، وحتى يتم إصداره، لدى سقراط أيضًا بعض الأسئلة:

    1. هل توافق على أن نسبة ساعة جاك للسيزيوم إلى درجة الحرارة هي 1؟

    2) هل تعتقد أن هذه العلاقة ستتغير إذا (ضع علامة على الإجابات الصحيحة):
    و. هل ستكون جيل في راحة؟
    ب. هل ستتحرك جيل بسرعة 100 كم/ساعة بالنسبة لجاك؟
    ثالث. هل ستتحرك جيل بسرعة 0.99 من سرعة الضوء بالنسبة لجاك؟
    رابع. هل ستؤدي جيل حركة الرقم ثمانية في الهواء، وحركة خلفية وركلة كونغفو؟
    ال. جيل لن تكون موجودة على الإطلاق؟
    و. هل جميع الإجابات A-F صحيحة؟
    ز. هل جميع الإجابات A-E غير صحيحة؟

    3) هل تعتقد أن إضافة جين آرثر أو يوهانس دي جروت إلى القصة سيغير نسبة 1:1 بين ساعات جاك؟ حتى لو كان آرثر يقود سيارته للخلف؟

    4) هل ترى تناقضا بين العلاقة بالانفجار العظيم في قصة السيارات المجهزة بعدادات المسافات ونظام تحديد المواقع؟

    5) هل ترى فرقًا جوهريًا بين قصة السفن الفضائية المزودة بساعة السيزيوم وساعة شابيرا الحرارية وقصة السيارات المجهزة بعدادات المسافات ونظام تحديد المواقع العالمي (GPS)؟ إذا كان الأمر كذلك، نخر.

    6) هل مازلت ترى أن التناقض بين النسبية والانفجار الأعظم نابع من اختراع ساعات درجة الحرارة؟

    الدفاع يستريح.

  108. 1) نعم
    2) نعم
    3) نعم
    4) نعم
    5) لم أفهم. هل يتحرك الجين بنفس سرعة جيل؟ (أفترض أن هذا هو ما تعنيه بـ "snoochronous"). ما علاقة المسافة بجاك؟ وماذا عن النظام المرجعي لجيل؟
    6) ليس حقا. إذا كانت جيل هي التي تشغل المحركات وتطفئ الدخان خلفها وتقترب من مجرة ​​المرأة المسلسلة حيث يوجد جاك، فصحيح أنه يمكنك أن تتفلسف وتقول إن جيل في الواقع تقف في مكانها والكون كله بما في ذلك جاك الذي يستريح فيه. هو الذي يتحرك. عندما تذهب للعمل هل أنت في راحة والعمل يأتي إليك؟؟
    7) لا. إذا كان جين وجيل، كما تقول، متزامنين، فلماذا يحدث هذا؟
    8) لا، من الأفضل أن أسألك إذا كنت تقبل ما تدعيه نظرية فسيولوجيا الخلية والكيمياء الحيوية.
    9) انظر الإجابة رقم 6. يمكنك أن تنظر إلى الأمر بهذه الطريقة، لكنها لا تقدمه إلى أي مكان، بل تؤدي فقط إلى تراكم الصعوبات. مثل القول إنه عندما تكون على متن طائرة من كاليفورنيا إلى إسرائيل، فإنك تستريح وتطير إسرائيل إليك.
    10) لا. لا تعتمد نسبة ساعة جاك على جيل جين أو جدة بوبسيك. ويعتمد ذلك على النسبة بين سرعة ساعة السيزيوم التي تحملها والإشعاع الذي يبرد الجدار الخارجي الذي تقاسه ساعة درجة الحرارة بواسطة شابيرا.
    11) في الواقع، أن جاك
    12) لا
    13) نعم إنك مخطئ ولم تستمع لما أكتبه لك منذ شهر. لن يؤثر النظام المرجعي، وبغض النظر عما يفعله، على العلاقة بين ساعات السيزيوم المحلية في نظام الدفع والساعات الحرارية التي تقيس الوقت وفقًا لإشعاع الخلفية في الكون.

  109. هذه ليست طريقة سقراط. يسأل سقراط سؤالاً واحدًا في كل مرة، وينتظر بفارغ الصبر الحصول على إجابة. ولم ينقذه ذلك من العقوبة في النهاية، لكنه تمكن من العيش بهذه الطريقة حتى بلغ السبعين من عمره

  110. نعم. دعونا نجرب طريقة سقراط مرة أخرى.

    1. هل تقبل الحقيقة الواردة في التجربة الأصلية بأن الاجتماع السابع يوضح أن زمن جيل أبطأ من زمن جاك؟ نعم لا

    2. هل تقبل أن هذا بسبب تحركات جيل بالنسبة لنظام جاك؟

    3. هل تقبل أن ما نعنيه بـ "نظام جاك" هو ببساطة جاك وساعته + الساعة الأخرى التي مرت بها جيل في طريقها إلى منزل جاك، وأن هاتين الساعتين جاك + أخرى بسرعة 0 بالنسبة لكل منهما أخرى ومتزامنة بينهما؟

    4. هل تقبل أننا كنا قادرين على إظهار أنه من خلال منطقك، فإن تسجيل الفيديو لساعات جاك فقط سيُظهر نسبة ثابتة تبلغ 1:1 وكبش جيل 1,000,000,000:1؟

    5. هل تقبل أنه إذا كانت هناك ساعة أخرى بعد جيل، اسمها جين، وهذه الساعة على نفس مسافة ساعات نظام جاك، ومتزامنة مع ساعة جيل، فإننا قد حصلنا على ما يسمى "النظام المرجعي لجيل" ؟

    6. هل تقبل أنه في الإطار المرجعي لجيل، فإن جاك هو الذي يتحرك بنفس السرعة التي يتحرك بها جيل بالنسبة لجاك فقط في الاتجاه المعاكس؟

    7. هل تقبل أنه، وفقًا للنسبية، عندما يتجاوز جاك جان، فإن وقته سيظهر أن الوقت أبطأ مما هو عليه عندما يتجاوز جيل؟

    8. هل تقبل ما تدعيه النسبية؟

    9. هل تقبل أن ما لدينا الآن هو نفس الصورة فقط في الاتجاه المعاكس، والآن جاك هو الذي يتحرك بالنسبة لنظام جيل؟

    10. هل تقبل أنه بنفس المنطق كما كان من قبل يجب أن تكون نسبة ساعة جاك الآن 1,000,000,000:1؟

    11. هل تقبل أن هذا هو نفس جاك الذي تبين حتى الآن أن نسبة 1:1 لديه؟

    12. هل ترى التناقض؟

    13. هل لديك تفسير؟

  111. إسرائيل،

    1) هيا! تبا، بعد كل شيء، قطعت جيل مسافة 600 كيلومتر في منحنى ينتهي على بعد كيلومتر واحد من نقطة البداية. مستمر تماما.

    2) "من الواضح تمامًا أن النظام الذي تتحرك به جيل هو نظام الراحة الخاص بجاك. ومن هنا أيضًا نسبة الدوران الدقيقة." حقًا؟؟ تعود مرة أخرى إلى عدم التمييز بين النظام المرجعي والعنصر المؤثر. وماذا لو كان جاك غير موجود؟ ألن تظهر ساعات جيل نسبة 1:600؟

    3) نفس الشيء. إذا كنت لا تفهم الاستقلال بين جاك وجيل، فليس لدي طريقة لشرح ذلك لك. خاصة:

    "ماذا سيحدث إذا ضرب جاك الغاز وانضم إلى جيل في الطيران بالمبنى؟" ==> فيما يتعلق بجيل وساعتها؟ لا شيء، لا تأثير
    "ماذا سيحدث إذا ضرب جاك الغاز وانضم إلى جيل في الطيران بالمبنى؟" ===> حتى أكثر لا شيء، أي تأثير
    "هل هذا هو ما سيجعل ساعة جيل الحرارية ترتفع؟" ==> لا سمح الله، لماذا تتباطأ ساعتها اعتمادًا على جاك؟ هل تعتقد أن هناك علاقة غامضة بين ساعة جاك وجيل؟

    "بعد كل شيء، الآن لا يوجد نظام آخر، فقط هو والظلام. ما الذي يسبب الفجوة بين الساعات الآن؟" ==> الفرق بين الساعات لم يكن سببه جاك. الفجوة بين الساعتين سببها أن ساعة السيزيوم الموجودة في سفينة الفضاء سوف تسرع مسارها لأن سفينة الفضاء تطير بسرعة بالنسبة لجزيئات التبريد في الخارج، وبالتالي فإن ساعة درجة الحرارة سوف تظهر مرور ساعة في الخارج، بينما ستظهر ساعة السيزيوم مليار سنة لأنها كانت في سفينة الفضاء حيث تم تسريع الوقت بالسرعة.

    "يشير إلى نظام افتراضي ما، والذي لم يعد موجودًا" ===> ؟؟؟

    في الختام: لا أعرف إذا كنت لا تمزح معي. إذا كان الأمر كذلك، فكن بصحة جيدة وتمتع بروح الدعابة الرائعة ودعنا نقول وداعًا هنا كأصدقاء قبل فوات الأوان 🙁
    إذا لم يكن الأمر كذلك، فأنا لا أفهم حقًا كيف لا ترى أنه لا يوجد أي اتصال بين جاك وجيل وأي نتيجة ستكون في ساعاتهم لا تعتمد على النظام الآخر.

  112. مقدمة دلالية: في الرياضيات هناك براهين. في الفيزياء هناك تأكيدات ودحضات، ولكن ليس هناك أدلة.
    الحل الذي وجدته يعتمد على النموذج الذي تحدثت عنه كثيراً وأثار غضب العظماء والحكماء الذين ماتوا بتلويح يدي - وهذا صحيح. لقد قدمت شرحًا سطحيًا (*) لم يولّد أي تعليقات. ولا فائدة من تقديم شرح متعمق، لأن ذلك يحتاج إلى فهم ودراسة متعمقة للنموذج من الأساسيات. نموذجي هو نظرية أخرى من النظريات التي تم وسيتم التضحية بها على مذبح الظواهر الفيزيائية، وكما يجسد في داخله حلا محتملا للمشكلة التي أثرتها، فليس من المستحيل أن تكون هناك حلول إضافية - بل أجمل - ستجده في نماذج أخرى.
    (* يعرض نموذجي حركة الفوتون و/أو الإلكترون كعملية ذات اتجاهين، ووفقًا لها فإن ما يعتبر مبكرًا له أيضًا جانب متأخر. وهذا ما يفسر الظاهرة المكتشفة في تجربة يانغ. دوران الإلكترون كما يلي: وكذلك يتم تحديد استقطاب الفوتون أيضاً في هذه العملية ذات الاتجاهين، وهذا ما يفسر نتائج تجربة أسبكت وما شابه)

  113. اليوبيل.
    أنت تدعي أنه قد يكون هناك احتمال آخر. على ما أذكر، لقد وعدتم: "إسرائيل! من فضلكم، لا تتعجلوا في القول: "لا مفر من الاعتراف بالحقيقة: الكون جزيرة محلية". أعتقد أنني توصلت إلى حل محلي للغز."

    أعتقد أن الوقت قد حان لرمي العظام.

    ر.ح.
    الإجابة على سؤالك: تحركت جيل مسافة 300 كم شمالًا + 300 كم جنوبًا.
    هذه ليست دالة متصلة (يمكنك الدوران في المنتصف)، لذا فإن التكامل يكون في أجزاء، وهو ما لن يعطي 0. صيغة فريدمان هي بالفعل دالة متصلة.

    سأسهل الأمر عليك: من الواضح تمامًا أن النظام الذي تتحرك به جيل هو نظام الراحة الخاص بجاك. وبالتالي نسبة الدوران الدقيقة.

    سؤال: إذا تم إجراء التجربة 3-3 بأكملها في حقل الاختبار الدائم الخاص بنا في تيزنبي، حيث لا توجد أنظمة إضافية، فقط الظلام فوق الهاوية، ماذا سيحدث إذا قام جاك بتشغيل الغاز وانضم إلى جيل في الرحلة في الهيكل؟ أم أنه سيختفي بكل بساطة؟ هل هذا هو ما سيؤدي إلى ارتفاع درجة حرارة ساعة جيل المؤقتة؟ بعد كل شيء، الآن لا يوجد نظام آخر، فقط هو والظلام. ما هو سبب الفجوة بين الساعات الآن؟ وهناك فجوة أخرى دقيقة جدًا، تشير إلى نظام افتراضي ما، والذي لم يعد موجودًا.

  114. إسرائيل،

    فيما يتعلق بسؤالك "لماذا تظهر جيل نسبة 1 مليار إلى 1، بينما نسبة جاك 1:XNUMX. فقط هذا.
    نسبة إلى ما يتحرك؟ ليست أشياء عامة، بل فقط النظام المحدد الذي تتحرك جيل فيما يتعلق به."

    يتم تحديد دورات الساعة من خلال اضمحلال السيزيوم. كان لدى جيل مليار انفصال وجاك مليون. الفرق هو أن جيل قاد السيارة. ونفس العدد من التفككات سيحدث في كل منهما حتى لو لم يكن الآخر موجودا.

    ما هو مماثل ل؟ سيقود جاك سيارة بطيئة مسافة كيلومتر واحد في نفس الوقت الذي ستقود فيه جيل مسافة 1 كيلومتر ذهابًا وإيابًا. كلاهما يملكان:
    1) يتم ربط عداد المسافات بعجلات السيارة
    2) نظام تحديد المواقع العالمي (GPS) الذي يقيس موقعهم بالنسبة لنقطة البداية.

    وهم الآن في نفس المكان يقارنون نسبة القياس بين عداد المسافات ونظام تحديد المواقع العالمي (GPS). نسبة جاك هي 1:1 ونسبة جيل هي 600:1.
    الآن دعنا نعود إلى سؤالك مع تعديل بسيط: "لماذا تظهر جيل نسبة 600 إلى 1، بينما نسبة جاك 1:1. فقط هذا. نسبة إلى ما يتحرك؟ ليست أشياء عامة، بل فقط النظام المحدد الذي تتحرك جيل فيما يتعلق به."

    الآن أجب على نفسك.

  115. كما ذكرنا، فإن إتقانك للمنطق جيد. الافتراض الخاطئ موجود في القسم 4: "هناك خياران". إذا قلت "هناك خياران فقط"، فأنت الجاني الرئيسي. ومع ذلك، فأنت تستشهد فقط بالآخرين، وبالتالي وجدت يحق لك بسبب الشك. كلا الاحتمالين هما نتيجة تفكير وليس دليل موجود في الميدان. ولا نستبعد وجود خيارات أخرى. نحن ندرك المصادفات، وبالتالي فإن حقيقة أن التجربة تؤكد الاستنتاج "بأغلبية ساحقة" قد تقنع هيئة المحلفين في محكمة متوسطة ولكنها لا تقول شيئًا ونصف عن حقيقة الاستنتاج.

  116. يوفال

    دعونا نذهب من خلال الحجة على مراحل. معرفة ما إذا كان بإمكانك الإشارة إلى الخطوة التي قد يكون هناك خطأ فيها:

    1. يترك إلكترونين مصدرًا مشتركًا في اتجاهين متعاكسين.

    2. وفقا لقانون حفظ الدوران، يجب أن يكون دورانهما معاكسين.

    3. يتم الدوران على طول 3 محاور. الدوران معاكس حول المحاور الثلاثة.

    4. هناك خياران:

    و. يتم تحديد الدوران عند الانفصال (المتغيرات الخفية، أينشتاين).

    ب. يكون السبين في حالة تراكب، أعلى + أسفل، ويتم تحديد قياس أحد الإلكترونات أخيرًا. (الكم، عدم اليقين، هايزنبرغ، بور)

    ثالث. إذا كان الوضع مثل B، فعند تحديد وضع الدوران، يختار شقيقه على الفور الدوران المعاكس. غير محلية.

    5. كيف سنختار بين أ و ب؟

    6. نظرية متباينة بيل: دليل رياضي على صحة الوضع ب.

    7. تجارب الجوانب وغيرها الكثير (على الفوتونات، ولكن المبدأ هو نفسه). تأكيد ما هو متوقع من جملة بيل بأغلبية ساحقة.

    8. سؤال: ما الذي يمكن أن يكون مختلفًا؟ ما هي الصفقة مع المسار؟ بعد كل شيء، تم إثبات النتيجة رياضيًا مسبقًا، ولم تؤكد التجارب إلا ما كان معروفًا بحدوثه. هل يمكن لأي شيء في التجربة أن يتعارض مع نظرية متباينة بيل ونتيجتها الطبيعية؟

  117. إسرائيل،
    لست مسؤولاً عما فهمته أو لم تفهمه، ولكن سأحاول مرة أخرى. يغادر الفوتون نقطة واحدة، ويصل الفوتون إلى نقطة أخرى (وفقًا لنموذجي، فهو ليس نفس الفوتون، لكننا سنترك نموذجي). تسجل التجربة بشكل تجريبي نقطة البداية ونقطة الوجهة وربما أيضًا عددًا محدودًا من النقاط على طول الطريق، ولكن ليس المسار بأكمله. ليس لدينا طريقة لمعرفة ما حدث بالضبط في الطريق. كل ما على الله أن يفعله هو استخدام التفكير التكميلي (على سبيل المثال، نظرية القيمة المتوسطة) وبالتالي استخلاص استنتاجات ليست صحيحة بالضرورة.
    أدخل المنطق، على سبيل المثال طريقة العمل: إذا كان A ثم B؛ و؛ وبالتالي ب. ولكن إذا لم تكن A صحيحة، فإن B كاذبة.

  118. يوفال، لست متأكدًا من أنني أفهم.

    برهان بيل رياضي. تجارب الجانب التجريبي. ماذا يمكن أن يكون الخطأ هنا؟ المنطق؟ ما هو الاستنتاج الآخر الذي يمكن أن يكون ممكنا؟ هل يمكنك أن تريني التفاصيل؟ إعطاء أي اتجاه، أي مثال؟

    ولم أفهم العلاقة بين تحركات الفوتون أيضًا.

  119. ر.ح. هذا ليس ما سألته.
    فقط:
    لماذا تظهر جيل نسبة 1 مليار إلى 1، بينما نسبة جاك 1:XNUMX. فقط هذا.
    نسبة إلى ما يتحرك؟ ليست أشياء عامة، فقط النظام المحدد الذي تتحرك جيل بالنسبة إليه.

  120. RH، شكرا على التوضيح.
    بالمناسبة، وجدت Flatlandia بعض ملفات PDF الرائعة، بما في ذلك نسخة ممسوحة ضوئيًا من الطبعة الأولى. يسعدني أن أنقلها إلى أي شخص يحتاج إليها. كما تم مؤخرًا إصدار نسخة فيلم روائي طويل.

  121. إسرائيل،

    لا أفهم اعتراضك على انتقال جيل وعدم قيام جاك بذلك. صحيح أنه يمكن للمرء أن يتفلسف أنه عندما تدخل جيل سفينتها الفضائية وتقود المحركات الذرية وتمضي في طريقها وتصل بعد فترة قصيرة إلى أندروميدا، في الواقع بقيت في مكانها وابتعد عنها جاك مع الأرض كلها وعن أندروميدا. فجأة جاء إليها. في الواقع، من الممكن أيضًا رؤية الأمر بهذه الطريقة... كما يعتقد ماخ.

    كيف يقوم ثلاثة فيزيائيين نسبيين بتركيب مصباح؟ واحد يحمل المصباح واثنان يدوران في الغرفة (أو العالم أو الكون، حسب اختيارك).

    الحجة الأخرى التي تجاهلتها هي أن النظام المرجعي هو مجرد نظام مرجعي وليس سببيًا. لقد زعمت أن شيئًا يتحرك بالنسبة لك لا يؤثر عليك حقًا. حقيقة أن هناك الآن سفينة فضائية في مكان ما في الفضاء تتحرك بالقرب من سرعة الضوء بالنسبة لنا لا تؤثر على اختياري للمعكرونة أو الأرز كطبق جانبي للدجاج.

  122. ر.ح.

    1. لنفترض أن الساعات المؤقتة وساعات المسافة المجرية وغرينتش العالمية جميعها تعطي نفس الوقت لكل نظام.

    فهل هذا يتعارض مع إطالة الأزمنة في النظام غير المتسارع - أعتقد ذلك. وأعتقد أيضًا أنني أثبتت ذلك بمثال جاك وجيل. إذا كنت تستطيع أن تثبت خطأي - سوط. إذا أضفت المزيد من أنظمة القياس، فقط لإظهار أن الساعات الحرارية ليست الوحيدة، فإنك لا تتعارض مع حجتي - بل تقويها.

    وليس لدي أي مسلمة. جميع التفسيرات والأدلة.

    2. كيف لم أقنعك إذا أوضحت لك أن جاك وجيل يجب أن يظهرا نفس الوقت على ساعاتهما إذا قبلا المسلمة أ؟ ربما لم تكن مقتنعًا، لكن حجتك بأن جيل تتحرك "حقًا" وجاك ليس كذلك، غير مقبولة بالنسبة لي. إذا كان هذا هو الشيء الوحيد الذي لديك، فسوف أحافظ على إيماني. إذا كان هناك شيء جديد - سوط.

    3. لا أقوم بتحليل كل التفاصيل الدقيقة للتناقضات بين جميع النظريات الموجودة. أنا أتحدث عن موضوع واحد صغير: طول الزمن وسرعة الضوء. إذا كان لديك شيء جديد - اجلده. ولكن هناك شيء واحد يجب أن أتأكد منه: في مثال CJ: هل التحفظ الوحيد لديك هو مشكلة أن جيل يتحرك بينما لا يتحرك جاك، أم أن هناك شيئًا آخر؟

    4. من القانوني العد. لكن في فيغاس، يعتبر الكازينو ناديًا خاصًا ولا يتعين عليه قبولك إذا لم يكن مهتمًا. في أتلانتيك، لا يُسمح لهم بمنعك من اللعب، ونتيجة لذلك تم تغيير قواعد اللعبة بحيث أصبح من المستحيل الربح حتى لو قمت بالعد. هذا هو أحد الأسباب التي تجعل الجميع يذهبون إلى فيغاس وليس المحيط الأطلسي. خلاصة القول، أنهم يؤذون أنفسهم.

  123. إسرائيل،
    وفيما يتعلق بالاستنتاجات التي يستخلصها الناس من نتائج التجارب، إذا استخدمت كلامك: "التجربة مسؤولة فقط عما تظهره". ادعاءات مثل "سرعة الضوء ثابتة في أي إطار مرجعي" أو "معلومات الدوران تنتقل بسرعة أكبر من الضوء" تعكس استنتاجات ولكن لم يتم إثباتها بشكل مباشر. عندما تقول إن EPR كان مخطئًا، فإنك تحكم على هذا النوع من الاستنتاجات. منطقك أرسطي تمامًا، لكن الخدعة المبنية على حقائق غير صحيحة تؤدي حتمًا إلى استنتاجات خاطئة. كما ذكرنا سابقًا، فإن الاستنتاج من تجربة بيل هو مجرد استنتاج محتمل. ليس لدينا (ولا توجد طريقة للحصول على) سجل مستمر لحركات الفوتونات على طول الطريق. وهذا هو العيب الذي أجده في حجتك. RH أكثر وضوحًا ومعرفة مني، وربما سيجد خطأً أكثر جوهرية.
    وبالمناسبة، انظر ماذا وجدت في قسم الآثار:
    http://www.haaretz.co.il/misc/1.815624

  124. اليوبيل,
    هذا ليس ما قصدته. ما أردته هو أن أُظهر لصديقنا (أنتم أصدقاء، أليس كذلك؟ هل تحبون الحالة؟) أن ساعات درجات الحرارة الخاصة به ليست شيئًا خاصًا ولكنها مجرد نظام مرجعي عادي آخر يتصرف وفقًا للنظرية النسبية المجيدة.

    إسرائيل،
    1) لقد تهربت من أناقة القنفذ. عظيم، لقد فكرت في الأمر أيضًا، فما هو الجواب؟ هل مازلت تعتقد أن الساعات الحرارية تختلف عن متوسط ​​غرينتش المجري؟ إذا كان الأمر كذلك فلماذا، وإذا لم يكن الأمر كذلك، ما هو معنى افتراضاتك المبدئية لمسلماتك؟

    2) "هل تجد خطأ في حجتي؟ سوط." أليس هذا ما كنت أفعله لعدة مئات من المشاركات؟ تتجاهل وتلوح بيديك أو تعطي إجابات هنا وهناك، لكنك مازلت لم تقنعني في المرحلة الأولى من بنيتك بوجود تناقض بين النسبية والانفجار.

    2.5) بالحديث عن الانفجار، فأنت الآن تدعي أنك لا تحب ذلك، هل لديك تفسير بديل لمسافة المجرات + درجة حرارة الخلفية؟ سوط!

    3)” وفقاً لهذا المنطق فإن النسبية خاطئة. تنتقل معلومات الدوران بشكل أسرع من الضوء. لقد كان أينشتاين مخطئًا بشأن EPR. هل نلغي العلاقة؟ هل ستمر الجزيئات عبر المسرع الآن؟ هل سنتوقف عن الخوف من القنبلة الإيرانية؟ ففي نهاية المطاف، ليس هناك شك في أن أينشتاين كان مخطئا!" ====> إذا كنت تعتقد أنك ستصفعني بـ "العلاقة خاطئة" وسأسقط من الكرسي، فأنت مخطئ. ففي نهاية المطاف، من الواضح والواضح أن ميكانيكا الكم لا تتوافق مع النسبية. ومن الواضح والواضح أن كلاهما صمد أمام كل اختبارات التفنيد التي أجريت حتى الآن ومع ذلك فإنهما يناقضان بعضهما البعض. تنهار صيغ النسبية عند مسافات صغيرة. الجاذبية في العلاقات هي انحناء المسافة وتحملها الجرافيتونات في العلاقات، EPR الخاص بك والمزيد. إذن ما الجديد هنا؟ نعلم جميعًا أن هناك مشكلة في النظريات. ولهذا السبب اجتمعنا هنا، ولهذا السبب يجلس كبار علماء الفيزياء ويحاولون تطوير نظريات الأوتار.

    4) الحجة الوحيدة التي لا أجدها صحيحة تتعلق بالأشباح. أوه، ولقد أقنعتني بشأن الكازينو أيضًا. وهل يجوز لهم أن يطردوا من جلس ويكسب؟ فهل هناك سبب قانوني لذلك؟ هل هناك قانون يمنع العد في القلب؟

  125. ر.ح.

    لقد تعافيت بالفعل، وأنا جاهز وجاهز للمعركة.

    1. فكرة جميلة، فكرت بها أيضًا. اقرأ البريد الإلكتروني التالي، ولاحظ التاريخ.

    بدءًا من: snuz2001@aol.com
    إلى: arot43@aol.com
    تم الإرسال: 12/24/2011 الساعة 2:30:44 مساءً بتوقيت المحيط الهادئ (أنا
    الموضوع: (بدون موضوع)

    مرحبًا جيل

    أود أن أشكركم على الوقت الذي أمضيتموه في مناقشة مسألة تمدد الزمن معي. بعد رحيلك، فكرت أن ما قد يوضح فكرتي عن الزمن المطلق هو المثال التالي: لنفترض أن لدينا جهاز إرسال قوي على الأرض يرسل، باستخدام الراديو، إلى الزمن الأرضي للكون في فترات زمنية قدرها 1 ثانية.

    لذا فإن النقطة 0 ستكون على سبيل المثال 31 ديسمبر 2011 الساعة 12 منتصف الليل بتوقيت جرينتش، وبعد ذلك سترسل كل ثانية إشارة قصيرة وقوية: 1، 2، 3، …..100^100.

    وفي كل نقطة في الكون القريب، يمكن للمستقبل الذي سيستقبل النبضة أن يحسب مسافة الأرض (في وقت الإرسال) من قوة الإشارة وإزاحة دوبلر، وبالتالي الوصول إلى ما سيسمى: "التوقيت المطلق العالمي". في مثالنا الخاص بالسفن المهاجمة التي تحاول مزامنة ساعاتها، فإن هذا التوقيت العالمي سيمنحها طريقة مفيدة لتنسيق الهجوم. ولاحظ أيضاً أنه لو كان هناك المزيد من مصادر الإشارات التي تم ترتيبها بنفس الطريقة الأصلية، وساعاتها متزامنة في البداية مع الأرض، وتتحرك بسرعة ثابتة في الفضاء، مهما بعدت وفي أي اتجاه، عندها ستتلقى كل سفينة في الكون منهم نفس "الوقت المطلق العالمي"، بغض النظر عن المصادر التي ستختار استخدامها.

    أ + ب. لا أعرف الجواب. أنا مشغول جدًا هنا بالمناقشات وتخطيط التجارب وجميع الأنشطة اليومية الأخرى، لدرجة أنني لم أتمكن من مناقشة الفكرة بالتفصيل. وقد عرضته في عدة منتديات فيزيائية ولم يجبني أحد.

    2. "أنت تدعي بالضرورة أن العلاقة غير صحيحة". وهذا استنتاج متسرع. مثل استنتاج يوفال، إذا قلت أن نظرية الانبعاث تشرح جيدًا نتائج تجربة MM، فهذا يعني أنها تشرح أيضًا حقيقة أن سرعة الضوء هي نفسها لكل جهاز قياس. وفي الحالتين ليس ما أقول، وأنا مسؤول فقط عما أقول، وليس عن الاستنتاجات التي أستخلصها من كلامي.

    قلت هنا 10 مرات على الأقل: لا أعرف. من الصعب علي أن أتقبل بشكل حدسي فكرة بداية الزمن (ما الذي جاء أولاً؟). أنا أحب الأناقة الرياضية للنسبية. هناك شيء غير صحيح. هذا كل شيء. ليس لدي أي ادعاءات لبناء مجموعة طلعت مصطفى.

    هل تجد خطأ في حجتي؟ سوط.

    3. "سيرن يدحض نظريتك. وتتحدث النظرية النسبية عن سرعة الضوء باعتبارها الحد الأعلى لكل شيء، سواء كان كتلة أو لا كتلة. وبهذا المنطق فإن النسبية خاطئة. تنتقل معلومات الدوران بشكل أسرع من الضوء. لقد كان أينشتاين مخطئًا بشأن EPR. هل نلغي العلاقة؟ هل ستمر الجزيئات عبر المسرع الآن؟ هل سنتوقف عن الخوف من القنبلة الإيرانية؟ لا شك أن أينشتاين كان مخطئا!

    هل تجد خطأ في حجتي؟ سوط

    4. على مدى جيل كامل، كانت صحراء سيناء مسرحًا للمناوشات مع مصر كل بضع سنوات. وكان هاريك الوحيد الذي كان هناك هو شارون، الذي كان يظهر مرتديا ضمادة وهو في طريقه إلى الجانب الآخر من القناة.

    5. حتى ما يقرب من 50 عامًا مضت، كانت لعبة البلاك جاك لعبة هامشية إلى حد ما. بعد أن كتب ثورب كتاب "تغلب على الموزع"، ارتفعت شعبية اللعبة، وحاول الجميع التغلب على المنزل، وبالفعل نجح الكثيرون. قام الكازينو على الفور بتغيير القواعد، بما في ذلك إضافة أوراق اللعب، لذلك توقف الناس عن القدوم.

    اليوم تم تحقيق الوضع الراهن: يمكن التغلب على المنزل، وهذه المعرفة تجذب الحشود وتجعل من لعبة البلاك جاك اللعبة الأكثر شعبية في الكازينو. ومع ذلك، فإن القليل منهم فقط هم من يتمكنون حقًا من تحمل العبء، وخاصة بالسرعة المطلوبة لحساب طاولة كاملة بها 6 لاعبين في لمحة. بالنسبة لهم، أنشأت الكازينوهات نظام مراقبة متطور يتضمن نظام التعرف على الوجه، وأجهزة كمبيوتر قادرة على معرفة بناءً على طريقة لعب الرهان إذا كان اللاعب كاتبًا أو مجرد مقامر، وحتى وكالة خاصة تسمى غريفين. ، وتتمثل مهمتها في التعرف على الكتاب، الذين يستخدم الكثير منهم التنكر.

    إنها تجربة غير عادية عندما تكون صغيرًا، خاصة إذا كنت تحب السفر. يمكنك السفر في جميع أنحاء العالم، والإقامة في أفضل الفنادق، وتناول الطعام في أغلى المطاعم، ورؤية جميع الحفلات الموسيقية المرغوبة، والاستماع إلى جميع المطربين، كل ذلك مجانًا - والمغادرة براتب بحجم جراح التجميل.

    خلاصة القول، وأنا أعني ذلك - إنه فخ. كان كين هيوستن أحد نجوم النظام المالي في سان فرانسيسكو. متزوج، أطفال، مهنة. لقد ترك كل شيء من أجل أسلوب حياة مدمن على الكحول، وكرس موهبته الرياضية الفريدة لحساب احتمالات البطاقة، وانتهى به الأمر وحيدًا في شقة في باريس محاطًا بالزجاجات.

    7. الشبح كما يسمى هو كائن لا حياة له وهدفه كله مضايقة المعلقين بدون تعليق موضوعي واحد (التصحيح نعم تعليق موضوعي واحد).

    هل تجد خطأ في حجتي؟ سوط.

    الأشباح - لماذا لا تشتكي من ابن عمك الشاب والناجح وتترك كبار السن يتذكرون؟

    ما الجديد تحت الصفر المطلق؟

  126. آر إتش،
    بافتراض أن درجة حرارة إشعاع الخلفية الكونية هي نفسها في كل مكان في الكون، فلدينا ساعة زمنية عالمية مطلقة ("جرينتش جالاكتيك"، وفقًا لتعريفك). لم أكن متأكدا مما تقصده.

  127. را بام,
    مرة أخرى لتناول الحبوب الوردية بدلا من تلك الأرجوانية؟ إنه ليس سيئًا، نم قليلًا، العب بالخارج في صندوق الرمل وسوف يمر الأمر.

    فقط كن حذرا حتى لا تنخفض إلى ما دون الصفر المطلق.

  128. شافيرة

    هذا كل شيء؟
    ابن عمي الذي هو أصغر مني، هو أكثر نجاحا منك (ولن أذكر الآخرين).

    وبالمناسبة، هذا ليس تمرينك يا إسرائيل.
    إنه عمرك.
    لوطي ستكون رائحتها مثل رائحة كبار السن هنا... ماذا تسميها؟ الشيخوخة؟

  129. إسرائيل،
    أنا سعيد بعودتك وأرى أنك عدت مليئًا بالطاقة والحيوية.

    1) يسأل سقراط: سوف نبني منشأة سنسميها مجرة ​​غرينتش والتي ستحتوي على ساعة سيزيوم في حالة سكون وتنقل الوقت في جميع الاتجاهات. هل تعتقد أن سفن الفضاء الخاصة بك التي تخطط لهجوم ستكون قادرة على تنسيق الهجوم إذا عرفت موقعها الدقيق وموقع غرينتش؟

    و. نعم ==> إذًا ما هو الفرق الأساسي عن ساعة درجة الحرارة لديك؟

    ب. لا ==> لماذا؟ ففي نهاية المطاف، كل ما تحتاجه كل مركبة فضائية هو أن تستقبل المرسل، وتحلل المسافة التي تفصلها عن غرينتش، وتضرب هذه المسافة في سرعة الضوء، وتضاف إلى نتيجة المرسل وهنا تعرف توقيت غرينتش بالضبط. بالمناسبة، اعتبارا من اليوم هناك العديد من هذه المرافق التي تعمل بنجاح كبير. يطلق عليهم الأقمار الصناعية GPS.
    وهذا ينطبق أيضًا على كلا الأنابيب الخاصة بك.

    2) أنت تزعم أن هناك تناقضاً بين الانفجار والنسبية. نظرًا لأنني لا أتذكر أنك قدمت أي ادعاءات ضد أدلة الانفجار (وصحح لي إذا كنت مخطئًا، بعد كل شيء أكثر من 1000 مشاركة) فإنك تدعي بالضرورة أن النسبية غير صحيحة.

    3) سيرين يدحض نظريتك. تتحدث النظرية النسبية عن سرعة الضوء باعتبارها الحد الأعلى لأي شيء، سواء كان له كتلة أو لا كتلة. التاكيونات هي جسيمات خيالية لم يتم توقعها بعد، ولا يمكنها أن تقل عن سرعة الضوء. تأتي وتقول: "نعم، فيما يتعلق بالمقال، هذا صحيح. موقع الويب النشط الخاص بي ليس فعالاً والنسبية تترسخ، ولا أهتم به على الإطلاق." ومع ذلك، فيما يتعلق بالفوتونات، فهذه قصة مختلفة." أليس هذا التلويح باليد وإدخال التعقيدات في النظام؟

    4) بالنسبة لي، سيناء مكان سحري للوضوح، مهرجان نويفا، العراة، المواد، صحراء مذهلة مذهلة. تيزانبي هي الزرقاء في موقع ليد جبل باروخ ليلاً بين المتتبعين الأحمر والأخضر والتي توقفت فقط عندما ظهر شوان إريك على قناة الشرق الأوسط.

    5) شكرا على الدرس. لكنها سألت. إذا كان الأمر بهذه البساطة ويمكن الوصول إليه، فلماذا لم يتم إخراج لعبة البلاك جاك من الكازينوهات؟ لماذا لا يستخدم الجميع هذه الأساليب؟

  130. هذا كل شيء، تلخيص انفلونزا النجوم.

    (بالمناسبة، هل تعرف أصل كلمة إنفلونزا؟).

    ر.ح. محبوب.

    1.

    هل سبق لك أن فعلت البراهين في الهندسة؟

    لا يمكنك الوصول إلى مرحلة متقدمة قبل إثبات المرحلة السابقة. مهما كان الأمر، في الجزء السفلي من السلسلة الغذائية، سوف تصادف البديهيات، تلك الافتراضات البديهية التي لا يمكن إثباتها، ولكن التي بنيت عليها كل جملة.

    ولن أستطيع أن أثبت لك صحة نظرية فيثاغورس إذا لم تقبل البديهية التي تقول إنه بين كل نقطتين في المستوى يمر خط واحد فقط، وهي أيضًا أقصر مسافة بينهما.

    إن ما يعادل البديهية في الفيزياء هو المسلمة.

    والمسلمة الأولى للنسبية (وليس النسبية فقط) هي تكافؤ جميع أنظمة القصور الذاتي. أي نظام لم يتم تسريعه يكون في الأساس في حالة سكون، ولا يمكن تمييزه عن نظام آخر لم يتم تسريعه.

    لقد تم تنظيم مناقشتنا حول إطالة الوقت كدليل هندسي، حيث كانت كل حجة جديدة مبنية على اتفاق كل منا على الحجة السابقة. في نهاية كل خطوة أود أن أطرح سؤالاً وأسألك إذا كنت توافق أم لا.

    لقد توصلنا إلى اتفاق على أن الساعتين المؤقتتين الموجودتين في نفس النقطة ستظهران دائمًا نفس الوقت، بغض النظر عن سرعتهما النسبية.

    في مثال سفن الفضاء الخاصة بجاك وجيل، توصلنا إلى اتفاق على أنه في نظام جاك، سيُظهر مقطع فيديو مشترك للساعة cc والساعات المؤقتة نسبة دوران تبلغ 1:1، أو نفس الوقت في كليهما دائمًا.

    ما زلنا منقسمين حول سفينة الفضاء جيل. أنت تزعم أن النسبة ستكون 1,000,000,000:1، لأن هذا النظام في حركة "حقيقية"، بينما أنا أدعي أنه لا يوجد شيء اسمه حركة حقيقية، انظر المسلمة أ. جيل في حركة غير متسارعة، وبالتالي أيضًا في نظام جيل ستكون النسبة 1:1، وستظهر كلتا الساعتين دائمًا نفس الوقت.

    ومن الواضح أنه من المستحيل مواصلة البرهان إذا لم نتفق على مثل هذه النقطة الأساسية. على الرغم من ذلك، أوضحت أنه إذا قبلت افتراضاتي السابقة، فلن يكون هناك إطالة للوقت لأنه إذا كان A=B وC=D وA=C، فإن A=B=C=D وبالتالي عندما يلتقي جاك وجيل سيرون أن فقرهم في نفس الوقت

    ولذا فقد أوضحت لك بمثال الأنابيب المتحركة، أنه بما أن جميع الساعات الموجودة في الأنبوبين تظهر دائمًا نفس الوقت، وبالتالي لا يوجد تمدد زمني، فلا مفر من الاستنتاج بأن نفس الفوتون يتحرك عند مستوى أعلى السرعة بالنسبة للأنبوب الذي يتقدم في اتجاه حركته.

    خاص.

    إلا إذا، كما ذكرنا سابقًا، فإنك لا تقبل المسلمة أ، أصل النزاع بيننا. ولكن إذا لم تحصل عليه، فكيف يمكنك الحصول على أي شيء في العلاقة على الإطلاق؟ بعد كل شيء، تم بناؤه عليه!

    أرني عيبًا آخر في الحجة إلى جانب هذا، ويمكننا أن نتجادل. بخلاف ذلك، يجب أن أستنتج أن السبب الوحيد الذي يجعلك متمسكًا بإطالة الوقت، هو ادعاءك المستمر بوجود نظام يتحرك "حقًا" (جيل على سبيل المثال). وفي هذه الحالة، فإنني أستمر في الاعتقاد بقلب هادئ أن دليلي لا تشوبه شائبة، حتى يثبت العكس.

    2. سيرين لا تدحض لي أي شيء من الناحية النظرية. لقد ذكرت عدة مرات أن هذا موضوع مختلف. اذهب إلى الموضوع. أتفق معك أن هناك تمديدا للمرات في الأنظمة المتسارعة. كل الأمثلة التي قدمتها لتتناقض مع ادعاءاتي، تجربة الطائرة، ساران، هي تجارب مع الأنظمة المتسارعة. أرني النقطة التي تظاهرت فيها بالتعامل مع الكتل، باستثناء نموذج الجاذبية والقصور الذاتي الذي تم ذكره طوال الوقت كعرض ثانوي فقط، حيث يتم حل مشكلة الاحتكاك في ليساج باستخدام النموذج الذي شرحته. منذ عام وأنا أتحدث عن مشكلة تتعلق بموضوع واحد: تمديد الوقت. ليس لدي مشكلة مع الباقي. وإذا فهمت خطأ فأنا أعتذر وأصحح. فقط تمديد للوقت. دعونا ننتهي من هذا - سنرى ما إذا كانت هناك قوة للباقي.

    وإلى جانب ذلك، قال ر.إ.ه. عزيزي، أنت تكرر وترتكب نفس الخطأ مرارًا وتكرارًا: الادعاء بأنني أدعي أن العلاقة خاطئة. هذا على الرغم من أنني أكرر وأؤكد أنه إذا لم يكن هناك تبريد مستمر للكون وفقا لصيغة فريدمان، كما تدعي نظرية الانفجار الكبير، فإن النسبية مطلوبة تماما. تكرر باستمرار موضوع "النظام الخارجي" الذي يمكن استخدامه لقياس الوقت، مثل ابتعاد المجرات. جاهز للتدفق معك: إذا كان بإمكانك بالفعل الاتفاق على وقت مطلق عن طريق المسافات المجرية، فلا يزال بإمكاني أن أثبت لك أنه لا يوجد تمدد زمني باستخدام الساعة المجرية، ولكن بشرط واحد: يجب عليك اعتماد المسلمة أ إلى قلب. كيف أفعل؟

    لذا تذكر - أنا لا أدعي أن العلاقة غير صحيحة. لكنها لا تتناسب مع نظرية الانفجار الكبير. لماذا لا تدعي أنني أخدع مفتزادول؟

    3. أرني النقطة التي زعمت فيها أن شرحي لموقع نشط يفسر عدم المحلية. كان رأيي دائمًا هو أنه يسمح بالأجهزة غير المحلية. ( انه ليس؟).

    أينشتاين هو الذي ادعى أن اللامكانية غير ممكنة بسبب النسبية (ألم يدعي؟ ألم يكن مخطئا؟).

    ملخص: إذا وجدت عيبًا في حجتي بخلاف مسلمتي في المسلمة أ - فليس لدي مشكلة في الاعتراف بالخطأ. بخلاف ذلك، سأستمر في التمسك بادعائي: هناك تناقض بين إطالة الوقت في العلاقات ونظرية الانفجار الكبير.

    4. من الذي تعرض للإهانة؟ هل سمعت صوت صراخ؟ الاسم قليل، حسنًا، كيف نقوله، مثير للاهتمام؟

    5. هذه هي مشكلتكم أيها الشباب. هذه الرائحة القوية للحوم الطازجة من الباكوم. أنك تقول تيزنبي وجبالاخرا، ولا تعلم أن هذه أسماء حقيقية للأماكن التي يخدم فيها البازمانيك مثلي (ويوفال ومئير كما أفترض) خدمتهم.

    6. دعونا نأمل ألا يغضب والدي كثيرًا. إذا كان الأمر كذلك، تحمل المسؤولية عن نفسك.

    أؤكد دائمًا أن هناك شيئين يبدوان معقدين، وهما في الواقع بسيطان للغاية. الأول هو النسبية الخاصة. والثاني هو عد البطاقات في لعبة البلاك جاك.

    في طريقة العد الكلاسيكية، طريقة HI-LOW، وهي نفس الطريقة التي تستخدمها جميع مجموعات MIT (التي لا تزال موجودة حتى اليوم)، يتم استخدام شيئين:

    1. الاستراتيجية الأساسية.
    2. عد وتلخيص البطاقات العالية والمنخفضة.

    الإستراتيجية الأساسية هي عبارة عن خط صارم من تعليمات العمل لأي موقف معين. على سبيل المثال: لديك 16 والتاجر لديه 10 - يجب أن تأخذ البطاقة. اذهب 5 أو 6 - يجب أن تقف. لديك 11 - يجب أن تضاعف.

    يمكنك التنزيل عبر الإنترنت أو الشراء بالدولار من الكازينو نفسه.

    2. البطاقات من 2 إلى 6 تحصل على قيمة +1.

    البطاقات رقم 7-9 لا قيمة لها.

    البطاقات رقم 10 (بما في ذلك جميع الصور) والآص تحصل على قيمة -1.

    مسار اللعبة في حزمة واحدة عندما تلعب بمفردك مو الموزع:

    1. ضع الحد الأدنى لمبلغ الرهان.
    2. العب بشكل مباشر وفقًا لقواعد الإستراتيجية الأساسية.
    3. قم بتلخيص جميع قيم البطاقة حسب المفتاح الذي قدمته. في نهاية الجولة، ستتبقى لديك ضريبة مضادة: سلبية، 0 أو إيجابية. إذا كانت قيمة رقم العداد أقل من 2، اترك مبلغ الرهان كما كان. مع هو 2، ثلاثة أضعاف ذلك. 3، خمس مرات. 4 أو أكثر، بقدر ما تستطيع، قبل أن يأتي الأمن ويرافقك إلى الخارج.

    إذا كان هناك طردان - قم بتقسيم الضريبة المحتسبة على ضريبة الحزمة المقدرة التي بقيت في "الحذاء" (الصندوق الذي توجد فيه الطرود. بهذه الطريقة سيتبقى لك ما يسمى بالعدد الحقيقي. مع 6 حزم - قسّم بمقدار 6 - بمقدار 600 وهكذا (لا بأس، الحد الأقصى هو 600 عبوات).

    بعد يومين من التدريب سترى أن الأمر أصبح تلقائياً تماماً، مثل القيادة أو تعلم لغة أجنبية تكون صعبة في البداية ثم لا تفكر فيها.

    المشكلة الأساسية ليست في العد، بل في كيفية البقاء باردًا كالثلج عندما تكون المبالغ مرتفعة، وكيفية منع الكازينو من منعك من اللعب (بلغة العلم - احجب ردودك).

    هذا كل شيء، وآمل الآن أن والدي لن يمنع تعليقاتي. لقد تم منعي من دخول الكازينوهات لسنوات، لحسن الحظ.

    خلاصة القول - مضيعة للوقت. بواسطة كين أوستن، الذي أضاع حياته ونقاط معدل ذكائه البالغة 170 نقطة هناك، ودمر نفسه وعائلته.

  131. اليوبيل,

    لم أدعي أنه من المستحيل حساب الوقت على أساس انخفاض درجة الحرارة، بل على العكس من ذلك، كان ادعائي أنه لا يوجد شيء مميز في ذلك وفي الواقع ستكون سفن الفضاء الإسرائيلية قادرة على جدولة موعد على الرغم من أن كل منها سافر في موعد محدد. سرعة متغيرة قريبة من سرعة الضوء بناءً على مجموعة متنوعة من الظواهر. ومن أمثلة الظواهر التي يمكن قياس الزمن "المطلق" لها بالشكل المطلوب، ابتعاد المجرات، ومعدل اضمحلال النجوم، وتبخر ثقب أسود، واضمحلال ذرة مشعة في مكان معين. وهذه الأخيرة تشبه ساعة السيزيوم الموجودة على كوكب، والتي يتعين على سفن الفضاء الإسرائيلية في كل مرة أن تتوقف وتتحقق من حالتها. أي أن هذه أيضًا ساعات نسبية ولا يوجد شيء غير نسبي كما حاول أن يدعي بالنسبة للساعات المؤقتة.

  132. روبي,
    إذا نظرنا إلى كل وحدة زمنية على أنها شيء محدد بذاته، فإن كوننا يتكون من أبعاد لا نهائية. ومع ذلك، بما أننا نقبل الزمن باعتباره بُعدًا محدودًا (مفردًا)، فإن الأبعاد الثلاثة التي نعرفها كافية للتوجيه في الفضاء. يمكن النظر إلى الأبعاد الإضافية لنظرية الأوتار على أنها فضاء فرعي من الأبعاد الثلاثة الموجودة.

  133. R. H.، شكرًا على الإشارة إلى Stohlandia. كتاب جميل.
    ولكن قبل أن تسارع إلى الدفع لأمازون، يرجى أن تضع في اعتبارك أن هذا نص انتهت صلاحية حقوق النشر الخاصة به منذ سنوات عديدة، لذا يمكنك تنزيله مجانًا من Project Gutenberg:
    http://www.gutenberg.org/ebooks/search.html/?format=html&default_prefix=titles&sort_order=downloads&query=flatland

    فيما يتعلق بساعة درجة الحرارة، أردت أن أخبرك أنه يبدو لي أنك لست على حق تمامًا. نظرًا لأنه يتم حساب عمر الجسم بناءً على اضمحلال أحد النظائر ("نصف العمر")، فمن الممكن حساب الوقت المطلق بناءً على قرب درجة الحرارة من الصفر المطلق. هذا على افتراض أن معدل فقدان الحرارة هو أيضًا أسي.

    تفسيري لظاهرة "العودة في الوقت المناسب" يعتمد على نموذجي. وبما أنني لا أريد تطويره هنا، سأطرحه بإيجاز فقط:
    لقد قدمت الفوتون (وكذلك الإلكترون) على أنه "قطعة من الفضاء الفارغ". ويؤدي دخول الجسيم إلى منطقة الفوتون إلى توقف الفوتون عن الوجود، ولكن في نفس الوقت يتم إنشاء فوتون جديد في المكان الذي كان فيه الجسيم من قبل. وبهذه الطريقة تكون حركة الفوتون في الفضاء ثنائية الاتجاه. وفي تجربة "الاختيار المؤجل"، يتم استبدال الجسيمات المراد أن يستخدمها الفوتون في طريقه بجسيمات أخرى بعد إخفاء الشاشة.
    إن تغير استقطاب الفوتون (في تجربة بيل)، وكذلك تغير دوران الإلكترون، تم شرحهما أيضًا بطريقة مماثلة، ولن أتوسع في ذلك الآن.

  134. نظرية الأوتار متفق عليها بالفعل والتي تسمح بوجود 10 أبعاد أو أكثر، ويبدو لي أن هذا المنتدى يحتاج إلى "دماء" جديدة لتحسين المناقشة.

  135. إسرائيل،

    ملخص أبنائي:

    1) تكرر وتنتقدني لرفض نسبية الزمن على الرغم من أنني أعتقد أنني أوضحت لك على مرأى من الجميع أن الأمر ليس كذلك وأن ساعات درجة الحرارة لديك نسبية مثل أي شيء آخر ويمكنك أيضًا النظر إلى أي ظاهرة بطيئة تحدث يحدث "هناك" مع مرور الوقت مثل تبريد النجوم، وابتعاد المجرات، وتبخر الثقوب السوداء، وما إلى ذلك.

    2) لقد تجاهلت دحضًا واضحًا لنموذجك حيث كان من المفترض أن تختفي الجسيمات الموجودة في المحور بسرعات عالية، لكن هذا لم يحدث. إن القول بأنك لا تتعامل مع الجسيمات ذات الكتلة هو التلويح باليد. هل من الممكن أن النسبية تنطبق فقط على الجسيمات ذات الكتلة؟

    3) تفسيرك لعدم المحلية بنفس الموقع النشط ضعيف للغاية. لم تجيبني كيف "يبلغ" الإلكترون هنا بالضبط الإلكترون الموجود في الجحيم خلف Tiz a Nebi لتغيير دورانه دون أن يكونا في الواقع وحدة واحدة.

    3.5) إن تفسير أن الإلكترونات مرتبطة في بعد آخر لا يقع في معقوليته من حقيقة أن الفوتونات تنتقل بسرعة أعلى من سرعة الضوء، فقط ما لا تلتقطه أجهزة الاستشعار لدينا للأسف.

    4) والأسوأ من ذلك كله أنك تستهين بما لا تعرفه. إذا كنت لا تمانع أن Flatland ليس كتاب أشباح ولكنه قصة رمزية رياضية مسلية ومثيرة للتفكير بقدر ما تبدو غريبة حول الأبعاد والطبقات الاجتماعية. كتاب موصى به للغاية
    http://www.amazon.com/Flatland-Dimensions-Illustrated-E-Reader-ebook/dp/B004AM5AZO/ref=sr_1_1?s=books&ie=UTF8&qid=1332208762&sr=1-1

    5) منصات ؟؟ ماذا كنت في اللواء؟ أليس هذا من جفعات هالفون؟

    على أية حال، يبدو لي أننا أنهكنا أنفسنا وبدأنا في مضغ الطعام القابل للمضغ. لذا، ما لم يكن لديك بعض الذكاء المذهل، فلنضع المناقشة هنا حتى تنشر نتائج تجربتك.

    في هذه الأثناء، هل يمكننا مناقشة كيف يمكنك عد أكثر من رزمة أو اثنتين دون وجود جهاز كمبيوتر في الحذاء، حيث سيؤدي العثور عليه إلى كسر عظامك أولاً على يد حراس أمن الكازينو ثم على يد رجال الشرطة الذين يتقاضون رواتبهم و رفع دعوى قضائية من هنا إلى بيلاجيو؟

  136. يوفال شكرا على المجاملات، ولكنني لا أفهمها.
    سوزان وتومر، مثل إسماعيل وإسحاق، هما واحد لأمهما وواحد لأمهما.

    لذا، نعم، الزراعة والري بالتأكيد، لكن النمو والاستثمار والفضل يعود إلى لوري.

    حل محلي؟ يجب أن أسمع هذا.

    ر.ح.

    1. "الاحتمال الأول لأينشتاين هو الأرجح بكثير"

    يبدو لي أنك إذا قبلت، كما قلت، فكرة أن شخصًا في سفينة فضائية شاردة يمكن أن يعرف أن اليوم صيام لأنه يوم الغفران بمجرد قياس درجة الحرارة - فأنت ترفض نسبية الوقت. والدليل هو تقني فقط. ولكن أعتقد أننا انتهينا، إلا إذا كنت ترغب في الاستمرار.

    2. "من خلال ادعائك بوجود جسيمات أسرع من الضوء، هل قمت بحل مسألة الموضعية؟" أنا أدعي ما كنت أطالب به دائمًا: الحل الخاص بي للموقع النشط يسمح ببرمجة غير محلية.

    كل شيء يمكن تفسيره من خلال الأبعاد الإضافية والسحر والشياطين والأرواح. تبدو فلاتلانديا مثل وحدة سيفتيا التي كانت لدينا في رفيديم.

    سؤال: بعد أن استوعبنا مسألة اللا محلية، هل يمكنك تفسير نتائج تجربة الاختيار المؤجل دون اللجوء إلى الأسوأ على الإطلاق: التأثير على الماضي من المستقبل؟

    3. أنا فخور جدًا. أفضل أن أكون أقل فخرًا وأجعلها قريبة من المنزل، لكن ماذا يمكنك أن تفعل، أنت لا تتوقع أن يذهب سانوزي إلى جامعة جنوب كاليفورنيا، أليس كذلك؟ كيف هي مثل أصدقائها الفارسيين الذين لا يسمح لهم آباؤهم الحكماء بالدراسة بعيدًا عن المنزل؟

    وكما قلت، الفخ الأمريكي. وعندما يحين الوقت، إذا استطعت، ستمنح الأطفال تعليمًا "ممتازًا"، حتى على حساب فقدان اتصالك اليومي بهم، وستجلس مع شريك حياتك في المنزل الفارغ. ولكن أيا كان - كن فخورا.

  137. إسرائيل إسرائيل إسرائيل توكل على الله.

    1) "استنتاج أينشتاين: سرعة الضوء ثابتة بالنسبة لأي جهاز قياس، وما يتغير هو الزمن والمسافة. السؤال: هل هذا هو الخيار الوحيد المتاح؟ هل أُغلق الأمر نهائيًا مثل جزيرة المحلة؟ جوابي هو لا. هناك احتمال نظري آخر: سرعة الضوء تتعلق فقط بالمشاهد".

    صحيح، أوافق، هناك دائما إمكانية نظرية. لكن الاحتمال الأول لأينشتاين هو الأرجح بكثير في ضوء المحاولات التي لا تعد ولا تحصى والتي قام بها زنادقة مثلك منذ عام 1905 وحتى اليوم بما في ذلك تجربة أوبرا، والتي فشلت في إثبات أي خطأ. إذا نجحت في محاولتك، فقد لعبتها وحتى ابنتك ستحييك. وعلى أية حال، كما قلت، سيكون الأمر ممتعا!

    2) اللامحلية - بادعائك أن هناك جسيمات أسرع من الضوء، هل قمت بحل اللامحلية؟ ماذا تقول؟ أعطني إستراحة! كيف تفسر أن إلكترونًا واحدًا من بين المليارات يرسل معلومات إلى إلكترون آخر صغير يبعد عنه بسنوات ضوئية ويختبئ في الطريق 7 مجرات و18 ثقبًا أسود وفي غابة من مليارات الإلكترونات المتماثلة؟ وكيف يعرف من المتورط معه؟ وله فقط؟
    التفسير الأكثر إقناعا الذي سمعته عن اللامكانية، حتى وإن لم يكن لها سند تجريبي، هو أن الإلكترونين متصلان ويشكلان في الواقع جسما واحدا عندما يتم الاتصال في بعد آخر حيث يكونان متجاورين فعلا. هل تعرف فلاتلانديا للكاتب إدوين أبوت؟ ويوضح هناك كيف يكون هذا الشيء ممكنا في الانتقال من بعد واحد إلى بعدين ومن بعدين إلى ثلاثة أبعاد.

    3) البيولوجيا الخلوية والجزيئية مثل قول الفيزياء الفلكية. إنه مجال واسع وممتد يمكن أن يشمل تحت أجنحته تقريبًا كل الأمراض المعروفة وكل كائن حي وكل مجال من مجالات علم الأحياء تقريبًا. لذلك عليك معرفة المزيد من التفاصيل. تحقق أيضًا من المنشورات الخاصة بالمختبر الذي تذهب إليه وسترى. على أية حال، جامعة بنسلفانيا هي جامعة عظيمة ويجب أن تكون فخورًا بها ولا تفكر فقط في نفسك وفي مناديلك (أنا بطل مع أطفال المدارس الابتدائية، وسوف تتغير آرائي في المستقبل على ما أعتقد).

    يوفال، هيا، نحن جميعًا متوترون، لقد قمت ببناء التوقعات، والآن سوف تنفجر.

  138. كل التوفيق للأب الذي زرع وسقى وربيا واستثمر 🙂
    إسرائيل! من فضلكم، لا تتعجلوا في القول: "لا مفر من الاعتراف بالحقيقة: الكون جزيرة محلية". أعتقد أنني توصلت إلى حل محلي للغز.

  139. ر.ح. سمعت.

    فلسفة.

    خصصت APR 99% من مقالتها للقضايا الفنية والصيغ. وفي النهاية، في جملة واحدة، قدموا البديل: اللامكانية. بدا الأمر وهميًا للغاية، و"مخيفًا" على حد تعبير أينشتاين، لدرجة أن عرضه ذاته كان من المفترض أن يغلق الباب أمام فكرة المتغيرات غير المخفية. بور نفسه لم يؤمن باللامحلية.

    إذا كنت تتذكر بحث نيك هربرت، فقد بدأ بقول نيك "سأثبت عدم وجوده" وانتهى به الأمر بالعثور على دليل أنيق للغاية على عدم المحلية.

    ونحن، يوفال، أنت وأنا، لم نصدق ذلك وبحثنا عن الخطأ: المستقطبون؟ الوسيط؟ الإعداد التجريبي؟ ففي نهاية المطاف، لا أحد يتوقع منا أن نصدق أن الكون بأكمله متصل بواسطة "وظيفة موجية" غامضة، والتي من خلالها تتدفق المعلومات، وفي وقت صفر!

    ومع ذلك، لا مفر من الحقيقة: فالكون جزيرة محلية.

    دعنا نعود إلى تجربة الأنبوب.

    يتحرك أنبوبان بالنسبة لبعضهما البعض ويتحرك الفوتون بالنسبة لكليهما. وتظهر القياسات أنه يتحرك بنفس سرعة الضوء بالنسبة لكليهما. استنتاج أينشتاين: سرعة الضوء ثابتة بالنسبة لأي جهاز قياس، وما يتغير هو الزمن والمسافة.

    السؤال: هل هذا هو الخيار الوحيد المتاح؟ هل أُغلق الأمر نهائيًا مثل جزيرة المحلة؟

    جوابي هو لا. هناك احتمال نظري آخر: سرعة الضوء مرتبطة فقط بالراصد. وكما أنك لن تكون قادرًا على رؤية شعاع ضوء الأشعة تحت الحمراء إلا إذا كنت تتحرك بسرعة معينة بالنسبة إليه، فلن تتمكن إلا من قياس مكون الفوتون الذي يكون عند سرعة الضوء بالنسبة لك. كل شيء آخر غير معروف بالنسبة لك، مثل الأشعة تحت الحمراء أو الضوء الفائق. إنه موجود - ولكنه شفاف بالنسبة لك.

    هلوسه؟ ربما. إن تغيير الوقت والمسافة يبدو مجرد وهم. وأيًا كان الأمر، فخلافًا لما هو الحال في غير المحلية، فإن هذا الاحتمال موجود.

    علاوة على ذلك: فهو يتناسب بشكل جيد مع افتراض أن الكون لديه درجة حرارة هي دالة مستمرة للزمن، ومع اكتشاف طبيعة الفوتون الاحتمالية والمنتشرة في الكون، وهو الأمر الذي عارضه أينشتاين بشدة طوال حياته. بالنسبة له، كان الفوتون عبارة عن كمية من الطاقة تترك النقطة أ وتطير بسرعة الضوء إلى النقطة ب، وهو ما نسميه في الحظيرة: فوتون – فوتون.

    هل هناك شيء في كلامي؟ على الاغلب لا. على الأرجح أن هناك ببساطة تفسيرًا، وربما يكون بسيطًا جدًا، لا أعرفه.

    الميزة - يمكنك القيام بالتجربة. سأواصل بالطبع البحث عن التفسير النظري، لكنني سأواصل أيضًا العمل على أفكار للتجربة. مهما كان الأمر، سيكون ملعقة.

    على أسئلتك:

    1. أعتقد أنه وفقًا لصورة الكون في عام 1905، فإن افتراضه كان منطقيًا للغاية. اليوم يبدو لي أنه كان مترددًا.

    2. مقاييس درجة الحرارة لا تقيس التغيرات على حد علمي. إنهم يقيسون درجة الحرارة، ويمكن أن تظل مستقرة لفترة طويلة. سيستمر مقياس الحرارة في إظهار ذلك طوال الوقت.

    3. فكرتي تحدثت عن الفوتونات التي توصف فيها بأنها موجات تتحرك في الموقع النشط. ولا يتعامل مع المقالات في هذه المرحلة. سؤالك مشابه لحقيقة أنني إذا عرضت خصائص الموجات الصوتية في الهواء، فسيتعين علي أيضًا أن أتناول خصائص الهواء، أو أي وسط آخر للموجات الصوتية. لا يعني ذلك أنها غير ذات صلة تمامًا، ولكن يجب التمييز بشكل واضح بين الموجة والمادة، بين الفوتون والبروتون، على الرغم من أن كلاهما يندرجان على ما يبدو تحت فئة: الجسيمات الأولية. هم ليسوا. يمكن للكتلة أن تتحرك بسرعات مختلفة، بما في ذلك 0. عادة ما تكون للموجة سرعة ثابتة، والتي تختلف دائمًا عن 0 (ما لم تكن ثابتة). وفقًا لنظريتي، فإن الفوتون ليس سوى موجة، مثل الموجة الصوتية، تحمل معها زخمًا ولا يمكن أن توجد في حالة سكون.

    4. قصدت أنه ربما يمكنك أن تشرح لي ما هو هذا المجال وما هو المتوقع منها. وكأن طفلك قال لك: أبي، سأصبح مديراً لقسم أمراض القلب في كابلان، ستمتلئ بالفخر وتبدأ بالادخار، لمساعدته براتب 5000 شيكل، على عكس أخيه المليونير جراح التجميل. .

    5. شكرًا لك وأيضًا يوفال على حسن الحظ، ليس لدي أي تفاصيل بالأمس فقط أبلغتني بها. على حد علمي، فإن "Snooze the Astronaut" هو بحث فقط، وليس ممارسة خاصة.

    مبروك.. عن ماذا بالضبط؟ عن حقيقة أننا حتى في السنوات العشر القادمة سنراها يومين في السنة كما في الخمس الماضية؟ كوننا وقعنا مثل أي شخص آخر في الفخ الأمريكي المتمثل في ملاحقة أفضل مدرسة دون أن نفكر فيما سيحدث لمصير الأسرة المشتتة؟ أين الإجازات؟ أين أبي؟ اين امي؟

    وأين المنديل؟

    لا يزال تومر أميًا.

  140. شيما إسرائيل,

    1) لا أعرف لماذا خصص أينشتاين الفصل الأول لمزامنة الساعات مع الضوء وليس مع مقياس الحرارة. ربما لأنه اعتقد أن درجة الحرارة هذه نسبية أيضًا؟ ماذا لو كان هناك أكوان أخرى؟ ربما لأنه لم يسمع عن الانفجار؟ ربما لأنه كان الجو باردا في سويسرا عام 1905؟

    2) إذا كنت لا تهتم بمقاييس الحرارة النسبية أيضًا، فعليك التحقق من تغير درجة الحرارة من الحرارة المرتفعة إلى الحرارة المنخفضة. في الكون الذي لا يتغير مع درجة حرارة ثابتة، لن تتمكن من قياس أي شيء.

    3) هل يميز نموذجك بين الجسيمات ذات الكتلة وتلك التي ليس لها كتلة؟ أي أنه إذا كانت هناك كتلة، فستحصل على سرعة الضوء كحد أعلى وكل النسبية صحيحة، ولكن عند التبديل إلى الفوتونات، يتغير كل شيء فجأة ويمكنها المرور عبر C، لكنها لم تعد قابلة للامتصاص بواسطة أجهزتنا ؟ يبدو لي هذا الانفصال بمثابة التلويح باليدين بشكل جدي للغاية.

    4) هايتكيست؟ استراتيجي؟

    5) تهانينا، أخيرًا شخص وضع قدميه على الأرض لعلاج أبي من الأنفلونزا. بالمناسبة، مع من ستدرس الدكتوراه ولماذا؟

  141. ر.ح.
    وإذا لم تكن هناك مشكلة، فلماذا خصص أينشتاين الفصل الأول من النسبية لمشكلة مزامنة الساعات باستخدام الأشعة الضوئية؟ لماذا لا تقوم فقط بسحب مقياس الحرارة وهذا كل شيء؟

    حسب فهمي فإن الزمن المطلق لا وجود له في العلاقات وكل ساعة ليست أكثر من عداد سرعة كما يدعي يوفال. يتطلب التزامن عبر المجرات المتراجعة استخدام الأشعة الضوئية ويخضع لجميع قيود التزامن المماثل الذي اقترحه أينشتاين، والذي ينتج عنه إطالة الوقت، والذي تُعفى منه الساعات المؤقتة.

    وألم أذكر أن مناقشتنا بأكملها حتى الآن لم تشير إلا إلى الزمن والفوتونات؟ أن الأجسام ذات الكتلة التي يتم تناول المحور فيها، هي موضوع جديد قد يأخذ 1000 إجابة أخرى؟

    وأن التناقض الخاص الذي أشرت إليه من نظرية بيل التي قالها أينشتاين نفسه لا يمكن أن يوجد لأن اللامكانية تتعارض مع النسبية؟ وليس الأمر أنه لا يوجد قرار بينهما، بل هناك قرار تجاه اللا محلية!

    بحار، فيزيائي...

    بالمناسبة، لست أنا، لكن ما تعريف حامل الدكتوراه في الرياضيات الذي يكسب رزقه من بيع الأجهزة الطبية؟ بائع متجول؟

    حسنًا، دعونا نستفيد من الاتصالات. وصلتني اليوم رسالة من الفتاة:

    أهلاً! سأذهب إلى جامعة بنسلفانيا للحصول على درجة الدكتوراه في علم الأحياء الخلوي والجزيئي

    لذا، بما أنك في الميدان، وأنا على اتصال بك أكثر بكثير مما كنت عليه مؤخرًا، هل يمكنك أن تشرح لي ما هي الأمور المفترضة؟

  142. إسرائيل،

    أنت لم تجب على أي من حججي. اتفقت على أنه من الممكن تحديد الوقت حسب الأشياء التي تحدث في نظام خارجي مثل درجة حرارة الكون أو بعد المجرات، لكنني لا أوافق على أن هناك ما يتناقض أو يطرح أي مشكلة.

    لم تجب على الإطلاق والشيء الرئيسي هو لماذا حقيقة عدم رؤيتك للجسيمات تختفي في المحور لا تشكل دحضًا صارخًا لنظريتك التي ترى أن الأشياء تتجاوز السرعة ولكن ببساطة لا يمكن التقاطها من قبلنا جهاز.

    نظرية بيل هي من ميكانيكا الكم وكلنا نعلم أن ميكانيكا الكم تناقض النسبية وهو تناقض حقيقي. ولهذا السبب يبحث العالم كله عن نظرية موحدة.

    أنا بحار كما أنت فيزيائي.

    شيء واحد ربما نتفق عليه كلانا هو صراخ هذا الطفيلي المسمى بالأشباح. الصفر المطلق.

  143. ر.ح.
    نسيت أن أذكر النقطة الأولى التي أثرتها، وهي التقصير المتسارع للأجسام.
    والسبب هو أنه بدون إطالة الزمان لا ينقص الأجسام مطلقًا. لقد طرحته فقط لأبين أنه حتى لو كان موجودًا، فهو غير ذي صلة.

    أيها الأشباح، أنا لا أتمكن من الإمساك بها، كما تقول دائمًا.

    لا أفهم لماذا تذمرت من الحديث الذي دار بيني وبين R.H، وأجبت على سؤال وكأنه يخص النقاش، لتعود وتكشف عن طبيعتك الطفيلية.

    من أنت، أي فيروس يغير تركيبته الجينية لخداع الجهاز المناعي للخلية؟ ففي النهاية، بمجرد أن تتعرف على فيروسات مثلك، فإنها تطردك على الفور لتعود إلى الحالة التي كانت لديك قبل أن تتمكن من إخفاء نفسك: عظمة ميتة هامدة.

    فإما أن تجيب، في الفيزياء، على السؤال الذي زعمت أنك تجيب عليه، أو ستعترف بأنه ليس لديك أدنى فكرة عما نتحدث عنه هنا على الإطلاق، ستغادر وربما ستندم، أو ستعود إلى بيتك: الصفر المطلق.

    وبما أن جميع المعلقين هنا هم من القدس والمنطقة المحيطة بها، فربما أنت تنتمي إلى هذا المكان أيضًا. سمعت أن هناك الكثير من الشقق المسكونة هناك.

    وأنت لم تفهم ما قلته. قلت إنني لست فيزيائيًا، وليس لأنني لم أدرس. لقد درست الفيزياء في الواقع في واحدة من أفضل الجامعات في العالم. وأنت صفر؟

  144. ر.ح. محبوب.

    يؤسفني أن أخيب ظنكم في افتتاح الألفية الثانية للتعليقات - فأنا لست فيزيائياً. (هل يوجد أحد هنا بالمناسبة؟ أقصد باستثناء المعالجين "إسرائيل لا تفهمونها")

    لا يعني ذلك أنني لم أدرس الفيزياء - لقد فعلت ذلك بالفعل - ولكن ربما ليس بما فيه الكفاية. وإلا لن نكون هنا.

    أطرح هذا الموضوع هنا كما هو الحال في منتديات الفيزياء الأخرى على أمل مقابلة شخص لديه الخلفية المناسبة، أو حتى لا، يمكنه أن يشير إلى خطأ في الحجة.

    ما زلت لم أتمكن من العثور على شخص مؤهل يكون قادرًا وراغبًا في معالجة المشكلة معي من البداية إلى النهاية، بما في ذلك الرسوم، كما فعلت أنت.

    ومع ذلك، يبدو لي أن النتيجة التي توصلنا إليها هي أنه من الممكن نظريًا وجود نظام يستقبل الوقت المطلق من الكون بنفس الطريقة التي يستقبل بها هاتفي الخلوي الوقت من AT&T، وأن هذا الوقت ستتم مزامنته تلقائيًا مع أي نظام آخر مماثل - يتعارض مع الفكرة الأساسية للنسبية، كما تم التعبير عنها في فصلها الأول: "تعريف التزامن".

    أما عن أسئلة سقراط:

    طوال المناقشة الطويلة، لم نذكر أبدًا الأجسام ذات الكتلة. الفوتونات فقط. هناك فرق بين الموجة، الموجة الصوتية على سبيل المثال، التي لا كتلة لها ولكن لها سرعة واتجاه، ويبدو أنها تحمل معها كمية دافعة، وبين الجزيئات التي "تستخدمها" كوسيلة للنقل (الهواء، الماء، الأسلاك).

    تشير الصيغة E=MC^2 ضمنيًا إلى الأجسام ذات الكتلة. إنها في الواقع تناسخ لمعادلة E=PC من أيام ماكسويل. وفقًا لمعادلة زيادة الكتلة النسبية، فإن كتلة الجسم ستزداد كلما اقتربت من الصفر، وستصل إلى ما لا نهاية بسرعتها الخاصة. يختلف الوضع مع التاكيونات، التي لم تتحرك أبدًا بسرعة أقل من سرعة الضوء.

    لذلك، رداً على سؤالك، ليس لدي أي مشكلة مع ما يحدث في سارن. هذا مجال مختلف

    إذا بدا لك أنه من المستحيل تجاوز سرعة الضوء، فكيف تفسر نظرية بيل وتجاربها الجانبية؟

    إذا بدا لك أنه لا يوجد تناقض مع النسبية، فقل ذلك، ويسعدني أن أقتبس من أحد معارفنا المتبادلين الذي يعتقد بالتأكيد أن التحول الدوراني بسرعة تتجاوز سرعة الضوء هو تناقض مع النسبية. أينشتاين، في ورقة بحثية تُعرف باسم مفارقة EPR.

    بالمناسبة، أيها الكيميائي، إذا وضعت البنزين على قاربك السريع، فهناك حاجز سرعة واحد لن تتمكن من تجاوزه مهما كان محرك القارب قويا وبطوليا: سرعة الصوت في الماء.

    وهذا هو السبب أيضًا في أنه يكاد يكون من المستحيل لطائرة ذات مروحية أن تتجاوز سرعة الصوت بمفردها.

    مائير - لم أنساك، كنت مشغولاً فقط كما ترى.. لدي فقط سؤال: هل سمعت أو قرأت كتاب "المعرفة غير المألوفة"؟

  145. شكرا لك مئير،
    في ذلك الوقت اقترحت تجربة موازية لتجربة بوينغ: ضع ساعة واحدة في الطابق السفلي من مبنى مرتفع وساعة ثانية على السطح. حجتي هي أنه بسبب الاختلافات في الجاذبية، فإن الساعة الموجودة في الطابق السفلي سوف تتأخر عن الساعة الموجودة على السطح. عندما يكون التأخر مرئيًا بالفعل، فسوف يقومون بالتبديل بين الساعات ومن ثم ستبدأ الساعة التي كانت متأخرة من قبل في التسارع. والغرض من هذه التجربة هو إظهار أن تجربة بوينغ لم تثبت أن الانحرافات في الزمن ترجع إلى السرعة. ألغى أستاذي اقتراحي وقال إنه بسبب اختلاف الارتفاع فإن الساعة الموجودة على السطح ستتحرك بشكل أسرع حول مركز الأرض 🙂

  146. اليوبيل,

    "ليس من الدقة القول بأن الساعات تقيس الوقت. جميع الساعات التقليدية، من الساعات الشمسية والساعات الرملية والمائية، والساعات البندولية إلى الساعات الذرية، لا تقيس الوقت ولكنها تحسب فقط نبضات الدورات التي تحدث داخلها وتعرض نتيجة المخزون.

    ذلك صحيح.

    (رغم أن من قام بتجربة بوينج ومن ذكرها كمرجع لشيء ما لن يعترف بصحة هذا القول حتى لو لم يكن لديه ما يقوله ضده من ذكاء)

  147. ر.ه.رفاعي.م
    لتجنب فقدان الرد الطويل، أكتبه مسبقًا في معالج النصوص، وهو ما أفعله الآن.
    لقد تحدثت عن نقطة مستقلة واحدة يمكن من خلالها مزامنة الساعات. ومع أن الأمور التالية، المتعلقة بالوقت والساعات، ليست مخصصة لك وحدك، ولكن بما أنك الوحيد الذي يرد هنا على الأمر وليس ساخراً، فإنني أرفقها بالرد الموجه إليك.
    ليس من الدقة القول بأن الساعات تقيس الوقت. جميع الساعات التقليدية، من الساعات الشمسية والساعات الرملية والمائية، والساعات البندولية إلى الساعات الذرية، لا تقيس الوقت ولكنها تحسب فقط دقات الدورات التي تحدث داخلها وتعرض نتيجة المخزون. لكي نقول إن عدادات النبض هذه تقيس شيئًا عالميًا، علينا أن نفترض أن شيئًا خارجيًا يؤثر على الجميع بالتساوي. لقد تعلمنا على مر الأجيال أن الظروف الخارجية تؤثر على عمل الساعات. فلو قارنا نتيجة مخزون الساعة المائية أو الساعة البندولية مع الدورة اليومية للشمس مثلا يبدو أننا في المنطقة القطبية سنحصل على نتائج مختلفة عنها في المنطقة الاستوائية وهذا بسبب اختلاف درجات الحرارة أو التغير في ثابت التسارع . وبالمثل، تتأثر الساعة الذرية أيضًا باختلافات درجات الحرارة وقوة مجال الجاذبية. من الملاحظات نرى أنه إذا تمكنا من الحفاظ على نفس ظروف درجة الحرارة والجاذبية لساعتين متطابقتين، فسوف تظهران بالفعل نفس نتيجة المخزون حتى بعد عدد كبير من الدورات.
    عندما نتحدث عن مزامنة الساعات، فإننا نعني التأكد من أن الساعتين ستظهران نفس القراءة. في العمليات التي تتطلب التنسيق بين الأطراف المختلفة (اجتماعات العمل، إطلاق الصواريخ، إلخ)، ضع في الاعتبار أنه حتى يتم الانتهاء من العمليات المنسقة، ستظهر الساعات المختلفة نفس الرقم أو قريبة بدرجة كافية من نفس الرقم. السؤال الأول المتعلق بالوقت، والذي يُقال إنه يثير اهتمام الفيزيائيين (بشكل أكثر دقة، فلاسفة الطبيعة)، هو ما الذي يجعل الساعتين الموضوعتين في مكانين مختلفين تستمران وتظهران نفس القراءة. إذا اعتمدنا فقط على البنية الداخلية للساعة، فسيتعين علينا أن نفترض، على سبيل المثال، أن الساعة تتمتع بنوع من الإحساس التخاطري الذي يسمح لها بمعرفة ما يحدث في الساعات الأخرى. ومثل هذا الافتراض غير معقول على الإطلاق، ونحن نرفضه جملة وتفصيلا. ومن ناحية أخرى، بما أننا نعلم بالفعل أن العوامل الخارجية تؤثر على عمل الساعات، فمن المرجح أن نفترض أن هناك عامل خارجي يؤثر على جميع الساعات بنفس القدر. لتوضيح هذا الافتراض، يمكن تقديم الكون بأكمله كما لو كان ينبض. ليست نبضات مستقلة فريدة لكل مكان، ولكنها نبضات موحدة في جميع أنحاء الكون بأكمله. لتوضيح ذلك، يمكننا أن نستشهد بمثل من عالم التشريح والطب: لكي يعمل الجهاز الدوري بفعالية، يجب تنسيق نبضات القلب والشرايين؛ ويتحقق هذا التنسيق من خلال خلايا خاصة تولد الإشارات الكهربائية؛ وعندما تتوقف هذه الخلايا عن العمل بشكل صحيح، يمكننا استبدالها بجهاز تنظيم ضربات القلب الاصطناعي الذي يقوم بنفس الوظيفة. إذا تم تنسيق نبضات الكون بواسطة جهاز تنظيم ضربات القلب هذا، فيمكن اعتباره النقطة الفريدة التي كنت تهدف إليها.

  148. إسرائيل،
    بالفعل سؤال جميل طرحته، هل تتدرب على الفصح؟ لذا:

    1) كما تعلم أنا لست فيزيائيًا ومعرفتي تخدش حد معرفتك، ولكن في رأيي المتواضع الخطأ هنا موجود في الجملة "حسب ديفيد إسرائيل أنه عند هذه السرعة المنخفضة يكون التقصير ضئيلًا للغاية لدرجة أنه قد يحدث" سيكون من الصعب كتابتها هنا". لأنه حتى هذا الإهمال سيؤدي إلى عدم وجود تناقض.
    دعونا ننظر إلى ما سيحدث إذا تحرك الأنبوب B بسرعات قريبة من سرعة الضوء. فهل تتفقون هناك على أن ما سيحدث هو أن طول الأنبوب سيتقلص وسيتحرك الزمن بشكل أبطأ من A، فلا يوجد تناقض؟

    إذا كان الأمر كذلك، ففي الحث البسيط، هذا أيضًا ما سيحدث عند السرعة المنخفضة والفرق الذي تسميه "ضئيلًا" هو الذي سيحدث الفرق.

    2) يسأل سقراط:
    ألا يشكل ما يسمى بالمعجل المحوري أو أي معجل آخر دحضا لنظرية اختفاء الجسيمات فوق سرعة الضوء؟
    وفقا لأينشتاين – E=MC^2، مما يعني أنه مهما استثمرت من طاقة، فلن تتمكن من تجاوز سرعة الضوء، وما سيحدث هو أن الكتلة ستزداد بشكل مباشر. استثمر طاقة لا نهائية، وستحصل على كتلة لا نهائية وسيظل C C.

    وبحسب شابيرا - فإن استثمار الطاقة في الجسيمات سيسرعها إلى سرعات عالية فوق سرعة الضوء ثم ما سيحدث هو أنها ستختفي لأن مستشعراتنا الضعيفة لن تراها وفقا لمبدأ البندول الباليستي. لذا، إذا قمنا برمي 100 بروتون وقمنا بالتسريع والتعجيل بواسطة المغناطيسات الهائلة الموجودة في المسرع بعد بضع دورات، فإن عدد البروتونات سوف يرتفع وينخفض.

    والسؤال الآن، ما الذي تعتقد أنك تراه في سارن:

    1) زيادة في كتلة البروتونات؟
    2) إزالة البروتونات والحفاظ على كتلة تلك التي لا تزال ممتصة؟

  149. لا أنوي الخوض في نموذجي مرة أخرى، ولكن فقط أتناول السؤال الذي طرح علي بعد استخدامي لـ "Notov":
    https://www.hayadan.org.il/astronomers-reach-new-frontiers-of-dark-matter-130112/#comment-332405
    في ذلك الوقت، وقبل العديد من الردود، قدمت الفوتون كقطعة من الفضاء الفارغ. عندما يدخل جسيم إلى المكان الذي يوجد فيه الفوتون، فإنه يترك وراءه فوتونًا. وبهذه الطريقة تتعايش حركة الفوتون مع حركة الجسيم في الاتجاه المعاكس.

  150. أشباح
    يجب علي أن أذهب لأنام الان. عندما يأتي أصحاب المعاطف البيضاء، لا تثوروا.
    وليس هناك حاجة حقًا للرد على تعليقاتي. لقد رأينا بالضبط ما أنت قادر على إنتاجه.
    لماذا لا تجرب شيئًا أبسط، ربما التطريز؟
    مساء الخير.

  151. إسرائيل
    الحانات؟ تل باروخ؟ ليس كل شخص لديه نفس التفضيلات مثلك.

    أبعد من ذلك، أردت الرد على تعليقك أعلاه، لكن بعد أن فهمت ما كتبته هناك، أدركت أنه لا داعي لتكرار كلامي لأنك ببساطة لا تفهم.

  152. أشباح
    كان أحد شروط إطلاق سراحك المبكر هو أن تستخدم الإجازة لدراسة الرياضيات والفيزياء. حسنا، في حالتك - الرياضيات والطبيعة.

    وفقًا لردك الأخير، فإننا نستنتج للأسف أنك أضعت وقتك في الحانات وفي تل باروخ، وبالتالي يتعين عليك إعادتك إلى منزلك الدائم - الجناح المغلق.

  153. ر.ح.

    دعونا نرى ما إذا كان بإمكاننا استخلاص أي استنتاجات مما توصلنا إليه حتى الآن.

    اذهب إلى الأنبوب.

    وهو طويل، طالما المنفى. كالعادة، كل شيء مبالغ فيه. لنفترض 100 ثانية ضوئية. قطرها 5 أمتار. سيتم وضع علامة على أنها

    كل 100 متر يتم تركيب الجهاز المعروف والذي يتضمن ساعة زمنية وساعة لدرجة الحرارة وكاميرا عالية الدقة. تتم مزامنة جميع ساعات تشيكوسلوفاكيا مع بعضها البعض بالطريقة التي اقترحها أينشتاين في المقالة الأصلية عن النسبية. تتم معايرتها لإظهار الوقت نفسه الذي تظهره ساعات درجة الحرارة، والتي يعرف الجميع أنها تتم مزامنتها بشكل مستقل.

    يوجد بداخله أنبوب مماثل ولكن بقطر 4 أمتار. سنضع علامة عليها بـ B

    المنشأة بأكملها موجودة في ساحة الاختبار الدائمة لدينا في تيزنبي.

    A في حالة راحة تامة بكل المقاييس.

    يمكن أن يتحرك B بحرية داخل A، على طول محورهما المشترك.

    مسار التجربة:

    1. نسحب B قليلاً بعيداً عن A، وبعد ساعة ندفعه بحيث تصبح سرعته بالنسبة إلى A 3 م/ث.

    ما يخرج هو أن B يخرج من اليمين وفي الوقت المحدد بـ T0، يلتحم الجزء الخلفي من الأنبوبين. يواصل B رحلته داخل A.

    2. في الوقت T0، يتم إطلاق فوتون واحد على طول المحور المشترك للأنابيب.

    تم تصوير الحدث بأكمله في الذاكرة، ولكن نظرًا لأن الجميع يعرف مدى خجل الفوتونات وضعيفتها، فلا شيء يصل إلى فوتوناتنا ووظائف glo، حتى لا تصطدم بنا في منتصف التجربة.

    الان:

    يشق فوتونينو طريقه على طول المحور بهدوء وهدوء. إنه لا يدرك على الإطلاق حقيقة أنه يتحرك الآن بالنسبة إلى نظامين يتحركان بسرعة 2 م/ث لبعضهما البعض.

    3. يصل فوتونينو إلى الطرف الآخر من A بعد 100 ثانية بالضبط وفقًا لساعات A.

    4. يصل فوتونينو أيضًا إلى الطرف الآخر من B بعد 100 عام بالضبط وفقًا لساعات B.

    5. ولكن عندما تصل الكرة إلى حافة B، تبرز B بالفعل حوالي 300 متر من الجانب الآخر من A.

    ولا أقول بالضبط 300 متر بسبب اختصار المسافات. حسب ديفيد إسرائيل أنه عند هذه السرعة المنخفضة يكون الاختصار ضئيلًا للغاية بحيث سيكون من الصعب تسجيله هنا. في النصف C يكون التقصير حوالي 13٪. هنا - جزء واضح.

    النقطة:

    6. إذا قبلنا الافتراض بأن جميع الساعات المؤقتة تظهر دائمًا نفس الوقت في كلا الأنبوبين، وأنه نظرًا لعدم تسريع الأنظمة، فإن الساعات C تظهر نفس الوقت مثل الساعات الحرارية، فإن أي صورة لأربع ساعات معًا من A سيُظهر +B دائمًا نفس الوقت في جميع الساعات.

    7. بما أن فوت وصل إلى نهاية B في نفس الوقت الذي وصل فيه إلى نهاية A، واستغرق الأمر بعض الوقت لقطع مسافة 300 متر إضافية، فلا مفر من الاستنتاج بأنه تحرك بشكل أسرع بالنسبة إلى B منه إلى أ.

    8. حتى الآن تعمدنا المبالغة ببطء، لتسهيل الحدس. وسوف تعمل كذلك في أي سرعة أخرى.

    9. سيعمل هذا حتى لو تحرك B في الاتجاه المعاكس. بهذه الطريقة يمكننا الحصول على فوتونات بطيئة كما نرغب.

    10. يتناسب هذا تمامًا مع الوصف الذي لدينا للفوتون كدالة موجية منتشرة في الفضاء.

    11. وهذا يفتح باباً لفهم اللامحلية.

    12. هذا يمكن أن يفسر نتائج تجربة "الاختيار المتأخر" التي أجراها ويلر دون الحاجة إلى الأسوأ على الإطلاق، وهو العودة بالزمن إلى الوراء. (التفاصيل - عند الطلب).

    بصرف النظر عن اعتراضك المعتاد على مبدأ الثبات، الذي ينص على أن A يتحرك بالنسبة إلى B بينما يتحرك B بالنسبة إلى A، هل يمكنك العثور على أي خلل في الحجة؟

    قرأت جميع أنواع الكتب عن أينشتاين وشاهدت أيضًا الأفلام. أنا من جماعته بما لا يقل عن جالي.

  154. إسرائيل
    والتفسير هو: بنظام مرجعي مختلف عن الضوء أو الأشعة الراديوية.
    على سبيل المثال، نقطة مفردة خيالية يتم النظر إليها من قبل هؤلاء الفضائيين الذين أتوا من الكون الذي تعيش فيه على الأرجح.

  155. إسرائيل،

    أنا شخصياً أجد صعوبة أكبر بالنسبة لي في فهم ما كان عليه الكون الأبدي دائمًا. حدسيًا، يبدو الكون ذو البداية منطقيًا (وليس هذا المنطق يلعب أي دور هنا). ماذا كان قبل ذلك؟ لم يكن قبل ذلك. هل تتذكر ماذا كان قبل ولادتك؟ وهذا ما كان عليه الحال قبل الانفجار الكبير. أو هناك طريقة أخرى للشرح كما قال صديقنا رفائيم في إحدى ذكرياته "درجة الحرارة أقل من الصفر المطلق"، هذا ما كان قبل الانفجار الأعظم 🙂

    فيما يتعلق بسيرة أينشتاين الذاتية، فمن المثير للاهتمام أن صديقي أوصى لي بسيرة والتر إيزاكسون (وهي في حد ذاتها قصة مثيرة للاهتمام) وسأحاول الحصول عليها. هل هذا ما قرأته؟

  156. ر.ح.
    4K؟ لقد راجعت الآلة الحاسبة. لقد ظهر منذ حوالي 6.5 مليار سنة. هل تعتقد أنه كانت هناك بالفعل سفن فضاء ميكانيكية في ذلك الوقت، أم أنهم ما زالوا يستخدمون المجاديف؟

    فيما يتعلق بترجيح عنصر آخر في صيغة فريدمان - القليل عن شركة أبل.

    فيما يتعلق بالعلاقات وMetzgadol

    اعتراف شخصي - ميت على النسبية ومات أيضًا على أينشتاين شخصيًا. لا أعلم إذا كنت قد قرأت سيرته الذاتية، أو كل أقواله في مواضيع عالمية. لقد وضع نيوتن سيئ المزاج أو ماخ رائد الفضاء في جيبه الصغير. وهو أيضًا أكثر تواضعًا وبساطة من فاينمان الأسطوري، على الرغم من أنه عبقري حقًا.

    ولم أتمكن قط من أن أفهم بشكل بديهي كيف أنه لم يكن هناك شيء قبل الانفجار الكبير ولماذا لم يكن الكون لانهائيًا. ولكن هذا ما يقوله أهل الفهم، والتعاليم لا تتفق في رأيي. ليس لدي أي تفضيل بين الاثنين.

    يبدو لي أنه يمكنني إثبات ذلك لك الآن باستخدام ما اتفقنا عليه، إذا قبلت معادلة جميع أنظمة القصور الذاتي. أعلم أن الأمر لا يبدو واضحًا على الإطلاق، ولكن من وجهة نظر نيوتن أو أينشتاين، فإن السمكة التي تسبح في المحيط بسرعة ثابتة يمكنها بالتأكيد أن ترى نفسها في وضع الراحة والمحيط وكل ما فيه يتحرك. الشيء نفسه ينطبق على الطائرة أو كرة البندقية. أي شخص في حركة غير متسارعة هو في الواقع في حالة راحة. الحركة لا تتعلق إلا بنظام آخر.

    دع ضربة الشمس الحالية تمر، وسأوضح لك لماذا أعتقد أنه إذا قبلت المبدأ الأول للنسبية، فإن استخدام ما اتفقنا عليه بالفعل، يمكن أن يتعارض مع تمدد الزمن.

    أشباح
    ماذا كان عند t=0؟
    بقدر ما أعرف لا أحد يعرف.

    لكن سؤالي كان:

    1. هل ستتمكن المركبة الفضائية من مزامنة ساعاتها بحيث ترى نفس الوقت دون استخدام أي وسيلة اتصال مثل الأشعة الضوئية أو الراديو؟

    وعلى ذلك أجبت بنعم. هل يمكنك شرح كيف؟

  157. وفقًا لما يحدث في المستعرات العظمى، يقول المنطق أنه بعد الانفجار الكبير، تم ضغط جزء كبير من المادة الأصلية مرة أخرى في النواة الفردية وتوزع الفائض في كوننا.
    ليس من الواضح بالنسبة لي لماذا في جميع المقالات والبرامج العلمية التي قرأتها، لا يتحدثون عن إمكانية ضغط المادة في النواة على شكل ثقب أسود أو أي اسم فردي آخر

  158. إسرائيل
    أنت لا تحصل على هذه النقطة.
    هل يمكنك الإجابة عن حالة المادة في الكون عند الزمن t=0؟
    ومن المعروف أن هناك بعض الإشعاعات التي تحولت إلى مادة في زمن لا يساوي t=0.
    أي أنه بمجرد حدوث أي تدخل من طرف ثالث تجاه الإشعاع، في تلك اللحظة يتحول الإشعاع إلى مادة.
    وقد وردت تعريفات لهذه اللحظة (بداية الانفجار) تتضمن المفهوم (التجريدي) للزمن، بعد تغير حالة المادة/الإشعاع/المكان. وينص التعريف على أن الزمن لم يكن صفراً عند خلق الكون.
    باختصار، (بالأمس بدأت بكتابة تعليق هنا وعندما كان بطول لفيفة أستير تم حذفه عن طريق الخطأ. لذلك كتبت الآن باختصار، وإذا كانت لديك أسئلة سأحاول الإجابة عليها.) سيكون هناك دائمًا بعض النظام المرجعي في هذا الكون.

  159. طالب، التخنيون، من أجل لا شيء
    وإذا كان لديك، أو أي شخص يقرأ هذا، رأي تود التعبير عنه أو تعليق للتعليق على الموضوع المعني، فيرجى عدم تزييف كلماتك. وهذا تناقض تام لكل ما نعرفه عن مفهوم الزمن. بعد أن تعلمنا بالفعل أن نختلف مع الفكرة المتجسدة في النظرية النسبية، والتي بموجبها يتحرك الزمن بمعدل متغير ولكنه مع ذلك يحافظ على اتجاه للأمام، هناك من بيننا من يحاول إقناع الجمهور البريء بأنه يتحرك بالفعل إلى الوراء. حتى لو ركزت على الخطوط الشخصية، فهذا جيد بالنسبة لي، لأن هذا بالضبط ما أفعله هنا بنفسي لريادة الفكرة.

  160. طالب، التخنيون
    ويبدو لي أن هذه المرة، على غير العادة، كنت واضحًا جدًا. "نحن" هي كل واحد منا، الفيزيائيون والعلمانيون على حد سواء. حتى أنا، الذي أدعي أنني أستطيع فك جميع أسرار الكون بنموذجي الغامض، لا أعرف حقًا.
    إذا كنت تعتقد أنني مخطئ، فهذا يعني أنك تعتقد أن هناك شخصًا يفهم جيدًا طبيعة وسلوك الفوتون و/أو الإلكترون ويمكنه شرح الظواهر غير الواضحة أعلاه، فيرجى إخبارنا بذلك.

  161. طالب، التخنيون
    سؤالك يذكرني بإشعياء ليبوفيتش عليه السلام. في إحدى المقابلات، عندما قال المحاور "نحن..."، قال ليبوفيتش "من نحن؟ نحن أنا وديداني!» 🙂
    يمكنك قبول ذلك، إذا كنت ترغب في ذلك، ولكن من فضلك اسمح لي أن أطلب منك شيئًا. هل لديك تفسير لجميع الظواهر المعروفة المرتبطة بالإلكترونات و/أو الفوتونات (على سبيل المثال نتائج التجربة التي قدمها اللورد إسرائيل شابيرا إلى M.R.S.T.) بناءً فقط على ما تعرفه عن بنيتها وكيفية تقدمها عبر الفضاء؟

  162. م.ر.س.ت
    يرجى أن نضع في اعتبارنا أنه قد تكون هناك ظواهر لسنا على علم بها. على سبيل المثال، عندما يتحرك فوتون من مصدر ضوء إلى هدف، فإن شيئًا آخر، نسميه "تقاطر لأسفل" (فوتون معكوس)، يتحرك في وقت واحد من الهدف إلى مصدر الضوء. وبهذه الطريقة، فإن حجب الشاشة بعد مرور الفوتون لم يؤثر على الفوتون الذي مر، بل حول التنقيط من المسار القصير إلى مسار بديل.
    كما ذكرنا، سأكون سعيدًا بالتواصل معك مباشرةً. ستجد عنواني هنا:
    https://www.hayadan.org.il/astronomers-reach-new-frontiers-of-dark-matter-130112/#comment-332386

  163. إسرائيل،

    فيما يتعلق بصيغة فريدمان، فإن حجتي بسيطة. من المحتمل أن تكون الصيغة صحيحة من السكون إلى التسارع المحدود. بسرعات عالية سيكون عليك إضافة مكون إضافي إليها. كما أن صيغ نيوتن صحيحة حتى السرعات العالية عند إضافة المكون النسبي إليها.

    سقراط:

    إذا كنت ترى تناقضاً بين النظرية النسبية والانفجار الأعظم (وهو ما سأعترف به وأعترف بأنني غير مقتنع بوجوده) فما هي حجتك؟ أنه لم يكن هناك انفجار أم أن النظرية النسبية غير صحيحة؟ (أنا أفهم أنك ضد النسبية، ولكن أريد أن أتأكد من أننا على نفس الصفحة).

    بخصوص الأسئلة أعلاه:
    1) نعم، 100 سفينة فضائية ستكون قادرة على الاتفاق على الوقت. سيكونون قادرين على تحديد بدء إطلاق النار عند 4 كلفن صفر صفر. يوافق.

    2) ما هو ذو الصلة؟ 1905؟ الكون اللانهائي؟ لا تفهم.

    الأنفلونزا - يوجد لقاح جيد يتم توزيعه كل عام. يأخذ؟ الحقيقة هي أنني قرأت أنه لم يكن ناجحًا جدًا هذا العام. مشكلة الأنفلونزا هي أنها كل عام تتغير وهناك سلالات جديدة. ولم ينجحوا بعد في إيجاد لقاح عالمي لجميع السلالات، بل سلالة واحدة فقط كل عام. ومن حسن حظنا أن المرض يبدأ في الانتشار في شرق آسيا ثم يهاجر غرباً، حتى يتسنى لأوروبا وأميركا الوقت لإنتاج اللقاح المناسب كل عام. المشكلة هي أنه في بعض الأحيان تكون هناك مفاجآت واللقاح الذي أعدوه ليس مناسبًا تمامًا للسلالة التي اندلعت أو طرأ عليها تغيير على طول الطريق.
    الجدري هو اللقاح الوحيد في التاريخ الذي أدى بالفعل إلى الانقراض الكامل لأي مرض. وهناك أسباب كثيرة لذلك، منها أن الجدري ليس له مضيف سوى الإنسان، فإذا قمت بالتطعيم فقد تخلصت منه. علاوة على ذلك، فإن الفيروس لا يتحور بكفاءة. من السهل إنتاج لقاح (حصل عليه جينر من فيروس بقري مشابه). ولم تعود مثل هذه المصادفة منذ ذلك الحين.

    بالمناسبة اليوم كان إدوارد جينر سيدخل السجن، لذلك يعتبر بطلا.

  164. اليوبيل,

    "نحن لا نفهم على الإطلاق بنية الفوتون و/أو الإلكترون وكذلك الطريقة التي يتقدم بها عبر الفضاء."
    - عندما تقول "لسنا كذلك" عمن تتحدث؟

  165. صباح الخير.

    ويبدو أن الأفق بدأ ينقشع قليلاً. حتى أنني تمكنت من أكل برتقالة. دعونا نأمل أن تكون هذه هي نهاية قصة أشعل النار هذه.

    أشباح لاحظت أنه في ردك أجبت بـ 2 نعم. هل يمكن ان توضح

    ر.ح.

    أعتقد أننا نوضح التناقض مع إطالة الوقت هنا الآن. النسبية هي أكثر بكثير من مجرد تمدد الزمن. إذا تمكنت أنت، أو أي شخص آخر، من إظهار مكان خطئي، فسوف أشكرك وأغادر بسعادة. آمل أن تكون وجبتهم أيضًا.

    أعتقد أن الجدول الزمني ليس أكثر من مجرد محور. إذا كنت أتذكر بشكل صحيح، فقد قال هوكينز بالفعل أن الانفجار الكبير خلق الوقت، الذي لم يكن موجودا قبله. هل حقا لا ترى الفرق الأساسي بين الزمن في عام 1905، والذي امتد من ناقص ما لا نهاية إلى ما لا نهاية، وصورتنا للوقت اليوم؟

    لكنني لا أريد الترفيه عن جبهة أخرى. يمكننا بالتأكيد أن نرى ما إذا كنا سنتوصل إلى استنتاجات إذا واصلنا في الاتجاه الذي بدأناه.

    فيما يتعلق بتجربتك الفكرية - ليس لدي أي فكرة عما سيكون التأثير. وأنا أفهم المنطق، وتقليص حجمها. لكن حتى أينشتاين لم يتعامل مع الزمن على أنه نفس البعد الثالث.

    فيما يتعلق بصيغة فريدمان - أنا لا أفهم حجتك. هل تزعم أنه من المستحيل معرفة عدد الثواني التي مرت منذ الانفجار الأعظم بمجرد قياس درجة الحرارة؟ هل الآلة الحاسبة الصغيرة الموجودة أسفل الرابط خاطئة؟ ولذلك يعد هذا إخلاء مسؤولية جديدًا تمامًا، لكنني أعتقد أننا أغلقنا المشكلة منذ وقت طويل.

    دعونا نعود إلى هذه النقطة. إذا اتفقنا على ذلك، يمكننا المضي قدمًا:

    1. وفقًا لأينشتاين، فإن المسافرين المختلفين في الفضاء الذين يلتقون ببعضهم البعض ولكل منهم ساعة تظهر وقتًا مختلفًا، لن يتمكنوا بأي شكل من الأشكال من تحديد الوقت الحقيقي. إذا اختطفتك كائنات حنان الفضائية، وسرعتك في جميع أنحاء المجرة ثم تركتك في سفينة فضائية ضائعة في قلب الفضاء، فلن تتمكن من معرفة الوقت في هيئة الصحة بدبي بأي شكل من الأشكال، إلا إذا قمت بالاتصال به. يمكنه بالتأكيد أن يدور حول قزم أبيض.

    2. على حسب ما ناقشناه هنا فهو ممكن أيضاً. جهاز قياس الإشعاع، والكمبيوتر، ولنعد إلى المنزل.

    إذا وافقت على 2، يمكننا المضي قدما.

    M.R.S.T.

    إذا كنت لا تزال غير مقتنع، اذهب إلى

    http://www.youtube.com/watch?v=3A6ageOaS-E

    توجد تفسيرات بديلة، لكن على حد علمي، فهي ليست سائدة.

    يوفال

    بعد عبارة "غسالة الدماغ" و"كلمة القواد" وغيرها من التعبيرات الملونة، ما هو معنى "زرع ستائر دخان ملونة وعطرة" بالنسبة لي؟

    تريد أن تكون منزعجا - لا مشكلة. فقط من فضلك لا تبدأ مع إسرائيل مرة أخرى.
    أعدك رسميًا ألا أبدأ معك.
    وبالمناسبة - لا أذكر أنني تعرضت لضغوط من قبل هذا المقال أو أي مقال آخر، ولا أعتقد أنني تجاهلت أي موضوع، إلا إذا طُلب مني على وجه التحديد تجاهله. ولكن من أنا لأعرف؟

    الشيء الرئيسي هو أننا انتهينا.

  166. إسرائيل
    ليس لدي مشكلة في أن تضربني بقوة أكبر عشر مرات أو أي عدد من المرات التي تراها مناسبة. ليس لدي سوى مشكلة واحدة "صغيرة" معك: أنت تتجاهل الأشياء التي لا تتدفق في الاتجاه الذي تريده وتزرع ستائر دخان ملونة وعطرة عندما تشعر بالتوتر. الحمقى مثلك عديمة الفائدة بالنسبة لي.
    "في العجل وأثناء الصلب" هي أيضًا من القادية وتُترجم "سريعًا في أيامنا هذه".
    ولجميع إسرائيل قالوا آمين

  167. م.ر.س.ت
    يجب أن نكون حذرين ونحاول ألا نقفز إلى استنتاجات متسرعة. نحن لا نفهم تمامًا بنية الفوتون و/أو الإلكترون وأيضًا كيفية تقدمه عبر الفضاء. أحد التفسيرات المحتملة لنتائج التجربة المعنية هو أن العامل المُقاس لا يتحول بين حالة الموجة والجسيم، ولكنه بدلاً من ذلك يحافظ على كلتا الحالتين في وقت واحد في أي لحظة معينة.
    أود أن أتوسع في بنية وسلوك هذه الجسيمات وفقًا للنموذج الذي قمت بإنشائه:
    ivrit.yuval00@googlemail.com

  168. م.ر.س.ت

    اذهب إلى

    ftp://ftp.biu.ac.il/pub/physics/optics/optics12_YoungMichelson.pdf

    قم بالتمرير لأسفل حتى تصل إلى "الاختيار المؤجل"
    لاحظ الفقرة التالية:

    "ومع ذلك، فإن القرار بشأن تحريك الشاشة أم لا لم يتم اتخاذه إلا بعد عملية انتقال
    الإلكترون في الشقوق. لذلك يبدو كما لو أنه بعد أن مر الإلكترون بالفعل عبر الشقوق، فإنه لا يزال حرًا في "اتخاذ القرار".
    في وقت لاحق، سواء مر عبرهما كجسيم أو كموجة، وهذا وفقًا لقرارنا بشأن نوع القياس الذي يجب إجراؤه".

    لاحظ أنه تم إجراء العديد من التجارب، معظمها بالفوتونات، والتي أكدت هذه النتائج الغريبة. الاستنتاج المقبول هو أننا نستطيع التأثير على الماضي من المستقبل.

  169. ر.ح.

    يمكننا التوسع بعد أن أتقدم في السن للعائلة المريضة.
    وفي الواقع، من المؤسف والمخز أنه في القرن الحادي والعشرين، بعد أن تمكنت من إيجاد ترتيب مناسب لمرض الجدري، لم تنته بعد من الأنفلونزا.

    في هذه الأثناء - سقراط.

    1. هل تقبل أنه باستخدام الجهاز الذي أسميه "الساعة المؤقتة" إذا كانت 100 سفينة فضائية تمر ببعضها البعض في نفس الوقت بسرعات مختلفة، وكل سفينة فضائية بها كاميرا ذات دقة عالية، ثم يتم التقاط الصور المشتركة لجميع الساعات في هل ستظهر كل صورة من الصور المائة نفس الوقت على جميع الساعات الموجودة في المركبة الفضائية؟

    2. هل يمكنك القيام بنفس التمرين مع 3 سفن فضائية وفقًا للنظرية النسبية في عام 1905؟

    فيما يلي أوقات ساعات السيزيوم للمركبة الفضائية أينشتاين:

    و. 1905.
    ب. 17 تريليون و83 سنة.
    ثالث. 564، 876، 456 سنة، 165 يومًا، 14 ساعة و43 ثانية.

    اليوبيل.
    أود أن أعود إلى كوني "أشوت" إذا كان بإمكانك الالتزام بالحقائق وتجنب الإصابة الشخصية. وإلا فإنني سأتقبل مصيري المرير بكل تواضع، ولن نتحدث.

    "في العجل وأثناء الصلب" - أليس هذا من القاديش؟

    م.ر.س.ت

    هناك بالتأكيد شيء من هذا القبيل. سأحاول أن أجد لك الرابط المناسب اليوم، أو سأشرح نفسي. الآن أنا انتهيت قليلا.

    روبي.

    تدعي ابنتي، التي ستبدأ دراسة الدكتوراه هذا العام، أنه بصرف النظر عن الجانب الاجتماعي، فإن الجامعة تنتمي إلى القرن العشرين. أي محاضرة عادية تسمعها في الجامعة تجدها على الإنترنت، و20 محاضرة أفضل منها. جميع خطط الدروس والدعم والتمارين والمستشارين، كل شيء موجود. ابحث في جوجل عن أي موضوع تريده، وابحث عن محاضرة تعجبك على اليوتيوب، وعلى عكس الجامعة الحقيقية، يمكنك دائمًا تكرار كل نقطة عدة مرات كما تريد، أو إذا لم تفهم مفهومًا ما، أوقف القطار وارجع إلى وذلك بعد أن تم شرح كل شيء لك.

  170. على أبعاد الزمان والمكان
    تابع من: https://www.hayadan.org.il/astronomers-reach-new-frontiers-of-dark-matter-130112/#comment-330693
    كما نعلم من علم الحساب، فإن جوهر الأعداد "السالبة" هو تعبير عن عملية الطرح كرابطة فردية محلية (أو اختصار لـ "صفر ناقص رقم"). وبطريقة مماثلة، النفي هو أيضًا اتصال مفرد محلي (التعبير "ليس شيئًا" هو اختصار لـ "الحقيقة وليس شيئًا") وهذا هو الحال أيضًا مع جذر النفي. وكما نعلم من العمليات على الأعداد المركبة، فإن التطبيق المتكرر للجذر السالب له أيضًا دورة مربعة: الجذر السالب → النفي → نفي الجذر السالب → موجب (نفي السالب)، ومرة ​​أخرى: الجذر السالب → سالب → سالب من الجذر السالب → موجب والعياذ بالله. الشحنة عضو محايد، لأن تطبيق أي علاقة ذات مكان واحد عليه ينتج قيمة العلاقة ذات المكان الواحد المطبقة. في هذه الدورة يتم الحصول على أربع قيم مختلفة، إحداها محايدة. ومع ذلك، يتم تكرار نفس القيم الأربع بلا حدود في دورات منفصلة. نحن نسمي الفرق بين دورة واحدة والدورة التالية "الوقت". الزمن أحادي البعد وطويل إلى ما لا نهاية، وبالتالي فإن أبعاد الكون لا نهائية، ولكن بسبب الدورية يتم الحصول على أربعة أبعاد للفضاء، أحدها محايد أو "منحط". ولذلك فإن الفضاء (إذا تجاهلنا لا نهاية الزمن) هو ثلاثي الأبعاد.

  171. التعليق السابق كان مخطئا.
    يجب أن يكون الجزء بأكمله بعد سبت شالوم بين "هل ستتغير النسبية كما هي" و"فيما يتعلق بصيغة فريدمان"

  172. إسرائيل،

    وماذا في ذلك؟ نحن نعرف كل هذا بالفعل. افترض أينشتاين في عام 1905 أن الكون لانهائي في الزمن، وأننا على "شبكة" لا نهائية من المحور X المكاني، والمحور Y، والمحور Z ومحور الزمن. لقد جاء الانفجار الكبير وأظهر أن الجدول الزمني على الأقل له بداية. وهذا هو، ر0.
    من هذا تستنتج تناقضا مع النظرية النسبية؟

    تجربة فكرية: لنفترض أنهم وجدوا يومًا ما جدارًا يحجب الكون من وجهة نظر مكانية، وهو نوع من القشرة التي تحيط بكل شيء (كما هو الحال في القصص القديمة الجيدة في Fantasia 2000 ZL)، فهل ستتغير النسبية بهذه الطريقة؟

    وفيما يتعلق بصيغة فريدمان، فهي صيغة نظرية تم حسابها بناء على الكثير من الافتراضات، أحدها أن التبريد هو نفسه في كل مكان ولا يؤثر عليه إلا الزمن، فما تقوله دائري. وأنا متأكد أن الوقت وحده هو الذي يؤثر على التبريد لأن صيغة فريدمان التي تفترض ذلك تقول أنه لا يوجد عامل إضافي. حقا؟

    على أية حال، أتمنى لك الشفاء التام وألا ينتهي الأمر بكلبي مثل الكلبة لايكا أو كلب آل باسكرفيل أو ليد العجوز الذي كاد أن يُؤكل.

    يوفال، إجابة خاطئة. ليس لفائف. خط مستقيم.

    شاب شالوم

    1) إذا كان الأمر كذلك، كيف؟
    2) إذا لم يكن الأمر كذلك، فلماذا يتعارض الانفجار معه؟

    الآن دعونا نفكر في الجانب الآخر. سنقوم بنقل بداية الجدول الزمني إلى النهاية. لنفترض أننا اكتشفنا أن الكون لا يتوسع حقًا، بل ينكمش باتجاه الانهيار الكبير. هل يؤثر على العلاقة؟

  173. صفقة عظيمة، انفلونزا النجوم. أساسا هذه مسألة فقدان الشهية. إذا استمر هذا لمدة أسبوعين آخرين، فسوف أعود إلى الشكل الذي كنت عليه عندما كان عمري 20 عامًا.
    و ر.ح. أنفلونزا الكلاب لا تنطبق على كلابي. كلبي ليس مستضعفًا عاديًا، يمرض، أو يكبر، أو يموت. كلبي هو كلب أسطوري، بحجم كلب ليد أو لايكا أو كلب آل باسكرفيل. لا، سوف يكون متحمسًا لبعض الفيروسات الرديئة.

    أيها الشبح، حجزك واضح ولكنه بسيط. إذا كنت قد درست الفيزياء، فمن المحتمل أنك تتذكر أن الأنظمة دائمًا ما تكون مثالية: يتم تجاهل الاحتكاك والإشعاع والحمير الوحشية. الفرق ليس كبيرا، ويمكن وزنه لاحقا. لقد ضربت مثال المنطقة المنسية في الفضاء وذلك للتغلب على اعتراض ر.ه. على افتراض أن أي نظام لا يتسارع هو في الواقع في حالة سكون. باستثناء أنه من الممكن نظريًا بناء نموذج مختبري لتلك المنطقة، حيث سيتم استيفاء جميع الشروط التي وصفتها، فقط أصغر بكثير، وهو ما لا يغير النقطة.

    يوفال، أنت تدمر السعادة. لكن بما أنك أشرت، فهذا صحيح: الفقرة التي أحضرتها هي من مقالة أينشتاين الأصلية والمعروفة على نطاق واسع بالنظرية النسبية الخاصة:

    http://www.fourmilab.ch/etexts/einstein/specrel/www/

    لكن انتبه بشكل أساسي إلى الفقرة التالية من المقال:

    يجب أن نضع في اعتبارنا بعناية أن الوصف الرياضي من هذا النوع ليس له معنى مادي ما لم نكن واضحين تمامًا بشأن ما نفهمه من خلال "الزمن". علينا أن نأخذ في الاعتبار أن جميع أحكامنا التي يلعب فيها الزمن دورًا هي دائمًا أحكام على أحداث متزامنة.

    وتقول لي: أليس كذلك عام 1905؟ لصورة عالمية للكون اللانهائي والأبدي، حيث كانت الأشياء دائمًا وستظل كذلك دائمًا. حيث يمكنك التفكير بهدوء في أحداث وقعت منذ تريليون تريليون سنة مضت، كما اعتقد بوانكاريه عندما أثبت قابلية عكس إنتروبيا الكون في الوقت المناسب، بينما أثبت طالب التخنيون أنه غير قابل للعكس حتى بالنسبة لنظام أصغر بكثير من الكون. مقدار عدد أفوجادرو خلال حياة الكون.

    لأنه وفقًا للنظرة العالمية لعام 1905، لا يمكنك حقًا معرفة الوقت عند نقطة لا ترتبط بها بشكل مباشر. لا يوجد وقت مطلق، ولكل مساح وقته الخاص. ولذلك إذا تابعت قراءة المقال يقترح أينشتاين مزامنة الزمن بين نقطتين باستخدام الأشعة الضوئية التي تكون سرعتها واحدة لكل جهاز قياس، ومن هذا يستنتج إطالة الأوقات وقصر المسافات.

    ومع ذلك، لدينا ميزة لم تكن لدى أينشتاين في عام 1905: وفقًا لنظرية الانفجار، ليست هناك حاجة لمزامنة أوقات الأنظمة على الإطلاق: فهي متزامنة بشكل طبيعي، وكل ما يتعين علينا القيام به هو قياسها، كما قلت. اقترح استخدام ساعات درجة الحرارة.

    وفقا لأينشتاين، عندما يلتقي مائة توأم تم فصلهم في الماضي مرة ثانية، لا يوجد شيء اسمه "الوقت الحقيقي" لأحدهم. الجميع على حق. وهذا يختلف بحسب نظرية الانفجار الأعظم. ومن المؤكد أنه من الممكن معرفة ما هو الوقت الحقيقي، وحتى قياسه. حجز R.H. فيما يتعلق بالحركة النسبية للأنظمة، يمكن صدها بسهولة نسبيًا من خلال النموذج الجديد للساعات المؤقتة، حيث يتم ترجيح السرعة النسبية في مخرجات الوقت. وينطبق هذا أيضًا على حجز مايكل الغائب بشأن "العودة في الوقت المناسب" للساعات المؤقتة.

    وفيما يتعلق بسؤال ر.ح. بخصوص عامل التبريد - الجواب هو صيغة فريدمان والآلة الحاسبة الصغيرة المرفقة في أسفل الرابط الذي قدمته لك. إذا كان هناك عامل آخر، فإن الصيغة لن تعمل.

    وسوف نأخذ استراحة لاستيعاب الأفكار والنقد. إذا كنت قد تحدثت كثيرًا من الهراء، فإن عذري هو مقياس الحرارة الذي يقرأ 104 فهرنهايت.

  174. آر إتش،
    سأخذ صندوقًا على شكل أسطوانة وألف الصفحة خارجه قطريًا بحيث يكون كل سطر (ما عدا الأول) استمرارًا للسطر الذي يسبقه. على الرغم من أن الخط الذي أقوم بتوصيل جميع النقاط به سيكون في الواقع حلزونيًا، إلا أنه من وجهة نظر هذا المستوى بالذات سيكون مستقيمًا.
    لا أعرف الكثير عن جاك السفاح. لقد شاهدت العديد من الأفلام التي استخدمت شخصيته (أحدها أخذه إلى الأمام في الوقت المناسب لسرقة الشابات في القرن العشرين: "مرة تلو الأخرى" عام 1979). إن ذكر قاتل رئيس الوزراء في سياق بنحاس مثير للاهتمام: أولئك الذين يعجبون بنحاس والذين يعجبون بهذا القاتل لديهم العديد من الخطوط الأيديولوجية المشتركة.
    ما بعد النصي. عندما اغتيل رابين ماتت البلاد من أجلي.

  175. يوفال، هل لديك أي توصيات حيث يمكنني الحصول على دورات أكاديمية حول هذه المواضيع؟ أنا مهندس بالتعليم. أنا مهتم جدًا بعلم الفلك والثقوب السوداء وما إلى ذلك.

  176. اليوبيل,

    اصنع لي معروفًا، فكر خارج الصندوق. يمكنك أيضًا القول إن جاك السفاح أو ييجال أمير فكرا خارج الصندوق.

    التفكير خارج الصندوق هو كيفية ربط 9 نقاط مرسومة على صفحة على شكل 3x3 بخط مستقيم واحد.

  177. آر إتش،
    فكر بينكاس خارج الصندوق. في بعض الأحيان تحتاج إلى أشخاص مثل هذا. وبينما كان موسى وكل جماعة بني إسرائيل واقفين عند باب المسكن يبكون، قام فينحاس وفعل شيئًا. إنه مثل، على النقيض من ذلك، ما فعله الإسكندر الأكبر في مستنقع كولومبوس... عفوا…. آسف. كنت مشوشه.

  178. روبي
    صحيح. نتوقع اكتشاف "حديقة حيوانات" كبيرة من الجسيمات. أنت "تخطئ" في التجارب الفكرية، مثلنا جميعًا هنا. مرحبا في النادي. ما تقوله، وهو أن الفرق بين السرعات الحقيقية والسرعات النسبية يترجم إلى طاقة زائدة (أعتذر عن صياغتي السيئة) يمكن بالتأكيد أن يكون بمثابة أطروحة لطيفة لدراسة مثيرة للاهتمام، ولن أتفاجأ إذا كان شخص ما قد فكر بالفعل منه قبلك. أدخلت النظرية النسبية العديد من المفارقات في حياتنا، ولكن من خلال وضع الأرقام بشكل صحيح في الصيغ المناسبة، يمكن التوفيق بين كل شيء. يبدو لي أنني لست أقوى منك في الرياضيات. من فضلك ضع البيانات في الأماكن الصحيحة وأخبرني والقراء الآخرين بما توصلت إليه.

  179. اليوبيل,

    وفي ضوء الطريقة التي قام بها هارون بتعليم حفيده بنحاس، الذي أصبح رمزا للتعصب والقتل الديني، فهو ليس نموذجي تماما.

    http://he.wikipedia.org/wiki/כהני_פינחס

    عدا عن أن شخصية هارون نفسه لا شيء. بعد كل شيء، هو الذي قاد خطيئة العجل وفي النهاية بسبب صلاته في الإدارة تم تبريره بالكامل بينما تم ذبح 3000 آخرين. لا تأتوني إما بشموئيل الذي ذبح أجاج بحماس، أو إيليا الذي ذبح كهنة البعل، أو إليشع الذي قتل أطفالاً ضحكوا عليه لصلعته.
    إذا أصررت فربما شاؤول أو شمشون بلين البريء.

  180. يوفال، شكرًا لك على الإجابة، لكننا ما زلنا نتوقع من الاصطدام العثور على جسيمات فيزيائية مكونة من بوزون هيغز الشهير.
    ما أريد قوله هو أن ناتج التصادم هو الطاقة التي تكسر البروتونات المتصادمة بكثافة MC**2 لكل بروتون بينما كانت سرعة كل بروتون نصف C وكانت الشدة MC**2/4 لكل بروتون . هذا يعني أنه ربما لا "يرى" البروتون المتحرك بسرعة C اصطدام البروتون بسرعة 2C ولكنه يشعر بحجم الاصطدام كما لو أن متجهات السرعة تتراكم.

  181. آر إتش،
    شكرا لمجهوداتكم 🙂
    وهناك قصص عن هرون الكاهن الذي أحب السلام وسعى للسلام وصنع السلام بين البشر. إذا تبرعت بعينة من الحمض النووي، أراهن أنك أحد نسله 😛

  182. م.ر.س.ت
    ليس لديك ما تسعى إليه. لا يوجد مثل هذا التفسير. المناقشات التي لدينا هنا هي في الأساس تجارب فكرية. تشير مثل هذه المناقشات عادة إلى عدد محدود من الحقائق أو الاستنتاجات وتتجاهل كل شيء آخر. وبهذه الطريقة، من الممكن التوصل إلى استنتاجات مثيرة للاهتمام ومثيرة للدهشة، ولكن كل شيء هو فقط من الناحية النظرية وعادة ما يكون بعيد المنال.

  183. روبي
    يفتح سؤالك الباب أمام نقاش مثير للاهتمام حول ما يتم تسريعه فعليًا في المسرعات. في الواقع، البروتون المتسارع هو عامل مهمل. يتم استخدامه فقط كـ "مركبة" صغيرة تنتقل عليها كمية كبيرة من الطاقة. إذا قبلنا صيغة العلاقة بين الكتلة والطاقة (E=MC^2) فسيبدو أن ما ينتقل إلى هناك هو طاقة بشكل أساسي. أحد استنتاجات النظرية النسبية هو أن الجسم الذي يتحرك بسرعة ينقبض. وفي سرعة الضوء، فإن انكماش الجسم سيجعل حجمه صفراً. إذا تسارع البروتون إلى سرعة الضوء، فإن كل ما سينتقل في النظام سيكون الضوء فقط.
    عندما يصطدم بروتونان متسارعان، يكون ناتج التصادم بينهما ضئيلًا مقارنة بمنتج التصادم بين حزمتي الطاقة التي أدت إلى تسريعهما. الجواب على سؤالك، لماذا تقوم بتسريع كلا البروتونين بدلا من تسريع أحدهما فقط وترك الآخر ثابتا، هو أن الغرض من التجربة هو إحداث تصادم بين شعاعين من الطاقة.

  184. أعاني من حمى من المحتمل أن تستمر لبضع ساعات. أود منك قراءة الفقرة التالية، ومعرفة ما إذا كانت تتناسب مع فكرة الوقت المطلق، والذي يمكن حتى قياسه باستخدام الجهاز الذي أسميه "الساعة المؤقتة" أو حتى اقتراحك R.H. ابتعاد المجرات أو أي عامل خارجي آخر. الشيء الرئيسي هو أنه يمكننا مزامنة الساعات دون أي اتصال بينهما.

    § 1. تعريف التزامن

    دعونا نأخذ نظامًا من الإحداثيات تكون فيه معادلات الميكانيكا النيوتونية جيدة.2 لكي نجعل عرضنا أكثر دقة ولنميز نظام الإحداثيات هذا لفظيًا عن الأنظمة الأخرى التي سيتم تقديمها فيما بعد، نسميه "نظام ثابت."

    إذا كانت نقطة مادية ساكنة نسبيًا بالنسبة لنظام الإحداثيات هذا، فيمكن تحديد موقعها نسبيًا باستخدام معايير قياس صارمة وطرق الهندسة الإقليدية، ويمكن التعبير عنها بالإحداثيات الديكارتية.

    إذا أردنا وصف حركة نقطة مادية، فإننا نعطي قيم إحداثياتها كدوال للزمن. الآن يجب أن نضع في اعتبارنا بعناية أن الوصف الرياضي من هذا النوع ليس له معنى مادي ما لم نكن واضحين تمامًا بشأن ما نفهمه من خلال "الزمن". علينا أن نأخذ في الاعتبار أن جميع أحكامنا التي يلعب فيها الزمن دورًا هي دائمًا أحكام على أحداث متزامنة. على سبيل المثال، إذا قلت: "يصل هذا القطار إلى هنا في الساعة السابعة"، أعني شيئًا من هذا القبيل: "إن الإشارة إلى العقرب الصغير في ساعتي إلى السابعة ووصول القطار هما حدثان متزامنان."

    قد يبدو من الممكن التغلب على جميع الصعوبات التي تواجه تعريف "الوقت" عن طريق استبدال "موضع العقرب الصغير لساعتي" بكلمة "الوقت". وفي الواقع يكون هذا التعريف مرضيًا عندما يتعلق الأمر بتحديد وقت حصرًا للمكان الذي توجد فيه الساعة؛ ولكن لم يعد الأمر مُرضيًا عندما يتعين علينا ربط سلاسل زمنية من الأحداث التي تحدث في أماكن مختلفة، أو -ما يصل إلى نفس الشيء- تقييم أوقات الأحداث التي تحدث في أماكن بعيدة عن الساعة.

    يمكننا بالطبع أن نكتفي بقيم زمنية يحددها مراقب متمركز مع الساعة عند أصل الإحداثيات، وينسق المواضع المقابلة للعقارب مع الإشارات الضوئية التي يصدرها كل حدث ليتم توقيتها، والوصول إليه من خلال الفضاء الفارغ. لكن من عيب هذا التنسيق أنه ليس مستقلا عن وجهة نظر الراصد مع الساعة أو الساعة، كما نعلم من التجربة. لقد وصلنا إلى قرار أكثر عملية على طول الخط الفكري التالي.

    إذا كانت هناك ساعة عند النقطة A من الفضاء، فيمكن للمراقب في النقطة A تحديد القيم الزمنية للأحداث في القرب المباشر من النقطة A من خلال إيجاد مواضع العقارب المتزامنة مع هذه الأحداث. إذا كانت هناك عند النقطة B من الفضاء ساعة أخرى تشبه من جميع النواحي الساعة عند A، فمن الممكن لراصد عند النقطة B أن يحدد القيم الزمنية للأحداث في الجوار المباشر للنقطة B. لكن ذلك غير ممكن بدون مزيد من الافتراض للمقارنة، فيما يتعلق بالوقت، حدث في A مع حدث في B. لقد حددنا حتى الآن فقط "الوقت A" و"الوقت B". لم نحدد "زمنًا" مشتركًا لـ A وB، لأنه لا يمكن تعريف الأخير على الإطلاق إلا إذا أثبتنا بالتعريف أن "الزمن" الذي يتطلبه الضوء للانتقال من A إلى B يساوي "الزمن" الذي يحتاجه للانتقال. من B إلى A. دع شعاع الضوء يبدأ عند "الزمن A" $t_{\rm A}$ من A باتجاه B، دعه ينعكس عند "الزمن B" $t_{\rm B}$ عند B في اتجاه A، والوصول مرة أخرى إلى A في "الوقت" $t'_{\rm A}$.

    وفقًا للتعريف، تتم مزامنة الساعتين إذا
    \begin{displaymath}t_{\rm B}-t_{\rm A}=t'_{\rm A}-t_{\rm B}. \النهاية{عرض الرياضيات}

    نحن نفترض أن هذا التعريف للتزامن خالي من التناقضات، وممكن لأي عدد من النقاط؛ وأن العلاقات التالية صالحة عالميًا: -

    إذا كانت الساعة عند B تتزامن مع الساعة عند A، فإن الساعة عند A تتزامن مع الساعة عند B.
    إذا كانت الساعة عند A تتزامن مع الساعة عند B وأيضًا مع الساعة عند C، فإن الساعتين عند B وC تتزامنان أيضًا مع بعضهما البعض.

    وهكذا، بمساعدة بعض التجارب الفيزيائية الخيالية، حددنا ما يجب أن نفهمه من الساعات الثابتة المتزامنة الموجودة في أماكن مختلفة، ومن الواضح أننا حصلنا على تعريف "المتزامن" أو "المتزامن" و"الوقت". "الوقت" لحدث ما هو الذي يتم تحديده بشكل متزامن مع الحدث بواسطة ساعة ثابتة موجودة في مكان الحدث، وتكون هذه الساعة متزامنة، بل ومتزامنة بالفعل لجميع تحديدات الوقت، مع ساعة ثابتة محددة.

    وبالاتفاق مع الخبرة، فإننا نتحمل أيضًا الكمية
    \begin{displaymath}\frac{2{\rm AB}}{t'_A-t_A}=c, \end{displaymath}

    ليكون ثابتًا عالميًا - سرعة الضوء في الفضاء الفارغ.

    من الضروري أن يتم تحديد الوقت عن طريق الساعات الثابتة في النظام الثابت، والوقت المحدد الآن مناسب للنظام الثابت الذي نسميه "وقت النظام الثابت".

  185. أنا جديد نسبيًا هنا ولم أفهم كيفية العودة إلى الماضي وتغييره. قال إسرائيل شابيرا أمس في الساعة 3:03 أنه يمكن القيام بذلك بمساعدة ميكانيكا الكم، لكنني لم أتمكن من العثور على تفسير.

  186. إسرائيل،

    "مركبة فضائية، في قلب الظلام المصري، قادرة على معرفة الوقت بناءً على قياسات مستقلة قامت بها. سيتم عرض هذا الوقت على وجه ساعة سفينة الفضاء بعدد الثواني التي مرت منذ الانفجار الأعظم." ==>

    نعم. بشرط ألا تبدأ في الانطلاق بسرعات أعلى من الحد المسموح به. إذا بدأت في التحرك، فسيتعين عليها استخدام تعديل لصيغة فريدمان الخاصة بك والتي تمت إضافة مكون نسبي إليها أيضًا يشير إلى التغير في الوقت وفقًا للسرعة. وطالما أنها تقع على عاتقها فهي مكون لا يكاد يذكر والصيغة قوية. يوافق؟

    "إذا أرسلت سفينة الفضاء هذا الرقم عبر الراديو، واستقبلته سفينة فضائية أخرى على مسافة ساعة ضوئية من سفينة الفضاء الأصلية، فسوف يتطابق تمامًا مع الرقم المعروض على وجه الساعة لسفينة الفضاء الثانية، والتي وصلت إليها بواسطة نفس مسار سفينة الفضاء الأولى، بعد تعويض الفارق الزمني بينهما." ==>

    مرة أخرى، بشرط ألا يتحرك أي منهم. فإذا تحرك أحدهما، يجب أن يؤخذ ذلك في الاعتبار عند حساب الوقت حسب معدل تغير درجة الحرارة.

    1. الموافقة على الشروط والأحكام التي ذكرتها.

    2. لا. ولكن بعد ذلك كان من الممكن القياس بشيء آخر. المعدل الذي تنحسر به المجرات، أو المعدل الذي يتلاشى به ضوء النجم القديم، أو أي شيء آخر يتغير بمرور الوقت. صحيح أنه لن يستمر أي قياس إلى الأبد، لكن تبريد إشعاعك لن يدوم إلى الأبد أيضًا، ولكنه سينخفض ​​بشكل مقارب إلى الصفر المطلق حتى لا يعد من الممكن قياسه.

    3. لم تجبني ما هي ثقتك بأن العامل الوحيد المؤثر على التبريد هو الوقت؟

  187. ر.ح.
    وفقا لصيغة فريدمان، يرتبط الوقت بدرجة الحرارة في صيغة دالة مستمرة. حتى مع وجود معلمات إضافية، في الحالة التي نناقشها، لا يتم عكسها.

    وبما أننا توصلنا إلى اتفاق بشأن 2، أود توضيح هذه النقطة قبل أن نواصل:

    مركبة فضائية، في قلب الظلام المصري، قادرة على معرفة الوقت بناءً على قياسات مستقلة قامت بها. سيتم عرض هذا الوقت على وجه ساعة المركبة الفضائية بعدد الثواني التي مرت منذ الانفجار الأعظم.

    إذا أرسلت المركبة الفضائية هذا الرقم عبر الراديو، واستقبلته مركبة فضائية أخرى على مسافة ساعة ضوئية من المركبة الفضائية الأصلية، فإنه سيطابق تمامًا الرقم المعروض على وجه الساعة للمركبة الفضائية الثانية، والذي وصلت إليه عن طريق نفس مسار المركبة الفضائية الأولى، بعد تعويض الفارق الزمني بينهما.

    1. هل توافق؟

    2. هل ستنجح الحيلة في كون متناحٍ وأبدي حيث تكون درجة الحرارة ثابتة دائمًا؟

  188. إسرائيل،
    1) لا أعرف، ربما يمكننا التفكير في لمعان آخر غير درجات الحرارة الخاصة بك. أي شيء خارج سفينة الفضاء يتغير بمرور الوقت سيساعدهم. ذرات مشعة ذات نصف عمر مرتفع تتجمع في الفضاء؟ تركيز الغبار بين النجوم؟. أفترض أنه حتى في البراري البعيدة، سيكونون قادرين على رؤية النجوم الزائفة في المسافة وإجراء بعض القياسات.

    2) نعم.

    قبل أن تبدأ في الجدال معي حول الإجابات في القسم 1. "قلت لا شيء! أَزِيز! لا يوجد شيء، لا يوجد غبار ولا توجد رشاشات" أريدك أن تعلم أن الإجابة الدقيقة ليست مثيرة للاهتمام وعلى الأرجح ما كتبته هناك غير صحيح، ولكن بالتأكيد يمكنك التفكير في شيء ما. وحتى لو لم يكن الأمر كذلك، فإن هدفي كله هو أن أوضح لك أن مقياس الحرارة الخاص بك، حتى لو كان ذو لمعان لطيف وربما مفيدًا في عصر الرحلات الجوية المجرية الكبيرة الذي يثير اهتمامنا، ليس مميزًا. إنهم يقيسون معلمة مثيرة للاهتمام خارج المركبة الفضائية والتي يمكن من خلالها حساب الوقت. صفقة كبيرة. هل هذا يعني أن هناك تناقضا في النظرية النسبية؟
    أيضًا، لكي أكون تافهًا، هل أنت متأكد من أن درجة الحرارة تتحلل مع مرور الوقت فقط؟ ربما هناك معلمات أخرى؟ الكثافة النجمية؟ الجاذبية حولها؟ ربما لا يتم توزيع الإشعاع بشكل موحد حقًا؟

  189. إن كلمة "تقريبًا" هذه مبدئية للغاية، كما رأيت في النصف الأول من الإجابة. إذا استثمرت كمية X من الطاقة للوصول إلى سرعة نسبية قدرها 0.99C بين البروتونات، فإذا استثمرت 1000X أخرى، فسوف تصل إلى سرعة نسبية تبلغ 0.999999C على سبيل المثال. إذا قمت بحسابات الطاقة بشكل صحيح وفقًا لمعادلات لورنتز، فسوف ترى أنه في الفارق الذي يبدو صغيرًا وهو 0.009999C، يتم دفن طاقة ضخمة تبلغ 1000X، أي أكثر بـ 1000 مرة من كمية الطاقة المطلوبة لإيصال البروتونات إلى مستوى قريب السرعة 0.99 درجة مئوية.
    وللوصول إلى C نفسها، لن تحتاج إلى أقل من طاقة لا نهائية.
    ربما تفهم لماذا اهتزت القشرة الأرضية عندما بدا أن بعض النيوترينو صفيق تجرأ على تجاوز سرعة الضوء؟ مستحيل يا عليبا ديسوت.

  190. إسرائيل، أعود إلى الفيزياء الأساسية: Ft=mV القوة والزخم. يجب أن يكون V هو المجموع المتجه للهيئتين (البروتونات).
    وفقًا لما كتبته، بغض النظر عن السرعة التي تزيد عن نصف C لكل بروتون، فإن مجموع المتجه سيكون دائمًا تقريبًا C، وبالتالي ستكون النتيجة Ft هي نفسها دائمًا.
    إذا كان الأمر كذلك، فهل يكفي تسريعها إلى نصف درجة مئوية ولا فائدة من استثمار طاقة إضافية؟

  191. روبي

    هناك فرق كبير جدًا بين طاقة البروتون عند 0.99 درجة مئوية وطاقته عند 0.999999999 درجة مئوية يمكن أن تصل بهدوء إلى آلاف الأمتار.
    الشق الثاني من السؤال غير واضح بالنسبة لي.

  192. إسرائيل، شكرًا على إجابتك، لكن ليس من الواضح بالنسبة لي سبب بذل الكثير من الجهد في تسريع البروتون إلى سرعة الضوء تقريبًا بحيث تظل السرعة النسبية في النهاية قريبة من سرعة الضوء.
    فيما يتعلق بطاقة الاصطدام، لم تجب عما إذا كانت ستكون نفسها عند سرعة نصف C إلى C لكل بروتون.

  193. ر.ح.
    يبدو لي أن أي جهاز كمبيوتر عادي سيكون قادرًا على حساب الوقت الذي انقضى من الانفجار إذا كانت سفينة الفضاء في حالة سكون. ستقوم أجهزة كمبيوتر Macintosh بذلك بشكل أسرع. وهذا ينطبق حتى لو كان كل ما نقيسه فعليًا هو طيف إشعاع الخلفية. ببساطة، يمكنك قياس درجة حرارة الإشعاع، وقياس سرعتك بالنسبة للإشعاع، والإزاحة. صيغة فريدمان، وهنا هو الوقت المحدد.

    لكن لا يهم. لنفترض أنه في منطقة معينة من الفضاء، في السهوب الأبدية التي تمتد في منطقة الشفق بين المحدود واللانهائي، لا توجد نجوم ولا سديم ولا حمير وحشية. فقط الظلام فوق الهاوية. لا شئ. ندى

    لنفترض أن سفينتين فضائيتين تبعدان مسافة ساعة ضوئية عن بعضهما البعض، وقد وقعا في نفس المنطقة الملعونة. سرعتهم هي 0 بالنسبة لبعضهم البعض وأيضا 0 وفقا لمعاييرك الصارمة تجاه أي نظام خارجي. (أي نظام؟ لا يوجد شيء هناك!).

    يقوم الطاقم في كل سفينة فضائية بسحب ساعة الوقواق الجديدة اللامعة من العبوة. الساعات، وفقا لتعليمات الشركة المصنعة، تظهر الوقت 0 وتبدأ العمل فقط عندما تضغط على عود الأكل أعلاه.

    يسأل سقراط:

    1. هل ستتمكن المركبة الفضائية من مزامنة ساعاتها بحيث ترى نفس الوقت دون استخدام أي وسيلة اتصال مثل الأشعة الضوئية أو الراديو؟

    2. هل سيكونون قادرين على القيام بذلك باستخدام الساعات المؤقتة؟

  194. روبي، هناك تحويلات لورنتز، والتي من خلالها يمكنك حساب السرعة النسبية للأجسام التي تقترب من سرعة الضوء.

    خذ مثالا متطرفا: المركبة الفضائية أ تطير بالنسبة للأرض بسرعة 0.9 درجة مئوية. تطير المركبة الفضائية B بسرعة 0.9C بالنسبة إلى A، في نفس الاتجاه. C بنفس السرعة بالنسبة إلى B، وهكذا حتى سفينة الفضاء T. ما هي سرعة T بالنسبة للأرض؟
    الجواب هو أن أقل بقليل من C.

    لذا فإن السرعة النسبية للبروتونات ستكون بين حوالي 0.7 درجة مئوية، إلى درجة مئوية تقريبًا، اعتمادًا على سرعتها.

    السبب وراء الحد الأقصى لسرعة الضوء منطقي بشكل أساسي. ويمكن إثبات أنه إذا خضع الجسم لعملية MHA، فإن النتيجة سوف تسبق السبب. ولولا معادلات النسبية لرأى كل جسم الآخر يتحرك بسرعة هي عبارة عن مجموع متجه بسيط، كما هو الحال مع نيوتن، وبالتالي أعلى من الأول.

  195. إسرائيل،

    0. http://www.alfavet.co.il/kennel_cough.html

    1. لا. لقد تحدثنا عن ذلك، سأتخذ من حرارتك ذريعة لمن نسيته. إذا كانت المركبة الفضائية تتحرك بسرعة قريبة من سرعة الضوء فسوف يتم الإبلاغ عن وقت قصير. إذا كان بنفس سرعة كوكبنا فسوف يبلغ 13.7 مليار سنة. وهذا يعني أن التقرير سيعتمد على السرعة. كيف تعمل هذه الساعة؟ يقيس درجة الحرارة الحالية، ويفترض درجة الحرارة t0. وبعد ذلك، وفقًا لمعدل التبريد الذي تم قياسه مسبقًا في النظام (وهو بالضبط العنصر المعتمد على السرعة!) ستقوم الساعة بحساب الوقت.

    2. نعم. يمكنهم أن يقرروا أنه عندما T=X سيجتمعون جميعًا. الأول سيصل بعد ساعة له والثاني بعد يومين (له) والثالث بعد عامين من التجوال (مرة أخرى له) ولكنهم سيجتمعون جميعا في نفس الوقت عندما T=X. ستمر 50 عامًا على SGH في النجم القريب، ولكن عندما يرى أن T=X، سينضم أيضًا.

    ومرة أخرى أقول، لا يوجد شيء مميز في درجة الحرارة هذه. ويمكنهم أيضًا أن يقرروا الالتقاء عندما تكون المسافة بين المجرتين A وB، اللتين تبتعدان عن بعضهما البعض، هي Y. وحتى في ذلك الوقت، سيسافر كل منهما بسرعته الخاصة ويمزق الفضاء بأكمله، ولكن عندما يكون الاثنان إذا وصلت المجرات إلى المسافة المطلوبة، فسوف تلتقي جميعها.

    3. لا.

    4. افعل ما تريد. أنا لست منتقيًا مجانيًا ولست روضة أطفال متطوعة.

  196. إسرائيل، لقد فهمت بشكل صحيح، اصطدام بروتون مع بروتون عندما يتسارع كل منهما في الاتجاه المعاكس بسرعة الضوء (تقريبا).
    إذا لم يكن هناك حد لسرعة الضوء، فسيرى KA أن السرعة النسبية تبلغ ضعف 2، ولكن بسبب حاجز سرعة الضوء، هل تدعي ذلك؟
    ماذا سيرى الله لو كانت سرعة كل بروتون نصف سرعة الضوء، وهي نفسها أيضًا؟
    ما هو زخم الاصطدام؟ إذا كانت سرعة كل بروتون بين نصف سرعة الضوء؟

  197. ر.ح.
    قضيت بضع لحظات هادئة بين نوبة حمى وأخرى. سؤال كشخص يفهم الفيروسات: كيف مرضت العائلة كلها باستثناء أطفالي؟ ربما هذا هو الاتجاه؟

    يؤسفني أنني غير قادر على إقناعك بتكافؤ جميع أنظمة القصور الذاتي، وأنه لا يوجد شيء اسمه نظام "متحرك" حقًا. لكن من الممكن الاستمرار حتى بدون ذلك، بمساعدة سقراط المسن.

    1. هل توافق على أن المركبة الفضائية الموجودة في الفضاء الفارغ ستكون قادرة على معرفة الوقت الدقيق الذي انقضى منذ الانفجار باستخدام الجهاز الذي أسميه "الساعة المؤقتة"؟

    2. باستخدام نفس المنشأة، هل يمكن لعدة سفن فضائية، بعد تمزيق المجرة بأكملها بتسارعات رهيبة، في كل الاتجاهات، وغير متزامنة تمامًا مع ساعات السيزيوم الخاصة بها، تنسيق هجوم مفاجئ مشترك على كوكب ما، في وقت لا يمكنهم فيه ذلك؟ وقد تمكنت من دون ساعات درجة الحرارة؟

    3. هل توافق على أنه وفقا للنسبية فإنه من المستحيل تنسيق مثل هذا الهجوم؟

    وفيما يتعلق يوفال - هل رأيت كم مرة طلبت منه النزول مني، عرضت عليه "التهدية" ووقعت عليه اتفاق سلام، ماذا يمكنني أن أفعل إذا استمر؟ أود بشدة أن تسير الأمور بسلاسة، لكنني لا أعتقد أنه يستطيع ذلك.

    روبي - الاصطدام بماذا؟ إذا فهمتك بشكل صحيح، فإن الاصطدام مع بعضنا البعض. الجواب حسب فهمي هو أقل بقليل من سرعة الضوء.

  198. سؤال للمنتدى الموقر
    في مسرع الجسيمات LHC في CERN، يتم تسريع البروتونات إلى سرعة 99.9999991 في المئة من سرعة الضوء، بعضها في اتجاه عقارب الساعة وبعضها عكس اتجاه عقارب الساعة.
    والسؤال هو ما هي السرعة التي يراها كل بروتون قبل الاصطدام وأيضا ما هو الزخم الناتج عن الاصطدام، أي سرعة الضوء أو ما يقرب من ضعف سرعة الضوء.

  199. إسرائيل،
    كيف يكون لدي الوقت لفعل أي شيء إذا كنت أتعامل طوال اليوم مع أشعة الضوء غير المرئية بسرعات عالية؟
    بالمناسبة، أنا أيضًا كنت مصابًا بالأنفلونزا حتى قبل يومين.

  200. لقد أصبت بسلالة عنيفة من أنفلونزا النجوم. سنواصل لاحقا.
    ر.ه.، ألم يكن من المفترض أن تجدي لقاحاً للأنفلونزا؟؟!؟

  201. إسرائيل،
    2) لحسن الحظ، فإن تصوراتي لأفكارك مدعومة بالفعل من قبل التيار الرئيسي ولهذا السبب أشعر بأنني على أرض صلبة، وإلا فلن أجادل كثيرًا حول موضوع ليس لدي سيطرة عليه بنسبة 100٪. سيتم إخبار جلالتك أنني تعلمت الكثير من هذه المناقشة. بالطبع، من الممكن أن أكون أنا ومعظم علماء الفيزياء في العالم مخطئين، ولكن سيتعين عليك العمل بجد وإثبات مثل هذا الخطأ في التجارب على الأضواء والسحب.

    – أحب أن أسمع أفكارك عن الأزمنة وتأثيرات الماضي والمستقبل وارتباطها بفكرة أشعتك الشفافة.

    3) أنا أتفق مع كل كلمة وبالتالي لا يوجد أي تناقض كما يتفق معي السادة الذين ذكرتهم. وبالمناسبة، لاحظ أنه لم يزعم أحد منهم أن النظام المرجعي للنظام المدروس مؤثر كما زعمت.

    6) لكنك تجيب على سؤالك بكتابتك الخاصة. لاحظ أنك كتبت "عربة قطار مختومة". ما هي الذرة؟ منفصل عن أي نظام مرجعي. من المحتمل أيضًا أن توافق على أنه إذا لم تكن العربة مغلقة، فهناك طريقة تعتمد على إطار مرجعي خارجي على شكل شجرة لتحديد ما إذا كنت تتحرك أم لا. وبالمثل جاك/جيل الذي يمكنه النظر إلى أندروميدا/بيتلجوس أو درجة الحرارة خارج سفينة الفضاء الخاصة به.

    7) هنا أيضًا فاتتك وجهة نظري. أنا لست نظام عدالة، ولا إحساس طبيعي بالعدالة، ولا يهمني من بدأه ومن أعاده. أرى نفسي صديقًا لكما ويسعدني أن أتواصل معكما، لكن يزعجني أنكما تتصرفان كطفلين. في الجيش كنا نأخذك إلى خيمة دورية في تسليم لمدة ليلتين، وستشعر بالوحدة.

    8) يوجد هنا أيضًا بحر (أكثر سوادًا من سواد) والتزلج (لا يوجد ثلوج هذا العام...) لذا لا تتكبر.

    9) أوافق على دعوة ميريست، ولست بحاجة إلى موافقتي. بالمناسبة، يوفال ليس متنمرا، لقد أراد فقط إشارة واقعية لكلماته.

  202. إسرائيل، من فضلك تجاهل يوفال المتنمر ولا تنتظر موافقة راه. استمر في فكرتك لأنها رائعة

  203. ر.ح.
    منتظر

    يوفال
    إن موهبتك في العيش في الواقع الافتراضي تصل إلى آفاق جديدة في كل مرة. أنت من يبدأ وأنت من يعزف. تحمل مسؤولية التغيير. اذهب إلى الموضوع. لنرى ما إذا كان بإمكانك تحديد النقطة التي بدأت فيها معك أو التي صرخت فيها. ربما ستجد أيضًا الحسابات التي أجريتها على جزيء الهيدروجين.

    وليس لدي أي سيطرة على حقيقة أنه من الصعب فهمك. المراجع التي أعطيتك إياها ممتازة. لو لم تحرق جسورك معي، كما فعلت هنا مع الجميع، كنت سأخوض الفكرة بأكملها معك ببطء. لكن الآن، اذهب وابحث عن الأصدقاء الذين لا تملكهم.

  204. ر.ح.

    2. سأكون سعيدًا بالبقاء في الاتجاه السائد، ولكن ماذا علي أن أفعل إذا كانت النظريات لا تتفق مع بعضها البعض؟ لم ننتهي من مناقشتنا حول ساعات درجة الحرارة. إذا واصلنا، قد يقنع أحدنا الآخر. إذا لم يكن الأمر كذلك، فأنا مضطر لمواصلة النظرية وإيجاد تفسيرات منطقية بالنسبة لي.

    إذا أردت، يمكنني أن أشرح لك كيف يمكن للحاضر، وفقًا لميكانيكا الكم، أن يؤثر على الماضي، وكيف يمكن لفكرتي أن تخرجنا من التشابك.

    3. فكرة أن كل نظام مستقل ولا يوجد شيء اسمه التحرك أو الراحة ليست فكرتي. ومن جاليليو ونيوتن وأينشتاين. هذا هو في الواقع الافتراض الأول في العلاقات. في مثال GPS الخاص بك، تتحرك القوارب بالنسبة لنظام GPS. إذا كان هناك نظام GPS آخر، فإن الإحداثيات ستتغير وفقًا لذلك. إذا اختار جاك وجيل أندروميدا، فسيكون إطارًا مرجعيًا اعتباطيًا جديدًا وسيتعين على الجميع إجراء تحويلات الإحداثيات اللازمة للانتقال إلى هذا الإطار. النتائج، بالمناسبة، سوف تكون هي نفسها تماما.

    لاحظ بالمناسبة، بما أننا لم ننته من مناقشاتنا، في نظام جاك-جيل ليست هناك حاجة لعامل خارجي إضافي.

    6. لا أفهم كيف يمكن أن تكون الأمور واضحة بالنسبة لك، إذا توصلنا إلى نتيجة مفادها أن درجة الحرارة وساعات السيزيوم يجب أن تظهر نفس درجة الحرارة في نظام القصور الذاتي، وإلا فإن ذلك يتعارض مع المسلمة النسبية الأولى:

    مبدأ النسبية:

    لا تتغير قوانين الفيزياء عند الانتقال من إطار مرجعي بالقصور الذاتي إلى إطار مرجعي بالقصور الذاتي آخر. على سبيل المثال، لا يستطيع شخص في عربة قطار مغلقة، من خلال أي تجربة أو قياس فيزيائي، تحديد ما إذا كانت السيارة تتحرك بسرعة ثابتة أم أنها في حالة سكون.

    بعد كل شيء، وفقًا لك، كل ما عليك فعله هو النظر إلى وجه الساعة. وفقًا لنسبة الدوران بين الساعة المؤقتة وساعة السيزيوم، ستعرف بالضبط ما إذا كنت تتحرك وبأي سرعة. (نسبة إلى لماذا، بالمناسبة؟).

    7. وفقًا لمنطقك، فإن المتنمر في الروضة والأطفال الذين يضربهم يتصرفون مثل الأطفال الصغار، أليس كذلك؟ الحقيقة هي أنهم يتعرضون للضرب. أو المغتصب وضحيته، الخ.

    يهتم النظام القانوني والشعور بالعدالة الطبيعية كثيرًا بمسألة من بدأ ومن عاد. الأول سيكون صالحا. الى السجن والآخر يدافع. بعنوان. حتى لو وجهت الثانية ضربات أقوى بكثير.

    في المواجهات بيني وبين يوفال، يبدأ يوفال دائما، رغم أنه تعهد صراحة بعدم القيام بذلك. الطريقة الوحيدة للتعامل مع المتنمرين هي ضربهم بقوة بمطرقة ثقيلة، قبل أن يسحبوا الكثير من مؤخرتهم. بواسطة الغفوة الأخرى التي تركتنا بالفعل.

    ولا تحتفظ بالكثير من ساحلك الشرقي لي. المعضلة الرئيسية التي تواجه سكان الساحل الغربي هي أنهم عندما يستيقظون في الصباح - حسنًا، لنكن دقيقين، في الساعة 11 صباحًا، بعد التمدد حتى الساعة 12 وتناول وجبة الإفطار حتى الساعة 1، أين يذهبون اليوم: إلى البحر. (نصف ساعة بالسيارة) أو التزلج (ساعة).

  205. التخنيون,
    انت على حق تماما. حقا لا يعمل بهذه الطريقة. أقترح أن نترك الأمر للساعة وعندما تسنح لي الفرصة سأقوم بنشر كل شيء بطريقة منظمة. الآن سأبحث عن ما هو "طول الاتصال". شكرا. مع السلامه

  206. اليوبيل,

    "التفسيرات موجودة، لكن في الوقت الحالي يطلب منك أن تثق بكلمتي فقط."

    - لا تثق. لا يعمل بهذه الطريقة.

    "ربما لم أفهم سؤالك حول مدة العلاقة."

    - لذا ابحث عن طول الرابطة.

    وآسف، ما زلت لا أفهم ما هي الحسابات التي لديك لجزيء الهيدروجين.

  207. إسرائيل،
    2) هذه ملاحظة نتفق عليها مع أينشتاين وبقية العالم. المشكلة في تفسيرك للسيارات الشفافة وتلك التي لم تصل. لكن لنفترض إذا أظهرت لي أن هذا التفسير له ميزة في تفسير ظواهر مثل تأثير الماضي على المستقبل (ما هو المقصود بالضبط؟؟) فلنتناقش.

    3) هذه نقطة لا أعتقد أنك تتناولها. طالما أن هناك نظامين فأنت على حق، كل منهما في حالة سكون والآخر يتحرك بالنسبة له. ومع ذلك، إذا أضفنا نظامًا مرجعيًا آخر مثل الشجرة في الخارج أو على المريخ، فسيكون من الممكن معرفة من يسافر ومن يستريح.

    سؤال: كيف يمكن تحديد موقع نقطة بالنسبة لنقطة أخرى بدون نظام مرجعي؟ باستخدام النظام المرجعي، يمكنك إعطاء إحداثيات XY أو القطبية (المسافة + الزاوية من المحور X). ولكن يجب أن يكون لديك نظام مرجعي تعسفي.

    مثال آخر - قاربين في وسط المحيط. وأيهما يتحرك وأيهما يستقر؟ ليس من الممكن أن نعرف. ومع ذلك، إذا نظر اثنان من البحارة في القوارب إلى نظام تحديد المواقع العالمي (GPS)، أي في نظام مرجعي عبر الأقمار الصناعية، فيمكنك معرفة من يتحرك ومن لا يتحرك. ولهذا السبب اخترعوا أنظمة مرجعية أو محاور للرسوم البيانية.

    وينطبق الشيء نفسه على مثال المركبة الفضائية الخاص بك. يمكن أن يتفق جيل وجاك على أن الإشعاع الكوني أو مجرة ​​المرأة المسلسلة أو أي عامل آخر خارج خلاياهما لا يتأثر بها، سيكون الإطار المرجعي. ومع نظام هيلز المرجعي، عرفوا بالضبط من انتقل ومن لم يتحرك.

    عن الدخان. أنت على حق في أنه كان ينبغي عليّ التأكيد على أن الدخان يشير إلى نظام متسارع. فالطائرة الشراعية التي ينبعث منها الدخان ستبقى في سحابة الدخان حتى لو تحركت للأمام، بينما الطائرة المتسارعة ستسحب أثرًا من الدخان خلفها. اعتقدت انه كان واضحا.

    6) لا أوافق على أن هناك تناقضًا أو مشكلة في الأمر وما زلت غير قادر على إقناع سبب تورط الفيزيائيين فيه. بالنسبة للشخص العادي تبدو الأمور واضحة. لكن ماذا أفهم؟ عالم الأحياء، كما تعلمون.

    7) ما أفهمه هو أنكما تتصرفان كطفلين صغيرين. بدأ قال لي هكذا ورد عليّ هكذا. ليس وسيمًا وغير مناسب وليس على شرفك. هيا، اشرب كأسًا افتراضيًا من النبيذ (وليس بعض العصير من وادي نابا، أو شيئًا حقيقيًا، أوروبيًا) وارجع إلى المناقشة مثل شخصين مثقفين مثقفين يحبان العلوم.

  208. التخنيون,
    أشكرك لأنك منحتني الفتح لتطهير اسمي. التفسيرات موجودة، لكن في الوقت الحالي يطلب منك أن تثق بكلمتي فقط. ببساطة، إنها مبنية على تعريفات لم يحن وقتها بعد.
    ربما لم أفهم سؤالك عن مدة العلاقة. ربما اعتقدت أنك تتحدث عن نصف القطر التساهمي للذرات المشاركة في الرابطة. يصعب حساب ذلك في ذرة الكلور ولكن من السهل (نسبيًا) حسابه في ذرة الهيدروجين. لقد وصفت نواة ذرة الهيدروجين، البروتون، بأنها مستعمرة من الجسيمات التي تتفاعل مع بيئتها. لم أتوصل بعد إلى حساب حجم هذه المستعمرة ولم أوضح بعد كيف يسمى الإلكترون وكيف يتم إنشاء القوى المستخدمة في الروابط بين الذرات. العمل على الموديل كثير والوقت المتاح لي قصير وليس لدي مساعدين. آسف.
    ليست لدي مشكلة مع المراجع، لكنه لم يحضر أي منها لأنه أرسلني للبحث عنها بنفسي وفي النهاية نصل إلى مقال يدحض ادعائه. وعلى ضوء ذلك كان ينبغي أن يقول إنه أخطأ، لكنه بدلاً من ذلك يصرف انتباه القراء عن عبثه وينشر الألفاظ (ولو باللغة الرفيعة، كما يليق بشخص "متحضر" مثله، لكن الألفاظ فجور) و ينزع مني أي رغبة في مواصلة الحديث معه.

  209. اليوبيل,

    "إن مسألة ما إذا كنت كذبت أم لا مفتوحة للتأويل."

    - بالتأكيد لا. إذا كان لديك تفسيرات لما كتبته عندما كتبته، فأنت لم تكذب. إذا لم تكن لديك، فقد كذبت.

    من كل ما كتبته لاحقًا، لم أفهم ما هي الحسابات التي لديك لجزيء الهيدروجين.

    "الخطاب هنا عام. إذا منعتني من الروابط وعمليات البحث، فأنت لا تزعجني فقط، بل تزعج كل من يتابعني أيضًا."

    - ما يجب القيام به، في المراجع العلمية هي جزء من المناقشة. هل سبق لك أن رأيت مقالًا لا يشير إلى الروابط؟

  210. اليوبيل.
    الجميع هنا يشير إلى الروابط. أحالني أحد الطلاب إلى العديد من الروابط في هذه المقالة وحدها. على عكسك، أنا أزعج نفسي بقراءتها.

    تشرح نظرية انبعاث الضوء نتائج M-M بشكل جيد، وهذا هو الشيء الوحيد الذي قلته. ومن الواضح أن هناك مشاكل أخرى فيها، مثل تفسير لورنتز وأينشتاين وتفسيرك أيضًا.

    وليس لدي أي مشكلة في مصداقيتي في أعينكم.
    لقد عرضت عليك التحدي. إذا احتفظت بها، يمكننا أن نخوض معركة جيدة. بمجرد عودتك إلى الأوراق الشخصية كالمعتاد، سوف تسرق 10 مرات.

  211. ر.ح.
    لقد عدنا من العمل.

    2. بالنسبة للعداد، المعنى هو أنه بغض النظر عن السرعة التي أنت بها، فإنك ستقيس الضوء على أنه C. إذا كنت تتذكر نموذج أوتوسترادا، فإن السيارات السريعة جدًا تكون شفافة بالنسبة لك، والسيارات البطيئة لم تصل بعد.

    3. في المثال الذي قدمته، تعتبر الشجرة نظامًا ذاتيًا تمامًا، وبنفس القدر يمكنك اختيار شجرة على المريخ. يبدو لي، بالمناسبة، أنه يجب عليك أن تستوعب هذه النقطة، أنه لا يوجد شيء اسمه نظام مطلق. كما قلت من قبل، لن تتمكن من "إخراج الدخان" من سفينة الفضاء وبالتالي معرفة ما إذا كانت سفينة الفضاء تتحرك أم لا. البروتون المتسارع إلى 0.999C يكون في حالة سكون بالنسبة له، ما دامت سرعته ثابتة. إن المجرة التي تبتعد عنا بسرعة قريبة من سرعة الضوء هي في حالة سكون، ومن أجلها نحن الذين نتحرك.

    4. مثل 3.

    5. لم أتقبل مبدأ أن كل حركة هي نسبة إلى الإشعاع الكوني. ولا أينشتاين فعل ذلك. ولم يكن يعرف عنها حتى في عام 1905.

    لم ننتهي من مناقشاتنا السابقة، ولكن أعتقد أنك رأيت الطبيعة الإشكالية للمسألة (صدقني الآن بعد أن وقع الفيزيائيون في مشكلة أيضًا؟). إذا كنت تريد، يمكننا الاستمرار، ولكنني سأتفهم بالتأكيد إذا كنت تريد الانسحاب.

    شيء واحد فقط - الضوء البطيء قادر على تفسير الظاهرة المحيرة أكثر من غير المحلية - التأثير على الماضي من المستقبل.

    وفي مسألة يوفال - هناك مسألة حق الدفاع عن النفس. انكم مدعوون للذهاب من خلال الموضوع في هذه المقالة وغيرها. أثبت أنني لا أسيء إلى المعلقين أبدًا. لكن ليس لدي رغبة في الامتصاص والانسداد.

  212. السادة المحترمون يوفال وإسرائيل،

    هل أنت مستعد للاسترخاء وتذكر أن هذه مناقشة علمية من أجل المتعة؟ إذا لم تستمتع به، فأنت تجعلني والآخرين لا نستمتع به أيضًا، وإذا كان الأمر كذلك فلا فائدة من ذلك ولنتوقف عن المناقشة.
    لسوء الحظ، لم يعد هناك ثلج ولا أستطيع الذهاب للتزلج، لأنه في المرة الأخيرة تسبب في حب كبير بينكما. لذلك أيها السادة، عودوا إلى رشدكم، واهدأوا، واشربوا بعض الماء وابدأوا في الحديث في صلب الموضوع وليس في الأسلوب والشرف وكل ما يخرج من هذا الهراء. لم يطلب منك أحد الزواج أو حتى الخروج إلى الحانة لتناول البيرة معًا. هيا، تهدئة.

  213. إسرائيل،
    والحديث هنا عام. إذا منعتني من الروابط وعمليات البحث، فأنت لا تزعجني فقط، بل تزعج كل من يتابعني أيضًا. كان من المناسب لك أن تشرح الأشياء بكلماتك الخاصة أو على الأقل أن تعطي إحساسًا بالمكان للأشياء الواضحة. على أية حال، لقد سئمت من "شرفي"، نظرت حولي ولم أجده، وأنا لا أصدقك.
    وما وجدته في الرابط أعلاه http://en.wikipedia.org/wiki/Emission_theory، يقدم التفسير الذي تتحدث عنه على أنه بعيد المنال.
    إذا كنت ترغب في الحفاظ على مصداقيتك، حتى لو كان ذلك فقط في نظر شخص غائم مثلي، فعليك أن تبذل جهدًا أكبر.

  214. التخنيون,
    مسألة ما إذا كنت كذبت أم لا مفتوحة للتأويل.
    يتظاهر نموذجي بتقديم جميع النظريات الأولية للنماذج الفيزيائية الموجودة على أنها ناشئة عن جسيم أولي واحد. بمجرد تنفيذ هذه المهمة لبعض النماذج المادية الموجودة، فمن الممكن الاستمرار منها. من الممكن، كتمرين، بناء كل شيء مباشرة من النموذج الأساسي. إنها وظيفة غير ضرورية ولكنها وضيعة محتملة. وإذا كان يوحي من كلامي أنني قمت بها لجميع النماذج المادية الموجودة، فأنا كذبت وأعتذر عن ذلك.
    لقد سألت عن طول الرابطة بين ذرة الهيدروجين وذرة الكلور. تحتوي ذرة الكلور على 17 بروتونًا وعددًا مماثلًا من النيوترونات. تضيف كل من هذه النيوكليونات معلمات إلى معادلة حساب نصف القطر التساهمي. تعتمد مساهمة كل نيوكليون على عدة عوامل، على سبيل المثال موقعه داخل النواة بالنسبة للنيوكليونات الأخرى. ولذلك فإن حساب نصف القطر التساهمي لذرة الكلور يتطلب الكثير من العمل. تحتوي نواة ذرة الهيدروجين عادة على نيوكليون واحد فقط، ومن الأسهل حساب نصف قطرها التساهمي.

  215. إسرائيل،
    حسنا، دعونا نستمر.
    1) نحن موافقون

    2) ما هو الأمر بالنسبة للمساح؟ هل الجملة التي لا تعادل "سرعة الضوء ليست نسبية"؟ هل يمكنك توضيح النظام الذي سيتصرف الضوء معه نسبيًا؟

    3) أوافق، باستثناء عبارة أن الإشعاع الكوني وحده هو نظام السكون. على الرغم من اسمها فهي ليست سحرية. بالنسبة لي، فإن أي نظام ثالث سيتم تعريفه على أنه نظام راحة سيخبرك بالضبط من يسافر ومن لا يسافر.
    مثال: أنت في قطار في محطة القطار. أمامك بجانب النافذة قطار آخر. نبدأ بالتحرك بين القطارين. كيف تعرف من يتحرك؟ قطارك أم الآخر؟
    الإجابة: أنت تنظر إلى الجانب الآخر من الشجرة الذي يمكن رؤيته من النافذة الأخرى. فيما يتعلق به (ولاحظ أنه ليس إشعاعًا كونيًا) يمكنك معرفة بالضبط من يسافر ومن يقف.

    4) مثل 3

    5) لماذا لا تستقر؟ إذا قبلت المبدأ القائل بأن كل الحركة مرتبطة بالإشعاع الكوني، فلماذا لا تكون كل الحركة في الزمن مرتبطة بالنقطة الصفر للانفجار الكبير؟

    6) سننتظر لحظة ونتحقق من 1-5

  216. ر.ح.

    لقد رأيت تعليقك للتو.

    وهنا الإجابات:

    1. هناك إطالة زمنية في الأنظمة المتسارعة. لا أعرف ما إذا كان هذا صحيحًا في الأنظمة غير المتسارعة.

    2. سرعة الضوء دائمًا هي C بالنسبة للمتر. الأسباب التي تجعل السرعات الأعلى أو الأقل من C شفافة بالنسبة لنا غير معروفة، وتم شرحها في مثال البندول الباليستي.

    3. لا يمكن التمييز بين نظامين غير متسارعين من هو المتحرك ومن هو الساكن مطلقاً، إلا إذا قارنت سرعتهما بالنظام الساكن للإشعاع الكوني. وبصرف النظر عنها، كل حركة نسبية فقط.

    4. إذا قمت بإضافة نظام آخر وقررت بشكل تعسفي أن هذا النظام في حالة سكون، فإن أي حركة ستكون نسبة إلى هذا النظام. وبالتالي فإن الجواب هو لا (على الرغم من أنني لم أفهم حقًا ما تقصده هنا بنعم أو لا). إذا تم تضمين الساعة المؤقتة في النظام المرجعي الجديد، فستكون الحركة مرتبطة بها.

    5. حجتي كلها هي أن النسبية غير متوافقة مع نظرية الانفجار الأعظم. أنا لا أدعي أن أقرر بين الاثنين.

    لكن إذا قبلت نظرية الانفجار الأعظم، فقد قبلت الزمن المطلق في نظام بالقصور الذاتي، ومن هنا يأتي التناقض مع النسبية حيث لا يوجد مثل هذا الزمن. وفقا لمناقشاتنا.

    6. إذا انتهينا من المناقشة الحالية أولاً، وعلى افتراض أنك ستقتنع بصحة ادعاءاتي، فسوف ترى، على ما أعتقد، أنه من الممكن أن نشرح بطريقة كلاسيكية سبب كون سرعة الضوء هي السرعة القصوى. نفس الشيء بالنسبة لكل قاس، ولماذا لا يمكن أن يكون هناك إطالة للوقت في نظام غير متسارع. ستكون قادرًا على رؤية ميزة فكرة السرعة المتغيرة (والتي ليست مجرد مسلمة، فهي مطلوبة في نموذج الموقع النشط) عندما نقترب من النماذج الكمومية.

  217. ليس لدي سيطرة على ما تفهمه من كلامي، ليس لدي سيطرة إلا على ما أقول.

    اقترحت عليك البحث عن نظرية انبعاث الضوء، لو فعلت ذلك لوجدتها هناك: نظرية الانبعاث (وتسمى أيضًا نظرية الباعث أو النظرية الباليستية للضوء) كانت نظرية منافسة للنظرية النسبية الخاصة، موضحة النتائج من تجربة مايكلسون-مورلي.

    حقيقة أنك كسول للذهاب إلى الروابط ليست مشكلتي.

    وأما غموضي: فأنا أقبله مجاملة من يشهد لنفسه أنه مشاغب يصعب فهمه.

    على أية حال، كنا نأمل أنه بعد أن أصبح لديك مدونتك الخاصة، ستكون مشغولاً بالرد على آلاف الأشخاص الذين يستجيبون لنموذجك، وخاصة أولئك من السويد، وأن تتوقف عن الصراخ بشأن أولئك الذين لا يريدون حقًا التعامل معك و مزاجك السيئ.

    هيا، أطلق النار على عدد قليل من القسام والغاردز لتظهر للجميع من هو الرجل هنا، وتردد الصدى.

    فقط لا تدعي أنه إذا قمت بالتنمر على إسرائيل أكثر من اللازم، فسوف يدمرون لك نصف غزة.

  218. إسرائيل،
    بسبب كثرة الدببة، لم يعد بإمكانك رؤية الغابة. دعونا نتراجع للحظة ونطلب المساعدة من سقراط المسن والمسموم.

    من فضلك، دعونا نلخص بوضوح ما هي مطالباتك:

    الرجاء الإجابة على الأسئلة التالية بنعم و لا و الشرح باختصار إذا لزم الأمر (لا تنسى الوقت و التاريخ)
    1) لا يوجد تمديد للوقت؟
    2) هل سرعة الضوء نعم نسبياً؟ (حسنًا، أعتقد أنني فهمت ذلك، أنت تدعي نعم، ولكن في أجهزتنا نرى فقط الضوء يتحرك في C ويكون أسرع وأبطأ شفافًا بالنسبة لنا لأسباب غير معروفة؟)
    3) إذا كان لدينا نظامان فلا يمكن تحديد من يتحرك ومن يستقر؟
    4) إذا أجبت بنعم على 3، إذا أضفنا نظامًا ثالثًا لا يعتمد على جيل أو جاك وعاملناه كنظام مرجعي، فهل ستظل الإجابة على 3 هي "نعم"؟ (النظام الثالث يمكن أن يكون، على سبيل المثال، Beetlejuice، أو ساعة درجة الحرارة، أو Edmond في سفينة فضائية لامعة)
    5) إذا وافقت على 4، هل مازلت ترى تناقضا في النظرية النسبية؟ إذا كان الأمر كذلك، فما هو؟
    6) هو النموذج الغريب للأشعة الضوئية في جميع السرعات التي تتحرك على جزيئات الأثير غير المعروفة التي تكتشفها مستشعراتنا بجميع أنواعها - الفيزيائية (التأثير الكهروضوئي)، أو البيولوجية - (الفقيرة) فقط عند وحدة السرعة C ميزة على ال الموجود وأيضا النموذج الغريب للضوء بسرعة ليست نسبية؟

    بالنجاح

  219. يوفال، هل كذبت هنا: https://www.hayadan.org.il/astronomers-reach-new-frontiers-of-dark-matter-130112/#comment-329366
    عندما كتبت معظم ما فصلته هنا: https://www.hayadan.org.il/astronomers-reach-new-frontiers-of-dark-matter-130112/#comment-329336
    هل يمكنك العثور على تفسير؟

    وكتبت أيضًا أن لديك حسابات لجزيء الهيدروجين، ما هي الحسابات؟

  220. إسرائيل،
    أنا لا أدعي أن لدي صبرًا لا نهاية له كصديق مشترك لنا. صبري ينفد بين الحين والآخر، وليس فقط بسببك. إن الدافع الذي يجعل صبري ينفجر مرارًا وتكرارًا في المحادثات معك هو التعتيم الذي تظهره، سواء كان حقيقيًا أو متخيلًا. قلت "السؤال: لماذا لا نأخذ التفسير الأبسط، وهو أن سرعة الضوء بسيطة بالنسبة إلى مصدر الضوء؟ وهذا من شأنه أن يفسر نتائج تجربة M-M بسهولة، أليس كذلك؟ (https://www.hayadan.org.il/astronomers-reach-new-frontiers-of-dark-matter-130112/#comment-332017). ومن هذا فهمت أنك تزعم أن سرعة الضوء بالنسبة لمصدر الضوء تفسر كيفية ثبات سرعة الضوء بالنسبة للراصد، وذكرت أنه يجب عليك إثبات كلامك هذا. ربما أسأت الفهم، ولكن حتى ذلك الحين يتعين عليك الإدلاء ببيان على غرار "تم إخراج كلماتي من السياق" أو شيء من هذا القبيل.
    تصبحون على خير/صباح الخير (اختر الخيار الصحيح).

  221. سأسألك للمرة الأخيرة: دليل على ماذا؟
    وإذا لم تجب بالضبط هذه المرة، سأطلب منك أن تتركني وشأني.
    ماذا لو كانت الساعة الثانية صباحًا في لوس أنجلوس؟ - طاب مساؤك.

  222. قلت ما؟ مرة أخرى من المفترض أن أخمن نواياك؟ لا أتذكر أنني تجنبت مطلقًا أي موضوع طرحته، راجع مناقشتي التفصيلية التي استمرت لمدة شهر مع R.H.
    لكن ليس لدي أي التزام بمناقشة المواضيع التي لا أهتم بها.

  223. أيها الطالب، التخنيون، مرحبا بكم مرة أخرى
    الجواب على سؤالك هو "ربما". واسمحوا لي أن أشرح: نموذجي يبدأ من أبسط العناصر ويتطور خطوة بخطوة. حتى الآن بالكاد أتطرق هنا إلى الجاذبية والقصور الذاتي لبروتون واحد. البروتون هو نظام معقد للغاية ("مستعمرة") ولم أتطرق إليه إلا في النهاية. ما زلت لم أوضح كيف "يجد" البروتون والإلكترون بعضهما البعض، وكيف يختلف النيوترون عن البروتون، وكيف تعمل القوة القوية وما الذي يسبب فقدان الكتلة في الانشطارات النووية. ويجب أيضًا أن أشرح القوة الضعيفة وعائلة كبيرة من الجسيمات دون الذرية، والعوامل الكامنة وراء ميكانيكا الكم (بما في ذلك الافتقار إلى الموقع في التشابك الكمي الذي لم أجد له تفسيرًا بعد)، وما إلى ذلك، وأخيرًا أيضًا تقديم "تنبؤات". لظواهر لم تُكتشف بعد" بالشكل المناسب للنموذج الذي يحترم نفسه. يتكون جزيء حمض الهيدروكلوريك من عدة أنظمة معقدة تتكامل مع بعضها البعض. لم يحن الوقت بعد للتعامل مع هذا الأمر ومع الجانب المحدد للغاية الذي تسأل عنه. وبعد كل هذا لا أزال أقول "ربما"، لأنه حتى الآن ليس لدي حسابات جاهزة تتجاوز جزيء الهيدروجين. قد أختار التعامل مع سؤالك أيضًا أو قد أترك الأمر لشخص يتابع الأمور، كتمرين. على كل حال لن أفعل ذلك هنا فوق صفحات العلوم ولكن في المدونة التي فتحتها لهذا الغرض (ولكم الشكر).

  224. إسرائيل! ليس من اللطيف ولا المشرف أن تتصرف بهذه الطريقة. يجب أن تتحمل مسئولية ما تقوله. لقد قلت شيئًا، فكن رجلاً والتزم بكلامك. طالما أنك لا تفعل ذلك، فلا أرى مفرًا من الاستنتاج بأنك مطارد متسلسل.

  225. يا صديقي، ألا نفعل شيئًا تكريمًا لعيد ميلاد أينشتاين؟

    بالطبع سوف يعطي جالي مقالا.

  226. يوفال، هل يمكننا رؤيتك تحسب طول رابطة حمض الهيدروكلوريك (HCl) باستخدام النموذج الخاص بك؟

  227. حسنا يوفال. هل يمكنك ربما تسلق شخص آخر؟ لقد طردتني من الرد على نموذجك. هل يمكنك أن تفعل الشيء نفسه مع كل تعليقاتي؟ مجرد ترك؟

  228. إسرائيل! هل تخوض حرب استنزاف؟ ارسل لي الروابط؟
    وعليك أن تقف وراء ما قلته، بالطريقة البسيطة التالية:
    النظرية أ، والتي منها تشتق النظرية ب، والنظرية ج، الخ... وبالتالي "سرعة الضوء بالنسبة لمصدر الضوء تشرح كيف أن سرعة الضوء بالنسبة لعين الراصد تكون ثابتة دائمًا".
    لا توجد "أوراق شخصية" هنا. هذا مجرد انتقاد للطريقة التي تدير بها خطابًا علميًا زائفًا.
    وحقيقة أنني لا أتدخل في محتوى المناظرة بينك وبين ر.ح. فقط لأنني أعتقد أن الافتراض الأساسي الذي تبني عليه كل حججك خاطئ، وقد سبق أن أخبرتك بذلك عدة مرات، لكنك اخترت لتجاهل كلامي بطريقة شاملة ومنهجية. ومن ناحية أخرى، فإن R.H. يفعل الشيء الصحيح. إنه يكسر حججك واحدة تلو الأخرى ويظهر لك المكان الذي تغضب فيه. ميزته علي هو صبره الذي لا ينضب. لقد برزت الألغام أكثر من مرة.

  229. ر.ح.

    "الخلاصة: هناك فرق بين نظامي جاك وجيل. ———-> صحيح"

    لماذا التمييز؟

    1. ألا ترى أن النظامين متماثلان تماماً؟ فكما يتحرك جيل بالنسبة إلى جاك، فهل يتحرك جاك أيضًا بالنسبة إلى جيل، بنفس السرعة تمامًا ولكن في الاتجاه المعاكس؟

    2. أليس هذا بالضبط ما سيقوله غاليليو ونيوتن وأينشتاين؟

    3. بسبب التناظر المطلق، ألا يعني ذلك أنه إذا تأخرت ساعة جيل بالنسبة لنظام جاك، فإن ساعة جاك ستتأخر أيضًا بالنسبة لنظام جيل؟

    4. أليس هذا بالضبط ما تقوله النسبية؟ هل يرى كل نظام أن الوقت الثاني أبطأ؟

    دعونا نقوم بتمرين فكري صغير. يقال أنه قبل وقت طويل من لقاء جاك بجيل، مرت عليه ساعة متزامنة مع ساعة جيل، وساعة جاك تظهر نفس الوقت في ساعة المرور + ثابت معين.

    لاحظ أن كل ما قمت به هنا هو عكس الأدوار: لقد حصلنا على بيانات المشكلة الأصلية، ولكن في الاتجاه المعاكس فقط. هذه المرة، جاك هو الذي يتحرك بالنسبة لنظام جيل، لذا فإن ساعته تتأخر عن ساعتها. لذلك، بسبب التناظر المطلق، نحن مضطرون إلى استنتاج أنه عندما يلتقي جاك وجيل، ستظهر ساعة جاك تأخرًا قدره مليار سنة بالنسبة لجيل، + نفس الثابت كما كان من قبل.

    لذا، إذا استخدمنا نفس القياس، فسيُظهر مقطع فيديو Jack نسبة دوران تبلغ 1,000,000,000:1، بينما سيُظهر فيديو Jill نسبة دوران تبلغ 1:1.

    وهذا يخالف الفيديوهات الموجودة لدينا والتي تظهر عكس ذلك.

    ولكن ماذا فعلنا تماما؟ هل وضعنا ساعة أخرى لا يعلم أحد بوجودها؟

    باستثناء أن يوفال يشير ضمناً إلى أنني أحاول تجنب شيء ما، أو أنني مخطئ، وهذا في حد ذاته دليل قوي على أنني ربما أكون على حق بنسبة 100٪.

    اليوبيل.
    عبء إثبات ماذا؟ أن سرعة الضوء بالنسبة للمصدر تفسر نتائج تجربة م-م؟ انظر إلى التجربة مرة أخرى، أو راجع نظرية الانبعاث الخاصة بدخول الضوء.

    أنا أيضًا لا أفهم لماذا يتم جر كل شيء معك دائمًا إلى مواجهة شخصية. لماذا لا تعترف أنه ببساطة ليس لديك أي فكرة عما أفعله أنا وآر.إتش. يتحدث؟ بشكل عام، لو مش قادر تتوقف عن كل الأوراق الشخصية، ممكن الأفضل تسكت وخلاص؟ أنت تعرف كيف تبدأ الحرب، وليس كيف تنهيها.

  230. آر إتش،
    من تجربتي مع هيلز أقول وبكل يقين أنه لا فائدة من انتظار إجابة معقولة منه. إنه يستمتع بالتسبب في أعمال شغب لكنه يعرف كيف يهرب بمجرد تسليمه المكنسة والمكنسة

  231. يرى جاك نسبة 1:1
    1) ترى جيل نسبة 1,000,000,000:1.

    الخلاصة: هناك فرق بين نظامي جاك وجيل. ———-> صحيح

    جاك في نظام غير المتسارع.
    جيل في نظام غير معجل.

    الخلاصة: لا يوجد فرق بين نظامي جاك وجيل. سنرى كم أنت غبي عندما تقول إنه لا فرق بالنسبة له إذا كان في المحطة أو في الحافلة، فهو في حالة راحة على أية حال.

    2) ولكن إذا كان نجم Pinkfloodjuice يتحرك بالنسبة إلى Beetlejuice بسرعة الضوء تقريبًا، فإن جيل قريب منه في حالة السكون وجاك في حالة حركة، أليس كذلك؟ هل ستغير نسبة الثورات بين ساعاتهم؟

    إسرائيل أي نوع من التفكير الملتوي هذا ما هي العلاقة بين ما يفعله أو لا يفعله البيتلجوس أو بينك فلويدجس أو ليد زيبلينيوس بالنسبة للنسبة بين ساعات جيل؟؟ أنت محتار هنا بين السبب ونظام الإسناد. النظام المرجعي لا يحدد أي شيء. فإذا حسبنا التاريخ بحسب خلق العالم، فإن ميلاد المسيح أو الهجرة لا يغير شيئا بالنسبة لمعدل تقدم الزمن. إنه مجرد نظام مرجعي أو نظام محاور. يبدو الأمر تافهًا جدًا بالنسبة لي وأنا أقدرك كثيرًا، لذلك أنا متأكد من أنني أفتقد شيئًا أساسيًا حقًا هنا. فقط قل لي ماذا؟

  232. ر.ح.

    يرى جاك نسبة 1:1
    ترى جيل نسبة 1,000,000,000:1.

    الخلاصة: هناك فرق بين نظامي جاك وجيل.

    جاك في نظام غير المتسارع.
    جيل في نظام غير معجل.

    الخلاصة: لا يوجد فرق بين نظامي جاك وجيل.

    يرجى حل التناقض.

    اقتراح:

    أنت تقول أن جاك في حالة راحة نسبية لبيتلجوس، وجيل قريب من سرعة الضوء. ومن هنا الاختلافات.
    لكن إذا كان نجم Pinkfloodjuice يتحرك بالنسبة إلى Beetlejuice بسرعة الضوء تقريبًا، فإن جيل يتحرك بالنسبة إليه في حالة السكون وجاك في حالة حركة، أليس كذلك؟ هل ستغير نسبة الثورات بين ساعاتهم؟

    لا تنس من أين بدأنا: تتحرك جيل فقط وفقًا لنظام الساعة المتزامن مع نظام جاك. بالنسبة لها، يتحرك جاك تمامًا كما تتحرك بالنسبة إليه. إذا أردنا التحقق من حركة جاك بالنسبة لنظام جيل، فسنرى أن الوقت يتباطأ بالنسبة له، ولكن ليس بالنسبة لجيل. وماذا في ذلك، فجأة سيتغير موقفهم لمجرد أننا قررنا أن النظام مختلف؟

    فكر في النظام المتزامن كقطار طويل بعربات تفصل بينها كيلومتر واحد. اقترح أينشتاين في مقالته الأصلية عن النسبية كيفية مزامنة الساعات باستخدام أشعة الضوء.

    إذا كانت السيارة تتحرك بسرعة على طول القطار، فسوف ترى في كل ساعة في السيارة التالية وقتًا أكثر وأكثر تقدمًا قليلاً من وقتها. هذا جيلنا.

    ولكن نفس المنطق يعمل أيضا في الاتجاه المعاكس. إذا كان جيل يجلس في قاطرة، وتتبعه العديد من السيارات المتزامنة، وكان جاك يقود قطارًا يتحرك في الاتجاه المعاكس، فإذا كانت الساعات تظهر نفس الوقت أثناء مرور القاطرات ببعضها البعض، فسيجد كل من جاك وجيل الوقت في السيارات التالية تتسارع أكثر فأكثر بالنسبة لها (وذلك لأن القاطرات، بالنسبة للقطارات، هي التي تتحرك، وبالتالي فإن الزمن فيها يتباطأ).

    على أية حال، التماثل هنا مثالي. النظامان متماثلان تمامًا، فمن وجهة نظر جاك، أو كل سيارة في قطاره، فإن جيل تتحرك، بينما من وجهة نظر جيل، وكل سيارة في قطارها، فإن جاك يتحرك.

    لذلك لا يمكن أن يكون هناك أي اختلاف في نسبة دورات الساعة. النجوم الخارجية، أو الجربوع المتنقلة، ليست ذات صلة على الإطلاق.

    إذا كانت سرعة الضوء نسبة إلى مصدر الضوء، فهذا يفسر بدقة تجربة م-م. هذا هو الشيء الوحيد الذي قلته. ومن الواضح أن هناك مشاكل أخرى في هذا، ولكن كذلك نظرية لورنتز (الانكماش) ​​أو النموذج الذي أعفي من الحديث عنه.

    الكازينوهات - لأسباب واضحة (الأطفال الذين يمكنهم تصفح الموقع على سبيل المثال) لا أستطيع التوسع دون موافقة والدي. بريدي الإلكتروني مع يوفال، إذا أردت سأرسل لك التفاصيل.

  233. بالإضافة إلى ذلك، سيتم قياس السرعة C بواسطة أي مقياس فراغ في عالمنا
    هناك حالة مفادها أن سرعة الضوء ستكون مختلفة في كون آخر، ولكن يجب استبعاد ذلك أو إثباته.

  234. إسرائيل ويوفال

    وأنا أتفق مع يوفال هنا في أن التفسير الأبسط، وهو أن سرعة الضوء بسيطة بالنسبة لمصدر الضوء، ليس له صلة بالموضوع ولا يعمل.

    وبحسب الورقة النقدية القديمة من فئة 5 ليرات والتي لا ينبغي ذكر اسمها في موقع العلوم، فإن سرعة الضوء ثابتة لكل مشاهد أو مشغل أو مصدر ضوء. بغض النظر عما يفعله أو لا يفعله، حتى لو وقف على رأسه أو قلب إلى الخلف، فإنه سيقيس دائمًا سرعة الضوء بـ C. وسيفعل الشيء نفسه من قبل جميع من حوله بتسارعاتهم وسرعاتهم ونقراتهم ، سيقيسون جميعًا كل ضوء ولا يهم من أين جاء ومتى، مثل السرعة C (تحت شرط واحد فقط، في الفراغ).

  235. سأبدأ بالإعفاء. أنا عادة لا أستعيد الهدايا إن كلمة "إعفاء" لأغراضنا تعني أنه لا يتعين عليك أن تشعر بأنك مجبر على الرد على ما أقوله.
    هل تقول أنه من حقيقة أن سرعة الضوء نسبة إلى مصدر الضوء يمكن استنتاج نتائج تجربة مايكلسون مورلي؟ إذا كان الأمر كذلك، فإن عبء الإثبات يقع على عاتقك.

  236. اليوبيل.
    "اقتراحك، "التفسير الأبسط، وهو أن سرعة الضوء بسيطة بالنسبة لمصدر الضوء"، ليس له صلة بالموضوع ولا يعمل."
    يعمل بشكل جيد لشرح نتائج التجربة M-M.
    وفيما يتعلق بنموذجك وإلغاء الإعفاء - فالشرط من جهتي كان ولا يزال طلباً صريحاً منك، والتزاماً منا كلانا بالالتزام بالأمور فقط.

  237. إسرائيل،
    هذا النقاش يدور ويدور.
    1) تقول: "تذكر أن كلاهما في راحة تامة". التحذير الآخر الوحيد الذي أعرفه في الحساب هو السرعة النسبية لنظام الإشعاع الكوني. لكن هذا ليس موضوعنا، ولا صلة له بالموضوع في الوقت الراهن. نحن في عام 1905، لا يوجد إشعاع".

    لا تحتاج بالضرورة إلى الإشعاع الكوني. يمكن لجيل وجاك أن يأخذا أي شيء ثالث يتم تحديده بينهما وهو الإطار المرجعي. لنفترض نجم بيتلجوس. سيتفق كل من جاك وجيل على أن جيل يتحرك بسرعة قريبة من سرعة الضوء بالنسبة إلى بيتلجوس بينما يتحرك جاك بشكل أبطأ (السرعة السلبية).

    الاستثناء الوحيد هو الضوء. أعتقد أنك في عداد المفقودين شيئا هنا. لنفترض أن جيل أرسل شعاعًا ضوئيًا وقام جاك بقياس سرعته، فسيحصل على C. جيل الذي يقيس سرعة ذلك الشعاع سيحصل أيضًا على C، وسيحصل Jorgenson على كوكب Beetlejuice على C من ذلك الشعاع.

    2) أنت تقول: "على أية حال، إذا عدنا إلى مناقشاتنا، فلا مفر من الاستنتاج بأن صورة لساعتي درجة الحرارة والسيزيوم ستظهر نفس الوقت في كليهما، في كل من جاك وجيل. إذا لم يكن كذلك، أرني أين."

    لماذا لا يوجد مفر؟ لأنك قررت؟ وكما ذكرنا، فإن جيل سيشهد تبريدًا بمعدل 1000 درجة في الساعة، بينما سيشهد جاك 1000 درجة خلال مليار سنة. ومن الواضح أنه عندما يقوم كلاهما بالتقاط صور للساعات، سيتم قياس نفس درجة الحرارة بالضبط. مرة أخرى مثل مثال القطار. إذا قام راكبان وجليسة وعداء بالتقاط صورة للمنظر الخارجي، فسيكون دائمًا نفس المنظر على الرغم من أن أحدهما قد قطع مسافة أكبر بكثير. لقد مضغناها قليلاً، أليس كذلك؟

    3) ما العلاقة الآن بالانفجار الأعظم؟ يتم تعريف الانفجار على أنه نقطة مفردة لا توجد حاليًا طريقة لمعرفة ما حدث من ماضيها الثاني، نعم الزمن، لا زمان، نعم مكان، لا مكان هي أسئلة غير ذات صلة لأنه لا توجد حاليًا أداة للتحقق منها. بشكل عام، لا يوجد وقت قبل الانفجار، وهذا سؤال غير ذي صلة. إنه يشبه الصوت الذي يصدر عن التصفيق بيد واحدة.

    4) الكازينو - أتركك من هراء أينشتاين ماخ وكل ذلك. إنه أكثر إثارة للاهتمام. كيف يمكن أن يكون عد 6 عبوات أمرًا بسيطًا مثل عد طرد واحد؟ أعطني بعض الكازينو للدمى.

  238. إسرائيل،
    لقد أحضرت تفسير لورينز فقط كمثال على تفسير محتمل. تفسيري مختلف. لقد قلت ذلك هنا عدة مرات، وحتى أنني قدمت تفصيلاً بناءً على النموذج الخاص بي:
    https://www.hayadan.org.il/astronomers-reach-new-frontiers-of-dark-matter-130112/#comment-329898
    اقتراحك، "التفسير الأبسط، وهو أن سرعة الضوء بسيطة بالنسبة لمصدر الضوء"، ليس له صلة بالموضوع ولا يعمل.

  239. جميل، لقد انتهى الجزء الأول. دعونا نحاول ب.

    لا تنس أنني لا أحاول دحض تمدد الزمن، بل أشير فقط إلى تعارضه مع نظرية الانفجار الأعظم. حاول أن ترى الأمر بهذه الطريقة:

    1. وفقا للانفجار، هل هناك أي معنى على الإطلاق لسؤال ما كان قبل 1000 سنة من الانفجار؟ ماذا حدث قبل 20 مليار سنة؟ أم أن مفهوم "الزمن" ذاته لم يكن محددًا قبل الانفجار؟

    2. وفقا للنظرية النسبية عام 1905، هل هناك مشكلة في سؤال ماذا حدث قبل 100 مليار سنة؟

    على أية حال، إذا عدنا إلى مناقشاتنا، فلا مفر من الاستنتاج بأن صورة لساعتي درجة الحرارة والسيزيوم ستظهر نفس الوقت في كليهما، في كل من جاك وجيل. إذا لم يكن الأمر كذلك، أرني أين.

    الكازينوهات: لا توجد مشكلة خاصة لدى الموزع في حساب 4 أو 8 أو 800 مجموعة من أوراق اللعب. المشكلة في القدرة على البقاء. في الواقع، هناك طريقة تم تكييفها خصيصًا للأماكن التي تحتوي فقط على طاولات مكونة من 6 طوابق أو أكثر.

    خلاصة القول: ليس صحيحا أن الكازينو يفوز دائما، ولكن الصحيح أن المقامر يخسر دائما. حتى مع أنه لديه ميزة على المنزل.

    إلا إذا كنت مهتمًا بتكريس حياتك للآيس أوفست والجلوس على الطاولات واستنشاق الدخان في الغالب.

    اليوبيل.

    "لست بحاجة إلى أن تكون باردًا لتخوض معركة جيدة".
    بل من الممكن التخلي عن مسألة الأمر الواقع، والانتقال مباشرة إلى العلم. لقد شرحت لك الإشكالية الموجودة في تفسيرك لتجربة MM. (أعتقد أنك قصدت تفسير لورنتز، الانكماش).

    سؤال: لماذا لا تأخذ بالتفسير الأبسط، وهو أن سرعة الضوء بسيطة بالنسبة لمصدر الضوء؟ هذا من شأنه أن يفسر نتائج تجربة M-M بسهولة، أليس كذلك؟

  240. سنحاول التكامل في الأجزاء مرة أخرى، وسنرى ما إذا كان سيتم تنفيذه.

    ر.ح. محبوب.
    رونان، إيه؟ ببطء وحذر، مثل القنافذ، سوف نكشف عن وجهك الحقيقي.

    3. الراحة: جاليليو، نيوتن، أينشتاين. أي جسم لا يتحرك بتسارع فهو في حالة سكون. قد يكون متحركًا بالنسبة للأنظمة الأخرى، لكن فيما يتعلق بنظامه فهو في حالة سكون.

    الضوء: يتحرك دائمًا بالنسبة لك بسرعة واحدة: سرعة الضوء. إذا كنت تقود شمالًا على الطريق الساحلي بسرعة 200 م/ث، فإن دراجة نارية تسير جنوبًا بسرعة 300 م/ث، وبمجرد مرور بعضكما البعض تقومان بتشغيل المصابيح الأمامية، فيتحرك الضوء بعيدًا عن كلاكما بنفس السرعة، سرعة الضوء، حتى لو قمت بقياس وإذا كانت الدراجة النارية. وذلك على الرغم من أن السرعة النسبية بينكما 500 م/ث. وسيكون الأمر نفسه حتى لو كانت السرعة بينكما 0.9 درجة مئوية.

    ولذلك، فإن جاك دائمًا ما يكون في السرعة C بالنسبة للضوء، وكذلك جيل. لكن كلاهما في راحة تامة بالنسبة لهما.

    4. جيل تتحرك بالنسبة لجاك بسرعة قريبة من سرعة الضوء، ولكن بالنسبة لها فهي في حالة سكون، وجاك هو الذي يتحرك. لا يمكن تفضيل نظام جاك على نظام جيل، أو العكس. هذه هي بالضبط المسلمة الأولى للنسبية.

    5. مبدئي للغاية. لأنه إذا لم يكن هناك فرق بين الأنظمة، وكانت نسبة دوران الساعة لجاك هي 1:1، فستكون هذه أيضًا النسبة لجيل.

    تذكر أن كلاهما في حالة راحة تامة. التحذير الآخر الوحيد الذي أعرفه في الحساب هو السرعة النسبية لنظام الإشعاع الكوني. لكن هذا ليس موضوعنا، ولا صلة له بالموضوع في الوقت الراهن. نحن في عام 1905، لا يوجد إشعاع.

    6-7. لا توجد سرعة في الفضاء لنظام لا يتسارع، بل فقط سرعة بالنسبة لنظام آخر. وما ثبت في تجربة الطائرة هو أن الزمن يطول في نظام متسارع. وهذا ممكن إذا تعاملت مع الوقت كعدد من الجسيمات لكل وحدة حجم، وفقًا لمناقشتي مع أحد الطلاب.

  241. ر.ح. محبوب.
    رونان، إيه؟ ببطء وحذر، مثل القنافذ، سوف نكشف عن وجهك الحقيقي.

    3. الراحة: جاليليو، نيوتن، أينشتاين. أي جسم لا يتحرك بتسارع فهو في حالة سكون. قد يكون متحركًا بالنسبة للأنظمة الأخرى، لكن فيما يتعلق بنظامه فهو في حالة سكون.

    الضوء: يتحرك دائمًا بالنسبة لك بسرعة واحدة: سرعة الضوء. إذا كنت تقود شمالًا على الطريق الساحلي بسرعة 200 م/ث، فإن دراجة نارية تسير جنوبًا بسرعة 300 م/ث، وبمجرد مرور بعضكما البعض تقومان بتشغيل المصابيح الأمامية، فيتحرك الضوء بعيدًا عن كلاكما بنفس السرعة، سرعة الضوء، حتى لو قمت بقياس وإذا كانت الدراجة النارية. وذلك على الرغم من أن السرعة النسبية بينكما 500 م/ث. وسيكون الأمر نفسه حتى لو كانت السرعة بينكما 0.9 درجة مئوية.

    ولذلك، فإن جاك دائمًا ما يكون في السرعة C بالنسبة للضوء، وكذلك جيل. لكن كلاهما في راحة تامة بالنسبة لهما.

    4. جيل تتحرك بالنسبة لجاك بسرعة قريبة من سرعة الضوء، ولكن بالنسبة لها فهي في حالة سكون، وجاك هو الذي يتحرك. لا يمكن تفضيل نظام جاك على نظام جيل، أو العكس. هذه هي بالضبط المسلمة الأولى للنسبية.

    5. مبدئي للغاية. لأنه إذا لم يكن هناك فرق بين الأنظمة، وكانت نسبة دوران الساعة لجاك هي 1:1، فستكون هذه أيضًا النسبة لجيل.

    تذكر أن كلاهما في حالة راحة تامة. التحذير الآخر الوحيد الذي أعرفه في الحساب هو السرعة النسبية لنظام الإشعاع الكوني. لكن هذا ليس موضوعنا، ولا صلة له بالموضوع في الوقت الراهن. نحن في عام 1905، لا يوجد إشعاع.

    6-7. لا توجد سرعة في الفضاء لنظام لا يتسارع، بل فقط سرعة بالنسبة لنظام آخر. وما ثبت في تجربة الطائرة هو أن الزمن يطول في نظام متسارع. وهذا ممكن إذا تعاملت مع الوقت كعدد من الجسيمات لكل وحدة حجم، وفقًا لمناقشتي مع أحد الطلاب.

    لا تنس أنني لا أحاول دحض تمدد الزمن، بل أشير فقط إلى تعارضه مع نظرية الانفجار الأعظم. حاول أن ترى الأمر بهذه الطريقة:

    1. وفقا للانفجار، هل هناك أي معنى على الإطلاق لسؤال ما كان قبل 1000 سنة من الانفجار؟ ماذا حدث قبل 20 مليار سنة؟ أم أن مفهوم "الزمن" ذاته لم يكن محددًا قبل الانفجار؟

    2. وفقا للنظرية النسبية عام 1905، هل هناك مشكلة في سؤال ماذا حدث قبل 100 مليار سنة؟

    على أية حال، إذا عدنا إلى مناقشاتنا، فلا مفر من الاستنتاج بأن صورة لساعتي درجة الحرارة والسيزيوم ستظهر نفس الوقت في كليهما، في كل من جاك وجيل. إذا لم يكن الأمر كذلك، أرني أين.

    الكازينوهات: لا توجد مشكلة خاصة لدى الموزع في حساب 4 أو 8 أو 800 مجموعة من أوراق اللعب. المشكلة في القدرة على البقاء. في الواقع، هناك طريقة تم تكييفها خصيصًا للأماكن التي تحتوي فقط على طاولات مكونة من 6 طوابق أو أكثر.

    خلاصة القول: ليس صحيحا أن الكازينو يفوز دائما، ولكن الصحيح أن المقامر يخسر دائما. حتى مع أنه لديه ميزة على المنزل.

    إلا إذا كنت مهتمًا بتكريس حياتك للآيس أوفست والجلوس على الطاولات واستنشاق الدخان في الغالب.

    اليوبيل.

    "لست بحاجة إلى أن تكون باردًا لتخوض معركة جيدة".
    بل من الممكن التخلي عن مسألة الأمر الواقع، والانتقال مباشرة إلى العلم. لقد شرحت لك الإشكالية الموجودة في تفسيرك لتجربة MM. (أعتقد أنك قصدت تفسير لورنتز، الانكماش).

    سؤال: لماذا لا تأخذ بالتفسير الأبسط، وهو أن سرعة الضوء بسيطة بالنسبة لمصدر الضوء؟ هذا من شأنه أن يفسر نتائج تجربة M-M بسهولة، أليس كذلك؟

  242. أينشتاين
    أينشتاين
    أينشتاين
    انهم على حق
    أقترح أن يرسل لنا النظام، نحن المشاركين الذين اجتازوا اختبار اللغة النظيفة، قائمة الكلمات التي بها مشاكل، وسنتعهد بإبقائها سراً 🙂

  243. الأب، مرحبا بعودتك. إذا كان هناك تجارب، يرجى المشاركة.

    آر إتش،
    تم حظر تعليقاتي عدة مرات، لكن إسرائيل ليست كلمة رئيسية ذات صلة. من ناحية أخرى، فإن قريب المغني المخضرم الذي أعطى اسمه لمعالج النصوص القديم يثير الشكوك.

    إسرائيل،
    لن أشعر بخيبة أمل منك أبدًا. سوف تعرف دائمًا كيفية مقاومة شعرها. والجزء الأفضل هو أننا لا نحتاج حتى إلى أن نكون هادئين حتى نخوض معركة جيدة.
    منذ أن بدأت مدونة مستقلة (بناء على نصيحة أحد الطلاب، التخنيون المشهورة)، سيطر علي رباطة جأش ولم أقم بتحميل أي شيء جديد. لكن قريبا إن شاء الله قريبا. أنا أعمل على الأشياء

  244. متشكك، يوفال لديه قاموس خاص وعليك أن تفرق بين ما يقوله يوفال وما يحدث بالفعل.
    عندما يقول يوفال: "أنت فقط لم تفهم" - فهو يعني "لم يفهم أحد".
    "أنت مراوغ" - يعني "لم تجب على ما أردت منك أن تجيب عليه".
    "استمتع بالأضواء" - المعنى هو "لماذا لا يتناول أحد القضية المهمة حقًا: نموذج يوفال".
    "غسالة الدماغ" - من الممكن أن تقنع شخصًا ما بما لا يصدقه يوفال أو لا يفهمه.
    "كلمة القواد" - على النحو الوارد أعلاه.

    "عدم المطابقة" - عسر القراءة.

    وبشكل عام، ليس فقط فيما يتعلق بيوفال: أولئك الذين يبدأون العبث معك شخصيا، بدلا من آرائك، لا يبحثون حقا عن نموذج علمي، بل عن نموذج الأنا. وإلا فإنه سيتركك لأجهزتك الخاصة وينتقل إلى التعليق التالي.
    وإلى جانب ذلك، يوفلي لطيف جدًا على الرغم من كل شيء.

  245. آفي بيليزوفسكي،
    إذا كانت شكوكي وشكوك إسرائيل صحيحة بالفعل أن اسمه واسم بلدنا والاسم الثاني ليعقوب أبينا كلمة محرمة في العلم، فأنا أعترض بشدة!

    إسرائيل،
    1) موافق
    2) موافق
    3) لا أعرف ما هي الراحة. لقد حددنا سرعة جاك بأنها 0 بالنسبة لسرعة الضوء أو جيل أو كوكب زحل أو الذبابة التي تحلق حوله.
    4) لا أعرف ما هي الراحة. لقد وضعنا جيل في سرعة قريبة جدًا من سرعة الضوء لدرجة أن مليارات سنوات جاك قد مرت في زمنها.
    5) لا أعرف ما هو من حيث المبدأ. جيل يتحرك بشكل أسرع من جاك. هل هو أساسي؟ الى جانب ذلك، فهي شقراء وهو آسيوي. هل هو أساسي؟
    6-7) لا يفهمون الأسئلة. وكما ذكرنا فإن اختلاف السرعة في الفضاء يؤدي إلى تباطؤ في سرعة الزمن الذي مضى. لا أفهم سبب إصرارك الشديد على هذا، بعد كل شيء تم إثباته في تجربة ساعة السيزيوم.

    روابط لجاليليو؟ هناك من يعرف كيفية عد البطاقات وهناك من يجد أي شيء، حتى ما هو مدفون في أقبية الإنترنت المظلمة. بالمناسبة، اعتقدت أنه منذ بيل كابلان من معهد ماساتشوستس للتكنولوجيا، لم يعد من الممكن عد البطاقات لأن الكازينوهات تستخدم عددًا كبيرًا من الحزم. أليس كذلك؟

  246. الوقت ذو صلة/مهم عندما تكون هناك أي حركة للأجسام أو الذرات أو أجزائها الأصغر.
    أما بالنسبة للإنتروبيا، فهي ذات صلة طالما أن هناك بنية جزيئية ما في الكون، لأنه بمجرد أن يصبح الحساء الكوني متطابقًا وموزعًا بالتساوي، فلن يكون له أي معنى.

  247. الوقت هو البعد الذي يقف من تلقاء نفسه،
    ويتم ربطه بكل نظام أبعاد بطريقة مختلفة.

    لقد عرفنا حتى الآن الوقت الذي يتحرك للأمام،
    سنكتشف لاحقًا الأنظمة التي يتصرف فيها الزمن بطريقة مختلفة.

  248. محاولة تسلل جزء من الاستجابة قبل أن تغفو:

    ر.ح.

    لقد رأيت للتو تعليقاتك. يبدو لي أن ذكر اسمي يثير غضب محاكم التفتيش. حاول بعد ذلك استخدام أمراض الكود. جرب "Hatzvi" أو "Yishuron" أو باختصار "جيش الدفاع عن.." سأفهم بالفعل أنك تقصدني وانتقل مباشرة إلى التعليق التالي.

    يبدو لي أنني بدأت أفهم جذور سوء التفاهم بيننا. الدافع وراء ذلك هو الجملة "النسبة بين ساعات جيل ستكون دائمًا نفس دالة سرعتها بالنسبة إلى حالة السكون أو إذا صح التعبير بالنسبة إلى سرعة الضوء."

    فيما يلي الأسئلة السقراطية المعتادة:

    1. هل نتفق على أن نسبة الدوران بين ساعات درجة الحرارة وساعات السيزيوم في منزل جيل هي 1,000,000,000:1؟

    2. هل نتفق على أن نسبة الدوران بين ساعتي درجة الحرارة وساعة السيزيوم عند جاك هي 1:1؟

    3. هل تعتقد أن جاك في حالة راحة؟

    4. هل تعتقد أن جيل في حالة راحة؟

    5. هل هناك فرق جوهري بين نظام جاك ونظام جيل؟ نعم لا

    6. إذا كانت الإجابة على الرقم 5 لا، فلماذا توجد مثل هذه الفجوة في العلاقة؟ لماذا لا يتم عكس الوضع فيما يتعلق بالدورات، لماذا لا تكون نسبة جيل 1:1 أو ما بينهما، 100:1 على سبيل المثال، كيف يمكن أن تتطابق نسبة جيل تمامًا مع سرعة جاك بالنسبة لها ؟

    7. إذا كانت الإجابة على الرقم 5 بنعم، فاشرح الفرق. ماذا تقصد بقولك: "بسرعته بالنسبة إلى حالة السكون أو إذا صح التعبير إلى سرعة الضوء". لماذا يختلف وضع جاك عن وضعها؟

    تحدث عن البطاقات البريدية، نحن جميعًا إخوة هنا. لا تظلم ولا تنقرض. فقط تذكر، ناهيك عن الاسم الصريح.

    بالمناسبة - من أين حصلت على الروابط الخاصة بجاليليو؟ ولم أكن أعلم بوجودهم. كالعادة، لم يخبرني أي شيء.

  249. ر.ح.

    لقد رأيت للتو تعليقاتك. يبدو لي أن ذكر اسمي يثير غضب محاكم التفتيش. حاول بعد ذلك استخدام أمراض الكود. جرب "Hatzvi" أو "Yishuron" أو باختصار "جيش الدفاع عن.." سأفهم بالفعل أنك تقصدني وانتقل مباشرة إلى التعليق التالي.

    يبدو لي أنني بدأت أفهم جذور سوء التفاهم بيننا. الدافع وراء ذلك هو الجملة "النسبة بين ساعات جيل ستكون دائمًا نفس دالة سرعتها بالنسبة إلى حالة السكون أو إذا صح التعبير بالنسبة إلى سرعة الضوء."

    فيما يلي الأسئلة السقراطية المعتادة:

    1. هل نتفق على أن نسبة الدوران بين ساعات درجة الحرارة وساعات السيزيوم في منزل جيل هي 1,000,000,000:1؟

    2. هل نتفق على أن نسبة الدوران بين ساعتي درجة الحرارة وساعة السيزيوم عند جاك هي 1:1؟

    3. هل تعتقد أن جاك في حالة راحة؟

    4. هل تعتقد أن جيل في حالة راحة؟

    5. هل هناك فرق جوهري بين نظام جاك ونظام جيل؟ نعم لا

    6. إذا كانت الإجابة على الرقم 5 لا، فلماذا توجد مثل هذه الفجوة في العلاقة؟ لماذا لا يتم عكس الوضع فيما يتعلق بالدورات، لماذا لا تكون نسبة جيل 1:1 أو ما بينهما، 100:1 على سبيل المثال، كيف يمكن أن تتطابق نسبة جيل تمامًا مع سرعة جاك بالنسبة لها ؟

    7. إذا كانت الإجابة على الرقم 5 بنعم، فاشرح الفرق. ماذا تقصد بقولك: "بسرعته بالنسبة إلى حالة السكون أو إذا صح التعبير إلى سرعة الضوء". لماذا يختلف وضع جاك عن وضعها؟

    تحدث عن البطاقات البريدية، نحن جميعًا إخوة هنا. لا تظلم ولا تنقرض. فقط تذكر، ناهيك عن الاسم الصريح.

    بالمناسبة - من أين حصلت على الروابط الخاصة بجاليليو؟ ولم أكن أعلم بوجودهم. كالعادة، لم يخبرني أي شيء.

    اليوبيل.

    "أنت مراوغ مرة أخرى، أيها الأنقليس الصغير، لكنني معتاد على ذلك."
    لهذا السبب كنت تعتقد أنني برج الحوت، بسبب ثعبان البحر، هاه؟

    لا. أعتقد أنك الشخص الذي يتجنب ذلك. اشرح لنا لماذا الزمن والقانون ب هما على ما هما عليه. لقد أثرت هذه النقطة.

    متشكك.

    نقطة مثيرة للاهتمام، ولكن بعد شهرين و900 رد، أليس من الأفضل أن نركز على القضايا المفتوحة قبل الانتقال إلى جبهات إضافية مثل الإدراك؟ سنترك ذلك للمقالات القادمة. ومع ذلك، يستغرق الأمر مني 5 دقائق للحصول على ردود من معظم التحميل.
    ولكنك مرحب بك بالتأكيد للبقاء معنا في كوزمو. يبدو أن لديك ما تقوله.

  250. سفكان ياكار، شكرا لاهتمامكم.
    يقوم Y.S بعادة منتظمة تتمثل في تجنب الأسئلة التي لا تناسب جدول أعماله. ربما يفهم أنني حاولت خداعه، لأنه لا يفتقر إلى الذكاء على الإطلاق. كل ما حاولت إظهاره، على العكس من ذلك، هو أن الوقت هو البعد الثابت الذي سيرتكز عليه كل النظام العالمي (والفوضى). لقد ظل لعدة أشهر يلعب بمعادلات النسبية التي تقدم الزمن ككمية متغيرة، وتوصل إلى مفارقات يقدمها لنا بصوت متواضع بطولي متجاهلاً الاحتمال "الصغير" بأن نتائج تجربة مايكلسون مورلي قد تكون لديك تفسير مختلف عن التفسير المقبول.
    وغني عن القول أن الأشياء التي تقولها جميلة بالنسبة لي ومقبولة بالنسبة لي.

  251. ي.ح. إسرائيل وكل الآخرين.

    الزمن، مثل أبعاد المكان، هي مفاهيم أنتجها إدراكنا بطريقة تطورية من أجل البقاء.
    إن كون الفيزياء *ملكية* لمفهوم الزمن لغرض نماذجها لا يعني أنها تملكه فعلياً.

    وبالتالي فإن مفهوم الزمن، كوسيلة لإدراك الواقع، لا يتم تعريفه بالضرورة بالإنتروبيا وحدها، أو بمفهوم فيزيائي آخر. كل عملية (مثل حركة الأشياء في بيئة الإنتاج الحي) تتطلب إدراكا للزمن، تماما كما تتطلب إدراكا للمكان. (بدون هذه المنظورات لن تحصل أشكال الحياة على موارد الحياة التي تحتاجها ولن تكون قادرة على الهروب من التهديدات التي تهدد وجودها).

    وفي رأيي أنه ينبغي التعامل مع مفهوم الزمن كمفهوم بدائي لا يحتاج إلى تفكير، كما أن مفاهيم الفضاء الثلاثي الأبعاد هي مفاهيم بدائية خالية من التفكير. في تقديري، أي محاولة لتعريف المفاهيم البدائية للمكان والزمان ستؤدي إلى مفارقات في التعريفات، ونوع من التمهيد في التعريفات، حيث أن الظواهر الفيزيائية المختلفة تستخدم ضمنيًا مفاهيم المكان والزمان (وبالتالي لا يمكن استخدامها لتفسير المفاهيم البدائية). الغرض من تحديد المفاهيم الأخيرة). وحقيقة أن الزمن غير محدد (أي يتم التعامل معه كمفهوم بدائي) لا يمنع خيار قياسه، على الأقل لأغراض عملية (من وجهة نظري، فإن علم الكونيات في الكون هو فرضية غير عملية، ولكن ذلك فهذه قصة أخرى).

    فيما يتعلق بزيادة الانتروبيا. من المحتمل أن يكون موجودًا في جميع الأنظمة الديناميكية التي يمكن فهمها، نظرًا لأن عكس اتجاه الإنتروبيا يعني عمليات يصعب إدراكها (وبالتأكيد يصعب تحليلها في النماذج الرياضية).
    لا علاقة لها بقانون فيزيائي أو آخر، يمكن تعريف الإنتروبيا على الأنظمة غير المادية أيضًا.

    أنا كسول جدًا لأقول المزيد عن الإنتروبيا، فهي معقدة جدًا وفلسفية، ومن المستحيل أيضًا تقديم تفسير كامل لما أعنيه. قول هذا فقط: الزيادة في الإنتروبيا على الخط الزمني تتوافق مع وصف السبب والنتيجة من النوع "*سبب واحد* يؤدي إلى العديد من النتائج"، من ناحية أخرى، فإن انخفاض الإنتروبيا على الخط الزمني يتوافق مع سبب -وصف التأثير من النوع "أسباب متعددة *متزامنة معًا* تؤدي إلى نتيجة واحدة". من الصعب جدًا إدراك وصف السبب والنتيجة للنوع الثاني، إذا كان ذلك ممكنًا.

  252. آري - أنا سعيد لأنك وصلت إلى نهاية تفكيري.
    من يدري، ربما في المركز توجد فقاعة تنتج أكوانًا وهذا ما يعطي قوة دافعة لتوسع الأكوان.

  253. روبي - أصبح من الواضح الآن من أين حصلت على البصل. تقولون عن البالون الموجود داخل بالون داخل بالون "هل هناك دليل على أن الأمر ليس كذلك؟" هذه الجملة يمكن أن تقال عن أي افتراض... هناك وحش السباغيتي الطائر؛ هل هناك أي دليل على أنه ليس كذلك؟ وأنا أقول: إذا كان هناك أكثر من بالون - فكل بالون هو كون مختلف (حتى لو كانوا داخل بعضهم البعض)!

  254. أنت مراوغ مرة أخرى، أيها الأنقليس الصغير، لكنني معتاد على ذلك.
    ومن الأساطير مثلاً: لا! الوقت والسرعة ليسا نفس الشيء ولكن الكلمة والعكس صحيح. لكن هذه ليست الطريقة التي تسير بها الأمور مع الوقت والإنتروبيا، حيث أنها تتقدم معًا.

  255. آسف لقد ابتعدت.
    كما يقول الحمار لشريك،
    مثال؟ (ماذا تقصد).
    بعد كل شيء، إذا قمت بإزالة الوقت من الصيغ، فلن يكون لديك السرعة. هل الوقت والسرعة نفس الشيء؟

  256. يوجد
    بما أنك تتجنب التقاط القفاز، سأفعل ذلك من أجلك. ادعائي هو أن "الزمن" و"القانون الثاني للديناميكا الحرارية" هما اسمان لنفس الشيء تمامًا. أزل الوقت من الصيغ ولن يكون لديك القانون الثاني للديناميكا الحرارية. ستتجاهل التغيرات في حالة النظام في الكون ولن تتمكن من قياس الوقت

  257. - في نهاية القرن التاسع عشر - بدأ البروفيسور - اعتقد الفيزيائيون أن عملهم قد انتهى تقريبًا.
    - أعطت قوانين الميكانيكا الإنجليزية لنيوتن، والكهرومغناطيسية الاسكتلندية ماكسويل، وصفًا مثاليًا تقريبًا للعالم المادي كتركيبة من الموجات والمادة.
    - كل شيء موجات - تمتم بالقانون.
    - الأمواج والأجسام - صححه الأستاذ.
    - الأمواج والموجات المضغوطة - أصر القانون.
    - وتابع البروفيسور: - الديناميكا الحرارية، بقانونيها القويين، تحولت مع اكتشاف الذرات إلى فرع من الميكانيكا الحركية، والآن، كما كان يأمل الجميع، يمكننا جميعًا الاسترخاء في العالم المريح والمنظم الذي رتبه السيدان البريطانيان. بالنسبة لنا، واحتساء كوب من الشاي بعد العشاء.
    - آه، تلك كانت الأوقات، لقد كان زمنًا حقيقيًا - تنهد الأستاذ ومسح حبات الحنين عن جبهته. - ضع في اعتبارك حجم الشاعرة، سحر الكون حيث المكان مطلق والزمن مطلق وقارنهما بالفوضى السائدة اليوم.
    - يقول جدي إن الموجة في مسقط رأسه كانت موجة، وكان الجسيم جسيمًا، ولم تجرؤ أي موجة على التصرف كجسيم أو العكس. كانت الإلكترونات تدور بسعادة حول النواة، واضحة، حادة، قابلة للقياس، في مسارات بيضاوية مثالية. لا تزال هناك بعض الأمور الصغيرة التي يجب تسويتها. إشعاع الجسم الأسود، وسرعة الأرض عبر الأثير - ليس في الحقيقة صيد الدببة والأسود، بل على الأكثر مطاردة كسول لفئران الحقل.
    - حسنا، ثم ماذا حدث؟ كان القانون مهتما.
    - سرعة الضوء رفضت التعاون - أجاب الأستاذ بفظاظة.
    - ماذا يعني ذلك؟
    - لم توافق على التلخيص.
    -ربما يكون من المفيد أن أوضح للقانون الفقير والغبي ما تقصده؟ غضب القانون.
    سمع هدير الموافقة من الجمهور.
    - لم يتفق على التلخيص الاتجاهي كأي سرعة أخرى. أظهرت تجربة ميشيلسون ومورلي أن سرعة الضوء ثابتة دائمًا، بغض النظر عن سرعة مصدر الضوء، وفي الواقع بغض النظر عن أي عامل.
    - وما المخيف في ذلك؟ القانون مرتبك.
    -ما هو السيء للغاية؟ الآن الأستاذ هو الذي غضب. - هل لا تفهم شيئا؟ كان كل شيء جاهزًا وجاهزًا بالفعل لنظرية موحدة كبرى بسيطة ومفهومة للفيزياء الكلاسيكية. كل ما أردنا معرفته هو سرعتنا في الموقع. هل هذا كثير على لاطلبه؟ التفت إلى الجمهور بصوت يبكي.
    وارتفعت تنهدات مكتومة من الحشد.
    - لكن لا، لم توافق المغنية الأولى على التسوية تحت أي شروط. تعرفون كم طلبنا، وكم توسّلنا... وفي النهاية، خطرت فكرة تقديم البائس إلى العدالة، لكنها لم تتحقق.
    -ماذا حدث؟
    - أعلن أينشتاين أنه نظرا للظروف الصعبة تقرر إلغاء الموقع، وتم الاتفاق على أن سرعة الضوء هي الثابت المطلق الوحيد في الكون. كل شيء آخر نسبي.
    -ما هو النسبي بالضبط؟
    - جميع العوامل الفيزيائية: الزمن، المسافة، الكتلة - كل شيء.
    -البروفيسور بولتزمانسكي - قال القانون بتشديد. هل من الممكن تجاوز سرعة الضوء؟
    -لا. هو الحد الأعلى.
    -لماذا؟
    - لأن السرعة الأكبر من سرعة الضوء ستكون تناقضاً مع... صمت الأستاذ.
    -تناقض لماذا؟ اعتقدت أنك قلت أنها كانت المطلقة الوحيدة.
    تمتم الأستاذ بكلمات غير مفهومة.
    -هل ستجيب على السؤال أم نعلنك شاهدا معاديا؟
    - تناقض القانون الثاني..
    -ما القانون الثاني؟ من الميكانيكا؟ الكهرومغناطيسية؟
    - الديناميكا الحرارية - همس الأستاذ.
    - والوقت؟ القانون يجلد بلا رحمة - كيف يتم تحديد اتجاه الزمن؟
    - اتجاه زيادة الإنتروبيا - إلى الأستاذ.
    - هل هي زيادة؟ وطالب القانون.
    - القانون الثاني للديناميكا الحرارية..
    -بروفيسور بولتزمانسكي - لقد تحدث القانون بلطف الآن - لم تعد متهمًا، وأنا على وشك إطلاق سراحك من المنصة، ولكن يجب أن تتذكر شيئًا واحدًا للأبد:
    - يوجد ثابت واحد فقط في الفيزياء وهو القانون الثاني للديناميكا الحرارية.
    كل شيء آخر نسبي.
    ابتسم الأستاذ مع وميض من الفهم المفاجئ.
    -هل تعلمنا الدرس اليوم؟
    أومأ الأستاذ برأسه.
    - لأنني لا أريد أن نضطر إلى تكرار هذا الدرس مرة أخرى.
    هز الأستاذ رأسه بالقانون ونزل من المنصة وسط تصفيق الحضور.

  258. وبافتراض صحة نظرية الانفجار الأعظم، فإن الكون يتوسع من المركز إلى الخارج.
    يصف تشبيه البالون كرة ذات طبقة واحدة تنتفخ إلى ما لا نهاية مع تسارعها.
    في رأيي أن جدار البالون له سمك وربما يكون عبارة عن عدد من البالونات القابلة للنفخ داخل بعضها البعض بحيث ينتشر الخارجي بشكل أسرع من الداخلي وهكذا باتجاه المركز.
    هل هناك أي دليل على أن هذا ليس هو الحال؟

  259. اليوبيل.
    الفلسفة ليست ثرثرة. إن الجمع بين الفيزياء والفلسفة ناجح جدًا في رأيي. ببساطة، إنه خارج نطاقنا في الوقت الحالي. هذا كل ما نفتقده الآن لرد 900، وهو اتجاه آخر للترفيه عنه.

    في رأيي، إذا قبلنا نظرية الانفجار الأعظم، فإن الزمن هو بالفعل عامل فيزيائي مطلق، يتأثر بالتسارع ولكن ليس بالحركة غير المتسارعة، ويمكن قياسه كدالة لتوسع الكون. قبل الانفجار، لم يكن لمفهوم الزمن أي معنى. لا يوجد شيء اسمه "ما حدث قبل 20 مليار سنة".

    ومن ناحية أخرى، إذا عدنا إلى عام 1905، فإن صورة الوضع هي كون أبدي، أو شبه أبدي. بالنسبة لبوانكاريه، فإن 100 مليار سنة هي لعبة أطفال، وهي مسألة وقت فقط قبل أن يكرر كل موقف نفسه، وتتناقص الإنتروبيا. ولذلك، في العلاقات، الوقت هو أمر ذاتي، ولكل قاس وقته الخاص، وليس هناك وقت مطلق.

    بالمناسبة، ادعاءك بأننا في مركز الكون لا تدعمه نظرية الانفجار الأعظم. ووفقا للنظرية لا يوجد شيء اسمه مركز الكون. يُسمح لكل نقطة أن ترى نفسها كمركز.

  260. روبي - لا يعني ذلك أنني خبير عظيم، لكن نموذج البصل الخاص بك غير مفهوم وغير مناسب لتقليل الأبعاد. أنت تحاول إحضار البصلة كنموذج ثلاثي الأبعاد وهذا ما لا يفهم. النموذج الذي يستحق الفهم هو نموذج ثنائي الأبعاد (سطح البالون)، فقط السطح منحني. إن كوننا هو نفسه فقط مع وجود بُعد إضافي (وليس بالضرورة كرويًا الشكل). أي أن سطح البالون يشبه الفضاء ثلاثي الأبعاد.
    يوفال: على حد علمي، لا أحد يعرف بوضوح ما الذي يسبب الطاقة المظلمة. أعتقد أن إحدى الأفكار هي طاقة الفراغ (EA)؟

  261. آريا سيتر، تعتذر عن الصياغة غير الواضحة.
    لم أكن أنوي أن أقدم لك اختبارًا للمعرفة العامة، لكني طلبت رأيك. ما الذي تعتقد أنه يسبب الطاقة المظلمة؟

  262. آريا - في رأيي، تشبيه البصل أفضل بالنسبة للحسابات المتناهية الصغر لفصل طبقات قشور البصل، أي التوسع المتسارع للكون.
    من خلال الملاحظات الفلكية، هل هناك انتقال للمجرات من قشرة إلى أخرى إذا واصلنا تشبيه البصل؟ إذا لم يكن الأمر كذلك، فيمكنك البقاء إذا كان هذا تشبيهًا، وإذا كان الأمر كذلك، فأنت بحاجة إلى التحقق من السبب.

  263. اليوبيل - استمرار الانفجار الكبير مع إضافة الطاقة المظلمة؛ لا نعم؟ روبي - لماذا البصل؟ البالون هو نموذجنا الذي تم اختصاره ببُعد واحد ليسهل علينا فهمه. المجرات لا تتحرك مع بعضها البعض مع غلاف البالون (تضخيم الفضاء)، فقط المسافة بينها تزداد مع توسع الفضاء.

  264. آري - أود أن أشبه الكون بقشر البصل الذي ينتفخ وليس بالبالون، لأن الكون ثلاثي الأبعاد مع إضافة ناقل الزمن.
    ولا تزال المجرات الموجودة على نفس الأصداف تتحرك معها بنفس السرعة (أكبر من سرعة الضوء؟) وهذا يتناقض مع بديهية أينشتاين كما أثبتها العلم حتى الآن باستثناء الانحرافات في حالات معينة.

  265. روبي - المجرات نفسها لا تتحرك؛ الكون الذي يحملهم هو الذي يتضخم. تصور بالونًا به بقع تمثل المجرات. ينتفخ البالون وتتحرك البقع البعيدة عن بعضها البعض بسرعة تفوق سرعة الضوء مع أن البقع نفسها لا تتحرك.

  266. اسد،
    اعتقدت أن سرعة الضوء كانت قصوى. كيف يمكن للمجرات التي هي مادة وليست طاقة أو فوتونات أن تتحرك بسرعة أكبر من سرعة الضوء؟
    أما بالنسبة للعمر المعروف للكون فهو حوالي 15 م سنة، وإذا كانت هناك مجرات تتجاوز مسافة 15 م سنة ضوئية وكانت نظرية الانفجار الأعظم صحيحة، فهذا يعني أن توسعها كان بالفعل يتجاوز سرعة الضوء.

  267. روبي - أبعد المجرات التي يمكن رؤيتها هي من بداية الكون. حتى المجرات التي لن يصل إلينا ضوؤها أبدًا، وحتى تلك البعيدة عنا، عمرها لا يتجاوز عمر الكون؛ لقد ابتعدوا ببساطة (وما زالوا يبتعدون) عنا بسرعة تتجاوز سرعة الضوء.

  268. مرحبا يوفال،
    صدقت، لقد خلطت بين الزمن والمسافة، لكن السؤال ما زال يطرح:
    لقد قطعت الأشعة المرصودة من المجرات البعيدة مسافة 14.5 مترًا ضوئيًا. أثناء قطع هذه المسافة، استمرت تلك المجرات في الوجود والابتعاد لفترة من الوقت لا أعرف ما هي (أود أن أعرف ما هي) ولكن لا بد أنها ضعف الوقت الذي استغرقته الضوء أشعة للوصول إلى هنا.
    ويجب أن يكون قطر الكون أكبر من 58 م سنة ضوئية بسبب تسارع التوسع كما لوحظ ونشر.
    كيف وصلت إلى 58 م سنة ضوئية؟ المسافة الأولية 14.5 م سنة ضوئية ومسافة أخرى مماثلة على الأقل بسبب التسارع مضروبًا في اثنين بسبب القطر.
    روبي

  269. يوفال - بل إن المجرات التي وراء الكون المرئي تبتعد عنا بسرعة الضوء وأكثر من ذلك - أليس كذلك؟ (موقعهم في الكون يبتعد عنا بهذه السرعات)

  270. روبي
    ويمكن أن نفهم من ادعائك أنه في اللحظة التي بدأ فيها الضوء طريقه من المجرات البعيدة إلينا، كان عمر المجرات 14.5 مليون سنة.
    أنت أيضًا تخلط بين مصطلحي "الزمن" و"المسافة" (السنة الضوئية هي وحدة مسافة وليست وحدة زمن) ومن المفهوم أنك تعتقد أن المجرات تبتعد عنا بسرعة الضوء.
    وأخيرًا، لست متأكدًا من كيفية وصولك إلى الرقم 59. هل تقصد الرقم 58؟

  271. مرحبًا آريا،
    أما بالنسبة للكون المرئي، فإذا كانت أبعد المجرات ترسل لنا أشعتها الضوئية منذ 14.5 م سنة ضوئية، فأنا أزعم أن عمر تلك المجرات ضعف أي 29 م سنة ضوئية لأنه في الوقت الذي استغرقته الأشعة للوصول إلى هنا كانت موجودة بل وانتشرت لنفس الفترة الزمنية تمامًا، أي 14.5 مترًا سنة ضوئية إضافية.
    أي أن قطر الكون حتى الآن (كما سيظهر بعد 14.5 م سنة ضوئية) هو 59 م سنة ضوئية.
    هل أنا مخطئ؟
    روبي.

  272. ارييه سيتر
    وفقا للنموذج السائد لـ "الانفجار الكبير" فإن عمر الكون وحجمه هما دالتان عكسيتان لبعضهما البعض (بالنظر إلى عمر الكون يمكن حساب حجمه، وبالنظر إلى حجمه يمكن حساب حجمه). عمر).
    ومن ناحية أخرى، أشاركك الرأي الذي عبرت عنه في عبارة "حقيقة أن الكون أكبر بكثير من الكون المرئي، لا تعني أن عمر الكون بالسنوات الضوئية هو حجم الكون". السبب الذي يجعلني لا أحب نموذج "الانفجار الكبير" هو أن الافتراض بوجود انفجار كبير بالفعل تفرقت منه المادة في الكون في كل الاتجاهات دون تفضيل اتجاه واحد على الاتجاهات الأخرى، بالإضافة إلى الملاحظات التي تظهر إن أفق الكون على مسافة ثابتة منا في كل الاتجاهات، يضعنا في مكان جيد في منتصف الكون، حيث حدث الانفجار الأعظم.

  273. إسرائيل،
    يبدو أنه قد أسيء فهمي مرة أخرى، كالعادة. لم أقصد التحدث عن الفلسفة. لقد تعلمنا من النسبية أن الزمن ليس ثابتًا (لأن سرعة الضوء ثابتة)، لذلك يمكن اعتباره عاملًا فيزيائيًا متغيرًا. إذا قلنا حتى اليوم أن الإنتروبيا (في الماكرو) تزداد مع مرور الوقت، والافتراض الحالي هو أن الوقت هو كيان مادي، فإن الوقت نفسه هو الذي يسبب كل هذه الفوضى. وحتى يومنا هذا، عادة ما تعرف البشرية السرعة على أنها مشتقة لشيء ما (المسافة مثلا) بناء على الزمن، ولكن بما أن الزمن ليس هو الثابت الحقيقي (بل سرعة الضوء كما ذكرنا)، فيجب تغيير موضوع الصيغة. وإعادة تعريفها وفقا لذلك. هل ستتمكن، في ظل هذه الظروف الجديدة، من إظهار كيف يقضم الوقت ("أسنان الزمن") النظام الجيد؟

  274. اصدقاء؛ أنت لم تجب على ما طلب روبي. إنه يخلط بين عمر الكون وحجم الكون. يمتد الكون المرئي إلى مسافة حوالي 14 مليار سنة ضوئية في كل اتجاه - لأنه من المستحيل رؤية شيء أبعد بسنوات ضوئية من عمر الكون لأن ما هو أبعد من ذلك - لن تصل إلينا أشعة الضوء أبدًا - بسبب تسارع التوسع. لكن حقيقة أن الكون أكبر بكثير من الكون المرئي، لا تعني أن عمر الكون بالسنوات الضوئية هو حجم الكون.

  275. وبسبب التوترات في الجنوب، أصبحت فترة انتظار الردود أطول. سنحاول التكامل في أجزاء. وإذا ظهرت نفس الأشياء مرة ثانية قريباً، فإن اللوم يقع على العجلة والانتظار.
    يذكرنا قليلاً بالانتظار قبل حرب الأيام الستة.

    يوفال
    "لقد اتضح أن القانون الثاني للديناميكا الحرارية موجود في غمضة عين."

    القانون الثاني هو أبو ليس ضررا صغيرا. إذا قمت بالتحقق بعناية، خلاصة القول هي أنه هو المسؤول عن أي مشكلة.
    لا يوجد الخير والشر. لا يوجد سوى النظام والانتروبيا. كل إنسان، صالحًا كان أم سيئًا، إذا كان الأمر يعتمد عليه، سيختار الخير لابنه. فإذا اختار الإنسان الشر لنفسه أو لغيره، فما ذلك إلا بسبب الظروف القاسية، التي تستند إلى القانون الثاني للديناميكا الحرارية.

    "هل يمكنك إظهار كيف أن الوقت ليس مجرد مفهوم أثيري يتم من خلاله قياس العمليات ("مع مرور الوقت"، "مع مرور الوقت"، وما إلى ذلك) ولكنه يسببها بشكل مستقل (على سبيل المثال، "سوف يأخذ الوقت مجراه")؟"

    العامل الفلسفي يثير اهتمامي أقل بكثير من العامل المادي. من الصعب جدًا تحديد الوقت في الفيزياء. أعتقد أن معالجة الزمن تختلف في النظريتين المقبولتين اليوم، الانفجار الأعظم والنسبية. وفقا للانفجار الكبير، لم يكن هناك وقت على الإطلاق قبل 13.7 مليار سنة (ما الذي كان موجودا قبل مليار سنة من الانفجار الكبير؟ ليس سؤالا لا معنى له؟). ومن ناحية أخرى، عندما تصور أينشتاين النسبية في عام 1905، كانت صورة الكون هي كون لا نهائي وأبدي. ومن هنا الاصطدام.

    في مناقشتي مع ر.ه. ويكرر الحجة التالية: "إنه مقياس حرارة يقيس معلمة خارج النظام". أحاول أن أجعله يرى التناقض الموجود بين البيانات المختلفة، وأن الحل هو أن المعلمة موجودة داخل النظام.

    فيما يتعلق بالإعفاء – الإلغاء ليس طلبًا صريحًا. يبدو لي أنك تنسجم بشكل جيد مع جميع الشركات الأخرى (أو ربما لا حقًا؟ بالمقارنة بما يقولون، أنا قطعة من العسل). لكني مازلت أعتقد أن الجميع هنا مهتم بسماع استمرارية النموذج، ومتى يخرج من مرحلة التعريفات إلى مرحلة النظرية والبراهين والصيغ واختبارات التفنيد.

  276. يوفال
    "لقد اتضح أن القانون الثاني للديناميكا الحرارية موجود في غمضة عين."

    القانون الثاني هو أبو ليس ضررا صغيرا. إذا قمت بالتحقق بعناية، خلاصة القول هي أنه هو المسؤول عن أي مشكلة.
    لا يوجد الخير والشر. لا يوجد سوى النظام والانتروبيا. كل إنسان، صالحًا كان أم سيئًا، إذا كان الأمر يعتمد عليه، سيختار الخير لابنه. فإذا اختار الإنسان الشر لنفسه أو لغيره، فما ذلك إلا بسبب الظروف القاسية، التي تستند إلى القانون الثاني للديناميكا الحرارية.

    "هل يمكنك إظهار كيف أن الوقت ليس مجرد مفهوم أثيري يتم من خلاله قياس العمليات ("مع مرور الوقت"، "مع مرور الوقت"، وما إلى ذلك) ولكنه يسببها بشكل مستقل (على سبيل المثال، "سوف يأخذ الوقت مجراه")؟"

    العامل الفلسفي يثير اهتمامي أقل بكثير من العامل المادي. من الصعب جدًا تحديد الوقت في الفيزياء. أعتقد أن معالجة الزمن تختلف في النظريتين المقبولتين اليوم، الانفجار الأعظم والنسبية. وفقا للانفجار الكبير، لم يكن هناك وقت على الإطلاق قبل 13.7 مليار سنة (ما الذي كان موجودا قبل مليار سنة من الانفجار الكبير؟ ليس سؤالا لا معنى له؟). ومن ناحية أخرى، عندما تصور أينشتاين النسبية في عام 1905، كانت صورة الكون هي كون لا نهائي وأبدي. ومن هنا الاصطدام.

    في مناقشتي مع ر.ه. ويكرر الحجة التالية: "إنه مقياس حرارة يقيس معلمة خارج النظام". أحاول أن أجعله يرى التناقض الموجود بين البيانات المختلفة، وأن الحل هو أن المعلمة موجودة داخل النظام.

    فيما يتعلق بالإعفاء – الإلغاء ليس طلبًا صريحًا. يبدو لي أنك تنسجم بشكل جيد مع جميع الشركات الأخرى (أو ربما لا حقًا؟ بالمقارنة بما يقولون، أنا قطعة من العسل). لكني مازلت أعتقد أن الجميع هنا مهتم بسماع استمرارية النموذج، ومتى يخرج من مرحلة التعريفات إلى مرحلة النظرية والبراهين والصيغ واختبارات التفنيد.

    محاكمة القانون الثاني.

    1. بولتزمانسكي.

    لكن المحاكمة هي كلام شفهي.
    في الواقع، ليس من المألوف أن يواجه قانون ما محاكمة جنائية... وليس أي قانون: القانون الثاني للديناميكا الحرارية، والذي ربما يكون القانون الأساسي بين جميع قوانين الفيزياء.
    ولم تكن محاكمة بالمعنى المعتاد للكلمة. أما القانون الثاني فلم يحضر إلى قاعة المحكمة وهو مقيد اليدين، ولم يصدر أي أمر بالقبض عليه، والأهم من ذلك أن القاضي لم يحضر المحاكمة أيضاً!
    القانون الثاني وضع الأمور في نصابها الصحيح منذ البداية: كشرط للموافقة على المثول أمام المحكمة، طالب القانون - وحصل - على الحقوق الكاملة لكل من محامي الدفاع والمدعى عليه. ستكون هناك محاكمة بالفعل، لكنها ستكون ذات اتجاهين، حيث يتمتع كل طرف بحقوق والتزامات متساوية. من هو القاضي ومن هو المدعى عليه - هذا ما يدعي القانون أنه سيتضح أثناء المحاكمة.
    لذا اسأل: ما الفائدة من مثل هذه الجملة؟ من سيصدر الحكم؟ ومن يملك صلاحيات التنفيذ؟
    الجواب على السؤال الأول هو أن المحاكمة في غاية الأهمية، وربما هي الأهم على الإطلاق، وهذا سيتضح أثناء المحاكمة. جواب السؤال الثاني : المحلفون . من سوف؟ وهذا أيضاً سوف يصبح واضحاً خلال المحاكمة. أما بالنسبة للثالث، صلاحيات التنفيذ، فلا بد أنك خمنت بالفعل... سيتم الكشف عنها أثناء المحاكمة.

    لذلك، لن نتلاعب بالألفاظ، وننتقل فورًا إلى مسار المحاكمة.
    تحدث المدعي العام للدولة أولاً.
    - شرف المحكمة .
    أود أن أعرفك على مجرم مميز جداً.
    إن المتهم الذي يقف أمامكم اليوم، المتهم المسمى "القانون الثاني للديناميكا الحرارية" والمعروف أيضًا باسم "الإنتروبيا"، ليس فقط أعظم مجرم على الإطلاق - بل هو أيضًا في الواقع المجرم الوحيد على الإطلاق.
    جاءت ميا الغاضبة من بين الحشد.
    -جريمته، كما يدعي الأشرار بصوته، هي سبب كل جريمة!
    كل خطأ، كل ألم، كل حزن، وكل انزعاج - مصدرهم هو القانون الثاني، هو وهو فقط.
    أزعم أن القانون الثاني كان له غرض واحد فقط منذ يوم ولادته قبل أربعة عشر ونصف مليار سنة حتى يومنا هذا: زيادة الفوضى في الكون.
    الكلمة ذاتها - الإنتروبيا - تعني الفوضى والفوضى وتفكك أي نظام. ستثبت الدعوى بما لا يدع مجالاً للشك عدم أخلاقية القانون، والشر المتأصل فيه، والأيديولوجية المشوهة....

    ومضى المدعي العام ليصف بيانياً مدى فظاعة وقسوة القانون. في مقعده، تجهم القانون، وأطلق سخرية وتعليقات ساخرة لنفسه. وفي مرحلة ما، نفد صبر القانون وقاطع كلام المدعي العام:
    - ربما التوقف عن الحديث والبدء في عرض الحقائق؟
    وسمع ضجة من الأصوات المهتزة في الحشد. تحول المدعي العام إلى شاحب. لقد أصبحت الآن بين يديه سنوات خبرته العديدة: لقد اعتاد دائمًا على الانضباط الصارم للمحاكم، والذي يتطلب إصدار أمر فوري لأي شخص يجرؤ على الانحراف عن الإجراء القانوني الصحيح.
    من عادته ألقى نظرة عاجزة على الكرسي الفارغ لأبي البلاط، وأدرك أن الخلاص لن يأتي من هناك اليوم، وقبل الحكم على نفسه.
    - أدعو إلى المنصة الشاهد الخبير الأول، البروفيسور بولتزمانسكي، أكبر فيزيائي في البلاد.
    أضاء وجه القانون الثاني. -فيزيائي! هتف بصوت عال. كم هو رائع! حقائق! دليل! منطق! خفض صوته كنوع من التحلية السرية مع الأستاذ. - أحب الفيزياء والفيزيائيين. يجب أن تعلم أنني أيضًا قانون فيزيائي لنفسي؟ يسعدني أن أضع نفسي في خدمتك يا أستاذ. يمكنك أن تتوقع التعاون الكامل مني.
    وألقى البروفيسور بولتزمانسكي نظرة صارمة على القانون قائلا: "سنرى في نهاية الشهادة إلى أي مدى ستحب الفيزيائيين، ومدى التعاون الذي يمكن أن نتوقعه منك".

    وأوضح البروفيسور أن القانون الأول للديناميكا الحرارية هو في الواقع قانون الحفاظ على الطاقة: لا يمكن أن تنشأ الطاقة أو تختفي من تلقاء نفسها. جوهر القانون هو كما يلي: في النظام المغلق، يكون مجموع الطاقات - الميكانيكية والكهربائية والإمكانية والحرارية وما إلى ذلك - ثابتًا دائمًا.
    - الأخ الأكبر - لملام القانون الثاني بشوق عندما ذكر القانون الأول - قانون جميل لكنه محافظ جداً جداً.

    - هذا القانون – وتابع الأستاذ، متجاهلاً الاضطراب، هو في الأساس عادل وأخلاقي. فهو يوفر عائد مناسب للعمل، ومكانة متساوية لجميع أشكال الطاقة. وأي طاقة يمكن استبدالها بأخرى، وجميعهم متساوون أمام القانون.

    انهار وجه القانون الثاني. "العمل؟"، تسنف، -من أين أتيت بهذا المصطلح فجأة؟ أين يمكنك العثور على وظيفة في نظام ديناميكي حراري مغلق؟ هل أنت فيزيائي على الإطلاق؟
    - هذه بالضبط هي النقطة التي ستحاكم عليها اليوم، أجاب الأستاذ آيس ببرود. - رغم أن العمل ممكن نظريا في أي نظام - ولم نسمع مثلا القانون الأول ضده - إلا أن وجود القانون الثاني يقتصر على العمل فقط على الأنظمة التي تتضمن عناصر بيولوجية.
    تدخل المدعي العام سعيدًا بالانتقام لشرفه المهين. - هل تقصد القول - التفت إلى الأستاذ - أنك في الكون الفسيح كله لن تجد عملاً في أي مكان إلا على الأرض أو غيرها من الأماكن، إذا وجدت، حيث توجد حياة؟ وكل هذا بسبب المتهم؟؟
    وكانت صيحات الجمهور موجهة نحو القانون الثاني. -متشرد! كسول! اذهب إلى العمل بدلاً من مقاطعة الحياة!
    - اسكت! صاح القانون مرة أخرى، - وأقول شكرا لك لأنك على قيد الحياة على الإطلاق، وشكرا لي!
    - أريد أن أقول - لقد ذاق البروفيسور بولتزمانسكي كل مقطع - أنه منذ خلق الكون وحتى ظهور الحياة، لم يتم عمل أي عمل فيه - بالمعنى المادي - وهذا بفضل صديقنا هنا السيد . الإنتروبيا، القانون الثاني للديناميكا الحرارية.
    -وماذا في ذلك؟ تم كسر القانون. - الذي يهتم على أي حال؟ لماذا لا تسمح لي بالراحة، وتفعل الشيء نفسه؟
    وسارع وكيل النيابة، الذي شعر بسقوط فكرة المحاكمة من بين يديه، إلى التدخل. - حدثنا عن طبيعة المتهم وشخصيته - قال للأستاذ، وأضاف في همس: - حوله إلى تراب ورماد.
    هز البروفيسور بولتزمانسكي رأسه قليلاً، وبدأ: - إذا كان القانون الأول يتناول حفظ الطاقة، فإن الثاني يحدد اتجاه حركتها داخل النظام.
    وينص القانون الثاني على أن الطاقة ستتحرك دائما في الاتجاه الذي يجعلها أقل تركيزا.
    -و.. تثاءب القانون.
    سوف تتدفق الحرارة دائمًا من الجسم الساخن إلى الجسم البارد. أبدا بالعكس.
    -مم-- تابع الأستاذ وهو يحمل قرصًا خشبيًا أحضره لغرض العرض.
    لقد ترك القرص يسقط على الأرض.
    سقط القرص بنقرة واحدة.
    -إذا دحرجتها، فسوف تتدحرج دائمًا إلى أسفل المنحدر.
    -إذا أشعلته فسوف يحترق.
    لكن العملية لن تحدث من تلقاء نفسها أبداً في الاتجاه المعاكس، رغم عدم وجود منع نظري لذلك.
    - ليس بالضبط... تمتم بالقانون.
    - توقف عن المقاطعة - أمر الأستاذ. -أنت تعلم مثلي أن هذه هي الطريقة التي تسير بها الأمور بالضبط.
    - حسنًا، لقد خفف القانون، لذلك هناك اتجاه مفضل لتدفق الطاقة في النظام. هل أنا أحاكم بسبب هذا؟ أنا أقر بالذنب على الفور. أنتم مشغولون، وأنا قانون مشغول، ربما ننتهي وننتقل إلى...
    - النهاية ما زالت بعيدة - رفع الأستاذ صوته. -في الواقع نحن في البداية فقط. وبعد أن وصفت الطبيعة المادية للقانون، سأشرع في وصف عواقبه المدمرة على الحياة اليومية.
    كان الجمهور مليئا بالترقب.
    -ومع ذلك، يجب أولاً أن أذكر حقيقة فيزيائية تنبع من القانون: في أي نظام نشط هناك دائمًا فقدان معين للطاقة المستخدمة لغرض القيام بالعمل-
    -مرة أخرى أنت وعملك-
    - تهدف جميع أشكال الطاقة دائمًا إلى تقليل الطاقة الحرارية. الحرارة - انكسر صوت بولتزمانسكي - الدفء اللطيف الذي نحتضنه - هنا أدنى شكل من أشكال الطاقة - أصبح صوته همسًا - الأكثر حقيرة -
    جاء الجمهور بالبكاء.

  277. روبي
    إذا بحثت في التعليقات في بداية المناقشة، فستجد أن السؤال الذي طرحته قد تمت الإجابة عليه بالفعل.
    وبشكل عام فإن اكتشاف الضوء القادم من المجرات البعيدة يعتمد على قدرات استقبال الضوء للأجهزة التي تنظر إلى تلك المجرات. وطالما أن قدرات الجهاز أكثر تقدما على المستوى التكنولوجي، فإنه سيكون من الممكن توقع مسافة أكبر باستخدام الجهاز. يمكن للتلسكوبات الحالية التقاط الضوء من مسافة بعيدة، ولكن لا تزال هناك مشكلة في التقاط الضوء في الوقت الفعلي. وماذا يعني: الضوء الذي يلتقطه التلسكوب من مسافة 14 م سنة ضوئية، يظهر صورة وضع المنطقة كما كان وقت خروج الضوء من نفس المنطقة التي كان التلسكوب ينظر إليها.
    ومن المعلوم (والمتفق عليه عند أغلب العلماء) أن معدل تمدد الكون يدفع المجرات بعيداً عن بعضها البعض.
    تشير هذه الظاهرة إلى حالة في عالمنا: أن المجرات البعيدة التي يتم رصدها من خلال التلسكوب ليست في الواقع في نفس المنطقة في الوقت (الحقيقي) الذي يتم رصدها فيه. وهذا ينطبق على كل مجرة. المجرة الأقرب إلينا تبعد عنا بضع سنوات ضوئية - وهذا يعني أنه عندما يتم استقبال الصور الثابتة من التلسكوب الذي يظهر المجرة، فإن الصور تظهر حالتها كما كانت قبل بضع سنوات.

    واليوم تظهر المجرات البعيدة حالتها كما كانت قبل 14 مليون سنة.
    وهذا يعني أنه اليوم (بسبب ظاهرة تسارع تمدد الكون) من المفترض أن تكون المجرات في منطقة تبعد عنا أكثر من 14 م سنة ضوئية. ووفقا للتكهنات، فإن سبب ابتعاد المجرات هو الطاقة المظلمة. لذلك، وفقا للحسابات (إذا أخذت الطاقة المظلمة في الاعتبار) - ينبغي العثور على المجرات البعيدة اليوم على مسافة 46 م سنة ضوئية. وهذا يعني عمليا، في الوقت الحقيقي (حاليا)، يجب أن تكون المجرات على بعد حوالي 46 مترا سنة ضوئية من الأرض.
    ومن ثم فإن القطر الحقيقي (حتى اليوم) للكون ليس 29 م سنة ضوئية، بل 93 م سنة ضوئية.

  278. روبي,
    لماذا تعتقد أنك مخطئ؟
    وفي كل اتجاه ننظر إليه، فإن الأفق يبعد عنا 14.5 مليار سنة ضوئية. إذا رأينا أنفسنا جالسين في مركز الكون (وعلينا أولاً أن نفترض أن الكون له مركز)، فإن قطر الكون يبلغ بالفعل 29 مليار شا. لكن هذا الافتراض ليس لديه ما يمكن الاعتماد عليه. وليس من المستحيل أن ينشأ هذا العدد من محدودية أدواتنا وليس من الحالة الحقيقية للكون.

  279. إسرائيل،
    "فقط نظام جيل هو المهم. ترى أن ساعة درجة الحرارة تدور أسرع بمليار مرة من ساعة السيزيوم. و لماذا؟ لأن جاك يسافر عدة مليارات من الأميال بالنسبة لها؟ ماذا سيحدث إذا غير جاك اتجاهه وطار بسرعة 0 بالنسبة لها، فهل ستتباطأ ساعة جيل الحرارية فجأة وتدور بنفس معدل ساعة السيزيوم؟

    ما هي العلاقة بين جيل وجاك؟؟؟ ستكون النسبة بين ساعات جيل دائمًا هي نفس دالة سرعتها بالنسبة إلى الراحة أو إذا صح التعبير إلى سرعة الضوء. حتى لو تم تدمير جاك، لا سمح الله، فإن النسبة بين ساعات جيل لن تتغير. لا يوجد اعتماد بينهما. هل تعتقدون أن هناك مثل هذه التبعية؟؟

  280. إسرائيل،
    لقد منحتك إعفاءً حتى تتمكن من تكريس وقتك للأشياء التي تحبها حقًا. لكن الإعفاء ليس حظرا. إذا كنت تتوق للنزول من النظريات الفائقة إلى مستواي البدائي، فلا تتردد.
    لقد اتضح أن القانون الثاني للديناميكا الحرارية موجود أمام أعينكم مباشرةً. هل يمكنك توضيح كيف أن الوقت ليس مجرد مفهوم أثيري يتم من خلاله قياس العمليات ("مع مرور الوقت"، "مع مرور الوقت"، وما إلى ذلك) ولكنه يسببها بشكل مستقل (على سبيل المثال، "سوف يأخذ الوقت مجراه")؟

  281. عندي سؤال عن عمر الكون للخبراء:
    وإذا كانت الأجهزة الموجودة بحوزة علماء الفلك تكتشف مجرات وصل نورها منذ 14.5 مليار سنة ضوئية، فهذا يعني أنه منذ لحظة خروج أشعتها في اتجاه الأرض، استمرت تلك المجرات في التحرك مبتعدة وموجودة في نفس الوقت. 14.5 مليار سنة ضوئية، مما يعني أن عمر الكون (على الأقل تلك المجرات) يجب أن يكون 29 سنة ضوئية حتى الآن وليس كما هو مذكور.
    هل يمكن لأحد أن يشرح لي أين أخطأت؟
    شكر
    روبي (رؤوفين)
    سعيد عيد المساخر والسبت شالوم

  282. ر.ح.

    إذا أجبت بنعم على السؤالين، فالمعنى الضمني هو (صححني إذا كنت مخطئا):

    1. إذا قارن جاك سوفير عدد الدورات بين الساعتين (ومن الممكن بالتأكيد ترتيب أن تظهر ساعات السيزيوم وساعة الحرارة الوقت في دورات الساعة) فسوف يرى نسبة 2:1. ليس عليه في الواقع الانتظار طوال الوقت حتى اللقاء. وفي أي فترة زمنية محددة، ستكون النسبة هي نفسها.

    2. إذا قامت جيل بالعد، فستحصل على نسبة مثلاً 1,000,000,000:1

    المشكلة هي هذه:

    1. كل من جاك وجيل في حالة راحة بالنسبة لهما. وأنظمتها ليست متسارعة، وتسمى في لغة النسبية "بالقصور الذاتي". ولو أمكن التمييز بينهما، مثلا، عملية عد الدورات، لكان ذلك مخالفا للمسلمة الأولى للنسبية.
    سيتم القبض على هذا التحفظ حتى لو قلت أن إحدى الحركتين تتعلق بنظام CMBR، على الرغم من أنه من الممكن الجدال مع مثل هذا التحفظ. (لم يحالفني الحظ. سنصل إلى هناك إذا وصلنا إلى CMBR).

    2. فكر فقط في نظام جيل. ترى أن ساعة درجة الحرارة تدور أسرع بمليار مرة من ساعة السيزيوم. و لماذا؟ لأن جاك يسافر عدة مليارات من الأميال بالنسبة لها؟ ماذا سيحدث إذا غير جاك اتجاهه وطار بسرعة 0 بالنسبة لها، هل ستتباطأ ساعة جيل الحرارية فجأة وتدور بنفس معدل ساعة السيزيوم؟

    يوفال: بالطبع يمكنك الاستمرار، لكن تذكر أن لدي إعفاءً.

    شبح أين ذهبت؟ لقد فاتني الهجمات الجامحة من معجبي السري.

    بالمناسبة هل هذا الرد :

    https://www.hayadan.org.il/confusion-about-evolution-3008116/#comment-319326

    هل تجعلك تغلق فمك الكبير الغبي؟

    أو وضع أي تعليقات من المحرر والمحرر العلمي للمجلة الشهرية التي ترعاها الجامعة العبرية؟

    شالوم إسرائيل ،
    استمع - منذ اللحظة التي بدأت فيها قراءة فصلك، لم أستطع التوقف... رائع.
    كان المحرر العلمي تسفي أتزمون متحمسًا أيضًا. أرجو قراءة رده ففيه تعليقات مهمة جداً.
    مرفق أيضًا النص الخاص بك مع تعليقات Zvi المحددة.
    (بعد أن ننتهي من هذا الفصل، سنتحدث عن التكملة... ويبدو لي أنه في الفصول القادمة المخطط لها تتفاقم المشكلة الموجودة في هذا الفصل أيضًا: فأنت تفترض أن جميع القراء تماما على دراية بـ Matria وبالتالي يمكنه بسهولة متابعة اللوم والتعليقات الساخرة. قد لا يكون الأمر سيئًا هنا، ولكن عندما تتعامل مع Godel وJacobians، وما إلى ذلك، قد يفقد القارئ العادي تمامًا...)
    ميكي

    أولاً، نحن الأوغاد، ويجب أن نحاكم إذا لم ننشر. بين، جلي.
    يجب ألا يكون هناك صراخ، ليس هناك مشكلة كبيرة. وهذا يستحق الجدال حوله.
    ثانياً، لم أفهم كل شيء، أعترف بذلك. هناك أيضًا مشكلة في هذا النوع (النوع) - فمن الصعب أن نعرف بثقة من ومتى يقول كل متحدث الحقيقة أو يكون ساخرًا أو ذكيًا. وقد يؤدي إلى تضليل القراء. ولهذا السبب من الضروري التأكد من إعطاء تلميحات شفافة عندما يكون الادعاء هو ما يعتقده المتحدث حقاً، أو عندما يكون ساخراً أو يحاول التضليل. هذه مشكلة تحتاج إلى معالجة.
    ثالثا: هناك أشياء على فرض أنني فهمتها لا أتفق معك فيها، أو أتساءل عن مدى دقتها. جانب الأمر السوسيولوجي السياسي: ليس من الواضح تماما بالنسبة لي إلى أي مدى تمثل الأشياء المكتوبة رأي المؤلف، أو عرضا لمناهج يعارضها.
    لقد وضعت علامة على كل هذا.
    الغزال

  283. المسؤول أم أحسب؟ 🙂
    الردود "القاتلة" التي تلقيتها حتى الآن، حتى لو كانت مبررة، تكرر نفسها ولم تعد مفيدة لي. كما أن التعليقات التشجيعية ليست مفيدة كثيرًا بخلاف تحسين الحالة المزاجية.
    لقد رأيت الكثير من مناقشات النمذجة المتقدمة هنا، لكنني لم أر الكثير من الاهتمام بالفيزياء الأساسية. هل من الممكن أن أستمر؟

  284. سمعت عن خدمة جديدة للعالم: الرد في انتظار الرد.

    في غضون ذلك، المزيد من المؤامرات من القانون الثاني.

    جلس البروفيسور بولتزمانسكي على كرسي المتهم، وهو يرتجف قليلاً من البرد ويخشى مما سيأتي.
    نظر إليه القانون الثاني للديناميكا الحرارية بنظرة دامت، كما بدا للأستاذ، إلى الأبد.
    -البروفيسور بولتزمانسكي- القانون تكلم بسرعة، - لماذا أنتم العلماء تلمسون قوانين الفيزياء وتخيفونها من مكانها؟
    -ماذا؟ وأضاف الأستاذ في كرسيه. -من هو الراعي لقوانين الفيزياء؟ قوانين الفيزياء جيدة وصادقة وصديقة للحياة. أنت المخرب الوحيد!
    -ذلك حقا؟ سأل القانون باستخفاف.
    -تمامًا هكذا! القانون الأول للديناميكا الحرارية هو قانون رائع. قوانين المرور -- عمل فكري. قانون الجذب عبقري. قوانين الكهرباء والمغناطيسية وديناميكية الموائع...
    - وميكانيكا الكم؟ قاطعه القانون.
    كان الأستاذ صامتا.
    -بنية الذرة؟ الجسيمات الأولية؟ الاضمحلال الإشعاعي؟
    ملأ الأستاذ فمه بالماء.
    -شيء ما حصل؟ وضع القانون على وجه القلق.
    الآن تحدث الأستاذ بسرعة. - أنا لا أحب ميكانيكا الكم. الحقيقة هي أنني لا أحب كل الفيزياء الحديثة. الجسيمات التي هي موجات، الإلكترونات التي تقفز من مكان إلى مكان دون أن تكون في المنتصف، القوة الشديدة التي لا تقدر إلا بالتقريب وتغير اتجاهها دون سابق إنذار.. عدم اليقين... عدم اليقين...
    - والنظرية النسبية؟ دراسة القانون.
    -نعم! الأستاذ سعيد. هذه هي نظرية السحر الساحرة! إلا أن النظرية النسبية هي في الأساس رياضية، رائعة ورياضية سماوية.. كان الأستاذ يمتلئ بالإثارة عندما تناول مادته المفضلة.
    - مبادئها، وخاصة النسبية العامة، يصعب فهمها إلى حد ما، ولكن منذ لحظة فهمها تكون متسقة ولا لبس فيها، على الرغم من - ضحك الأستاذ في نفسه - أن هندستها ملتوية بعض الشيء.
    -شيء مضحك؟ وطالب القانون.
    - لا.. لا شيء.. نكتة خاصة.. كنت أفكر في ريمان...
    - هل يمكنك أن تقتبس لنا شيئاً من المعادلات الشهيرة للنظرية النسبية؟
    -مسار! أجاب الأستاذ عن علم. - تحويلات لورنتز التي تربط كتلة الجسم وطوله بسرعته، أو الصيغة المعروفة: E=MC تربيع، طاقة الجسم تساوي كتلته مضروبة في مربع سرعة الضوء.
    - متهم - القانون قاطعه بجفاء - هل صحيح أنكم في الماضي كنتم فيزيائيين تريدون محاكمة سرعة الضوء؟
    دهشة وصدمة في قاعة المحكمة التي تحمل اسم سولونيو!
    -لا!!! صاح الأستاذ. ليس تماما!! أنت تخرج الأمور عن سياقها!
    -لا؟ هل تريد منا أن نستدعي سرعة الضوء؟ إنها صديقة قديمة لي، كما تعلمين.
    - أنا... نحن... الأستاذ واجه صعوبة في استيعاب التحول السريع في الجملة، وخيانة الجمهور الذي صفّر عليه الآن. - على أية حال، لماذا أنا متهم؟ هذه هي جملة القانون الثاني! أنا أحتج! أريد العودة إلى ديارهم! استدار لمغادرة الرصيف ولكن تم إيقافه من قبل اثنين من المرشدين الأقوياء.
    - كنت تعلم أن هذه محاكمة ذات اتجاهين عندما تطوعت للإدلاء بشهادتك، والآن سوف تطيع القانون - قال القانون بصرامة. تأخذ مكانك أريد أن أعرض الدليل رقم 1 - أخرج مصباحًا يدويًا - واستدعاء شاهد الادعاء رقم 1 للشهادة، سرعة السرعة - ضغط على زر المصباح - سرعة...
    -لا لا! صاح الأستاذ. انا اعترف! فقط دعني أشرب كوبًا من الماء وأتعافى، وسأخبرك بكل شيء.
    - القانون الثاني أطفأ الفانوس وانسكبت سلسلة النصر على وجهه.

  285. مرة أخرى الرد في انتظار التأكيد. فقط لا يصدق.
    يبدو لي أن اسمك إسرائيل هو الكلمة الإشكالية.

    هنا التعليق السابق بدون الاسم الصريح

    حتى يتم إصدار إجابتي السابقة، فلنبقي الأمر بسيطًا.
    1) نعم
    2) نعم

    ملاحظة: من أجل حساب الوقت من درجة الحرارة أو الاضمحلال الإشعاعي (أي السيزيوم) يجب عليك إضافة سرعة المقياس إلى الصيغة. عند السرعات المنخفضة يكون لا يكاد يذكر، ولكن عند السرعات القريبة من الضوء يكون مهمًا. أنا لم أقل ذلك، إنه أينشتاين.

    ومازلت لا أفهم أين التناقض؟

  286. إسرائيل،
    حتى يتم إصدار إجابتي السابقة، فلنبقي الأمر بسيطًا.
    1) نعم
    2) نعم

    ملاحظة: من أجل حساب الوقت من درجة الحرارة أو الاضمحلال الإشعاعي (أي السيزيوم) يجب عليك إضافة سرعة المقياس إلى الصيغة. عند السرعات المنخفضة يكون لا يكاد يذكر، ولكن عند السرعات القريبة من الضوء يكون مهمًا. أنا لم أقل ذلك، إنه أينشتاين.

    ومازلت لا أفهم أين التناقض؟

  287. إسرائيل،
    بادئ ذي بدء، "الساعة المؤقتة" لا تدور. سوف يشهد انخفاضًا من 6000 ألف إلى 3 في كلتا الحالتين. بالنسبة لجيل، سوف يستغرق الأمر ساعة واحدة، وبالنسبة لجاك، سيستغرق الأمر مليار سنة.
    أعتقد أننا على حد سواء متفقون، أليس كذلك؟

    والآن أنت القنفذ،
    1) ما هو التناقض إذن؟

    2) ما الذي ستظهره ساعات جيل وجاك والسيزيوم ودرجة الحرارة في لحظة لقائهما؟

    فيما يتعلق بنظام تحديد المواقع. ما حاولت فعله هو أن أعرض على البعد "العادي" ما يحدث في الوقت المناسب. لاحظ أن نظام تحديد المواقع العالمي (GPS) له ثلاث وظائف (هناك المزيد، ولكن هناك ثلاث وظائف مهمة لهذا الغرض):
    1) مقياس الموقع - يعطي إحداثيات - مماثلة لساعة درجة الحرارة لديك
    2) عداد المسافات - يخبرك بالمسافة التي قطعتها —— على غرار ساعة السيزيوم
    3) عداد السرعة، يخبرك بمدى سرعة قيادتك بالنسبة لبعض أنظمة الإحداثيات (عادةً WGS84)

  288. ر.ح.
    أعتقد أن مثال نظام تحديد المواقع العالمي (GPS) يمكن صياغته على النحو التالي: تركب جيل دراجة ذهابًا وإيابًا، وفي كل مرة تمر بجوار جاك، يقارنون عدادات السرعة. تقطع جيل أميالاً، ويقرأ عداد السرعة الخاص بجاك الكسول 0، أو قليلًا، أو أيًا كان: أقل من جيل. هذا على الرغم من أن كلاهما متفقان على أنهما على نفس المسافة من أوري. أنا أفهم ذلك أليس كذلك؟

    لاحظ أن هذا النظام نيوتوني بالكامل، بغض النظر عن لاحظ أيضًا أنها فاتتها الحجة المهمة التالية: العلاقة بين درجة الحرارة والوقت يتم الحصول عليها من خلال صيغة فريدمان، وهي دالة متصلة وذات قيمة واحدة. حجتك: "إنه مقياس حرارة يقيس معلمة خارج النظام." سنرى إن كان بإمكاننا التوصل إلى اتفاق على أن نصل إلى تناقض بين فرضيات إطالة الزمن في العلاقات ونظرية الانفجار الكبير، أو سأتقبلها وأستمتع بها، أو سنتيأس ونفشل معًا. لكن لا يمكننا القيام بذلك إلا إذا عملنا مثل القنافذ: السؤال الدقيق - الإجابة الدقيقة.

  289. ر.ح.
    حجتي مبنية على مراحل. إذا اتفقنا على خطوة واحدة، يمكننا الانتقال إلى الثانية. في هذا الرد:

    https://www.hayadan.org.il/astronomers-reach-new-frontiers-of-dark-matter-130112/#comment-331078

    الذي يقال فيه:

    الاستنتاج أ:
    ومن خلال قياس درجة الحرارة وحدها، من الممكن معرفة الوقت الذي انقضى منذ الانفجار الكبير في أي مكان في الكون، بأي دقة نريدها وهذا ممكن تقنيًا.

    الخطوة التالية: تثبيت الساعات المؤقتة.
    يمكن القيام بذلك عن طريق توصيل جهاز كمبيوتر بمقياس الحرارة. (لا يهم الآن إذا كانت هذه هي درجة حرارة CMBR).

    الاستنتاج ب:

    يمكن تجهيز أي شخص بساعات مؤقتة.

    اتفقنا على موضوع ساعات درجة الحرارة. من الصعب أن تأتي وتدعي:

    "حقيقة أنك تسميها "ساعة مؤقتة" لا تجعلها ساعة تقيس الوقت."

    هناك شيء صغير وبسيط يمكنك القيام به، ولكن مثل القنفذ، من شأنه أن يزيد من حدة الموضوع:

    و. قيل أن جيل كان لديه جهاز أسميه "الساعة المؤقتة". بجانب هذه الساعة توجد ساعة السيزيوم. يلتقط الفيديو كليهما. في نهاية رحلة جيل التي استغرقت عامًا، والتي انخفضت فيها درجة الحرارة من 6000 ألف إلى 3 ألف، تقوم بتشغيل الفيديو بسرعة، وتحسب عدد الدورات التي قامت بها الساعة المؤقتة وتقسمها على عدد الدورات التي تقوم بها ساعة السيزيوم.

    1. هل توافق على أننا سنحصل على رقم في حدود المليارات؟ نعم لا؟

    ب. نفس الترتيب موجود أيضًا في سفينة الفضاء التي طارت جيل عبرها. بالنسبة لها، لقد مرت 13.6 مليار سنة على تلك السفينة الفضائية. هنا أيضًا، يمكنك حساب عدد الدورات التي تقوم بها الساعة المؤقتة وتقسيمها على عدد الدورات التي تقوم بها ساعة السيزيوم.

    2. هل توافق على أننا سنحصل على رقم في حدود 1؟ نعم لا؟

    إذا كانت إجابتك على 1 أو 2 لا، فاشرح السبب.

  290. إسرائيل،
    حقيقة أنك تسميها "ساعة مؤقتة" لا تجعلها ساعة تقيس الوقت. إنه مقياس حرارة يقيس معلمة خارج النظام.
    وبالمثل، استطاع جيل وجاك النظر إلى معدل المسافة بين أندروميدا ودرب التبانة.
    ولو سألت جل (الذي يتحرك بسرعة) ما هو معدل المسافة؟ ستقول أنه في غضون ساعة ابتعدت المجرات (على سبيل المثال) 100 سنة ضوئية. إذا سألت جاك سيقول أن المعدل هو 100 سنة ضوئية في مليار سنة.

    فيما يتعلق بمقياس درجة الحرارة الذي يعرف أو لا يعرف أنه سيلتقي بمقياس آخر، في رأيي أنك تفتقد نقطة أساسية هنا. قياسات جيل وجاك ليست ذات صلة. كلاهما يقيس درجة الحرارة أثناء القياس. السؤال هو مدى سرعة وصولك إلى هذه النقطة.

    فكر في الوقت باعتباره بُعدًا. ومع ذلك، وعلى عكس أبعاد الموقع، فإننا نتحرك باستمرار في البعد الزمني بمعدل ثابت.
    صورة جيل وجاك مرة أخرى في القطار بسرعة ثابتة. كانت جيل تركض ذهابًا وإيابًا وتقطع المسافة المقطوعة في القطار، وكان جاك جالسًا مذعورًا. في كل مرة يجتمعون فيها ينظرون إلى نظام تحديد المواقع العالمي (GPS) الخاص بهم (المؤقت التناظري). بأعجوبة، سيُظهر نظام تحديد المواقع العالمي (GPS) أن جيل قد قطعت مسافة كبيرة (مماثلة للوقت) بالنسبة لجاك، لكن نظام تحديد المواقع العالمي (GPS) سيُظهر أيضًا أن موقعهما في لحظة اللقاء هو نفسه على الرغم من اختلاف المسافة التي قطعاها.

  291. كنت أشير إلى المناقشة التي دارت منذ شهر مضى، لكنني أتفق مع نيكو على أننا كنا على وفاق.

  292. إسرائيل،

    كان سؤالي ردًا على سؤال كتبته من قبل: "لماذا، إذا طبقنا نفس المنطق بالضبط، لن يتحرك النظام نحو إنتروبيا أعلى في الماضي أيضًا؟" ففي نهاية المطاف، كل شيء هنا قابل للعكس تمامًا فيما يتعلق بقوانين نيوتن."

  293. طالب علم
    "سؤال لك - هل فهمت وقبلت الشرح الوارد في المقالات حول رتابة الإنتروبيا وحل "مشكلة عدم التماثل" التي سألت عنها؟"

    بالطبع فعلت. أنا أيضًا لم أفهم سؤالك "ليس من الواضح بالنسبة لي سبب إصرارك أنت وبوينت على تقديم تفسيرات خاصة بك لما تم تقديمه بالفعل تفسيرًا معقولًا إلى حد ما (كما هو مذكور في المقالة) من قبل عظيمكم ( بولتزمان، فاينمان، بنروز...)". كان الأمر برمته هو سؤالي إذا تم إعطاؤه ببساطة "تحديد كمي" لحظة معينة. لم أحاول مطلقًا الادعاء بأنه لا يوجد تفسير، أو أنه غير جدير بالاهتمام.
    ولكن يبدو لي أننا قد استنفذنا الموضوع إلى حد ما، والآن نحن على نفس الصفحة.

    ر.ح. - الرد عليك في الفحص الأمني.

  294. ر.ح.
    سأتناول أولاً مشكلة المتاعب التي أثرتها.
    وبما أنني الوحيد بيننا الذي يعرف مسألة "الهجوم على المؤسسة" كما تقول، فيمكنني أن أؤكد لك بأمانة أنه لا يوجد شيء أبعد عن الحقيقة. تم التواصل عبر رسائل البريد الإلكتروني، وكان الأشخاص الذين اتصلت بهم يتعاونون بحماس. وفي الخلاف بيننا في المسائل الجوهرية، تمكنت من إقناعهم بصحة ادعاءاتي. عندما تعمقنا أكثر، في معظم الحالات تم الإعراب عن اعتذار لعدم القدرة على حل المشكلة التي أثرتها.

    كان الادعاء الأول الذي أثير بانتظام هو أن كل نظام يرى الآخر على أنه خاضع بشكل أبطأ، كما تدعي النسبية.

    ويمكن دحض هذا الادعاء بسهولة عن طريق استخدام كاميرات عالية الدقة وعن طريق الفيديو، لأنه من الصعب للغاية الجدال مع صور لا لبس فيها، والتي تظهر الوقت المحدد الذي تظهره كل ساعة خلال الاجتماع.

    سنتناول الآن حجتك، ونحاول تحقيق نفس مبدأ الصور، الذي يمكن ملاحظته بعد فترة طويلة من وقوعه.

    «ساعات السيزيوم ستظهر فرقًا، ساعة في حالة واحدة ومليار سنة (حسب مبالغتك) في الحالة الأخرى. سيُظهر مقياس الحرارة لكلا التوأم نفس درجة الحرارة (نحن نتفق على ذلك). אם שני החבר'ה האלו היו מציירים גרף של מד הטמפ' לאורך זמן היינו רואים שקצב ההתקררות במקרה של ההוא שנח הוא איטי (נגיד מ – 6000K ל- 3K במילארד שנה) בעוד שהגרף של השני היה מראה קצב מהיר 6000K ==> 3K בתוך الساعة. بسيطة جدا، أليس كذلك؟ إلا إذا كنت أفتقد شيئًا أساسيًا للغاية وسأكون سعيدًا إذا قمت بتنويري"

    هنا هو التوضيح:

    في الحالة التي ناقشناها، يتعلق الأمر بالأنظمة التي لم يتم تسريعها. فتاة لطيفة تدعى جيل موجودة في سفينة فضاء في حالة سكون تام. بجانبها ساعتان: إحداهما ساعة السيزيوم والأخرى ساعة درجة الحرارة. يلتقط الفيديو كلتا الساعتين معًا.

    1. إذا كان "الرسم البياني للآخر سيظهر معدلًا سريعًا يبلغ 6000 ألف ==> 3 آلاف خلال ساعة." بعد كل شيء، سيُظهر مقطع الفيديو الخاص بالساعتين أن الساعة الحرارية تدور بشكل أسرع بكثير من ساعة السيزيوم. في الواقع، مقابل كل دورة في ساعة السيزيوم، تدور الساعة الحرارية عدة مليارات من الثورات.

    فلماذا؟ لأنه في مكان ما على الطريق ستلتقي جيل بساعة تتحرك بالنسبة لها؟ كيف تعرف الساعات الموجودة في سفينة جيل الفضائية مدى سرعة تحرك تلك الساعة البعيدة، ولهذا السبب يتعين عليها تغيير معدل دورانها؟
    يفرض المنطق أنه طالما لم يتم تسريع النظام، فإن ساعة السيزيوم وساعة درجة الحرارة تتحركان بنفس المعدل وستعرض كل صورة نفس الوقت في كليهما، لأن النظام في الواقع في حالة راحة.

    نكمل. في وقت الاجتماع، تمر جيل بساعة سيزيوم أخرى بجوارها ساعة أخرى لدرجة الحرارة. وهذا لا يمنع الكاميرا عالية الدقة الموجودة في سفينة جيل الفضائية من تصوير الساعات الأربع معًا. لنفترض أن هناك أيضًا كاميرا في النقطة التي تقع فيها الساعات البعيدة، خصمًا بالنسبة للساعات الموجودة، وتقوم بالتقاط صور للساعات الأربع أيضًا.

    2. هل ستكون الصور التي التقطتها الكاميرتان لحظة اللقاء هي نفسها؟ أو هل تعتقد أن الصور سوف تظهر أن كل نظام يتخلف عن الآخر؟

    3. كيف ستبدو الصور الواضحة؟

    لأن هذه هي النقطة الأساسية: كل نظام في حالة سكون، وبالتالي فإن معدل تقدم ساعات السيزيوم وساعة الحرارة في كل نظام هو نفسه. لأننا اتفقنا على أنه خلال الاجتماع ستظهر الساعات المؤقتة نفس الوقت، وكل ساعة مؤقتة تظهر نفس الوقت لساعة السيزيوم المجاورة لها (نظام غير متسارع)، ولأنه إذا كان a=b، c=d، و a=c ثم a=b=c بعد كل شيء، يجب أن تظهر جميع الساعات نفس الوقت في لحظة الاجتماع.

    وهذا يتعارض مع الحجة الأصلية حيث يوجد فرق قدره 2 مليار سنة بين ساعتين السيزيوم.

    حاول أن تفعل ذلك مثل القنفذ، ببطء وحذر، واكتب كل الأوقات ومعدلات الفيديو، أعتقد أن هذه نتيجة حتمية.

    ومن ناحية أخرى، إذا كنت متعبًا قليلاً من ذلك، فسوف أتفهم ذلك. آخر شيء أريده هو أن أكون مصدر إزعاج.

  295. إسرائيل،

    "هل هناك فقرة في المقالة الأخيرة أو في مقالة أخرى، حيث يمكنك إعطاء كل لحظة في التاريخ قيمة عددية محددة، بالدقة التي تريدها، وكذلك ربطها بالعوامل المادية؟"

    - إنتروبيا؟ انظر إلى نهاية المقالة الأولى التي قمت بربطها، هناك قيم تقريبية مذكورة (بواسطة بنروز) للإنتروبيا في حالات الكون المختلفة. (لا أعتقد أنه من الجدية أن نسأل عن "الدقة التي نريدها" لمثل هذا الرقم)

    فيما يتعلق بالسؤال الثاني، فأنا لست على دراية بتجربة ويلر. أعتقد أن البروفيسور أهارونوف قام بالبحث/البحث في موضوعات مثل التأثير على الماضي/الحاضر من المستقبل (أو أشكال مختلفة من هذا القبيل)، قد ترغب في البحث عن مقالاته حول هذا الموضوع. وعلى ما أذكر، يتعلق هذا بما يسمى بالقياسات الضعيفة في نظرية الكم.

    سؤال لك - هل فهمت وقبلت الشرح الوارد في المقالات حول رتابة الإنتروبيا وحل "مشكلة عدم التماثل" التي سألت عنها؟

  296. حل الشروط الأولية لا يكفي.
    في رأيي الحل هو أن الكون لديه العديد من الحالات الممكنة، ووعينا ينتقل عبر تلك الحالات التي تسمح بوجوده، أي في تلك الحالات التي نرى فيها الإنتروبيا تتزايد. هناك حالات أخرى (أكوان؟) حيث لا يحدث هذا، ووعينا ليس هناك.

    وبهذا المعنى يمكن القول أن كل المواقف الممكنة تحدث، ونحن فقط في تلك المواقف التي تسمح لاختراعاتنا. بالنسبة لمراقب خارجي، يبدو أن الوعي يتسبب في انهيار الدالة الموجية بطريقة معينة.

  297. إسرائيل،
    وبما أن الرد لم ينشر حتى الآن فقد قمت بنسخه محررا بدون الرابط الذي كان مرفقا ونتمنى أن يمر هذه المرة.

    إسرائيل،

    هذه المناقشة تتكرر بطريقة دائرية. لا أرى أي تناقض ولا أستطيع أن أفهم ما التناقض الذي تراه.
    الإجابة على أسئلتك بسيطة ومتماسكة مع مفهوم تسريع الزمن حسب السرعة
    ستظهر ساعات السيزيوم فرقًا، ساعة في حالة واحدة ومليار سنة (حسب أقصى تقديرك) في الحالة الأخرى. سيُظهر مقياس الحرارة لكلا التوأم نفس درجة الحرارة (نحن نتفق على ذلك). אם שני החבר'ה האלו היו מציירים גרף של מד הטמפ' לאורך זמן היינו רואים שקצב ההתקררות במקרה של ההוא שנח הוא איטי (נגיד מ – 6000K ל- 3K במילארד שנה) בעוד שהגרף של השני היה מראה קצב מהיר 6000K ==> 3K בתוך الساعة. بسيطة جدا، أليس كذلك؟ إلا إذا كنت أفتقد شيئًا أساسيًا للغاية وسأكون سعيدًا إذا تمكنت من تنويري.

    أبعد من ذلك، لا أحب حقاً إشارتك إلى أنك لجأت إلى الأساتذة والعلماء المؤهلين وهم لا يفهمون. كان هناك نقاش أعلاه حول المسائل الرياضية والعلمية وهجوم موجه ضد يوفال. إحدى الخصائص الرئيسية للمتنمر العلمي هي أن المتنمر عادة ما يهاجم "المؤسسة العلمية" ويقلل من شأن العلماء الحاضرين الذين يفشلون في رؤية ما يناقشه المتنمر بحماس. أقترح عليك ألا تسير في هذا الاتجاه، حتى لا تصبح واحدًا منه. ماذا تحتاج لمهاجمة "المؤسسة"؟ التزم بعلمك وصدقني أنك إذا أثبتت نظريتك فسوف يتم قبولها دون مؤامرات أو غموض مؤسسي.

  298. ربما فاتني شيء.
    هل توجد فقرة في المادة الأخيرة أو في أي مادة أخرى، يمكن فيها إعطاء كل لحظة من التاريخ قيمة عددية محددة، بالدقة التي نريدها، وربطها أيضًا بالعوامل المادية؟
    هل تعرف شيئا مماثلا؟
    سأكون سعيدًا بتلقي التوضيحات أو المراجع.
    سؤال آخر: هل أنت على دراية بتجربة ويلر التي يبدو من الممكن فيها التأثير على الماضي من الحاضر؟

  299. نقطة،

    "ما الأمر يا طالب، الأشياء والمشاكل التي ذكرتها هي بالضبط المشاكل التي تشير إليها هذه المقالات.
    لهذا السبب أعتقد أنك لم تقرأها."
    - "لماذا"؟ المقال يعرض "المشكلة" ويطرح الحلول، كما ادعيت وجودها وأنكرتها (ولهذا السبب ربطتها بها).

    إسرائيل،

    نعم. لاحظ أن ذلك في الفقرة التي تتناول الجانب الفلسفي لمفهوم الزمن (وهو ما لم يتطرق إليه في المقالة الثانية التي ربطتها - ومن بين أمور أخرى، السبب الذي دفعني إلى القول بأن المقالة الثانية أكثر إيجازا).

    "ومن هنا مناقشاتنا السابقة. وفقا لفكرتي، من الممكن أن نحدد تماما ما تعنيه عبارة "مع الوقت".
    ويرتبط الوقت ارتباطًا مباشرًا بكمية الجزيئات، أو أي شيء آخر، في وحدة الحجم. وهكذا يتم تعريف كل لحظة من الزمن بشكل لا لبس فيه، ويتم فصل المستقبل عن الحاضر والماضي برقم عداد بسيط.
    - لقد فهمت ما تقصده (ونقطة) بـ "المشكلة" التي تتحدث عنها بدءًا من أحد تعليقاتك الأخيرة، ولهذا السبب ربطت مرة أخرى بالمقال أعلاه.
    ليس من الواضح بالنسبة لي سبب إصرارك أنت وبوينت على اختراع تفسيرات خاصة بك لما تم بالفعل تقديم تفسير منطقي إلى حد ما (كما هو مذكور في المقال) من قبل العظماء منك (بولتزمان، فاينمان، بنروز...).

  300. طالب.
    من مرجعك في الرد :

    https://www.hayadan.org.il/astronomers-reach-new-frontiers-of-dark-matter-130112/#comment-325851

    في كتاب بعنوان سهم الزمن ونقطة أرشميدس"

    من مرجعك الأخير:

    بمجرد أن نقبل تفسير بولتزمان لماذا العيانية
    تتطور الأنظمة بطريقة تجعل SB يزداد
    مع مرور الوقت، تبقى المشكلة المزعجة (التي
    كان بولتزمان يدرك جيدًا) ما نعنيه بـ "مع".
    الوقت

    ومن هنا مناقشتنا السابقة. وفقا لفكرتي، من الممكن أن نحدد تماما ما تعنيه عبارة "مع الوقت".
    ويرتبط الوقت ارتباطًا مباشرًا بكمية الجزيئات، أو أي شيء آخر، في وحدة الحجم. وهكذا يتم تحديد كل لحظة من الزمن بشكل لا لبس فيه، ويتم فصل المستقبل عن الحاضر والماضي برقم عداد بسيط.

    نقطة.
    هل يمكنك التوسع في موضوع سهم الزمن أحادي الاتجاه نتيجة انهيار الدالة الموجية؟ لماذا يمنع زيادة الانتروبيا أيضا في الاتجاه الماضي؟

    ر.ح. ؟؟؟؟!؟

  301. ما الأمر أيها الطالب، الأشياء والمشاكل التي ذكرتها هي بالضبط المشاكل التي تشير إليها هذه المقالات.
    لذلك أعتقد أنك لم تقرأهم.

  302. نقطة،

    انا اعرف. لقد كتبه أستاذ مشهور عالميًا في مجال الفيزياء الإحصائية.
    يبدو أنك تحاول مفاجأتي بما هو مكتوب في المقال، وهو أمر مسلي للغاية. على أية حال، أنصحك بقراءة الموضوع كاملاً من البداية إلى النهاية - ربما تغير رأيك. بالطبع، يمكنك البقاء/اختلاق أي تفسيرات تريدها، وتطوير النظريات من هنا إلى معهد كاليفورنيا للتكنولوجيا، ولكنك على الأرجح "ستخسر" في النهاية.

    إسرائيل،

    حسنًا، أفهم سؤالك. أقترح عليك أن تقرأ مرة أخرى (أريد أن أصدق أنك قرأت مرة واحدة بالفعل) مقال البروفيسور ليبوفيتش، الذي ربطتك به من قبل، وهنا النقطة: http://www.scholarpedia.org/article/Time%27s_arrow_and_Boltzmann%27s_entropy - يتحدث عن سؤالك، من بين أمور أخرى.

    وأما الكتاب المذكور - فلا أعرفه، ولم أره في الروابط التي أعطيتك إياها.

  303. نقطة جميلة. نحن متفقون فيما يتعلق بثنائية اتجاه الإنتروبيا فيما يتعلق بضغط الوقت.
    هل يمكن للانهيار أيضاً أن يفسر مسألة التأثير المفترض للحاضر أو ​​المستقبل على الماضي؟

  304. Studanczyk، المقالة التي أرسلتها فكرة جيدة. وأقترح عليك قراءتها أيضًا.

    بولتزمان: "إن التحول من حالة محتملة إلى حالة غير محتملة في الطبيعة لا يحدث كثيرًا كما يحدث العكس، ويمكن تفسيره بافتراض حالة أولية غير محتملة جدًا [SB صغيرة] للكون بأكمله المحيط بنا. وهذا افتراض معقول، لأنه يمكننا من شرح حقائق التجربة، ولا ينبغي للمرء أن يتوقع أن يكون قادرا على استنتاجه من أي شيء أكثر جوهرية.

    بمجرد أن نقبل التفسير الإحصائي لسبب تطور الأنظمة العيانية بطريقة تجعل SB يتزايد مع مرور الوقت، تظل هناك مشكلة مزعجة (والتي كان بولتزمان يدركها جيدًا) حول ما نعنيه بـ "مع الوقت": بما أن القوانين الديناميكية المجهرية موجودة متماثل، فإن اتجاهي متغير الزمن متكافئان قبليًا وبالتالي يجب أن يظلا كذلك بعديًا.

    وأنا أقول إن الأمر لا يتعلق بالظروف الأولية، بل بانهيار الدوال الموجية.

  305. نقطة،

    1. إذا تم تفسير قانون فيزيائي من خلال نظرية رياضية، ألا يمكن اعتباره قانونًا أساسيًا للطبيعة؟ إنها لحقيقة أن الأشخاص الذين من المرجح أن يكون رأيهم حول هذا الموضوع أعلى من رأيك. وفي معظم السياقات التي درستها أيضًا، تم ذكر قوانين الديناميكا الحرارية باسم "القوانين الأساسية للفيزياء". وقد تم ذكرهم أيضًا على هذا النحو في ويكيبيديا الإنجليزية.

    2. لقد قلت هذا عدة مرات وأجبت ولكن لن أكرر كلامي.

    3. أنت على حق أن التوزيعات التي ذكرتها هي بالفعل مستمدة من افتراضات معينة، ولكن هذه الافتراضات هي المفترضة، وليس التوزيعات نفسها.

    المقصود بـ "... ليس هناك أي سبب من جانب قوانين الفيزياء لوجود أنظمة التوازن هذه في الواقع." لا أفهم، ولكن في الفيزياء والميكانيكا الإحصائية يتم التخلي عن الوصف الدقيق "مقدمًا". بشكل عام، هناك عدد قليل جدًا من الأنظمة الفيزيائية التي يمكن وصفها بدقة. والحقيقة هي أن الفيزياء الإحصائية تعمل بشكل رائع. ويعتمد عليه الكمبيوتر الذي تكتب عليه، من بين أشياء أخرى.

    4.
    "القانون الفيزيائي هو معادلة شرودنجر لهذه المسألة، والقانون الإحصائي هو القانون الديناميكي الحراري. أعتقد أن الأمر واضح."
    - بوضوح؟ ولم أسمع قط عن مثل هذا التقسيم.

    "لا يفصل القانون الإحصائي مجموعة من الجزيئات التي تشكل الغاز عن مجموعة من الجزيئات التي تشكل خلية لاقحة. على الرغم من أن النتائج الجسدية لهذين الاثنين مختلفة تماما."
    - آسف، لم أفهم ما الذي كنت تتحدث عنه. هل يوجد فرق بين جزيئات الغاز والخلية البيولوجية؟ ما هي النتائج البدنية المختلفة؟

    "محليًا، القانون الإحصائي غير صالح، وقوانين الفيزياء صالحة، ولكن عندما تنظر إلى الصورة الكبيرة فهي بالتأكيد صالحة في عالمنا، وإلا فلن يكون لدينا سهم مادي للوقت. والسؤال هو كيف تصبح مجموعة من قوانين الفيزياء القابلة للعكس شيئا لا رجعة فيه. وأعتقد أن الأمر يتعلق بآلية انهيار الموجة."
    - وبما أنني لا أدعو إلى تكرار عمل الآخرين (وأكبر مني)، فسوف أحيلك إلى المقال:
    http://www.scholarpedia.org/article/Time%27s_arrow_and_Boltzmann%27s_entropy، في حال أردت ذلك. وأعتقد أنه سوف تساعدك.

  306. إسرائيل
    يعرّف قانون نيوتن الثاني القوة بأنها تغير في سرعة الكتلة، وللتحقق من هذا القصور الذاتي (الثبات) يشير إلى السرعة
    العادية أو الراحة قراءة الرابط من آخر مرة.

  307. نقطة.
    هل تعرف كتلة المادة الباريونية في الكون؟ ما هو نصف قطر الكون؟ إذا كان الأمر كذلك، يرجى الكتابة هنا، أو الاتصال على الرابط.

  308. نار.
    كيف يمكنك تعريف F=MA؟ لا يوجد قانون القصور الذاتي؟

    طالب.
    دعونا نتأكد من أننا نتحدث عن نفس الشيء.
    إذا كان لديك نظام في حالة إنتروبيا معينة، فإنه بسبب المبادئ الإحصائية سوف يميل إلى الانتقال إلى حالة إنتروبيا أعلى في المستقبل. أعتقد أنه لا يوجد جدل حول ذلك، والرابط الذي قدمته يتحدث عنه أيضًا.

    السؤال هو:
    لماذا، إذا طبقنا نفس المنطق بالضبط، لن يتحرك النظام نحو إنتروبيا أعلى في الماضي أيضًا؟ بعد كل شيء، كل شيء هنا قابل للعكس تماما من حيث قوانين نيوتن.

    وهذا لا علاقة له بالعمليات الديناميكية الحرارية القابلة للعكس. هذا موضوع مختلف.

    تمت مناقشة الموضوع في كتاب في أحد الكتب في الرابط الذي أرسلته لي "سهم الزمن ونقطة أرخميدس" ص 56. كما تم وصف جاذبية بوانكاريه هناك أيضًا، الذي ادعى أن ليس فقط انخفاض الإنتروبيا ممكن، لكنه أمر لا مفر منه.

    ر.ح.
    تحتاج إلى إنشاء سلسلة لجميع التعليقات قبل إرسال التعليق، وإعدادهم للأسوأ.
    كتبت زانوها أليك...
    كل الأحداث في كوزمو!

  309. إسرائيل، لست واضحًا على الإطلاق بشأن ما تريد. لقد أخبرتك ما هي صيغة نصف قطر شفارتزشيلد للثقب الأسود.
    من أين تحصل على M للكون؟ من بيانات WMAP؟ ففي نهاية المطاف، هذه هي نفس البيانات التي تظهر أن المادة الباريونية تشكل 5% فقط.
    أو ربما تريد اللعب الفردي؟ يمكنك البدء من الرقم 1 وباستخدام الوحدات تصل إلى أي معادلة فيزيائية تريدها. بعد كل شيء، يتأكد الفيزيائيون من أن المعادلات منطقية (متوازنة مع الوحدات على كلا الجانبين).

  310. 1) قوانين الديناميكا الحرارية ليست أولية، والحقيقة أنك درست الميكانيكا الإحصائية التي تشرح كيفية الحصول على قوانين الديناميكا الحرارية.

    2) لا توجد إجابة لأن القوانين الأولية المجهرية متماثلة في الزمن، والقوانين العيانية للديناميكا الحرارية ليست كذلك. ومع ذلك فإن كلاهما موجود في عالمنا. وليس هناك تفسير للانتقال من العالم الجزئي إلى العالم الكلي والنقاش كله يدور حول هذا الأمر. ربما عندما يبدأون بالحواسيب الكمومية سنعرف المزيد عن هذا الأمر.

    3) التوزيعات في الأنظمة التي تصفها مبنية على عدة افتراضات. وهذه هي النقطة بالضبط، أنه لا يوجد أي سبب من قوانين الفيزياء لوجود أنظمة التوازن هذه في الواقع.
    ولماذا يتم استخدام مثل هذه الأنظمة؟ لأن معادلة شرودنغر (أو معادلاتها النسبية) لا يمكن حلها لعدد كبير من الجسيمات. إذا كان ذلك ممكنًا، فلن يضطروا إلى استخدام الميكانيكا الإحصائية على الإطلاق.

    4) القانون الفيزيائي هو معادلة شرودنجر لهذه المسألة، والقانون الإحصائي هو قانون الديناميكا الحرارية. أعتقد أن الأمر واضح. لا يفصل القانون الإحصائي مجموعة الجزيئات التي تشكل الغاز عن مجموعة الجزيئات التي تشكل الخلية الملقحة. على الرغم من أن النتائج الجسدية لهذين الاثنين مختلفة تمامًا.
    محليًا، القانون الإحصائي غير صالح، وقوانين الفيزياء صالحة، ولكن عندما تنظر إلى الصورة الكبيرة فهي بالتأكيد صالحة في عالمنا، وإلا فلن يكون لدينا سهم مادي للوقت. والسؤال هو كيف تصبح مجموعة من قوانين الفيزياء القابلة للعكس شيئا لا رجعة فيه. وأعتقد أن الأمر يتعلق بآلية انهيار الموجة.

  311. الرد أعلاه هو في صلب الموضوع.

    إسرائيل،

    "وأيضًا في المرة الأخيرة التي أجبتك فيها أنه يبدو لي أنني أحدث فرقًا. وفقا لدورة كارنو. وهذا لا علاقة له بعكس العملية".
    – أنت من وضع الانقلابات وليس أنا. فإذا أردت حذف هذا المفهوم والعودة إلى ما كتبته:
    "لماذا، إذا كان كل شيء قابل للعكس ومتماثل، وقوانين الاحتمالية صالحة في كلا الاتجاهين، ألن تزداد الإنتروبيا أيضًا في اتجاه الماضي؟"
    - لماذا تزيد الإنتروبيا إذا عكست اتجاه السهم (للأمام)؟ بعد كل شيء، من تعريف الإنتروبيا سوف تقبل أن انخفاض الوزن الإحصائي سوف ينخفض.

    "ما أتحدث عنه، وأعتقد أنه نقطة مهمة أيضًا، هو أنه من الناحية النظرية، فيما يتعلق بسهم الزمن، يجب أن تزيد الإنتروبيا نحو المستقبل ونحو الماضي، بينما في الممارسة العملية فإنها تزيد فقط نحو المستقبل. "
    - أنا لا أفهم ما لا تفهمه أنت (أو أنت). أعتقد أن الرابط الذي قدمته لك يقدم شرحًا شاملاً لما تسميه مشكلة هنا. ربما إذا أعطيت مثالاً محددًا يمكنك توضيح ما تتحدث عنه.

    "إذا كنت تعتقد أنه لا توجد مشكلة هنا، فأنت على خلاف مع بوانكاريه وبولتزمان."
    - لقول.

  312. "بشكل عام، في الرابط الذي أعطيته الإجابة ليست جادة ولا دقيقة على الإطلاق."
    - ورفض مثل هذا الادعاء دون أن يذكر ما هو - أمر خطير ودقيق للغاية. بشكل عام، يمكن قول الشيء نفسه عن كلماتك. وبما أن مؤلفي الروابط التي قدمتها هم طلاب دراسات عليا ودكتوراه من معهد وايزمان، فإن المصداقية، حتى يثبت العكس، تظل معهم.

    "قوانين الديناميكا الحرارية ليست أساسية على الإطلاق."
    - إنهم كذلك، وهذا لم يرد فقط في الروابط التي قدمتها، بل في معظم السياقات التي درستها حتى الآن.

    "والجواب في رأيي يتعلق بالانتقال من العالم الكمي الجزئي إلى العالم الكلي الكلاسيكي ويرتبط بانهيارات الموجات بطريقة خفية."
    - يعني ليس لديك إجابة وترفضين قبول الموجود؟

    "تفترض الميكانيكا الإحصائية وظائف توزيع الجسيمات. ولم يتمكن أحد من حل معادلات شرودنغر للحصول على هذه الدوال.
    - لم أفهم ما هي العلاقة بين دوال التوزيع في الميكانيكا الإحصائية (التي ليست مفترضة على الإطلاق، يتم الحصول عليها من تحليل النظام - سواء كان كلاسيكيًا هو ما يؤدي إلى ماكسويل-بولتزمان أو ما إذا كان كميًا هو ما يؤدي إلى فيرمي-ديراك، أو بوز-آينشتاين أو البدائل البديلة مثل الإحصاء)، وعلاقة معادلة شرودنغر وانهيار الدالة الموجية، لهذا النقاش.

    "وإذا حاول شخص ما الوقوف في الطابور، فلن يتمكن من الوصول إلى وظائف التوزيع هذه، لأن آلية الانهيار ليست جزءًا من معادلة شرودنغر. إنها آلية أخرى غير واضحة".
    – مرة أخرى، أنا لا أفهم الادعاء أو ما تقصده. ففي النهاية، معادلة شرودنغر ليست ضرورية على الإطلاق (لكن من الضروري أن نأخذ في الاعتبار الطبيعة الكمومية للجسيمات) للوصول إلى توزيع فيرمي-ديراك (على سبيل المثال)، فأين الاتصال؟

    "إن مناقشة السؤال هي كما لو كانت فلسفية. هذا سؤال مادي لكل شيء. وحقيقة أنها لا تملك إجابة في الوقت الحالي لا يجعلها فلسفية."
    - قلت إن النقاش حول مسألة ما هي "قوانين الفيزياء" أو ما هو الاحتمال ولماذا يعمل، هو نقاش فلسفي.

    "ومرة أخرى فيما يتعلق بالتطور، مازلت في حيرة من أمرك. التطور مثال على أن ما يحكم الطبيعة هو القانون الفيزيائي، وليس القانون الإحصائي، وهذا لا يعني أن قوانين الديناميكا الحرارية قد تم انتهاكها، بل يعني فقط أنهم لا يستطيعون التنبؤ بأي شيء مثل الحياة على الإطلاق.
    1. ما هو/من هو القانون الفيزيائي والقانون الإحصائي؟
    2. لماذا تعتقد أنهم لا يستطيعون التنبؤ؟ لأنه ليس لديك طريقة لوصف نظام معقد مثل ذلك الذي أدى إلى تطور الحياة؟
    3. أعتقد أنك تناقض نفسك. من ناحية أنت تقول أن قوانين الديناميكا الحرارية صالحة (غير منتهكة) ومن ناحية أخرى تدعي أنها لا تستطيع التنبؤ بالسيناريو الذي سيحدث (مثل خلق الحياة) - وهو جزء من جوهر القانون الثاني .

  313. إسرائيل،

    ردي ينتظر التأكيد في أقبية الستاسي، المسكينة أنهم مزقوها إرباً بالتعذيب ومع ذلك لم تعترف بالتصيد أو أي ضرر آخر. أعتقد أنه سيتم إطلاق سراحها قريبا.

  314. نقطة.

    G = ثابت الجاذبية.
    ج = سرعة الضوء.
    M = الكتلة المقدرة للكون. (يمكن العثور عليها في ويكيبيديا).
    R = نصف القطر المقدر للكون. (نفس ما ورد أعلاه).

    إذن: GM=RC^2 تقريبًا.

    إذا استخدمنا تحليل الأبعاد، بعد كل التخفيضات على جانبي المنارة، لم يتبق لدينا سوى:

    و = ماجستير

    قانون نيوتن الثاني، قانون القصور الذاتي.

    السؤال:

    فكيف يتناسب هذا مع فكرة أن هناك كتلة أكبر بخمس مرات على الأقل مما هو مقبول؟ ألا ينبغي لنا أن نضيف عاملاً آخر في مثل هذه الصيغة الأنيقة؟ ألا ينبغي لهذا التناقض وحده أن يقتل فكرة الكتلة المظلمة برمتها؟ بعد كل شيء، ليس هناك شك تقريبًا في قيم G وR وC.

    أو ربما M في الصيغة يتضمن بالفعل الكتلة المظلمة؟

    طالب.

    "- لماذا وكيف توصلت إلى استنتاج مفاده أن العملية الموضحة في الرابط الذي قدمته قابلة للعكس؟ حتى في المرة الأخيرة التي ناقشت فيها معك، كان لدي انطباع بأنك لا تميز بين العملية القابلة للعكس والعملية التي لا رجعة فيها".

    وأيضاً في المرة الأخيرة التي أجبتك فيها أنه يبدو لي أنني أصنع فرقاً. وفقا لدورة كارنو. وهذا لا علاقة له بعكس العملية. ما أتحدث عنه، وأعتقد أنه نقطة مهمة أيضًا، هو أنه من الناحية النظرية، فيما يتعلق بسهم الزمن، يجب أن تزيد الإنتروبيا نحو المستقبل ونحو الماضي، بينما في الممارسة العملية فإنها تزيد فقط نحو المستقبل.

    إذا كنت تعتقد أنه لا توجد مشكلة هنا، فأنت على خلاف مع بوانكاريه وبولتزمان. هناك عدة تفسيرات لهذه الظاهرة، والكم هو أحد هذه التفسيرات.

    كنت أحاول أن أبين أن نظرية الانفجار الأعظم قد تلقي بعض الضوء على المشكلة، لكننا نستمتع قليلاً، لذا سنترك الأمر عند هذا الحد.

    ر.ه.؟؟؟ الساعات، الجوزاء، الأميات.

  315. تسعى جميع أشكال الطاقة دائمًا إلى تقليل الطاقة الحرارية. الحرارة - انكسر صوت بولتزمانسكي - الدفء اللطيف الذي نحتضنه - هنا أدنى شكل من أشكال الطاقة - أصبح صوته همسًا - الأكثر حقيرة -
    جاء الجمهور بالبكاء.
    - قف، قف - طبقت القانون. - الحرارة والطاقة والعمل - ما كل هذا وأنا؟ انت مثيرة؟ شغل المكيف الهوائي!
    - وماذا يفعل مكيف الهواء؟ زأر الأستاذ - تبريد نظام مغلق على حساب آخر؟ - يجب أن تفهم، مخاطبًا الجمهور، أن مكيف الهواء، على عكس الفرن على سبيل المثال، لا يمكن تضمينه في نظام مغلق. وهذا هو سبب تثبيتها عادة على الحائط - نصفها في نظام مغلق، المنزل، والنصف الآخر في الهواء الطلق، ومن الضروري دائمًا إمدادها بالطاقة من مصدر خارجي. هل تفهم العبثية؟ رفع الأستاذ إصبعه عتابًا - علينا أن نستثمر الطاقة لإخراج الطاقة من الغرفة. حقا، إلى أي مدى يمكن أن تذهب؟
    - لنفس السبب تمامًا، لا يمكننا إنتاج عمل مفيد من الحرارة مباشرة. لن تتمكن الطائرة من التحرك للأمام عن طريق تبريد الهواء الذي تمر عبره. جميع الأنظمة التي تنتج العمل من الحرارة - المحرك البخاري على سبيل المثال - يجب أن تكون أكثر سخونة من البيئة المحيطة. وتقتصر كفاءتها في استخدام الطاقة، وهي منخفضة للغاية، على الفرق في درجة الحرارة بين النظام والبيئة، مقسومًا على درجة الحرارة المطلقة.
    -بروفيسور بولتزمانسكي- تدخل المدعي العام بلطف، -لا أنوي الدفاع عن المتهم، لكن ألم تنحرف قليلاً عن الموضوع؟ ففي نهاية المطاف، لا تتم محاكمة المدعى عليه بسبب نشاطه الديناميكي الحراري. هل يمكنك أن تصف لنا الأضرار التي سببها نشاطه للحياة؟
    - صحيح جدًا - اقرأ القانون - أنا وصديقي الجديد المدعي العام وأصدقائي من الجمهور الذين قمت بتخديرهم بشكل جميل للغاية أريد أخيرًا أن أفهم: ماذا فعلت؟ لماذا انا هنا
    -فوضى. أجاب الأستاذ.
    -ماذا؟ سأل القانون والمدعي العام وعدد قليل من الأصوات الأخرى في الجمهور.
    -لقد أحدثت فوضى. فوضى سياسية. فوضى. بسبب ميل الطاقة إلى التبدد، فإن اتجاه الأشياء في الطبيعة هو دائمًا زيادة الإنتروبيا، أو الفوضى. يصبح المنزل دائمًا فوضويًا من تلقاء نفسه - ولكن من أجل ترتيبه، علينا أن نبدأ العمل. سوف ينكسر الكوب الصيني إذا سقط، لكن الشظايا لن تلتصق ببعضها البعض من تلقاء نفسها. سوف تصدأ السيارة وتصبح قديمة، ولن تصبح جديدة مرة أخرى بدون عمل خارجي.
    - ليس بالضبط.. تمتم بالقانون مرة ثانية.
    -نعم بالضبط. لقد أدركت بالفعل اتجاه تدفق الطاقة - إلى الحرارة، الطاقة الأدنى - أقر الآن بالاتجاه الذي تقود فيه كل الأشياء في العالم: التفكك والفوضى. لولا الحياة، لكان كل شيء في الكون يسعى باستمرار إلى أدنى شكل ممكن من النظام.
    إن مفهوم الشغل في حد ذاته - والذي تم تعريفه في الفيزياء على أنه نتاج القوة على طول الطريق - هو في الواقع عكس الإنتروبيا. العمل المستمر موجود فقط في الأنظمة البيولوجية أو في الأنظمة التي أنشأتها الأنظمة البيولوجية. في الحياة الساكنة، لا توجد مكيفات هواء، أو أي آلات أخرى، ويمكن للنظام أن يقلل من الإنتروبيا المحلية بطريقة واحدة فقط: التوسع.
    ومن ناحية أخرى، في الأنظمة الحية، يمكن تقليل الإنتروبيا المحلية عن طريق امتصاص الطاقة الخارجية - كما تفعل النباتات في عملية التمثيل الضوئي. الآلات التي يصنعها الإنسان تفعل ذلك أيضًا. وفي الحقيقة الحياة هي تناقض كامل للقانون الثاني وأهدافه الضائعة، انتصار الخير والنبيل على الشر والمدمر..
    -إذا كان جيدًا جدًا فلماذا سيئًا جدًا؟ سأل القانون باللغة الروسية.
    - توقف، السخرية لن تساعدك. تحاول تقديم نفسك على أنك مرقط ومؤذ، لكننا سنكشف عن وجهك الحقيقي: المخرب! مدمرة! أنت سبب كل مشاكلنا وأمراضنا! أنت سبب الأعطال في حياتنا، في شيخوخةنا، أيها القاتل! أنت تجعلنا جميعا....
    بكى الحشد بمرارة.
    - حقًا يكسر القلوب - تذمر الناموس، - وأنت أيضًا ستسلك طريق كل الجسد -
    - خلصت الأستاذ مسح عيونه الدامعة بالمنديل. - يمكننا الاستمرار في وصف المآثر الإجرامية للمتهم. لكن المبدأ سيبقى كما هو: إلى أن يُلغى القانون الثاني، أو تتغير طبيعته التدميرية، فإن الحياة ستظل تعاني إلى الأبد من اضطراباتها ومضايقاتها.
    - إما نحن - وإما هو.

  316. بشكل عام، في الرابط الذي أعطيته الإجابة ليست جادة ولا دقيقة على الإطلاق.
    قوانين الديناميكا الحرارية ليست أساسية على الإطلاق. هذه هي القوانين الإحصائية التي تنطبق في العالم الكلي، عندما يكون هناك عدد كبير من الجزيئات. والسؤال برمته هو كيف يحدث ذلك. والجواب في رأيي يتعلق بالانتقال من العالم الكمي الجزئي إلى العالم الكلي الكلاسيكي ويرتبط بانهيارات الموجات بطريقة خفية.
    بالطبع، في الدورة لا تتعرف على انهيار الدوال الموجية. في الميكانيكا الإحصائية، يتم افتراض وظائف التوزيع للجسيمات. لم يتمكن أحد من حل معادلات شرودنغر للحصول على هذه الدوال. وإذا حاول شخص ما الوقوف في الطابور، فلن يتمكن من الوصول إلى وظائف التوزيع هذه، لأن آلية الانهيار ليست جزءًا من معادلة شرودنغر. إنها آلية أخرى غير واضحة.
    مناقشة السؤال كما لو كانت فلسفية. هذا سؤال مادي لكل شيء. حقيقة أنها لا تملك إجابة في الوقت الحالي لا يجعلها فلسفية.

    ومرة أخرى فيما يتعلق بالتطور، لا تزال في حيرة من أمرك. التطور مثال على أن ما يحكم الطبيعة هو القانون الفيزيائي، وليس القانون الإحصائي، وهذا لا يعني أن قوانين الديناميكا الحرارية قد تم انتهاكها، بل يعني فقط أنهم لا يستطيعون التنبؤ بأي شيء مثل الحياة على الإطلاق.

  317. إسرائيل،
    آسف، للوهلة الثانية رأيت أن التعليق على التطور هو نقطة قالها، فالجواب في هذا الموضوع موجه إليه.

  318. نقطة،

    أنت مخطئ. هناك شرح لضغط الوقت، وقد تم تقديمه بشكل شامل في المقالتين اللتين ربطتهما بهما. ولا أرى سبباً لإصرارك على أنه لا يوجد أي تفسير. على أية حال، بالطبع لا أستطيع القراءة لك.

    بعض التعليقات:
    1. تعتبر قوانين الديناميكا الحرارية من القوانين الأساسية للطبيعة.
    2. يتم تقديم تفسير القانون الثاني في إطار الفيزياء / الميكانيكا / الديناميكا الحرارية الإحصائية. تدخل ميكانيكا الكم في الفيزياء الإحصائية، لكنني لم أسمع عن انهيار الدالة الموجية في الدورة، ولكي أكون صادقًا، لا أعرف حقًا ما الذي تتحدث عنه في سياق المناقشة.
    3. أنت تسحب المناقشة في اتجاه فلسفي ولست مهتمًا بالذهاب إلى هناك. ما هو الاحتمال وما هي "قوانين الفيزياء" هي أسئلة فلسفية. ومن الناحية العملية، لا توجد دالة موجية ولا قوانين ولا فيزياء، بل يوجد فقط ما نستنتجه من دراسة العالم من حولنا.

    "لقد أحضرت التطور كمثال على أننا لا نحصل دائمًا على أدنى مستويات الطاقة."
    - والمكيفات ليس من المفترض أن تعمل؟ لا يوجد شيء في هذا البيان. انت كتبت:
    "لا يوجد عامل إحصائي من شأنه أن يتسبب بالضبط في حدوث المكون الذي يحتوي على أكبر عدد من الحالات. وأفضل مثال على ذلك هو التطور."
    وهذا ليس مجرد مثال جيد. إن خلق الحياة، حتى لو لم تدرك ذلك أو تراه، زاد من إنتروبيا الكون.

  319. إسرائيل،

    هذه المناقشة تتكرر بطريقة دائرية. لا أرى أي تناقض ولا أستطيع أن أفهم ما التناقض الذي تراه.
    الإجابة على أسئلتك بسيطة ومتماسكة مع مفهوم تسريع الزمن حسب السرعة
    ستظهر ساعات السيزيوم فرقًا، ساعة في حالة واحدة ومليار سنة (حسب أقصى تقديرك) في الحالة الأخرى. سيظهر مقياس درجة الحرارة نفس الشيء تمامًا. אם שני החבר'ה האלו היו מציירים גרף של מד הטמפ' לאורך זמן היית רואה שקצב ההתקררות במקרה של ההוא שנח הוא איטי (נגיד מ – 6000K ל- 3 במילארד שנה) בעוד שהגרף של השני היה מראה קצב מהיר 6000K ==> 3 בתוך الساعة.

    ما السر؟؟؟ ما الذي يعتقد الأساتذة والعلماء المؤهلون أنك لا تفهمه، لا تفهمه؟
    في إسرائيل، كان هناك نقاش هنا حول المسائل الرياضية والعلمية التي كانت موجهة إلى يوفال. ومن أهم علامات الفتنة العلمية الهجوم على المؤسسة والاستهتار بالعلماء الحاضرين الذين لم يروا ما يرى، فلا ينبغي لك أن تسير في ذلك الاتجاه، لئلا تصبح كذلك.
    انظر على سبيل المثال هنا:
    http://www.gadial.net/?category_name=%d7%94%d7%91%d7%9c%d7%99%d7%9d-%d7%a4%d7%a1%d7%90%d7%95%d7%93%d7%95-%d7%9e%d7%aa%d7%9e%d7%98%d7%99%d7%99%d7%9d

    فيما يتعلق بالإنتروبيا، على الرغم من أنك تعرف عددًا من العدادات أكثر مني وقرأت مقالتك عن غاليليو التي اقتبستها أعلاه، أعتقد أنك مخطئ عندما تقول "لا يوجد عامل إحصائي من شأنه أن يتسبب بالضبط في المكون الذي يحتوي على أكبر عدد من حالات حدوثها." بالطبع هناك. العب لمدة 5 دقائق بالمكعب المجري وسترى الرخام.

    "من وجهة نظر إحصائية حيوية، فإن الحالة التي لا توجد بها حياة لديها طاقة أقل من الحالة التي بها حياة. ومع ذلك حدثت الحياة. إنه يظهر فقط أنه لا يوجد عامل إحصائي يفرض نفسه على الكون.

    – هذه إحدى الحجج الخلقية القديمة والمتهالكة. الأرض ليست نظامًا مغلقًا، وهناك حقنة من الطاقة من الشمس، لذلك محليًا هناك انقلاب لسهم الإنتروبيا. وبنفس الطريقة يمكن للمرء أن يسأل كيف تشتعل النار؟ بعد كل شيء، أدنى حالة طاقة هي شجرة ميتة، ولكن حقن الطاقة فيها وسوف تضيء والنار سوف تستمر وحتى تنتشر.

  320. مرحبًا أيها الطالب،

    انظر، وفقا لقوانين الفيزياء، الزمن متماثل، ولا ينبغي لنا أن نقبل أي فرق بين الماضي والمستقبل.
    القانون الثاني للديناميكا الحرارية ليس قانونًا أساسيًا للطبيعة (على الرغم من وجوده دائمًا في عالمنا)، ولكنه نتيجة للتجارب ومبادئ الحفظ.
    الفيزياء الكلاسيكية المتناظرة في الزمن لا يمكنها تفسير نتيجة مثل القانون الثاني. العامل الوحيد الذي نعرفه والذي يمكن أن يفسر شيئًا مشابهًا للقانون الثاني هو آلية الانهيار (غير المعروفة) للدالة الموجية، لأنها آلية غير متماثلة ننتقل فيها من التراكب إلى حالة واحدة.

    أيها الطالب، الاحتمال ليس جزءًا من قوانين الفيزياء، لذلك ليس من المفترض أن يعمل في عالمنا بهذه الطريقة. المكان الوحيد الذي يقدم احتمالية الحدوث هو تفسير معين لميكانيكا الكم.

  321. إسرائيل،

    "لماذا، إذا كان كل شيء قابل للعكس ومتماثل، وقوانين الاحتمالية صالحة في كلا الاتجاهين، ألن تزداد الإنتروبيا أيضًا في اتجاه الماضي؟"
    - لماذا وكيف توصلت إلى استنتاج مفاده أن العملية الموضحة في الرابط الذي قدمته قابلة للعكس؟ حتى في المرة الأخيرة التي ناقشت فيها معك، كان لدي انطباع بأنك لا تميز بين العملية القابلة للعكس والعملية التي لا رجعة فيها.

    نقطة،

    "لا يوجد عامل إحصائي من شأنه أن يتسبب على وجه التحديد في حدوث المكون الذي يحتوي على أكبر عدد من الحالات."
    - لا يوجد عامل من هذا القبيل، بل يحدث فقط. هل تتساءل لماذا يعمل الاحتمال؟ لماذا يوجد القانون الثاني؟ هذا بالفعل سؤال فلسفي. معظم القوانين الأساسية في الطبيعة ليس لها تفسير لـ "لماذا". الرابط الخاص بالموضوع :
    http://davidson.weizmann.ac.il/online/askexpert/physics/%D7%9E%D7%93%D7%95%D7%A2-%D7%9E%D7%AA%D7%A7%D7%99%D7%99%D7%9D-%D7%94%D7%97%D7%95%D7%A7-%D7%94%D7%A9%D7%A0%D7%99-%D7%A9%D7%9C-%D7%94%D7%AA%D7%A8%D7%9E%D7%95%D7%93%D7%99%D7%A0%D7%9E%D7%99%D7%A7%D7%94-%D7%A9%D7%A8%D7%95%D7%9F

    "من وجهة نظر إحصائية حيوية، فإن الحالة التي لا توجد بها حياة لديها طاقة أقل من الحالة التي بها حياة. ومع ذلك حدثت الحياة. إنه يظهر فقط أنه لا يوجد عامل إحصائي يفرض نفسه على الكون.
    - لا. إن خلق الحياة لا يتعارض مع القانون الثاني للديناميكا الحرارية وأن إنتروبيا الكون تزداد في عملية التطور.

    "لذلك، لا يوجد تفسير كلاسيكي حقيقي لظاهرة الإنتروبيا."
    - أعتقد أن هناك.

    "وحدها ميكانيكا الكم يمكنها تفسير ذلك."
    - لقول.

  322. إسرائيل، لم توضح ما هي الثوابت في صيغتك. هل تعلم أن نصف قطر الثقب الأسود هو R=2Gm/c^2؟

    من الناحية المادية، لا يوجد شيء خاص في وقت معين. حقيقة أن وعيك يمر بوقت معين ليس لها أي معنى مادي (كما نفهم الفيزياء اليوم، الوعي ليس له وظيفة). لذلك يمكن القول أن وعينا ككل يتجول حسب الزمن. يبدو الأمر كما هو الحال في الفيلم، حيث لا يمكنك التساؤل عما إذا كان ما حدث الآن سيؤثر على ما سيحدث في المستقبل أو في الماضي للفيلم (لأن الفيلم بأكمله قد تم تحريره بالفعل).

    تعتقد أنني سأمر بكل ما كتبته.
    اطرح سؤالاً محددًا (وليس سؤالًا طنانًا كما لو كنت تحاول جرني إلى مناقشة معينة).

  323. إلى نظام المعرفة
    في الآونة الأخيرة، تأخرت العديد من ردودي لفترة أطول من المعتاد. هذا يجعلني أبدأ في الشك في أن هذه ليست صدفة عشوائية. الرد المنطوق الحالي هو
    https://www.hayadan.org.il/astronomers-reach-new-frontiers-of-dark-matter-130112/#comment-331370
    يرجى التحقق وإخباري عن أسباب التأخير وكيف يمكنني تجنب تأخيرات مماثلة في المستقبل. وغني عن القول أيضًا أنني سأكون ممتنًا إذا تم إصداره

  324. نقطة.
    إذا تحدثنا هنا فقط عن الأشياء غير المعروفة، فمن الصعب أن نتحدث على الإطلاق.
    أعتقد أنه قد يكون هناك سبب آخر، وهو ضغط الوقت، لكن الأمر لا يستحق النظر في اتجاهات أخرى في الوقت الحالي.
    تعليقاتك الأخيرة منطقية ومفيدة وتظهر معرفة كبيرة. سأكون مهتمًا بأفكارك حول بعض المواضيع التي تم طرحها مؤخرًا:

    1. إذا أخذنا الصيغة GM=RC^2، فلقد حصلنا على صيغة جميلة تصف العلاقة بين الثوابت، ومن بينها كتلة الكون وليس الكتلة المظلمة.

    فكيف يتناسب هذا مع فكرة أن هناك كتلة أكبر بخمس مرات على الأقل مما هو مقبول؟ ألا ينبغي لنا أن نضيف عاملاً آخر في مثل هذه الصيغة الأنيقة؟ ألا ينبغي لهذا التناقض وحده أن يقتل فكرة الكتلة المظلمة برمتها؟ بعد كل شيء، ليس هناك شك تقريبًا في قيم G وR وC.

    أو ربما M في الصيغة يتضمن بالفعل الكتلة المظلمة؟

    2. كلامك: "فيما يتعلق بالتأثير من الحاضر إلى المستقبل ونحوه. لا توجد مثل هذه الأشياء. بمعنى ما، الماضي والمستقبل موجودان بالفعل، ونحن نمر عبرهما فقط. بالطبع، أسئلة ميكانيكا الكم تأتي هنا أيضًا."

    لا أفهم. يبدو قليلا الفلسفية. هل يمكنك التوسع

    3. إذا تمكنت من المتابعة والعثور على خطأ في المناقشة التي أجريتها مع R.H. فيما يتعلق بما يسمى التناقض بين إطالة الزمن في العلاقات ونظرية الانفجار الأعظم. بدءا من:

    https://www.hayadan.org.il/astronomers-reach-new-frontiers-of-dark-matter-130112/#comment-330884

    إذا كان ذلك ممكنا، فقط الإجابات الفردية لهذه النقطة، لا التعميمات.

    شكرًا لك، وليلة سعيدة من لوس أنجلوس.

  325. أيها الطالب، مفهوم الفوضى الجزيئية لا يمكن أن يفسر الفوضى. لولا ميكانيكا الكم، لم يكن هناك عامل إحصائي من شأنه أن يتسبب بالضبط في حدوث المكون الذي يحتوي على أكبر عدد من الحالات. وأفضل مثال على ذلك هو التطور. من وجهة نظر الطاقة الإحصائية، فإن الحالة التي لا توجد بها حياة لديها طاقة أقل من الحالة التي بها حياة. ومع ذلك حدثت الحياة. إنه يظهر فقط أنه لا يوجد عامل إحصائي يفرض نفسه على الكون.
    لذلك، لا يوجد تفسير كلاسيكي حقيقي لظاهرة الإنتروبيا. فقط ميكانيكا الكم يمكنها تفسير ذلك.

  326. إسرائيل مسألة سهم الزمن مشكلة معروفة.

    يبدو أن الحل تم العثور عليه في عامل مهم جدًا في ميكانيكا الكم غير واضح على الإطلاق، وهو انهيار الدوال الموجية. فقط هذا العامل غير الواضح يمكنه تفسير الاتجاه غير الواضح للوقت (وهو في الواقع رمية النرد الشهيرة).

    وأما التأثير من الحاضر إلى المستقبل ونحو ذلك. لا توجد مثل هذه الأشياء. بمعنى ما، الماضي والمستقبل موجودان بالفعل، ونحن نمر عبرهما فقط. بالطبع، أسئلة ميكانيكا الكم تأتي هنا أيضًا.

    على كل حال هذه مسائل معروفة ولا جديد فيها.

  327. وبخطوات واثقة تقدم وكيل النيابة إلى المنصة وبدأ تحقيقاته في القانون الثاني:

    الذي - التي. أيها المدعى عليه، هل يمكنك أن تخبر المحكمة لماذا تزيد الإنتروبيا دائمًا؟

    أ. بدون سبب يحدث فقط.

    الذي - التي. لكن لماذا؟ هل من المستحيل تركها كما هي؟ الحد منه؟

    أ. ممكن، ولكن من غير المرجح.

    الذي - التي. ماذا يعني ذلك؟

    أ. تعطي قوانين العشوائية الرياضية احتمالية منخفضة للغاية لتقليل الإنتروبيا.

    الذي - التي. لا أفهم هل تزعم أن الرياضيات، وهي البنية الأكثر منطقية وتنظيمًا، هي سبب الفوضى المتزايدة؟ هل ذلك منطقي بالنسبة لك؟ وخاطب المدعي العام الجمهور بسخرية.
    صفير الجمهور في ازدراء.
    أ. نعم.

    قال المدعي العام باشمئزاز: "لا أصدق ما أسمعه". "لكنني لست عالم رياضيات محترفًا. أود استدعاء البروفيسور ليبنوفيتس، رئيس معهد غاوس للأبحاث الحسابية، للإدلاء بشهادته".

    القانون مملوء بالبهجة. "جمال! من المؤكد أن عالم الرياضيات سيثبت ادعاءاتي وصحتي. أنا أحب البراهين الرياضية. أتمنى أن يكون هذا دليلاً سلبيًا، فهذه هي الأشياء المفضلة لدي." اتكأ على ظهر الكرسي متوقعا.
    أخذ البروفيسور ليبنوفيتس مكانه على المنصة وفتح الباب. "من الواضح أن القانون صحيح. ومن حيث الاحتمالية الرياضية البحتة، هناك حالات فوضى في النظام أكثر بكثير من حالات النظام."
    ابتسم القانون بتواضع قليلاً وهو يقول: "حسناً، ألم أخبرك؟"
    قام الأستاذ بسحب مجموعة من البطاقات المختومة وفتحها أمام الجمهور. "نرى؟ البطاقات في حالة الحد الأقصى للطلب. في البداية سلسلة الوجه، عندما يتم ترتيب البطاقات بالتسلسل من الأصغر إلى الأكبر. وبعد ذلك سلسلة القلب، ثم البرسيم وأخيراً سلسلة الماس عندما يتم ترتيبها جميعها بنفس الطريقة، من الأصغر إلى الأكبر".
    "الآن، إذا قمت بتبديل المجموعة بخفة، فسوف ينخفض ​​الترتيب في المجموعة. ستبقى هناك بالفعل سلسلة صغيرة من البطاقات التي "تلتصق" ببعضها البعض، لكن الترتيب في المجموعة سيتناقص، وسيستمر في الانخفاض مع استمراري في الخلط، حتى تصل إلى حالة من الفوضى القصوى، أو الإنتروبيا.
    وفي كلمته، يخلط الأستاذ الأوراق أمام الحضور، فيما يشمت القانون من كرسيه.
    "نفس الشيء سيحدث لغرفة مرتبة تهب فيها رياح عاصفة. سوف ينخفض ​​مستوى النظام مع مرور الوقت. إذا قمنا بتصوير الغرفة في مواقف مختلفة ثم قمنا بمزج الصور، فيمكننا دائمًا ترتيبها مرة أخرى بترتيب زمني وفقًا لحالة الاضطراب في الصورة: كلما زادت الفوضى أو الإنتروبيا في الصورة، كلما كانت الصورة متأخرة. هل توافق؟"
    هتفت الجماهير بالموافقة، بينما أشرق القانون بالسعادة.
    "وهذا هو السبب أيضًا في تعريف سهم الزمن على أنه اتجاه زيادة الإنتروبيا. في الطبيعة، في نظام مغلق، مع مرور الوقت، تزداد الإنتروبيا، أو تظل ثابتة لسوء الحظ. ولا ينخفض ​​إلا في حالات نادرة وخاصة.
    والسبب في ذلك هو الرياضيات البحتة. هناك ببساطة العديد من الحالات المحتملة ذات الإنتروبيا العالية، أكثر من الحالات المرتبة."
    "خاص." قال القانون بسرور. كنت أعلم أن الرياضيات ستكشف براءتي!"
    "غير صحيح!" قال الأستاذ. الرياضيات تثبت العكس تماما! إنه خطؤك!"
    "ماذا؟" صرخ القانون: "لقد أثبتت لنا للتو أن الإنتروبيا يجب أن تزداد دائمًا لأسباب رياضية بحتة تتعلق بالاحتمال!"
    "صحيح، ولكن هذه مجرد صورة جزئية! ويظل السؤال مفتوحا: لماذا؟".
    قال القانون ببرود: «يبدو لي أنك شرحت الأمر جيدًا بمثال مجموعة أوراق اللعب. "هناك العديد من الحالات ذات الإنتروبيا العالية أكثر من الحالات المرتبة" - نقلاً عن.
    هذا صحيح، ولكن لماذا تتحول الإنتروبيا في الاتجاه الذي تختاره مع مرور الوقت؟ لماذا لا يقوم خلط الأوراق بترتيب الأوراق في المجموعة بدلاً من إفسادها؟ لماذا لا ترتب الريح الغرفة؟ لماذا لا نصبح أصغر سنا مع مرور الوقت بدلا من الشيخوخة؟
    تمتم بالقانون: "كل هذا خطأي، كالعادة".
    "بالضبط" وافق الأستاذ. "أنت في حالة من الفوضى تماما. أنت تعرف فقط كيف تسبب الكثير من المتاعب. هل هناك أي تفسير آخر لماذا تزداد الإنتروبيا دائما مع مرور الوقت؟ قوانين نيوتن للحركة لا تتطلب ذلك! إذا قمنا بتغذية جهاز كمبيوتر ببيانات نظام نجمي معين، فسيكون قادرًا على التنبؤ بدقة كبيرة بحالته خلال ألف عام - وأيضًا بحالته قبل ألف عام! لماذا لا ينطبق مبدأ التكافؤ هذا أيضًا على النظام الديناميكي الحراري؟"
    "كما قلت، الرياضيات .."
    "يا!" وكان الأستاذ متحمسا. "لقد ارتكبت خطأً في هذه المسألة، ويمكنني إثبات ذلك إذا أحضرت لي سبورة وطباشير".
    يمتد الجمهور في مقاعدهم في ترقب متوتر، بينما تهتز الشمس في قاعة السبورة المحمولة.

  328. طالب علم
    من الرابط الذي أرسلته لي:
    "يمكننا أن نسأل أنفسنا السؤال التالي: من ناحية، قوانين نيوتن لا تميز بين الماضي والمستقبل - معادلات الحركة متناظرة بالنسبة للزمن (أي يمكنك استبدال t بـ t في المعادلات، و سيكونون على حق بنفس القدر)."

    أنظر إلى الصورة المرفقة مع الرابط. لديها ثلاث حالات من الإنتروبيا وسهم زمني يشير إلى اليمين، إلى المستقبل.
    الحالة اليسرى هي حالة من الإنتروبيا المنخفضة، والوسطى من الإنتروبيا المتوسطة، والحالة اليمنى من الإنتروبيا العالية.

    إذا تصورنا الموضع الأوسط في المضارع، يمكننا أن نسمي صاحب اليد اليسرى الماضي، وصاحب اليد اليمنى المستقبل.

    1. انظر إلى الموضع الأوسط فقط، وقم بعكس اتجاه السهم إلى الاتجاه السابق.
    2. انظر إلى الاقتباس أعلاه.
    سؤال: لماذا، إذا كان كل شيء قابل للعكس ومتماثل، وقوانين الاحتمالية صالحة في كلا الاتجاهين، ألن تزداد الإنتروبيا أيضًا في اتجاه الماضي؟

  329. إسرائيل،

    http://davidson.weizmann.ac.il/online/askexpert/physics/%D7%9E%D7%94%D7%95-%D7%9B%D7%90%D7%95%D7%A1-%D7%9E%D7%95%D7%9C%D7%A7%D7%95%D7%9C%D7%A8%D7%99-%D7%95%D7%9B%D7%99%D7%A6%D7%93-%D7%94%D7%95%D7%90-%D7%9E%D7%A9%D7%A4%D7%99%D7%A2-%D7%A2%D7%9C-%D7%AA%D7%A4%D7%99%D7%A1%D7%AA-%D7%97%D7%A5-%D7%94%D7%96%D7%9E%D7%9F-%D7%91%D7%A2%D7%91%D7%95%D7%93%D7%AA%D7%95-%D7%A9%D7%9C-%D7%91%D7%95%D7%9C%D7%A6%D7%9E%D7%9F-%D7%9E%D7%99%D7%9B%D7%90%D7%9C-0

    بخصوص سؤالك:
    "هل تعتقد أنه من الممكن بأي شكل من الأشكال التأثير على الماضي من الحاضر أو ​​المستقبل؟
    ... "
    - ليس لدي إجابة (أو رأي) في هذا السؤال وما بعده. أعتقد أن البروفيسور ياكير أهارونوف قام بالبحث/البحث في المجالات المتعلقة بمسائل مثل هذه.

  330. طالب.
    في ذلك الوقت، عندما ناقشنا موضوع الإنتروبيا، كانت النقطة التي أردت منا أن نصل إليها وفشلنا في ذلك، هي إمكانية عكس الإنتروبيا في كلا الاتجاهين الزمنيين.

    الفكرة هي كما يلي: إذا كانت قوانين نيوتن قابلة للعكس فيما يتعلق بالوقت، وإذا كان الكمبيوتر قادرًا على حساب موقع الأجرام السماوية بعد 1000 عام من الآن، ولكن أيضًا قبل 1000 عام، إذا كان النظام الديناميكي الحراري لا يختلف بشكل أساسي عن كرات البلياردو على سطح الأرض. الجدول، إذا كانت قوانين الاحتمال تتنبأ بأن الإنتروبيا يجب أن تزيد مع مرور الوقت، فلماذا لا تزيد عندما نعكس اتجاه الزمن إلى اتجاه الماضي؟

    إذا كانت هناك عاصفة من الرياح تهب في الغرفة ويمكننا معرفة ما هو مبكر وما هو متأخر في الصور من الغرفة فقط من خلال درجة الفوضى في الصور، وهذا يرجع فقط إلى اعتبارات الاحتمالية، فلماذا لا ألا تتنبأ نفس القوانين بأن الإنتروبيا ستزداد أيضًا في اتجاه الماضي؟ إذا كان الجليد في البحيرة يذوب في الربيع باتجاه المستقبل، فلماذا لا يذوب تجاه الماضي، وهذا بالضبط من نفس الاعتبارات الاحتمالية؟

    لكني لا أريد فتح جبهة جديدة. أنا منشغل جدًا بالنسبية الآن. أود فقط أن أطرح عليك بعض الأسئلة، باعتبارك شخصًا يفهم الإنتروبيا والكميات.

    1. في رأيك هل يمكن بأي شكل من الأشكال التأثير على الماضي من الحاضر أو ​​المستقبل؟

    2. إذا كانت الإجابة على 1 هي لا، فهل توافق على أن هذا هو بالضبط (التأثير على الماضي) ما تتوقعه ميكانيكا الكم؟ (حسب تجربة ويلر).

    3. إذا كان الجواب على 2 هو نعم، ألا يتناقض هذا مع كل ما قلناه من قبل حول عدم رجعة الإنتروبيا في الوقت المناسب؟

    4. هل تعرف أي تفسير لتجربة ويلر لا يتطلب العودة بالزمن إلى الوراء؟

    5. لا أعلم إذا كنت قد تابعت النقاش بيني وبين ر.ح.، لكن إذا كانت الإجابة على 1 لا، والإجابة على 2 نعم، والإجابة على 3 نعم، فهل ترى مخرجاً من هذا؟ الأسوأ من ذلك كله (التأثير على الماضي) من خلال ما تم طرحه في المناقشة؟

    شكرا، وآسف على كثرة الأسئلة.

    مئير.

    لقد تجاهلت CMBR عمدًا في الوقت الحالي، لكنها ليست حاسمة بالنسبة للسؤال الرئيسي: هل يتفق التوأمان اللذان انفصلا عندما كانت درجة حرارة الكون 6000 كلفن على درجة الحرارة اليوم، 3 كلفن، على الرغم من مرور سنة بيولوجية واحدة فقط على أحدهما؟ هم؟

    ما لتشغيل؟ التدفئة أو تكييف الهواء؟

  331. يا إسرائيل لن تختطف. ليس هذه المرة.
    لا أتذكر بالضبط متى خطرت ببالي فكرة أن المساحة الفارغة ليست فارغة، لكني أتذكر بالفعل في صف اليود (1969) أنني قررت أنها مقيدة بالواقع.
    لقد قمت بصياغة نظرية الجسيمات الأولية بشكل شبه نهائي، عندما درست علم الأحياء في السنة الأولى في الجامعة العبرية على جبل المشارف في العام 1977-78. لقد اكتشفت العلاقة بين الخط المقارب للقطع الزائد والبصريات الهندسية عندما كنت منغمسًا بعمق في النموذج (هذا الاكتشاف بالذات، بالمناسبة، جاءني في المساء عندما فاز يفغينز هابر كوبوهافن بجائزة يوروفيجن. لذلك لم أفعل ذلك لدي جهاز تلفزيون في المنزل من حيث المبدأ. سمعت الأغنية مرارا وتكرارا من الجيران. وأتساءل عما إذا كنت سأكتشف ذلك لو كان لدي جهاز تلفزيون..).
    وبعد السنة الثانية تركت دراستي لصالح الأسرة والأطفال وضريبة الدخل. من وقت لآخر كنت ألقي نظرة على النموذج لإجراء تلميع بسيط. عدت إلى الجامعة في عام 1983 ودرست الفلسفة والرياضيات وعلوم الكمبيوتر لمدة ثلاث سنوات أخرى. فكرة أن "هناك" تبدأ من "لا شيء" جاءتني في حلم عام 1983، بعد مناقشة فلاسفة ما قبل سقراط، لكنني لم أربطها بعد ذلك بالنموذج. ثم تركتها لفترة طويلة جدًا ولم أعد إليها إلا منذ عام أو نحو ذلك عندما أمضيت سنتي الأولى في الفيزياء في جلاسكو وأصبحت على دراية كبيرة بالمادة المظلمة. قررت العودة إليها لأن سلوك المادة المظلمة يكاد يكون مطابقًا لسلوك الجسيمات الموجودة في نموذجي. على مر السنين، قمت بتغيير معالجات النصوص (المحرر البدائي على الحاسوب المركزي في الجامعة، آينشتاين، Kiottext DOS، Kiottext Windows، Word 2، Word 6...) وتم تدمير القرص الصلب الخاص بي مرتين دون أن أتمكن من عمل نسخة احتياطية. بدأت أتحدث عنه هنا بدافع الفضول، وفي هذه الأثناء كان لدي تفكير مصاب بجنون العظمة بأنه ربما لديه القدرة على الحصول على ميدالية ذهبية من يدي ملك السويد. أقوم الآن بإعادة البناء والتحرير، وفي هذه الأثناء أشارككم هنا المعرفة.

  332. اليوبيل.
    لا أريد أن أزعج. وأعتقد أيضًا أنني تعلمت خلفية المواضيع التي أناقشها بطريقة صارمة، وفي المؤسسات الأكاديمية المناسبة. ببساطة، هناك شيء لا يسير على ما يرام، وببساطة، أعتقد أنه شيء بسيط أيضًا.

    إن الأسئلة التي طرحتها وأطرحها هنا، طرحتها أيضا أمام أساتذة وعلماء ذوي كفاءة عالية. لم أحصل على إجابة مرضية قط. في الواقع، توصلت إلى نتيجة (ذاتية بالطبع)، مفادها أنهم لا يفهمون القضية حقًا.

    مشكلتي مع كلماتك مختلفة قليلا: لا يبدو لي، وصحح لي إذا كنت مخطئا، أنك تعلمت الأشياء التي تتحدث عنها بطريقة صارمة (مؤسسة أكاديمية معترف بها، الواجبات المنزلية، الامتحانات، مختبرات).

    السبب وراء قولي هذا هو أن العمليات الحسابية البسيطة ستظهر لك أن النموذج كان موجودًا قبل أن تتمكن حتى من دراسة الفيزياء في الجامعة. ببساطة، كنت صغيرا جدا.

    هذا لا يعني أنه مخطئ - ولكن في رأيي، سيتعين عليك إحضار صيغ جميلة، أو تجربة مقنعة، لشرح أذن مستمعيك.

    إذا كنت قد درست الجبر الخطي، فمن المحتمل أنك تعلم أنه من الممكن بناء "نماذج" أو "حقول متجهة" مغلقة تمامًا رياضيًا، وبدون تناقضات، لكن هذا لا يجعلها واقعية.

    أبسط مثال هو عالم بأكثر من 3 أبعاد مكانية. لا يختلف الأمر على الإطلاق من وجهة النظر الرياضية عن العالم الذي نعرفه، لكن عالمنا، على الأقل وفقًا لنيوتن، له 3 أبعاد.

    فلماذا إذن بناء نموذج؟ من أجل الأنا؟ بالنسبة للأم التي ستكون فخورة جدًا بطفلها الذكي حتى أنه لديه نموذجه الخاص على الإنترنت؟

    سألتني عدة مرات لماذا لا أكتب كتبًا. الجواب بسيط للغاية: لماذا ذلك؟ هل هناك أي كتب مفقودة في العالم تفوق بكثير أي شيء أستطيع إنتاجه؟

    هذا. أتمنى أن لا أبالغ في كلامي.

  333. واحدة واثنان وثلاثة أرباع، تجربة
    لقد قمت للتو بتأسيس موقع على شبكة الإنترنت:
    https://sites.google.com/site/darkmattermodel/
    هناك مقال يمكنك تنزيله: نمط الموجات العائدة.pdf
    هذه المقالة "مستقلة" لذا فهي صحيحة رياضيًا بغض النظر عن بقية النموذج
    يرجى القراءة بعناية وتذكر أن هذا قد كتب منذ سنوات عديدة
    R.H. وإسرائيل، لقد تلقيتها بالفعل عبر البريد الإلكتروني

  334. إسرائيل،

    رأيت للتو فقط:
    "كيف تفسر درجة حرارة الكون؟ ونظام CMBR؟ والعلاقة بينهما؟"

    ليس لدي أي تعليق بخصوص العلاقة المتأصلة بين الاثنين. لدي أشياء مهمة لأقولها عن "الاستكمال" (علامات الاقتباس تعبر عن رأيي فيه)، والذي يتم من خلاله استنتاج درجات الحرارة في الجسيمات الفرعية للثانية الأولى من الانفجار (حرفيًا). سأقول لهم في مقال أنيق.

    فيما يتعلق بساعات درجة الحرارة التي تقترحها، أود أن أعلق (بغض النظر عن نظرية MCS) على أنه يجب أن تنشأ تباينات الخواص على ما يبدو في قياساتها على طول محور الحركة، أي اختلاف بسيط في درجة الحرارة الناتج عن دوبلر التفاضلي في الأطوال الموجية للموجة. يتم قياس CBR مع وضد اتجاه الحركة (وبالطبع أيضًا نصف الاختلافات مقارنة بالقياسات المتعامدة مع اتجاه الحركة). يجب أن يشير هذا الاختلاف إلى السرعة المطلقة بالنسبة لنظام الراحة في CBR. يجب أن يثير هذا تساؤلات حول نسبية الزمن، لأنه يجب أن يقرر بين مسافرين بالقصور الذاتي يسافران وبأي سرعة.

  335. إسرائيل،

    "إذا بدا لك أن يوفال، أو يهودا، أو أنا، أو أيًا كان، هم شخصيات فينلايسون، فلماذا لا تتصفح؟"
    - أفعل ذلك بالفعل. مع العلم أن الوحيد الذي دخلت معه في نقاش علمي هو أنت، وكان ذلك فقط في المواضيع التي رأيت أنها تستحق النقاش معك. أحد الأشياء المهمة التي تتعلمها في التخنيون هو كيفية استخدام الوقت، يمكنك التأكد من أنني لن أضيعه في المناقشات مع مثيري الشغب أثناء الخدمة.

  336. إسرائيل شابيرا، شكرًا لك على الحماية
    عندما يدينونني كمثير للمشاكل، فهذا ليس خبيثًا، لأنني كذلك بالفعل، وليس فقط من وجهة نظرهم. لاحظ لي R.H، وهو محق تمامًا، أنني كنت أتصرف كمثير للمشاكل (ليس بهذه الكلمات بالطبع 🙂) وقدم لي صياغة أكثر نجاحًا اعتنقتها بكلتا يدي. ليست لدي مشكلة جدية في أن يطلق علي لقب مثير للمشاكل. أعلم أنني هكذا، وهذا ليس من اليوم. تعليقات Studentechnion وPoint وR.H. Refai.M (أين هو؟ يرجى التأكيد له أننا نحبه وندعوه للعودة)، بعيدًا عن اللغة الشائكة إلى حد ما، مفيدة للغاية وأنا أرحب بها. في النهاية، سأحصل على نموذج جميل ومصمم بشكل صحيح من يدي - ويرجع الفضل في ذلك كثيرًا إليهم.
    بالمناسبة، جملةك "أشتاق إلى اللحظة التي أكتشف فيها أنني ارتكبت خطأً، وأفهم أين" عاشت معي لسنوات عديدة.
    في ب

  337. طالب.

    مشكلة مثيري الشغب في الفيزياء مشكلة قديمة ومألوفة، خاصة في مسألة الجاذبية. وقد ذكر ذلك فاينمان في كتابه "ما يهمك ما يعتقده الآخرون". جوجل أيضا CRACKPOTS الفيزياء.
    لكن معاصرينا يتمتعون بميزة لم تكن موجودة من قبل. لذلك كان عليك الجلوس في الاجتماعات والاستماع إلى كل غفوة. اليوم في عصر الإنترنت يمكنك فقط التمرير. سوف يسامحني بعض المعلقين، لكن هذا ما أفعله بتعليقاتهم.

    إذا بدا لك أن يوفال أو يهودا أو أنا أو أي شخص آخر هم أعضاء في شخصية فينلايسون، فلماذا لا تتصفح؟ إذا واصلت الجدال مع دوف هينيس، فإن السؤال الأول الذي سأطرحه على نفسي هو ما هو دافعي للقيام بذلك. مشاعر التفوق؟ متعة سادية؟
    وسامحني ديب. وربما كان على حق تماما فيما قاله. ببساطة، ليس لدي أي اهتمام.

    لا أستطيع إلا أن أتحدث عن نفسي: أشتاق إلى اللحظة التي أكتشف فيها أنني مخطئ، وأفهم أين. وهذا ما أفعله في المناقشة الحالية مع ر.

    ومع ذلك، سيكون من الصعب جدًا بالنسبة لي أن أتجادل مع شخص تكون نقطة بدايته هي أنه بالتأكيد على حق وأنا بالتأكيد على خطأ. حتى لو كنت أعلم أن قياساته النفسية أعلى مني، وخلفيته الأكاديمية في المجالات التي أناقشها أغنى من خلفيتي.

    ر.ح.

    في الرابط الذي أرسلته، عندما تصل جيل إلى الساعة C2، تظهر 10 ثوانٍ وتظهر ساعتها 8.

    دعونا نمدها قليلاً، لجعلها ملائمة للعمل: 10 مليار سنة و8 مليار.

    السؤال 1: إذا كانت هناك أيضًا بجانب الساعة C2 ساعة درجة حرارة، P2، وأظهرت C2 10 مليارات، فماذا ستظهر P2؟
    السؤال 2: إذا كانت بجانب ساعة جيل "C" ساعة درجة الحرارة "P"، وكانت "C" تظهر 8 مليار، فماذا سيظهر "P"؟

    لا تنس الأنظمة التي لا تتسارع وتلك التي تكون في حالة سكون.

    و الاهم من ذلك:

    السؤال رقم 3:

    قيل لنا أن لدينا كاميرا عالية الدقة. خلال الاجتماع، تستطيع الكاميرا تصوير الساعات الأربع معًا. ما الأوقات التي ستظهر على كل ساعة في الصورة المشتركة الواضحة؟

  338. اليوبيل,

    وأفضّل أن يبقى في إطار المناقشة هنا. يمكن تحميل الملفات على الإنترنت من خلال جميع أنواع المواقع. إذا كنت تريد، فاكتب "تحميل ملف" في Google وأعتقد أن الأمر سينجح هناك.

  339. إسرائيل،
    انا لا افهم ما تريد. وعلى كل حال فإن الرابط الذي أرسلته يبين تماما أنه لا يوجد أي تناقض في الأمور.
    وأنا أيضًا لا أتفق مع هذا التعريف "الوقت الحقيقي"، ما هو الوقت الحقيقي؟ هل تقصد أن التوأم 2 الذي يفترض أنه يستريح هو الوقت الحقيقي؟ لذا أطلق عليه "وقت الراحة" مقابل وقت الحركة. لقد كان لديهم جميعًا وقت حقيقي، إلا أن إيقاعه يختلف من توأم إلى توأم بسبب حركتهم. اسأل أينشتاين.

    وأعتقد أن الأمر المحير هنا هو أننا، على عكس الأبعاد الأخرى، لا نعرف وقت "الراحة" بل فقط الوقت الذي يتحرك بمعدل ساعة واحدة في الساعة. إنه مشابه للأشخاص الذين يتحركون طوال الوقت في القطار، بعضهم يجلس والبعض الآخر يركض. هل يمكنك تحديد مسافة "حقيقية" لأولئك الذين يجلسون والذين يركضون في القطار ليست حقيقية؟

  340. اليوبيل,

    لقد تم اختراع "الرسومات" التي تحتاجها منذ زمن طويل - وهي تسمى الرياضيات. ليس من قبيل الصدفة أن يدرسها طلاب العلوم في الجامعة.

    ونصيحة: يمكنك تحضير ما تريد (صور، معادلات، رسوم توضيحية ورسوم بيانية) في برامج مثل word وexcel أو حزمة openoffice، وتحويل الملفات إلى PDF وتحميلها على الإنترنت. لا تقصر نفسك على الكلمات فقط بسبب نظام التعليق هنا.

  341. ر.ح.
    في الواقع، لا تنس أنه قبل انفصال التوأم كانا معًا في نفس النظام غير المتسارع، وحتى بعد عودتهما معًا كانا مرة أخرى في نظام غير معجل. إذا كان هناك 10 توائم بدلاً من 2 تم فصلهم والتقيهم مرة أخرى، فيمكن ترتيب ذلك بحيث تمر سنة واحدة بالضبط لجميع التوائم المسافرين، على الرغم من اختلاف الطرق التي سلكوها. بالطبع، إذا كان هناك حاسوب عملاق في السفينة التوأم المسافرة، فإن التكامل الخطي لجمع السرعات يجب أن يتضمن حسابات نسبية.

    على أية حال، يمكن تخطي المشكلة برمتها التي أثرتها إذا تحولنا إلى الأنظمة غير المتسارعة. لهذا، إذا أمكن، قم بإلقاء نظرة على الرابط التالي:

    http://galileoandeinstein.physics.virginia.edu/lectures/time_dil.html
    لاحظ أنه في هذا المثال، لم يتم تسريع أي نظام، وبالتالي فهو في حالة سكون.

    (عذرًا على التشويش، إذا كان الأمر معقدًا، يمكنك أيضًا المتابعة بدون الرابط، على الرغم من أهميته). لكن يجب أن أتأكد من اتفاقنا على النقطة الرئيسية: في وقت الاجتماع، حتى مع وجود 10 توائم معًا في نفس سفينة الفضاء، سيتفقون جميعًا على أن الوقت "الحقيقي" هو وقت التوأم رقم 2، والدليل هو التدفئة الجارية في سفينة الفضاء.

  342. آر إتش،
    شكرا على النصيحة الحكيمة. والحقيقة أنني اتبعت ذلك في بعض ردودي، لكن ربما لم أؤكد عليه بما فيه الكفاية. وكما لاحظت نقطة، وأكدت، فأنا أقوم ببناء عالم "كما هو الحال في محرك رسومي لألعاب الكمبيوتر". ومع ذلك، على عكس ألعاب الكمبيوتر، فإن لبنة البناء محددة جيدًا مسبقًا ولا أقوم بإضافة تعريفات مخصصة لاحقًا.

    بوينت، الطالب، التخنيون وشركائهم في الرأي،
    إن الافتقار إلى الخيارات الرسومية في نظام تعليقات العلماء يجعل من الصعب علي توضيح كلماتي، ويجبرني على الشرح بالكثير من "التلويح باليد". أكتب تعليقاتكم، رغم أنها "قاتلة"، وأتعلم منها على أمل أن أتمكن من صياغتها بشكل أفضل في المستقبل.

  343. نقطة،
    أنا أتفق مع المراجعة. لا يمكنك أن ترفض ما يكتبه يوفال باعتباره "هراء".
    يقول يوفال "دعونا نفترض بديهيًا أن هناك A، B، C." ومن هنا سننطلق ونرى ما هي الاستنتاجات من هذه البديهيات. فإذا شرحوا دون تناقض الأشياء المثيرة للجدل في الفيزياء الحالية، فما هو الجيد وما هو الممتع.
    يمكنك أن تستنتج الجمل والاستنتاجات الناشئة عن مقدمات البداية، لكن لا يمكنك أن تستنتج البديهيات.

    لمعلوماتك، تم بناء السلاسل بنفس الطريقة تمامًا. "لنفترض أن هناك خيطًا واحدًا يشكل كل المادة الموجودة في الكون والتي تهتز عند أطوال موجية مختلفة." هذا هو المكان الذي يبدأ كل شيء.

    اليوبيل,
    المراجعة لك أيضًا وقد كتبتها لك من قبل. يجب أن تبدأ بجملة "لنفترض ذلك... ونرى ما يخرج منه" وإلى الألف بجملة "هناك جزيئات دقيقة ومساحة فارغة ولا شيء، الخ" وكأن هذه حقيقة موجودة.

  344. اليوبيل,

    لسوء الحظ، مع تقدم المناقشة، أتفق مع رأي بونكوت، ولدي انطباع بأنك مجرد مثير للمشاكل الجنسية.

  345. إسرائيل،

    أتفق مع كل ما قلته. ولكن هناك شيء مفقود هنا.
    ما هو مفقود في وصف الآلة الحاسبة التي قدمتها هو البيان الذي يفيد بأن الآلة الحاسبة المذكورة أعلاه صالحة فقط للراحة.
    في الواقع، كان لا بد من إدخال عامل السرعة النسبية. سيكون الحساب مختلفًا لكل سرعة (ولا تسألني عن الصيغة) وبعد ذلك ينجح كل شيء ولا يوجد أي تناقض.
    أي أنه في مثالنا، فإن انخفاض درجة الحرارة خلال 13.6 مليار سنة في حالة الراحة يساوي نفس الانخفاض لمدة ساعة واحدة عند السرعة X.

  346. إسرائيل ويهوذا! أحبه من أجل الثناء 🙂
    للحظة فكرت في إغلاق البسطة والعودة إلى المنزل، لكن بسببك يجب أن أستمر في طحن دماغي هنا 😀

    نقطة،
    تعليقاتك "القاتلة" مفيدة جدًا. سأحاول إحضار المزيد من "اللحوم" هنا. شكرا.

  347. إلى يوفال تشيكين
    أتفق معك من المفترض أن تشرح النظرية فقط ما هو معروف بأبسط طريقة وتكون مثالية إذا فعلت ذلك!. النبوة أعطيت فقط للأنبياء وليس من المفترض أن يكون العلماء أنبياء. لقد كان نيوتن متقناً في نظرياته، ولم يكن من المفترض أن يشرح في نظرياته ما كان معروفاً سابقاً عن النظرية النسبية.
    أقترح عليك قراءة مقالتي هنا في مجلة العلوم بعنوان "تطور النظريات" والتي أبين فيها تشابهًا كبيرًا بين تطور الحياة وتطور النظريات.
    آسف لأني مشغول جدا بشؤوني ومشاكلي ويصعب علي أن أجد الوقت للتعليق.
    يوم جيد
    سابدارمش يهودا

  348. لست متأكدًا من أنك أمسكت برأس يوفال.
    في رأيي، يوفال لا يحاول تفسير الفيزياء الموجودة بالطريقة التي اعتدنا عليها أو نفهمها، لكنه يحاول خلق فيزياء بديلة. لذلك، بالنسبة له على الأقل، يجب عليه أولاً تحديد المفاهيم الأساسية حتى يتمكن من الاستمرار في ربط فيزياءه بالفيزياء التقليدية.
    بالنسبة لمراقب خارجي، قد يبدو هذا بمثابة تفريغ للدماغ، وربما يكون هذا هو الحال حقًا. ولكن يمكن قول الشيء نفسه عن نظرية الأثير لماكسويل، باستثناء أن ماكسويل هو الذي جلب المعادلات.
    فكر في القبعة الكبيرة التي ستضطر إلى ابتلاعها، إذا تمكن يوفال من استخلاص معادلة شرودنغر الأساسية من نموذجه. ألن يبدأ الجميع بالحديث فقط عن الجسيمات الدقيقة والمساحات الفارغة؟

    لذلك لا تمنع الثور باستخدام داشو. ليس قبل مرحلة الصيغ التي وعد بها يوفال.
    (وصحيح الكلام سهل عليا عندي إعفاء).

  349. وظيفتي هي أن أهاجم أولئك الذين يصنعون المسرحيات. وأي شخص يكتب مليون تعليق دون أن يقول أي شيء تقريبًا، فهو يقوم بمسرحيات.

  350. نقطة، لماذا تهاجم يوفال؟ هل فكرت يومًا كيف سيكون رد فعلك إذا طرح شخص ما هنا لأول مرة الفكرة الوهمية للأكوان الموازية؟ أو أبعاد متعرجة؟ أو الثقوب الدودية؟ هل سبق لك أن نظرت إلى نظرية ماكسويل، مع كل الأنفاق والحذافات والتيارات والموجات العمودية؟
    هل يمكنك أن تبين لنا ولو ردًا واحدًا من ردك لا يرقى إلى مستوى: "كل هذا هراء!" أو "اذهب للدراسة!" هل سبق لك أن حاولت حل المسائل الفيزيائية التي لم يتم حلها بنفسك، أم أن وظيفتك الوحيدة هي أن تقع على عاتق أولئك الذين يحاولون؟

    بعد كل شيء، لقد وعدتنا بروابط لحل مشكلة عدم المحلية في التشابك الكمي. أين هم؟ العالم يتساءل ويمل.

  351. نقطة،
    نظرًا لأنني تلقيت عددًا قليلًا جدًا من التعليقات هنا، فقد اعتبرت المراسلات معك طريقة رائعة للخروج. أشكركم على ردودكم وأعتذر عن تضييع وقتكم الثمين.

  352. يوفال، أنت تدعي في الواقع أن الاختبار يعتمد على سذاجة الفاحص. كلما كان الممتحن أكثر سذاجة، ويتجاهل الأشياء غير الضرورية، كلما كان نموذجه أبسط وبالتالي أكثر صحة.

    هذا هو تقريبًا النموذج الذي تقدمه. لقد كتبت بالفعل عشرات الآلاف من الكلمات هنا، وبصرف النظر عن الجسيم الحاد والمساحة الفارغة، ليس لديك أي فكرة عما يعنيه هذا الخليط من الكلمات. أنت لم تقل أي شيء هنا.
    وأتساءل عن عدد الكلمات التي تضيعها هنا على الموقع فقط لتقول إن كل شيء بالنسبة لك يأتي ببطء.

    لاحظت أن ما يأتيك ببطء هو لا شيء. فقط الجزيئات الدقيقة والمساحات الفارغة التي ليس لها معنى.

    باختصار، لقد غيرت رأيي بشأنك، والآن بعد مراوغتك وتجاهلك للأسئلة البسيطة التي تطرح عليك هنا، فإنني أتقيأ عليك باعتبارك مزبلة دماغ عادية.

  353. المحاولة الأخيرة لاجتياز حاجز التأخير (12 ساعة):
    نقطة، العدالة معك،
    هذا هو بالضبط ما أفعله: بناء عالم يشبه محرك الرسومات لألعاب الكمبيوتر. ولكن هذا ليس نقطة. المغزى هو أنني أستخدم لبنة بناء بسيطة جدًا، وأحاول أن أبين أنها كافية لبناء العالم كله بحيث يكون مطابقًا للعالم الذي نعرفه من الفيزياء.
    يتكون تعريف هذا العنصر الأساسي من أربعة أقسام (في الواقع، وصلت إليه من تعريف أبسط ولكني لا أذكره هنا بسبب الصعوبات التقنية: فهو يحتاج إلى رسم توضيحي):
    أ) يتكون العالم من عدد متزايد من الجسيمات وكمية لا حصر لها من الفضاء الفارغ.
    ب) مع وجود مساحة فارغة كافية، تمتص الجسيمات نفسها في محيطها لتكوين جسيمات ذات حجم خاص بها. جسيم محاط بجسيمات كثيفة للغاية يغير حالته إلى قطعة من الفضاء الفارغ.
    ج) يشغل جسيم واحد حجما محددا في الفضاء، ولا يشغل أي جزء منه في نفس الوقت جسيم آخر.
    د) للجسيم حركة ذاتية مستقلة تختلف عشوائيًا في اتجاهه وسرعته، باستثناء قيد واحد ناشئ عن القسم السابق: لا تتم حركة الجسيم داخل الحجم الذي يشغله جسيم آخر.
    إن تعدد سلوكيات مجموعة الجزيئات مستمد من هذا التعريف وحده، دون إضافات.
    ولهذا السبب، تم إجراء فحص لنموذج التنبؤات الجديدة للمستقبل (ميشبلهيفكت دريميمشكنا)، تشامبفيستوتووبوبي كولي ذاسبايره فيها. النموذج بالنسبة لي بسيط، تماما مثل ما سبق، وما هو معنى النموذج للفيزياء بأكملها. A m h a y t c l i h b m y m o t h u , h r y h a v a b a r a t h m b h a n b h c h a l h g l l l a t h a zi و T.

    الرد في انتظار الموافقة.

  354. الطالب والتخنيون وكل من يتابع،
    بعد عدة زيارات لمدونة مئير، كنت مقتنعا بأن نموذجه هو استمرار فوري لنموذجي. أحضر وصفًا بلاستيكيًا، ويضع مئير الصيغ فيه. أنوي اعتماد صيغ مائير كأساس للحسابات الكمية في نموذجي. حتى أنتهي من العمل، أنتم مدعوون لتصفح مدونته. من فضلك شاهد "فيزياء MCS".

  355. آسف على التقليد :)

    الرسالة السابقة هي بالطبع مني إلى إسرائيل (إسرائيل لا تتحدث مع نفسها عادة)!

  356. إسرائيل،

    المساحات عبارة عن مساحات كثيفة يشكل تسلسلها اللزج "قماش" الفضاء. تجاه الجسيم في لحظة تأثيره، يكون القماش ثابتًا. "تزحف" النبضات الكهرومغناطيسية داخل هذا النسيج في خطوات محسوبة، طول كل منها مسافة واحدة في اتجاه عشوائي، اعتمادًا على الموضع النسبي الذي تنشأ فيه النبضات الكهرومغناطيسية بين أقسام القماش في لحظة التشغيل. بين العملية، تكون النبضات الكهرومغناطيسية والنسيج شفافين لبعضهما البعض. الفاصل الزمني بين الفعل والفعل (زمن دورة الجسيم) ثابت، وكبير جدًا (>>) من زمن التفاعل، وهو في الواقع "قفزة نوعية" إلى مسافة مسافة واحدة، عند "~" "سرعة لا نهائية"، والتي يبلغ متوسطها سرعة الضوء في جذر اثنين عند قسمة طول القفزة على وقت الدورة.

    من أجل فهم الطاقة "المحتملة" كحركة، يمكننا أن نشير إلى طاقة الجاذبية المحتملة (MCS هي نظرية الجاذبية، وشد الزنبرك يتضمن طاقة "وضعية" كهربائية):

    طاقة الجاذبية الكامنة هي عدم تناسق في مجال الجاذبية الذي يتعرض له الجسيم الأولي، والذي يتم التعبير عنه بحركة الجسيم (دائمًا، حتى عندما نشير إليه على أنه "جسيم ساكن"). من الممكن الجدال والادعاء بأن "عدم التماثل" المحلي للمجال تجاه الجسيم هو "طاقة محتملة". ضد هذا يمكن للمرء أن يجادل ويدعي أنه لكي يكون لهذا عدم التماثل المحلي معنى، يجب أن يكون هناك جسيم له إمكانات فعل في المجال، وبالتالي فإن الجسيم يعبر أيضًا عن "طاقة الوضع" وهي قدرته على القيام "بشيء ما". الى الحقل.

    لذلك، أعتقد أنه من الصحيح أن الطاقة هي التفاعل نفسه، أي "تحقيق" الإمكانات (على جانبي الحاجز) والتي تكون حركية دائمًا، وبالتالي تبقى مع نوع واحد من الطاقة: التفاعل بين الجسيم والفضاء، وهو ما ينعكس في حركة الجسيم نحو الفضاء بسرعة الضوء.

  357. وفي تلك المناسبة سأشير إلى أن التفسير (وبشكل عام، النموذج) يجب أن يكون صارمًا ورياضيًا أيضًا.

  358. نقطة باختصار حتى يتم تأكيد الرد المتأخر:
    رأيي أن اختبار النموذج ليس في التنبؤ بظواهر لم يتم ملاحظتها من قبل، بل في بساطته وقدرته على تفسير الكثير

  359. طالب، التخنيون
    أنا أفهم قلبك. لو كنت مكانك لكنت انفجرت بفارغ الصبر.
    ولكن إذا كتبت الآن شيئًا مثل: "مسار حركة الضوء زائدي، لكننا نشير إلى الخط المقارب وبالتالي يبدو أنه يتبع خطًا مستقيمًا؛ "وهذا ما يفسر لماذا زاوية التأثير تساوي زاوية العودة" قد ترفع حاجبك في عدم فهم أو ازدراء تام.
    تمكنت بالأمس من استعادة أجزاء من القرص الصلب الذي تم تدميره منذ 15 عامًا، ووجدت فيه بعض الأشياء التي كنت قد نسيتها. أنا آسف على البطء. هذا كل ما أستطيع أن أجيبك عليه الآن

  360. اليوبيل,

    ما زلت لا أفهم شيئا. إذا كان لديك تفسيرات لما كتبته، لماذا لا تنشرها؟

  361. والدي نظام العلوم:
    وتأخير الردود، مهما كان مبررا، يفسد ديناميكية الحوار. ألا يوجد مجال لإتقان الطريقة؟
    ونرجو بهذه المناسبة نشر التعليق رقم 331216 للنشر
    شكر

  362. طالب، التخنيون
    بالنسبة لي، التفسيرات تأتي ببطء. آسف، ولكن هذا ما هو عليه. لم أجد تفسيرا لظاهرة واحدة، ولكن أعتقد أن ذلك فقط لأنني لم أكن أبحث. هذه هي مسألة عدم المحلية في التشابك الكمي التي أثارها زميلنا هنا. وفي المستقبل القريب سوف أتعمق فيه، وإذا تمكنت من شرحه دون إضافة بنود إلى التعريف الأصلي للجسيم، فسوف أقدم الشرح دون تأخير.

  363. نقطة، العدالة معك،
    هذا هو بالضبط ما أفعله: بناء عالم يشبه محرك الرسومات لألعاب الكمبيوتر. ولكن هذا ليس نقطة. المغزى هو أنني أستخدم لبنة بناء بسيطة جدًا، وأحاول أن أبين أنها كافية لبناء العالم كله بحيث يكون مطابقًا للعالم الذي نعرفه من الفيزياء.
    يتكون تعريف هذا العنصر الأساسي من أربعة أقسام (في الواقع، وصلت إليه من تعريف أبسط ولكني لا أذكره هنا بسبب الصعوبات التقنية: فهو يحتاج إلى رسم توضيحي):
    أ) يتكون العالم من عدد متزايد من الجسيمات وكمية لا حصر لها من الفضاء الفارغ.
    ب) مع وجود مساحة فارغة كافية، تمتص الجسيمات نفسها في محيطها لتكوين جسيمات ذات حجم خاص بها. جسيم محاط بجسيمات كثيفة للغاية يغير حالته إلى قطعة من الفضاء الفارغ.
    ج) يشغل جسيم واحد حجما محددا في الفضاء، ولا يشغل أي جزء منه في نفس الوقت جسيم آخر.
    د) للجسيم حركة ذاتية مستقلة تختلف عشوائيًا في اتجاهه وسرعته، باستثناء قيد واحد ناشئ عن القسم السابق: لا تتم حركة الجسيم داخل الحجم الذي يشغله جسيم آخر.
    إن تعدد سلوكيات مجموعة الجزيئات مستمد من هذا التعريف وحده، دون إضافات. في رأيي، اختبار النموذج ليس في التنبؤ بالظواهر التي لم يتم ملاحظتها من قبل (حتى لو كان لها تأثير درامي مقنع)، ولكن في بساطته وقدرته على تفسير الكثير. يتم تعريف نموذجي ببساطة، كما هو مذكور أعلاه، ويدعي أنه يبني نموذجًا لـ כל الفيزياء المعروفة. فإذا نجح في مهمته، فقد اجتاز الاختبار بنجاح حتى بدون توقعات.

  364. مئير.
    من أين تأتي اللغة الإنجليزية الممتازة؟ يمكن للأب إبن أن يأخذ مثالاً.

    أسئلة حول الفصل الرابع

    كما أفهمها، أنت تدعي أن تكافؤ الكتلة والطاقة ينتج عن تفاعل جسيمات الفضاء (التي تسميها Spacent) مع الجسيمات الأولية التي تسميها EMP.

    وبما أن النبضات الكهرومغناطيسية تتحرك بسرعة جذر 2 مرات C، فإننا نحصل على التكافؤ كطاقة حركية.

    1. هل فهمت بشكل صحيح؟
    2. ألا ينبغي أن يكون العكس؟ الفضاء هو الذي يتحرك مثل جزيئات الهواء وتتحرك النبضات الكهرومغناطيسية من خلالها؟
    3. في نظرك، كل أشكال الطاقة تنشأ من الحركة (يبدو معقولاً جداً). ماذا عن الطاقة المحتملة؟ ربيع مضغوط مثلا؟
    حدسيًا، يبدو لي أنه وفقًا لنموذجك، من الواضح تمامًا السبب هنا أيضًا، لكنني سأكون مهتمًا بسماع تفسيرك.

  365. يوفال، لا أستطيع أن أصدق أنك كتبت ما كتبته على محمل الجد.
    يمكنك أيضًا القول بأن مجرد حقيقة وجود العالم هو دليل على نموذجك. انها مجرد سخيفة.

    اقترح تجربة تتنبأ بنتائج مختلفة عما تتنبأ به النظريات المختلفة.

    الأمر هو أنك غير قادر على القيام بذلك، لأن نموذجك لا يتنبأ بأي شيء، فهو نموذج مرن يسمح بالتكيف مع أي كون يمكن تخيله.
    هذا ما يتم فعله باستخدام المحركات الرسومية لألعاب الكمبيوتر، حيث يمكنك بناء جميع أنواع "العوالم" بها.

  366. اليوبيل,

    أوه ونعم، أنا (وأعتقد أن العديد من الأشخاص الآخرين الذين يناقشون معك) ما زلت أنتظر توضيحات حول المواضيع التي ذكرتها منذ وقت ليس ببعيد. إذا لم يكن لديك أي شيء، من فضلك قل ذلك.

  367. أيها الطالب، التخنيون، العدالة معك. اللدغة خرجت من الكيس 😛 فلنكمل:
    على الرغم من عدم وجود مبدأ حفظ الزخم على مستوى الجسيمات، فقد رأينا أن هذه الظاهرة موجودة على مستوى البروتون. إن حركة البروتون هي تعبير عن نسب الكثافة بين أحد جانبيه والجانب المقابل، وتحافظ نسبة الكثافة هذه مع مرور الوقت. حركة البروتون هي في الواقع حركة موجة. بروتونان يصطدمان ببعضهما أثناء حركتهما، ويتبادلان نسب الكثافة، وهذا بالضبط ما يبدو عليه التصادم المرن في الفيزياء. أثناء التصادمات المرنة، تتبادل البروتونات الزخم. لكن البروتونات لا تحصل على الزخم من بعضها البعض فحسب، بل تولد أيضًا الزخم من نفسها. ما الذي يسبب الزخم الذاتي للبروتونات؟
    دعونا نتفحص ما يحدث في "قلب" البروتون: كثافة الجزيئات الموجودة في مركز البروتون عالية. قد "يموت" الجسيم ذو الكثافة العالية ويتحول إلى قطعة من المساحة الفارغة - حفرة. في هذه الحفرة، يمكن لبعض الجسيمات أن تدخل وتترك وراءها ثقبًا يتم إدخاله أيضًا في مرحلة ما بواسطة جسيم تاركًا وراءه ثقبًا. وهكذا، بسبب كثافة البروتون العالية، يتشكل ثقب في منطقته الوسطى، فيومض للخارج. وبما أنه لا يوجد تصادم مرن بين الجسيمات، بدأت الكثافة في مركز البروتون في الزيادة. وفي لحظة معينة تشكلت هناك فجوة أخرى وتومض للخارج أيضًا. وقد تتكرر هذه العملية إلى ما لا نهاية. مثل هذا الثقب، الذي ينشأ في قلب البروتون ويرفرف للخارج، يعني أنه في لحظة معينة تكون الكثافة المحلية في منطقة واحدة من غلاف البروتون أصغر من الكثافة في المنطقة المقابلة. يؤدي اختلاف الكثافة إلى انتقال البروتون إلى المنطقة الأكثر كثافة. يرفرف ثقب جديد تم إنشاؤه في مركز البروتون للخارج دون تفضيل الاتجاه وقد يتسبب في تحرك البروتون في اتجاه جديد.

  368. ر.ح.
    حسنًا، اتفقنا على أنهما سيتفقان على درجة الحرارة.
    الخطوة التالية: اذهب إلى الموقع:
    http://hyperphysics.phy-astr.gsu.edu/hbase/astro/expand.html#c3
    هناك صيغة هناك - صيغة فريدمان للعلاقة بين درجة الحرارة والزمن الذي انقضى منذ الانفجار الأعظم.
    توجد أيضًا آلة حاسبة - يمكنك إدخال درجة الحرارة والحصول على الوقت بأي دقة تريدها، أو العكس.

    الاستنتاج أ:
    ومن خلال قياس درجة الحرارة وحدها، من الممكن معرفة الوقت الذي انقضى منذ الانفجار الكبير في أي مكان في الكون، بأي دقة نريدها وهذا ممكن تقنيًا.

    الخطوة التالية: تثبيت الساعات المؤقتة.
    يمكن القيام بذلك عن طريق توصيل جهاز كمبيوتر بمقياس الحرارة. (لا يهم الآن إذا كانت هذه هي درجة حرارة CMBR).

    الاستنتاج ب:

    يمكن تجهيز أي شخص بساعات مؤقتة.

    الخطوة التالية: ساعات السيزيوم.

    تُظهر هذه الساعات الوقت الفريد للنظام، وتتأثر بالتسارع.

    الاستنتاج ج:

    إذا مرت سنة بيولوجية واحدة فقط للتوأم 1 منذ انفصاله عن التوأم 2، فسوف تظهر ساعة السيزيوم الخاصة به أيضًا مرور عام واحد فقط.

    وإذا مر التوأم 2 بـ 13.6 مليار سنة، فإن ساعة السيزيوم الخاصة به ستظهر أيضًا أن 13.6 مليار سنة قد مرت.

    (أفترض أن هذا ما قصدته بقوله: "سيكون الجدال بينهما هو ما هو معدل تبريد الكون. الأول سيقول أن الكون يبرد من X أولية إلى 2.75 في الساعة والثاني سيقول - لا، المعدل هو X-2.75 مقسومًا على 13.6 مليار سنة.").

    الاستنتاج د:

    إذا زودناهما بساعات مؤقتة + ساعات سيزيوم، فسيظهر مقطع فيديو للساعتين معًا أن إيقاع الساعات في التوأم 2 هو نفسه أو تقريبًا نفسه، بينما في التوأم 1 إيقاع الساعة الحرارية أسرع بكثير من ساعة السيزيوم.

    خاتمة الرب:

    عندما تلتقيان، ستظهر الساعتان الحراريتان نفس الوقت (13.7 مليار سنة) بينما ستظهر ساعات السيزيوم وقتًا مختلفًا تمامًا (100,001 سنة و13.7 مليار سنة).

    من فضلك قم بمراجعة الاستنتاجات وأخبرني إذا كنت توافق على ذلك، وإذا لم يكن أين.

  369. إسرائيل،
    متفق عليه، سيشاهد كلاهما نفس درجة الحرارة في الاجتماع الأول - دعنا نقول X (بضعة آلاف من الدرجات) وفي الاجتماع الثاني اليوم دعنا نقول 2.75 ألف.
    وسيكون النقاش بينهم هو ما هو معدل تبريد الكون. الأول سيقول أن الكون يبرد من X الأولي إلى 2.75 في الساعة والثاني سيقول - لا، المعدل هو X-2.75 مقسومًا على 13.6 مليار سنة.
    ليس هناك خطأ وصواب. كلاهما على حق، ولهذا السبب هناك حروب في العالم.

  370. اليوبيل,

    أنت تتحدث عن "نموذجك" أكثر من الإعلان عنه فعليًا. إنه يخلق صورة شقي - فقط لعلمك.

  371. من الواضح، في النظام العلمي أنهم لا يعملون في أيام السبت. بخلاف ذلك، ليس من الواضح بالنسبة لي ما الذي يجعل ردي البريء ينتظر الموافقة لمدة يوم تقريبًا.

  372. ر.ح.
    نحن قريبون حقًا من جذر المشكلة. بعد إذنك، سأنتقل مباشرة إلى سؤالك 2 لمعرفة ما إذا كان بإمكاننا التوصل إلى اتفاق. سنرى بأنفسنا إلى أين يقودنا ذلك. سأحاول أيضًا استخدام الطريقة السقراطية في الأسئلة والأجوبة كوسيلة لاستكشاف الحقيقة.

    2) فيما يتعلق بـ "ساعة درجة الحرارة" ما زلت أعتقد أنك مخطئ ولا يوجد أي تناقض. توأم واحد يطير ويعود بعد سنة. سيكون مندهشًا جدًا عندما يرى أن شقيقه المتبقي يبلغ من العمر 10 سنوات. وسوف يتفاجأ أيضًا أنه وفقًا لساعة درجة حرارة شابيرا، فقد مرت 10 سنوات أيضًا. ولو نظر إلى ساعة مماثلة كانت معه في سفينة الفضاء لرأى أن الساعة تعمل باستمرار أسرع بعشر مرات من المتوقع. أين التناقض؟

    وفي المقال مع هاسوي قدمت مثالا متطرفا وهو أن الفارق الزمني بين التوأم هو 13.6 مليار سنة. يدعي التوأم 1 أنه لم يمر سوى عام واحد على الانفصال، بينما يدعي 2 أن 13.6 مليار قد مرت.

    وذكرت أيضًا أنه في رأيي 2 كانا على حق و 1 كان على خطأ. الدليل: الجو بارد في الخارج، بارد جدًا، وكلاهما متفقان على ذلك. إذا كان الأول على حق، كان ينبغي عليهم تشغيل مكيف الهواء. حقيقة أن التدفئة قيد التشغيل، تظهر أن الرقم 2 صحيح.

    اسئله:
    1. هل تتفق معي على أن التوأم خلال اللقاء سيتفقان على درجة الحرارة؟

    2. هل تتفق معي على أن كلاهما سيتفقان على أنه من حيث درجة الحرارة يكون الجو باردًا جدًا في الخارج (3K)، وليس بضعة آلاف من درجات الحرارة كما كان عند انفصالهما؟

  373. نقطة، كان ردي عليك "في انتظار التأكيد" لمدة 12 ساعة تقريبًا.
    باختصار اقتراحك مقبول بالنسبة لي، لكن أود أن أضيف أن التجربة التي تؤكد هي بالفعل مقنعة، ولكنها ليست الاختبار الوحيد. الأدوات الأخرى هي اختبار البساطة واختبار العمق. أدعي أن نموذجي أبسط وأكثر تفسيرية من النماذج الموجودة. كما أنني أعتذر عن جلب الأمور شيئًا فشيئًا.
    شكر

  374. إسرائيل،

    1) لقد أجبت على سؤالي حول الحد الأدنى لسرعة الضوء: ". نقطة جميلة. لكن لا تنسوا أمرين: الأول، أنني لا أعرف إذا كان الضوء يتحرك بجميع السرعات وفقًا للنموذج. وقد يتحرك بسرعات معينة، حسب سرعات المذبذب الأصلي. والثاني، المهم، هو مدى السرعات التي يحساس لها الكاشف، أو الراصد. تمامًا كما لو حاولت قياس المسافة إلى قوس قزح، فسوف ترى دائمًا أنه على مسافة معينة وثابتة منك أيها المشاهد، بغض النظر عن مكان وجودك."

    سامحني، لكن هذه ليست إجابة، بل موجة من الأيدي لا تعني الكثير. إن القول بأن الضوء يتحرك في نطاق محدود من السرعات يشبه القول إنه يتحرك بسرعة واحدة. وثانيًا، هل من الممكن أن يكون الكاشف حساسًا للضوء فقط بهذه السرعة النسبية الدقيقة؟ إذا كان الأمر كذلك، فهذا سؤال مثير للاهتمام للغاية، فكيف لم يقم أحد ببناء كاشف قادر على قياس الضوء بسرعات مختلفة؟ (ربما هذا هو الاتجاه الذي يجب أن تسلكه، فمن الممكن أن نفترض أن الشخص الذي صنع مثل هذا الكاشف ألقى به في سلة المهملات لأنه اعتقد أنه لا يعمل؟)

    2) فيما يتعلق بـ "ساعة درجة الحرارة" ما زلت أعتقد أنك مخطئ ولا يوجد أي تناقض. توأم واحد يطير ويعود بعد سنة. سيكون مندهشًا جدًا عندما يرى أن شقيقه المتبقي يبلغ من العمر 10 سنوات. وسوف يتفاجأ أيضًا أنه وفقًا لساعة درجة حرارة شابيرا، فقد مرت 10 سنوات أيضًا. ولو نظر إلى ساعة مماثلة كانت معه في سفينة الفضاء لرأى أن الساعة تعمل باستمرار أسرع بعشر مرات من المتوقع. أين التناقض؟

  375. إعادة صياغة الرد الذي كان في انتظار الموافقة لمدة 9 ساعات.
    نقطة بخصوص كلماتك: "اقترح تجربة يمكن لأي شخص إجراؤها وتثبت أن منطقك الداخلي يتحدث بالفعل عن الواقع... هذه هي الطريقة التي يعمل بها العلم، وهذا هو معنى التجربة."
    ما تقوله جميل وربما يكون مقنعا، لكنه لا يكفي لاستخدامه كدليل. في الفقرة التي تحمل عنوان "تقلص لورنس وتقلص الجاذبية" (https://www.hayadan.org.il/astronomers-reach-new-frontiers-of-dark-matter-130112/#comment-329898) لقد اقترحت، في الواقع، إجراء تجربة مايكلسون مورلي وتجربة إدينجتون وتوقعت (بعد فوات الأوان) نتائجهما. لو كنت قد نشرت نموذجي في منتصف القرن التاسع عشر، قبل عصر النسبية، لكانت تلك التجارب قد أكدت ذلك. لا يتضمن اختبار النظرية العلمية تجربة تنبؤات تتحقق فحسب، بل يشتمل أيضًا على تفسير أبسط من التفسير الموجود. وأزعم أن النموذج الذي أحضره أفضل من النماذج المادية الموجودة اليوم في أمرين: أنه أبسط وأكثر تفسيرا. لكن حتى هذا لا يعني أن نموذجي صحيح؛ إنه فقط يجعل الأمر أكثر احتمالا.
    هاريني يعتذر عن جلب الأشياء بالتنقيط الرقيق. كما ذكرنا سابقًا، هذه الكثير من المواد القديمة التي يجب علي استرجاعها من أجهزة التخزين "القديمة" وفك تشفيرها وتحريرها.
    لقد اقترح عليّ أن أفتح مدونة وأركز الأمور فيها، وأنا أفكر في ذلك جدياً.

  376. نقطة بخصوص كلماتك: "اقترح تجربة يمكن لأي شخص إجراؤها وتثبت أن منطقك الداخلي يتحدث بالفعل عن الواقع... هذه هي الطريقة التي يعمل بها العلم، وهذا هو معنى التجربة."
    ما تقوله جميل وربما يكون مقنعا، لكنه لا يكفي ليكون دليلا. في الفقرة التي تحمل عنوان "تقلص لورنس وتقلص الجاذبية" (https://www.hayadan.org.il/astronomers-reach-new-frontiers-of-dark-matter-130112/#comment-329898) لقد اقترحت، في الواقع، إجراء تجربة مايكلسون مورلي وتجربة إدينجتون وتوقعت (بعد فوات الأوان) نتائجهما. لو كنت قد نشرت نموذجي في منتصف القرن التاسع عشر، لكانت تلك التجارب قد أكدت ذلك. ولكن منذ أن جلب أينشتاين النظريات النسبية مع تلك التنبؤات، أصبحت مقبولة. لا يتضمن اختبار النظرية العلمية تجربة تنبؤات تتحقق فحسب، بل يشتمل أيضًا على تفسير أبسط من التفسير الموجود. أزعم أن النموذج الجزيئي للمادة المظلمة الذي أحضره هو أبسط من النماذج الفيزيائية الموجودة اليوم ويشرح أيضًا أكثر منها. لكن حتى هذا لا يعني أن نموذجي صحيح تمامًا؛ إنه فقط يجعل الأمر أكثر احتمالا.
    هاريني يعتذر عن جلب الأشياء بالتنقيط الرقيق. كما ذكرنا سابقًا، هذه الكثير من المواد القديمة التي يجب علي استرجاعها من أجهزة التخزين "القديمة" وفك تشفيرها وتحريرها.
    لقد اقترح عليّ أن أفتح مدونة وأركز الأمور فيها، وأنا أفكر في ذلك جدياً.

  377. تو-أوب.
    هناك جملة لا يمكن لأي شمالي يحصل على درجة الدكتوراه في علوم العشب أن يقاومها:

    "إنزلق دافين، كفى مع الأفانتي".

  378. شكرا يوفال. وهنا تتمة:

    روى راندو قصته بصوته الرقمي المعدني. والعديد من الأشخاص الذين تابعوا "تمرد الشهداء" في وسائل الإعلام - إعادة صياغة لقصة معروفة حدث فيها العكس تماما - مئات الملايين من الناس الذين يسألون أنفسهم كل يوم ما هو الهدف والمعنى من حياتهم اللطيفة كان، أشخاصًا لم يكونوا جميلين (بالنسبة لمن؟)، ليسوا طويلي القامة (نسبيًا لماذا؟)، ليسوا أغنياء (مقارنة بجرو معدم ولكن سعيد؟)، ليسوا أذكياء (مقارنة بقرد؟ أرنب؟ أو أشخاص آخرين) ، بلا قلب (هل يمكنهم حقًا فعل أي شيء آخر؟)، الذين عملوا، إذا عملوا على الإطلاق، في وظائف يكرهونها مقابل أجر زهيد دون قدرة حقيقية على المضي قدمًا، لقد حصلوا أخيرًا على تفسير الغرض من حياتهم دون رضا وأمل: لتكون خزانًا للحرارة المنخفضة التي تسمح للنظام النفسي الميكانيكي العظيم بالتكشف.

    مئير، أنا لم أنساك، أحاول أن أخصص بضع ساعات حتى أتمكن من مقارنة نظريتك مع شيماء.

    سؤال: كيف تفسر درجة حرارة الكون؟ ونظام CMBR؟ وما العلاقة بين الاثنين؟
    بالمناسبة، هل تعرف أنت أو أي شخص ما إذا كان من الممكن قياس درجة حرارة الفضاء باستخدام مقياس حرارة كلفن بسيط؟ وإذا كان الأمر كذلك، فماذا سيراه في المنطقة المظللة؟

  379. سيقدم يوفال، بكل بساطة، تجربة يمكن لأي شخص القيام بها والتي تثبت أن منطقك الداخلي يتحدث عن الواقع، وليس عن أوهامك. هكذا يعمل العلم، وهذا هو معنى التجربة.

  380. نقطة،
    ما تقوله صحيح جدا. كل شخص لديه منطقه الداخلي الخاص، والاختبار هو الواقع الخارجي. المنطق الداخلي الخاص بي يجتاز اختبار الواقع بنجاح (حسب رأيي 😛 )، لكن المشكلة التي أواجهها منذ أكثر من ألف عام هي كيفية جعل منطقتي الداخلية مجالًا عامًا.

  381. R.H.، شكرا لك 🙂
    على وجه التحديد في إنكار النفي لا يوجد تصوف. نعلم جميعًا هذا الإجراء من الحياة اليومية باسم "الدفع". "الغموض" يكمن في ابتكار سبنسر الذي أتقبله بحرارة. بما أن الأرقام "التخيلية" موجودة في المجموعة الخارجية للأرقام "الحقيقية"، كذلك يوجد "الجذر السالب" في مجموعة قيم الحقيقة الخارجية للمجموعة {"الحقيقة"، "الخطأ"". كنت بحاجة إليه لتجنب التعريف الدائري لـ "هناك" من "هناك" (لأن فعل النفي الذي نعرفه موجود أيضًا في عالم "هناك").
    في ذلك الوقت، أثار إسرائيل شابيرا سؤالاً حول ما هو جذر i. الإجابة على ذلك هي ± نصف جذر مرتين (1+i) ولا تتطلب أرقامًا خيالية إضافية. وبهذه الطريقة، يكون جذر النفي أيضًا تعبيرًا يتكون من قيم الحقيقة في المجموعة {"النفي"، "جذر النفي" } ولا يتطلب قيم حقيقة إضافية.
    وفي رأيي أن العائق الوحيد في فهم هذا الأمر هو اعتمادنا المفرط على الحدس.

  382. ربما لا يكون الأمر ممتعًا للغاية، لكن إحدى أكثر حلقات مملكة الرياضيات غموضًا يجب أن تنكشف في منبوذ... ربما الفصل الأكثر قتامة... ثورة الأعداد البسيطة تحت القيادة الشجاعة لراندو المتواضع وعدد عشوائي من جميع الأرقام.
    عندما انضم الصغير إلى عائلة الرياضيات، أقاموا وليمة على شرفه، والتي كانت مخصصة، كما وعدوا، لعالم الأرقام بأكمله. من لم يكن هناك؟ كل رجل سمين في الرياضيات، كل دوق وكل كونت، وكل من هو صغير الحجم. وسار في المجمل أعمدة لا نهاية لها، والتي تجمعت خصيصًا لهذه المناسبة. تم إعداد مراحل ترفيهية للمسلسل. ومن المفهوم أنه تم وضع حواجز علوية وسفلية في جميع الشوارع لمنع البروليتاريا من الاحتكاك بالرعاع والنبلاء. وبعد ذلك، عندما أُعطيت الإشارة، أعلن المذيع: "سيداتي وسادتي، أطلب من الجميع أن يركعوا، واسمحوا لي أن أقدم أمامكم أمراء الرياضيات الخمسة، 1، 0، i،e، وπ."
    أنت تعرف بالفعل 1، 0 وأنا. π هي بالطبع النسبة بين قطر الدائرة ومحيطها، وهي 3.14 تقريبًا. تبلغ القيمة العددية لـ e حوالي 2.72، ويتم تعريفها في حساب التفاضل والتكامل كرقم لوغاريتمه الطبيعي يساوي 1.
    ووقف الخمسة منهم على منصة الشرف أبناء الآلهة مرفوعين من الشعب، فيما يفصل المذيع نسب كل منهم وفضائله. "وها نحن هنا، لقد وصلنا إلى اللحظة العظيمة، الصيغة التعويضية التي ستوحد إلى الأبد مجالات الجبر وحساب التفاضل والتكامل والهندسة!"
    خفتت الأضواء، وعلى أصوات الأبواق والطبول، أضاء نقش ناري ضخم فوق المسرح، أضاء سماء الليل والحشد المبتهج:

    ه^أناπ+1=0

    وفقط تلك الأرقام التي لم تتم دعوتها إلى الحفلة، 1995 الرقم المعروف باسم راندو، ورفيقه 763 المعروف باسم ويلو، استدارا وبدأا بالسير نحو منطقة الشفق بين الأعداد المنتهية واللانهاية، الصفصاف الأبدي حيث يمكنك المرور بالعديد من ملايين الأعداد الصحيحة المتتالية دون الالتقاء حتى برقم أولي واحد. وسرعان ما كان هناك عمود متزايد هندسيًا من الأرقام البسيطة خلفها، أرقام مجهولة وصعوبات يومية، لم يتم ذكرها مطلقًا في أي كتاب، أرقام لم تكن جميلة، وليست مثالية، وليست أولية، ولا حتى إيجابية بالضرورة.
    كل هذا الموكب الضخم يشق طريقه ببطء أمام المنصة التي لا يزال أمراء الرياضيات يقفون عليها، ونظرات الرعب في أعينهم، لأنه عندما بدأت جميع الأعداد البسيطة في الهروب من خط الأعداد، فقدت جميع الأعداد الأولية دعمها لقد كان لديهم دائمًا على اليمين واليسار، وسرعان ما تم تقليصهم جميعًا إلى نقطة واحدة واحدة: 0 .

    لأن هذه هي طبيعة محور الأعداد: كل رقم في حد ذاته، مهما كان مهما ومتميزا، ليس أكثر من نقطة لا أبعاد لها، لكن الجمع المتتالي لجميعها يحولها إلى خط له بعد طول.

  383. يعتمد يوبيل العلم على أشياء يمكن للآخرين فهمها. إذا لم يفهمك أحد، فذلك لأن لديك منطقك الداخلي الخاص، وتعتقد أنه صحيح (بشكل طبيعي). الجزء الصعب نفسيًا من العلم هو التخلي عن ثبات هذا المنطق الداخلي، لأن ما يحدد ليس المنطق بل الواقع. ويتم تعريف الواقع على أنه نفس الشيء المشترك بين الجميع.

  384. اليوبيل,
    لا تجعل مثل هذا الوجه حزينا. انا احاول ان افهم. حتى لو لم أتفق مع الكثير من الأشياء التي تقولها.
    على سبيل المثال، فيما يتعلق بجذر ناقص واحد (i)، يتبع ذلك ببساطة إذا قمت بإضافة محور رقم رأسي إلى المحور الحقيقي. أي، إذا قمت بإنشاء نظام ديكارتي حيث X هي الأعداد الحقيقية كمضاعفات للعدد 1 وY هي الأعداد التخيلية كمضاعفات للعدد i. يتم تمثيل كل رقم في الشبكة بإحداثيتين للمكون الحقيقي والمكون المحاكى له، لذلك يسمى رقمًا مركبًا.. لا يوجد شيء خاص أو باطني في نفي النفي وما إلى ذلك.
    علاوة على ذلك، من الممكن تعقيد وإضافة المزيد من المحاور والأبعاد (على الرغم من أنه من الصعب حقًا تخيل ما يحدث خارج المحور الثالث).

  385. ولمن لا زال يتابع، ملحق لقسم "الأسطوري" في الرد التالي: https://www.hayadan.org.il/astronomers-reach-new-frontiers-of-dark-matter-130112/#comment-329001
    لقد كان من السهل بالنسبة لي أن أتخلص من النفي الذي نعرفه في الحياة اليومية، وهو آلية الجوهر الذي يسبق كل "هناك". ومع ذلك، بهذه الطريقة أصرح بشكل اعتباطي أن "اللا شيء" الذي يسبق كل "هناك" له خصائص موجودة في واقعنا "الموجود". هذا تعريف دائري وبالتالي لا يمكن أن يكون هذا النفي هو عامل التكوين. في الحياة اليومية، نحن لا نعرف علاقة منطقية محلية يؤدي تطبيقها على نفسها إلى النفي. واجه علماء الرياضيات مشكلة مماثلة عندما حاولوا العثور بين الأعداد المعروفة على رقم تكون نتيجته عند ضربه في نفسه رقمًا سالبًا. ومع تطور الفكر الإنساني وتعمق فهم طريقة العالم، أصبح من الواضح أن وجود مثل هذا العدد مقيّد بالواقع وإن كان في تجلياته الخالصة لا يعبر عنه بالحدس اليومي المعروف. الواقع. حصل هذا الرقم على لقب "خيالي" غير الناجح. في الإسقاط من الأرقام إلى المنطق، يمكن استخدام النظير المنطقي لهذا الرقم كجذر للنفي. مثل العدد "الخيالي" في الرياضيات، لا نعرفه حدسيًا ولا نستخدمه في الحياة اليومية؛ ومع ذلك، في استخدامه كمصدر للنفي، فهو ليس إسقاطًا من واقعنا اليومي، وبالتالي فإن تعريفه لـ "نعم" ليس دائريًا.
    الكشف الكامل: فكرة الجذر السالب ليست فكرتي الأصلية ولكنها فكرة عالم رياضيات بريطاني يدعى جورج سبنسر براون. يمكن العثور على تفاصيل حول تعاليمه في الكتاب
    قوانين الشكل

  386. إسرائيل،

    شكرا للمجاملة.

    "أو ربما تحتوي الصيغة M بالفعل على الكتلة المظلمة؟"

    وفقًا لواجبي المنزلي، M لا تتضمن الكتلة المظلمة، ولا الطاقة المظلمة (حصلت على أن M أصغر بمقدار 16.4 مرة من الكثافة الحرجة).

    في رأيي، على الرغم من أن الكتلة الكونية بما في ذلك الظلام ربما تكون حوالي 10٪ من الكثافة الحرجة، لذا فإن العلاقة عدديًا مقصودة تمامًا. لكن فيما يتعلق بالفيزياء، فإن هذا الارتباط غير ضروري بالنسبة لي. في بضع عشرات من مليارات السنين، بعد فترة طويلة من غياب الشمس، ستكون كتلة الكون المرصود صغيرة وتساوي كتلة المجرة، وسيظل G هو R، وG سيكون R، وC سيكون C، أي أن العلاقة ستكون غير ذات صلة.

  387. مئير.
    لقد بدأت في الاطلاع على مقالاتك قليلاً. أولاً، احصل على الثناء: ليس هناك شك في أنها مكتوبة بشكل أفضل بكثير من معظم المقالات التي مررت بها حول ما أعرّفه بـ "النظريات البديلة".

    سيستغرق الأمر بعض الوقت للاطلاع على كل التفاصيل، فأنا مشغول جدًا ويجب علي أيضًا التعامل مع فكرتي. لكن لدي سؤال يتعلق بالفصل الثالث، المتعلق بالطاقة المظلمة، وفي الواقع أيضًا بهذه المقالة المتعلقة بالمادة المظلمة.

    إذا أخذنا صيغة صديقنا من المدونة، والتي كما ذكرنا يمكن كتابتها بالشكل GM=RC^2، فقد حصلنا على صيغة جميلة تصف العلاقة بين الثوابت، ومن بينها كتلة الكون ولكن لا توجد كتلة مظلمة .

    فكيف يتناسب هذا مع فكرة أن هناك كتلة أكبر بخمس مرات على الأقل مما هو مقبول؟ ألا ينبغي لنا أن نضيف عاملاً آخر في مثل هذه الصيغة الأنيقة؟ ألا ينبغي لهذا التناقض وحده أن يقتل فكرة الكتلة المظلمة برمتها؟ بعد كل شيء، ليس هناك شك تقريبًا في قيم G وR وC.

    أو ربما M في الصيغة يتضمن بالفعل الكتلة المظلمة؟

    الذهاب إلى النوم، وسوف نناقش في وقت لاحق.

  388. مئير
    لا بد لي من الطيران للعمل. أعطني بعض الوقت، قد يكون الحل الذي تقدمه هو ما هو مطلوب. لكن كما ترى، بعد كل شيء، أينشتاين... شيمي... ماخ...
    استمتع بالثلج. ألا تجعل رقاقات الثلج القانون الثاني للديناميكا الحرارية أكثر صعوبة بعض الشيء؟

  389. شيء آخر. عندما ادعت شيماء أن حرف G يتغير، لم يقل ذلك من أجل إنقاذ ماخ من مطالبة مئير عميرام.
    والسؤال الذي يطرح نفسه، كيف يحدث أنه بعد مائة عام من الحرث بمبدأ ماخ، يحتاج المواطن العادي إلى اكتشاف ذلك فعليًا لكي يتغير تقعر الماء كدالة للكتلة العالمية وفقًا لمبدأ ماخ، يحتاج G بالضرورة إلى التغيير مع الكتلة العالمية.

  390. من العار أن تهتم. هذه ليست القضية. أعلم أن دينيس شيمي يدعي هذا، وأفترض أنه عندما تتحدث عن الشكلية، فإنك تتحدث عن تفسير شيمي لمبدأ ماخ.

    والسؤال هو ما إذا كان ماخ نفسه قد ادعى هذا. من الواضح أنه لا، لأن شييم اخترعه بعد أن كان ماخ موجودًا بالفعل في عالم خالٍ من الجاذبية.

    بعد كل شيء، بدأ كل شيء عندما سألتني لماذا كتبت أنه وفقًا لماخ فإن الماء سوف يفيض بسبب ذرة الهيدروجين التي تبعد عشرة مليارات سنة ضوئية. شرحت السبب. وبأي طريقة ننظر إليها، من المحير أن ماخ لم يلاحظ أن الماء سيحافظ على التقعر (لأنه لم يقل أن G يتغير. إذا أراد أن يقول ذلك، لكان قد نادى الطفل باسمه). .

    فصحيح أنه بحسب شيمي له بعث (غريب، وأن محاولة اكتشافه بالقياسات باءت بالفشل)، لكن هل مبدأ ماخ التاريخي يخضع لفرضيات شيمي المثيرة للجدل؟

  391. أعتقد أنه يحدث عندما تكون الكتلة الكونية أكثر تركيزًا. أعطني بعض الوقت، سأحاول أن أقدم لك كل الشكليات.

  392. إسرائيل،

    "بسيط جدًا: قبل 5 مليارات سنة، بنفس سرعة الدوران، كان من الممكن أن تحصل على المزيد من الانقلابات. السبب: كتلة أكثر تأثيرا وأقرب."

    ماذا عن الجاذبية؟ أليس هو لاعب في هذه اللعبة؟

    ارسم بنفسك متجهات القوة المؤثرة على جزيء ماء واحد اكتسب ارتفاعًا في الدلو.

    ففي نهاية المطاف، إذا كان يدور بنفس السرعة الحالية، فإن تسارعه الشعاعي هو نفس التسارع الحالي. إذا زعمت أن قوة الطرد المركزي المؤثرة عليه تبلغ ضعف ما هي عليه اليوم، فإن الشيء الوحيد الذي تغير هو كتلته.

    نفس التسارع، قوة الطرد المركزي المزدوجة = مضاعفة الكتلة.

    إذا كانت كتلته مضاعفة، فإن قوة الجاذبية المؤثرة عليه تكون مضاعفة (ولقد قمت بافتراض لك هنا).

    الآن اشرح ماذا يحدث لجزيء الماء الذي يكون في حالة اتزان بين قوتين عندما تزداد قوة القوتين بنفس النسبة؟

  393. الفترة، هيا.
    ربما يمكنك بالفعل التركيز على موضوع معين؟
    إذا كانت "السطحية" تصف بعدين، فما الذي يصف "السطحية"؟ 0 الأبعاد؟
    ماذا عن الروابط إلى غير محلية؟ العالم يحبس أنفاسه.
    إذا كان لديك شيء محدد لتقوله من البداية إلى النهاية، فهذا هو المكان المناسب. خلاف ذلك، يرجى تجنب التعميمات الغبية، والانتقال إلى مقالات أخرى. هناك مقالة مفتوحة عن القرود. ألا تعتقد أن هذا هو مكانك الطبيعي؟

  394. مئير.
    بداية دعوني أوضح موقفي:
    1. أنا لست منغلقًا على نظام Mach.
    2. على حد علمي، كان أينشتاين متحمسًا لماخ في البداية، ثم أصيب بخيبة أمل منه لاحقًا. ادعى أينشتاين في النسبية العامة، مثلك في الواقع، أنه حتى في الكون الفارغ فإن الدلو سوف يدور. أنا لا أفهم كيف.
    3. حدسيًا - أنا أتفق مع ماخ.
    4. منطقي - إذا أنكر أينشتاين ماخ أخيرًا، فيجب علينا أن نفهم السبب، ويجب أن ننطلق من افتراض أن الخطأ كان من نصيبي.
    5. أنت تدعي ما ادعى أينشتاين. سوف يدور الماء في الدلو مهما حدث.
    6. ومن هنا مناقشاتنا.

    "لا تتأثر درجة التسارع الشعاعي بحجم الكتلة." لا أعرف إذا كنت تقصد كتلة الماء أم كتلة الكون. أظن أنك تقصد الماء. وفقًا لماخ، فإن مقياس كتلة الكون هو الذي يحدد قوة القصور الذاتي المؤثرة على وحدة كتلة الماء، لذلك لا يوجد تناقض في نظرية ماخ.

    "إذا كانت جزيئات الماء الموجودة في الدلو عديمة القصور الذاتي تقريبًا (= كتلة قصورية صغيرة)، فما الذي سيمنعها من التسلق وخلق الفيضان؟"

    لا شيء سيمنعهم، لكن لا شيء سيسببهم أيضًا.

    "سيبقون بالضبط في نفس التوازن بين قوة الجاذبية التي تمارسها عليهم الأرض نحو الأسفل وقوة الطرد المركزي التي تسرعهم نحو الجانب."

    وفي مثال الكون الفارغ، لا توجد أرض ولا قوة طرد مركزية. إذا كنت تقصد كونًا فارغًا لا يوجد فيه سوى الدلو وما إلى ذلك، فهذا نظام مختلف عما رأيناه حتى الآن.

    "هل تريد أن تدعي ادعاءً وهميًا بأن كتلة القصور الذاتي يتم ضبطها وفقًا لكتلة الجاذبية على وجه التحديد في كون له نفس كتلة كوننا تمامًا؟"

    أنا لا أفهم هذا الادعاء. إذا كان ذلك ممكنا، التفاصيل.

    "إن الكتلة الثابتة للسديم الذي تشكلنا منه كانت أثقل بكثير لأنه تعرض لكتلة كونية أكبر بثماني مرات."

    أفترض أنه وفقًا لمبدأ ماخ، كانت القوة المؤثرة على الدلو في تلك الأوقات أكبر بثماني مرات. لا أرى ما هي المشكلة.

    "لقد كان الرجل مشغولاً للغاية بمسألة القصور الذاتي لدرجة أنه نسي أن يأخذ في الاعتبار القوى الأخرى المسؤولة عن الشكل المقعر للمياه التي تتحرك في الدلو."

    ما القوى؟ كوريليوس؟ وفقا لماخ، لا ينبغي أن يكون هناك المزيد من القوات.

    "كيف ولماذا سيتأثر محيط تجاويف الماء في الدلو الدوار على الكوكب X لمدة 14 مليار سنة في كون افتراضي حيث يوجد اضمحلال خطي للكتلة"

    الأمر بسيط جدًا: قبل 5 مليارات سنة، بنفس سرعة الدوران، كنت ستحصل على المزيد من الانقلابات. السبب: كتلة أكثر تأثيرًا وأقرب.

  395. إسرائيل، أنتم تقومون فقط بتوسيع وتضخيم الأمور التي تمت مناقشتها واستخراجها في الماضي. وتظن أنك تكتشف شيئًا جديدًا. إنه أمر مثير للسخرية بعض الشيء.

  396. إسرائيل،

    تحقق من نفسك. لا تتأثر درجة التسارع الشعاعي بحجم الكتلة.

    ادعى ماخ أن دوران الدلو ليس له معنى إذا لم يكن هناك ما يرتبط به. ولهذا الغرض ولهذا الغرض فقط أحضرت ذرة الهيدروجين. لدينا هنا ذرة هيدروجين على بعد 10 مليارات كيلومتر، وهنا لدينا ما ننسب إليه دوران الدلو.

    هل سيتم جرف دلو يدور في كون فارغ (لا يوجد فيه سوى مراقب ضئيل على مسافة 10 مليارات كيلومتر) أم لا؟
    أعتقد أنهم سوف يتداخلون بنفس القدر الذي يتداخلون به في عالمنا. ماخ وأنت تدعي أنهما لن يتداخلا.

    هو دلو يدور في كون فارغ (لا يوجد فيه سوى دلو من الماء معلق على حامل ثلاثي الأرجل، وكرة أرضية وحيدة في الظلام موضوع عليها، وأضفتها لأسباب عملية: لا يدخل) في السؤال المتعلق لماذا يدور الدلو، وعلى ماذا يعلق، ولماذا يبقى الماء في الدلو ولا يطفو في ظروف نقص الوزن)، هل سيتم غسلها أم لا؟ أعتقد أنهم سوف يتداخلون بنفس القدر الذي يتداخلون به في عالمنا. ماخ وأنت تدعي أنهما لن يتداخلا.

    سؤال:
    إذا كانت جزيئات الماء الموجودة في الدلو عديمة القصور الذاتي تقريبًا (= كتلة قصورية صغيرة)، فما الذي سيمنعها من الصعود وإحداث الفيضان؟ وبنفس القدر الذي فقدوا فيه كتلة الثبات (نتيجة لفقدان تأثير الكتلة الكونية)، فقدوا أيضًا كتلة الجاذبية. قوة الجاذبية المؤثرة عليها صغيرة مثل قوة الطرد المركزي. سيبقون بالضبط في نفس التوازن بين قوة الجاذبية التي تمارسها عليهم الأرض نحو الأسفل وقوة الطرد المركزي التي تسرعهم نحو الجانب.

    هل تريد أن تدعي ادعاءً وهميًا بأن كتلة القصور الذاتي تتلاءم مع كتلة الجاذبية على وجه التحديد في كون له نفس كتلة كوننا تمامًا؟

    بالمناسبة، لاحظ أنه وفقًا لادعائك (وكشخص يتبع المسار الذي حدده ألبرت) فإن قوانين الفيزياء تتغير بمرور الوقت، نظرًا لأن كتلة الكون المرئي تصبح أصغر. في كل يوم من أيام توسع الكون، نفقد مجرات ثمينة. قبل 7 مليارات سنة، كان المحتوى الكتلي للكون المرئي حوالي ثمانية أضعاف ما هو عليه اليوم، مما يعني أن كتلة الثبات للسديم الذي تشكلنا منه كانت أثقل بكثير لأنه تعرض لكتلة كونية أكبر بثماني مرات.

    يمكنني أن أخرج مبدأ ماخ من هذا المثابر باحترام نسبي، لكن لا أستطيع أن أنقذه من الخطأ المحرج فيما يتعلق بالدلو. لقد انشغل الرجل بمسألة القصور الذاتي لدرجة أنه نسي أن يأخذ في الاعتبار القوى الأخرى المسؤولة عن الشكل المقعر للماء الذي يدور في الدلو.

    إذا كنت تعتقد أن هذا ليس خطأ محرجًا من جانبه، فلا ترسلني إلى الأقراص الدوارة (التي ناقشناها وسمحنا بها بالفعل)، ولكن اشرح لي بقدرتك التفسيرية الرائعة المعتادة، كيف ولماذا محيط ستتأثر تقعرات الماء في الدلو الدوار على الكوكب X لمدة 14 مليار سنة في الكون الافتراضي حيث يوجد اضمحلال خطي للكتلة.

  397. مئير
    وفقًا لمبدأ ماخ، إذا كان الدلو بمفرده في الكون الفارغ، فلن يفيض الماء، بغض النظر عن سرعة دوران الدلو. إن حقيقة أنك أضفت بروتونًا واحدًا في مكان ما سيكون لها تأثير ضئيل بحيث يمكن ببساطة تجاهل التأثير.
    إذا قمت بإضافة كل الكتلة الموجودة اليوم في الكون في نفس مكانها بالضبط ولكن مقسومة على 2 - فإن درجة التداخل ستنخفض بالنسبة المناسبة. ثلاثة أضعاف كمية الكتلة - سوف تزيد بنسبة مناسبة. (ليس 3 مرات، يمكنك حساب المبلغ بالضبط).

    هذا هو بالضبط سؤالي الأصلي: إذا كان هناك قرصان يدوران بالنسبة لبعضهما البعض - ما هو القرص الدوار الحقيقي - بالنسبة إلى ماذا؟
    إليبا داماخ - نسبة إلى المادة الموجودة في الكون.

  398. إسرائيل،

    "لدي سؤال فقط: لقد قمت بمراجعة مدونتك قليلاً. لماذا تعتقد أنه وفقًا لمبدأ ماخ (وهو خطأ بالنسبة لك) إذا كان هناك بروتون واحد في الكون الفارغ يحيط بالدلو على مسافة ملايين الكيلومترات منه، فإن الماء سوف يفيض الدلو؟ من الناحية النظرية نعم، ولكن إلى حد لا يذكر بحيث لا يوجد أي تناقض مع المبدأ نفسه.

    لماذا إلى حد الصفر؟
    التسارع الشعاعي دائمًا هو Vsquared/R، بغض النظر عن الكتلة (في حالتنا، كتلة الثبات). ولذلك فإن تقليل كتلة الثبات لن يغير من درجة فيضان الماء في الدلو.

    تجربة فكرية: بالنظر إلى سرعة زاوية معينة، هل سيكون تقعر الزئبق الذي يدور في دلو مختلفًا عن تقعر الأيزو بنتان أو الماء أو أي سائل آخر (مع الأخذ في الاعتبار الكتلة، وتجاهل الاختلافات الناتجة عن اختلاف اللزوجة)؟

    بالطبع لا، لأنه كلما كانت الكتلة أصغر، كان من الأسهل أن تنحرف عن المركز وتتراكم عند المحيط.

  399. R.H. Rafai.M،
    مسألة ما إذا كنت أشعر بالخجل أم لا ليست ذات صلة. إذا كان كل ما تراه في أشيائي هو مادة خام للتذمر، فلا يسعني إلا أن أشعر بالأسف من أجلك. أنا لا أدعي بيقين مطلق أن الكون مبني من الجزيئات التي أعرضها هنا. كل ما أدعي القيام به هو إظهار كيف أنه من الممكن، بوسائل بسيطة، بناء شيء يشبه الكون الذي نعرفه. ربما الجسيمات التي أتحدث عنها هي بالضبط الجسيمات التي يتكون منها الكون، أو ربما لا. ولكن هذا ليس نقطة.

  400. انكماش لورنتز وتسارع الجاذبية:
    تعتمد حركة الجزيئات بشكل عكسي على الكثافة التي تم ترتيبها بها، وبالتالي تعتمد أيضًا على حركة الثقوب. ومع زيادة الكثافة التي تترتب فيها الجزيئات، يقل حجم الثقوب الموجودة بينها وتقل حركتها. ويتجلى ذلك، من بين أمور أخرى، في تقليل سرعة تقدم الثقوب. دعونا ننظر إلى بروتون مرة أخرى. وكلما زادت سرعة حركته، زادت النسبة بين كثافة الجزيئات التي أمامه وكثافة الجزيئات التي خلفه. إن حركة الثقب (أو موجة الثقوب) التي تقع في بيئة كثيفة ستكون أبطأ من حركتها في بيئة أقل كثافة.
    وإذا قارنا فوتونات الفيزياء بثقوب النموذج، يظهر أن سرعة الضوء تختلف باختلاف كثافة البيئة التي يتحرك فيها. ولذلك فإن سرعة الضوء تجاه الجسم المتحرك هي نفس سرعته تجاه الجسم الساكن.
    يحتفظ البروتون في بيئته المباشرة بجزيئات ذات كثافة أعلى من كثافة البيئة البعيدة عنه. البيئة الكثيفة تجذب جزيئات أكثر من البيئة المتفرقة وبالتالي تجذب المزيد من الثقوب إليها. وبالتالي فإن فرصة تحرك الثقب بالقرب من البروتون في الاتجاه نحو البروتون أكبر من فرصته في التحرك في الاتجاه المعاكس.
    وهنا أيضاً، إذا قارنا فوتونات الفيزياء بثقوب النموذج، يبدو أن ما تسميه الفيزياء "انحناء الفضاء" ليس أكثر من تغير في كثافة جسيمات المادة المظلمة بحسب قربها من السماء. غرام.

  401. اليوبيل!!

    لقد بدأت تعليقك: "المزيد عن الجسيمات والمساحة الفارغة:
    لم يتم تعريف مصطلح "قطعة من الفضاء الفارغ" بالتفصيل حتى الآن: فهي مساحة متصلة من الفضاء الفارغ..."

    قل لي، ألا تخجل؟
    هل تريدنا أن نستمر في إزعاجك؟

  402. نقطة، شكرا لهذه المعلومة.
    أحتاج حقًا إلى نموذج كمبيوتر يساعدني في توضيح الأشياء وربما يحل بعض الأسئلة المفتوحة بالنسبة لي. أفترض أنك بكلمة "ساذج" تقصد ما أحمله هنا. وهذا صحيح، ويرجع ذلك أساسًا إلى الطريقة الساذجة التي أتناول بها الأشياء بدون صيغ رياضية في الوقت الحالي. حتى عندما يحين دور الصيغ الرياضية، فإنها لن تكون معقدة. تنشأ الحاجة إلى الرياضيات الجامعية (المصفوفات والمتجهات والمعادلات التفاضلية وما إلى ذلك) عند ربط عدد كبير من الهياكل الفردية ولكن بها. لكني الآن مازلت على مستوى الهياكل الفردية.

  403. يوفال، سوف تتعلم برنامج OpenGL وستكون قادرًا على تحميل عالمك على الكمبيوتر وستكون فخورًا بعملك.
    الكون الساذج لا علاقة له بفيزياء كوننا.

  404. المزيد عن الجسيمات والمساحة الفارغة:
    لم يتم تعريف مصطلح "قطعة من الفضاء الفارغ" بالتفصيل حتى الآن: فهي منطقة مستمرة من الفضاء الفارغ يمكن لجسيم معين أن يتحرك فيها بحرية. بهذه الطريقة، يمكنك أيضًا النظر إلى الفضاء الفارغ كما لو كان يتكون من نوع من الجزيئات المنفصلة وليس مجرد "سحابة". أدناه، من أجل الإيجاز، سوف نسمي قطع الفضاء الفارغ "الثقوب" والجسيمات الدقيقة "الجسيمات". سننظر الآن إلى الطريقة التي يتحرك بها الثقب: عندما يتشكل الثقب في منطقة معينة (على سبيل المثال بعد تحول جسيم إلى مساحة فارغة بسبب الكثافة العالية) يمكن لجزيئات من البيئة أن تتحرك فيه. عندما يتحرك جسيم ما إلى داخل ثقب، فإنه يقلب المساحة الفارغة في الاتجاه المعاكس وينتج عن ذلك إنشاء ثقب في مكان جديد. يمكن للجسيمات أيضًا أن تدخل هذا الثقب الجديد وتتحرك الثقب. وبالتالي يمكن للثقب أن يتحرك لمسافات كبيرة في حين أن كل من الجزيئات التي ساهمت في حركته لا تتحرك إلا لمسافة قصيرة.
    الآن سنرى كيف أن المساحة الفارغة لها سلوك مزدوج:
    إذا كانت هناك منطقة تنتج ثقوبًا باستمرار، والتي ستسمى أدناه "البؤرة" (وسنرى لاحقًا كيف يمكن أن يوجد مثل هذا الشيء)، فستنتشر الثقوب دون تفضيل الاتجاه من البؤرة إلى الخارج. كل ثقب عبارة عن وحدة فردية، لكن مجموعة الثقوب معًا تشكل جبهة موجية.

  405. ر.ح.
    أعيد قراءة رد الأسماك الأصلي. ليس هناك إشارة إلى الضوء. هناك إشارة إلى السمكة. السمكة ليست خفيفة. السمكة هي سمكة. يعتقد الكثير من الناس أنهم عندما يكتبون سمكة فإنهم يقصدون سمكة. سيتمكن قراء العقول من معرفة أنهم عندما يكتبون سمكة فإنهم يقصدون الضوء. أنا لا أقرأ العقول. أشير فقط إلى ما هو مكتوب. لقد كتب السمك. كنت أشير إلى الأسماك. لو كان مكتوبًا "اقرأ مرة أخرى ما كتبته وبدلاً من السمكة اقرأ الضوء ثم اتخذ قرارك" كما في تعليقك الأخير، لكنت قد فعلت ذلك. لقد شككت في أن هذا هو ما تقصده، لكنني لم أكن متأكدا. ولهذا السبب أشرت فقط إلى الأسماك. لصيد الأسماك

    في الحقيقة
    يبدو لي أنك انتقلت من تجربة M-M إلى نتيجة أينشتاين. وهذا موضوع جديد قد نتطرق إليه لاحقا. ومع ذلك، فأنا لا أتفق مع تأكيدك على أن "تجربة MM أرادت اختبار سرعة السمكة (الضوء) بالنسبة إلى العوامة (الأرض) وما وجده هو أنه بغض النظر عن كيفية ومكان قياسها، فإن السمكة تسبح بسرعة نفس السرعة." إنها ليست دقيقة، ولا حتى دقيقة جدًا. يمكن تفسير نتائج تجربة MM بسهولة من خلال افتراض أن الضوء يتحرك بنفس السرعة (سرعة الضوء) بالنسبة لمصدر الضوء، مثل رصاصة بندقية تتحرك بنفس السرعة بالنسبة للبندقية. إذا كنت لا تصدق ذلك، قم بالتجربة، وسترى أن الأمر واضح جدًا.

    ولهذا السبب آمن الكثيرون أيضًا بنظرية الانبعاث، وهي نظرية منافسة للنظرية النسبية، والتي بموجبها يتحرك الضوء بسرعة ثابتة بالنسبة للمصدر. ولكن هناك العديد من المشاكل مع هذه النظرية كما يمكنك أن تقرأ على ويكي (كنافو). على أية حال - لم أتلق بعد إجابة لسؤال الصلاحية المنطقية لتجربة MM.

    إذا أردت، يمكننا أن نترك هذا السؤال مفتوحًا الآن ونراجع استنتاج أينشتاين، ولماذا أعتبره مشكلة، وكيف يمكن لفكرتي أن تساعد في حلها.

    وفيما يتعلق بتجربة دوروثي/إيشباه، فإن إحدى التجارب الشهيرة هي أن الفزاعة ستعلن دائمًا "والآن تعزف آلة موسيقية، معلنة قدوم ملك أوز!" قبل دخول أيوب الجوبنيك.

    مئير.
    لدي سؤال فقط: لقد تصفحت مدونتك قليلاً. لماذا تعتقد أنه وفقًا لمبدأ ماخ (وهو خطأ بالنسبة لك) إذا كان هناك بروتون واحد في الكون الفارغ يحيط بالدلو على مسافة ملايين الكيلومترات منه، فإن الماء سوف يفيض الدلو؟ من الناحية النظرية نعم، ولكن إلى حد لا يذكر بحيث لا يوجد أي تناقض مع المبدأ نفسه

    اليوبيل…
    مشاكل بسيطة!

    أنا لا أشك في أي شيء. ببساطة، كتبت أن الكثيرين فهموا كيفية وجود الجاذبية في نموذجك حتى بدون رد الفعل الذي ذكرته، وأن سوء التفاهم لا يكون إلا مع إسرائيل المغفل، كما هو الحال دائما. وبما أنني لا أعتقد أنه من الممكن فهم كيفية وجود الجاذبية في النموذج الخاص بك دون هذه الاستجابة، فقد خلصت إلى أنك أرسلته إلى الخبراء فقط. الآن، أعرف الإجابة: ربما لم يفهمها أحد، لأنه لم يفهمها أحد.

    لكن اترك الهراء يا أخي. أعتقد أنني بدأت أرى إلى أين أنت ذاهب، لكن بما أن الطبيب أعطاني إعفاءً، فسوف أحقق أقصى استفادة منه.
    لكن كن حذرًا: يبدو لي أن النموذج، على الأقل من حيث الجاذبية، هو مجرد لاسيج مقنع.
    ربما من الأفضل عدم إلغاء الإعفاء..

  406. الطالب، التخنيون (وجميع غيرهم من الصبر)،
    أنا آسف لأنني لا أتمكن من تلبية الطلب بالسرعة التي تناسبك. أنا أتعامل مع استعادة المواد من معالجات النصوص في عصور ما قبل التاريخ وتصحيحها. بالإضافة إلى ذلك، أنا مشغول جدًا خلال النهار.

  407. إسرائيل،

    أنا آسف لكنك لم تفهم مثل السمكة. إما أنني لم أكن واضحا بما فيه الكفاية أو أنك قرأت على عجل. في النهاية سأفكر مثل يوفال أنك لا تهتم حقًا بالتعليقات الموجهة إليك.
    أعد قراءة ما كتبته وبدلاً من السمكة، اقرأ الضوء ثم اتخذ قرارك.
    كما أنك تستمر في تكرار عبارة "المحيط مرتبط بالأرض". انسَ أمر الأرض، ففي المثل أعلاه لا توجد أرض. المحيط هو الفضاء المملوء بالماء (الأثير). أرادت تجربة MM اختبار سرعة السمكة (الضوء) بالنسبة إلى العوامة (الأرض) وما وجده هو أنه بغض النظر عن كيفية ومكان قياسها، فإن السمكة تسبح بنفس السرعة. ومن هنا كان استنتاج أينشتاين، والذي قبله مجتمع الفيزيائيين بأكمله، هو أن سرعة الضوء ثابتة، وليس أنه عند السرعة العالية تصبح السمكة شفافة أمام أجهزة القياس. لست متأكدا من أي من الاستنتاجات أكثر غرابة، ولكن بقدر ما أفهم، فإن العديد من التنبؤات بالنظرية النسبية قد تحققت حتى الآن. انتظر لحظة، سأقوم بتوصيل نظام تحديد المواقع العالمي (GPS)، ثانية، أوه، أيها العبد! تجربة أخرى أكدت تنبؤات النظرية النسبية كانت ناجحة.

    أحببت دير بلاك باللغة الإنجليزية 🙂 وأما أيوب فلا أفهم من أين وصلوا أنه كان مستشارًا لفرعون، على الأكثر كان مستشارًا لدوروثي.

  408. يوفال، باختصار، لقد فشلت في تفسيراتك وفي منطق التفسيرات. من ناحية، أنت لم تبدأ من البداية (تم توفير الوقت والمكان لك دون أي تنازلات وشرح سبب اختيارك لثلاثة أبعاد للمكان)، ومن ناحية أخرى، مازلت لم تصل إلى أي نتيجة.

    "العالم" الذي تحاول بناءه هو أكثر ملاءمة لتلك العوالم الافتراضية الموجودة في ألعاب الكمبيوتر، فهي مكونة من جزيئات دقيقة (مثلثات) تتحرك في مساحة فارغة (المسافة بين المثلثات) وهكذا نحصل على عرض تقديمي لعالم ثلاثي الأبعاد على شاشة جهاز الكمبيوتر الخاص بنا.
    وهذا لا علاقة له بالفيزياء. هذا مجرد خيال.

    لقد بحثت، ربما لديك بعض المبادئ الفلسفية المثيرة للاهتمام، بدأت بـ "الوجود مخلوق من العدم"، وهي عبارة تحتوي على تناقض. لم تشرح لها، لقد ركضت فقط كما لو كنت أشرح كل شيء، لكنك لم تشرح أي شيء، لقد ذكرت للتو أن الأمر كذلك. ولم توضح كيف من العدم خلق شيء من هذا القبيل ولا يوجد غيره.. باختصار لقد فشلت في محاولاتك.

  409. اليوبيل,

    حسنًا، متى سنرى الارتباط بفيزياء الكم؟ الحسابات على طول الرابطة الكيميائية والمعادلات المطلوبة والارتباط بالتحليل الطيفي؟ هل سنحصل على تفسيرات حول الرنين المغناطيسي النووي (NMR) وESR وتكثيف بوز أينشتاين؟ ولتذكيرك، لكي تدخل في مجلة علمية، ستحتاج إلى أكثر من مجرد قصة خيالية عن الجسيمات الدقيقة، والمساحات الفارغة، والإلكترونات ذات الطبقات الرقيقة، والفوتونات ذات القطر.

  410. إسرائيل!
    هل تحاول الإشارة ضمنًا إلى أنك تعلم (وليس فقط الشك) أنني أخفي شيئًا ما؟ إذا كان الأمر كذلك، يرجى الكشف عن مصادرك. وإذا لم يكن هناك، فيرجى الاحتفاظ بشكوكك لنفسك.
    صباح الخير لمدينة الملائكة

  411. نقطة،
    إن افتراضك حول المكان والزمان صحيح بالفعل. ولم أقدم أسباب وجود فضاء الدير ثلاثي الأبعاد (الذي يمكن فيه التحرك ذهابًا وإيابًا) والفضاء الزمني أحادي البعد أحادي الاتجاه. تنتمي هذه الأشياء إلى خطوة سابقة في النموذج الذي فضلت تخطيه هنا.
    وصحيح أيضًا أنني أقوم ببناء نموذج للعالم من "طوب الليغو".
    أنت أيضًا على حق في تعليقك الأكثر عمومية بأن مجموعة الحقائق التي قدمتها حتى الآن لا تثبت أي شيء. المشكلة هي أنني أكتب ببطء ولم أتمكن بعد من الوصول إلى الأشياء "الدرامية".
    وإذا أراد موقع العلوم حذف كلامي فلن أقف معه. لكن موضوع البحث الحالي هو المادة المظلمة، ونموذجي يتناول ذلك بالضبط.

  412. نقطة.
    أنت لا تجيب على السؤال الذي طرحته. اترك الآن مع سؤال الصواب أو الخطأ.

    إذا تابعت المناقشة هنا، في رأيي أن نموذج الموقع صحيح بالفعل.

    مساء الخير.

  413. يوفال، مجموعة "الحقائق" التي تدعيها لا تفسر أي شيء. بشكل عام، بطريقة غير واعية، يبدو أنك تفترض أن "المكان" و"الزمن" هما كائنان موجودان، ولم تحدد مما يتكون هذا الفضاء الفارغ، وما إلى ذلك. باختصار، أنت تحاول فقط بناء عالم من قطع الليغو البسيطة التي لا علاقة لها بعالمنا.

    مثل هذه المحاولات لا علاقة لها بالموقع العلمي.

  414. وماذا عن هذا التعليق؟

    https://www.hayadan.org.il/astronomers-reach-new-frontiers-of-dark-matter-130112/#comment-329304

    ألم تشاركها مع أحد قبل نشرها على الموقع؟

    وكيف تعرف ماذا أفعل بحياتي؟ ليس هذا ما أعرفه، لكن كيف حالك؟
    ليلة سعيدة اليوبيل. ولم أتلق طلبًا صريحًا بعد. وفي هذه الأثناء سأستفيد من الإعفاء وأذهب إلى ما بعده.

  415. إسرائيل، لا أفهم إلى أين أنت ذاهب. أنا لا أفهم ما لا تفهمه.

    تاريخيًا، كان يُعتقد أننا نتحرك داخل الأثير. الفكرة هي أن النجوم ساكنة (وهذا في حد ذاته لا يمكن أن يكون صحيحا بسبب الجاذبية) داخل الأثير، والإطار المرجعي هو النجوم الساكنة (مثل الشمس).

    تبين أن كل هذا غير صحيح.

    ماذا عليك أن تحفر في شيء تبين أنه غير صحيح، كان هناك الكثير والكثير من المشاكل التي كانت موجودة آنذاك في التصورات وكنت على دراية بها أيضًا وحاولت إيجاد الحل الأكثر منطقية وفقًا للمعرفة الموجودة في ذلك الوقت.

  416. إسرائيل،
    أنت موهوب بما فيه الكفاية لكتابة الكتب المنتجة ذاتيا. ألن يكون من الأفضل أن تتوقف عن الاقتباس وإعطاء التفسيرات السخيفة وتبدأ في فعل شيء مفيد وذو معنى في حياتك؟

  417. اليوبيل.
    للتذكير، يجب أن تخبرني إذا كنت مهتمًا بتعليقاتي بخصوص النموذج.

    كيف كانت الاقتباسات من أيوب؟ كتاب كبير، إيه؟ يدعي مجتمع الكابالا أنه كان مستشارًا لفرعون.

    وكيف أعجبتك الخطبة في قضية بيلك؟ لقد كان مجرد سوء فهم، وأن موشيه ويشوع المسكينين لم يتحدثا لغة موآبية جيدة، ولهذا السبب أرسلوا إلى بيلك رسالة باللغة الإنجليزية. فتح الرسالة، وقرأ السطر الأول: دير بلاك! خذ المظلة وأرسل إليهم بلعام، لكي يتعلموا درسا ولا يصعدوا مرة أخرى إلى العشب مع جميع الشعب وينطحوه مثل الثور.

  418. أشباح
    لقد كان ردك في صلب الموضوع بالفعل، وإذا تعهدت بمواصلة الرد على الأمر، وعدم الانزلاق في الأحاديث الشخصية، وإذا تم استدعاؤك للأمر بذلك بالاعتذار والتوقف، فيمكننا الاستمرار. بناءً على تعليقات مئير وأرييه سيتر.

    وأتعهد أن أفعل نفس الشيء.

    ولكن يجب عليك الالتزام. حرفياً

  419. إسرائيل
    كان ردي في صلب الموضوع.
    لقد كتبت في كثير من الأحيان شيئًا على غرار: "لكن بكل صدق، أعتقد أن هذه مشكلة معي...".
    وأنا أربط مشكلتك هذه بشكل واقعي، وأجيبك بأن المشكلة في الحقيقة معك. ليس في فكرتك فكرتك سخيفة. المشكلة معك وهي أنك لا تفهم أن فكرتك بعيدة المنال.
    وكل المحاولات تهدف إلى مساعدتك على فهم ذلك.

    انت تكتب:
    "وفقا لصورة العالم في عام 1887، فإن الكون لانهائي ومتجانس ومتناحي الخواص. هل يمكن لمثل هذا الكون أن يكون لديه نظام راحة لـ "محيط" الأثير؟..."

    وكلنا نسألك:

    مر. إسرائيل شابيرا، من كريات عينافيم، لماذا لم تكتب: "وفقًا لصورة العالم في عام 2012، الكون هو..."؟
    و"هل يمكن لمثل هذا الكون أن يكون لديه نظام راحة لـ..."؟

  420. ر.ح.

    الرد عليك في انتظار التأكيد.

    الأشباح (أدعوكم بذلك لأنه من الأسهل الكتابة).

    وكما ذكرت عدة مرات، إذا كنت تريد مني الرد على تعليقاتك، عليك أن تتعهد بالبقاء في مجال الأعمال فقط، وعدم الدخول في الأوراق الشخصية.
    لقد تعهدنا أنا ويوفال بالفعل. الأمر متروك لك.

    نقطة.

    لقد تناولنا في الأشهر الماضية هذا المقال وغيره في محاولة لشرح نفس الشيء الغريب الذي أشرت إليه.

    ولكن لفهم ما هو هذا الشيء الغريب، حاول معرفة ما إذا كان بإمكانك الإجابة على السؤال الذي طرحته:

    وفقا لصورة العالم في عام 1887، فإن الكون لانهائي ومتجانس ومتناحي الخواص. هل يمكن لمثل هذا الكون أن يكون لديه نظام راحة لـ "محيط" الأثير؟ ألا ينتهك إيجاد نظام الراحة هذا افتراض التجانس والتناحي؟ لو تم العثور على النظام، لما اضطررنا إلى التساؤل: لماذا هذا النظام بالضبط؟ أليس هذا تشبيهًا لإيجاد مركز خط مستقيم لا نهائي؟

  421. إسرائيل، ما يهم ما هي تجربة ميكلسون مورلي، نوايا التجربة ليست مهمة. المهم هو التجربة نفسها.
    وأظهرت التجربة أن سرعة الضوء في اتجاهين متعامدين هي نفسها. فإما أن الأرض تستقر، أو أن شيئًا غريبًا يحدث. غريب على المستويات التي لا يستطيع فك شفرتها إلا شخص مثل أينشتاين.

  422. مئير.

    Lesage بالنسبة لي هو عرض جانبي. هذا هو مجال يهوذا المختفي.
    من الصعب بعض الشيء التخلي عن العلاقة، أليس كذلك؟ القنابل النووية، الإيدوش، اختبارات الطائرات مع تمديد الوقت، نظام تحديد المواقع العالمي (GPS)، المفاعلات؟

    مشكلتي مع النسبية مختلفة، إذا كان أي شخص مهتمًا فيمكننا إعادة النظر في فكرة التناقض الظاهري بالكامل مع الانفجار الأعظم. لكن بكل صدق، أعتقد أن هذه مشكلة معي وليست في العلاقة.

    أعطني بعض الوقت للبحث في النموذج الخاص بك. على الأقل معك، كل شيء منظم نسبيًا، وليس فوضى مثلنا.

  423. اليوبيل.
    تذكر أن النموذج يتم تحريره وترتيبه في رأسك فقط. نحن، كمعلقين، ليس لدينا طريقة لمعرفة رأيك. لذلك، إذا قمت بتحريره شيئًا فشيئًا، فيجب علينا مراجعة الموضوع بأكمله لمحاولة فهم الاستجابة المحددة التي يرتبط بها موضوع معين.

    على سبيل المثال: أعرف بالضبط أين يمكنني العثور على إجابة السؤال الأخير لـ R.H.، لأنني أتحكم في النموذج الخاص بي. هل تستطيع على الاغلب لا.

    من ناحية أخرى، إذا تم تحرير كل شيء وتنظيمه في مدونة، أو في تعليق، أو في مجموعة معينة من التعليقات، فيمكننا دائمًا الذهاب إلى هناك ومعرفة ما يفترض أن تعنيه الأشياء.

    ما هو مكتوب في تعليقك هو هذا:

    https://www.hayadan.org.il/astronomers-reach-new-frontiers-of-dark-matter-130112/#comment-329304

    رأيت الموقع لأول مرة منذ 3 أيام. لدي شعور بأن الآخرين حصلوا عليه أولاً.

    ر.ح.
    هل سبق لك أن غطست في البحر المفتوح؟
    أنت باللون الأزرق الداكن، على عمق 30 مترًا تحت مستوى سطح البحر، لا توجد أرض ولا سماء، بل محيط فقط. لكل حاجة واهتمام لا حصر له. إذا لم تفعل، تخيل.
    سمكة تسبح وحدها. نسبة إلى ماذا ؟؟؟؟ بالنسبه للمحيط نفسه إذا حددنا نقطة في الماء، لنفترض جزيء ماء مشع، فسنعرف بالضبط حركة السمكة. (حسنًا، لكن سرعة المحيط نفسه هي 0 بالنسبة إلى الأرض. وهي عالية جدًا بالنسبة إلى الشمس أو المذنب العابر).
    الآن أنت غواص. تنظر إلى سمكة تبتعد عنك وتقيس سرعتها.
    تقيس سرعتها بالنسبة إلى البالون العائم في الماء، فتجد أن السمكة تسبح بنفس السرعة التي قستها من قبل. (صحيح. لأن سرعة البالون تساوي 0 بالنسبة للمحيط).
    أنت الآن غواص تسبح بسرعة، وقمت بقياس سرعة السمكة ولدهشتك ما زالت تسبح بنفس السرعة. (غير دقيق. تسبح السمكة بنفس السرعة بالنسبة للمحيط، ولكن ليست بالنسبة لي أنا الغواص. إذا كانت سرعة السمكة 10 م/ث بالنسبة إلى الماء، وسرعتي 1000 م/ث بالنسبة إلى الماء إلى الماء في الاتجاه المعاكس، فإن سرعتي بالنسبة للأسماك هي 1010 م/ث).
    تركب دراجة نارية تحت الماء وتقيس سرعة السمكة وهي لا تزال تسبح بنفس السرعة. (نفس ما ورد أعلاه).
    الآن وضعت كلبك على عوامة تحت الماء مزودة بمقياس سرعة وأنت تسبح بسرعة كبيرة، وقمتما بقياس سرعة السمكة التي اصطدتماها، ولدهشتك في كلا القياسين أنها تسبح بنفس السرعة! (كما هو مذكور أعلاه. الأمر مختلف قليلاً مع كلبي، لأن كلبي سيأكل السمك).
    علاوة على ذلك، ابتعدت السمكة عنك وانتقلت إلى جانب كلبي بنفس السرعة!

    ما هو استنتاجك؟

    1) أن السمكة تسبح دائما بنفس السرعة ولا تتناسب سرعتها مع أي شيء؟ (سرعة السمكة هي دائمًا نفس السرعة بالنسبة إلى المحيط. إذا كان للمحيط نفسه سرعة بالنسبة إلى الشمس، فإن سرعة السمكة بالنسبة إلى الشمس هي المجموع المتجه للسرعتين: سرعة السمكة بالنسبة إلى الشمس) السمك بالنسبة للمحيط + سرعة المحيط بالنسبة للشمس.

    وعلى النقيض من ذلك، فإن سرعة الضوء ستكون هي نفسها دائمًا، بغض النظر عن سرعة السمكة أو المحيط، لأي قياس).

    2) أن جزيئات الماء تتحرك بسرعات مختلفة من 0 إلى ما لا نهاية وتجرفها أسماكنا. في الواقع، يتحرك في جميع السرعات من 0 إلى ما لا نهاية، ولكن لسبب غير معروف فهو شفاف لأجهزة القياس لدينا فوق وتحت السرعة المقاسة، لذلك يبدو لنا أنه في سرعة ثابتة. (لماذا؟ متوسط ​​سرعة جزيئات الماء بالنسبة للأرض هو في حدود عدة كيلومترات في الثانية. ومع ذلك، فإنها تقابل بعضها البعض، بحيث يكون المجموع المتجه لها جميعًا صفرًا بالنسبة إلى المحيط. لدى الأسماك سرعة بالنسبة للمحيط، وهو ناقل آخر. ليس هناك شك هنا في اللانهاية).

    دعونا ننتقل إلى الهواء للحظة. تبلغ سرعة الجزيئات المتوسطة حوالي 480 م/ث بالنسبة إلى الأرض، لكن سرعة الهواء نفسه تبلغ 0 بالنسبة إلى الأرض. إذا كانت هناك رياح، فهذا ناقل آخر. إذا كان هناك طائر يطير، فهذا ناقل آخر.

    لكن ما هي سرعة هواء المحيط نفسه؟ وهو يختلف بالنسبة لكل قياس. فيما يتعلق بالأرض (إذا لم تكن هناك رياح) 0. فيما يتعلق بالطائرة، سرعة الطائرة. وفيما يتعلق بالشمس، سرعة الشمس.

    لكن تجربة MM حاولت إيجاد سرعة "المحيط" الأثيري بالنسبة للأرض. فإذا وجده أين التجانس؟ لماذا هذا واحد؟ كيف هي أفضل من أي سرعة أخرى؟

    الشفافية ليست هلوسة جامحة. وهذا أمر شائع جدًا بالنسبة للجسيمات الأولية السريعة. أ. اليورانيوم 235 ونفاذيته للنيوترونات والنيوترينوات والتاكيونات (والتي، في حالة وجودها، لا يمكن اكتشافها بدقة للسبب الذي تمت مناقشته: أنها أسرع من الضوء).

  424. إسرائيل،
    والخبر السار هو أنني كتبت تعليقا. الرسالة الثانية هي أن الرد في انتظار الموافقة.

    خلاصة القول كانت:

    "باختصار، لا أعتقد أنه من المناسب قياس MCS وفقًا لمعايير شيماء، لكنك على حق :)"

    ليلة راحة من القدس

  425. إسرائيل،

    "إذا قبلت فكرة الكون اللانهائي والمتجانس والمتناحي"
    أنا أقبل فقط فكرة نطاق كبير بما يكفي بحيث لا يتعارض مع إجراء عمليات القياس الفيزيائية. ما الذي يهمني فيما بعد؟

    "نقطة البداية للفكرة بأكملها هي البندول الباليستي - فوق سرعة معينة، أقل بكثير من اللانهاية، تصبح جميع الجزيئات شفافة" لا أتراجع عن مقولة أن الفكرة عبقرية. ومع ذلك، فهو يدعي أنه ولد من أجل تبرير آلية (Le Sage) التي تستهلك الكثير من الموارد وتنتج القليل جدًا. على أي حال، لم أفهم الارتباط بالسؤال الذي طرحته: الاحتكاك الذي كنت أتحدث عنه بين النظام أ والنظام ب هو أمر افتراضي. لقد سألت ببساطة ما الذي يمنع نظامين افتراضيين لا نهائيين من الحركة النسبية؟

    "النقطة الأساسية هي مسألة ما إذا كنت تقبل أو ترفض ما هو مكتوب عن أينشتاين:
    إذا لم تقبل"

    أنا لا أقبل

    "سأحاول الاطلاع على جميع أدلة شيما (بدون روابط، إنه كتاب يرجع تاريخه إلى ما يقرب من 50 عامًا) لمعرفة مدى ملاءمتها لنظريتك."

    أنكم يا إسرائيل ستعتمدون على النظرية النسبية لفحص طفلي؟! لماذا تحتاج إلى موافقة النظريات الأخرى حتى تتمكن من الحكم على نظرية جديدة؟ أقول لك مقدما أنه لا يتناسب مع نظريتي. تعتمد فكرة شيما، من بين أمور أخرى، على تأثير الحركات المستقبلية للجماهير في الكون على ثبات الكتلة هنا والآن. مناسبة للنظرية النسبية والتعامل مع الزمن كبعد يمكن اللعب به. بالنسبة لي: حكايات خرافية.

    إن اختبار النظرية هو مدى مطابقتها للواقع والقياسات وتنوع الظواهر التي تدعي أنها تفسرها. ومن أجل فحص هذا التوافق، لا بد من فحص الواقع من الافتراضات الأساسية للنظرية التي يتم اختبارها، وليس من الافتراضات الأساسية للنظريات التي جاءت لتحل محلها. أدعي أن نظريتي في الجاذبية توفر توافقًا أفضل مع الواقع من النظرية النسبية. ما مدى نجاح النظرية النسبية؟ من الافتراض المريح والخاطئ بأن "المشاهدة أثناء الراحة" هي حقيقة مادية. هذا الافتراض يجعل من الممكن التعامل مع أداة المختبر كمراقب في حالة سكون، والادعاء بأنها تتحرك للأمام في الزمن لأنها تستقر في الفضاء، مقارنة بشعاع ضوئي "يتقدم بأقصى سرعة ممكنة في الفضاء"، وبالتالي يستريح في الوقت المناسب. المشكلة هي أن هذه الحكاية الخيالية المترجمة إلى تحويلات ومعادلات صحيحة من الناحية الإحصائية، وهي محض هراء من الناحية المادية. لذلك، مثل الإحصائيات، تعمل بشكل جيد في معظم الحالات بأعداد كبيرة وليس لمسافات قصيرة جدًا، وتقع على عاتق الفرد.

    كما لاحظت، لقد أعطيت النظرية النسبية الفضل، ولكن فقط مقدار الفضل الذي تستحقه.

    كتبت في مقال عن القصور الذاتي عن نوع التجارب والملاحظات التي يمكنها التمييز بين تنبؤات النسبية وتنبؤات فيزياء MCS عندما يتعلق الأمر بالقصور الذاتي. وقد تم تنفيذ بعضها عمليًا وتم تعريف نتائجها على أنها شاذة وفقًا للنظرية النسبية.

    بالإضافة إلى ذلك، هناك ظاهرة معينة تتنبأ فيها فيزياء MCS بنتيجة معاكسة لتنبؤات النسبية (أي أنه وفقا للنظرية النسبية فإن تغير قيمة تجريبية معينة هو دالة مباشرة للتغير في قيمة أخرى، وفقا بالنسبة لفيزياء MCS، ستتغير هذه القيمة التجريبية كدالة عكسية للقيمة الأخرى)، وهو ما يسمى "اختبار يوم القيامة". تجربة سهلة التنفيذ، وسيتم مناقشة المزيد حولها.

    باختصار، لا أعتقد أنه من المناسب قياس MCS وفقًا لمعايير شيماء، لكنك على حق :)

  426. إسرائيل،

    هل سبق لك أن غطست في البحر المفتوح؟
    أنت باللون الأزرق الداكن، على عمق 30 مترًا تحت مستوى سطح البحر، لا توجد أرض ولا سماء، بل محيط فقط. لكل حاجة واهتمام لا حصر له. إذا لم تفعل، تخيل.
    سمكة تسبح وحدها. نسبة إلى ماذا ؟؟؟؟ بالنسبه للمحيط نفسه إذا حددنا نقطة في الماء، لنفترض جزيء ماء مشع، فسنعرف بالضبط حركة السمكة.
    الآن أنت غواص. تنظر إلى سمكة تبتعد عنك وتقيس سرعتها.
    تقيس سرعتها بالنسبة إلى البالون العائم في الماء، فتجد أن السمكة تسبح بنفس السرعة التي قستها من قبل.
    أنت الآن غواص تسبح بسرعة، وقمت بقياس سرعة السمكة ولدهشتك ما زالت تسبح بنفس السرعة.
    تركب دراجة نارية تحت الماء وتقيس سرعة السمكة وهي لا تزال تسبح بنفس السرعة.
    الآن وضعت كلبك على عوامة تحت الماء مزودة بمقياس سرعة وأنت تسبح بسرعة كبيرة، وقمتما بقياس سرعة السمكة التي اصطدتماها، ولدهشتك في كلا القياسين أنها تسبح بنفس السرعة!
    علاوة على ذلك، ابتعدت السمكة عنك وانتقلت إلى جانب كلبي بنفس السرعة!

    ما هو استنتاجك؟
    1) أن السمكة تسبح دائما بنفس السرعة ولا تتناسب سرعتها مع أي شيء؟
    2) أن جزيئات الماء تتحرك بسرعات مختلفة من 0 إلى ما لا نهاية وتجرفها أسماكنا. في الواقع، يتحرك في جميع السرعات من 0 إلى ما لا نهاية، ولكن لسبب غير معروف فهو شفاف لأجهزة القياس لدينا فوق وتحت السرعة المقاسة، لذلك يبدو لنا أنه في سرعة ثابتة.

    اختر الإجابة الأكثر احتمالا في رأيك.

  427. إسرائيل،
    آمل أن الطريقة التي أقوم بها الآن بإصدار النموذج شيئًا فشيئًا، شاشة واحدة في كل مرة، تساعده على الفهم والتعلم. أفضله على هذا النحو، لأنه يساعدني أيضًا في إعادة صياغة وتصحيح الأشياء التي كتبت منذ عقود مضت. عندما أنتهي من نقل كل شيء وأحصل على ما يكفي من مراجعة النظراء هنا، كنصيحتك المفيدة، سأنشر كل شيء في مقال أنيق في مجلة علمية أو مدونة خاصة.

  428. دعونا نأخذ استراحة قصيرة من البروتونات للحظة ونتحقق مما يحدث للمساحة الفارغة.
    على الرغم من أن الفضاء الفارغ غير دقيق، إلا أنه يمتلك سلوك الجسيمات الدقيقة. عندما يتحرك جسيم إلى قطعة من المساحة الفارغة، فإنه يترك وراءه قطعة من المساحة الفارغة. على الرغم من أن قطعة المساحة الفارغة التي دخلها ليست هي تلك التي تركها وراءه، إلا أنهما مساحات فارغة وبالتالي متشابهتان في خصائصهما. تحدث حركة جسم حاد من النقطة أ إلى النقطة ب بالتزامن مع حركة قطعة من الفضاء الفارغ من المكان ب إلى المكان أ. لكن بينما يتحرك الجسيم الدقيق في مجمله، فإن قطعة الفضاء الفارغة التي يتحرك فيها تصبح أصغر فأصغر أثناء الحركة، وفي الوقت نفسه تكبر قطعة الفضاء الفارغة التي خلفه. وبهذه الطريقة، إذا تحرك جسم حاد جزءًا فقط من الطريق، فإن قطعة المساحة الفارغة قد انقسمت إلى قطعتين صغيرتين. ومن ناحية أخرى، فإن مجموعة القطع الصغيرة من المساحة الفارغة هي في الواقع قطعة واحدة كبيرة. وعلى عكس الجسيم الذي له شكل ثابت وحجم ثابت، فإن المساحة الفارغة ليس لها شكل ثابت وتمتد على مساحة كبيرة. لذلك، في حين أنه يمكن حساب الجسيمات الدقيقة في وحدة الفضاء كعدد صحيح غير سالب وكحجم، إلا أنه لا يمكن قياس المساحة الفارغة إلا بوحدات الحجم. حجم الجسيمات في وحدة الفضاء بالإضافة إلى حجم الفضاء الفارغ في وحدة الفضاء هذه يكون حجمًا ثابتًا. كثافة الجسيمات في وحدة الفضاء هي مقياس لتناثر حجم الفضاء الفارغ في وحدة الفضاء تلك.
    كما ذكرنا في القسم السابق، فإن حركة البروتون أثناء الجاذبية أو القصور الذاتي هي نتاج نسب الكثافة بين ضلعيه الأمامي والخلفي. يمكن أن نرى كما لو أن الفضاء الفارغ يدفع البروتون بينما تجتذبه الجسيمات. وكلما زادت نسبة الكثافة، زادت سرعة حركة البروتون. بشكل حدسي، هناك تلميح سميك إلى حد ما هنا: في حين أن الجسيمات الدقيقة قد يُنظر إليها في وعينا على أنها مسؤولة عن "الكتلة" (المعبر عنها بالجاذبية والقصور الذاتي)، فإن الفضاء الفارغ مسؤول عن "الطاقة"؛ وكما ذكرنا، هذه مجرد وجهة نظر بديهية؛ وسنتناول التعريفات التفصيلية والدقيقة للكتلة والطاقة لاحقًا.

  429. أنا لا أتحمل مسؤولية ما لست مسؤولا عنه. ليس لدي عسر القراءة.

    لقد مررت بجميع رسائل البريد الإلكتروني. فقط الفصل 1 والأنماط الهندسية.
    لا يهم. إذا كنت تريد إلغاء إعفائي، أرسل كل شيء أو انشره على الموقع.

  430. إسرائيل شابيرا، هل تريد المغفرة؟ الحصول على لفة المغفرة!
    ما نقلته للآخرين، بما في ذلك RH، في البريد الإلكتروني هو بالضبط ما نقلته إليك. البعض تجنب القراءة، مثلك، والبعض الآخر تكبد عناء التعمق أكثر. لقد تحدثت عن مستعمرات الجسيمات هنا في أحد تعليقاتي، ولم يشر إليها R.H فقط. بشكل عام، أعتقد أن هناك الكثير من أوجه التشابه بيني وبينك، وعسر القراءة هو أحدها. ما رأيك لو رفعنا دعوى جماعية للمغفرة من يساد هآرتس؟

  431. ر.ح.
    من الواضح أن المدونة فكرة جيدة، بعد اجتياز الغربال الأولي لمراجعة النظراء.

    المحيط. يتمتع المحيط بنظام استراحة واضح - مثل المحيط. ولكن ما هو نوع نظام الراحة الذي يمكن أن يتمتع به المحيط في الكون اللانهائي؟ نسبة إلى ماذا؟

    لنفترض أن نجما يصطدم بنجم يتحرك بالنسبة إليه بسرعة 100 كم/ثانية باتجاه الشمس ويندمج معها. ما سنحصل عليه هو نظام راحة جديد للمحيط الموسع.

    ولكن ما هو النظام الباقي لعدد لا نهائي من المباريات بكل السرعات وفي كل الاتجاهات تتصادم وتندمج؟

    مئير.

    "أشك في لانهاية الأنظمة اللانهائية، ولكننا سوف نتدفق. اشرح لي لماذا لا يمكن لنظام لا نهائي A أن يكون له سرعة مقارنة بنظام لا نهائي B، على افتراض أن كلا النظامين يتكونان من عدد لا نهائي من العناصر المنفصلة. ما هو المنطق الذي قد ينتهك إذا كان كل عنصر في النظام (أ) يواجه احتكاكًا بمعدل ثابت مع عناصر من النظام (ب) تتقدم في اتجاه معين، والعكس صحيح."

    إذا قبلت فكرة الكون اللانهائي والمتجانس والمتناحي، فكيف يمكنك التحدث عن محدودية الأنظمة؟ ألا يتطلب ذلك تفضيل نظام على آخر، أو منطقة في الفضاء على أخرى؟

    نقطة البداية للفكرة بأكملها هي البندول الباليستي - فوق سرعة معينة، أقل بكثير من اللانهاية، تصبح جميع الجزيئات شفافة. وفيما يتعلق بالاحتكاك، فإن نفس منطق الاحتكاك يمكن أن يلتقط الغاز الموجود في خزان أميشرجاز. والسبب في عدم قراءتها هو أن جميع التصادمات مرنة تمامًا. (بالمناسبة، على النقيض من نموذج ليساج الأصلي الذي يتطلب تصادمات غير مرنة وبالتالي احتكاكًا).

    فيما يتعلق بالنموذج الخاص بك:

    النقطة الأساسية هي مسألة قبول أو رفض ما هو مكتوب عن أينشتاين:

    ولاختبار الفكرة، اعتبر أينشتاين جسمًا صغيرًا ساكنًا داخل غلاف دوار من
    المواد في مساحة فارغة خلاف ذلك. وباستخدام نظريته النسبية، حسب ما سيحدث. اتضح أن الجسم يجب أن يشعر بالفعل بقوة الجاذبية المغناطيسية الصغيرة

    إذا لم تقبل، فسأحاول الاطلاع على جميع أدلة شيما (بدون روابط، إنه كتاب يرجع تاريخه إلى ما يقرب من 50 عامًا) لمعرفة مدى ملاءمتها لنظريتك.

    افهم، أنا لا أدعي أن لديك أي خطأ، بل على العكس. كما انسحب أينشتاين من مبدأ ماخ. لم يخطر ببالي أبدًا السبب، وقد تكون نظريتك هي التفسير.

    نقطة أو أسد أو من يقرأ:

    ما رأيك في السؤال التالي :

    جاءت تجربة مايكلسون مورلي لمحاولة العثور على حالة سكون الأثير.

    سؤال:

    وفقا لصورة العالم في عام 1887، فإن الكون لانهائي ومتجانس ومتناحي الخواص. هل يمكن لمثل هذا الكون أن يكون لديه نظام راحة لـ "محيط" الأثير؟ ألا ينتهك إيجاد نظام الراحة هذا افتراض التجانس والتناحي؟ لو تم العثور على النظام، لما اضطررنا إلى التساؤل: لماذا هذا النظام بالضبط؟ أليس هذا تشبيهًا لإيجاد مركز خط مستقيم لا نهائي؟

    إذا أمكن، في إشارة إلى السؤال وليس السؤال.

    يوفال
    "الرد في انتظار التأكيد." ربما لأنني كتبت الوظيفة.

  432. اليوبيل.

    "إسرائيل!
    ماذا حدث لك حتى أنك اقتبست فجأة أيوب؟!"

    كنت أتوقع منك أن تفهم.

    هذا ليس أيوب، هذا هو جواب الله لأيوب. تجدر الإشارة إلى إجابة غير مناسبة للغاية. الرجل الفقير يخدش في التراب، ومات الأطفال، وهربت الزوجة، وتم كباب اللحم البقري، كل ذلك لأن الرئيس تدخل مع S ولم يرغب في الخروج. عندما صرخ أيوب بحق، "لقد دفعت الأرض ليد الأشرار!" ويطلب تفسيرًا، يكشف الله له نفسه من العاصفة، ويجعله فاحصًا مفاجئًا في خلق العالم، كما لو أن أيوب المسكين تظاهر يومًا بمعرفة كيفية خلق العوالم وإدارتها.

    ذكرني قليلاً بجميع الأسئلة المطروحة عليك حول النموذج الخاص بك.

    "يا إسرائيل، على حرقك المتأخر:
    وفيما يتعلق بمسألة تأسيس الدولة، فقد ترددت هنا كثيرًا واصطدمت بجدار من الصبر، كنت أنت أيضًا أحد طبقاته. الأمور ليست معقدة وتتطلب فهمًا أساسيًا وبسيطًا للغاية لمنطق الحياة اليومية، ومع ذلك فقد تجاهلت ذلك. النقطة السوداء التي وضعتها في مذكراتي بجانب اسمك يرجع إلى أنني لاحظت أنك تميل إلى تجاهل الأشياء التي لا تتوافق مع جدول أعمالك. ربما أنت لا تفهم، أو ربما لا تقرأ على الإطلاق. إذا حاولت أن تفهم ولم تنجح، سأرسل لك فاكسًا وأتحمل اللوم على نفسي."

    تحمل المسؤولية كان لدي ادعاء أساسي واحد ضدك طوال الوقت: أنني لا أعرف ما هو نموذجك على الإطلاق. لقد قمت بإعادة توجيهها في رسالة بريد إلكتروني خاصة إلى الآخرين، ولكن ليس لي. لقد ظللت تسألني أسئلة حول النموذج، وأظل أخبرك أنني لا أستطيع الإجابة عليك إذا كنت لا أعرف ما هو.

    كيف أعرف أنك حولته إلى R.H. مثلا وليس أنا؟ ر.ح. ارجع إلى مستعمرات جزيئاتك التي ذكّرته بالمستعمرات البكتيرية. لم أتلق أي شيء عن هذه المستعمرات يا م.س.ل.

    باختصار، يوفلي محرم عليك. لقد اتهمتني زورا بإهمالك الإجرامي. يجب أن تسقط على وجهك، وتطلب مني المغفرة، وتدعني أكتشف الجوانب الرحيمة في طبيعتي.

  433. إسرائيل، لحرقتك المتأخرة:
    وفيما يتعلق بمسألة تأسيس الدولة، فقد ترددت هنا كثيرًا واصطدمت بجدار من الصبر، كنت أنت أيضًا أحد طبقاته. الأمور ليست معقدة وتتطلب فهمًا أساسيًا وبسيطًا للغاية لمنطق الحياة اليومية، ومع ذلك فقد تجاهلت ذلك. النقطة السوداء التي وضعتها في مذكراتي بجانب اسمك يرجع إلى أنني لاحظت أنك تميل إلى تجاهل الأشياء التي لا تتوافق مع جدول أعمالك. ربما أنت لا تفهم، أو ربما لا تقرأ على الإطلاق. إذا حاولت أن تفهم ولم تنجح، سأرسل لك فاكسًا وأتحمل اللوم على نفسي.

  434. نقطة،
    على الرغم من أنني أقدم أيضًا حسابات كمية، إلا أن أي شخص يتوقع أن يجد معادلات معقدة معي سيصاب بخيبة أمل. أنا لا أخترع رياضيات جديدة ولكني أستخدم رياضيات أساسية جدًا. لم أهتم بالمعادلات التفاضلية، ولكن إذا أردت، أعتقد أنه يمكنك العثور في الفيزياء على ما يكفي من المواد الخام لهذه المعادلات أيضًا. يدعي النموذج الذي أقدمه هنا أنه يبني العديد من النظريات الأساسية للفيزياء من مواد خام بسيطة وقليلة. إذا كنت تتوقع أن تجد التعصب والحصار في داخلي، فيرجى قبول اعتذاري العميق والصادق مقدمًا.

  435. إسرائيل،

    أشك في لانهاية الأنظمة اللانهائية، ولكننا سوف نتدفق. اشرح لي لماذا لا يمكن لنظام لا نهائي A أن يكون له سرعة مقارنة بنظام لا نهائي B، على افتراض أن كلا النظامين يتكونان من عدد لا نهائي من العناصر المنفصلة. ما هو المنطق الذي قد ينكسر إذا تعرض كل عنصر في النظام "أ" للاحتكاك بمعدل ثابت مع عناصر من النظام "ب" تتقدم في اتجاه معين، والعكس صحيح؟

    نظرت إلى الرابط. شكرا. لم أكن أعرف ريموند على وجه التحديد، ولكنني التقيت بالعديد من أمثاله وهم يتلمسون طريقهم في الظلام بحثًا عن الشيء الصحيح. ويمكن القول أنه بعيد عن الحقيقة مثل بعد الشرق عن الشمال الغربي.

    ومن حسن الحظ أنني تحررت من أمراض الطفولة التي يعاني منها، لأنني على عكسه، تمكنت من تأسيس نظرية كاملة لجاذبية الجسيمات قبل أن يتضح لي أنني بدون قصد ودون تغيير أي شيء، حصلت على القصور الذاتي كمكافأة، بينما هو (وهو ليس الوحيد في هذه الحالة) يهتدي بحدس (صحيح) مفاده أن مصدر القصور الذاتي هو في الجاذبية الذاتية ويحاول استخدامه لتطوير طريقة لفهم الجاذبية.

    المفتاح موجود في الهندسة الموضحة في الرسم الذي أحضرته أعلاه. أعلم أنها على حق لأنه من المستحيل أن أحصل على كل الأشياء التي حصلت عليها من خلالها كمكافأة إذا كانت مخطئة. وأولئك الذين لا يضعون هذا المخطط في أذهانهم سوف يستمرون في البحث عن الإبرة في كومة القش. لاحظ أنه بدون هذا الرسم، بدون هذه الهندسة الخاصة، عندما تقوم بإزاحة جسيم فإنك تحركه بعيدًا عن مركز المجال الذاتي، وكلما حركته أبعد، أصبح من الأسهل عليك "تحليله" أسفل الحقل . يدرك ريموند أن هذا لا ينتج القصور الذاتي (وأنا أفترض أن هذا هو السبب في أن ماخ نفسه لم يأخذ في الاعتبار إمكانية الجاذبية الذاتية)، لذلك اخترع أن دفع الجسيم يضغط المجال. لو كان أمام عينيه الهندسة الصحيحة، ولو فهم أن الجسيم يعمل في دورات، لأدرك أنه لا يوجد "اتصال" بين الجسيم والمجال. لا يستطيع الجسيم "ضغط" الحقل، لكنه يمكنه تجديده في ظل ظروف فتح جديدة.

    الفرق هو أولاً وقبل كل شيء بين هندسة الكرة التي تقع على قمة الجبل (ريموند وآخرون)، وهندسة الحلقة التي تقع في منتصف جانب الجبل، وتحيط به من جميع الجوانب ( مئير عميرام).

  436. عزيزتي إسرائيل،

    ومن قال لك أن تصمت أو تصمت؟ لكن يوجد طريق. لو سألتني سأقول:
    "بما أن هناك مشاكل في الفيزياء اليوم حيث لا تتناسب إحدى النظريات مع نظرية أخرى، فمن الواضح أنه يجب البحث عن تفسير آخر (وهذا تافه ومعروف). أقترح البدء من بديهيات مختلفة عن تلك المقبولة وتفسيرًا مختلفًا لنتائج تجربة MM بناءً على احتمال عدم تمكن D-E-T-Y من العثور على نظام راحة لموقع في عالم لا نهائي ومتجانس. إذا قبلنا هذه الافتراضات، فسوف أرى أن المشاكل أ، ب، وحتى ج قد تم حلها ويتم الحصول على التوراة كاملة دون تناقضات. من أجل تأكيد نظريتي وتفسيري لنتائج MM، أقترح تجربة نقوم فيها بإجراء A. B. C. إذا أظهرت نتائجه X، فسيتم استعادة نظريتي ولن نضطر إلى العودة إلى المكتب."

    لكن في الحقيقة الأمر يتعلق فقط بالصياغة والأسلوب. ما أعرفه هو أنني إذا تلقيت مقالًا من Revio Sheiktol، فلنفترض أن تجربة Selson-Stahl (التي أظهرت تكاثر الحمض النووي شبه المحافظ) "غير منطقية" سأكون معارضًا لها على الفور وسأكون حتماً رأيًا غير جيد من كاتب المقال.

    وعن مدونة؟ لقد كان عرضًا وديًا وبناءً، لا تريده فلا تقبله.

    وفي واقع الأمر، فإنك تستمر في تكرار الشعار "دعونا نقول أنهم عثروا عليها". لنفترض أنه يتحرك بالنسبة إلى النظام الشمسي بسرعة 345 كم/ثانية باتجاه كوكبة الأسد." ألا تفهم أن نظام الراحة لا يتحرك بالنسبة للأشياء الموجودة فيه، بل هم بالنسبة له؟ إنه مشابه للأسماك في المحيط حيث تأتي وتدعي أن المحيط بأكمله يتحرك بالنسبة لسمكة نيمو، لكن ماذا عن والد نيمو الذي يتحرك في الاتجاه المعاكس تمامًا؟ لا. جميع الأسماك تتحرك نسبة إلى المحيط.

  437. يوفال، لديك معادلات. لأنه يمكن لأي شخص أن يروي قصصًا عن الجزيئات الدقيقة والجنيات الجيدة.
    المعادلات التفاضلية هناك؟

  438. نقطة شكرا لاهتمامكم
    لقد ركزت حتى الآن على جسيمات النصب. لكن المساحة الفارغة بينهما تلعب أيضًا دورًا مهمًا في الفيزياء. سأتوقف الآن لضيق الوقت. سأعود في فترة ما بعد الظهر وأشرح.

    RH وإسرائيل، يبدو لي أننا نتقاسم أكثر من مصلحة مشتركة 🙂

  439. ر.ح.
    لنفترض أنك ترغب في شرح فكرتي، دون غطرسة، دون غطرسة، في الواقع، إن أمكن، بموضوعية تامة. هل يمكن أن تجد بعض الصياغة التي تغفل النقطة الأساسية، وهي بطلان محاولة M-M دون أن تبدو متعالية ومتعالية؟

    إذن ما هو البديل - ناهيك عن الشيء الرئيسي؟

    إليك الصياغة الدقيقة التي اخترتها:

    "لذلك لو قيل أن ميكلسون تمكن من العثور على نظام السكون للأثير، وقيل أنه كان يتحرك بالنسبة إلينا بسرعة 1887 كم/ث باتجاه كوكبة الأسد، لكان علينا أن نسأل : لماذا هذا بالضبط؟ ما هو المميز عنها؟ أين التجانس؟

    إنه مثل العثور على النقطة المركزية للكون اللانهائي. لماذا هذا واحد؟

    وهنا أعرف أنني يجب أن أكون مخطئا. لا، من المستحيل أن أفكر في الأمر ولم يفعل لورنز ذلك. صحيح أنه تم قبولي في جامعة هارفارد، ولكن للحصول على شهادة جامعية فقط. لقد تم قبولي كمنظفة حمامات في جامعة هارفارد، ولكن فقط للطلاب الجامعيين. لقد استأجروا شخصًا أكثر ملاءمة لمنصب الموظفين.

    لذا فإن حقيقة أنني لا أعرف أن ماكسويل ولورينز قد أثارا هذا التحفظ، توضح لي منطقيًا أنني مخطئ.

    لكن حتى أعرف أين الخطأ، يجب أن أحاول حل اللغز، حتى أتمكن من الموت سعيدًا."

    قرأته ولا أشعر أن هناك أي غطرسة هنا. فقط شكل من أشكال التعبير النموذجي لمؤلف الفقرة.

    الى حد، الى درجة.

    لقد كنت مقتنعًا تمامًا أنه بعد أن أثرت هذه النقطة، ربما يقوم شخص ما، ربما أنت، أو ربما آرييه، بتوجيهي بسرعة كبيرة إلى رابط يوضح لي مكان الخطأ.

    لأنه من الواضح (على الأقل بالنسبة لي) أن هناك احتمالين فقط: إما أن لدي خطأ - أو لا.

    إذا لم أكن مخطئًا، وكانت تجربة MM غير منطقية حقًا - فلا مفر من الاستنتاج بأن جميع الآخرين كانوا مخطئين، وليس مجرد خطأ، خطأ فادح.

    وهو أمر غير مقبول بكل بساطة.

    وفي المقابل، لا مفر أيضاً من الاستنتاج بأن القلة التي قرأت الرد حتى الآن، وأغلبهم بالتأكيد أذكياء ومتحكمون في المادة، لم ينبهوا إلى الخطأ. ومن هذا أستنتج أنني لست الأحمق الوحيد الذي لا يرى ما هو واضح.

    ماذا تنصح أن تفعل في مثل هذه الحالة؟ هل تصمت فقط خوفًا من الظهور بمظهر المتغطرس؟ أعلم جيدًا أن الصورة أهم بكثير بالنسبة لي من مكاني فيها. ليس هناك خيار، سأستمر في طرح نفس السؤال، وسأختطف كما هو الحال دائما.

    لدي الكثير لأقوله، عن الرفض العرضي، في رأيي، لنظرية ماكسويل، وعن التناقض الذي قد يكون موجودًا بين النظريتين النسبية والانفجار الأعظم، لكننا اتفقنا بالفعل على أننا انتهينا.

    سمعت عن عنوان في بلعام. حول "دير بلاك!" هل سمعت

    ويوفال، إذا كنت تقرأ، رغم أنك أعطيتني إعفاءً: هل من الممكن وفقًا لنموذجك معرفة وقت ولادة يائيلة إيلا؟ ومن أنجب قطرات الندى؟ وبشكل عام أين كنت في باسدي إيرتس؟ قل لو كنت تعرفها!

  440. إسرائيل،
    أعتذر إذا كنت قد أساءت. معاذ الله أن أنضم إلى معسكر كارهي إسرائيل. كما أنني لا أفهم سبب غضبك، أردت انتقاد العارضة، قرأت ما كتبت وحاولت أن أفهم وأثارت تحفظات وتساؤلات، أليس هذا هو القصد؟

    النقطة السوداء بالنسبة لي في الأقسام التي ذكرتها هي 3، وما زلت أعتقد أن القول في مثل هذه الأحكام بأن تجربة مايكلسون مورلي غير منطقية هو غطرسة. ليس لدي مشكلة مع النظريات الثورية والمبتكرة، فأنا أوافق على أن العصور الوسطى نتجت عن تثبيت عقلي عميق للكنيسة الكاثوليكية بشكل رئيسي وأن التفكير خارج الصندوق هو ما يتقدم العالم.
    لكن، وهذا أمر كبير، لكنك لا تبدأ نظرية بـ "المحاولات السابقة التي تدرسونها جميعًا وتقتبسونها لسنوات هي هراء (أي غير منطقية)". لاحظ أن هذا ليس انتقادًا للنموذج أو تجربتك المخطط لها على الإطلاق، بل هو انتقاد للمنهج.
    ولكن إذا كنت منزعجًا من مثل هذه التعليقات، فأنا أعدك بأن أكون عمليًا بحتًا من الآن فصاعدًا.
    بالمناسبة، إجابة لطيفة على سؤالك "بالمناسبة، ألا تعتقد أن القسم 3 الخاص بي ذو صلة أكثر من أي شيء آخر؟ وإذا كان الأمر كذلك، أليس من الغريب أن جميع القراء الأذكياء الذين يتقنون المادة (بما فيهم أنت) الذين قرأوا سؤالي لم يجيبوا عليه بعد؟"
    بالفعل في عام 1905 أعطاك واحدًا اسمه أ. استند أينشتاين على هذه التجربة غير المنطقية إلى نظرية كاملة مفادها أن الناس هبطوا على القمر وأنك تجد طريقك عبر شوارع لوس أنجلوس حتى لو كنت في أحيائها غير المألوفة. إذا ماذا تريد مني؟

    بالمناسبة، إذا كنت مهتمًا ببلعام بالفعل، فهل سمعت بعنوان بلعام؟

  441. أشباح
    عندما يكون لديك شيء ذو صلة لتكتبه لي، بدلًا من الهوس بشأن هويتي وما أنا عليه، سيكون من الممكن التحدث معك.
    حتى ذلك الحين، قيشتا.

  442. إسرائيل
    مع كامل احترامي، أنت لست أستاذا في الفيزياء النووية، وحتى لو كنت كذلك، فأنت لست بمستوى العلماء الذين يمكنهم مناقشة الموضوع.
    ومع ذلك، أنت في الواقع تحاول التظاهر بهذه الطريقة.
    "إنك تضرب حصاناً ميتاً" كما يقولون في بلدك.

  443. مئير.
    خذ طريقًا لا نهائيًا. هل يمكنك معرفة ما هي نقطة المنتصف؟ لماذا هذا واحد؟
    خذ الكون اللانهائي. نفس ما ورد أعلاه.
    لنأخذ على سبيل المثال كونًا لا نهائيًا ومتجانسًا ومتناحيًا. لنفترض أن لديها نظام راحة كاملاً كما حاول ميشيلسون ومورلي العثور عليه. لنفترض أنهم وجدوها حتى. لنفترض أنه يتحرك بالنسبة للنظام الشمسي بسرعة 345 كم/ثانية باتجاه كوكبة الأسد.
    السؤال: لماذا هذا؟ أين التجانس؟
    هذه هي القصة كلها.

    قبل أن نعود إلى القصور الذاتي، ربما يمكنك إلقاء نظرة على الرابط

    http://www.reocities.com/perfectfluid/

    شكرا.

  444. إسرائيل،

    هل يمكنك توضيح المزيد (لصالح شخص غير ذكي يحاول المتابعة) لماذا لا يسمح الكون اللانهائي والمتجانس بوجود نظام راحة في الموقع، ولو محليًا فقط؟

    على العكس من ذلك: ففي نهاية المطاف، إحدى المشاكل التي حاول ميلر التغلب عليها في التجارب المتكررة هي سحب الموقع "ربما" مع النظام المحلي. أي أنه افترض إمكانية تحديد موقع نظام داخل نظام.

  445. ر.ح. صديقي
    دعونا نأخذ كلماتك واحدة تلو الأخرى ونرى ما إذا كانت ستصمد أمام اختبار الواقع.

    1. "يوفال (ويهودا إسرائيل وكل من يفكر في النماذج ويريد قراءتها والتعليق عليها)"

    لا أستطيع أن أتحدث باسم الآخرين ولكن فقط لنفسي. من خلال قراءة الموضوع في هذه المقالة، يبدو في الواقع أنك أنت من نقر لإخبارك بفكرتي وليس العكس. انظر هذا التعليق وما بعده:

    https://www.hayadan.org.il/astronomers-reach-new-frontiers-of-dark-matter-130112/#comment-324601

    لم أحاول أبدًا "تسويق" أي نظرية. على العكس تماما. لقد أصررت بشدة على أنه إذا كنت مهتمًا بالفكرة، فيجب عليك الزحف إلى بعض الروابط المتعلقة بالموضوع وقراءتها، حتى أنني تحدثت عن "التجارة" في الأفكار (مثل لقاح الأنفلونزا).

    إن طرح الأمر وكأنني "ميت" حتى تتم قراءة أفكاري والتعليق عليها، هو خطيئة ضد الحقيقة وضد الإحساس بالعدالة. صحيح أنني أريد أن أخضع أفكاري للنقد، ولكن فقط لأولئك المهتمين بها والذين لديهم معرفة معقولة بالمادة. لقد أثبتت كلا الأمرين، من خلال طلب مناقشة الفكرة صراحة، ومن خلال قراءة النص الموجود في الرابط الخاص بماكسويل (ص 136، تذكر؟).

    يجب أن أعترف أنك فاجأتني بحدة إدراكك للتفاصيل، كما أنني شكرتك من قبل على تناولك الفكرة بالتفصيل. ولكن كان اختيارك، من فضلك لا تسقط الحقائب.

    2. "لماذا لا تقومون بإنشاء مدونات مثل مئير عميرام أو تلك الفتاة وهناك سوف تشرحون مشناه الخاص بكم بطريقة منظمة وواضحة وليس من خلال مشاركات مختلطة ومربكة هي رد على مقال تم نسيانه منذ فترة طويلة موقع هيدان؟"

    لقد شرحت لك في وقت سابق. لا فائدة من إنشاء مدونة إذا لم تكن متأكدًا من صحة ما تريد قوله على الإطلاق. فمن الأفضل أن نسأل أولا، لمنع خطأ في العداء. هذا ما فعلته.

    ولو قيل أننا كنا سنأخذ مقالاً مختلفاً، قل مقالاً مهملاً لنحماني منذ 5 سنوات وعلقنا عليه، هل كان سيحدث فرقاً؟ من يلزمك، أو نقطة، أو أي شخص آخر غير مهتم حتى بإلقاء نظرة سريعة على هذه المقالات؟ ألا تشير حقيقة أن هذه المقالة وحدها التي تلقت عددًا من الردود مثل جميع المقالات الأخرى الموجودة على الموقع طوال عمرها، إلى أن هناك اهتمامًا كبيرًا بهذه المواضيع؟

    3. "فيما يتعلق بـ 2، 3، كما ذكرنا، فمن السخافة أن نذكر، مرة أخرى، إذا قرر المجتمع العلمي بأكمله "فجأة"، كما تقول، أنه لا يوجد موقع وأن تجربة M-M مقبولة منطقيًا، إذن ربما يجب عليك التحقق من افتراضاتك المتعجرفة مرة أخرى؟ وهذا ليس هجوما شخصيا، في رأيي هو غطرسة وجرأة في التشكيك في كل ما هو مقبول اليوم في الفيزياء الحالية التي يتم بحثها واختبارها بلا كلل من قبل أفضل العقول في العالم دون أي نتائج تجريبية أو ذرة من الأدلة. علاوة على ذلك، أن نأتي ونقول إن جميع علماء الفيزياء المتعلمين، دون سخرية، لا يرون أن تجربة MM غير منطقية؟؟؟
    أنت تعرف النكتة القديمة عن ديفيد ليفي الذي اتصلت به زوجته وقالت له: "كن حذرًا، قالوا في الأخبار أن هناك سائقًا يقود سيارته عكس اتجاه حركة المرور على الطريق رقم 1". "أيها؟" سأل "الآلاف!"

    غطرسة؟ وجه جريء؟ من خلال طرح سؤال والقول مقدمًا أنه من المحتمل أن يكون لدي خطأ ولكن أريد أن أعرف مكانه؟ في الواقع كنت أتوقع إجابة على غرار: انظر إسرائيل، هنا رابط لهذا وذاك، السؤال الذي طرحته قد تم التحقيق فيه بالفعل وهنا هو الجواب. انظر ديفيد ليفي، الدائرة الحمراء ذات الخط الأبيض تعني "ممنوع الدخول".

    ناهيك عن التناقض الداخلي مع كلامك:

    "بالإضافة إلى ذلك، ليس من الواضح ما هي حجتك بشأن ماكسويل ولورينز. فقط لأنهم لم يفكروا في الأمر أولاً يعني أنه غير صحيح؟ وبسبب طريقة التفكير هذه، ظل العالم عالقًا في العصور الوسطى لمئات السنين لأنه "إذا لم يقل أرسطو ذلك أولاً، فمن المحتمل أن يكون هذا غير صحيح". اخجل لمدة دقيقة واستمر على الفور في خط تفكيرك. معظم الاكتشافات والاختراعات هي تلك التي تقول فيما بعد "واو، كيف لم يفكروا في هذا من قبل؟؟" حقيقة. لم أفكر في ذلك من قبل! "

    لاحظ أيضًا أن تجربة جاليليو الفكرية بشأن تساوي سرعة السقوط لجميع الأجسام هي تجربة فكرية بسيطة وواضحة لا مثيل لها، ومن المدهش حقًا أن التفكير فيها استغرق ألفي عام.

    باختصار، RH، كما هو الحال دائمًا، أدعو إلى انتقاد أفكاري. لكن بالنسبة لسؤال إسرائيل: هل تجربة M-M مقبولة منطقيا، هل يمكن أن يكون هناك نظام راحة لموقع في كون لا نهائي ومتجانس، تماما كما يمكن أن تكون هناك نقطة مركزية لللانهاية؟ أونا ر.ه: لماذا من أنت على أية حال، وما هي قصتك بالضبط؟ أنا مجبر على أن أختتم بكل أسف أنك انتقلت، دون أي سبب منطقي، إلى معسكر كارهي إسرائيل.

    بالمناسبة، ألا تعتقد أن القسم 3 الخاص بي ذو صلة أكثر من أي شيء آخر؟ وإذا كان الأمر كذلك، أليس من الغريب أن جميع القراء الأذكياء والمطلعين (بما فيهم أنت) الذين قرأوا سؤالي لم يجيبوا عليه بعد؟

    هيا يا بلعام، استدر وارجع إلى كونك R.H. الجيد

    ومن يعرف ما هو السبب الحقيقي الذي دفع بالاق إلى إرسال بلعام ليلعن إسرائيل؟

  446. لقد كتبت "معظم الأشياء التي قمت بتفصيلها تجد تفسيراً بداخلي". لذلك اعتقدت أن الشرح موجود وكان عليك فقط نشره.

  447. مرحبًا يوفال،

    مثير! استمر في ذلك، وسنرى أين ستنتهي.
    ولا تلتفتوا إلى كل المتشككين والمترددين، فليكن
    يعطيك العافية وتحية لك على مجهودك وشجاعتك.

    ؟؟؟؟

  448. نقطة،
    نعم. نظرية كل شيء. البديهية الأساسية هي "الوجود مخلوق من لا شيء". في مرحلة معينة من تطورها توصلت إلى نظرية جسيمات المادة المظلمة والتي تسير على النحو التالي:
    أ) يتكون العالم من عدد متزايد باستمرار من الجسيمات الدقيقة وكمية لا حصر لها من الفضاء غير المبالي (أو الجسيمات غير الدقيقة).
    ب) مع وجود مساحة كافية غير مبالية، فإن الجسيمات الدقيقة تنقع محيطها لتكوين جسيمات دقيقة مثلها. إن الجسيم الحاد المحاط بجزيئات حادة ذات كثافة عالية جداً يغير حالته إلى جسيم غير حاد (يصبح قطعة من الفضاء الفارغ، أو "ميتاً").
    ج) يحتل جسيم كروي واحد حجمًا محددًا في الفضاء، ولا يشغل أي جزء منه في نفس الوقت جسيم كروي آخر.
    د) يمتلك الجسيم الملاحظ حركة ذاتية مستقلة تختلف بشكل عشوائي في اتجاهه وسرعته (باستثناء قيد واحد: لا تحدث حركة الجسيم الملاحظ داخل الحجم الذي يشغله جسيم ملاحظ آخر).

  449. اليوبيل,

    "معظم الظواهر التي طرحتها يتم شرحها وحسابها وقياسها بواسطة النماذج الفيزيائية الموجودة. يدعي نموذجي أنه يبني نفس النماذج المادية."
    - هذه ليست روح كلامك في آخر التعليقات. لدي انطباع بأن نموذجك يمكن أن يفسر الظواهر التي تفسرها فيزياء الكم. إذا كنت تدعي أن الظواهر موضحة، فما الذي تحاول تفسيره؟ كما أنني لم أفهم ما هو النموذج الذي يبني النموذج.

    "كتمرين، يمكنك الوصول إلى الظواهر المختلفة مباشرة من النموذج الخاص بي."
    - من فضلك تبين لنا.

  450. يوفال، لم أتابع المحادثة بأكملها هنا، ولكن من جميع المراسلات أفهم أنك تحاول القيام بشيء مثير للاهتمام.

    صححني إذا كنت مخطئًا ولكنك تحاول بناء بعض النظريات... هل تحاول بناء نظرية لكل شيء؟ أم أنني في حيرة من أمري؟

    وإذا كان الأمر كذلك، ما هي الافتراضات أو البديهيات الأساسية التي تبني عليها أفكارك؟

  451. التخنيون,
    معظم الظواهر التي جلبتها تم شرحها وحسابها وتحديد كميتها من خلال النماذج الفيزيائية الموجودة. يدعي نموذجي أنه يبني نفس النماذج المادية. كتمرين، يمكنك الوصول إلى الظواهر المختلفة مباشرة من النموذج الخاص بي. ولكن، قياسًا على علوم الكمبيوتر، فإن الأمر يشبه كتابة تعليمات برمجية معقدة باستخدام لغة الآلة فقط.

  452. اليوبيل,

    من الممكن ملء عدة رفوف في المكتبة بالموضوعات التي كتبتها لك حتى الآن. لقد قلت أن لديك تفسيرًا لمعظم ما قمت بتفصيله - وأحب أن أرى هذه التوضيحات.

  453. إسرائيل،
    قصدت بالمقالة المنسية أن المناقشة التي تطورت لا تتعلق بالمقالة (ربما بطريقة غير مباشرة) وقد اتخذت بالفعل العديد من المنعطفات الخاصة بها. ولا يمكن متابعته وهو غير منظم وترابطي.
    إذا كتبت مقالًا فقط عن نموذجك وتوسعت في النقاط التي كتبتها أعلاه وقام يوفال بوصف نموذجه ويهودا ومئير كل على حدة، فستكون المناقشة أكثر تركيزًا ووضوحًا.
    هذه سلطة كاملة.

    فيما يتعلق ببنودك ليس لدينا أي جدال حول 4.
    أما بالنسبة لـ 2، 3، كما ذكرنا، فمن السخافة أن نذكر، ومع ذلك، مرة أخرى، إذا قرر المجتمع العلمي بأكمله "فجأة"، كما تقول، أنه لا يوجد موقع وأن تجربة M-M مقبولة منطقيًا، فربما يجب عليك التحقق من الافتراضات المتعجرفة الخاصة بك مرة أخرى؟ وهذا ليس هجوما شخصيا، في رأيي هو غطرسة وجرأة في التشكيك في كل ما هو مقبول اليوم في الفيزياء الحالية التي يتم بحثها واختبارها بلا كلل من قبل أفضل العقول في العالم دون أي نتائج تجريبية أو ذرة من الأدلة. علاوة على ذلك، أن نأتي ونقول إن جميع علماء الفيزياء المتعلمين، دون سخرية، لا يرون أن تجربة MM غير منطقية؟؟؟
    أنت تعرف النكتة القديمة عن ديفيد ليفي الذي اتصلت به زوجته وقالت له: "كن حذرًا، قالوا في الأخبار أن هناك سائقًا يقود سيارته عكس اتجاه حركة المرور على الطريق رقم 1". "أيها؟" سأل "الآلاف!"

  454. تابع أيها الجمود:
    وكما ذكرنا، إذا كانت كثافة البيئة على أحد جانبي البروتون أقل من كثافة البيئة على الجانب الآخر، فإن البروتون سيتحرك نحو المكان الأكثر كثافة. وبما أن البروتون أكثر كثافة من محيطه، فإنه ينتج جسيمات أكثر من تلك التي ينتجها محيطه. وتتناثر الجسيمات التي ينتجها البروتون في أي اتجاه دون تفضيل، وبالتالي يتم الحفاظ على نسبة كثافة البيئة بين جانبي البروتون ومعها ميل البروتون لمواصلة حركته.

  455. إسرائيل،

    ليس من الواضح بالنسبة لي ما هو المثير في الحصول على قانون نيوتن الثاني من صيغة سرعة الإفلات. خذ قانون التربيع العكسي وأوجد أن F=m*g وأن g=M*G/R^2
    ترتبط جميع قوانين نيوتن للحركة ببعضها البعض وبقانون التربيع العكسي.

    اقسم طرفي المعادلة التي قدمتها على مربع نصف القطر وستحصل على تسارع في كلا الطرفين. اضرب في كتلة الجسيم الأولي وستحصل على القوة التي تؤثر بها الكتلة الكونية على الجسيم الأولي الموجود على حافة الكون. هل تؤثر عليه بهذه القوة المعطاة لأنه الكتلة الكونية، أم لأنه جسيم أولي له الكتلة المعطاة؟ هل سيتغير أي شيء في هذه المعادلة إذا كانت الكتلة الكونية عُشر ما هي عليه (أو جزء من مائة، أو جزء من المليون)؟

    ليس هناك ما يثير الدهشة في أن تكون هناك علاقة بين حدود الكون المرئي وسرعة الضوء، وليس هناك ما يثير الدهشة في أن الكتلة الموجودة داخل حدود الكون المرئي هي الكتلة الكونية الكلية للكون المرئي. هذه مجرد طرق مختلفة لقول نفس الشيء.

    إذًا ما الذي يثير الدهشة في أن الكتلة الكونية تؤثر بقوة معينة على جسيم له كتلة معينة على مسافة معينة؟ كيف يرتبط هذا بخاصية القصور الذاتي للجسيم أكثر من الكتلة الذاتية لهذا الجسيم؟

    כל עוד לא נמצא בכתובים אחרת, דבר אחד ברור: לא מאך ולא דניס שימה הביאו בחשבון את הפתרון של מאיר עמירם בטרם טרחו להמציא אינטראקציות מאגיות כדי להסביר תופעה שניתן להסביר אותה באמצעות חוק היפוך הריבועים של ניוטון ועוד לקבל מתוך זה כבדרך אגב את שלושת חוקי התנועה له. شيء محرج قليلا، ولكن قليلا من الشيء.

  456. ر.ح.

    "يوفال (وكذلك يهودا إسرائيل وكل من يفكر في النماذج ويريد أن يقرأها ويعلق عليها)"
    ألا تعتقد أنه من الأفضل قبل البدء بالمدونة مراجعة الأشخاص المفكرين الآخرين إذا لم يكن هناك خطأ جوهري في الفكرة التي يحاول الشخص تقديمها؟

    أعتقد ذلك بالتأكيد. وهذا ما فعلته هنا. لاحظ أنني قدمت عدة أقسام للمراجعة:

    1. كانت نظرية الأثير لماكسويل تعتبر وهمية إلى حد ما، لولا الصيغ الشهيرة وتجربة هيرتز.
    2. بما أنه تم إثباته فمن المستحيل تجاهله والقرار فجأة بعدم وجود موقع.
    3. تجربة MM غير مقبولة منطقيا. لا يمكن أن يكون هناك نظام راحة لموقع ما في كون لا نهائي ومتجانس، تمامًا كما لا يمكن أن تكون هناك نقطة مركزية لللانهاية.
    4. يكفي أن يتبين أن نيوترينو واحد فقط تمكن من تجاوز سرعة الضوء لينهار النسبية.
    5. على النقيض من التشابك الكمي، حيث لا مفر من الاستنتاج بأن معلومات الدوران تمر بشكل فوري، في حالة الضوء هناك على الأقل احتمال نظري بأن سرعة الضوء هي نفسها في جميع الأنظمة المرجعية، وهذا دون تمديد الوقت في الأنظمة غير المتسارعة.

    حتى الآن لم أتلق إجابة.

    وإليك كلامك فيما يتعلق بالقسم 3، قلب الحجة:

    "تعليق واحد فقط حول فقرتك 3. لقد ادعيت في الماضي أنك تنحني لماكسويل لورينز وآخرين. لقد زعمت أيضًا أن نظرية LS الغريبة، إن لم نقل بعيدة المنال، حقيقية لأن الكثير من الناس، بما في ذلك نيوتن، فهموا أن هناك نوعًا من الجاذبية هنا ولم يبق سوى مشكلة الاحتكاك. ولكن من ناحية أخرى، أتيت إلى هنا وقتلت تجربة MM، التي تعتبر التجربة رقم واحد على الإطلاق في الفيزياء (حسنًا، على الأقل في المراكز العشرة الأولى) وقد اعتمدها المجتمع العلمي الفيزيائي بأكمله وقبلها، وبعد ذلك هي، النظرية النسبية. من الجنون الحكم على الموضوع، لكن لدي شعور بأنه من غير المرجح أن يكون جميع الفيزيائيين في القرن العشرين أغبياء ولم يروا أنها كانت تجربة غير منطقية كما تدعي".

    وجوابي: "وهنا أعرف أنني لا بد أن أكون قد ارتكبت خطأ. لا، من المستحيل أن أفكر في الأمر ولورينز لم يفعل ذلك."

    وهذا هو بالضبط سبب تقديم الفكرة هنا. لمعرفة ما إذا كان يمكن لأي شخص أن يشير إلى خطأ، أو بعض التفسير، أو رابط لمقالتي غير المعروفة.

    بالمناسبة، إذا لم أفعل ذلك بشكل صريح حتى الآن، شكرًا لك على استثمارك ومراجعتك.

    ويبدو لي أنك مخطئ بعض الشيء بشأن نسيان المقال. هذه واحدة من أكثر المقالات قراءةً وترويجًا في تاريخ الموقع.

  457. اليوبيل,

    "ST: القائمة طويلة. أغلب الأشياء التي تفصيلتها تجد في تفسيرها. في الوقت الحالي، أنا أتعمق في التحدي الذي طرحه عليّ Y.S فيما يتعلق باللا محلية في التشابك الكمي."

    أعتقد أن أعضاء اللجنة سيكونون سعداء برؤية هذه التفسيرات (الأولى هي حساب طول الرابطة الكيميائية لحمض الهيدروكلوريك من الطيف الذي قمت بربطه).

  458. يوفال (وكذلك يهودا إسرائيل وكل من يفكر في النماذج ويريد أن يقرأها ويعلق عليها)،

    لماذا لا تقومون بإنشاء مدونات مثل مئير عميرام أو تلك الفتاة وهناك ستشرحون تعاليمكم بشكل منظم وواضح وليس فوق منشورات مختلطة ومربكة هي رد على مقال طال نسيانه على الصفحة موقع هيدان ؟
    يمكنك مواصلة المناقشة هنا، ولكن بعد ذلك يمكنك إرسال الروابط وعدم الإشارة إلى تعليقاتك المجهولة والمنسية من الماضي.
    القليل من العمل، نعم، لكنها دفعت ثمنه.

  459. اليوبيل,
    إذا تمكنت بالفعل من الإجابة على تحديات طالب التخنيون وبعض التحديات الشريرة الأخرى التي يمكن التفكير فيها بسهولة وأيضًا تقديم تفسير لظواهر غير واضحة اليوم مثل التشابك والمادة المظلمة وما إلى ذلك، فسيكتسب نموذجك زخمًا. لكن ماذا أقول لك؟ هل تعلم أن.

    بالنجاح

  460. إلى جميع المعلقين،
    ST: القائمة طويلة. أغلب الأشياء التي تفصيلتها تجد في تفسيرها. أنا حاليًا أتعمق في التحدي الذي يطرحه عليّ Y.S والذي يتعلق باللا محلية في التشابك الكمي.
    نعم: ربما ستشعر بالارتياح لأن كل شيء يمر مع الحياة.
    RR: أنت في مجموعة جيدة وواسعة من الأشخاص الذين لا يركزون على تفسيراتي. كما ذكرنا، فقد أعفتك من التعريفات الأساسية (التي تبدأ بـ "لا شيء") وعرضت عليك قبول التعريفات الحالية كبديهية. وأيضًا، على الرغم من أنك دقيق تمامًا في التعريفات الفيزيائية التي تقدمها، إلا أنني لا أعمل في الفيزياء، لذا فإن تعريفاتي مختلفة. في النهاية، يجب أن تتطابق إعداداتي مع الإعدادات الفعلية. وبالنسبة لأسئلتك الأخيرة: الفوتون له حالتان، موجة وجسيم؛ مثل الموجة، قطرها كبير جدًا؛ كجسيم صغير. "المساحة الفارغة" هي الحجم الذي لا يشغله جسيم ثابت.

  461. طالب علم
    وبحلول الوقت الذي ينتهي فيه من الاختبار، سيكونون قادرين على قياس الفوتونات في أندروميدا.

    يوفال
    في الرياضيات - تستخدم الحروف لترمز إلى شيء ما. "a" كعضو على سبيل المثال، "i" كرقم وهمي، "E" كمفهوم ما. مجرد مثال.
    في الرياضيات هناك دلالات مختلفة عما هي موجودة في الأدب مثلا، ومن المؤسف أنك لا تفرق بينهما.
    "ما يحدد تردد الفوتون" هو النسبة بين سرعة الطور وطول موجته.
    وليس "القطر (المتوسط) للجسيم" (على الرغم من أنه يعمل بشكل جيد مع جزيئات الأردواز :)).
    بالمناسبة، ما هو قطر الفوتون، هل تعلم؟
    لقد كتبت أيضًا "الإلكترون عبارة عن طبقة رقيقة من الفضاء الفارغ".
    مرة أخرى، ما هو "الفضاء الفارغ"؟
    كيف يمكنك تعريف "المساحة الفارغة"؟

  462. يوفال
    آسف، اعتقدت أنك كنت تتحدث معي في اليومين الماضيين، لكنني الآن لاحظت حقًا أن الأمر كان مع الأشباح. لقد شعرت بالارتباك، ربما بسبب مرض الزهايمر.

    وعلى أية حال فإن الإعفاء هو إعفاء. إذا كنت تحتاج لي، حدد. على الرغم من أنه كما أدركتما منذ وقت طويل، فأنا لا أفهم حقًا ما أتحدث عنه، لذلك لا توجد خسارة كبيرة هنا.

    إسرائيل الغبية غبية.

  463. وبعض المواضيع الأخرى التي تستحق التحقق منها إذا كان النموذج الخاص بك يشرح:
    1. الجدول الدوري.
    2. كيف يعمل الرنين المغناطيسي النووي (NMR) والرنين المغناطيسي الإلكتروني (ESR)؟
    3. كيف يحدث تكثيف بوز أينشتاين ولماذا لا تخضع له الفرميونات؟
    4. لماذا يتكون جزيء الهيدروجين من ذرتين وليس 20؟
    5. كيف تعمل الامتيازات والرهونات البحرية؟
    6. نثر رامان.

  464. شيء آخر يستحق التحقق: بمساعدة النموذج الخاص بك، هل يمكنك حساب (ولو بشكل تقريبي) ما هو تردد الضوء الذي يمتصه، على سبيل المثال، جزيء البنزين.

  465. التخنيون,
    ربما. لم أتحقق بعد. ما يحدد تردد الفوتون هو القطر (المتوسط) للجسيم، ومتوسط ​​الكثافة المحلية، ومتوسط ​​التردد الاهتزازي، وما إلى ذلك. يمكن حساب هذه الأحجام بناءً على الملاحظات الفردية والتحقق مما إذا كانت مناسبة أيضًا لمجموعة واسعة.

  466. إسرائيل!
    هل نسيت أنك معفى من الرجوع إلى النموذج الخاص بي؟
    المشكلة التي أواجهها معك هي أنك تندفع للأمام دون فهم الأساسيات. أنا لا أقرأ ببطء فحسب. أنا أيضا أكتب ببطء. إذا لم تتحلى بالصبر فلن نتمكن من التحدث.

  467. R.H. Rafai.M،
    الحروف هي رموز لكل شيء.
    سأدرس فيزياء الكم قريبًا، إذا ساعدني. لكنني سأثبت شيئًا بالفعل:
    الإلكترون عبارة عن طبقة دقيقه من المساحة الفارغة المحيطة بالبروتون من جميع أعضائه. كما ذكرنا، الفوتون هو قطعة من الفضاء الفارغ ذات حجم معين (والذي يختلف حسب عدة عوامل). عندما يضرب الفوتون إلكترونًا فإنه يساهم في زيادة حجمه. والإلكترون الذي يزداد حجمه "يسعى" إلى الحفاظ على سمكه الرقيق وبالتالي يزيد قطره. إذا كان القطر الجديد مناسبًا تمامًا لقشرة رقيقة تمامًا، فإن الإلكترون يستقر عند المستوى الجديد. إذا لم يكن الأمر كذلك، فإنه يطلق فوتونًا مرة أخرى. نقيس الفوتونات حسب ترددها (اللون) ونجد أن هناك علاقة بينها وبين شيء نسميه "الطاقة". ولكن ليس عن ذلك الآن.

  468. يوفال
    أخيراً!

    تنشأ عدة أسئلة من النموذج.

    1. أليس شرطاً أنه لكي تنشأ الجاذبية ألا تبدأ الجسيمات الموجودة على الجانب الآخر من المستعمرة بالموت أيضاً، وبالتالي يتم خلق توازن يمنع المستعمرة - البروتون - من الموت. من التحرك في اتجاه معين؟

    2. ما أسباب تلك الوفاة؟ كيف يمكن أن يؤثر بروتون على القمر على بروتون على الأرض؟

    3. هل كان من المفترض أن أخمن كل هذا؟

  469. يوفال

    يتم التعبير عن الرياضيات بالرموز وليس بالحروف.

    دعونا نجرب طريقة أخرى.
    أخبرنا كيف تتناسب فيزياء الكم مع نموذجك.
    بمعنى، في أي مرحلة، ما هو العامل/العوامل التي تربط نموذجك بالفيزياء على المستوى الكمي؟

  470. وتابع نموذج يوفال. ومنذ ذلك الحين، أصبح كل شيء يتعلق بالرياضيات، ومعظمها الهندسة والاحتمالات.
    بروتون، بروتونان، الجاذبية:
    نظرًا لأن الجسيمات الدقيقة للمادة المظلمة (المشار إليها فيما يلي بـ "الجسيمات") تتكاثر في ظل كثافة محلية منخفضة وتموت عندما تكون الكثافة المحلية أكبر من عتبة معينة، فإن الكثافة التي يتم ترتيبها فيها تتأرجح في نطاق معين حول حجم ثابت. سوف نسمي هذا الحجم "متوسط". بافتراض أن الجسيم يشغل حجمًا محدودًا، عندما يموت، "يحرر" الجسيم جزءًا ("الكم") من المساحة الفارغة بنفس الحجم. ويتناقص متوسط ​​الكثافة دفعة واحدة تبعا للحجم الذي يحرره الجسيم عند موته. وعند هذه الكثافة المنخفضة تزداد فرصة ولادة جسيم جديد ولا قدر الله. في أي لحظة هناك مناطق تكون فيها الكثافة أعلى من المتوسط ​​ومناطق تكون فيها الكثافة أقل من المتوسط.
    سننظر الآن إلى مجموعة من الجسيمات ذات كثافة أعلى من المتوسط. يمكن تشبيه هذه المجموعة بالمستعمرة (مثل الإنسان أو البكتيريا)، لذلك سنسميها كذلك. يتم تنظيم الكثافة القصوى للمستعمرة من خلال موت الجزيئات ولا يمكن أن تزيد عن حجم معين؛ ومن ناحية أخرى، بعيدًا عن المركز الهندسي للمستعمرة، تكون كثافة الجسيمات هي متوسط ​​الكثافة. تترك الجزيئات المستعمرة إلى البيئة وتنضم إليها جزيئات من البيئة ويتم الحفاظ على التوازن الديناميكي. على مسافة معينة حول مركز المستعمرة يوجد غلاف حيث متوسط ​​عدد الأشخاص المغادرين لكل وحدة زمنية يساوي عدد الأشخاص المنضمين. سوف نحدد هذا الظرف بأنه "حدود المستعمرة".
    إذا كانت المستعمرة متناظرة، أي أن متوسط ​​الكثافة المحلية هو نفسه في كل نقطة حول غلافها، فإن التوازن الديناميكي سيؤدي إلى عدم تغيير المستعمرة لموقعها على الرغم من أنها تتبادل الجزيئات مع محيطها. ومع ذلك، إذا كان متوسط ​​الكثافة المحلية في إحدى مناطقها أعلى منه في منطقة أخرى، فإن المنطقة الكثيفة ستنضم إليها جزيئات أكثر من المنطقة المتناثرة، وفي المنطقة المتناثرة ستتركها جزيئات أكثر من المنطقة الكثيفة. وهكذا، أثناء تبادل الجزيئات مع البيئة، ستنتقل المستعمرة إلى المنطقة ذات الكثافة الأعلى.
    إذا كانت هناك مستعمرتان بينهما مسافة محددة، فإن المنطقة الواقعة بين المستعمرات ستكون أكثر كثافة من المناطق الأخرى. وبسبب هذا ستتحرك المستعمرات تجاه بعضها البعض وسيحدث تجاذب بينها. وسيسمى هذا الجذب أدناه "الجاذبية"، وستسمى المستعمرات "البروتونات".

  471. مثال الحلقة غير ناجح، لأن تأثير تحريض القوة المغناطيسية على الجسم الجالس في مركز الحلقة لا يتحقق بسبب دوران الحلقة ولكن بسبب المجال الكهربائي المتغير والذي يمكن أيضًا أن يكون يتم الحصول عليها في الجزء الثابت للمحركات والتي، كما يوحي اسمها، ثابتة.
    من حقك أن تستخلص استنتاجات من السطر الأخير فقط من الظواهر، لكن هذا سيمنحك على الأكثر جائزة نوبل للآداب.

  472. اليوبيل.
    ليس لدي أي فكرة متى ستفهم ما أعنيه. أستطيع أن أشرح بأفضل ما أستطيع، لكن ليس لدي سيطرة على ما يفهمه الناس من كلماتي.

    أعرف فقط أنه عندما لا أفهم شيئًا ما، أحاول القراءة مرة أخرى، أو أطلب التوضيح. ولا أشير على الفور إلى أن الطرف الآخر "يصيغ كلامه بطريقة غير ناجحة"، أو "يُدخل الجنس في غير الجنس".

    في رأيي المثال مع الخاتم ممتاز، ويوضح الفكرة الرئيسية للعلاقة بين الجاذبية والقصور الذاتي. وكما نعلم فإن القوة الكهرومغناطيسية أقوى بكثير من قوة الجاذبية. ولذلك فإن هذا المثال هو توضيح "لضغط" المادة لزيادة قوة الجاذبية كما شرحت في
    https://www.hayadan.org.il/astronomers-reach-new-frontiers-of-dark-matter-130112/#comment-329007

    من وجهة نظر الجسم، لا فرق بين أن تكون القوة المؤثرة عليه قوة جاذبية أو كهربائية أو مغناطيسية. لذلك، إذا كان الجسم المشحون كهربائيًا يدور بسبب دوران الحلقة، فمن الطبيعي أن نستنتج أن الجسم سوف يدور بسبب دوران "إطار" ثقيل جدًا. وهذا أيضًا ما يدعيه ألبرت.

    وفيما يتعلق بالنموذج الخاص بك. لقد قلت بالفعل عدة مرات أنني لا أستطيع التعليق عليه لأنني لا أعرف ما هو. إذا، كما تزعم، فإن الصعوبات الإدراكية التي أواجهها هي وحدها التي تقف في طريق فهم كيفية عمل الجاذبية وعدم المحلية في نموذجك، بينما يفهم الآخرون جيدًا بالفعل، ربما يكون من المفيد لهم أن يشرحوا لي، وبعد ذلك سأقوم بذلك. يمكن أن تتصل.

    في غضون ذلك، وإلى أن تتمكن من الرد بشكل واقعي على العبارات المتعلقة بمبدأ ماخ، يجب أن أتفق مع أينشتاين (أ.ي.)، وماخ (أ.ي.) وشيما (أ.ي.).

  473. إسرائيل! ماذا يكون؟ متى سأفهم قصدك؟
    القوى هي قوى، لكن ما يبدو لي هنا ليس القوى بل المصدر الذي تنشأ منه. في رأيي، العلاقة بين الشحنة الكهربائية وشحنة الجاذبية ليست سوى علاقة غير مباشرة. يمكنك النقر والقول أن كلاهما يتمتعان بعدد من الخصائص المتشابهة، ولكن هذا لا يشير بالضرورة إلى أصل مشترك. [حسب نموذجي، التشابه ينشأ من أن الأوساط التي تمر عبرها (وسط الجزيئات الدقيقة ووسط قطع الفضاء الفارغ) متكاملة داخل بعضها البعض، لكنك معفى من الإشارة إليها] .

  474. "وبمناسبة وجودي هنا بالفعل، أردت أيضًا أن أشير إلى أن الشحنة الكهربائية ليست شحنة جاذبية."
    ماذا حقا؟ كيف لم يخبروني؟ وما هو الاتصال؟ ألسنا نتحدث عن القوى هنا؟

  475. إسرائيل! أنت مبدع للغاية. شبو إن قدرتك على إشراك الجنس في غير الجنس تفوق قدرتي بكثير.
    وبمناسبة وجودي هنا بالفعل، أردت أيضًا أن أشير إلى أن الشحنة الكهربائية ليست شحنة جاذبية.

    التعليق السابق في انتظار الموافقة. انتظر

  476. إسرائيل! أنت مبدع للغاية. شبو درجة عبادتك الشيطانية أعلى بعدة مرات من درجة عبادتي.
    وبمناسبة وجودي هنا بالفعل، أردت أيضًا أن أشير إلى أن الشحنة الكهربائية ليست شحنة جاذبية.

  477. حسنًا ، لم أشعر بالإهانة.

    أنا في الواقع أتطلع لسماع Ifka. ولكن إذا كان ذلك ممكنا، بعض التفسيرات والأمثلة والصيغ والروابط (ليس للقطط).

    من الصعب بعض الشيء بالنسبة لي قبول حجة على غرار: Mach - Yoke. بديهي.

    ألا تظن أننا إذا رتبنا حلقة مشحونة بشحنة كهربائية، فإذا وضعنا جسماً مشحوناً في المركز سيتأثر بدوران الحلقة؟ وبعيداً عن مسألة الإشعاع، ما هو الفرق فعلياً بين القوة التي تؤثر بها الحلقة على الجسم والقوة التي تؤثر بها كتل الكون على كتلة دوارة في المركز، فقط على عكس الأدوار؟

  478. رافد Dyslect.
    مشكلة كبيرة، عسر القراءة. ماذا سأقول؟ أيضًا عسر القراءة، وأيضًا فقدان الذاكرة، وأيضًا اضطراب نقص الانتباه (ADD)، وأيضًا مرض باركنسون.
    لقد تقدمنا ​​في السن مع اليوبيلات، وسنبلغ قريبًا الثلاثين عامًا.
    مرة أخرى اختلط علي الأمر، بسبب الخرف؟
    حقيقي حقيقي. 20.

    على أي حال. لقد أثيرت صيغة أينشتاين لأنك كتبت: "هل يمكن ليوئيل شابيرا أن يشرح للأحمق ولأمثالي ما يعنيه الشاعر عندما قال: "إن القصور الذاتي يرتبط بكل الكتل في الكون من خلال علاقة صيغية خطية بسيطة"؟"
    وهي مثال على هذه العلاقة. هذا كل شيء. لا علاقة لها بقصصنا. مجرد مثال. مثال على علاقة خطية بسيطة. مثال. صله. خطي. فقط. مثال.

    ما هو المرتبط وكيف تكون الصيغة GM=RC^2. إنها ليست حتى خطية، إنها مدببة أكثر، مثل نقطتنا. النظامي فقط. ولكن هناك شيء رائع جدًا في هذه الصيغة. إذا قمت بإجراء تحليل أبعاد لها - ZA فسوف تقوم بتقليل كل الأمتار والثواني والكيلوجرامات والأرقام الموجودة على طرفي المعادلة، وسوف ينتهي بك الأمر بالنيوتن فقط على الجانب الأيسر وحاصل ضرب الكيلوجرامات بالمتر مقسومًا على الثواني المربعة .

    أو باختصار:

    و = ماجستير

    قانون نيوتن الثاني، قانون القصور الذاتي.

    هذه ليست حالة. (على الرغم من أنه من الممكن بالطبع تغيير الحدود في المعادلة والحصول على علاقة مختلفة على ما يبدو). والسبب في ذلك هو الطريقة التي وصلنا بها إلى المعادلة.

    الطريقة موصوفة جزئيًا في الرابط الذي أرفقته بمقالة البروفيسور وودوارد. في ذلك الوقت، أحالني إلى كتاب لدينيس شيما (بريطاني من أصل يهودي سوري، أحد طلاب ديراك، كامبريدج، أحد آباء علم الكونيات الحديث، لا بأس، أعلم أنك لست اسمًا مقطوعًا). . كان شيما من أتباع مبدأ ماخ كثيرًا، وفي الواقع بالنسبة له على الأقل، أثبت إلى حد كبير العلاقة الوثيقة بين الكتلة في الكون والقصور الذاتي وفقًا لمبدأ ماخ. ومن هنا الصيغ لدينا.

    وإذا كانت الصيغ، فمن الصعب بعض الشيء أن يجادل. وخاصة مثل هذه الصيغة التي جناحها الأيسر هو كتلة الكون، وفي تحليلها الأبعاد نحصل على قانون نيوتن الثاني، وتطويرها يعتمد على مبدأ ماخ.

    هل هذا يعني شيئا؟ ليس بالضرورة. ولكن يبدو معقولا بالنسبة لي.

    وربما الأهم من ذلك هو أنك عدت إلى كونك يوفال اللطيف. خشيت أن تفتح فمك في وجهي، فخرجت مباركًا، مثل بلعام في زمانه.

    هذا. هذه قصتي كلها.

  479. إسرائيل!
    ما زلت غير متأكد من أنني أفهم. بدا لي أنك كنت تبحث عن علاقة ذات صيغة خطية بسيطة بين القصور الذاتي لجسم واحد وجميع الكتل في الكون، ولكنك الآن وضعت E=MC^2 في المعادلة.
    كنت سأترك الدعوى القضائية معك لمئير. لا أعلم ماذا حدث لي حتى أدخل أنفي في نقاش ليس لي. أرجوك أن تغفر وتغفر.

  480. إسرائيل!

    إذا نسيت، دعني أذكرك بأنني مصاب بعُسر القراءة. فقط إذا شرحوا لي ببطء، أتمكن من الفهم بسرعة. لتوضيح مدى يأس وضعي، سأطلب منك أن تصدقني أن جملة "القصور الذاتي يرتبط بكل الكتل في الكون في علاقة صيغة خطية بسيطة" لا تتجاوز عتبة فهمي. وكما ذكرنا، كان رد فعلي الفوري هو أن أقول لك "لا". ولكني خشيت أن أخطئ في حقك، لأنه ربما كان في سؤالك شيء ذكي أخطأت فيه بسبب محدوديتي المعرفية الخاصة. فقط من فرط الخجل والكبرياء فضلت تقديم الأمور وكأن العيب فيك، لكن الحقيقة أنك تكتب ببلاغة ووضوح.

    ملحوظة: مازلت لم أفهم ما الذي تقصده بالضبط.

  481. مئير.
    انتبه إلى ما هو مكتوب فوق ردك، في ردي:

    ام:
    G = ثابت الجاذبية.
    ج = سرعة الضوء.
    M = الكتلة المقدرة للكون. (يمكن العثور عليها في ويكيبيديا).
    R = نصف القطر المقدر للكون. (نفس ما ورد أعلاه).

    إذن: GM=RC^2 تقريبًا.

    مذهل أليس كذلك؟ وما لا يقل إثارة للدهشة هو أننا إذا استخدمنا التحليل البعدي، بعد كل التخفيضات على جانبي المنارة، لم يبق لنا سوى:

    و = ماجستير

    قانون نيوتن الثاني، قانون القصور الذاتي.

    تم اشتقاق الصيغة GM=RC^2 من تطوير البروفيسور وودوارد ودينيس شيما. يتم تطويرها من خلال التكامل على أساس مبدأ ماخ. يمكنك العثور على بعض منه في

    http://physics.fullerton.edu/~jimw/kill-time/

    أفهم أنك تعتقد أن كتلة الكون أقل بأربع مرات من الكتلة المقبولة. لكن الحقيقة ذاتها، وهي أن التكامل الذي يتم على أساس مبدأ ماخ، يؤدي إلى حساب الكتلة المقبولة للكون، تظهر في رأيي أنه من المستحيل تجاهل مبدأ ماخ.

    هل تعرف نظرية ريموند جروديس؟

    بالمناسبة، هل فكرت يومًا أنه سيكون من المثير للاهتمام أن نتساءل كيف يمكن أن تكون صيغة العلاقة بين نصف القطر والكتلة هي نفسها بالنسبة للثقب الأسود والكون؟ بالتأكيد لا يبدو أننا نعيش في ثقب أسود!

    يجب أن أذهب، سنتناقش عندما أعود.

  482. إسرائيل،

    كتبت أعلاه أنني أشك في أهمية هذه العلاقة. أعني أنه لا يبدو لي أنه من الممكن أن نستخرج منه نوعاً من المعلومات التي تفتقر إليها البشرية.

    لتلخيص ذلك في جملة واحدة، يؤكد هذا الارتباط أن سرعة الهروب من الكون تساوي سرعة الضوء تقريبًا.

    لكن حقيقة أن الكون هو نوع من الثقب الأسود، تعرفه البشرية بالفعل منذ فجر شبابها :)

  483. يوفال، مئير.
    وكالعادة «الرد في انتظار الموافقة». كيف لم أفاجأ؟

    حتى نهاية التحقيق، آخذ كلبي للنزهة. وفي هذه الأثناء، إذا كان ذلك ممكنا، والنظر في

    https://www.hayadan.org.il/astronomers-reach-new-frontiers-of-dark-matter-130112/#comment-326922

    وأخبرني ما معنى الصيغة التي تظهر هناك. (مئير، هذه مدونة شركتك).

  484. ر.ح.

    يجب عليك الانتظار حتى نهاية الاختبارات.
    في غضون ذلك، راجع تعليقًا منذ أسبوع على:

    https://www.hayadan.org.il/astronomers-reach-new-frontiers-of-dark-matter-130112/#comment-327662

    "وفقًا لهذا الاقتباس، فمن الممكن أن تكون مشكلة الأوبرا في المحور بشكل عام هي المعدات النهائية، التي تتكون من مكونات المعدات (الجسيمات الأولية على سبيل المثال) التي تتحرك بسرعة عالية في اتجاه حركة النيوترينو".

    وهذا يشمل بالتأكيد المذبذبات.

    اليوبيل.

    أنت مدعو للنظام.

    لقد التزمت بما يلي:

    https://www.hayadan.org.il/astronomers-reach-new-frontiers-of-dark-matter-130112/#comment-328506

    ليس بغرض إيذاء المشاركين شخصيًا عن قصد، ولكن كرد فعل على ضرر شخصي لك.

    ما يخالف كلامك:
    "إنه يصوغ كلماته بطريقة غير ناجحة بحيث يصعب علي أن أفهم ما يريد قوله بالضبط."

    تحمل المسؤولية حقيقة أنك واجهت صعوبة في الفهم، لا تعني أن الأمور تمت صياغتها بطريقة غير ناجحة. يبدو لي أن مئير فهم على الفور.

    كيف يمكنك صياغة العلاقة بين الكتلة والطاقة في صيغة أينشتاين؟ هل يمكنك التفكير في صياغة أفضل من "علاقة الصيغة الخطية البسيطة"؟

    إذا كان الأمر كذلك، فهذه هي الفرصة. تمت صياغته.
    ؟؟؟؟

  485. صباح الخير لوس أنجلوس، مساء الخير للمدينة المقدسة،
    مئير! من الخصر أقول أنه من نموذجي يمكنك استنتاج نموذجك. ولذلك، فإن ردي على إسرائيل، في ظاهر الأمر، هو "لا" بشكل لا لبس فيه. لكنه يصوغ كلماته بطريقة غير ناجحة بحيث يصعب علي أن أفهم ما يريد قوله بالضبط. فهل من الممكن أن يشرح يوئيل شابيرا للأحمق وأمثالي ما قصده الشاعر بقوله "إن القصور الذاتي مرتبط بكل كتل الكون بعلاقة صيغية خطية بسيطة"؟

  486. إسرائيل،

    لقد سألت يوفال وفقا لنموذج يوفال. لست على دراية بنموذجه حتى أتمكن من الإجابة وفقًا له.

    رأيي الشخصي واضح. لا توجد علاقة، لا خطية ولا غير خطية، بين المحتوى المادي للكون والقصور الذاتي.

  487. إسرائيل،

    "1... يمكننا معرفة السرعة الزاوية التي تدور بها الأقراص، حتى في غرفة مغلقة. ويظل السؤال: التدوير بالنسبة إلى ماذا؟ يقول أينشتاين - نسبة إلى الفضاء الزمني المطلق. ومع ذلك، إذا فهمت ادعاءك بشكل صحيح، فإن القصور الذاتي ينشأ من حركة الجسم من مركز ثقله. إذا كان الأمر كذلك، فكيف يمكننا أن نقول أن القرص A يدور بسرعة 5 دورات في الثانية والقرص B 7؟ نسبة إلى ماذا؟"

    لنأخذ جميع الجسيمات الأولية على طول خط نصف قطر وهمي للقرص، والذي أدعي أن سرعة دورانه مطلقة، ونقيس لكل جسيم درجة عدم تناسق مجال الجاذبية الذاتية بالنسبة إلى النقطة المركزية للجسيم . وفي القرص غير الدوار، لن نجد مثل هذا عدم التماثل في أي من الجسيمات الموجودة على طول الخط. إذا أردنا، يمكننا استخدامه كمرجع لجميع الأقراص الأخرى المحيطة به. في القرص الدوار سنجد أن عدم التماثل يزداد كلما ابتعدت عن المركز على طول نصف القطر.
    كلما زاد عدم التماثل على مسافة معينة، زادت سرعة دوران القرص.

    يتضمن الشرح أعلاه أيضًا ردًا على تعليق يوفال.

    الدوران المطلق بالنسبة إلى ماذا؟ نسبة إلى الفضاء. ليست نسبة إلى الزمكان المطلق (لا يوجد مثل هذا الحيوان)، بل نسبة إلى الحالة السابقة للمكان المتغير (والتي يمكن التعامل معها على أنها مطلقة في أي لحظة، حتى التغيير المنفصل التالي الذي يحدث فيه، والله حرم).

    "2. لنأخذ المثال (الممتاز) ليوفال على إطار به دلو من الماء في المنتصف. صحيح أنه عندما يكون الإطار في حالة سكون، فإن قوة الجاذبية داخل المنطقة المحيطة به تساوي 0. لكنني أعتقد أن الأمر مختلف عندما يدور الإطار بالنسبة للدلو. إذا تمكنا من ضغط مادة إلى تركيز 1000 طن لكل متر مكعب، وقمنا بتجميع الإطار من هذه المادة، فإن هذا الإطار الدوار الذي يبلغ نصف قطره الداخلي 2 متر ونصف القطر الخارجي 2.5، سيزن عدة ملايين من الأطنان لذلك إذا وضعنا الدلو سيظهر أن الماء يدور ويكتسح. سيحدث نفس الشيء إذا بقي الإطار في حالة سكون ودار الدلو."

    غير صحيح. كل جزيء لنفسه. ويسحب كل جسيم معه مجال جاذبية ذاتية له عدم تناسق في اتجاه الحركة، وهو عدم تناسق يتناسب عند السرعات التقليدية مع سرعته الخطية. لا يوجد أي تفاعل بين الماء الموجود في الدلو والإطار. ادعى ألبرت أن هناك سحبًا للإطار. لا يوجد مثل هذا الحيوان. الجسيمات الأولية تعرف فقط نفسها وما يحدها ماديًا. بتجاهل ظاهرة تمدد الفضاء (من أجل المناقشة، وبما أن تأثيرها على البيئة المباشرة للجسيم الأولي هو صفر)، فإن البيئة المباشرة التي يعيشها الجسيم الأولي تكون دائما ثابتة. فالجسيم الأولي لا يعرف ولا يمكنه أن يعرف أن شيئًا يبعد عشرة سنتيمترات (أو ميكرومتر) يتحرك. الشيء الوحيد الذي "يعرفه" هو المعلومات الثابتة التي يكتسبها من البيئة المباشرة في لحظة معينة من دورة عمله. لكي يتمكن إلكترون معين E في دلو الماء من "معرفة" أن العجلة المضغوطة تدور، من الضروري أن يؤدي مجموع مساهمات جميع الجسيمات الأولية التي تشكل العجلة المضغوطة إلى خلق البيئة المباشرة لـ أن يكون الإلكترون E غير متماثل في اتجاه الدوران. لن يكون هناك مثل هذا عدم التماثل لأن مجموع كل مساهمات عدم التماثل المحلية في مجالات الجاذبية الذاتية لجزيئات العجلة أثناء حركتها هو صفر بالنسبة إلى موضع الإلكترون E وبالنسبة إلى موضع كل جسيم أولي آخر يشكل الماء.

  488. من سيقوم بالتجربة أنا؟ من أنا، ما أنا، من فضلك أنا قادم، من فضلك اذهب.

    إنه أينشتاين نائم مرة أخرى بسبب هراءه. أنت بحاجة إلى التحدث إلى Punkut وأبي، الذي سيتصل به لحل المشكلة.

    لا بد لي من الذهاب إلى السرير قريبا. لكن لدي سؤال بسيط ومباشر حول القصور الذاتي، وسأكون سعيدًا لو تمكنت من الحصول على إجابة بسيطة ومباشرة منك فيما يتعلق بصورتك النموذجية:

    1. هل يبدو لك أن القصور الذاتي يرتبط بكل الكتل الموجودة في الكون في علاقة صيغية خطية بسيطة؟

    2. إذا كانت الإجابة على 1 إيجابية، أليس هذا بالضبط مبدأ ماخ؟

    سأكون سعيدًا إذا تمكنت من الحصول على إجابة بنعم أو لا.

  489. إسرائيل،
    جميل. لقد وجدت لنفسك تجربة بسيطة للتعديل. يرجى متابعته. ضع كاميرا على القرص الدوار، ومن المؤكد أنك ستحصل على تأكيد آخر للنظرية النسبية. أطلب قبعة منخفضة السعرات الحرارية.
    وكما تعلمون فإن البحث عن العملة المفقودة تحت الفانوس عملية أسهل بكثير من البحث عنها في المكان الذي فقدت فيه بالفعل.

    لكنني أتفق معك في شيء واحد: إن النقاش حول أصل القصور الذاتي لا يزال مفتوحًا.

  490. نقطة، إقناع والدي بحظر المناقشات حول هذا الموضوع. وفي هذه الأثناء، أعتقد أننا سنستمر.
    ماذا عن الروابط التي وعدت بها لتفسير اللامكانية في التشابك الكمي؟ العالم ينتظر وينتظر.

  491. بالنسبة لأولئك الذين ما زالوا يعتقدون أن هذا النقاش برمته ربما يكون غير مثمر، حيث أن هناك بعض الاتفاق العام حول أصل الجاذبية والقصور الذاتي، وكل ما هو مطلوب هو الجلوس والدراسة قليلاً، يوصى بالنظر إلى الرابط:

    http://www.enotes.com/topic/Inertia

    ومنه:

    مصدر القصور الذاتي

    لا توجد نظرية واحدة مقبولة تشرح مصدر القصور الذاتي. الجهود المختلفة التي بذلها فيزيائيون بارزون مثل إرنست ماخ (انظر مبدأ ماخ)، ألبرت أينشتاين، دي سياما، وبرنارد هايش واجهت جميعها انتقادات كبيرة من المنظرين الأحدث.

  492. اليوبيل.

    نلقي نظرة على

    http://www.padrak.com/ine/INERTIA.html

    ونرى:

    ولاختبار الفكرة، اعتبر أينشتاين جسمًا صغيرًا ساكنًا داخل غلاف دوار من
    المواد في مساحة فارغة خلاف ذلك. وباستخدام نظريته النسبية، حسب ما سيحدث. اتضح أن الجسم يجب أن يشعر بالفعل بقوة الجاذبية المغناطيسية الصغيرة

    أعتقد أنني أفهم بالتأكيد ما يقوله مائير. إن العمل باستخدام الأنماط الدورانية أسهل بكثير من العمل بالحركة الخطية.
    .

  493. تفترض جميع النظريات الفيزيائية اليوم "طاولة عمل" تحدث عليها الظواهر. نفس النظريات لا تشرح بنية سطح المكتب هذا، لأنه لا يوجد شيء نسبيًا يمكن تفسيره.
    ولذلك فمن الواضح أنه لا توجد نظرية موجودة اليوم يمكن أن تكون كاملة.

    وثانياً، يعني أيضاً أن ذلك التفسير الأولي يجب أن لا يعتمد على شيء، وبما أنه يمكن إثبات أننا لا نستطيع أن نفكر في شيء كهذا (باختصار، تفكيرنا ترابطي ويربط شيئاً بشيء آخر، لذلك لا يمكن أن نفكر في شيء كهذا) التفكير في شيء لا يتعلق بشيء آخر)، ويترتب على ذلك أننا لن نتمكن أبدًا من فهم الواقع كما هو.
    هذا الأمر سهل الفهم، وقد أربك كانط الجميع عندما كتب كثيرًا عن هذا الموضوع. الأمر بسيط.

  494. إسرائيل! حسنًا، لقد أخذت الطعم ["مثال يوفال (الممتاز)"].
    المثال الذي ذكرته (بالمناسبة مازحا) في الحقيقة ليس ممتازا وغير ناجح على الإطلاق، لأن الأسطوانة مهما كان وزنها ستمارس نفس الجاذبية سواء دارت أو ألغيت.

    وبما أنني هنا بالفعل، سأضيف أنه يبدو لي أنك لم تفهم حجة مائير. كان يتحدث عن مجال الجاذبية الذاتية لبروتون واحد ويتحرك في خط مستقيم، بينما أنت تتحدث عن قرص متعدد البروتونات. للانتقال من الحركة الخطية للبروتونات إلى الحركة الدورانية للقرص، يجب إجراء التصحيح المناسب.

  495. اليوبيل.
    لا يوجد مشكلة. اسمحوا لي أن أعرف إذا كنت مهتما بالتعليقات في المستقبل. أنتم بالتأكيد موضع ترحيب لمواصلة الرد على تعليقاتي.

    مئير.
    أنا معجب بعملك. لقد ساعدتني على فهم كيف يمكن لأينشتاين أن ينكر مبدأ ماخ. لكني مازلت لا أفهم بعض الأشياء في فكرتك. بعد إذنك نواصل الأسئلة.

    1. وفقًا لكلماتك أيضًا، يمكننا معرفة السرعة الزاوية التي تدور بها الأقراص، حتى في غرفة مغلقة. ويظل السؤال: التدوير بالنسبة إلى ماذا؟ يقول أينشتاين - نسبة إلى الفضاء الزمني المطلق. ومع ذلك، إذا فهمت ادعاءك بشكل صحيح، فإن القصور الذاتي ينشأ من حركة الجسم من مركز ثقله. إذا كان الأمر كذلك، فكيف يمكننا أن نقول أن القرص A يدور بسرعة 5 دورات في الثانية والقرص B 7؟ نسبة إلى ماذا؟

    2. لنأخذ المثال (الممتاز) ليوفال على إطار مع دلو من الماء في المنتصف. صحيح أنه عندما يكون الإطار في حالة سكون، فإن قوة الجاذبية داخل المنطقة المحيطة به تساوي 0. لكنني أعتقد أن الأمر مختلف عندما يدور الإطار بالنسبة للدلو. إذا تمكنا من ضغط مادة إلى تركيز 1000 طن لكل متر مكعب، وقمنا بتجميع الإطار من هذه المادة، فإن هذا الإطار الدوار الذي يبلغ نصف قطره الداخلي 2 متر ونصف القطر الخارجي 2.5، سيزن عدة ملايين من الأطنان لذلك إذا وضعنا الدلو سيظهر أن الماء يدور ويكتسح. سيحدث نفس الشيء إذا ظل الإطار في حالة سكون ودار الدلو.

    ألم نحصل على "مبدأ ماخ" في إنفين الصغيرة؟

  496. R.H. Rafai.M،
    لقد سألتني إذا كنت أنا من حدد الجزيئات الدقيقة والمساحة الفارغة المحيطة بها. الحقيقة هي نعم، ولكن لا بد لي من توضيح شيء ما. في البداية، قبل أربعين عاما، كنت أبحث عن نموذج "بسيط"، وافترضنا أنه لن يحتوي على العديد من الافتراضات (ثم قيل لي للتو عن ماكينة حلاقة أوكهام، وأعجبني ذلك). لقد قمت بتعريف الجزيئات بالطريقة التي قدمتها هنا. ومع مرور السنين أدركت أن هذا التعريف مرهق للغاية وبحثت عن تعريف أبسط له. ولهذا الغرض "جندت" النفي وخلق "نعم" من "اللا شيء" وحده دون تدخل طرف خارجي. لم أرهم في أي معادلة. لقد قمت ببناء الوصف (الرياضي) بنفسي. عبارة "الفيزياء غير موجودة" ليست عذرًا، لأنني في هذا النموذج أتظاهر ببناء الفيزياء.
    ولم يأت بعد المساهم في الرسومات التخطيطية. يمكنني، على سبيل المثال، تحميلها على الويب كمستند Google ومشاركتها مع العالم أجمع.
    أنت تقول أنه من الممكن الاعتقاد بأن اللغة هي الشيء الوحيد الذي كان موجودًا في تلك اللحظة البدائية. يبدو الأمر جميلاً، لكنني لم أخوض في هذا السؤال. الرياضيات بشكل عام هي لغة، وهي في تطور مستمر.
    وعن سؤالك "العادل" الذي يبدو لي أكثر أهمية من الأسئلة الأخرى: "كيف يمكن دحض النظرية؟":
    وبما أن النموذج رياضي في المقام الأول، فيمكن اختباره باستخدام أدوات رياضية. على سبيل المثال، إذا "اخترعت" رياضيات جديدة، فلن تكون رائحتها طيبة. وبعد مرحلة الوصف التشكيلي تأتي مرحلة التعريفات، تليها مرحلة الصيغ والمقارنة مع الملاحظات في الطبيعة. لذلك سيكون من السهل العثور على العديد من نقاط الضعف. من المتوقع منك الكثير من العمل 🙂

  497. آر إتش،
    مقدمة "أسطورية" قصيرة: بدأ الكون (بدون "افترض ذلك") مليئًا باللا شيء، "لا شيء". وذلك لأنه إذا افترضنا أن الكون بدأ من شيء ما، فلا بد أن لهذا الشيء بداية أيضًا. يفضل البعض تقديم الكون كشيء أبدي ويقولون "لقد كانت المادة دائمًا". في رأيي، هذا ليس خطأً فادحًا، لأن مفهومي "الخلود" و"دائمًا" يمكن اعتبارهما جوهرًا موجودًا خارج الزمن، و"العدم" كذلك أيضًا لأنه عندما لا يوجد شيء، يوجد أيضًا لا وقت.
    السؤال الأول الذي يلفت الأنظار على الفور هو كيف تم خلق شيء من "لا شيء"؟ وبجانبه السؤال: كيف يوجد مثل هذا الاختلاف الكبير في الكون؟ لذلك، حتى لا يُنظر إلى هذا الادعاء على أنه بيان فارغ، يجب على المرء أن يفهم الآليات التي يصبح فيها "لا شيء" "هناك" ويصبح "لا شيء" "كثيرًا".
    وكما أن افتراض أن الكون بدأ من "كائن" ما يحول السؤال من "كائن" إلى "كائن" آخر (سابق)، فإن افتراض وجود خالق للعالم هو تحويل بنفس الطريقة تمامًا، لأن إن افتراض وجود خالق (وهذا دون أن يتضمن السؤال اللاهوتي "عاقل أم لا؟") يثير التساؤل عن أصل ذلك الخالق. ولذلك، فإن "اللا شيء" الذي خلق منه "النعم" يجسد أيضًا في ذاته الآلية التي تخلق "النعم" من "اللا شيء"، أي أنه "اللا شيء" نفسه.
    ليس لدينا طريقة لمعرفة هذا "اللا شيء" السابق للإبداع، لأننا نعيش في عالم "نعم". ولكن يمكننا أن نحاول إسقاط "العدم" الأولي من خلال ما نعرفه من معرفتنا ببعض الظواهر اليومية. وهنا يأتي "افترض ذلك": لنفترض أن "لا شيء" ما قبل البدائي هو نفي. وهذا الافتراض ليس من فراغ. عندما نتحدث عن "لا" فإننا نقدمه كنفي لـ "هناك"؛ "لا" هي "لا يوجد". أي أنه ليس مجرد نوع من "الفراغ" المحايد مثل الرقم صفر في عملية الطرح (أو الجمع)، ولكنه فراغ فاعل مثل الصفر في عملية القسمة (أو الضرب). إنه يخدع.
    "لا شيء" ما قبل البدائي ليس لديه ما ينكره إلا نفسه، لأنه لا يوجد شيء خارجه. نعلم من الحياة اليومية أن نفي النفي يعطي شيئًا جديدًا نسميه "إيجابيًا". وقد عرضت هنا إمكانية وجود آلية لخلق الوجود من العدم دون تدخل أي عامل خارجي، لكن سؤال التنوع الكبير للظواهر في الكون لا يزال قائما.
    "لا" النفي هو عامل فعال بينما "هو" غير فعال. ومرة أخرى نحتاج إلى آثار من العالم الذي نعرفه. من الممكن، على سبيل المثال، رؤية فعل تطبيق "لا شيء" على نفسه كقسمة (في القسمة أو الضرب - لا يهم). "اللا شيء" البدائي انقسم إلى "لا شيء" اثنين. كل واحد منهما في حد ذاته هو "لا شيء" فاعل، لكن كلاهما معًا يظهران كـ "نعم" محايدة. لا يوجد ما يمنع كل "لا شيء" من الانقسام مثل "اللا شيء" قبل الخلق، وبالتالي يتم إنشاء العديد من "العدم" بلا حدود. كل زوج من "لا" لديه سلوك مختلف عن سلوك "لا" واحد ونحن نسميه "نعم". مرة أخرى، من خلال استقراء عالمنا، يمكننا أن ننظر إلى مجموعات مختلفة من "ليست" و"موجود" كسلاسل من "ليست"، وهناك عدد لا حصر له من هذه السلاسل.
    من هنا وحتى تحديد خصائص الجسيمات الدقيقة للمادة المظلمة، وكذلك أجزاء الفضاء الفارغ، لا يزال هناك طريق طويل لنقطعه، لكنني آمل أن يكون الفضول البديهي قد وجد بعض الرضا.

  498. إسرائيل،
    وحتى إشعار آخر، اعتبر نفسك معفى من الرد على كلامي. سأحاول أيضًا عدم الإجابة على الأسئلة التي تطرحها هنا، أيضًا من أجل الإنصاف ولكن بشكل رئيسي لأن مئير يؤدي المهمة بشكل جيد (على الأكثر، يمكنني العثور على شيء أضيفه إلى كلماته ولكني لا أرى حاجة لاستبداله).

  499. إسرائيل،

    "السؤال 1: هل يمكنك فقط من خلال مشاهدة مقاطع الفيديو معرفة ما يظهره مقياس الطاقة على الأقراص؟ لا تنس، على الرغم من أنه من الواضح أن السرعة الزاوية النسبية بينهما هي دورتان في الثانية، إلا أننا لا نستطيع معرفة ما إذا كانت السرعات هي 2 و7، أو 5 و17، أو حتى 19+ و1-، أو 1 و0. "

    لا أستطيع، إلا إذا أظهرت مقاطع الفيديو أيضًا صورة لنجوم أعرف أنها بعيدة.

    "السؤال 2: هل يمكننا معرفة ما يظهره عداد الطاقة بمجرد النظر إلى النجوم؟"

    כן

    "السؤال 3: إذا كانت الإجابة على السؤال 2 إيجابية، ألا يظهر هذا وجود علاقة بين النجوم والقصور الذاتي، أو باختصار مبدأ ماخ؟"

    وهذا يدل على الارتباط، ولكن بالتأكيد ليس مبدأ ماخ. كما ادعى نيوتن أن النجوم البعيدة هي مؤشرات لحالة الماء في الدلو، ومع ذلك أطلق ألبرت على المبدأ اسم "مبدأ ماخ"، لأنه على عكس نيوتن، ادعى ماخ أن النجوم هي التي "تقوم بالقصور الذاتي" ميكانيكيًا، و وليست مجرد مؤشرات تظهر لنا الفضاء الفارغ الساكن الأبدي الموجود أيضًا بدون نيوتن.

    يمكنك أن تجعل الأمر صعبًا على نيوتن كيف تجرأ على الاعتقاد بأن النجوم البعيدة رتبت نفسها في حالة سكون بالنسبة إلى الفضاء. فقط عن طريق الصدفة؟! هذا هو جوابي، أنه ليس من قبيل الصدفة. لن أتحدث نيابة عن نيوتن (الذي لم يفترض أن قانون الجاذبية الذي وضعه يضع حدًا أعلى لسرعة حركة الكتل)، ولكن عن نفسي: ليس من قبيل الصدفة أن تكون النجوم البعيدة مؤشرات على السكون بالنسبة إلى الفضاء. وذلك لأن سرعة حركتهم في الفضاء محدودة. عند المشاهدة لمسافات طويلة، سيتم أخذ أسرع مكان في الاعتبار. في ساحة البلدة يندفع الجميع من حولك. وعندما تنظر منه إلى نهاية الجادة في الأفق، يبدو كل شيء هناك هادئًا وساكنًا. هذه ليست فيزياء، هذه هندسة.

  500. يوبيل، ليس لدي القوة لتحمل كل هذه المشاعر مرة أخرى هذه المرة. أجبني بصراحة لنبقى أصدقاء:
    أرد أم أصمت؟
    لأنني لن أتمكن من الرد إذا فاتني فصول في النموذج.

    أشباح!
    احصل على إعجاب لتحسين أسلوبك في الكتابة والصياغة. (وهذا على الرغم من القص). فخورة جداً بك هذه المرة.

  501. R.H. Rafai.M،
    شكرا. لقد قلت بعض الأشياء المثيرة للاهتمام، وأنا أستعد للحديث عنها مطولاً، في وقت لاحق اليوم أو الليلة.

    آر إتش،
    قررت حفظ المقدمة المهمة لأن مناقشتها تسببت في تأخير كبير. وسوف الخوض في هذا في وقت لاحق

    إسرائيل!
    لقد ذهبنا بالفعل إلى تشيلي مرة واحدة. دعونا لا ننجر إلى هذا مرة أخرى. أرجو أن تفهم أن لدي مشاكل معك

  502. مئير.
    إجابة لطيفة، وربما صحيحة، لأن هذا ما يقوله أينشتاين أيضًا.

    عدم الفهم معي. دعونا نحسن المشكلة.

    يقال أن لدينا مقاطع فيديو للقرصين يصوران بعضهما البعض في الفضاء.

    يُظهر مقطع الفيديو رقم 1 من الكاميرا رقم 1 القرص 2 وهو يدور بسرعة زاوية تبلغ دورتين في الثانية في اتجاه عقارب الساعة.

    الفيديو رقم 2 من الكاميرا رقم 2 يظهر القرص 1 وهو يدور بسرعة زاوية تبلغ دورتين في الثانية عكس اتجاه عقارب الساعة.

    السؤال رقم 1:

    هل يمكنك فقط من خلال مشاهدة مقاطع الفيديو معرفة ما يظهره مقياس الطاقة على الأقراص؟ لا تنس، على الرغم من أن السرعة الزاوية النسبية بينهما تبلغ دورتين في الثانية بشكل واضح، إلا أنه لا يمكننا معرفة ما إذا كانت السرعات هي 2 و7، أو 5 و17، أو حتى +19 و-1، أو 1 و0.

    السؤال 2: هل يمكننا معرفة ما يظهره عداد الطاقة بمجرد النظر إلى النجوم؟

    السؤال 3: إذا كانت إجابة السؤال 2 إيجابية، ألا يدل ذلك على وجود علاقة بين النجوم والقصور الذاتي، أو باختصار مبدأ ماخ؟

    نقطة.

    لقد صنعت شجرة أو بلاطة، وخرج الكم.

    وهذا هو الرابط الذي أطلبه:

    شرح فيزيائي لكيفية انتقال معلومات السبين من الإلكترون A إلى الإلكترون B على مسافة سنة ضوئية في زمن الصفر. تفسير فيزيائي - وليس "انهيار الدالة الموجية" (جسم رياضي)، وليس "انظر ما قاله بوهم" وليس محاكاة 17 بُعدًا في كون بدرجة حرارة سلبية.

    إذا تمكنت من العثور على مثل هذا الرابط، أو شرحه بنفسك، فسوف يتم تكريمك وتقديرك. جميع المستجيبين في الموقع سوف يقومون بالرد عليك أنت فقط. سوف يقوم حزب يرفع شعار: دا دا، فترة. ناهيك عن بعض جوائز نوبل الصغيرة على الجانب.
    والحافز الأكبر: سأضطر إلى إغلاق فمي الكبير - وأغلقه بإحكام، إلى الأبد.

    ولكن إذا لم تنجح، لدي طلب واحد فقط: القليل من الصمت. أسبوع، لا أكثر.

    فماذا سيكون: دا دا دوت، أو دوت دا؟

    ر.ح.
    ينقط ، إيه؟ تركنا للعمل والذهاب لركوب الأمواج؟ هل تفعل المسارات السوداء؟

    اليوبيل.
    وما زلت في انتظار المعلومات المفقودة. هل يمكنك النقل الفوري؟

  503. اليوبيل,
    هل ترى ما كتبه رافا * ايم؟ هذا بالضبط ما قصدته بشأن المقدمة المفقودة من النموذج الخاص بك. إذا رميته في الوجه فإنه يسبب الحيرة ورفع الحاجبين بشكل مبرر.

    عليك أن تمضي قدمًا وتقول "لنفترض أن العالم يتكون من كذا وكذا من الجسيمات الدقيقة وغير الدقيقة و..."
    وبعد ذلك سوف تعطي الأسباب المنطقية لكل هذه الافتراضات الغريبة:
    "إذا افترضنا كل هذا وفتحنا الفكرة من هنا، فسنرى فجأة أن بعض الأسئلة التي لم يتم حلها اليوم مثل XYZ تتلقى إجابة مثيرة للاهتمام وبسيطة، وبالتالي هناك نقطة في القصة أعلاه"

    وإلا فإن كل هذا ميثولوجيا وليس نموذجا.

  504. يوفال
    لقد سحبتني للرد عليك مرة أخرى. حسنًا، لقد قمت بفحص نموذجك بإيجاز وماذا يمكنني أن أقول لك؟
    لنفترض أن السؤال "2) تهتز الجسيمات الدقيقة بشكل عشوائي، وتتحرك دون تحديد الاتجاه والسرعة في أجزاء الفضاء الفارغ المحيطة بها."
    أو "(لأنهم تم تعريفهم على أنهم دقيقون)." - ومن قام بتعريفهم؟ انت؟ هل رأيتهم في مكان ما في نوع من المعادلة؟ أم أن عذرًا مثل عدم وجود الفيزياء هو عذر كافٍ بالنسبة لك؟
    هل هذا هو نوع حجة الفوز الخاصة بك؟ حتى لو قمت بتغيير الصورة إلى إيريك وبنز وقمت بتسمية نموذجك بنظرية الأوتار يوفال تشايكين، فلن يساعدك ذلك. إذا كان هذا هو النموذج الخاص بك، فيبدو لي أنه سيتعين عليه دفع إيجار درجك لمدة 40 عامًا أخرى.
    وأين اتفاقكم؟ اسأل المحرر، فهو شخص جيد، ومن المؤكد أنه سيفتح لك مكانًا حيث يمكنك تحميل المخطط. الجميع سوف يحكمون ليس فقط على أصدقائك.
    عندما تقول ما تقوله، قد يعتقد المرء أن اللغة هي الشيء الوحيد الذي كان موجودًا في تلك اللحظة البدائية.
    مجرد سؤال، إذا أردت فمرحبا بك للإجابة: (حتى لو لم تنته من إخبارنا بفكرتك بعد) كيف يمكن دحض نظريتك؟

  505. نقطة مهمة نسيتها:
    في القسم الأول من الأقسام الثلاثة، نسيت أن أضيف أن الجسيمات الدقيقة لديها القدرة على التكاثر وكذلك الموت. إذا توفرت مساحة فارغة كبيرة بما فيه الكفاية بالقرب من جسيم حاد، فإن جسيمًا حادًا جديدًا يولد فيه. إذا تجمعت الجزيئات الدقيقة بكثافة شديدة، يتحول بعضها إلى مساحة فارغة. وأسباب ذلك مفصلة في الأقسام السابقة التي أنقذتها من قارئين ونصف.

  506. إسرائيل،
    "السرعة المماسية - بالنسبة إلى ماذا؟ إلى المادة المظلمة؟"
    سرعة الضوء مرتبطة بكل مراقب. في هذه الحالة بالنسبة لك، من يقيس سرعة دوران القرص الكبير.

    "لماذا لا يمكننا القول أن السرعة العرضية للقرص B، الذي يدور، هي سرعة الضوء بالنسبة للقرص A، الذي لا يدور؟"

    لأن سرعة الدوران هي السرعة الزاوية. عندما تكون السرعة العرضية للقرص الكبير هي سرعة الضوء، تكون سرعته الزاوية صفرًا. وسيكون هذا أيضًا مصير السرعة الزاوية لأي قرص ذي حجم مماثل. لن يتمكن الراصد الجالس على القرص A (القرص الكبير) من الادعاء بأن السرعة الزاوية للقرص B (طبق طائر، أو كوكب، أو مجرة) تساوي صفرًا، لأنها لا تستوفي التعريف.

    أفهم أنك تريدني حقًا أن أقول إن ما يحدد المرجع هو متوسط ​​السرعة الكونية، أو مجموع الزخم الكوني. أنا لا أقول ذلك، لأنه ببساطة غير صحيح. وهذا ليس السبب وراء ظهور النجوم البعيدة ثابتة.

    السبب ذو الصلة هو أنه عندما تنظر إلى مسافات كبيرة فإن الكون بأكمله يكون في حالة سكون، لأن جميع الكائنات الموجودة فيه محدودة بسرعة الضوء. يمكنك رؤيتها كأضواء تحدد الفضاء، لكنها لا تؤثر ولا تحدد أي شيء في حركة الأجسام البعيدة. حتى لو لم يكن مجموع الزخم الكوني صفرًا، وحتى لو كانت جميع النجوم البعيدة تتحرك بسرعة الضوء في اتجاه محدد كأمر عسكري، فستظل قادرًا على تحديد متى تدور اللوحة B من خلالها، وبأي سرعة زاوية.

    لذلك، مثل نيوتن وعلى عكس ماخ، حتى في الكون الفارغ، ستستمر لوحة الدوران الخاصة بك في الدوران (باستثناء هذه العزلة المضيئة وفي غياب الأشعة الخارجية) سيتعين عليك الاعتماد على مقياس قوة الطرد المركزي المثبت عليها.

  507. إسرائيل ويوفال

    انا فخور بك. لقد تركتك لمدة يومين لأغراض التزلج وهنا تحققت له رؤيا إشعياء "وعاش الذئب مع خروف... وجسد جميل!"

  508. مئير،
    ما زلت لا تحصل عليه.

    "نحن نعرّف القرص بأنه لا يدور إذا كان قطره لانهائيًا، وكانت سرعته العرضية هي سرعة الضوء."

    السرعة العرضية - بالنسبة إلى ماذا؟ إلى المادة المظلمة؟
    لماذا لا يمكننا القول أن السرعة العرضية للقرص B، الذي يدور، هي سرعة الضوء بالنسبة للقرص A، الذي لا يدور؟

    نقطة

    "افتح الكتاب المدرسي وتعلم."
    هل يمكنك الإشارة إلى كتاب دراسي واحد، بما في ذلك رقم الصفحة، حيث تم شرح هذا الموضوع؟ ربما لديك رابط لمشاركته معنا جميعا؟

    لأنه إذا لم يكن لديك، فربما تتوصل أخيرًا إلى نتيجة مفادها أنه لا يوجد حتى الآن تفسير لما طلبته؟ وما نتحدث عنه هنا هو محاولات لإيجاد تفسير وليس مجرد كلام؟

  509. إسرائيل، يسعدني أن أشرح لك ذلك، لكن الأمر هو أن الأمر لا يتعلق بالأفكار التي تتم مناقشتها.
    افتح الكتاب المدرسي وتعلم. يمارس لن تتعلم أي شيء بالحديث عنه.

  510. إسرائيل،

    لا توجد سرعة أعلى من سرعة الضوء، لذلك في الفضاء الذي يكون توسعه متناحيًا لا توجد كائنات تتحرك بسرعة أعلى من سرعة الضوء بشكل عمودي على الخط الذي يربطها بجسم بعيد. إذا كان القطر اللانهائي يزعجك، فسنرضى بأقل من ذلك. إن القطر المحدود الذي تحده النجوم البعيدة (= الأضواء) كافٍ ليظل التعريف الذي قدمته صالحًا.

    نظرًا لأن الكائنات الموجودة في عالمنا تشترك في نفس المساحة، والتي تحدد أيضًا، من بين أمور أخرى، سرعة الضوء كحد أعلى لها، فإن القرص المحدد كما حددته لا يدور دائمًا، ولن يتم الشعور بأي قوة طرد مركزية دائمًا فيه.

  511. اليوبيل.
    أثناء انتظار رد رفائيم على سؤالك، هل يمكنك أن ترسل لي الفصول المفقودة حتى أعرف ما هي موضوعها؟

  512. الفترة، الأرض. إذا كنت خبيرًا في معادلات الموتر، تعال واشرح لنا كيف تعمل الجاذبية، وكيف يخضع دوران الإلكترون للتشابك الكمي. إذا لم يكن الأمر كذلك، أطلقوا سراحنا. هناك العديد من المقالات الأخرى التي تتطلب حضورك، ومع كل الصعوبات التي ينطوي عليها الأمر، يبدو لي أننا سنتدبر الأمر بطريقة ما بمفردنا.

  513. إسرائيل شابيرا,
    سيتعين على غير المحليين الانتظار قليلاً. سأبدأ بالجاذبية (والمثابرة).
    لكن أولاً وقبل كل شيء، يعد وصف النموذج ضروريًا لصالح أولئك الذين لم يتلقوا مني مادة عبر البريد الإلكتروني:
    بعد تخطي عدة خطوات (بدءًا بالآلية التي ينطبق فيها "اللا شيء" على نفسه لإنشاء "هناك" والاستمرار في كيفية وكيفية إنشاء جسيم المادة المظلمة) توصلت إلى الوصف التالي:
    1) يسكن الكون بالكامل مادة مظلمة شكلها عبارة عن جسيمات دقيقة (أي لا تسمح باختراق جسيمات أخرى فيها) وجسيمات غير دقيقة (والتي يمكن تسميتها "الفضاء الفارغ" للإيجاز [ ولكن يجب تمييز هذا الفضاء الفارغ عما تسميه الفيزياء العادية "الفضاء الفارغ"، لأن الفضاء الفارغ في الفيزياء هو الفضاء الذي تسكنه الجسيمات المعنية]).
    2) تهتز الجسيمات الدقيقة بشكل عشوائي، وتتحرك دون تحديد الاتجاه والسرعة إلى قطع من الفضاء الفارغ حولها.
    3) عندما يقابل جسيم حاد جسيمًا حادًا آخر فإن رد الفعل الوحيد بينهما هو عدم دخولهما في بعضهما البعض (لأنهما عرفا بأنهما حادان). وعلى هذا المستوى لا وجود للفيزياء، لذلك لن يحدث بينهما شيء معروف من الفيزياء، مثل الاصطدام المرن مثلا.
    وكيفية تعريف الجسيمات المعروفة في الفيزياء وكيفية حدوث الجاذبية بينها سيتم شرحها لاحقاً بعد الأسئلة والأجوبة الخاصة بهذا الجزء. ر.ه.رفاعي م: هيا! من فضلك كن أول من يسأل.

  514. يوفال

    زيف بخير. سأكون أيضًا أكثر من سعيد بقراءة النموذج الخاص بك. (على الرغم من أنني أعتقد أن ما قرأته خاطئ تمامًا، إلا أنه في رأيي على الأقل يبدو منطقيًا أكثر من النموذج الإسرائيلي، يا صديقك. ومن المؤكد أنه يجعل وقتي يمرر بالمرح).

    إسرائيل

    أي اتفاق سلام؟ أقول لك رأيي الصادق. وأنت مستاء. إذا كنت تريد مني أن أتوقف عن إبداء رأيي الموضوعي فقط حتى تتمكن من التوقف عن التعرض للأذى، فمن المحتمل أن تستمر في التعرض للأذى.

  515. جمال اليوبيل. هل أستطيع إرجاع الصورة القديمة؟ أفضل بكثير في نظري).
    أشباح! هل أنتم مستعدون للانضمام إلى اتفاق السلام ووقف الهجمات الشخصية؟ أنا أكثر من راغب.

    اليوبيل.
    العودة إلى الأسئلة القديمة.
    1. كيف تعمل الجاذبية في النموذج.
    2. كيف يعمل غير المحلية؟

    يمكنك أن ترسل لي الروابط، أو كتابة الشرح هنا. ولكن من فضلك، ليس جمل مثل: "لقد تحدثنا عن ذلك بالفعل"، أو "يظهر في الفصل 7". أنا مشغول بأموري، كما نحن جميعا مشغولون بأمورنا، ولا أتذكر كل تفصيلة أو تعليق كتبته.

  516. مئير

    "نحن نعرّف القرص بأنه لا يدور إذا كان قطره لانهائيًا، وكانت سرعته العرضية هي سرعة الضوء."

    ألا ينطبق التعريف "غير الدوار" على أي سرعة عرضية محدودة أخرى؟ ماذا عن 100C، 2C؟ وحتى في تلك الحالة، إذا كان القطر لا نهائيًا، فإن القرص لا يدور. لا؟

    وأعترف أيضًا أنني لم أفهم كيف يفسر هذا أي من الأقراص يدور "حقًا" وأيها لا يدور.

  517. الى! الى! إسرائيل! وقعت وقعت
    R. H. مدعو لفتح زجاجة الشمبانيا (على حسابي) أثناء قلب الملعقة.
    نهائي MM و LMD موجودان بجوار بعضهما البعض على لوحة المفاتيح، وآمل وأعتقد أن القطة ولدت من خطأ مطبعي ولم يتم تحديدها لصالح أحد.
    R.H. Refai.M، من فضلك لا تأخذ الأمر على محمل الجد.
    قريبًا، إذا ساعدني، سأستمر وأشرح بالتفصيل بنية المادة المظلمة وفقًا لرؤيتي للعالم. سأشير مقدما إلى أن هناك الكثير من التشابه مع تعاليم مئير عميرام. يرجى الاستمرار في المتابعة.

  518. "سلايم"، اقتباس مباشر من المايسترو، أليس كذلك؟

    وهذا صحيح، مباشرة بعد أن اعتذرت (لماذا بالمناسبة؟ ما الذي بدأت من أجله؟) واصلت. انظر هذا التعليق:

    https://www.hayadan.org.il/astronomers-reach-new-frontiers-of-dark-matter-130112/#comment-327515

    ويبدو لي أكثر أن هناك أشخاصًا يحتاجون إلى دروس خصوصية من طويل فضائح، وزيرة الغضب الجامح الفلسطينية من لطمة. ما يجب على الأشخاص الذين يأكلون الأنا أن ينتقدوه في كل مرة يتم فيها التعبير عن رأي يختلف عن آرائهم، دون حتى أن يكلفوا أنفسهم عناء الاستماع إلى نهاية الجملة أو التوقف.

    وبما أنني لا أرغب في توجيه مطرقة 5 كيلو إلى رأس الإيرانيين، وهي طريقتهم في إثبات وجود الكتلة المظلمة، فإلى أن تتحسن لهجة الحديث هنا، ليس لدي أي نية للدخول في الخوض في الأمر. الجدال مع أولئك الذين تسترشدهم اعتبارات الأنا وليس العلم.

  519. حسناً، ماذا يحدث لإسرائيل؟ هل أنت من الوحل؟
    ولماذا تذكرني أصلاً؟ ما الذي يثير اهتمامك في رأي الصبي الأخضر الغبي؟ هل يؤلمك شيء في عينك؟
    لماذا لا تتناول الفكرة التي طرحتها (رداً على يوفال)؟ لماذا تستمر في كتابة الهراء؟

  520. إسرائيل

    "هل تريد أن تأتي على القطة؟" ؟؟ هذه النكتة غير منطقية لأنه من غير المنطقي أن ينمو الشخص خصيتين على جبهته. (يوفال، أعلم أنك تضحك الآن 🙂)

    على أي حال،

    ومن الملائم جدًا أن نأتي ونقول: إسرائيل الأحمق.
    ومن الأسهل أن تقول لاحقًا: أوه، آسف، لم أقصد ذلك.

    وبالحكم على ما كتبته: عليك أن تقبل اعتذاري. واضح وسلس.

  521. الصورة الغاضبة مرة أخرى؟

    (روكي في كولورادو). لا أتذكر أنني كتبت رونيكًا على الإطلاق. الفيسبوك، إيه؟

    سيكون من الصعب جدًا الاستمرار عندما تتعرض لخطر الإصابة الشخصية في كل مرة تجرؤ على الانتقاد. المحلفون، الذين يؤدون اليمين. لذلك، ومن دون إعلان نوايا، وفي ظل التجربة الماضية معك ومع الأشباح، سيكون من الصعب علي أن أواصل النقاش الجاد معك في أي موضوع.

    مثال شخصي:

    أنا، إسرائيل شابيرا، أتعهد بموجب هذا بعدم إيذاء المستجيبين شخصيًا عمدًا، ولكن كرد فعل على الأذى الشخصي الذي يقصده المدعى عليه. إذا أسأت عن غير قصد، ودعاني صاحب التعليق للطلب، سأعتذر ولن أكرر خطأي.

    ولهذا السبب جئت إلى الموقعين أدناه:

    إسرائيل شابيرا
    ------------

    ماذا عنك؟ تريد أن تأتي إلى التوقيع؟

    أشباح؟ هل تريد أن تأتي على القطة؟

  522. إسرائيل،

    سوف نقوم بتعريف القرص غير الدوار إذا كان قطره لا نهائيًا، فإن سرعته العرضية هي سرعة الضوء.
    سنقوم بوضع مصابيح LED في نقاط مختلفة على القرص، كما نرغب.
    سيتم استخدام هذا القرص لقياس سرعة دوران جميع الأقراص الأخرى.

  523. تشيلابا (أي ليس طلبًا صريحًا يطلب الإجابة) عزيزي،
    أولاً، شيء يتعلق بالاتساق:
    عندما تطلب kasach وتتحدث في نفس الوقت عن كلاب Veronit في جبال Rocky، فلا تتفاجأ إذا تم اختطافهم أيضًا.
    ثانيا النظارات:
    لم أكتب في أي مكان أن هناك علاقة بين القصور الذاتي والنجوم البعيدة. قلت فقط أن هناك ترابطاً بين الملاحظات، ولست مسؤولاً عن أي استنتاج تتوصل إليه بمبادرة منك.
    ثالثاً: الاعتبار الجاد:
    يمكنك الحفاظ على حقك في التزام الصمت، لكنك بذلك تخاطر بأن رأي هيئة المحلفين لن يكون متحيزًا لصالحك.

    إسرائيل شابيرا,
    في ذلك الوقت سألت عن جذر الرقم "الوهمي" i وأجبت أنه من الممكن حسابه. سأكون ممتنًا إذا أكدت أنك قرأت وفهمت، وإذا كنت ترغب في الحصول على شرح أكثر تعمقًا، فسأقدم لك ذلك بكل سرور.

  524. مئير
    أنا أفهم تماما تفسيرك. لقد ذكرت ذلك أيضًا عدة مرات. لكن الحقيقة لا تزال قائمة، وهي أن الدوران نسبي بالنسبة للنجوم البعيدة. إذا أمكنك هنا على الموقع شرح كيفية عمل مصابيح LED في حالة السكون، وتحديدًا بالنسبة للقرص A (حالة الراحة) وليس للقرص B (الدوران)، فسيسهل ذلك المناقشة بشكل كبير.

    مساء الخير.

  525. إسرائيل،

    لقد سألت كيف يمكن إنكار مبدأ ماخ، وأجبتك. ينص مبدأ ماخ على أن الكتلة الكونية تحدد درجة مقاومة كل كتلة على حدة للتغير في حالة حركتها. وقد وافق ألبرت على ذلك عندما ذكر أن القصور الذاتي هو ظاهرة التفاعل بين الكتل. أنا أنكر ذلك، وبينت أيضًا كيف يكون ذلك ممكنًا.

    مبدأ ماخ ليس الارتباط، بل الفرضية التي اشتقها ماخ منها. يمكن أيضًا استخلاص نفس الفرضية من مصدر ضوء بعيد عديم الكتلة. النقطة المهمة هي أن الكتلة الكونية بدت لماخ شيئًا أكثر إثارة للإعجاب، حيث يمكن تعليق التأثير على مسافة، على الرغم من أنه لم يوضح كيف.

    جاء مئير عميرام، وادعى أن الكتلة الكونية، بقدر ما هي مثيرة للإعجاب، تتكون من كتل الجسيمات الأولية التي تتكون منها. لذلك، بدلاً من حك الأذن اليمنى باليد اليسرى والادعاء مثل ماخ أن كل جسيم أولي ينشر تأثيره السحري على جميع الجسيمات الأولية الأخرى في الكون، دعونا نحك الأذن اليمنى باليد اليمنى وندعي أن كل جسيم أولي ينشر تأثيره السحري على جميع الجسيمات الأولية الأخرى في الكون. يحتفظ بتأثيره السحري لنفسه.

    كما ادعى مئير عميرام أننا إذا فعلنا ذلك، فسنكتشف أن قوانين نيوتن الثلاثة للحركة مستمدة مباشرة من قانون التربيع العكسي لنيوتن. فهو لم يدعي فقط، بل أظهر أيضًا كيف.

    في الختام، لدينا ماخ الذي ادعى أن كل جسيم أولي "يساهم" من خلال آلية غير معروفة وغير معروفة في التأثير على جميع الجسيمات الأولية الأخرى في الكون.

    ومن ناحية أخرى لدينا عميرام الذي يدعي أن كل جسيم أولي يؤثر على نفسه بطريقة معروفة ومألوفة.

    وبطبيعة الحال، فإن قبول مبدأ أميرام هو نفي لمبدأ ماخ، لأنه يضع النجوم البعيدة في فئة أضواء LED، والتي، حتى لو كان من الممكن استخدامها كإضافات لتسجيل الفيديو، لا تشارك بشكل فعال في الحبكة كما يدعي ماخ.

  526. أشباح
    إذا لم تجد أي فائدة من الرد، هيا يا تاف، ماذا تفعل - لا ترد.
    كما يمكنك العودة إلى مكانك الطبيعي: الصفر المطلق - وما تحته!

    اليوبيل.
    إذن هل هناك الآن علاقة بين القصور الذاتي والنجوم البعيدة؟ اعتقدت أننا كتبنا منذ وقت ليس ببعيد "فقط لنقول إن ماخ كان مخطئًا".

    لكننا انجرفنا قليلاً. لقد أخطأت في السابق بالرد على تعليقاتك دون إذن صريح، فوبخته على ذلك، دون أي مبرر. وكما ذكرت سابقا، لن أكرر الخطأ مرة ثانية. وبدون طلب صريح منك، لن أرد على ما تكتبه بشكل مباشر. إذا أجبت، بعد هذا الطلب الصريح، فسيكون ذلك فقط بالطريقة الوحيدة التي أعرفها سعيًا وراء الحقيقة: الطريقة السقراطية في الأسئلة والأجوبة. ستكون الأسئلة مدببة وغير شخصية ومتسقة ومنطقية. سأنتظر الإجابات وفقا لذلك. إذا لم يكن واضحًا لك ما يدور حوله هذا الأمر، فراجع تحقيق الشتي والعربي الذي أجراه ر.ح.

    إذا شعرت أنك تستخدم عبارات شخصية، أو تتجنب الإجابات، أو في رأيي تجيب خارج السياق، فسوف أستخلص استنتاجاتي وفقًا لذلك.

    وهذه، بالمناسبة، هي الطريقة الوحيدة في رأيي لمنع العارضات من التحول من النماذج المادية إلى نماذج الأنا. وهذا أيضًا هو السبب الذي يجعلني أدعو دائمًا إلى النقد كصح لكل فكرة أتوصل إليها: لسد الخطأ المتراكم في العداء.

    الكرة بين يديك. تذكر: B K S H M F O R S T

    أستطيع أن أعيش بشكل جيد للغاية حتى دون أن أتعرض للخطر بسبب محاولتي المساعدة.

    إسرائيل، غاسلة الدماغ التي لا يمكن الوثوق بها.

  527. إسرائيل شابيرا,
    لا يقول أنيو (وأعتقد أيضًا مئير) أنه لا يوجد ارتباط بين الملاحظتين (الفيديو ومقياس الطاقة). لكن ادعائي هو أن هذا الارتباط هو منتج ثانوي. تظهر الكاميرا الحركة النسبية بين الجسم والنجوم، ومقياس القوة يظهر التفاعل بين الجسم والمادة القريبة منه، والذي تستمر في تجاهله بعناد ملحوظ.

  528. إسرائيل
    لقد طلبت الرجوع إلى محتوى الأشياء.
    إذا كان الأمر كذلك، سأشير بعد إذنكم إلى مضمون الرد والجملة التي وردت فيه: إسرائيل الديماغوجي.
    أعتقد أنه كان عليك أن تكتب: إسرائيل المهرج.
    أما بقية كلامك فلا أجد أي فائدة في الرد عليه.
    اسبوع جيد.

  529. مئير.

    لا أدعي أن لدي تفسيرًا لمبدأ ماخ، أو أنه من غير الممكن بناء تجربة معينة قد تنحرف عن القاعدة. أنا فقط أزعم أنه يجب أن يكون هناك علاقة بين قوة الجذب المركزي والنجوم البعيدة. أنت لا تصدق ضع كاميرا فيديو تواجه السماء على جهاز طرد مركزي، ضع فيها عداد الطاقة، خذها إلى أي نقطة تختارها في القارة التي تعيش فيها، وقم بتدويرها بسرعة زاوية ثابتة، وأرسل لي الفيديو على اليوتيوب.

    أعتقد أنه فقط من خلال مشاهدته بالفيديو يمكنني أن أخبركم عن القوة التي يظهرها مقياس الطاقة في جهاز الطرد المركزي، وكم من اليورانيوم الذي يمكنه فصله.

    وكل هذا من فيديو لبعض النجوم الفقراء.

    ليس ساحر الكون؟

    بالطبع، قد يكون هناك تفسير لسبب تواجد النجوم في مكانها بالضبط وفقًا لفكرتك.

    وفيما يتعلق بالتجربة المقترحة هنا فهي تتعارض مع مبدأ ماخ: إذا أخذنا بيانات حقيقية، حيث يبلغ وزن السائل في الدلو حوالي 15 كجم، ويتم وضع النظام ليكون آمنا عند نقطة جزيرة الوزن بين الأرض و القمر، ثم الأسطوانة التي يبلغ ارتفاعها مترا واحدا ومركز كتلتها في نصف قطر متر واحد من الدلو، ستكون كتلتها حوالي 100,000 ألف طن لتعادل الشمس وحدها، ناهيك عن بقية الكواكب. كون. إنها تجربة مكلفة للغاية، والأكثر من ذلك، لا أعتقد أن ماخ كان سيعترض على تفسير تأثير الأسطوانة على الدلو والعكس صحيح. T.L.H.

    يوفال اركمان

    لقد أساءت تفسير إهانتي. أنا متحمس لأي شيء، طالما أنه في المستوى. ولقد ذكرتك دائمًا بشكل إيجابي باللغة العبرية الممتازة، عبر البريد الإلكتروني (الخاص بك).

    حتى عندما اقتبست عبارة "ماذا فعلت" كنت متحمسًا للغاية، لأنها ذكّرتني بأفلاطون شارون: "ماذا فعلت في مدينة زبل؟"

    ولكن في ضوء تجربة الماضي الحزينة، سأكون راضيًا بما جاء في عاموس 5: 14.

    إسرائيل الديماغوجيه.

  530. القواد الذي يسرق الكلمات بالخطأ - يعترف ويترك يروحام أم يغرق ويزحف في يده؟
    لقد أوضحت وجهة نظرك. لقد أظهرت أنني لا أحتكر الإهانة. إذا أخذت ما أقوله على محمل الجد لمرة واحدة، فسوف أتراجع عن كل الكلمات القاسية - باهتمام.
    ادعاء مئير (وأنا) هو أنه بالقرب من الأجسام (الأقراص الدوارة، على سبيل المثال) توجد مادة على الرغم من أننا لا نراها، إلا أن هناك تفاعلًا بين الأجسام وبينها. يسميه مئير "مجال الجاذبية الذاتية للأجسام" (وأنا أعرّفه بالمادة المظلمة للمجرات وأنسب إليه طابعًا جسيميًا كميًا واضحًا مع لمحة من السلوك الموجي).
    الادعاء الذي تتشبث به بالعناد الفردي هو أن الشخص المسؤول عن السلوك الخاص للأجساد ليس بالضرورة (أو ليس فقط) هو الأمر المعني بين الأجساد، بل الجماهير في الكون كله. وتعذر ذلك بالأرصاد التي تعكس النجوم البعيدة ولا تظهر هذه المادة (التي كما ذكرنا غير مرئية).
    والسؤال الذي أطلب منك أن تأخذه على محمل الجد، وهو ليس طريقك، هو لماذا تصر على تجاهله؟. ربما يمكنك أن تدعي أن وجودها لم يثبت (مؤكد، على وجه الدقة [الشكر للطالب])، ولكن هذا لا يعني أنها غير موجودة (في هذه الأثناء تلقينا تأكيدا على وجود المادة في الفضاء بين النجوم والذي، على الرغم من أننا لا نراه، يتداخل بقوة مع عمليات الجاذبية). يمكنك القول إنه أمر مخالف للحدس، لكننا تعلمنا بالفعل أن نعرف أن الحدس ليس المقياس الوحيد للواقع. يمكنك قول أشياء كثيرة، لكن من فضلك كن متسقًا ومنطقيًا وحاول تجنب القصص عن الكلب والمرأة وما إلى ذلك قدر الإمكان.

  531. إسرائيل،

    فيما يتعلق بأقراص الغزل:

    لنفترض أنهم يتحركون في كون خالي من أي كتلة أخرى. لا يزال هناك في هذا الكون مساحة نسيجها عبارة عن سلسلة متواصلة من الرغوة الكمومية التي يتم من خلالها توزيع جاذبية كل قرص من الأقراص.

    مجالات الجاذبية لكل قرص لا "تدور"، لأنه في رأيي لا يوجد إطار متعرج، وبالتالي فهي ثابتة بالنسبة لبعضها البعض. ولكنها عبارة عن مجموع مجالات الجاذبية المجهرية الناتجة عن الجسيمات الأولية التي تشكل الأقراص. إذا تسببت قوة خارجية في دوران أحد الأقراص، فإن مجالات الجاذبية الصغرى الخاصة به ستعاني من عدم التماثل في اتجاه الدوران، مما يؤدي إلى إدامة حركة الدوران، والتي بدورها ستؤدي إلى إدامة عدم التماثل هذا.

    القرص الذي تعاني مجالات الجاذبية المجهرية لجسيماته الأولية من عدم التماثل هو قرص دوار. وتعتمد درجة عدم التماثل في هذه المجالات على سرعة الدوران. كل هذا بغض النظر عن حالة حركة القرص الآخر. هذه هي سرعة الدوران المطلقة بالنسبة لنسيج الفضاء المحلي.
    إذا توسع الفضاء في هذا الكون (بسبب إضافة الرغوة الكمومية)، إلى الحد الذي يكون فيه هذا التوسع متناحيا (بسبب إضافة الرغوة الكمومية) فلن تحدث حركة نسبية بين الأقراص نتيجة هذا التمدد باستثناء المسافة المتبادلة دون تأثيرات القصور الذاتي.

  532. إسرائيل،

    ولنفترض لغرض المناقشة أن الأرض كانت تدور حول محورها مرة كل ربع ساعة، وأنت تقف بثبات على القطب الشمالي وتنظر إلى النجوم البعيدة من حولك مرة كل ربع ساعة.
    إن الدوران في مكان ما بمعدل مرة كل خمس عشرة دقيقة ليس ظاهرة ينبغي أن تشعرك بالحركة على جسدك، لذلك من خلال النظر إلى النجوم لن تتمكن من تحديد ما إذا كانت الأرض تدور حول محورها، أم أنها تدور حول نفسها. ما إذا كان الكون يدور حول الأرض.

    الآن تحصل على الدوار، الذي يدور دورة واحدة في الثانية بالنسبة للبلد، وتلتقط الفيديو. سيظل لديك فيديو تسجل فيه الدوران بالنسبة للنجوم البعيدة، بتردد يقارب 100% نفس تردد دورانك بالنسبة للأرض، أي بالنسبة لأي فيديو آخر ستختاره لغرض التسجيل التناوب.
    هل يمكنك أن تستنتج بناءً على هذا الفيديو فقط ارتباطاً بين الدوار والنجوم البعيدة، أكثر من ارتباط بينه وبين الأرض؟

    ولأغراضنا فإن النجوم البعيدة بعيدة جدًا، لدرجة أنه حتى لو كان هناك دوران نسبي بين المجرة وبينها، فلا يمكننا أن نحدد من خلال دوران الدوار ما إذا كانت المجرة تدور أم أن الكون يدور حولها. في المسافات المعنية، سيكون هناك دائمًا تطابق بنسبة 100% بين تردد دوران صورة الفيديو للنجوم البعيدة وتردد دوران صورة الفيديو التي تستكشف المشهد الأرضي.

  533. ترشيد
    لكي لا تحزن، أنا مستعد لإضافة مداخلة أخرى حول بوزون هيغز،
    فما رأيك؟
    (:))
    يوم جيد
    سابدارمش يهودا

  534. مئير.
    إينال زاففور بوبلوفا. ومرة أخرى الرد في انتظار التأكيد. حوالي الساعة الرابعة صباحًا في لوس أنجلوس. ليلة سعيدة (صباحا؟)

  535. مئير.

    لا أستطيع أن أقول. ولكن يبدو منطقيًا بالنسبة لي أنه إذا كان بإمكاني، فقط من خلال فيديو النجوم البعيدة، أن أعرف بالضبط، دون حتى النظر، ما هي القوة التي سيتم قياسها بمقياس قوة الطرد المركزي في الدوار، فمن المحتمل أن يكون هناك نوع من الارتباط بينهما... وإلا يمكنني أن أستنتج القوة من أي فيديو آخر، أليس كذلك؟

    كما لو كان معظم الأطفال في السويد أشقر بعيون زرقاء، فهناك بعض الارتباط بين العيون الزرقاء والشعر الأشقر، وهناك احتمال أكبر قليلاً أن يكون للطفل السويدي الأشقر عيون زرقاء مقارنةً بطفل زنجي من قبيلة الزولو على سبيل المثال. . اسأل R.H. فهو يفهم علم الوراثة والسويد. لا؟

    كما يبدو من المعقول بالنسبة لي أن أجبر يهودا على إطعام يائيلي بجزيئات الآيس كريم الداكن، أليس كذلك؟

    ومن المنطقي أيضًا بالنسبة لي أنه إذا كانت الساعة 3.30:XNUMX في لوس أنجلوس، فهذا هو وقت النوم، أليس كذلك؟

  536. مرحبًا يهودا،

    لقد تحققت مرة أخرى، لقد كنت على حق وكنت مخطئًا - لقد راهننا على جسيم الكتلة المظلمة... لذلك يبدو أن الآيس كريم الخاص بي ذهب إليها!
    عندما تدخلت معك، بنيت على حقيقة أنهم أغلقوا مخلوق هيغز في نطاق 125 GEV، وما تبقى هو فقط زيادة اليقين في القياسات إلى خمسة سيجما. بالنسبة للجسيم ذي الكتلة المظلمة، سأحتاج إلى الكثير من الحظ لتناول الآيس كريم في العام المقبل...

    ؟؟؟؟

  537. إسرائيل،

    تكتب "ليس من الواضح بالنسبة لي كيف يمكن إنكار مبدأ ماخ. بعد كل شيء، يكفي أن تدور في مدينة الملاهي وتلتقط مقطع فيديو للنجوم أعلاه. إذا نظرت إلى الفيديو، ألن تكون قادرًا على معرفة من خلال الفيديو فقط مقدار دوران رأسك أثناء الدوران؟"

    كيف تثبت من هذا الموقف أن الشيء الذي تتسارع بالنسبة له الجسيمات الأولية التي تشكل رأسك الدوار هو كتلة النجوم البعيدة (باستخدام آلية غير معروفة حتى ماخ نفسه لم يتمكن من اقتراح ماهيتها) وليس مجالات الجاذبية الذاتية للجسيمات الأولية التي تشكل رأسك (باستخدام الآلية التي اقترحها مئير عميرام)؟

    بالمناسبة، كحجة جانبية: لاحظ أنه لا يمكنك استنتاج أي شيء من حالة حركة النجوم البعيدة لأنها حتى لو كانت تتحرك بسرعة الضوء بشكل عمودي على الخط الذي يصل بينك وبينها (أو في أي اتجاه آخر) اختر)، فسيظلون يظهرون لك في حالة سكون بسبب المسافة الهائلة التي تفصلهم.

  538. ترشيد
    نحن نراهن على جسيم الكتلة المظلمة

    كما قلت في 25.1.2012 يناير XNUMX
    "التنبؤ هو أن الجسيم المسؤول عن "الكتلة المظلمة" سيتم العثور عليه هذا العام في عام 2012" نهاية الاقتباس.
    جسيم الكتلة المظلمة هو جسيم الأجنحة
    بوزون هيغز هو الجسيم الذي من المفترض أن يعطي كتلة للجسيمات الأولية. لا يبدو لي أن هذا كان القصد، ويبدو لي أنك تحاول انتزاع الآيس كريم مني، أنا الرجل البريء، بادعاءات كاذبة، ناهيك عن الغش.
    سأطلب من المعلقين أن يذكروا ما إذا كان اكتشاف بوزون هيغز، في نظركم، سيكون أيضًا دليلاً على اكتشاف جسيم الكتلة المظلمة، وبسبب ذلك أيضًا أتقاضون مني ثمن الآيس كريم؟
    انتظر ردك. وأتعهد بالحصول على رأي الأغلبية.
    يوم جيد
    سابدارمش يهودا

  539. يهودا – لا تقل أبداً أبداً

    صحيح أنه يبدو من السخافة أن يكون للجزيء زخم سلبي (درجة الحرارة)، ولكن عندما تستمر في التفكير في الأمر، فمن الصعب التأكد. في العام الماضي، اكتشف في LHC أن جسيمات المادة المضادة تتصرف بشكل مختلف عن جسيماتها التوأم. كيف ستبدو العين المؤقتة في عالم المادة المضادة؟
    هناك مشكلة البرودة الفائقة في التحولات الطورية، حيث يمكن أن يتجمد الماء عند درجة حرارة C-15، في ظل ظروف معينة. ما هي الظروف التي تؤدي إلى الصفر المطلق اليوم؟ هل يمكن التأثير عليهم؟ تغيير الصفر المطلق؟

    مرفق رابط حول المادة المضادة. كان يُعتقد سابقًا أن المادة المضادة هي الفرق في الشحنة الكهربائية. ومنذ ذلك الحين تم العثور على نيوترون مضاد وتوسع التعريف:
    https://www.hayadan.org.il/anti-helium-2704115/

    بالمناسبة، قرأت في مجلة Scientific أنه من المتوقع أن يعلق الباحثون في Tevatron (الذي تم إغلاقه بالفعل) على اكتشاف هيغز في مارس... هل يمكن أن يكون الآيس كريم قادمًا؟
    ؟؟؟؟

  540. مئير.
    هل تدرك المشكلة التي قد تكون موجودة في نموذج القصور الذاتي؟ إذا كان قرصان يدوران بالنسبة لبعضهما البعض في الفضاء، بعيدًا عن أي كتلة، فكيف يمكننا أن نحدد بشكل مؤكد أن القرص A في حالة سكون والقرص B يدور؟ وبنفس الوزن: إذا كان الجسم A يتسارع بالنسبة إلى الجسم B، فلماذا لا نعكس المعادلة ونقول إن الجسم B يتسارع بالنسبة إلى الجسم A؟
    أتصور أن هناك حلاً، لكني أود أن أسمع منك أولاً.

    إسرائيل القواد.

  541. يوفال، يهودا

    نعم، بالطبع هذا استثناء للفيزياء. في رأيي، هناك "هنا" (سنسميها نقطة الانفجار - في حالة وجود اتصال بين ذلك الجسيم الغازي والفوتون) اتصال مباشر بمسألة التناظر. في رأيي، من بين أمور أخرى، في هذه اللحظة الحرجة، يحدث كسر التماثل.
    على أية حال، يوفال، فيما يتعلق بطلبك للحصول على التفاصيل، ربما في بعض الأحيان.

  542. يتم تعريف الصفر المطلق عمومًا على أنه عدم وجود حركة في الجزيئات. فكيف يمكنك تحديد درجة الحرارة المنخفضة؟ والسرعة السلبية؟ يبدو لي أن هناك شذوذًا في الفيزياء هنا.
    مساء الخير
    سابدارمش يهودا

  543. أرييه سيتر,
    إن انكماش لورنتز مقبول لدى الفيزيائيين وهو عامل حاسم في النظرية النسبية على الرغم من أننا لا نعرف أسبابه.
    معنى كلامك واضح بذاته، لكني لست واضحا بشأن العلاقة مع ريشا.

  544. R.H. Rafai.M،
    عندما تتحدث عن درجة حرارة أقل من الصفر المطلق، فإنك تخرج عن حدود الفيزياء. إنه يُظهر التفكير الإبداعي الذي أشجعه بشكل عام. من الصعب بالنسبة لي أن أرى إلى أين تتجه فكرتك. لذلك، إذا كان بإمكانك توضيح المزيد، فقد يساعد ذلك.

  545. يوفال - لا أعتقد أنه من الضروري وصف آلية تقلص فيتزجيرالد لورينز، لأنه إذا قمت بالقياس من الجسم المتحرك نفسه أو من جسم يتحرك بالتوازي معه - لا يوجد انكماش.

  546. يوفال

    لا أعتقد أن هناك مشكلة مع بقية الأشياء التي كتبتها (باستثناء الفوتون).
    ما رأيك في الفكرة التالية: أن هناك جزيئات غازية (أو في حالة تراكم بلازما - لم تغلق بعد على هذا) درجة حرارتها أقل من الصفر المطلق؟ في رأيي أن مثل هذا الجسيم موجود ويتفاعل مع الفوتون. الرابطة بين الجزيئين تخلق النيوترينو. (صحيح أن هذه الصياغة للفكرة هي أيضا إشكالية وليست دقيقة بما فيه الكفاية، لكن الفكرة في مهدها فقط، وحتى لو كانت دقيقة فلا تزال هناك مشكلة اختراع التكنولوجيا لإجراء التجربة).

  547. لفهم الآلية التي تؤدي إلى انكماش لورنتز أثناء الحركة، يجب على المرء أولاً أن يفهم الآلية التي تخلق الحركة. تعلمنا من أينشتاين أن ظاهرة مبدأ الثبات (الذي صاغه غاليليو ويسمى اليوم "قانون نيوتن الأول للحركة") وظاهرة قوة الجذب بين الكتل هما في الواقع ظاهرتان متطابقتان. لذلك، هناك سبب لافتراض أن نفس مجال الجاذبية الذي يحيط بجسم ضخم ما هو المسؤول أيضًا عن الحركة المستمرة للجسم في الفضاء. لقد تمت مناقشة هذا بالفعل هنا من قبل مئير عميرام، الذي قدم أيضًا الصيغ الرياضية المناسبة، وسأكون ممتنًا لو شاركنا التفاصيل.

    لكي نفهم ما هي الحركة، يجب علينا أن ننظر إلى ما هو أبعد من حدسنا. من وجهة نظرنا، يحتوي أي جسم في داخله على كل ما هو ضروري لتحديد خصائصه، بما في ذلك الكتلة. لكن هذا الدليل ينبع من الطريقة الملائمة لنا لإدراك الواقع. إن العلاقة المتبادلة بين جوهرنا المهم والجوهر الذي نوليه أهمية في الواقع من حولنا ليست بالضرورة الشيء الأساسي. المثل ما هو مماثل ل؟ عندما نقوم بأي عمل بدني، فإننا لا ندرك عمل كل خلية عضلية وكل خلية عصبية تشارك في العملية، وكذلك لا ندرك تدفق الدم أو الخلايا العصبية والمشابك العصبية في دماغنا التي تخلق الأفكار؛ كل ما يهمنا هو النتيجة النهائية، وهي تنفيذ الإجراء. هكذا رأينا أنفسنا وكل الأجسام من حولنا ككيانات محددة ذاتيا ومنفصلة عن الفضاء المحيط بها ليس بالضرورة تصورا صحيحا للأشياء. فإذا نظرنا إلى الأجسام كجزء لا يتجزأ من بيئتها وبيئتها كشريك فيها وفي عملية حركتها، فيمكننا أن نفكر بطريقة لا تنشأ بها ظاهرة ثبات الجسد من الجسد. وحيد. نحن نعلم أن الجاذبية هي عملية يشارك فيها عامل خارجي (ننسبه إلى جسم آخر) في حركة الجسم. ولذلك يمكن البحث عن تورط طرف خارجي أيضا في ظاهرة الثبات.
    لقد قيل الكثير هنا عن نموذج LaSage والموقع، وقد نُسبت إليهما طبيعة جزيئية. ولكن بما أن وحدة الضوء لها أكثر من مثيل، فإن هناك مجالًا أيضًا للحديث عن أكثر من مثيل لجسيمات La Sage أو الأثير. سأحاول التحدث عن ذلك لاحقا.

  548. R.H. Rafai.M،
    صحيح أن الأمر يبدو غامضا بعض الشيء، لكن لدي شعور بأنني فهمت روح الأشياء.
    نحن نعرف نمطين مختلفين للضوء، الموجة والجسيم. وعلى الرغم من أنهما يبدوان متناقضين مع بعضهما البعض، إلا أن وجودهما تم تأكيده من خلال الملاحظات. أعتقد أنه في تجلياته كموجة ضوئية يتحرك بسرعة تعرف باسم سرعة الضوء، لكنه كجسيم يخضع لمجموعة مختلفة من القوانين. على سبيل المثال، يمكن أن يُحتجز داخل الإلكترون ويتسبب في تغيير حجمه. أرى الإلكترون كقشرة رقيقة تحيط بالبروتون. يتسبب الفوتون المحاصر بداخله في زيادة سماكته و"يسعى" إلى العودة إلى سمكه الرقيق عن طريق زيادة قطره. ولكن مع هذا القطر المتزايد يصبح أقل استقرارًا وينتقل إلى حالة أكثر استقرارًا أثناء إطلاق الفوتون. والفوتون المنطلق، إذا لم يلتقطه إلكترون آخر، يعود إلى شكله المتموج. أحد الاستنتاجات التي قد تنشأ من هذا هو أن الإلكترون هو "خزان" من الفوتونات، وبالفعل يُظهر الإلكترون أيضًا صفة مزدوجة - جسيم وموجة.
    على الرغم من أنني لم أجب على كلامك بالضبط، إلا أن نيتي كانت التوسع قليلاً في الحديث عن الفوتون.

  549. يوفال
    "سرعة الضوء ثابتة" عبارة إشكالية.
    وهذا يؤدي أيضًا إلى أخطاء مثل - "قد لا تكون سرعة الضوء ثابتة".
    في رأيي يجب أن نكون دقيقين ونقول أن سرعة جسيم الضوء ثابتة.
    المشكلة تكمن في عبارة "سرعة الضوء". إنها ليست سرعة الضوء، بل الطول الموجي للفوتون. سرعة الفوتون محدودة. المشكلة هي عندما يمر الفوتون عبر وسط ما...
    طول موجي معين للفوتون يخلق -إذا صح التعبير- "وهمًا بصريًا" (مثل تأثير دوبلر على سبيل المثال)، بسبب استحالة الأجهزة الموجودة في تمييز أو فصل الطول الموجي للفوتون عن الطول الموجي للإشعاع غير المعروف 🙂
    (يبدو الأمر غامضًا بعض الشيء ولكن أتمنى أن تفهم المعنى)

  550. لماذا سميت النظرية النسبية بهذا الاسم؟
    أطلق عليها أينشتاين اسمًا آخر: "نظرية الثبات" (مترجمة بشكل فضفاض من الثبات). قرر أن يأخذ سرعة الضوء كثابت لأي إطار مرجعي، ويرى إلى أين سيؤدي ذلك. وقد أعطاها ماكس بلانك الاسم المقبول اليوم، ويقال إن أينشتاين لم يعجبه.
    قدم هندريك لورنز تفسيرا للنتيجة المفاجئة لتجربة مايكلسون مورلي: "الأجسام تنكمش مع حركتها"، لكنه لم يقدم الآلية التي تحدث هذا الانكماش. لقد تغلب أينشتاين على العقبة، والباقي هو التاريخ.
    في الواقع، تقبل نظرية أينشتاين انكماش لورنتز، بل وتستخدم صيغه بدقة شديدة. وبما أن لورنتز بدأ من فرضية أن سرعة الضوء ليست ثابتة، ولكنها تظهر لنا بهذه الطريقة فقط بسبب الانكماش، وبما أن أينشتاين اعتمد على صيغ لورنتز، فإن أينشتاين قام فقط بتوحيد الفيزياء وفقًا لهذه الظاهرة.
    وبما أن ملاحظاتنا تعتمد كثيرًا على البصريات، وبما أن سرعة الضوء تبدو ثابتة، فإن الملاحظات تتوافق مع استنتاجات نظرية الثبات/النسبية وهذا ما رسخ الاعتقاد بأن سرعة الضوء في الفراغ ثابتة في كل مكان. في الكون.
    ومن تحديد سرعة الضوء حصلنا على تنبؤ جميل ودقيق للعديد من الظواهر، ولكن بقينا أمام مفارقات مثيرة للقلق، وآخرها، ولكن الأقوى على الإطلاق، هي الطاقة المظلمة. سيتم تقديم حل لجميع هذه المفارقات عندما يتم العثور على الآلية التي تسبب تقلص لورنتز

  551. تجربة ماخ:
    خذ أسطوانة كبيرة من المعدن الثقيل للغاية (سنكتفي بالرصاص، لأن أسعار الذهب والبلاتين فقط ترتفع). قم بإنشاء مساحة فيه حيث يمكنك وضع دلو فيه. قم ببناء هذا النظام بحيث يمكن للأسطوانة أن تدور حول الدلو ولكن الدلو سيكون أيضًا مستقلاً عن الدوران (الرسم سيشرح ذلك بسهولة). داخل الاسطوانة، على سقف المساحة، يتم وضع كاميرا ترى الدلو. يمتلئ الدلو بسائل ثقيل (ربما زئبق) وتسجل الكاميرا حالة السائل الموجود في الدلو.
    للتجربة مرحلتان: في المرحلة الأولى يتم تثبيت الدلو وتدوير الأسطوانة حوله. وبحسب ماخ فإن الكتل الموجودة في الكون كله تؤثر على الدلو في دورانها النسبي نحوه. لذلك، يجب أن يكون للأسطوانة الثقيلة تأثير أيضًا. علاوة على ذلك فإن تأثير الأسطوانة الثقيلة أكبر من تأثير كل الكتل الموجودة في الكون، لأنها قريبة من برج الدلو. في الخطوة الثانية، قم بالعكس: قم بتثبيت الأسطوانة وتدوير الدلو. بعد ذلك يتم دمج صور سطح السائل في المرحلتين.
    ملاحظة: إذا تبين أن دوران الأسطوانة كان له تأثير أقل من دوران الدلو، فيمكن الافتراض أن كتلة KHA تدخلت وعطلت التجربة. ولذلك يمكن محاولة القيام بذلك في مكان بعيد عن مكة. ومن الواضح أن تأثير KAHA حتى على مسافات كبيرة (بل إن البعض يدعي أنه يصل إلى ما لا نهاية)، ولذلك يجب إجراء التجربة عدة مرات، وفي كل مرة على مسافات مختلفة عن KAHA.
    من ناحية أخرى، من الممكن أيضًا توفير نفقات التجربة والقول ببساطة أن ماخ كان مخطئًا.

  552. تصحيح الخطأ (ربما لا أحد يتابع، ولكن لحسن التدبير...)
    لقد جادلت هنا عدة مرات بشغف بأن الضوء الذي يصل إلينا من المجرات البعيدة يتسارع في طريقه. لقد تلقيت بريدًا إلكترونيًا من أحد المعلقين هنا الذي قام بالحساب ووجد أن الضوء يجب أن يتباطأ بالفعل (ليكون متسقًا مع ادعائي بأن سرعة الضوء ليست ثابتة). لقد حسبت أيضًا ووجدت أنه كان على حق. الضوء في رحلته التي تستغرق مليار سنة من المجرات البعيدة يتباطأ في طريقه إلينا، وهذا ما يسبب ظاهرة الانزياح الأحمر التي تم تفسيرها وكأن المجرات تتسارع.

  553. ر.ح.

    صحيح، ولكن على عكس يهوذا، بالضبط فيما يتعلق بالنقطة التي أثرتها، كتبت:

    "وهنا أعلم أنني يجب أن أكون مخطئا. لا، من المستحيل أن أفكر في الأمر ولورينز لم يفعل ذلك".

    وأضفت:

    "ولكن حتى أعرف أين الخطأ، يجب أن أحاول حل اللغز، حتى أتمكن من الموت سعيدا."

    ثم كتبت الفكرة بأكملها هنا على الموقع.

    فما رأيك: أين الخطأ؟ هل يمكن، منطقيًا، أن يكون هناك نظام سكون لموقع ما في كون لا نهائي ومتناحٍ؟

    اسد.

    سيكون من الأسهل التفكير في الأمر إذا تخيلت مركبة فضائية على مسافة ساعة ضوئية من جهاز الإرسال. في مثل هذه الحالة، يمكن للإشارة A أن تسبق الإشارة B بدقيقة كاملة على سبيل المثال. بالنسبة للإشارة، ما يمكنه اكتشافها هو كاشف ذو سرعة خطية معينة. نظرًا لأن الهوائي الأول يتحرك بالنسبة إليه بسرعة عالية/منخفضة، اعتمادًا على أي جانب من القرص الذي تنظر إليه، فإن السرعة العرضية تكون منخفضة/عالية وفقًا لذلك، ووفقًا لفكرتي يمكن استخدامه كـ "فخ" للإشارة، لأنها بالنسبة إليها تتحرك بسرعة أقل من سرعة "الهروب". يتم توصيل الهوائي بمؤقت يسجل وقت وصول الإشارة (في المثال الذي قدمته، بالمناسبة، لا تحتاج إلى مؤقت. يمكنك رؤية الإشارة أو سماعها مثل أي إشارة راديو أخرى).

    وبعد دقيقة واحدة تصل الإشارة الثانية، التي يلتقطها النظام العادي.

    قم بتضييق نطاق الأمر كله، وهنا جهازي.

    ويمكن فعل الشيء نفسه مع استقبال الإشارات من نبضات بعيدة، لكن الأمر قد يكون معقدًا. النظام الذي وصفته غير كامل بالمناسبة. هذه مجرد فكرة عامة

    هيا، دعنا ننتقل إلى النماذج المهمة حقًا.

    إسرائيل من لصوص كريات.

  554. إسرائيل شابيرا - لا تبدو فكرة الهوائي القرصي الدوار، والذي بالنسبة لك هو نفسه من حيث تأخير الاستقبال كهوائي يتحرك بعيدًا عن جهاز الإرسال بنفس السرعة العرضية للقرص الدوار، إلي. بعد كل شيء، يحتاج كل هوائي إلى الاتصال بجهاز الاستقبال حيث سيتم استقبال الإشارة وقياس الوقت. أجهزة الاستقبال نفسها على نفس المسافة بالنسبة إلى جهاز الإرسال وحقيقة أن أحد الهوائيات يدور - لن يؤثر ذلك. يجب أن ينقل الهوائي الدوار الإشارة المستقبلة إلى جهاز الاستقبال من خلال عمود سيكون التوصيل الكهربائي للإشارة المستقبلة وطريقة النظر إليها - لن يتسبب دورانه في تأخير الإشارة - أي سيتم استقبال الإشارة في بطريقة متزامنة في كلا المستقبلين.

  555. اليوبيل,
    من المؤسف أنه من الأسهل أن تغضب ومن الأصعب أن تسامح، لذلك في رأيي أعظم الناس في التاريخ هم الأشخاص الذين نادوا بالفرج في موقف كان من الأسهل أن ينفعلوا ويتحمسوا مثل غاندي، مارتن لوثر كينغ، نبدأ بعد التلانا والسادات ورابين.

    إسرائيل،
    أعتقد أننا بعنا. ملاحظة واحدة فقط حول الفقرة 3. لقد ادعيت في الماضي أنك تنحني لماكسويل لورينز وآخرين. لقد زعمت أيضًا أن نظرية LS الغريبة، إن لم نقل بعيدة المنال، حقيقية لأن الكثير من الناس، بما في ذلك نيوتن، فهموا أن هناك نوعًا من الجاذبية هنا ولم يبق سوى مشكلة الاحتكاك. ولكن من ناحية أخرى، أتيت إلى هنا وقتلت تجربة MM، التي تعتبر التجربة رقم واحد على الإطلاق في الفيزياء (حسنًا، على الأقل في المراكز العشرة الأولى) وقد اعتمدها المجتمع العلمي الفيزيائي بأكمله وقبلها، وبعد ذلك هي، النظرية النسبية. من الجنون الحكم على الموضوع، لكن لدي شعور بأنه من غير المرجح أن يكون كل الفيزيائيين في القرن العشرين أغبياء ولم يروا في ذلك تجربة غير منطقية كما تزعم.
    وكذلك الحال بالنسبة للقسم الثاني، فإذا كان من الواضح أن هناك أثيرًا ولا يمكن تجاهله كما تقول، فكيف يتجاهله جميع الفيزيائيين ويعتبر مثالًا كلاسيكيًا لنظرية مؤقتة تقول فشل؟
    لقد بدأت تبدو مثل صديقنا يهودا الذي يرى في الفيزياء كلها نوعًا من المؤامرة السرية التي تهدف إلى دفع الكتلة المظلمة، كما لو أن شخصًا ما يمتلك مخزونًا من الكتلة المظلمة. من المؤسف أنه لا يوجد لأنني سأشتري، والكثير.

  556. حسنًا، يوفال، أنا أفهم مدى الحساسية. دعونا نرى كلانا متحررين.

    حسنًا، آبي، أتفهم حساسية WordPress. لن أكتب بعد الآن "أينشتاين".

    حسنا، الفترة. ويجب أن يكون هناك ركن لمن "ليس لديهم أدنى فكرة".

    ر.ح.

    1. لا توجد مثل هذه المشكلة. دعونا نبالغ في البيانات: يقال أن أجهزة الكشف موجودة مع رائد فضاء على بعد ساعة ضوئية واحدة بالضبط من الأرض. إذا تمكنت من إرسال الرقم 7 إليه، وإذا كانت هناك تعليمات، اضرب الرقم الذي تلقاه في 5 وأرسله مرة أخرى إلى هيئة الصحة بدبي، واستلمت 35 خلال ساعة ونصف، فقد تمكنت من إرسال إشارة مع المعلومات أسرع من الضوء.
    قم بتقليل كافة البيانات، وبذلك تكون قد حصلت على تجربتي.

    2. نقطة جميلة. لكن لا تنسوا أمرين: الأول، أنني لا أعرف إذا كان الضوء يتحرك بجميع السرعات وفقًا للنموذج. وقد يتحرك بسرعات معينة، حسب سرعات المذبذب الأصلي. والثاني، المهم، هو مدى السرعات التي يحساس لها الكاشف، أو الراصد. مثلما لو حاولت قياس المسافة إلى قوس قزح، فسوف ترى دائمًا أنه على مسافة معينة وثابتة منك أيها المشاهد، بغض النظر عن مكان وجودك.

    3. معلومات هامة ومفيدة. وهذا وحده جعل فكرة مراجعة النظراء بأكملها تستحق العناء. شكرا.

    يمكنك بالطبع الاستمرار في الفكرة، وتوضيح سبب اعتقادي أنها تحل بعض المشكلات الكمية، وما إلى ذلك. ولكن يبدو لي أننا لخصنا ذلك، إلا إذا كنت ترغب في التوسع. طلبي لك وللجميع إذا وجدت خطأ في النقاط التالية:

    1. كانت نظرية الأثير لماكسويل تعتبر وهمية إلى حد ما، لولا الصيغ الشهيرة وتجربة هيرتز.
    2. بما أنه تم إثباته فمن المستحيل تجاهله والقرار فجأة بعدم وجود موقع.
    3. تجربة MM غير مقبولة منطقيا. لا يمكن أن يكون هناك نظام راحة لموقع ما في كون لا نهائي ومتجانس، تمامًا كما لا يمكن أن تكون هناك نقطة مركزية لللانهاية.
    4. يكفي أن يتبين أن نيوترينو واحد فقط تمكن من تجاوز سرعة الضوء لينهار النسبية.
    5. على النقيض من التشابك الكمي، حيث لا مفر من الاستنتاج بأن معلومات الدوران تمر بشكل فوري، في حالة الضوء هناك على الأقل احتمال نظري بأن سرعة الضوء هي نفسها في جميع الأنظمة المرجعية، وهذا دون تمديد الوقت في الأنظمة غير المتسارعة.

    فيما يتعلق بالنقطة 5: هل هناك أي تجربة معروفة حول تمديد الوقت في الأنظمة التي لا يتم تسريعها أو التي تتضمن تأثير دوبلر؟ تجربة الطائرات وإصدار الميون في الأنظمة المتسارعة.

    والأكثر من ذلك، الآن بعد أن أصبحت الفكرة واضحة، وتطور موضوع التجربة، هل لدى أحد اقتراح لتحسين التجربة التي اقترحتها؟

    وبشكل عام: هل هناك أي خلل منطقي في كل الفكرة التي طرحتها ربما فاتني؟

    هذا. يمكنك الانتقال إلى مواضيع أكثر أهمية، ولكن على الأقل 9 تعليقات أخرى، أليس كذلك؟ يجب أن تتجاوز 500.

    مئير.
    أشعر بالفضول الشديد لمعرفة ما تقوله عن مسألة الارتباط بين القوة الطاردة المركزية والنجوم البعيدة. لا أفهم كيف يمكن تجاهل ذلك، لكنني أيضًا لا أفهم كيف حل أينشتاين المشكلة، لذا فمن الواضح أن المشكلة معي.

  557. أبي، أنصحك بفتح موقع على شبكة الإنترنت وتسميته "مبدعي الأفكار" حيث يستطيع كل من "لديه فكرة" الكتابة.

  558. مصافحة؟ ليس في التجسد الحالي.
    لقد تخليت عنه تمامًا. يتظاهر بالقراءة ثم يتبين أنه ينظر فقط إلى شكل الحروف. حتى أنه جعلني أشعر أنني كنت مخطئًا لأنني لا أكتب بوضوح. لذلك سمحت للآخرين بالقراءة، ففهموا جيدًا. كانت معدتي ممتلئة للتنفيس عنه، وكان الدافع عبارة عن جملة وجدها في المادة التي أرسلتها إليه واستغلها للاستفادة منها بشكل سخيف. أعني، ليس فقط أنه لا يهتم بقراءة الأشياء بشكل صحيح ويجعلني أشعر أن هناك خطأ ما في قدرتي على التعبير، لكنه يرى أشيائي ليست أكثر من مادة خام للنكات.
    دعه يذهب للبحث عن أصدقائه. أنا متأكد من أنه ليس واحدا منهم.

  559. إسرائيل،
    1) كيف ستعرف أن ما تراه ليس بسبب تأثير نسبي على الزمن؟
    2) إذا كانت سرعة الضوء غير ثابتة ويوجد فوتونات في جميع السرعات فلا نرى الفوتونات السريعة لأنها سريعة ولكن أين الفوتونات البطيئة؟ لماذا تكون سرعة الضوء ثابتة حتى من الأسفل؟
    3) أنظر هنا: http://www.n3kl.org/sun/index.html . توجد هنا بيانات تجمعها ناسا عن الشمس من محطات من الأرض والأقمار الصناعية. ربما يمكنك استخراج شيء من فروق السرعة والوقت بين المحطات؟ على سبيل المثال، هل يتم قياس الانفجارات في وقت سابق (بعد تطبيع إحداثيات الموقع) في المحطات السريعة مقارنة بالمحطات البطيئة؟ لا أعرف إذا كان ذلك ممكنًا ولكن قد يكون الأمر يستحق المحاولة.

    يوفال وإسرائيل
    أعتقد أنه كان هناك نوع من الاعتذار هنا. دعونا نصافح الأيدي الافتراضية ونمضي قدمًا. ستكون إسرائيل أكثر حساسية قليلاً، وسيكون يوفال أقل حساسية قليلاً. من المؤسف أنه حتى تكون هناك مجموعة مناقشة مثيرة للاهتمام وذكية هنا تناقش العديد من المواضيع، فإنك تبدأ بهراءك. إذا واصلتم على هذا النحو فسوف ينتهي بنا الأمر مع الشياطين والأشباح فقط.

  560. يوفال، لقد أخر الجستابو ردي على التحقيق.
    أعجبني "ما الذي كنت تفعله إلى جانب محاولة التفكير بشكل غير ذي صلة واستخدام الأفكار المشوهة التي لا تخصك بغرض الإساءة إلى أولئك الذين لا يبدو أنهم منضبطون بدرجة كافية لتناسب ذوقك المتعجرف؟"

  561. يوبيل، يوبيل، يوبيل.

    في الوقت الذي كتبت فيه لي: "أكثر من إصراري على التحقق مما (لا) يفهمه جمهوري، أحاول أن أفهم لماذا لا أستطيع أن أشرح".

    أعتقد أن لدي إجابة.

    أنت تفترض أن الناس يعرفون ما يدور في ذهنك، دون أن تتواصل معهم.

    تريد مثالا؟ كم مرة سألتني عن رأيي في موديلك، ونسيت شيئًا واحدًا: أنه ليس لدي أي فكرة عن موديلك. لقد طلبت معرفة ما أقوله عن نموذج المادة المظلمة، لكنك لم ترسل لي (حتى الآن) أفكارك حول تلك المادة المظلمة. أنت لم تكتب على الموقع أيضا.

    أعلم أنك تتواصل مع العديد من المشاركين، وتعرف تفاصيل شخصية عنهم وتتبادل الأفكار معهم. ليس لدي أي فكرة عن هوية R.H.، نقطة، أو شبح، أو والدي G. لا أعرف كم عمرهم، وأين يعيشون (أعتقد أن R.H. موجود في بوسطن، بسبب تشارلز).

    عندما بدأت الكتابة على الموقع، اعتقدت أن يهودا يجب أن يكون عمره 25 عامًا، فقط بعد أن شاهدت الفيديو أدركت أن هذا شخص كبير في السن.

    لهذا السبب عندما بدأ مئير الكتابة، اعتقدت أنه قد يكون طالبًا في المدرسة الثانوية محبًا للعلوم، وكنت قلقًا من أنه قد يكون لديه انطباع بأن ما نناقشه هو الاتجاه السائد، وهذا ما سيتم تثبيته في رأسه. ومن هنا التحذيرات. من الواضح، بعد أن اتضح لي أنه شخص بالغ، يتحكم في المادة، وله أفكاره ومدونته الخاصة، من الممكن أن يخفف من القيود ويصبح جديًا.

    أبذل قصارى جهدي للإشارة طوال الوقت إلى أن أفكاري هي مجرد تكهنات، حتى يتم إثباتها، من خلال الصيغ أو التجربة الأفضل، لمكانها الصحيح في لالا لاند. أعتقد أن هذا صحيح بالنسبة لنا جميعا.

    "الاستخدام المشوه لأفكار ليست ملكك بغرض الإساءة إلى أولئك الذين لا يبدون منضبطين بدرجة كافية لتناسب ذوقك المتكبر؟"

    مرة أخرى، ليس لدي أي فكرة عما تقصده ومن تقصد. للأشباح؟ لا أعرف كم عمره، كل ما أعرفه هو أنه بدأ يعالجني فجأة دون أي استفزاز مني. ثم توصلنا إلى هدنة، لكنه تعافى قليلاً.

    لأقول لك الحقيقة - إذا كنت تقصده، فلا أعتقد أنه قد جرحني على الإطلاق. ربما مسليا. رأسه منتفخ وأنا أختلس النظر، والأمر هو أن رأسي كذلك أيضًا. أيها الأشباح، أنتم مدعوون للخروج من تراجعكم العقلاني والرد والاختفاء.

    "يمكن لأي فيزيائي أن يبني هذا من القصور الذاتي." ولكن هذا هو بالضبط السؤال: كيف يعمل القصور الذاتي؟ إن نظرية أينشتاين ليست بديهية، أو كما تقول، بدائية. ماخ نعم. ولم نصل بعد إلى صيغة مئير عميرام. هل يجب أن أخمنها أيضًا؟

    وبالإضافة إلى ذلك، أنا لا أفهم مشكلتك الشخصية معي. هل أقوم بعملية غسيل دماغ؟ بالكاد أعرف كيف أغسل الأطباق في المنزل. هل أنا في دائرة الضوء؟ من يمنعك من مراسلة من تريد إذا كان مهتماً بالرد. على عكس ما ورد في إحدى النقاط، لا توجد مشكلة تتعلق بوقت البث هنا. إذا لم يكن شخص ما مهتمًا بشيء ما، فيمكنه تخطيه. يرد دوف هينيس من وقت لآخر، لكن لا يستجيب الكثيرون لردوده. حاولت مرة، وأدركت أنني لم أفهم ما يقوله، فتوقفت. أجد اهتمامًا كبيرًا بنظرية مئير، وسأكون سعيدًا جدًا إذا تمكنا من تطويرها ومناقشتها. من يزعجه؟ وسأكون سعيدًا جدًا أيضًا بمناقشة النموذج الخاص بك، إذا كنت أعرف أو أفهم ما هو عليه.

    لذلك، باختصار، يوفال، كفى مع التشيليبا. إذا كنت تريد مني أن أرد على كلامك في المستقبل، فاذكر ذلك بوضوح، فأنا لا أقرأ الأفكار، ولا أريد أن أؤذيك أو أي شخص، وليس لدي أي اهتمام بمعارك الأنا.

  562. ر.ح.
    أولاً، دعونا ننهي موضوع L.S.

    الكتلة حسب L.S. إنها شبكة ذات ثقوب. البعض يمر والبعض الآخر لا.
    الثقب الأسود هو جدار خرساني. لا شيء يمر
    تم اختبار النظرية من قبل شامنا وسيلتا، بما في ذلك نيوتن نفسه. لم يزعم أحد أن الجاذبية لن تحدث (على حد علمي). كما لم يدعي أحد أن الشكل الهندسي كان مهما. وكانت المطالبات مختلفة، وأهمها مشكلة الاحتكاك. أعتقد أن فكرتي يمكن أن تحل هذه المشكلة، ولكن كما ذكرت عدة مرات، فهي مجرد عرض جانبي. القضية المهمة بالنسبة لي هي العلاقة. وقد اقترب هذا الآن.

    "من الناحية الفنية، سيكون هناك العديد من المشاكل لأن المستشعر المتحرك سيكون على مسافة مختلفة من مصدر الضوء عن المستشعر الساكن."

    هذا كل شيء، لا. والفكرة هي وضع كلاهما قريب جدًا من بعضهما البعض.

    كانت فكرتي الأولى هي وضع كاشفين قريبين من بعضهما البعض، أحدهما طبيعي والآخر بالقرب من أنبوب أشعة الكاثود حيث يمكن للإلكترونات الوصول إلى سرعات عالية جدًا. أطلق فوتونًا على الإلكترون المتحرك، ثم، بسبب تأثير كومبتون وانحراف الإلكترون، قم بقياس موقع ووقت تأثير الإلكترون المنحرف ومقارنته بزمن تأثير الكاشف المجاور له، حيث يكون الوضع الطبيعي يضرب الفوتون.

    أما الفكرة الثانية فكانت معرفة ما إذا كان من الممكن فعل شيء ما باستخدام مسرعات الجسيمات. ولهذا التقيت بجيل تريفيش من مختبر مسرع الجسيمات بجامعة كاليفورنيا. لن أزعجك بالتفاصيل، لكن خلاصة القول هي لا. ليس الآن.

    أما الفكرة الثالثة فهي تقنية كما أحب. حاول القراءة ومعرفة ما إذا كان هناك أي خطأ منطقي أو فني فيه.

    ما نريده هو وجود كاشفين في نفس النقطة، أحدهما في حالة سكون والآخر يتحرك بسرعة. وهذا لا يغضب أينشتاين.
    يمكن القيام بذلك بالطريقة التالية: يرسل جهاز إرسال لاسلكي إشارة مستهدفة إلى نقطة تبعد 300 كيلومتر عنها. يوجد في النقطة جهازي استقبال: الأول له هوائي عادي، والثاني له هوائي على شكل قرص يوازي الأرض ويدور بسرعة هائلة. ما تم التوصل إليه هو أنه، فيما يتعلق بالإشارة التي تصل إلى جهاز الإرسال B، فإن السرعة العرضية للقرص هي نفسها بالنسبة له كهوائي يطير بنفس السرعة.

    الوقت الذي يستغرقه انتقال الإشارة من جهاز الإرسال إلى أجهزة الاستقبال هو حوالي جزء من الألف من الثانية. الموقتات التي اخترتها دقيقة إلى مستوى البيكو ثانية تقريبًا. أرغب في استخدام مؤقت الفيمتو ثانية، لكن هذه أكثر تعقيدًا.

    هذه هي الفكرة. تحقق، واكتساح، والعودة.

  563. قلت: "إن حركة جميع الكتل في الكون مرتبطة بالمادة المظلمة. إن الحركة المتبادلة بين الجماهير ليست سوى منتج ثانوي".

    فأجاب: "أولاً ينبغي عدم الخلط بين القراء بين الآراء المقبولة وآرائنا الخاصة. مبدأ ماخ، إليبا داماخ، هو كما قدمته. الإضافة: "إن حركة جميع الكتل في الكون مرتبطة بالمادة المظلمة". إنها فكرتك، ويجب الإشارة إليها، حتى لا تربك القراء الذين قد يعتقدون أنها الرأي السائد. بقدر ما أعرف، لا. أنا لا أقول إنها ليست فكرة مثيرة للاهتمام، ولكن حتى يتم إثباتها، يجب التعامل معها على أنها تكهنات"، وأغلقت الملف عليها.

    تخلص من "تكهنة" واحدة لإفساح المجال للآخرين؛ تجاهل رأيي الخاص لتعزيز آرائه الخاصة؛ ولتحذير القراء من أن رأيي ليس ثابتا في السائد، وكأن آرائه كذلك. الدهماوي! قواد الكلمات وغسالة الدماغ.

    وعلى عكس تكهناته، التي تملأ الفضاء بالكثير من الافتراضات الوجودية، فإننا نعرف بالفعل عن وجود المادة المظلمة.

    إسرائيل! لقد قدمت لك خصما، ولكن هنا فقط على الموقع. لن يكون هناك المزيد من رسائل البريد الإلكتروني الخاصة. لقد سألت كيف يحافظ محور العجلة الدوارة على اتجاهه في الفضاء. يمكن لأي فيزيائي أن يبني هذا من القصور الذاتي. إذا كنت تسأل ما الذي يخلق القصور الذاتي، فقد ربط ألبرت أينشتاين بالفعل القصور الذاتي بالجاذبية (القس ط. الهخليت) وأظهر مئير عميرام أيضًا صيغته الأصلية في هذا الشأن. أقدم الجاذبية/القصور الذاتي كتفاعل بين المادة المظلمة ونفسها، وربما سأستخدم أيضًا صيغ مئير في مرحلة الحساب. وماذا فعلت إلى جانب محاولتك التفكير بشكل غير ذي صلة واستخدام أفكار مشوهة ليست من أفكارك بغرض الإساءة إلى أولئك الذين لا يبدون منضبطين بما يكفي لذوقك المتكبر؟

  564. إسرائيل،
    أولا حظا سعيدا.
    ثانيًا، يبدو لي أنك ستحتاج إليه لأنه من الناحية الفنية سيكون هناك العديد من المشكلات حيث أن الحساس المتحرك سيكون على مسافة مختلفة من مصدر الضوء عن الحساس الساكن وستحتاج أيضًا إلى قياس مسافة دقيق جدًا جدًا حتى تتمكن من ذلك لتعويض تأثير المسافة (ما لم تقم بإجراء التجربة بين الأرض والمرأة المسلسلة وبعد ذلك سيكون تأثيره ضئيلًا ولكن من المحتمل أن تنشأ مشاكل فنية أخرى).

    نقطة أخيرة، بعد الانتهاء، على افتراض أنك لاحظت فارقًا زمنيًا، وأقمت حفلة، وفتحت الشمبانيا، ثم طرقت الباب. ومن سيقف هناك إن لم يكن أينشتاين. وما سيقوله لك هو: "جبل شابيرا، من الواضح أن نظام الدفع سوف يرى الفوتونات قبل ذلك، وهذا افتراض لأنه، كما زعمت في نظريتي النسبية الخاصة، في نظام الدفع الجبلي، سيتم تقصير المسافات وسوف يمر الوقت بشكل أسرع. علاوة على ذلك، يضيف ويقول: "لقد تم بالفعل إثبات هذا التأثير تجريبيًا بواسطة ساعتين ذريتين، إحداهما طائرة والأخرى ساكنة على حد ما أستطيع أن أتذكر، والتي للأسف لم تعد كما كانت في تلك السنة الرائعة" 1905، أظهر لك ر.ه. مرة واحدة في أحد تعليقاته."
    لا تنس أن تصنع له المسكر فهو يكره الشمبانيا.

  565. ر.ح.
    فرق الضغط ليس ريحًا، على الرغم من أنه يمكن أن يسبب الرياح. في كرة القدم التي يركلها أطفالك، هناك فرق كبير في الضغط بين الخارج والداخل، ولكن يتم التعبير عنه بالقوة، وليس الروح أو الطاقة. وهذه أيضًا هي الطريقة التي يتم بها التعبير عن الجاذبية، اللاساجية أو النيوتونية، في قوة الجذب. الكرسي الذي تجلس عليه يبذل أيضًا قوة، ولكن فقط إذا كانت هناك حركة في اتجاه القوة، يمكنك التحدث عن الطاقة، كما هو الحال في القارب الذي يتسارع باستخدام الشراع.

    مباشرة إلى الاختبار.
    يمكنك التقاط قوقعتنا إذا قمت بتحريك الأرض في اتجاه حركة القذيفة. فإذا كانت سرعة القذيفة 12 كم/ث بالنسبة إلى الأرض، فإنها ستفلت ولن تعود أبدًا. ومع ذلك، إذا قمت بتحريك الأرض بسرعة 1 كم/ث في اتجاه حركة القذيفة، فإن السرعة النسبية للقذيفة إلى الأرض ستنخفض إلى 11 كم/ث، وسيتم التقاطها.

    لذلك، إذا كان هناك أي شيء في هراءي، وقمت بإرسال شعاع من الضوء، يتكون من العديد من الفوتونات، وسيكون للمعدات النهائية نظامان للكشف مزودان بمؤقتات دقيقة، أحدهما يستريح بالنسبة لمصدر الإشعاع والآخر يتحرك بسرعة وبسرعة عالية نسبيًا لأول مرة، سيلتقط النظام الثاني فوتونات كانت ستكون شفافة له لو لم يتحرك، وهذا قبل أختها الثابتة.

    وبما أن النظام الأول يستقبل فوتونات تتحرك بسرعة الضوء، فإذا استقبل النظام الثاني فوتونات حتى قبل الأول (وسوف يرى الموقتون ذلك)، فإنه يستقبل فوتونات تتحرك بسرعة تتجاوز سرعة الضوء.
    خاص.

    وبحسب هذا المقطع فمن الممكن أن تكون المشكلة في الأوبرا في سارن بشكل عام في المعدات النهائية التي تتكون من مكونات المعدات (الجسيمات الأولية مثلا) التي تتحرك بسرعة عالية في اتجاه حركة النيوترينو.

    يمكننا أن ننتقل إلى التناقضات التي أعتقد أنها موجودة عند أينشتاين (هل تتذكر ساعات درجة الحرارة؟)، لكنني أعتقد أننا لخصناها قليلاً في الوقت الحالي. والأكثر من ذلك، ربما وجدت بعض المعدات التي يمكن أن تكون مؤقتًا بدقة بيكو ثانية، لذلك سنرى إلى أين سيصل ذلك.

    مئير.
    ليس من الواضح بالنسبة لي كيف يمكن إنكار مبدأ ماخ. بعد كل شيء، يكفي أن تدور في مدينة الملاهي وتلتقط مقطع فيديو للنجوم أعلاه. إذا شاهدت الفيديو، ألن تكون قادرًا على معرفة من خلال الفيديو فقط مقدار دوران رأسك أثناء الدوران؟

  566. إسرائيل،

    الجواب فيما يتعلق بالسرعة النسبية لجزيئات LS أنا لا أتخذ قراري. من الصعب بعض الشيء بالنسبة لي الآن أن أنقل هذا الانزعاج إلى تفسير مقنع، لذلك عندما أكبر سأكتبه.

    فيما يتعلق بماخ، فإن الاختبار ليس هو سبب دوران الصحن الطائر بالنسبة إلى النجوم البعيدة، ولكن ما إذا كان سيشعر بذلك عندما ندور النجوم البعيدة حول الطبق، وإجابتي هي "لا".
    بالطبع، في رأيي، مبدأ ماخ غير صحيح.

    تكتب "ومع ذلك، في رأيي، أي حل بديهي، وهذا ما أبحث عنه، لآلية الجاذبية، يجب أن يتضمن آلية تشبه LS. وإلا فكيف يعلم المشتري أني أحرك يدي في إسرائيل؟

    ليس من الواضح بالنسبة لي سبب بحثك عن حل بديهي. الحدس هو شيء السائل.
    كما أنه ليس من الواضح بالنسبة لي على أي أساس أنت متأكد من أن المشتري يعرف أنك تحرك يدك في إسرائيل.

    في رأيي، يعرف كوكب المشتري أنك تحرك يدك في إسرائيل، ولكن بدقة أقل بكثير مما يمكن توقعه من جاذبية La Sage. إنه يعرف ذلك من خلال آلية لا يمكن تفسيرها تمامًا (وبالتالي جميلة أيضًا) والتي من المحتمل أن أصفها في المقالة العاشرة. وهنا لا أسمح لنفسي إلا أن أذكر للغرض ما سبق أن كتبته في مقالات سابقة، وهو أن الجاذبية سائلة، وتنتشر حول كتلة ليس بطريقة خطية ولكن بطريقة حجمية. يملأ المساحة دون ترك "مسافات".

    سيتلقى كوكب المشتري رسالة حول تحريك اليد، لكن هذه الرسالة سوف تكون غير واضحة وتدور أثناء رحلتها إلى كوكب المشتري، وتندمج مع مجال الجاذبية العامة للأرض حتى تصبح غير قابلة للاكتشاف (كقصة رمزية فقط، سيشعر الشخص بخوض اليد في البحيرة). شخص يقف بالقرب منه، ولكنه سيتشوش مع المسافة). من ناحية أخرى، إذا اختفت بعض الكتلة من الأرض، على سبيل المثال في شكل إشعاع (أو في شكل تجربة فكرية)، فسيشعر المشتري بالتغير في كتلة الأرض على الفور.

    إليكم الأمر (وفي الوقت نفسه من المستحيل التشهير بمقالاتي من خلال ذكر مسدس في الفصل الثاني دون إطلاق النار في الفصل الخامس) وهي آلية ليست مثل LS. مما يعلن عن وجود كتلة بسرعة أعلى بكثير من سرعة الضوء.

  567. إسرائيل،

    من الواضح أنني لا أفهم LS لأنه حتى بعد النظر في نظريته عدة مرات (حسنًا، أعترف بذلك في ويكيبيديا) لا أستطيع أن أفهم كيف أنه إذا تحدث عن اختلاف الضغط على جانبي الجسم، فلن يتم إنشاء أي رياح . الرياح = حركة الهواء من مكان الضغط المرتفع إلى مكان الضغط المنخفض، لم أقل ذلك، قال داني روب. ويتضح من الرسوم التوضيحية في نظريته أن تدفق الجزيئات يتم إنشاؤه إلى مكان الضغط المنخفض. وماذا تكون إن لم تكن روح؟؟
    ومنذ متى والشكل الهندسي للجسم لا يؤثر على الروح؟ لقد قلنا بالفعل المئات.

    وفي رأيي أن هناك أيضًا فرقًا كبيرًا جدًا بين نيوتن الذي يشير إلى مركز الكتلة كنقطة محورية للجاذبية وبين ل.س. وفقًا لنيوتن، لا توجد علاقة بأبعاد الكتلة (الثقب الأسود يمكن أن يحتوي على كتلة أكبر من كتلة المشتري على سبيل المثال)، وتنشأ الجاذبية من نقطة هندسية لا أبعاد لها في مركز الكتلة.
    من ناحية أخرى، وفقًا لـ LS، يتغير أيضًا حجم الكتلة وأبعادها. ولذلك فإن الدحض على LS هو نجم ذو نفق. ووفقا لنيوتن، ستنشأ حركة توافقية في النفق لأن الصدفة ستنجذب إلى مركز الكرة. وفقًا لـ LS (وأيضًا وفقًا لموقع الويب الخاص بك) لا يوجد سبب لحركة متناغمة.

    شيء آخر ينبغي للسيد إل إس، لو كان على قيد الحياة أو أنت ويهودا، ممثليه على الأرض، أن يشرحوه، وهو كيف يمكن في ضوء "اختلافات الضغط لديكم" أن يكون لثقب أسود يبلغ قطره مترًا جاذبية أقوى من الشمس؟

    فيما يتعلق بلغزك، إذا قلت "نحن، كغرباء عن النظام، لدينا قوى خارقة لتحريك الكتل" فكل ما علينا فعله هو زيادة كتلة الأرض وسيتم الاستيلاء على القشرة. هل هذا ما قصدته؟

    هيا قم بالتجربة وسننتهي منها.

  568. مئير
    يتم قياس سرعة جسيمات LS، مثل أي سرعة أخرى، فقط بالنسبة لبعضها البعض، أو بالنسبة لمراقب أو جهاز قياس خارجي. في النموذج L. س. في الأصل، سرعتها لا تقل عن 100,000 ألف ضعف سرعة الضوء. بالنسبة لرؤوفين نير، سرعتهم هي سرعة الضوء. (وأنا أسأل: إذًا ما هي سرعة النظام الذي يشمل جميع الجسيمات في وحدة الحجم؟ نسبة إلى ماذا؟ ما لم تفترض نوعًا من نظام السكون المطلق، فارجع إلى القيمة التجريبية مم.). معي، لكل متر عند أي سرعة للمتر، سيكون للجسيم الواحد سرعة مختلفة، مثل سرعة جزيئات الهواء بالنسبة إلى أمتار مختلفة. ولذلك تتحرك الجسيمات بسرعات من 0 إلى ما لا نهاية، في كل الاتجاهات.

    فيما يتعلق بالنموذج الخاص بك. ما زلت لم أفهم كيف يعمل مع مبدأ ماخ. إذا كان مصدر القصور الذاتي هو الجسيم نفسه، فلماذا يدور أيضًا طبق طائر دوار يقيس قوة الطرد المركزي بالنسبة للنجوم البعيدة؟ كيف ترتبط؟

    لكن لا تيأس. يمكن أيضًا طرح نفس السؤال حول الفضاء الزمني لأينشتاين (لقد سألت، لقد سألت)، لذا فإن عدم الفهم ربما يكون عليّ. ولكن إذا كنت تستطيع أن تشرح، فهذا أمر رائع حقًا.

  569. ر.ح.
    صحيح أن سرعة الموجة نسبة إلى الوسط بالطبع. لكن بالنسبة لجميع الوسطاء المعروفين، فإن الهواء والماء والأسلاك والسرعة 0 بالنسبة لبعض العوامل الخارجية. ولهذا السبب تحدثت عن "النظام المغلق". إذا كنت لا تصدق، أعط مثالا مضادا. في "الموقع النشط" النظام مفتوح (تذكر ماكس؟). إن سرعة الموجات تتعلق فقط بالراصد أو القيَّاس، وبالتالي فهي نفسها في أي نظام مرجعي.

    لقد أجبتك بالفعل بخصوص النجم ذي الثقب: "الجاذبية في ليساج مطابقة تمامًا لجاذبية نيوتن، فقط ليساج هو الذي يعطيها سببًا، ونيوتن لا يفعل ذلك. ولذلك، ستكون هناك حركة متناغمة للقذيفة التي اقترحتها."

    أعتقد أنك (أو أنا) قد لا تفهم تمامًا نظرية LS. لأنك تستمر في ذكر "الروح" والشكل الهندسي للجسم. والتي لا تظهر في نظرية LS ادما. الجاذبية في L.S. كما ذكرنا، فهو مطابق تمامًا لنيوتن، وبما أنه لا يوجد معنى للشكل الهندسي بالنسبة لنيوتن، كذلك بالنسبة لـ L.S. لا.

    "أنت لم تجيبني بعد عن أسلوبك في التعامل مع الجاذبية. هل تعتقد أن جزيئات الأثير النشط هي أسباب انخفاض السرعة عن طريق دفع الجاذبية؟ مثل لاساج المشهور؟"

    لا أعلم. أنا حاليًا مهتم أكثر بمشكلة سرعة الضوء. لقد أشرت للتو إلى أن نفس الحل المتعلق بسرعة الضوء يمكن أن يحل مشكلة الاحتكاك في L.S. نظرًا لأنني لست في الصنابير الجماعية المظلمة، وقد أقنعني مايكل في الماضي أنه تم اختباره مع مراعاة دفع الجاذبية، لذا فإن الموضوع لا يزال مفتوحًا، على الأقل بالنسبة لي، على الرغم من أنه من المثير للاهتمام أن نرى كيف سيتم التكامل العمل على الكتلة المظلمة إذا استبدلنا الجاذبية النيوتونية بالجاذبية الدافعة، وأضفنا مبدأ ماخ. ومع ذلك، في رأيي، أي حل بديهي، وهذا ما أبحث عنه، لآلية الجاذبية، يجب أن يتضمن آلية شبيهة بآلية LS. وإلا فكيف يعرف المشتري أني أحرك يدي في إسرائيل؟

    في غرفة محكمة الإغلاق سيكون هناك جاذبية وفقًا لـ L.S. لا تشكل الجدران حاجزاً كاملاً ضد جزيئات LS، ولا شيء يشكل عزلاً ضد جزيئات LS. مثلما لا يمكنك عزل الجاذبية. لكن نعم، الجدران منجذبة قليلاً لبعضها البعض في L.S. (أيضا مع نيوتن).

    "خلاصة القول، ما أفهمه من النموذج الخاص بك هو أنك تعتقد أن هناك موقعًا يحتوي على جسيمات ذات سرعة متغيرة من 0 إلى ما لا نهاية. فوق سرعة الضوء تكون شفافة للجماهير ويمكن أن تمر من خلالها. وأقل من ذلك، فإنهم يدفعون الجماهير ويخلقون الجاذبية."

    نعم. السرعة فقط هي التي ترتبط ببعضها البعض، فلا يوجد شيء اسمه سرعة غير نسبية. تمتلك الجسيمات أيضًا الخصائص التي أشار إليها ماكسويل، الخاصة بثنائيات القطب المغناطيسي.

    سننتظر الآن مع التجربة، لدي فخذ في عيني، ومن الصعب الكتابة. ولكن هنا لغز يوضح فكرة التجربة: في مثال القذيفة التي تصل إلى نفق في إسرائيل فوق سرعة الإفلات، وعلى افتراض أنها قذيفة في غاية الأهمية، كيف يمكننا نحن الغرباء أن نظام يتمتع بقوى خارقة لتحريك الجماهير، ولكن ضمن نطاق الفيزياء المعروفة، هل يمكنه التقاط القشرة الموجودة في الأرض؟

  570. إسرائيل،

    بينما قمت بحل نسبة سرعة جسم إلى متوسط ​​سرعة جسيمات LS. (والتي تكون دائمًا صفرًا طالما أننا نناقش تلك الجزيئات LS التي لا يكون جسمها الضخم شفافًا)، فإنك لم تحل فيما يتعلق لسبب قياس سرعة جزيئات LS. أنفسهم.

    حقا فيما يتعلق بماذا؟ بالنسبة للفراغ المطلق عند نيوتن؟

  571. إسرائيل،
    أعتقد أنني لا أستطيع أن أوضح نفسي. وما علاقة أن المشاهدين يعرفون الاتجاه؟؟
    والمقصود هو أن سرعة الموجة نسبة إلى الوسط أو الجسيمات التي تحركها، وليس إلى شيء خارجي.
    بالإضافة إلى ذلك، حسب فهمي، الموجة هي تطور إثارة أو حالة الجسيمات (وليس الجسيمات نفسها). المذبذب الذي تتحدث عنه هو الذي يخلق الموجة وليس له علاقة.

    قبل التجربة. مازلت لم تجيبني ما هو أسلوبك في الجاذبية. هل تعتقد أن جزيئات الأثير النشط هي أسباب انخفاض السرعة عن طريق دفع الجاذبية؟ مثل لاساج الذي يتذكره الجميع؟ إذا كان الأمر كذلك، فما زلت لا أفهم لماذا لا تؤثر هندسة الكتلة على شكلها. كما أنك لم تتطرق إلى التجربة التي اقترحتها بخصوص نجم به ثقب.
    إن نموذجك، إذا فهمت بشكل صحيح، لا يتعامل مع الجاذبية كنقطة في مركز الكتلة مثل نموذج نيوتن.
    في الواقع، بعد مزيد من التفكير في غرفة محكمة الإغلاق وفقًا لنماذج الجاذبية بالدفع، لا ينبغي أن تكون هناك جاذبية على الإطلاق، تمامًا كما لا توجد رياح.

    باختصار، ما أفهمه من نموذجك هو أنك تعتقد أن هناك موقعًا به جسيمات متغيرة السرعة من 0 إلى ما لا نهاية. فوق سرعة الضوء تكون شفافة للجماهير ويمكن أن تمر من خلالها. وأقل من ذلك يدفعون الجماهير ويخلقون الجاذبية.

    إذا بدا لك أنني لا أفهم النموذج، فيمكنك تلخيصه في 3-4 نقاط.
    إذا فهمت يمكنك مواصلة التجربة.

  572. بالمناسبة، لاحظت أنه في أقسام TN للوظائف الموجية، قمت بعكس ترتيب الوقت/المسافة. لكن النية كانت جيدة.
    وفيما يتعلق بالكازينوهات - فأنا أذهب فقط إلى المعارض. لقد كان لدي صفقة معهم. كانوا يعطونني أجنحة، ووجبات، وعروضًا، ومروحيات، وكل شيء مجانًا - وفي المقابل كنت آخذ أموالهم. لكن الآن انقلب عليّ البخلاء مثل الرافد، حتى أنني نادرًا ما أذهب إلى هناك بعد الآن.

  573. اليوبيل.
    كما تريد. على الرغم من أنني لست متأكدًا مما قلته مرة أخرى هذه المرة. لقد سئمت من تخمين نواياك. اذا كان لديك ما تريد قوله، قله.

    ر.ح.

    بادئ ذي بدء، الموجة في الملعب ليست موجة طبيعية. يمكن لأي مشاهد أن يقرر مقاطعته أو عكس اتجاهه. الأمر مختلف مع الأمواج.

    1. في حالة الموجة الصوتية، هناك شيئين هنا - سرعة تقدم الموجة، والتي يتم تحديدها من خلال متوسط ​​سرعة جزيئات الهواء نسبة إلى متوسط ​​مركز سرعتها لكل وحدة حجم قادرة على حمل الموجات الصوتية.

    وفي حالة الهواء فإن متوسط ​​سرعته يزيد بحوالي 40% عن سرعة الصوت. نسبة إلى ماذا؟ إذا وضعت إصبعك في نقطة معينة في غرفة لا توجد بها رياح، حيث تبلغ سرعة الصوت حوالي 330 م/ث، وقمت بحساب متوسط ​​جميع سرعات الجزيئات بالنسبة للإصبع، ستجد أن المتوسط تبلغ السرعة حوالي 480 م/ث (إذا كنت أتذكر بشكل صحيح).

    2. إذا هبت رياح، فإن سرعة الصوت ستكون الآن 330 م/ث بالنسبة إلى ورقة تهب في الريح، والتي تبلغ سرعتها 0 بالنسبة للريح، لكن سرعة الورقة والرياح يمكن أن تكون 50 م/ث بالنسبة للأرض.

    ولا علاقة له بسرعة الإثارة. وإلا فسنحصل على سرعات صوت مختلفة للمذبذبات المختلفة بسرعات مذبذب مختلفة. الأمر ليس كذلك. والاتصال الوحيد بالمذبذب - أو الإثارة - هو درجة الصوت، ولكن ليس سرعة تقدم الموجة.

    3. ليس لدي أي فكرة عن سبب كونه تشيليبا مرة أخرى هذه المرة. أنا مهتم بالتكنولوجيا وليس بعلم النفس.

    4. عندما تصبح جاهزًا، يمكننا الانتقال نحو الاختبار الدحضي والتجربة.

  574. إسرائيل،

    أنا لا أتفق مع قولك "مركز سرعة الأشخاص في الملعب هو الملعب نفسه. ولهذا السبب تتقدم الموجة أيضًا نسبيًا نحو الملعب". ضع ألف رائد فضاء في الفضاء واطلب من الأول أن يرفع يديه ويخفضهما. قل للثاني أن يفعل نفس الشيء بمجرد أن يرى الأول يخفض يديه ثم الثالث والرابع وسترى موجة تتقدم للأمام. الملعب ليس له علاقة بالموجة أو سرعتها. يتم تحديد ذلك كدالة للوقت المنقضي بين الجسيمات الخاضعة للإثارة (رواد الفضاء يرفعون أيديهم في المثال أعلاه) والمسافة بينهما.

    وبالإضافة إلى ذلك، إذا كنت شخصًا غير مرغوب فيه في الكازينو، فلماذا تذهب إلى فيغاس كثيرًا؟
    وبالإضافة إلى ذلك، لماذا تزعج يوفال؟

  575. ر.ح.
    مركز سرعة الأشخاص في الملعب هو الملعب نفسه. ولهذا السبب تتقدم الموجة أيضًا بالنسبة إلى الملعب. فحتى الموجة الصوتية في الريح ستتحرك بسرعة الصوت بالنسبة إلى الريح، وليس إلى الأرض.

    يمكنك أن ترى هذا جيدًا إذا أخذت معادلات الموجة الكلاسيكية. تتكون الدالة من متغيرين، المسافة والزمن. إذا قمت باشتقاق الدالة بالنسبة إلى المسافة جزئيًا، فستحصل على معادلة الحركة التوافقية البسيطة. لكن إذا قمت بقصها جزئيًا بالنسبة للوقت، فستحصل على موجة جيب التمام المجمدة، نسبة إلى نظام المحاور، والذي سيكون في مثال الملعب هو نفس نظام الملعب.

    ولكن ما الذي ستحصل عليه في مثال الموقع النشط؟

    وفيما يتعلق بماكسويل ولورينز: كان جوهر حجتي هو أنني لست مثلهم. يتطلب الأمر دافنشي، أو ديكارت، أو غاليليو، لتحدي أرسطو. انا لا.

    صحيح أن الكازينوهات الشريرة تفكر بشكل مختلف، وقد علقت وجهي القبيح في نظام التعرف على الوجه، لكنني أعرف الحقيقة المرة. افتراضيا - لدي خطأ في الحسابات.

  576. يوفال
    صحيح أن التمرين لم يكن ناجحا. أي شخص نظر إلى تعريف النموذج الذي قدمته، كان يجب أن يتوصل إلى نتيجة مفادها أن هذا نموذج وهمي تمامًا، وهو ساذج يرتدي قشًا مشوشًا. في النص الأصلي، كان من المفترض أن يدخل الشبح الساذج في الفخ، ويقص النموذج بطريقته التصويرية المعتادة، ويرسلني إلى المصحة العقلية مرة أخرى.

    كان من المفترض أن أتفق معه من حيث المبدأ، وأومئ برأسي بالموافقة، وأستفسر عما إذا كان يمكنه ترتيب سرير مريح لي في المؤسسة، ويفضل أن يكون بجوار النافذة، ثم أشير بشكل عابر إلى أن هذا هو نموذج ماكسويل للأثير، وهو النموذج العلمي. تحفة القرن التاسع عشر التي حسب بها سرعة الضوء من ثوابت الكهرباء والمغناطيسية ومعادلاتها الأبدية.

    كما ذكرنا - الحيلة لم تنجح. اشتم الشبح العاري رائحة الفخ، وتجنب الدخول فيه. وهو ما يقودني إلى اقتراح للأشباح: تعال وانضم إلينا في لالا لاند. احصل على كأس من البيرة، وأحضر إحدى الأفكار التي لديك (لاحظت أنك مهتم بالرياضيات)، وتوقف عن القلق بشأن ما سيفكر به "المطلعون" وأرسل أفكارك لمراجعة النظراء.

    إذا قررت البقاء إلى جانب خبراء القص - فإليك اقتراح لتحسين هذه التقنية. بدلًا من مهاجمتنا شخصيًا، قم بقص أفكارنا. أنت أخضر للغاية، ونحن كبار السن ومتعبون جدًا، بحيث لا يمكن أن نتأثر بهراءك. ومن ناحية أخرى، إذا وضعت النظريات في نقطة ضعفنا، فيمكنك أن تفقد توازننا، كما فعلت معي مع الشحنة الكهربائية المتسارعة.

    ابدأ بنظرية مائير. ما يقوله هو شيء بسيط: كل جسيم يخلق الجاذبية، وبالتالي إذا حاولنا تحريك الجسيم من مركز جاذبيته، ستنشأ قوة معاكسة للحركة، أو باختصار، القصور الذاتي.

    1. هل هناك أي مشكلة متأصلة هنا؟
    2. كيف يتناسب هذا مع مبدأ ماخ؟ ففي نهاية المطاف، بحسب ماخ، فإن التسارع نسبي بالنسبة إلى النجوم البعيدة، انظر مثال الصحون الطائرة الدوارة، التي لا تقاس قوة الطرد المركزي، كما أنها لا تدور بالنسبة إلى النجوم.

    بهذه الطريقة، ستلهم كلاكما الاحترام، وإذا كان الأمر مهمًا بالنسبة لك، فستكون قادرًا على مساعدتنا في رؤية الأشياء التي ربما فاتناها.

  577. إسرائيل،
    ويجب أن أقول أنني لا أفهم.
    الموجة الصوتية في الهواء هي اهتزاز قوي للجزيئات في مكان معين نسبة إلى ما كانت عليه من قبل وليست نسبة إلى الأرض أو أي شيء آخر. يمكنك رؤيتها بشكل جميل في موجة في مباراة كرة قدم. يجلس الناس، وينهضون، ويجلسون، ثم يفعل من بجانبهم الشيء نفسه ويتم إنشاء موجة. الموجة لا تتحرك بالنسبة للملعب بل بالنسبة لحالة الناس أولا.
    هناك "إثارة" مؤقتة للأشخاص أو الجزيئات فيما يتعلق بحالتهم السابقة وعندما يتقدم هذا الإثارة تكون موجة.
    وفي الختام، فإن ما يتحرك في الموجة هو حالة الجسيمات وليس الجسيمات نفسها.

    بالإضافة إلى ذلك، ليس من الواضح ما هي حجتك حول ماكسويل ولورنتز. فقط لأنهم لم يفكروا في الأمر أولاً يعني أنه غير صحيح؟ وبسبب طريقة التفكير هذه، ظل العالم عالقًا في العصور الوسطى لمئات السنين لأنه "إذا لم يقل أرسطو ذلك أولاً، فمن المحتمل أن يكون هذا غير صحيح". اخجل لمدة دقيقة واستمر على الفور في خط تفكيرك. معظم الاكتشافات والاختراعات هي تلك التي تقول فيما بعد "واو، كيف لم يفكروا في هذا من قبل؟؟" حقيقة. لم أفكر في ذلك من قبل!

  578. إسرائيل،

    أتفق معك في أن تجاهل الحل البسيط الذي قدمته، فالجمود أمر معقد.

    وبعد أن نفهم من هو أبو وأم القصور الذاتي 🙂 يمكننا أن نتخيل أكثر من مثال لكون بلا جمود:

    على سبيل المثال، ستكون هناك كتل ثابتة في الفضاء، ولكنها تنفجر في حركتها بسرعة الضوء عندما يتم تطبيق قوة عليها بكثافة تتجاوز مستوى عتبة معين (الارتباط الفرويدي بالتأثير الكهروضوئي).
    أو على سبيل المثال الكون الذي تستجيب فيه الكتل لقوة تؤثر عليها بشكل مستمر، وبسرعة تتناسب طرديا مع القوة المؤثرة، وتتوقف في مكانها بمجرد زوال القوة.

    بعض الأكوان التي ليس لها قصور ذاتي تكون مظلمة. وإذا تابعنا في المستقبل كتابات من وحد قوانين نيوتن، فسوف نكتشف أنه ليس علينا بالضرورة أن نهاجر إلى أكوان موازية لنتمتع بقلة الجمود. وستكون هناك أمثلة على عدم وجود القصور الذاتي في الأجزاء المظلمة من عالمنا.

  579. إسرائيل،
    حسنًا، في هذه الأثناء، ننتظر الشركة الطيبة لتنتهي من فرز التعليقات الخاضعة للرقابة، ليس لدينا سوى الاستمتاع بكتابنا المقدس. وفيما يلي آية يوم:

    وفي السنة الرابعة عشرة جاء قيدار لعمر والملوك الذين معه وضربوا الاشباح

    الموتى لن يعيشوا، والأشباح لن تقوم، لذلك أنت تأمرهم وتدمرهم وتدمر كل ذكرى في حياته.

    عيش منك يا جيفة سيأتي الصيف ويبكي سكان التراب فالطل ينير الندى وتسقط أرض الأشباح

    رجل يضل عن طريق الحكمة بصحبة الأشباح.

    قل، أليس الوقت مبكرًا بعض الشيء بالنسبة لك في لوس أنجلوس؟

  580. أرجوك يا عرف، نظرت إلى آية الأشباح، وماذا وجدت؟

    "من الذي أغوىني؟ يسر هنا!"

    ولم يعلم أن هناك أشباحا في أعماق الهاوية.

  581. ر.ح.

    وأخذت الشرطة ردي للتحقيق. دعونا نأمل أن يطلقوا سراحها قريبا. وأمرتها بعدم الكشف عن أي معلومات باستثناء الاسم والرتبة والرقم الشخصي وبعض الغمغمات الوهمية التي تليق بوالدها الحقيقي.

  582. ر.ح.
    هنا تأتي اللحظة: تحديد سرعة الضوء في أي نظام مرجعي.
    أولاً، اعتراف مخزي: أنا لست مهتماً حقاً بـ ليساج. وتركها ليهوذا. وسر رهيب آخر: نمط الصدفة لم يكن كاملاً. ولم يتم طرحه إلا لتهيئة القلوب لقلب الحجة: شفافية المادة بالنسبة للجسيمات الافتراضية فوق سرعات معينة.

    دعنا نعود إلى ماكسويل.
    ورأينا أنه تمكن من استخدام نموذجه لتحديد سرعة الضوء من خلال ثابتي الكهرباء والمغناطيسية. السؤال هو: سرعة الضوء بالنسبة إلى ماذا؟

    افترض ماكسويل ولورنتز والعديد من معاصريهم تلك النسبية للموقع. ومن هنا محاولة ميشيلسون ومورلي: محاولة تحديد ما هو النظام الباقي للموقع.

    الآن: طوال الوقت، أقول إنه من المحتمل أن يكون هناك خطأ في نموذجي. لقد حان الوقت لأقول لماذا، منطقيا، يجب أن أكون مخطئا.

    موقفي من أسماء مثل "ماكسويل" أو "لورنز" يشبه موقف اليهودي المتطرف من أسماء مثل "رامبام" أو "حاخام لوبوفيتز" مع فارق واحد: يعتقد اليهودي المتطرف أن هناك من هو فوق رأسه الأبطال. أعتقد أنهم الشيء نفسه.

    عندما خطط ميكلسون لتجربة تهدف إلى تحديد ما هو نظام الراحة للأثير، فإن السؤال الأول الذي يتبادر إلى ذهني هو: ما هو نظام الراحة؟

    ففي نهاية المطاف، كانت صورة الكون في عام 1887 هي كون لا نهائي ومتجانس ومتناحي الخواص. بالفعل هناك تحفظ لدى SEELIGER، حول توزيع النجوم، لكنه ليس له علاقة بقضايانا.

    ولذلك، لو قيل إن ميكلسون تمكن من العثور على نظام سكون الأثير، وقيل إنه يتحرك بالنسبة إلينا بسرعة 1887 كم/ثانية باتجاه كوكبة الأسد، لوجب علينا أن نتساءل: لماذا هذا بالضبط؟ ؟ ما هو المميز عنها؟ أين التجانس؟

    إنه مثل العثور على النقطة المركزية للكون اللانهائي. لماذا هذا واحد؟

    وهنا أعرف أنني يجب أن أكون مخطئا. لا، من المستحيل أن أفكر في الأمر ولم يفعل لورنز ذلك. صحيح أنه تم قبولي في جامعة هارفارد، ولكن للحصول على شهادة جامعية فقط. لقد تم قبولي كمنظفة حمامات في جامعة هارفارد، ولكن فقط للطلاب الجامعيين. لقد استأجروا شخصًا أكثر ملاءمة لمنصب الموظفين.

    لذا فإن حقيقة أنني لا أعرف أن ماكسويل ولورينز قد أثارا هذا التحفظ، توضح لي منطقيًا أنني مخطئ.

    لكن حتى أعرف أين الخطأ، يجب أن أحاول حل اللغز، حتى أموت سعيدًا.

    إذن هذه هي الفكرة.

    دعونا نفكر في موجة صوتية في الهواء. إنه عديم الوزن، ولكن لديه قوة دفع وسرعة. تماما مثل الفوتون. السؤال هو: السرعة بالنسبة إلى ماذا؟
    لدينا هنا إجابة جميلة وبسيطة: نسبة إلى متوسط ​​سرعة جزيئات الهواء. إذا لم تكن هناك رياح، بالنسبة إلى الأرض. إذا كانت هناك ريح، نسبة إلى الريح. إذا كان في أوريون، نسبة إلى أوريون.

    ولكن ماذا لو كان الكون نفسه، اللامتناهي، المتجانس، مليئًا بجزيئات الهواء؟ بالنسبة إلى ماذا كانت الموجة الصوتية تتحرك؟

    بعد كل شيء، إذا كان الكون لا يتوسع (لا تنسوا، 1887، كون لا نهائي ومتناحي الخواص)، وكان مليئًا بجزيئات الهواء، فيجب أن نستقبل موجة صوتية، وحتى بسرعة معينة ومطلقة، ولكن نسبة إلى ماذا؟

    وهنا يأتي دور نموذج الطريق السريع لمساعدتنا: فبدلاً من بعد واحد، نفتحه على ثلاثة أبعاد. بدلاً من 3 مسار لكل بُعد، مسارات لا نهائية لكل بُعد. هذا هو المنطق الوحيد الذي يمكنني العثور عليه.

    وكما في مثال الطريق السريع، فإن الجزيئات الوحيدة التي توجد بالنسبة لنا بأي سرعة هي تلك التي تقل عن سرعة معينة. جميع السرعات أكثر شفافية بالنسبة لنا. إذا انتقلنا إلى سرعة مختلفة، فإن الجزيئات التي أثرت علينا سابقًا ستصبح فجأة شفافة، والجزيئات التي كانت شفافة ستصبح فجأة حقيقية.

    والموجة الصوتية (موجة جيب التمام! وليست جيبية) سوف تتقدم دائمًا بسرعة واحدة وثابتة بالنسبة إلينا، بغض النظر عن النظام المرجعي أو السرعة التي نصبح عليها. لأنه كما أظهرنا في مثال الطريق السريع، عند أي سرعة نعود إلى نفس الوضع بالنسبة للسيارات، أو بالنسبة إلى "الموقع النشط" في نموذجنا.

    هذا كل شيء في الوقت الراهن. توقف عن هضم الأفكار والقيل والقال. ماذا لو كانت الساعة الخامسة صباحًا في لوس أنجلوس؟

  583. كاسبر،
    لم تقل أنك ستغادر؟ والحقيقة أنك اخترت بيتاً جميلاً. يروهام يشكر ويغادر.

    إسرائيل،
    أعتقد أنك مخطئ ومضلل. الموجات في النموذج الأصلي الذي تشير إليه هي موجات جيب التمام وليست جيبية وهذا ما يصنع الفرق ويخلق التأثير الكهرومغناطيسي. كما أنها تتفق مع معادلة بنروز وحدسية غولدباخ للأعداد غير النسبية.

  584. إسرائيل!

    "الكون مليء بنوع من الجسيمات الصغيرة التي تشبه الكرة، مع تدفق التيارات من أحد قطبي الكرة إلى القطب الآخر في كل كرة. وبين الكرات حذافة ومحاور، عجلات خاملة، تتدفق وتشكل دوامات، تتقدم على شكل موجات جيبية متعامدة مع بعضها البعض في الكون، وهكذا تنشأ الموجات الكهرومغناطيسية.

    لماذا تفعل هذا!؟ أهكذا تعلمتم التصرف!؟

  585. في الواقع، أنا مثل ذلك. هذا يكفي، استمر بالفعل.
    لا أقصد القتال معك. حقًا. لدي نية متعمدة في انتقاد النموذج. حاول أن تخبرني عن رأيك في النموذج الذي قمت بتحميله فيه https://www.hayadan.org.il/astronomers-reach-new-frontiers-of-dark-matter-130112/#comment-327500

    واستخدم أكبر قدر ممكن من اللغة الرسومية.

  586. إسرائيل

    لطيف.
    هل تريد النقد البناء؟ حسنا هنا هو:

    أنت أول من يدرك حجم المشكلة، وأول من يفهم كيفية التعامل معها.
    أنت أذكى من مورلي، وأكثر تصميماً من ميكلسون.
    أنت أيضًا تتحرك بشكل أفضل من هوكينج.
    مفيش ناس كبار وذوي خبرة مثلك ومثلك يعرف..
    وبدون نظريتك، فإن الكون ببساطة لن يكون كما هو الآن.
    نيوتن يتقلب في قبره بعد قراءة النموذج الخاص بك. (كيف عرف هذا العبقري، لابد أنه يسأل نفسه)
    حلولكم عالمية المستوى، ماذا يقول العلماء عنها؟

    الأمثال 9، الآية 8

  587. مئير
    شكرا للمجاملة. عادةً، إذا تلقيت مجاملة على الموقع، يكون الأمر مثل: أنت تكتب بشكل جميل، على الرغم من أنك أحمق. ومن الجميل أيضًا أن أرى أن هناك بالفعل شخصين كاملين يقرؤون بالفعل ما أكتبه.

    كما كتبت، فإن مشكلة القصور الذاتي أكثر تعقيدًا في رأيي مما يمكن وصفه في بضع جمل. إنها ليست مسألة ماخ، أو آينشتاين، أو جسيمات هيغز. هذا سؤال كوني ووجودي، بحجم الأسئلة التي يتعين على يوفال أن يجيب عليها. السؤال في رأيي يجب أن يكون: كيف، من وجهة نظر منطقية، هل من الممكن حتى وصف الكون دون القصور الذاتي؟

    لرؤية ذلك، حاول أن تتخيل الكون بدون جاذبية. وفي هذه الحالة يمكنك أن تضع الأرض والقمر بجانب بعضهما البعض مسافة متر، ولن يكون هناك تجاذب بينهما. غريب، ولكن من المنطقي.

    ولكن كيف سيبدو الكون بدون القصور الذاتي؟ ما هي النتائج النهائية للاصطدام بين كرة بندقية والقمر؟ كيف يمكننا حساب النتائج كميا دون قانون حفظ الزخم؟ هل ستكون عشوائية تمامًا؟ لأنه إذا كان هناك منهج ومنطق وصيغ، فلا بد أن تتضمن في محتواها استنتاج أن الأجسام تقاوم تغير حالة حركتها يتناسب طرديا مع كمية الكتلة التي تحتويها، ومن هذا حصلنا على: القصور الذاتي. وذلك بطريقة منطقية، دون وصف للآلية.

    وفيما يتعلق بموديل LS. - للقداس في L.S. هناك دور واحد فقط: وهو الشراع الذي يقاوم حركة الجزيئات. نقطة. وعلى حد علمي، هذه هي الآلية الوحيدة التي تشرح بشكل حدسي سبب تأثير ما يحدث هنا على ما يحدث هناك. إلا إذا كنت تفضل: "الكتل تشوه الزمكان" (كيف بالضبط؟).

    أحتاج إلى إعداد إجابة تشرح سرعة الضوء واختبار الدحض. لقد تركنا الضوء الأخير للمنطق الخالص للانضمام إلى بقية الحكماء المتقاعدين. لقد تركنا أيتاما مثل شجرة مهجورة، بلا أب أو يد ترشدنا. مجموعة مثيرة للشفقة من "العلماء" يتعاملون مع البراز، ويرفعون فزاعاتهم لتحطيمها وإرسالها إلى مزبلة النظريات الوهمية..

    يا أخي، يا لها من ملعقة، يا له من صمت، يمكننا أن نتنفس أخيرًا!

    ذكي..

    عندما أسمع كل النحاتين، الذين ليس لديهم ولو ذرة من فكرة أصلية، ويعرفون فقط كيف يجدون العيوب في الآخرين، أفكر في صديقي زفيكا، المخترع، والمهندس في جوجول، والرجل الذكي حقًا، وعطلات نهاية الأسبوع الرائعة التي نقضيها في مناقشة الأمور العلمية، وحماية قلوب النساء المهملات. زفيكا يحب الاستمتاع بالأفكار، نبحث عن حلول للمشاكل معًا، دون تنازل، دون غطرسة، والأكثر من ذلك، زفيكا، خريج التخنيون، يدعي، بموافقة ويكيبيديا، أنه مع كل الاحترام الواجب لمستوى أعلى مؤسسات التعليم في إسرائيل، لا ترقى إلى مستوى الجامعات في الخارج، بالنسبة لمئات الآلاف من الشرقيين الذين يملأونها، الصينيين الموهوبين والمجتهدين الذين لا نهاية لهم، والذين يدرسون 16 ساعة معتمدة قبل الذهاب إلى العمل في المساء لمساعدة أسرهم، يكملون الدكتوراه وعمرهم 25 عاماً، وكل هذا بابتسامة خجولة على شفاههم واستعداد للمساعدة، ودون التفاخر بعلمهم وتعليمهم.

    حسنًا، لقد انجرفت قليلاً، فلنعد للرد على R.H.

  588. عند هذه النقطة سأتخلص من الأسئلة المتعلقة بالضوء والفوتونات. هذه أسئلة حسابية قد يكون لها حل غير ضار.

  589. إسرائيل،
    أنا أتابع. لقد فهمت كيف تغلبت على حجة فاينمان. من العبقرية.
    لكنني لا أحب L.S.

    بعض الأسئلة:
    ألم ترمي الماء مع الطفل؟ في النموذج الأصلي كان هناك تفسير لأحد جوانب القصور الذاتي: تسارع الكتلة يجمعها مع المزيد من الجزيئات عكس اتجاه الحركة، وبالتالي مقاومة الكتلة للتسارع. في النموذج الخاص بك، هناك حاجة إلى آلية منفصلة لهذا الجانب (لم أتعمق في نظرية LS، ولكن في ظاهر الأمر، يبدو أنه في كل نموذج لهساجي لا يوجد تعامل مع مسألة ما هي الكتلة، حيث أن LS فالجسيمات نفسها تظهر بكتلة مدمجة، كما لا يوجد أي تناول لمسألة كيف "تجد طريقها" في الفضاء لتحافظ على حركتها بسرعة ثابتة في خط مستقيم).

    كيف يتم "عدسة" شعاع الضوء بواسطة جسيمات LS؟ دون أن يتم الترفيه؟ كيف التأثير العشوائي لجزيئات LS في الفوتون الذي يشق طريقه مليارات السنين الضوئية في الفضاء، ألا يجعل لاساجي مساره فوضويًا؟

  590. الرفاعي م، انتظر لحظة، لا تغادر بعد. أحتاج إلى حدسك الحيواني. هناك نموذج ما يجب أن أقبل نقدًا صحيًا له.

    ما رأيك في النموذج التالي: الكون مليء بنوع من الجسيمات الصغيرة الشبيهة بالكرة، مع تدفق التيارات من أحد قطبي الكرة إلى القطب الآخر في كل كرة. ويوجد بين الكرات حذافات ومحاور، عجلات خاملة، تتدفق مكونة دوامات، تتقدم على شكل موجات جيبية متعامدة مع بعضها البعض في الكون، وبالتالي تتولد الموجات الكهرومغناطيسية.

    مراجعة صادقة. لديك الإذن بقص محتوى قلبك واستخدام أي لغة تختارها.

  591. يوفال

    أرى أنك تستمتع بما يسمى بالاستمناء الفكري بينك وبين إسرائيل (بمساعدة عرضية من مهر.ح)،
    كل ما أستطيع أن أتمنى لك هو حظا سعيدا.
    سأفعل مثل الحكماء الآخرين وسأنسحب من المناقشة. (لكنني أعدك بأنني سأواصل القراءة، أنتم بالتأكيد مسليون يا رفاق 🙂 ).

  592. وسأعود بكل سرور بعد توضيح النموذج الخاص بك والاتفاق على كل شيء. وفي هذه الأثناء، كان مطلوبًا مني أن أتطرق إلى مسألة تسارع الضوء خلال رحلته التي تستغرق مليار سنة من المجرات البعيدة إلينا. لقد تلقيت إشارة مفادها أنه من الممكن أنه يتباطأ بالفعل، وإذا كان هذا صحيحًا، فيجب علي تغيير شيء أساسي في القصة. يمنحك هذا إقامة لمواصلة الاحتفال بمفردك في دائرة الضوء. RH يطحنك جيدًا. أحسنت وعزيز عليه. وأيضا الرفاعي.م تذكرته للأبد. قد لا يكون متقدمًا مثلك في الأمور، لكن لديه حدس سليم (انظر المزامير الفصل 8 الآية 3)

  593. ومرة أخرى، الرد في انتظار التأكيد. لقد أصبحت آلية التحكم هنا مجنونة تمامًا.
    أين يوفال ومئير؟ لقد قلبنا يوفلي تمامًا رأسًا على عقب، حتى أنه قام بتغيير الصورة. بالمناسبة، أرسل لي جزءًا من نموذجه. بالنسبة لأولئك منا الذين لم يروه من قبل، واعتقدوا أنه يعتمد فقط على "في البداية لم يكن هناك شيء لم يكن في حد ذاته" يمكنني أن أؤكد لكم أن النموذج مذهل ومادي وهندسي تمامًا. ربما بدأ اليوبيل المرتبك للتو من افتراض أننا جميعًا نعرف النموذج، ربما من خلال التخاطر.

    لذا اترك نظريات الشماوري. ولا شك أن الموقع يخلو من رافد من السماء إلى حد بعيد. من سيوصي بعربات اليد؟ من يرضينا بلغة ماضية؟ من سيعلمنا تاريخ شعبنا؟
    لذا اترك كل شيء الآن واتصل به على الفور لترتيب ما يلي:
    اليوبيل يعود!

  594. كما استخدمنا النيوترونات البطيئة في القنابل التي صنعناها في الكيبوتس. خاصة في القنابل الخلفية.
    وعندما نطلق النار على البطيخ، فإن الرصاص البطيء يحدث كل الضرر. بالكاد تصطدم الصواريخ السريعة بنواة بيضاء.
    ولماذا نذهب بعيدا؟ دائمًا ما يتحدث شيوخ الديناصورات في ركن الحيوانات بالمزرعة عن النيازك التي كانت تضرب البلاد. أما السريعون، الذين يصلون بسرعة نصف سرعة الضوء، فيحدثون ثقبًا صغيرًا ويستمرون دون أن يلاحظهم أحد. كان الهايتيون هم الذين أحدثوا الفوضى والانقراضات دائمًا.

    وبشكل عام: الجميع، دون استثناء، لا يتحدثون دائمًا عن مليارات النيوترينوات التي تمر عبرهم كل يوم. إنهم لا يخبرون، لأنهم لا يدركون وجودها على الإطلاق، لأنه رغم كل أجهزة القياس يكاد يكون من المستحيل قياسها، وهذا على الرغم من عدم وجود شك في وجودها. كما أنها سريعة جدًا. ما الذي قد يجيب على السؤال: في السؤال: "سيكون الجسم A شفافًا بالنسبة للجسم B إذا تحرك الجسم A بسرعة معينة عبر الجسم B."

    ر.ح.

    أنا مهتم بالفيزياء والنقد البناء. ليس هناك شك في أنه قام بتحسين لهجته وأدلى بالتعليق الصحيح. إذا واصلنا على هذا النحو، ليس لدي أي مشكلة معه.

    لكن كل هذا لا علاقة له بشؤوننا على الإطلاق. حتى مثال الصدفة ليس كاملًا، وقد تم تقديمه فقط لتحديد ما نعرفه من الحدس، بطريقة غير كاملة، وهو أنه فوق سرعة تصادم معينة، يتضاءل تأثير الاصطدامات بين الأجسام.

    لا تصدق؟ قم بتعليق كتلة من البلاستيسين على خيط وأطلق عليها مقذوفات بسرعات مختلفة. انظر إلى من تستجيب المجموعة له أكثر.

    يصعب علي قبول تفسيرك لتأثير الشكل الهندسي على الجاذبية في نموذج ليساج. ولم نتناول الموضوع بالتفصيل، لكن لا ننسى أن النظرية تم اختبارها من قبل الجميع تقريبًا، ولم يتم طرح هذا التحفظ أبدًا. لا يهم ما إذا كانت النظرية ستنجح أم لا. يجب أن ينجح الأمر، ليس هناك شك تقريبًا، باستثناء أنه وفقًا لنموذج ليساج الأصلي، يجب أن يكون لدينا جاذبية، بغض النظر عن الشكل الهندسي، ولكن أيضًا الاحتكاك.

    ومن وجهة نظر الجسيمات، فإن التفاعل كله يكون فقط بين الجسيمات الأولية التي لها نفس الشكل الهندسي. لا يظهر مفهوم "الروح" في نظرية ليساج. يرجع اختلاف الضغط في المحاكاة إلى وجود شراعين في الهواء الساكن، بينهما كمية مخففة من الهواء. يمكنك تسميتها الرياح، لكنها تختلف عن الرياح في البحر، إلا إذا فكرت في الرياح التي تضغط على شراع واقف، فهي لا تختلف عن ضغط الهواء العادي.

    إن جاذبية ليساج هي نفسها تمامًا مثل جاذبية نيوتن، فقط ليساج هو الذي أعطى السبب، بينما نيوتن لم يفعل. وبالتالي ستكون هناك حركة توافقية للقذيفة التي اقترحتها.
    كيف سيعمل الشراع الذي اقترحته بالقرب من كتلة كبيرة، إذا كان في حالة سكون من حيث الجسم أثناء السقوط الحر؟

    يجب أن أتحرك، في عيد الحب. امرأة أو شوكولاتة أو زهور أو نبتة في الرأس. وسنواصل التجربة لاحقا.

  595. إسرائيل شابيرا
    من تجربتي مع القنابل الذرية التي صنعتها عندما كنت طفلاً في الحي الذي أعيش فيه، كانت النيوترونات البطيئة هي الأكثر نجاحًا وأظهرت النتائج..
    والسبب هو أن الأجسام السريعة ستطير عبر 235 ولم تطلق نيوترونات تقريبًا للعملية المتسلسلة. ومن ناحية أخرى، ابتلع اليورانيوم 235 النيوترونات البطيئة وحاول تحرير نفسه من الضيف غير المدعو حتى، بسعال شديد، انقسم إلى ذرتين أصغر ونيوترونات أكثر استمرت في السلسلة وكتلة أكبر. وربما أيضاً أعصابه التي تحولت إلى طاقة
    لكنه كان في حيي، لا أعرف كيف كان في أحياء الشمال.
    يوم جيد
    سابدارمش يهودا

  596. إسرائيل،

    لقد رأيت أنك بدأت بالهلوسة والقيام بجلسات تحضير الأرواح مع الشياطين والأرواح لذا لم أرغب في الإزعاج.

    دعنا نعود إلى القشرة والأرض.
    ووفقا للنظرية المقبولة، كلما زادت الكتلة، زادت قوة جاذبيتها. لا يهم شكله أو إذا كان به ثقوب.
    وفقًا لنظرية ليساج الخاصة بك، ودعونا لا ننسى السيد سابراديش المحترم، فإن الجاذبية ناتجة عن ضغط الجزيئات على الكتلة (لقد أضفت أيضًا السرعة العالية التي تلغي الجاذبية، لكن هذا ليس ذا صلة في الوقت الحالي).
    مما أفهمه وقد ناقشنا هذا من قبل، لكنك الآن أوضحت نظريتك بشكل أكثر وضوحًا، في نموذجك، يجب أن يكون للشكل الهندسي للكتلة تأثير على جاذبيتها. على الأقل ثقب في منتصف الكرة حيث لن تكون هناك جاذبية لأنه ستكون هناك حركة حرة للجسيمات التي تحركها. لذا، مهما كانت سرعة وصول القذيفة، فلن تؤثر عليها الجاذبية، وبقدر ما يتعلق الأمر فهي في الفضاء الفارغ. لا تحتاج إلى سرعة الهروب أو أي شيء.

    ومن هذا أيضًا تأتي تجربة تفنيد لنظريتك، عن كل من يهودا وسيج.
    خذ نيزكًا ذو الجاذبية X. احفر نفقًا من النهاية إلى النهاية. أدخل أي كائن في النفق. إذا كنت على حق، فلن يؤثر عليه أي جذب ولن تكون هناك حركة توافقية. إذا تم إنشاء واحد، فأنت مخطئ.

    أيضًا في مثال الأوتوسترادا الخاص بك فإن الشكل الهندسي للبلاستيك أو الشراع سيكون له تأثير على اتجاه حركته مثل الشراع الثلاثي للقارب سيدفعه في أي زاوية ما عدا ما بين -45 +45 (ما يعرف بالحديد) ). وبالمثل، فإن شراع الجاذبية (في الواقع أي كتلة) بالقرب من كتلة أخرى يجب أن يتصرف مثل الشراع.
    أوافق على أنه لا توجد "رياح" في الفضاء لأن الجزيئات تتوازن مع بعضها البعض، لكن في محيط كتلة كبيرة توجد "رياح" وبالتالي يجب أن يكون لهندسة الكتلة تأثير.
    ولكن ماذا؟ أظهر جاليليو أن الأمر لم يكن كذلك، فبعد كل شيء، لديك اختبار دحض تم إجراؤه بالفعل.

    على أية حال، هيا، ما الذي يميز سرعة الضوء وما هي تجربتك الدحضية؟

  597. سؤال ليهودا

    إذا كنت نظير اليورانيوم 235، فمن الذي تخاف منه أكثر: النيوترونات السريعة أم النيوترونات البطيئة؟

    إذا كنت تستطيع، اشرح أيضًا السبب.

  598. كيف يمكنك أن تقول أنها تشع؟ - بسبب ما يتم الحصول عليه في حسابات الديناميكا الكهربائية الكمومية.
    لماذا لا يمكن قياسها؟ بسبب مبدأ عدم اليقين.

    على أية حال، إجاباتي محدودة بسبب معرفتي المحدودة بهذا الموضوع.
    أولئك الذين يمكنهم الإجابة على أسئلتك بجدية ودون خربشات في هذا الموقع هما إيهود ومايكل روتشيلد.
    وسأكون سعيدًا جدًا أيضًا إذا شارك واحد منهم على الأقل في هذه المناقشة.

  599. جميل. أنا فخور بك. وهذه نقطة لم أفكر فيها. وهذا هو بالضبط سبب طرحي الأفكار للمراجعة.
    لكن انتبه إلى نقطتين: 1. يشع الإلكترون المتسارع بغض النظر عن انتقال الزخم إلى الحلقة. 2. لا يهم حقًا هذه النقطة - لقد كان مجرد مثال، وأنا أعترف أنه لم يكن ناجحًا (لقد مرت سنوات عديدة منذ أن تعاملت مع هذه القضية).

    وشيء آخر. أنا لا أحاول بيع نظرية ما، بل على العكس. أحاول العثور على ما بها حتى أتمكن من اللجوء إلى أشياء أخرى.

    لسوء الحظ، حقيقة أن المثال غير ناجح لا يغلق الباب أمام الفكرة. حاول أن تفعل الشيء نفسه في جسم غير مشحون كهربائيًا، مثل القشرة، أو بشكل عام، بما أننا نتحدث عن أجزاء افتراضية (جسيمات لاسيج)، فكيف يمكنك القول إنها تشع؟ بصرف النظر عن مسألة الإشعاع، هل سينجح مثالي؟

    سأذهب للنوم الآن (1.30 في لوس أنجلوس). لا أعتقد أن هذا يغلق الباب أمام الفكرة، ولكن إذا كنت تعتقد ذلك، فاشرح السبب.
    شكرا جزيلا على المراجعة البناءة.

  600. يهودا

    المشكلة الرئيسية هي إقناع إسرائيل بأن الشحنة الكهربائية التي تتحرك بتسارع تبعث إشعاعات (وحتى الشحنات الساكنة تبعث فوتونات افتراضية).

  601. للجميع
    التعليقات هنا تطول وتطول ويجب أن أطلب من والدي متابعة التاريخ لأنه في وقت قصير ستصبح الستة مليارات سنة قديمة وسيتعين علينا تحديثها!.
    يوم جيد
    سابدارمش يهودا

  602. وماذا لو تحركت جزيئات الهواء أو الإلكترونات أو البوزيترونات أو الجرافيتونات أو أي شيء آخر عبر الثقب الموجود في الحلقة واصطدمت بالإلكترونات التي تعبرها؟ بعد كل شيء، تقوم إلكتروناتك بنقل الزخم عندما تنجح في اختراق الحلقة، وأيضًا عندما تتحرك عبر الثقب الموجود في الحلقة، فإنها تنقل الزخم إلى الجزيئات الموجودة هناك. ألا تعتبرهم مثلاً؟

    على سبيل المثال فقط: إذا وضعت إصبعك في الحلقة وشكل إصبعك حاجزًا أمام إلكتروناتك، فهل ستخترق إلكتروناتك - "بسرعة الهروب" - إصبعك دون ترك أي زخم؟

    لا أستطيع أن أفهم حججك، على محمل الجد. إذا كان بإمكانك كتابة ملخص لفكرتك.

  603. حسنًا، يجب أن أفكر قليلًا، يبدو لي أن نمط الصدفة قد يكون مربكًا. دعنا نعود إلى مثال الشاحن الكهربائي. كما أنها أكثر واقعية فيما يتعلق بالشفافية.

    لنفكر في حلقة معدنية تتدلى من سلك ومشحونة بشحنة كهربائية سالبة. تقترب الإلكترونات من المركز من اليمين بسرعات مختلفة.

    ادعائي هو: تحت سرعة معينة VM، ستدفع جميع الإلكترونات بعيدًا عن الحلقة وبالتالي تنقل الزخم إليها. من ناحية أخرى، من سرعة VM وما فوق، سوف تمر الإلكترونات بالحلقة إلى اليسار، وتتسارع عائدة وتعود إلى سرعتها الأصلية دون ترك أي بصمة على شكل زخم على الحلقة. خلاصة القول، الإلكترونات والحلقة شفافة بالنسبة لبعضها البعض.

    أخطاء؟

  604. مئير
    وفي الحركة التوافقية لا ينتقل الزخم إلى نظام الأرض/الصدفة؟

    لا بد لي من الذهاب إلى العمل، وسوف نتناقش عندما أعود.

  605. إسرائيل،
    لم يتم نشر ردي منذ 24 ساعة. ربما كان طويلاً جدًا، لذا سأختصره إلى 20%:

    فيما يتعلق بالقذيفة التي تم التقاطها، لم أفهم سبب عدم دخولها في (مثل) التذبذب التوافقي، وبالتالي لن تنقل الزخم (بعد كل شيء، اتفقنا على أنه لا يهم ما إذا كانت قد اكتسبت سرعتها الأولية بالقرب من المركبة الفضائية بعد إطلاق النار أو بعد السقوط الحر من مسافة أكبر).

    أما فيما يتعلق بالنظرية المتطابقة فإنني أشكك في هويتها في ظل أمر التعرف السريع. لا يوجد وضع.

  606. لا، هذا ليس ما أدعيه.
    ربما لم أخوض في التفاصيل الكافية. الوقت المناسب للقيام بذلك هو الآن.
    وفي الحقيقة فإن أي جسم يصل إلى الأرض من الفضاء ويمر عبر نفقنا، ولا تتأثر به إلا اعتبارات الجاذبية، لن يترك بصمته على الأرض على شكل قوة دفع.
    سوف يتسارع الجسم في اتجاه الأرض، ويصل إلى سرعة الإفلات ويصل إلى أعلى وجه الأرض، ويصل إلى أقصى سرعة له في مركز الأرض، ويترك الجانب الآخر بنفس السرعة التي وصل بها بالضبط، والتي منذ ذلك الحين فإذا كانت أعلى من سرعة الإفلات، فإن الجسم سيهرب بالفعل من جاذبية الأرض وسيكون جاهزًا للقيام بنفس التمرين مرة أخرى مع أي كوكب آخر.

  607. بخصوص القشرة 12

    وهذا مشابه للادعاء بأن الطائرة التي تسير بسرعة الصوت تتجنب الاحتكاك بالهواء.

  608. إسرائيل

    إذا كنت تبحث عن النتيجة الأكثر دقة، فعند حساب حركة الأرض وحركة الشمس، لا يمكنك فقط مراعاة بيانات الأرض والشمس. يجب عليك أيضًا تضمين القيم التي لا ترتبط بشكل مباشر بـ KDA والشمس مثل تأثيرات الكواكب الأخرى. التأثيرات الناجمة عن الظواهر/العمليات التي تحدث حول الأرض، وحول الشمس، وما إلى ذلك.
    ونظرًا لأنه من المستحيل تحديد هذه البيانات بشكل مثالي، فإن نتائج القياسات أيضًا ليست مثالية.
    يتم التعبير عن هذا بشكل أساسي في الحالات التي تحاول فيها حساب كتلة النيوترينو، على سبيل المثال، أو حجم الكون.

  609. نسيت أن أضيف - ألا ترون الفارق الواضح والصريح بين القذيفة 11 التي نقلت أكبر قدر من الزخم إلى إسرائيل، دون أية اعتبارات كمية، وبين القذيفة 12 التي كانت تحتاج إلى هذه الاعتبارات؟

  610. نعم. لنأخذ نفس المنطق ونطبقه على أي نظام آخر يتضمن الجاذبية. وماذا عن حركة الأرض حول الشمس؟ هل من المفترض أيضًا أن يتم إبطاؤها؟ ماذا عن حركة البندول؟ ماذا عن حركة جزيئات الهواء داخل زجاجة مغلقة؟

    والشيء الأكثر أهمية: ما الذي يهم بالضبط بالنسبة للحجة نفسها؟ قيل أنك على حق وبعد مائة مليار عود ثقاب أرضي سنثبت أنه بالفعل انخفضت سرعة القذيفة بمقدار متر كامل في الثانية! فهل يغير هذا من افتراضنا الأساسي أنه طوال رحلة القذيفة لم تترك أي علامة على الكواكب التي عبرتها أثناء طيرانها على شكل قوة دفع يمكننا قياسها؟ لأنه إذا لم نتمكن من القياس، فكيف يمكننا أن نعرف أنه كان هناك في أي وقت مضى؟ ألم يصبح "نيوترينو" - نعلم أنه موجود، لكننا لا نستطيع قياس تأثيره؟

  611. نعم نوعا ما.
    بقدر ما أفهم نظرية الكم (وهي قليلة) - إذا مرت القشرة عبر النفق، فحتى عند سرعة الهروب - ستظل القشرة خاضعة لتأثيرات خارجية (تغير في بعض القيم مثل كتلة الزخم.. حتى مستوى معين) لأنه حتى في الفراغ نفسه توجد تقلبات كمية. أي أن القشرة التي ستمر عبر النفق بسرعة الهروب - حتى لو لم تتمكن من قياس أي تأثير عليها - سيظل لها تأثير ضئيل عليها كما تظهر الحسابات الرياضية (لا تطلب مني أن أعرض لك الحسابات 🙂 ).

  612. دعنا نرى.

    لنأخذ مثال النفق عبر KDA. لقد فهمت أنك تدعي أنه بسبب التأثيرات الكمومية فإن حركة القشرة سوف تتباطأ. أنا أفهم ذلك أليس كذلك؟

  613. المشكلة هي أنه مع المعرفة الحالية، لم يعد بإمكانك القول بوجود مثل هذا النظام في الطبيعة من شأنه أن يشكل مجموعة من "القوى الصفرية". أي أنه لا يوجد نظام يمكنك وصفه ولا يؤثر بأي شكل من الأشكال على الجسيم الذي يمر عبره.

  614. بالمناسبة، فيما يتعلق بـ "أساليب الكتابة" - شخصيًا أفضّل المحتوى على الأسلوب. من جهتي، الكتابة حتى مع وجود أخطاء إملائية، الشيء الرئيسي هو أن يكون لديك محتوى يمكن من خلاله استخلاص استنتاجات منطقية وذات مغزى.

  615. إسرائيل

    حسنا، أنا أحب لهجتك.
    (يجب أن أقول إن ما قلته عن R.H. كان مسليًا 🙂 ولكن دعنا نترك الأمر عند هذا الحد، فهو ليس ذو صلة).

    أهلا بك أن تقرأ تعليقاتي مرة أخرى، وتوضح لي ما هو الخطأ في كلامي الذي يتناقض مع كلامك.
    بالطبع سأقرأ أيضًا، وإذا وجدت شيئًا غير منطقي في كلامي - الذي تمكنت إجاباتك من دحضه - فسوف أرد عليه أيضًا.

  616. ر.ه.رفاعي.م

    مرة أخرى ردي على R.H. بانتظار التأكيد.

    أنا دائما منفتح على النقد. ولكن على الأفكار، وليس هجوما شخصيا.

    قد أكون مخطئا بالطبع. ماذا عنك؟ هل أنت فوق الخطأ؟

    خذ كل تعليقي وحاول أن تبين لي أين أخطئ. حاول التحدث بالفيزياء فقط. بمجرد التبديل إلى الخطوط الشخصية، قل وداعًا في طريقنا إلى السلام.

    وفي رأيي، أنت مخطئ بشأن R.H.
    إلى جانب كونه أحد أكثر الكتاب عقلانية ومنطقية في الموقع، فهو أيضًا كاتب جميل وممتع وممتع، ولن يفوتني تعليق منه أبدًا، سواء اتفقت معه أم لا. الشيء نفسه ينطبق على يهودا ويوفال. يمكنك أن تتعلم منهم الكثير فيما يتعلق بالأسلوب، خاصة من يوفال. من المؤسف أن غاي تركنا، لقد كتب جيدًا أيضًا.

    هناك كتاب منطقيون للغاية ولكن جافون إلى حد ما هنا. على الرغم من أن العلم ليس مسلسلًا تليفزيونيًا، إلا أنني أعتقد أنه من المستحسن الحفاظ على الحيوية والفكاهة، وإلا فسوف ننام جميعًا. لهذا السبب أحاول إدخال بعض الهراء والقصص الشخصية.

    لذا من فضلك، أرني أين تعتقد أنني مخطئ، وأنا دائمًا أدعو شخصًا ما للتحقق من أفكاري. لكن هذا لا يعني أنني يجب أن أتفق معك، ولا أتحدث بشكل شخصي على أي حال.

  617. نعم، أيها الراعي، ولكن هناك مشكلة أخرى

    لقد صاغ نيوتن قانون الثبات، مثل قانون الجاذبية، لكنه لم يقدم لهم أي تفسير.

    وفيما يتعلق بالمثابرة، تحدث عن "الفضاء المطلق" الجوي، بروح الله (بكلمات نيوتن الخاصة) وأعطى مثال الدلو الدوار، وهو التجسيد السابق لـ BSA في يهوذا، لتوضيح المشكلة. إذا كانت هناك آلية للقصور الذاتي فسرها نيوتن جيدًا، فلماذا كانت مطلوبة بعد 200 عام؟ أو أينشتاين؟

    الشيء نفسه ينطبق على الجاذبية. وصف نيوتن الظاهرة وحدد كميتها، لكنه لم يتظاهر أبدًا بشرح الآلية، وعلق بمرارة فقط "سأترك الأمر للقارئ ليقرر كيفية عمل الجاذبية".

    ولكنك قفزت خطوة واحدة إلى الأمام. لقد افترضت أنني كنت أحاول شرح القصور الذاتي باستخدام الآلية التي وصفتها. انها ليست دقيقة. لا يمكنك تفسير القصور الذاتي عن طريق نقل الزخم، الذي ينشأ من القصور الذاتي. سنصل إلى ذلك لاحقا. ومن ناحية أخرى يمكننا أن نتحدث عن الجاذبية.

    إذا في مثال الطريق السريع بدلًا من شراع واحد، وضعنا شراعين على مسافة معينة من بعضهما البعض، فوفقًا ليساج والمنطق سيكون هناك تجاذب بين الأشرعة. أعتقد أننا حللنا مشكلة الاحتكاك الديناميكي الحراري للجزيئات مع الكتلة، بمثال تفاعل الغلاف مع DA. يتم نقل الزخم والدفع، ولكن لا يوجد احتكاك. ولكن ماذا عن مشكلة الاحتكاك الثانية التي تحدث عنها فاينمان، وهي مشكلة احتكاك الكواكب في حركتها في الفضاء مع الجزيئات؟

    دعونا نحسن المشكلة. يقول فاينمان: صحيح أنه سيكون هناك تجاذب في نموذج ليساج، لكن النجوم والكواكب في حركتها ستصطدم بالجزيئات، وهذه ستقاوم الحركة مثل الريح ضد الشراع.

    ومع ذلك، لاحظ أن هذه المشكلة غير موجودة في نموذج Autostrada. ومهما كانت سرعة الشراع، أو البلاستيسين، فإنهم في حالة سكون بالنسبة لهم. إذا أضفنا بعدًا آخر إلى مثالنا أحادي البعد، إذا تحرك البلاستيسين أيضًا شرقًا أو غربًا، وكانت نفس آلية الأوتوسترادا تعمل في هذا البعد أيضًا، فإن البلاستيسين بالنسبة لها هو بسرعة 0 بالنسبة للسيارات ولا يوجد احتكاك.

    ولذلك، إذا كانت الأرض تجذب القمر، ويتحرك أيضًا في حركته الدائرية في اتجاه عمودي على اتجاه الجذب، فلن يحدث أي احتكاك مع الجسيمات. القمر بالنسبة له في حالة راحة.

    فيما يتعلق بالقصور الذاتي، فالسؤال أكثر تعقيدًا، رغم أنه يبدو واضحًا بديهيًا: تكون الأشرعة في حالة سكون عند أي سرعة ثابتة، ولكنها أثناء التسارع تواجه العديد من السيارات، التي تعارضها حتى يستقر الشراع عند سرعة ثابتة جديدة، ومن ثم كل القصور الذاتي. توازن القوى مرة أخرى.

    ورغم البساطة الظاهرة، فإن الشرح هنا أكثر تعقيدا، ومن الضروري أيضا إعطاء إجابة للسؤال الذي طرحه ماخ: ما العلاقة بالنجوم البعيدة؟ لماذا في مثال الصحون الطائرة، الصحن الوحيد الذي لا يدور، والذي لا يتم قياس قوة الطرد المركزي فيه، هو أيضًا افتراض بالنسبة للنجوم البعيدة؟

    دعونا نترك الأمر عند هذا الحد في الوقت الراهن. أريد أن نغلق موضوع الجاذبية، حتى نتمكن من الانتقال إلى الجزء المثير للاهتمام حقًا: ثبات سرعة الضوء في أي إطار مرجعي. إذا كنت قد صمدت إلى هذا الحد، فسوف أعدك بمكافأة لطيفة. في نهاية المناقشة المملة، يوجد اختبار دحض، وهو نفس التجربة التي أتحدث عنها، والتي يمكن أن تؤكد ما إذا كان هناك شيء ما في الفكرة بأكملها، أو إذا كانت مجرد تمرين فكري لوقت الفراغ.

    أخطاء؟

  618. إسرائيل

    أنا دائما أسأل الأسئلة عندما لا أفهم.
    لا أستطيع أن أسألك، على سبيل المثال، لأن الإجابة التي تقدمها ستكون خاطئة على الأرجح. وبعد أن يشرحوا لك سبب خطأها، ستظل مصرًا على أنك على حق. حتى لو أثبتت فتاة صغيرة أنك مخطئ، فإنك لن تقبل الحقيقة.

    أدعوك لدراسة المزيد من الفيزياء (على الرغم من أنك كبير جدًا في السن لدرجة أنك على ما يبدو أكبر شخص يتمتع بأكبر قدر من الخبرة في الحياة في العالم).
    وافهم بنفسك أين أخطأت، بدلاً من أن تطلب مني أو من الآخرين أن يوضحوا لك أين أنت على خطأ.

    أما بالنسبة لـ R. H.، إذا بقيت على قيد الحياة في هذا الموقع لفترة أطول قليلاً، فستتمكن من إثبات أنها في الواقع بكتيريا تعالج البكتيريا الضارة (لكن بدلاً من معالجتها، تبقى قريبة منها دون أي تأثير عليها 🙂).

    على أية حال، إذا كنت مثابرًا على دراسة الفيزياء، فمرحبًا بك لقراءة تعليقاتي مرة أخرى والتعليق عليها.
    وبالطبع أنا مستعد لفتح صفحة جديدة معك إذا فهمت أين كانت أخطائك وكنت على استعداد للاعتراف بها دون أن تشعر بالإهانة.

  619. إسرائيل،
    حتى الآن الأمر واضح. في رأيي المتواضع كان هناك رجل واحد اسمه نيوتن قام بصياغتها عام 1687 دون البلاستيسين وحتى بدون الطرق السريعة. وقد أطلق عليه اسم قانون نيوتن الأول:

    Lex I: Corpus omne المثابرة في حالتها المستقرة أو التحرك الموحد في الاتجاه المباشر، لا يوجد مصدر لتأثير قوي على الحالة المتغيرة.

    أو باللغة الماضية:
    "يحاول الجسم الحفاظ على سرعته طالما أن مجموع القوى المؤثرة عليه يساوي صفراً"

    يمكنك المضي قدما.

    أما بالنسبة لصديقك، ترى؟ لقد تناول الحبوب وهدأ وحاول التحدث عن الأمر الواقع. ولكن قريبا سوف يخرج الولد الشرير مرة أخرى.

  620. لقد كتبت ردًا على RA الذي ينتظر الموافقة. وفي هذه الأثناء يجب أن نوضح بعض الأمور:

    ر.ه.رفاعي.م

    إذا كنت لا تنوي الإساءة، فحاول تجنب الصفات مثل "غبي". سيعتقد الكثير من الناس لسبب ما أن نيتك هي الإهانة بالفعل.

    إذا لم تفهم شيئًا ما، فنحن نرحب بك دائمًا لطرح السؤال. قد يكون فهمي خاطئًا، ولكن قد يكون فهمك كذلك. ولهذا السبب نجري مناقشة.

    وعلى حد علمي، لن ينشأ أي احتكاك بسبب التأثيرات الكمومية، وإلا فإن نفس المنطق سينطبق على جميع المذبذبات الطبيعية في الطبيعة. إذا كنت قد درست الفيزياء، فانظر في أي كتاب مدرسي قياسي لطلاب الفيزياء في السنة الأولى في الفصل الذي يتناول الجاذبية، مثال النفق عبر KDA.

    على أي حال - لا يهم على الإطلاق. هذا مجرد مثال لفكرة يمكن بالتأكيد اعتبارها تخمينية حتى يتم دعمها بالصيغ أو التجربة. (لقد ذكرت ذلك بنفسك). إذا لم تلاحظ، فنحن نتعامل هنا مع نماذج نظرية، خارج التيار الرئيسي، قادرة على تفسير أشياء لا يوجد لها تفسير مرضٍ على حد علمنا. الأشياء هي:

    1. الجاذبية.
    2. الجمود.
    3. ثبات سرعة الضوء في أي نظام مرجعي.
    4. غير محلية.
    5. الكتلة والطاقة المظلمة.

    من الواضح لمعظمنا أن كل ما أقوله أنا أو يوفال أو مئير أو يهودا هو مجرد تخمين، إلا إذا تمكنا من تقديم الصيغ، أو حتى تجربة أفضل.

    إذا أردت التوضيح فاسأل. يمكنك بالتأكيد أيضًا تخطي التعليقات إذا وجدت صعوبة في ذلك. يمكنك أيضًا الخروج بأفكارك الخاصة.

    إذا كنت تريد فتح صفحة جديدة، فلا مشكلة. ولكن إذا هاجمت - فسيتم اختطافك.

    يائيل.
    يا له من جمال! تخرجت ابنتي مؤخرًا بدرجة البكالوريوس في الكيمياء الحيوية. تنوي تكريس حياتها للبحث الأكاديمي. إنها مدفونة باستمرار في الكتب والدراسات وهي مشتتة تمامًا.

  621. الصف التاسع ودرجة البكالوريوس في الفيزياء؟
    حقا يجب أن يكون هناك مصعد لهذا ..
    أين أنت تفعل شهادتك؟ في الجامعة المفتوحة؟

  622. ر.ح.

    ومن أجل الاستمرار، من المهم أن نتوصل إلى اتفاق على المبدأ الأساسي. ولذلك سأتطرق إلى التنازلات:

    ب. هذا ما تعلمناه في السنة الأولى. من المؤكد أن ذلك كان منذ سنوات عديدة، لذلك ربما أكون في حيرة من أمري أو أنه تم تغييره منذ ذلك الحين.

    على أية حال، كان ينبغي أيضًا أن تؤخذ في الاعتبار مسألة احتكاك الجاذبية في حالة سقوط جسم داخل النفق في حالة سقوط حر، حيث أنه مكتوب في الكتب أنه سيتم إنشاء حركة توافقية بدورة مدتها 84.2 دقيقة. ، فمن الواضح أنه لا يوجد احتكاك... (الاحتكاك بماذا؟ بعد كل شيء، لا يوجد هواء. جميع المذبذبات التوافقية الطبيعية تعمل على نفس المبدأ لملايين السنين).

    ثالث. انظر ب.

    رابع. سمعت. لكل عملية في معجل الجسيمات هناك مليارات العمليات دون ضرر. عن طريق الجو ولهذا السبب قلت "الأكثر".

    ال. سيتم مناقشة التاكيونات بالضبط وكيفية التقاطها إذا وصلنا إلى فكرة التجربة التي أخطط لها.

    و. ربما يمكننا التحدث مع والدي إذا لم يتمكن من حجبه في الزاوية بحاجز الفم، ثم على الأقل مضاعفة جرعة الحبوب؟

    بعد التفكير مرة أخرى، المضاعفة لن تكون كافية.

    ثلاثية!

    خلاصة القول، كان الغرض من كل هذه المناقشة هو الوصول إلى حقيقة أنه من الممكن أن تكون هناك جسيمات مشابهة لجسيمات لازاج تتمتع بالخاصية التالية: عندما تصطدم بالمادة، فإنها تنقل إليها كمية الحركة دون إهدارها على شكل حرارة، ولكن أعلى من ذلك. بسرعة معينة يمرون بها في المادة دون أن يؤثروا عليها أو عليهم.

    اذهب إلى الطريق السريع.

    Autostrada هو نموذج Lesage في بعد واحد من الطول. فبدلاً من الجسيمات الموجودة في جميع الاتجاهات وبجميع السرعات، نركز فقط على تلك الجسيمات التي تتحرك على طول خط مستقيم معين، في كلا الاتجاهين وبجميع السرعات. سنركز على سرعات تصل إلى 500 م/ث.

    يمكننا مقارنته بطريق سريع يضم 100 حارة، 50 في كل اتجاه، شمالًا وجنوبًا. تتحرك السيارة بسرعة 10 م/ث، 20 م/ث، …. ما يصل إلى 500 م / ث. لغرض المناقشة، نقوم بتركيز السيارات التي لها نفس السرعة على نفس المسار. لذلك، في المسار رقم 1، ستكون جميع السيارات بسرعة 10 م/ث، ثم 20 م/ث حتى الخط 50 حيث توجد سيارات بسرعة 500 م/ث. نفس الشيء في الاتجاه المعاكس.

    1. إذا رسمنا خطًا وهميًا عبر الطريق السريع، فإن الزخم الإجمالي للسيارات التي تعبر الخط يبدو 0 لأن السيارات من كلا الاتجاهين تقابل بعضها البعض.

    2. إذا استخدمنا بدلاً من الخط كيانًا، على سبيل المثال مجال القوة الكهربائية، أو من أجل التوضيح شراعًا أو مادة بلاستيكية خاصة يمكن للسيارات المرور من خلالها دون تشويهها، فيبدو أننا حصلنا أيضًا على 0 زخم على البلاستيسين.

    3. لنفترض حسب ما استنتجناه سابقاً أن البلاستيسين حساس فقط للسرعات التي تصل إلى 100 م/ث. السيارات التي تمر بالبلاستيك بسرعة أكبر من 100 م/ث تكون شفافة بالنسبة للبلاستيك وهو شفاف بالنسبة لها. أقل من هذه السرعة النسبية، ستؤثر السيارات بقوة على البلاستيسين.

    4. دعونا نرى ما يحدث عندما نضع البلاستيسين على الطريق السريع:

    جميع السيارات التي تبلغ سرعتها 110 م/ث فما فوق، في كلا الاتجاهين، تكون شفافة بالنسبة للمركبة

    5. تؤثر السيارات التي تبلغ سرعتها 100 م/ث أو أقل قوة على النبض، ولكنها تقابل بعضها البعض. القوة الإجمالية المطبقة على النبض تساوي 0 وتبقى في مكانها. (يحدث نفس الشيء لأي شراع في الهواء الساكن. فجزيئات الهواء السريعة تقابل بعضها البعض).

    6. ماذا سيحدث إذا وضعنا شراعًا مستويًا أو شراعًا ثانيًا يتحرك بسرعة 10 m/s باتجاه الشمال بالنسبة للطريق السريع؟

    السيارات على الطريق رقم 1 المتجهة شمالًا، والتي كانت تتحرك بسرعة 10 م/ث بالنسبة إلى المستوى A، ستكون سرعتها 0 بالنسبة إلى المستوى B. أولئك الذين تحركوا في الطريق 2 بسرعة 20 م/ث بالنسبة إلى أ، سيتحركون بسرعة 10 م/ث بالنسبة إلى ب، وما إلى ذلك. سيتم رؤية جميع السيارات الموجودة على جميع الطرق في اتجاه الشمال في المستوى B وهي تسير بسرعة أقل بمقدار 10 م/ث مما يراها في المستوى A. من ناحية أخرى، سينظر B إلى السيارات المتجهة جنوبًا على أنها تسير بسرعة أعلى بمقدار 10 m/s من المستوى A الذي سيقيسه.

    ومع ذلك، لاحظ أنه بما أن كل سيارة تبلغ سرعتها 100 م/ث أو أعلى في أي اتجاه تكون في الواقع شفافة للبلاستيك، فإن مجموع القوى على المستوى B هو أيضًا 0. ومن ثم، ستبقى بنفس السرعة البالغة 10 م/ث باتجاه الشمال. إذا عدنا إلى الشراع، حتى بسرعة 10 م/ث لن نشعر بأي رياح، وبنفس المنطق أيضًا بسرعة 200 م/ث، أو 350 م/ث، ولا يهم أي اتجاه. مهما كانت السرعة التي نضع فيها الشراع أو الشراع بالنسبة للطريق السريع، فسوف يظلون عند ذلك، عندما يكونون بالنسبة لهم في نظام "الهواء الراكد".

    سنأخذ فترة راحة لاستيعاب الأفكار والأسئلة، والتي سنناقشها لاحقًا.

  623. مرحبًا السيد إسرائيل شابيرا،

    نعم، أنا أدرس في الصف التاسع، لكني أحصل أيضًا على درجة البكالوريوس في الفيزياء. ولكي أكون صادقًا تمامًا، فإن بعض الأشياء التي كتبتها هنا هي أشياء أخبرني بها والدي.

    أتمنى أن لا أكون أسأت لأحد وما كتبه ر.ح. Refai.m لم يكن حقا في نواياي.

  624. إسرائيل

    مجروح
    (أنا آسف، لم أقصد أن أؤذيك)
    بخصوص النموذج الخاص بك فهو خاطئ من البداية.
    حتى لو قمت بحفر ثقب عبر CDH - فإن جدران CDH (التي تحيط بالفتحة) ستظل لها تأثير (حتى لو كان ضئيلًا جدًا) على كل جسيم يمر عبرها.
    حتى لو مرت 12-20 قذيفة عبر نظام الدفاع الصاروخي، فإنها ستظل تفقد جزءًا صغيرًا من زخمها.
    وحتى لو حاولت التفكير في نظام آخر بدلا من «ثقب في الأرض» من شأنه أن يشكل فراغا، فلن تجده، لأنه حتى في الفراغ هناك تقلبات كمية من شأنها أن تؤثر على الجسيم الكمي.

    - ويرجى ألا تنزعج من الأشخاص الذين يثبتون خطأك.

    ر.ح

    والأمر الغريب هو أنك تختار دائمًا الوقوف إلى جانب الأشخاص المخطئين (وأحيانًا الذين يكذبون أيضًا). حسنًا، هذه هي شخصيتك وهذا هو أنت.

  625. إسرائيل،
    و. أنا أتفق معك
    ب. توافق على أنهم سوف يمرون، غير متأكد من "لن ينقل الزخم". أعتقد أن الجاذبية بعد مرورهم بالمركز ستكون قوة معاكسة، أو نوع من الاحتكاك، مما سيخفض سرعتهم.
    ثالث. إذا كنت على حق في B - B فهذا غير صحيح وفي النهاية سيتم سجنهم في أحد النجوم.
    رابع. لا أوافق على ذلك، "التصادمات المرنة بين الجسيمات الأولية في معظم الحالات لا تسبب ضررًا أو احتكاكًا" - هل سمعت عن مسرع الجسيمات؟
    ال. وبقدر ما أفهم، فإن النيوترينو فريد من نوعه في هذه الميزة. جميع المشرعين الآخرين يتعارضون مع بعضهم البعض. انظر مسرع الجسيمات. تتصادم الفوتونات وتمتصها المادة وتنقل طاقتها إليها، فلا يمر الضوء عبر الشاشة. وفي نهاية المطاف، تتصادم النيوترينوات أيضًا، وهي حقيقة التقطتها أجهزة الكشف. ذات مرة عندما كانت قصص الخيال العلمي مثيرة للاهتمام حقًا وليست مجرد هراء خيالي، تحدثوا عن جسيم يسمى التاكيون الذي يتحرك فقط بسرعة أعلى من سرعة الضوء ويمر عبر أي شيء وبالتالي لا يتم اكتشافه.
    و. فهو شيء غريب. مصاب بالفصام، يحاول أحيانًا أن يقول أشياء معقولة، ولكن فجأة يخرج الولد الشرير ويعود إلى غرفته. ربما هي الأيام التي نسي فيها تناول الحبوب الأرجوانية.

  626. יעל
    أتمنى ألا تشعر بالإهانة لأنني سألت إذا كنت يائيل من الصف التاسع. أتذكر يائيل هذه من إحدى المقالات وتساءلت عما إذا كانت أنت. إنه في الواقع أمر رائع جدًا مع فتاة مهتمة بهذه القضايا.

  627. ر.ح.
    قبل أن نصل إلى الطريق السريع، أود أن نوضح بالضبط أي تقاطع وصلنا إليه، ونتزامن مواقفنا لتجنب سوء الفهم.

    دعونا نصف تجربة فكرية:

    1. يتم حفر نفق بين قطبي هيئة الصحة بدبي.

    2. وصول 20 قذيفة تتراوح سرعتها من 1 كم/ث إلى 20 كم/ث إلى مدخل النفق بفارق يوم واحد.

    3. لا نأخذ في الاعتبار حاليًا النظام الذي يتضمن أصل المقذوف وتاريخه. لنفترض أنها ليست ذات صلة حاليًا بالحجة. ( هذا لا).

    التأكيد أ: سيتم التقاط المقذوفات رقم 1-11 عن طريق سحب DCA وستنقل زخمها إلى نظام الأرض/القذائف.

    المطالبة ب: المقذوفات رقم 12-20 لن تنحصر بالجاذبية الأرضية ولن تنقل إليها أي قوة دفع، وستواصل طريقها إلى الفضاء متحررة من الجاذبية الأرضية.

    المطالبة ج: إذا وصلت لاحقاً في رحلة المقذوفات 12-20 أو أي مقذوف آخر أمام الأرض بسرعة تتجاوز سرعة الإفلات 11.2 كم/ث، فسيكون هناك تطابق لأرض أخرى، أو حتى لأرض أخرى. ألفاً على التوالي، وستتكرر العملية عندما تمر المقذوفات على الكواكب دون أن تترك أثراً عليها على شكل قوة دفع. وفي نهاية العملية، ستبقى المقذوفات بنفس السرعة تمامًا لكل خطوة معينة في العملية.

    المطالبة د: على عكس الاصطدامات بين القذائف والأرض، فإن التصادمات المرنة بين الجزيئات الأولية في معظم الحالات لا تسبب ضررا أو احتكاكا، كما في حالة جزيئات الغاز المحبوسة في منطاد أميشيرجاس، والتي يمكن أن تصطدم ببعضها البعض لسنوات عديدة دون أن تفقد متوسط ​​سرعتها أو تغير تركيبها.

    الادعاء 5: هذا النموذج ليس نموذجًا في ألما من أجل الجدل - هكذا تحدث الأشياء بالفعل في كثير من الحالات في عائلة الجسيمات الأولية، بسبب النيوترينو وتغلغله الكبير في المادة.

    المطالبة 6: القزم المعروف باسم R.H. Refai.M هو ثرثارة ميؤوس منها دون الحد الأدنى من فهم قوانين الفيزياء. في المواقع التي تتم مراقبتها عن كثب، كان من الممكن أن يتم حظره منذ فترة طويلة بسبب مضايقة المعلقين بشكل متسلسل.

    إذا كانت لديك أي شكوك حول هذه المطالبات، فهذا هو الوقت المناسب لتوضيحها قبل السير على الطريق السريع.

  628. إسرائيل،

    لقد فهمت مبدأ الشفافية فيما يتعلق بالأرض ذات الثقب. وهذا سيلزمك مستقبلاً أن تشرح ما يعادل الثقب في الجزيئات لأنه من الواضح أنه في قياس الأرض، إذا لم يكن هناك ثقب، مهما كانت سرعة القشرة/النيزك، فإنني سأفعل ذلك. لا أريد أن أكون على مقربة من التأثير (وبعيدًا عن البيئة)، لكن دعونا نترك الأمر الآن.
    يمكنك الاستمرار على الطريق السريع.

  629. رافين المعتوه.

    من الأفضل توجيه هذا السؤال إلى مايكل. أنت تابعه الأحمق، بالمعنى الحرفي للكلمة. على الرغم من أنه أمر غريب، إذا كان الشخص موهوبًا جدًا، فسوف يضيع وقتًا ثمينًا في DPR الخاص بالموقع.
    باختصار، خذ قيلولة بعد الظهر.
    هيا، اغضب، وابدأ بالسب. ولكن شيئا جيدا، إيه؟ ليس المستوى الفرعي المعتاد.

  630. إسرائيل

    وفقًا لأي توراة تقول: "سيكون الجسم (أ) شفافًا بالنسبة للجسم (ب) إذا تحرك الجسم (أ) بسرعة معينة عبر الجسم (ب)"؟
    في الأساس، إرحل. لن تفهم حتى لو ضربتك فتاة في الصف التاسع على رأسك بمطرقة وزنها 5 كيلو ومكتوب عليها "غبي".

  631. مائير، لقد راجعت النموذج الخاص بك قليلاً. إنه لطيف بلا شك، لكنه يعاني من مشكلة أنه ليس أساسيا. قرأت نفس النظرية منذ 6-7 سنوات، لكن لم أتمكن من العثور عليها في الروابط الموجودة على الكمبيوتر. إذا كنت ترغب (كثيرًا)، سأحاول معرفة ما إذا كان لدي مقال عن أجهزة الكمبيوتر القديمة على Boydam.

  632. انه لا يهتم. ومن الممكن أيضًا أن يكون كويكبًا من الفضاء. كانت المركبة الفضائية لأغراض العرض فقط. النقطة المهمة هي كما يلي: فوق سرعة معينة لوصول القذيفة إلى البلاد (في حالتنا - سرعة الهروب 11.2 كم/ثانية) سيمر الجسم عبر البلاد دون التأثير عليه، وبالتالي فهو "شفاف" بقدر ما كما يتعلق الأمر. ما أجاب على سؤال R.H. :

    "إذا قلت أن الجاذبية والتسارع ناتجان عن دفع الجزيئات للمادة، فلا يمكنك أن تأتي من الجانب الآخر وتدعي أن المادة شفافة بالنسبة لها. هل تقصد أنه عند السرعة المنخفضة تكون الجاذبية/التسارع وفي السرعة العالية تكون شفافة؟"

  633. إذا كان ما يحدث في منطقة المركبة الفضائية (أو أبعد) لا يهم، إذن "ما الذي يهم" الأرض سواء اكتسبت القذيفة سرعتها الأولية نتيجة طلقة مركبة فضائية أو نتيجة السقوط الحر من مكان أبعد؟

  634. لا ترتبط منصة الإطلاق بنظام القشرة/الأرض. يمكن أن تكون على بعد 100 سنة ضوئية من الأرض وليس لديها طريقة لمعرفة ما إذا كانت القشرة قد استحوذت عليها جاذبية الأرض أم لا.

  635. إسرائيل،
    تتسبب القذيفة المطلقة أيضًا في ارتداد منصة الإطلاق. إن إضافة المتجه V مضروبًا في كتلة الغلاف يساوي إضافة متجه في الاتجاه المعاكس لسرعة المركبة الفضائية مضروبًا في الكتلة.
    نعم، أنا من بين أمور أخرى من مناقشة إطالة الوقت من بعض الوقت. لم أعد أتذكر موضوع المناقشة وما إذا كنت مع أو ضد، ولكن من الواضح بالنسبة لي شخصيًا (وليس أن الملاحظات والتجارب التي تدعي خلاف ذلك قد مرت دون أن يلاحظها أحد) أن الجسيمات الأولية لها ساعة داخلية لا تتغير دقتها. تعتمد على سرعة حركتها لسبب بسيط وهو أنهم جميعا يتحركون بسرعة الضوء، وأن "الساعات في حالة سكون" هي تكهنات ملائمة للاستخدام في حسابات معينة، ولكن ليس لها أي أساس فيزيائي.

  636. יעל

    لا أذكر أنني تحدثت يومًا عن الكتلة المظلمة.

    هل أنت يائيل من الصف التاسع؟

    مئير

    إنه ليس صاروخًا، ولا يوجد به غازات عادم. مجرد قذيفة 155 ملم. فكر في الأمر كجسم في حالة سقوط حر مع متجه سرعة مضاف V في اتجاه الأرض.

    هل أنت مستنير بمناقشة العمل الإضافي في العام الماضي؟

  637. إسرائيل،
    إن الزخم الذي تكتسبه المركبة الفضائية في الاتجاه المعاكس لحركة الصاروخ و/أو زخم غازات العادم التي تسرع الصاروخ، اعتمادًا على طريقة الإطلاق) يعوض زخم الصاروخ من حيث المساهمة في زخم الإطلاق في الأرض.

  638. عذرا إسرائيل

    من الممكن أن يكون الخطأ مني: لا أفهم كيف يمكن الحديث عن نظرية عندما لا تكون هناك قاعدة بيانات يمكن الرجوع إليها. من السهل فحص الظروف المادية على الأرض، وربما حتى من خلال الحدس والخبرة الشخصية فقط. لا أستطيع أن أفهم كيف تعرف ماذا وكيف يحدث مع الكتلة المظلمة والطاقة المظلمة؟ تبعد عنا ملايين ومليارات السنين الضوئية. هل أنت مرتبط ببيانات البحث حول الموضوع؟

    الرابط الذي قدمته يفعل ذلك بالضبط - تم أخذ عدد من النظريات الرائدة واختبارها مقابل البيانات التي تم جمعها مؤخرًا من الأقمار الصناعية والتلسكوبات.

  639. آر إتش،
    وبما أننا تخطينا الخطوات الأساسية، فليس لدينا خيار سوى قبول حركة جسيمات المادة المظلمة كبديهية. لشرح الأذن، سأذكر فقط أن سلاسل "اللا شيء" ديناميكية.
    في الفيزياء نربط كل حركة بمفهوم "الطاقة". ولكن هذا المفهوم في حد ذاته لم يتم تعريفه في الفيزياء. إن تقديم الطاقة ككتلة، والعكس صحيح، هو أمر دائري ولا يحدد أيًا منهما. وفقًا لنموذجي، فإن تذبذبات جسيمات المادة المظلمة هي التي تحدد كلاً من الطاقة والكتلة

  640. R.H. Rafai.M،
    أولاً، أنا لا أدرس في جامعة هارفارد.
    ثانيًا، ليس هنا فقط، بل أيضًا في الجامعة التي أدرس فيها، لا يفهمون أفكاري.
    أنا لست الشخص الذي يصنع السلطة. الأفكار التي أطرحها بسيطة، لكنها لا تتماشى مع الحدس الجسدي. وكما قلت سابقًا، فإن الفيزياء هي نتاج متأخر للنموذج، وبالتالي لا ينبغي أن يُنسب إليه أي قانون فيزيائي.

  641. مئير.
    في الحالة الأولى، حيث يتم التقاط القشرة بواسطة جاذبية الأرض، يجب أن ينتقل الزخم إلى نظام القشرة/الأرض. وإلا أين ذهب زخم القذيفة؟ أين هو الحفاظ على الزخم؟
    وفي الحالة الثانية، تستمر القذيفة في حركتها بنفس السرعة التي أطلقت بها من المركبة الفضائية، وبالتالي ومن نفس اعتبارات الحفاظ على الزخم، لم يتم نقل أي زخم إلى إسرائيل.

    علاوة على ذلك، هذا ليس مجرد مثال لأغراض التوضيح. هذه هي الطريقة التي تعمل بها الأشياء على مستوى الجسيمات الأولية، مثل وصول الإلكترون أو البروتون إلى منطقة بها شحنة كهربائية، ولا يهم إذا كانت الشحنة تحمل نفس الإشارة أو العكس.

    ر.ح.

    "فإذا لم تكن هناك طاقة ولا قوى، فكيف تتحرك الجسيمات المظلمة؟"
    لا تحتاج الجسيمات أو الأجسام إلى طاقة أو قوى لتتحرك. بالنسبة لهم، طالما لا يوجد تسارع، فهم في حالة راحة.

    يائيل.

    من فضلك أرني أحد تعليقاتي بغض النظر عن الواقع، حتى أتمكن من التحسن.
    ولم أفهم حقًا أهمية الرابط الذي أرسلته. هل يمكن ان توضح

  642. إسرائيل،
    في رأيي، في جميع حالات الصدفة التي وصفتها، لن يكون هناك نقل صافي للزخم في نهاية العملية.
    في حالة قذيفة الرصاصة من مسافة بعيدة (لنفترض أنها تطارد الأرض) فإنها ستبطئ سرعة الأرض عند اقترابها (مع اكتساب زخم إضافي للقذيفة) وستسرع الأرض أثناء تحركها بعيدًا ( مع إرجاع نفس القدر من الزخم الذي اكتسبه أثناء الاقتراب).

  643. صباح الخير يا يوفال، إسرائيل ويهوذا،

    وأنا أتابع هذا الحديث، مع أنني يجب أن أشير إلى أنني لم أر أي شيء مثير للدهشة من حيث الابتكار، ناهيك عن الارتباط بالحقائق والواقع.

    إذا أردت، هناك رابط ذو صلة بموضوع المناقشة:

    http://arxiv.org/abs/1202.0892

  644. اليوبيل,
    تقول "في هذه المرحلة ليس هناك ما يمكن الحديث عنه حول الطاقة، لأن هذا المصطلح غير محدد على هذا المستوى من جسيمات المادة المظلمة".
    فإذا لم تكن هناك طاقة ولا قوى، فكيف تتحرك الجسيمات المظلمة؟

  645. في حياة يوفال، ماذا يعلمونك هناك في جامعة هارفارد؟ أعيد قراءة ما كتبته ويجب أن أخبرك أنه لو لم نتواصل في "حيدان" لكنت على يقين أننا كنا في مطعم "البابور" في الجليل الغربي - من غالبية السلطة التي قمت بإعدادها بين المفاهيم.

  646. يوفال

    هذا هو بالضبط ما أحاول أن أشرحه لك، عليك أن تفهم الأشياء الأساسية.
    في قاعدة النموذج الخاص بك، أنت تدعي أنه لا يوجد شيء - كيف يمكنك ادعاء ذلك عندما يكون "لا شيء" موجودًا عمليًا - حتى لو كان غير محدد؟
    أنت تمضي قدمًا من هناك وتدعي أنه حتى قبل ظهور الفيزياء، كانت هناك جميع أنواع جسيمات المادة المظلمة والميكانيكا موجودة بينها.
    - كيف يكون ذلك ممكنا؟ كيف تكون ميكانيكا الجسيمات ممكنة قبل وجود الفيزياء؟ إما أنني غبي أو أنت عبقري.

    هل ترى ما تفعله أنت تستبدل الفيزياء الحديثة بفيزياءك الخاصة. ويفعل ذلك بدون براهين على شكل معادلات رياضية.

    والطريقة الصحيحة هي، مرة أخرى، استخلاص الفيزياء التي لم نكتشفها بعد من الفيزياء الموجودة. وليس العكس. (لقد أحضرت لك بعض الأفكار التي تساعد نموذجك لكنك تتجاهلها).

  647. طالب علم
    "يبدو أنك لا تعرف ما هي العملية القابلة للعكس أو لا تفهم ما تعنيه."
    في الحقيقة أعتقد ذلك، لكن من أنا لأحكم؟

    ر.ح.

    "إذا قلت أن الجاذبية والتسارع ناتجان عن دفع الجزيئات للمادة، فلا يمكنك أن تأتي من الجانب الآخر وتدعي أن المادة شفافة بالنسبة لها. هل تقصد أنه عند السرعة المنخفضة تكون الجاذبية/التسارع وفي السرعة العالية تكون شفافة؟

    هذا بالضبط ما أقوله. مثال:

    يقال أنك تقوم بحفر نفق من القطب إلى القطب في هيئة الصحة بدبي. إذا أسقطت كرة في أحد القطبين، فإنها ستهبط بسرعة متزايدة إلى مركز الأرض ثم تبدأ في التباطؤ حتى تصل إلى القطب الآخر، حيث تتوقف وتبدأ رحلة العودة. إذا لم نقم بإزعاجها، فإنها ستبقى إلى الأبد في نفس الحركة التوافقية البسيطة.

    ماذا سيحدث لو تم إطلاق قذيفة مدفع من المركبة الفضائية المتوقفة على مسافة معينة من هيئة الصحة بدبي في اتجاه النفق؟ إذا وصلت القذيفة إلى سطح الأرض بسرعة أقل من سرعة الإفلات (11.2 كم/ث حسب أفضل ما أتذكر)، فسوف تلتقطها جاذبية الأرض وستبدأ أيضًا حركة توافقية، عندما تكون هذه المرة الحركة التوافقية تتجاوز حدود سطح الأرض. في هذه الحالة، ستقوم القشرة بنقل الزخم إلى نظام الأرض/الصدفة وتدفع الأرض للأمام، دون اتصال مباشر بها أو احتكاك أو أي تشوه.

    ومن ناحية أخرى، إذا كانت سرعة القشرة أعلى من سرعة الإفلات، فإن القشرة، بعد تحركها مسافة كبيرة من الأرض، ستترك الأرض بنفس السرعة التي كانت عليها من قبل تمامًا، ولن يكون للأرض القدرة على الحركة. نفس التأثير على سرعة القذيفة، والتي ستعود إلى نفس السرعة التي أطلقتها بها المركبة الفضائية على نفس المسافة من الأرض. لم يتحرك أي زخم، ولم يحدث احتكاك، ولم يحدث تشويه، وعاد الجميع إلى وضعهم السابق، راضين تمامًا. القشرة جاهزة للقيام بنفس التمرين مع مباراة KDA القادمة أو المليار التي تليها دون مشكلة وبدون طاقة إضافية.

    "ثم من يحتاج إلى الورق الشفاف؟ كيف يضيفون المعرفة؟"

    لذلك - في نموذج أوتوسترادا. لكن يجب أن أتأكد من أنك قبلت مبدأ الشفافية. لا يمكنك المضي قدما بدونها.

    اليوبيل.

    "عندما تأخذ ظاهرة فيزيائية وتسقطها على ظواهر أخرى، فإنك لا تقدم أي شيء ولكن فقط تخلق ادعاءً دائريًا." (كثيرون يفعلون ذلك. انظر نظرية ماكسويل. نجاح كبير.) على سبيل المثال، أنت تستنتج من "البندول الباليستي" نتائج تجربة مايكلسون مورلي، وبالتالي تدعي في الواقع أن "جسيمات" الضوء لها زخم. (لقد فعلوا ذلك. صيغة أينشتاين الشهيرة مبنية على هذا.) أنت تتجاهل ازدواجية الضوء الموجي والجسيمي (إيه؟ أين؟ لم نصل إلى ذلك بعد) أو تعتبر أنه من المسلم به أن رصاصة البندقية تضرب كتلة من الضوء. البلاستيسين لديه حركة موجية. (لا أفهم ماذا تقصد. لماذا جاليت؟) هناك الكثير من الانتقادات، ولكن مهلا مهلا.
    حاولت في براءتي أن أبين أن هناك حلولًا أخرى لأسئلتك، (حسنًا، أين؟) لكنك مشغول فقط بإخراج أشياءك. (لا يشارك في أي عملية خارجية. ويخضع للنقد. معظم التعليقات هنا تدور حول كلماتك. هل أنت أيضًا خارجي؟). إذا كنت ترغب في دراسة النموذج الخاص بي بعد الانتهاء من أفكارك، فمرحبًا بك. (أبذل قصارى جهدي لتعلم نظريتك، لكن الأمر صعب للغاية عندما لا تعرف ما هي. لقد اكتشفت للتو أنك تتوافق مع R.H. ربما أرسلت له النموذج. لم أفعل، لذلك أنا ليس لدي أي طريقة لمعرفة ما الذي تتحدث عنه بالضبط. ولا أنت تجيب على الأسئلة. سألتك ما الذي يثبت عجلة الدراجة النارية. ما هو التفسير الذي لديك لتجربة MM. كيف تفسر عدم الموضعية. الجاذبية . الأسئلة مركزة، والإجابات ليست كذلك.)

    علاوة على ذلك، بحسب ادعائك حول اختلاف سرعة الضوء على مسافات كبيرة من الأرض: ألا يتطلب ذلك قيمًا مختلفة لثوابتي الكهرباء والمغناطيسية؟

    حسنًا، الآن يبتسم الجميع.
    ؟؟؟؟

  648. اليوبيل,

    هذا هو التخمين في أحسن الأحوال. يمكنك تأكيد الفرضية بالتجارب التي ستجريها، إذا تمكنت من الوصول إلى القمر وقياس سرعة الضوء هناك. أو إلى أندومادا.

  649. يوفال

    لقد شككت في أن هذا هو الحال.
    الميل للخطأ هو بالتحديد في هذه النقطة حيث تعتقد أن حركة الضوء تتسارع، بدلا من الاعتماد على القياس (الصحيح) الذي ينص على أن سرعة الضوء ثابتة، وما الذي يسبب التسارع (تأثير دوبلر) ) هو تأثير آخر لم يتم شرحه بشكل كامل بعد ولكن يتم تعريفه على أنه الطاقة المظلمة. (مشكلة التحديد هي أنه بين التنبؤ الكمي والتنبؤ الكوني هناك فجوة في النتائج يتم التعبير عنها بحوالي 120 مرة من حيث الحجم - حسب ويكيبيديا)

  650. التخنيون,
    بدلاً من! يرجى إحضار ملاحظة من أندروميدا.
    وبالمناسبة، لدي تأكيد بأن سرعة الضوء ليست ثابتة. لنأخذ على سبيل المثال ظاهرة تحول نطاقات الامتصاص في الطيف القادمة من المجرات البعيدة. إن الاعتماد على الادعاء بأن سرعة الضوء ثابتة، مع إسناد الظاهرة إلى تأثير دوبلر، يؤدي إلى الاتفاق على أن المجرات تبتعد بسرعة متزايدة، وإلى الاعتقاد بوجود "الطاقة المظلمة". . ومن ناحية أخرى، فإن رؤية حركة الضوء متسارعة خلال رحلته إلينا يفسر الظاهرة دون الحاجة إلى الطاقة المظلمة.

  651. يوفال

    ماذا عن الفوتونات التي تأتي من المستعرات الأعظم؟ حقيقة أن الحسابات تظهر نتائج تشير إلى أن الفوتون يتحرك بسرعة الضوء، أليس هذا كافيا؟

  652. يوفال
    لا يمكنك القول أن الفيزياء في نموذجك قد تم بناؤها لاحقًا إذا لم يكن لديك دليل على كيفية حدوث ذلك. وهذه البراهين لا يمكن أن تكون إلا معادلات رياضية تشرح الفيزياء، ويمكن أن يستمد منها المعنى الفلسفي للظاهرة. وليس العكس.
    وكما قال ستيفن هوكينج: الفلسفة ماتت.

  653. التخنيون,
    وبقدر ما أعرف، فإن جميع التجارب التي تؤكد أن سرعة الضوء ثابتة في الفراغ أجريت في المنطقة المجاورة مباشرة لكوكبنا. لتوضيح حجتي، لن أطلب الذهاب إلى أبعد من كوكب المشتري وزحل، على سبيل المثال، لكنني سأكتفي بمثل هذه الملاحظة التي تم إجراؤها على سطح قمرنا.

  654. R.H. Rafai.M،
    صحيح أنني لم أتعمق بعد فيما قلته، لكن ذلك لأن وقتك لم يحن بعد. أقدم حاليًا مرحلة أساسية جدًا من النموذج الخاص بي. وستأتي مساهمة الفيزياء ومفاهيمها بعد حل الأسئلة الحالية

  655. اليوبيل,

    والدليل ليس في الرابط (أو في أي مكان آخر، لا توجد براهين في العلم). وينص الرابط على أن سرعة الضوء في الفراغ ثابتة وهذا ما تم تأكيده بالتجارب.

  656. R.H. Rafai.M،
    تتحدث في الفقرة الأولى عن الفراغ والطاقة والكميات والفيزياء بشكل عام. وهذا العلم ومصطلحاته غير موجودة في هذه المرحلة من النموذج الذي أطرحه ولكن يتم بناؤه لاحقًا، ومن العار أن نتوقف عندها الآن.
    بناءً على نصيحة ر.ه.، تخليت مؤقتًا عن قصة "لا شيء" وحدها، ووافقت مع بعض الأسف على الانتقال إلى مرحلة أكثر تقدمًا. أود العودة إليه، ولكن بسبب عدم قدرتي على شرحه، أخشى ألا أفعل ذلك قريبًا.

  657. يوفال
    مع كامل احترامي لـ R.H والطالب التخنيون - وهناك احترام - يشرح لك R.H النقاط التي ذكرتها أمامك (ولم تتطرق إليها). وقد ذكر أحد الطلاب بالفعل أنه ليس على دراية جيدة بفيزياء الكم ويفضل عدم التحدث عن أشياء لا يفهمها. صادق. و حقه

  658. طالب، التخنيون
    أنا لا أقول أنك لا تقول الحقيقة، ولكن في الرابط الذي قدمته لا يوجد دليل على أن سرعة الضوء في الفراغ ثابتة. هناك مثل هذا الادعاء، ولكن لا يوجد دليل. والإشارة إلى حقيقة أنها تدرس في جميع جامعات العالم (وليس فقط في التخنيون في السنة الثانية) لا تغير من حقيقة أن سرعة الضوء في الفراغ لم يتم قياسها على مسافة عشرات الكيلومترات بالخارج الأرض، ناهيك عن النظام الشمسي وخارجه. إذا لم يكن الأمر واضحًا منذ البداية، فإن ادعائي يتعلق بسرعة الضوء ليس فقط في بيئتنا المباشرة ولكن في كل مكان في الكون.

  659. مايكل روتشيلد

    إذا كنت تقرأ هذا التعليق بالفعل، فلماذا لا تنضم إلى المناقشة؟ (أعلم أنك مشغول بـ "الحروب الصليبية" 🙂 حتى أنك كتبت هنا بعض الأسئلة ليوفال تشايكين في بداية المناقشة، لكن مع ذلك، أسقط بعضًا من لؤلؤة الحكمة للقراء بيننا، حتى لا نحصل على بالملل على الأقل 🙂)

  660. يوفال
    سرعة الضوء في الفراغ ثابتة. ويتم تعريف الفراغ نفسه بأنه طاقة نقطة الصفر.
    فلا يوجد فراغ آخر غيره (في العالم الذي نعيش فيه. {ربما في عالم الخيال نعم..}).
    طاقة نقطة الصفر، لأن 0 درجة كلفن (-273.15 درجة مئوية)، هي الحالة التي يكون فيها النظام (المادي) في حالته الأساسية. عند درجة الحرارة هذه (وفقًا لنظرية الكم)، يكون كل جسيم كمي في أدنى حالة طاقة. إذا كانت حالة الطاقة لجسيم ما هي 0، فإن الجسيم سيكون في حالة سكون (أي بدون إزاحة)، وسيكون موضعه مؤكدًا. وهذا وضع مستحيل كميًا لأنه وفقًا لمبدأ عدم اليقين، من المستحيل الحصول على معلومات حول موضع وسرعة حركة الجسيم. ولذلك، وفقًا لميكانيكا الكم - لا يمكن لجسيم أن يكون في حالة طاقة نقطة الصفر، وبالتالي فإن الفراغ نفسه لا يمكن أن يكون في حالة الصفر المطلق. يجب أن توجد بعض التقلبات الكمومية داخل الفراغ نفسه.

    ما تقترحه هو حل (لا لا) الذي -بحسبك- يسبق الفيزياء وكل شيء، بينما
    الحل الخاص بك يقع ضمن حدود الفيزياء نفسها (وحتى يحاول استبدال الفيزياء الموجودة). وليس لديك طريقة لبناء شيء من شيء غير موجود. يمكنك فقط البناء على الفيزياء الموجودة والمساعدة في إيجاد حلول لها، بدلاً من بناء نموذج يشرح الفيزياء الموجودة.

  661. اليوبيل,

    تم توضيح ذلك في الرابط الذي قدمته (نرحب بقراءته بالكامل). وبما أنني لن أعلمك كيفية قراءته، سأذكر كمرجع أن سرعة الضوء في الفراغ، c، هي ثابت فيزيائي، ويتم تدريس ذلك، على سبيل المثال، في دورة الفيزياء 2 في التخنيون.

  662. التخنيون,
    عندما تقول إنك تقدم هذا الادعاء من أجل القراء الآخرين، فإنك تلمح إلى أنني مخطئ ومضلل. وحتى لا تقع في نفس الفئة، يُنصح بإحضار الدليل الصريح على كلامك.

  663. آر إتش،
    الوضع الذي ذكرته مستحيل هو بالفعل مستحيل في الفيزياء. لكن هذا النموذج من المفترض أن يبني الفيزياء من الميتافيزيقا، بحيث لا توجد فيه التعريفات المستخدمة في الفيزياء. في هذه المرحلة لا يوجد ما يمكن الحديث عنه حول الطاقة، لأن هذا المصطلح لم يتم تعريفه على هذا المستوى من جسيمات المادة المظلمة.
    ما لم يروه في مسرعات الجسيمات هو ما لم يبحثوا عنه. لكن ليس عليك أن تذهب إلى هذا الحد. هل الحالة الطبيعية للبروتونات في الكون هي أيونات موجبة أم أنها نوى ذرية للهيدروجين؟ كيف، وفقًا للفيزيائيين، تمكنت جميع البروتونات في كل نقطة في جميع أنحاء الكون من العثور على إلكترونات لنفسها؟

  664. اليوبيل,
    أنت تقول: "عندما يصطدم جسيمان، لا يحدث شيء. لا يوجد حفظ للزخم أو تصادم مرن معهم."
    ومن ناحية أخرى، أنت تدعي أن الجسيمات دقيقة، وهذا يعني أنه في نقطة ما في الفضاء في وقت معين لا يمكن أن يكون هناك جسيمان. وهذا وضع مستحيل وفقًا للفيزياء نظرًا لأن جسيمين يتحركان عكس بعضهما البعض. كلاهما لديه طاقة حركية. أم أنهما سيتوقفان ويلتصقان ببعضهما ثم أين ذهبت الطاقة الحركية؟ أو سيتحرك كلاهما مثل كرات البلياردو ثم يحدث تصادم مرن.

    بالإضافة إلى ذلك، لا أفهم كيف يتم إنشاء إلكترون من الثقوب الوامضة الموجودة في مركز البروتون؟ كيف لم يتم رؤية أي من هذه في مسرعات الجسيمات؟

  665. آر إتش،
    كلامك صحيح وليس فيه تناقض. في الواقع، في وسط المباني المتنمرة، يتم إنشاء "ثقوب" وتخرج هذه الثقوب. وهكذا، على سبيل المثال، يتكون الإلكترون المحيط بالبروتون في ذرة الهيدروجين.
    عندما يتصادم جزيئان، لا يحدث شيء. ليس لديهم مبدأ حفظ الزخم أو الاصطدام المرن. حركتهم عشوائية وعفوية. يمكنهم البقاء بالقرب من بعضهم البعض أو الاستمرار في المشي. وبما أنها لا تميل إلى الهروب من بعضها البعض، فإن العديد من هذه الجسيمات يمكن أن تتجمع بكثافة عالية (وكما ذكرنا أعلاه، عند الكثافة العالية جدًا سيكون هناك بعضها الذي سيصبح "ثقوبًا" في مساحة غير مبالية).
    تم العثور على دليل بحثي على اختلاف سرعة الضوء من فراغ إلى فراغ في تجربة إدينجتون. حركة الضوء في الفراغ القريب من الشمس أعلى منها في الفراغ البعيد عن الشمس. ويتجاهل الفيزيائيون هذا التفسير، وذلك لأنه بعد تجربة أخرى أجريت قبل بضعة عقود (مايكلسون مورلي)، تقرر قبول الاستنتاج القائل بأن سرعة الضوء ثابتة في كل فراغ.

  666. اليوبيل,
    هناك تناقض هنا. فمن ناحية تقول أن المادة المظلمة المركزة = المادة الباريونية. ومن ناحية أخرى، فأنت تقول أنه كما هو الحال في لعبة الحياة، تتحول المادة المظلمة المركزة إلى لا شيء. ومن ثم، في وسط أي مادة متنمرة يجب أن يكون هناك "فجوة" لما تسميه المساحة اللامبالاة. وقد لوحظ أي شيء من هذا القبيل؟ انا لا اظن ذلك.

    الشيء الثاني، أنت تتحدث عن الدقة، بمعنى، إذا كنت أفهمك بشكل صحيح، في مساحة معينة يمكن أن يكون هناك جسيم دقيق واحد فقط في وقت معين، أليس كذلك؟ إذا كان الأمر كذلك، فماذا يحدث عندما يصطدم جسيمان؟

    الشيء الثالث، أنت تزعم أنه "كلما زادت الكثافة، زادت سرعة الضوء". صحيح أن سرعة الضوء تختلف من وسط إلى متوسط، لكن في الفراغ، حسب الفيزياء المتعارف عليها، سرعته القصوى هي: ج. أنت تزعم أن هناك أنواعا من الفراغ تعتمد على تركيز المادة المظلمة. هل تم قياس سرعة الضوء في الفراغ بخلاف C؟ هل هناك أي دليل بحثي على الأنواع الفارغة؟ لا أعتقد ذلك أيضًا.

  667. ر.ه.، بعد إذنك سنقوم ببعض الترتيبات.
    إن القصة شبه الأسطورية حول "اللا شيء" و"الموجود" تهدف أساسًا إلى شرح شيء ما في تعريفات آليات المادة المظلمة. جسيمات المادة المظلمة ليست نتيجة مباشرة فورية. قصة "السلاسل" توضح أولاً وقبل كل شيء مفهوم الزمن. يمكنك تخطي هذه الخطوات والبدء مباشرة من المادة المظلمة وقبول تعريفاتها ببساطة كبديهية. سأرسل لك شيئًا عبر البريد الإلكتروني قريبًا، ولكن أولاً سأغلق بعض الزوايا هنا.
    نظرية جسيمات المادة المظلمة، ثلاثة بنود:
    * يتكون الكون من عدد متزايد من جسيمات المادة المظلمة وكمية لا حصر لها من الفضاء اللامبالي ("الفراغ"). مع وجود مساحة كافية حول جسيم المادة المظلمة، فإن الجسيم يحفز نفسه في بيئته لتكوين المزيد من الجسيمات؛ ونظرًا لوجود عدد كبير جدًا من الجسيمات في محيط مساحة صغيرة غير مبالية، تموت الجسيمات بسبب الكثافة وتترك وراءها مساحة غير مبالية.
    * الدقة: يشغل جسيم واحد مظلم حجمًا محددًا في الفضاء، ولا يشغل أي جزء منه في الوقت نفسه جسيم مظلم آخر (المساحة غير المبالية ليست دقيقة ويمكن للجزيئات اختراقها).
    *للجسيم المظلم حركة ذاتية مستقلة تختلف عشوائيا في اتجاهه وسرعته بلا حدود، باستثناء ما يتطلبه القسم السابق: حركة الجسيم المظلم لا توجد ضمن الحجم الذي يشغله في تلك اللحظة جسيم أولي آخر .
    في الأقسام الثلاثة أعلاه، لم يتم تعريف الكتلة والطاقة ولم يتم تعريف القوى. بشكل عام، لا يوجد شيء نعرفه من الفيزياء على هذا المستوى. "الفضاء اللامبالي" ليس "اللا شيء" الأسطوري. والجسيم المظلم ليس "لا شيء" الذي أصبح "نعم" أو العكس. باستخدام قصة سفر التكوين "لا شيء" من الممكن بناء هذه الأقسام الثلاثة من جوهر أبسط، ولكن كما ذكرنا قمنا بتخطيها (للأسف، لأن المسار نفسه جميل).
    جسيمات المادة المظلمة هي الزبالون. قد تصل مجموعات منها إلى بعض الكثافة، ولكن سيكون هناك دائمًا قدر معين من المساحة غير المبالية بين الجسيمات. إن كثافة الجسيمات التي تتجاوز عتبة معينة تخلق المادة الباريونية، في حين أن الفضاء اللامبالي هو المسؤول عن الظواهر الكهرومغناطيسية. يتم التعبير عن العلاقة بين كثافة المادة المظلمة والظواهر الكهرومغناطيسية في سرعة الضوء. كلما زادت الكثافة، زادت سرعة الضوء، ولكن عند كثافة عالية تتجاوز عتبة معينة (في جسيمات الباريون) لا يمكن للضوء المرور. كما أن الضوء يكشف عن سلوك مزدوج يمكن فهمه من هذا الوصف.
    الكتلة ليست خاصية للجسيم الباريوني، بل هي خاصية لكثافة المادة المظلمة المحيطة به. إنه يتجلى في الجاذبية (وكما يوضح مئير عميرام، أيضًا في القصور الذاتي) وفي التفاعل مع الموجات الكهرومغناطيسية (تنافر الجاذبية).
    إن الاختبار الدحض الذي اقترحته، "كيف يمكن أن توجد مجرات بدون مادة مظلمة؟"، ليس اختبارًا مثاليًا لأن كل مجرة ​​لها كثافتها الفريدة من المادة المظلمة ومن الممكن الإجابة على السؤال وقول ذلك في المجرات في السؤال، المادة المظلمة لها كثافة أقل من المجرات الأخرى.

  668. اليوبيل,
    حتى الآن، حتى قبل أن أفهم نموذجك، أشعر أنك تحاول القفز عاليًا جدًا وشرح كل شيء في كل شيء.
    لماذا لا تبدأ بعبارتك البسيطة: "المادة الباريونية المعروفة هي مادة مظلمة كثيفة". هذا الادعاء، في رأيي، لديه توقعات لدحض بسيط. على سبيل المثال، كيف يمكن أن تكون هناك مجرات بدون مادة مظلمة؟ لماذا تتركز كل المادة المظلمة هناك على شكل مادة باريونية؟

    قصة العين والهيش معقدة ومعقدة وفي رأيي لا يوجد أي دعم لها على الإطلاق وتبدو لي أشبه بالأساطير منها بالعلم.
    ولكن كما كتبت مازلت لا أفهم العلاقة المباشرة بين الجزء الأول (العدم) والثاني (المادة المظلمة) ومن الممكن أن الجزء الثاني لا يمكن أن يوجد بدون الأول.

  669. اليوبيل,
    ما زلت لا أفهم العلاقة المباشرة بين البداية والاستمرار.
    هل سلاسل "الجزيرة" هي المادة المظلمة؟ أو كل زوج من الجسيمات الأيونية (ما هي الجسيمات الأيونية؟) أصبح جسيم مادة مظلمة وما بينهما هو "جسيم" أيوني؟
    هل جسيمات "الفضاء" تلك الموجودة بين المادة المظلمة، السحابة المتموجة، هي جسيمات أيونية؟
    وأخيرا، ما هو "الميت"؟ العودة إلى كونها عين؟

  670. آر إتش، شكرًا لك
    صحيح. لقد تخطيت الشرح التفصيلي طويل ويحتوي على صور توضيحية.
    قادتني جمعية البريونات في ذلك الوقت (منذ 40 عامًا، عندما لم نسمع بعد عن البريونات والأبقار المجنونة) إلى تسمية نموذجي "بيولوجيا الفيزياء" :-).
    الطريقة التي أوضح بها كيف أن لا شيء يحل نفسه ولكنه يظل نشطًا هي من خلال بناء "السلاسل". كلمتان متجاورتان من "لا" معًا هما "نعم"، ولكن كل منهما على حدة هو "لا".
    جميع الجسيمات التي نعرفها هي نتيجة مرحلة متأخرة. حتى المادة المظلمة، وحتى كل شيء، هناك تماثل في كل شيء. يبدأ الاختلاف فقط في مرحلة المادة المظلمة، وهو ليس سوى كثافات مختلفة ترتب فيها المادة المظلمة نفسها. سبب الكثافات المختلفة هو هندسي. ميل الجسيمات في الطور الذي يسبق المادة المظلمة هو التكاثر. ولكن عند كثافة تتجاوز عتبة معينة "يموتون" (تذكرنا بلعبة الحياة). لكن الترتيبات الهندسية المختلفة تملي كثافات قصوى مختلفة.
    تتجمع المادة المظلمة معًا لتشكل الهياكل. عند كثافة ثابتة، تحافظ الهياكل على الاستقرار عن طريق التوازن الديناميكي. في الفيزياء نسمي البنية المستقرة "البروتون"،
    إن الفضاء بين جسيمات المادة المظلمة والذي أسميه "الفضاء الفارغ" (ولا ينبغي الخلط بينه وبين الفضاء الفارغ في الفيزياء والذي ليس فارغا على الإطلاق وفقا للنموذج) يشكل أيضا نوعا من الجسيمات. لكن هذه ليست جسيمات "دقيقة" مثل جسيمات المادة المظلمة. يمكن أن تظهر على شكل جسيمات عندما تكون محاطة بجزيئات المادة المظلمة، ولكنها يمكن أن تظهر أيضًا كجزء من نوع ما من "السحابة". وهي ذات طبيعة مزدوجة، بحيث يمكنها التحرك كموجة، ولكن عندما تواجه مثل هذه الموجة حاجزًا فإنها تعود إلى التصرف كجسيم. يعتبر البروتون حاجزا لأن كثافة المادة المظلمة بداخله تحد من حرية حركة الجزيئات وبالتالي أيضا حركة الفضاء الفارغ بينها.
    لقد رأيت للتو النتيجة النهائية. سأرسل لك رسالة بالبريد الإلكتروني قريبا. ولكن بما أنني كتبت بالفعل، فأنا أقوم بالنشر.

  671. في ضوء ردك، قمت بمراجعة إعدادات الحظر واكتشفت أنه في الماضي كان هناك شخص يعرف نفسه بنفس الطريقة ويميل إلى الرد بتعليقات مسيئة وغير مدروسة.
    وبما أن هذا الشخص توقف عن إزعاجي على ما يبدو، فقد قمت بإلغاء الحظر وأتمنى أن تواجه مشاكل أقل من الآن فصاعدًا

  672. إسرائيل،

    ونسيت أن أذكر أنه يوجد بالفعل تعليق في تعليقاتي. لماذا - سؤال ممتاز. في الماضي قيل لي أن السبب هو الروابط التي أرفقتها، لكن في التعليقات الأخيرة لا يوجد رابط.

  673. اليوبيل,
    انتظر، انتظر، أنت أيضًا. أنت تعمل بسرعة كبيرة بالنسبة لي
    أعجبتني فكرة التفاعل المتسلسل حيث تصبح نواة "اللا شيء" شيئًا وتبدأ في تغيير تلك المجاورة لها وفقًا لذلك (يذكرني بالبريونات البيولوجية من عالم آخر، وزمن آخر ودراسة أخرى، ولكن هذا هو ارتباطي).

    هناك مشكلة هنا. ماذا يحدث الآن ليش + إن؟ إذا كان الجواب لا، فستبقى العملية عند مستوى لا + لا = نعم ويتم تدمير هذا نعم. إذا كانت الإجابة بنعم خاملة ولا تستجيب بأي شيء، فسيكون كل شيء سريعًا "نعم".

    لذا، إذا تم تنفيذ مثل هذا التفاعل المتسلسل، في أي وقت (يعتمد على سرعة هذا التفاعل المتسلسل، هل يقتصر على سرعة الضوء؟) سيتحول كل شيء إلى لا شيء، أليس كذلك؟

    الخطوة التالية، هذا هو المكان الذي علقت فيه. ما هي طبيعة هذه الـ "نعم" الأولية؟ كيف أصبحت كل الجسيمات التي نعرفها؟ ولماذا لا يكون موحدا إذا جاء من زي "لا شيء"؟
    كيف يمكن استخلاص تعريفات المادة المظلمة وازدواجية الجسيمات وما إلى ذلك من هذا؟
    إذا كنت تشعر بعدم الارتياح أثناء الجدال أثناء الرد على مقال ما، فيمكنك إرسال بريد إلكتروني إليّ.

  674. إسرائيل،
    انتظر، انتظر، قبل الطريق السريع.
    لا يمكنك أن تمسك عصا من كلا الطرفين أو تأكل كعكتك وتتركها كاملة.
    فإذا قلت إن الجاذبية والتسارع ناتجان عن دفع جميع الجزيئات للمادة فلا يمكن أن تأتي من الجانب الآخر وتدعي أن المادة شفافة بالنسبة لها. هل تقصد أنه عند السرعة المنخفضة تكون الجاذبية/التسارع وفي السرعة العالية تكون شفافة؟
    إذن من يحتاج إلى الورق الشفاف؟ ماذا يضيفون إلى المعرفة؟
    النقطة الثانية، حسب فهمي المتواضع للفيزياء، الكرة التي تخترق كتلة بسرعة هائلة لا تترك دون تأثير، يجب أن تنخفض سرعتها. العملية التي وصفتها للإلكترون يجب أن تؤثر على الإلكترون بطريقة ما. أي أنه إذا كانت السرعة المنخفضة تؤثر على المادة وتتفاعل معها، فمن غير الممكن أن يختفي التفاعل عند السرعة العالية. حسنًا، لن تتحرك الكتلة، لكن الكرة "ستشعر" بالتحول في الحرارة وفقدان السرعة. ويبدو لي أنني أكرر نفسي قليلاً. لذا يرجى التوضيحات.

  675. إسرائيل،

    يبدو أنك لا تعرف ما هي العملية العكسية أو لا تفهم ما تعنيه. إذا كان النظام أو محيطه بعد كل جولة من النجاح في حالة مختلفة (لأي سبب، بما في ذلك الجسيمات المخترعة)، فإن هذه العملية لا رجعة فيها بحكم التعريف. إذن، لا أعرف لماذا تبحث عن تفسيرات "خاصة بك" - فهناك بالفعل بعض منها. إذا كانت العملية لا رجعة فيها، تزداد الإنتروبيا. إذا كان قابل للعكس، لا.

  676. القليل من النقد لـ "نموذجك" ، حسنًا:
    عندما تأخذ ظاهرة فيزيائية وتسقطها على ظواهر أخرى، فإنك لا تقدم أي شيء، بل تخلق فقط ادعاءً دائريًا. على سبيل المثال، تستنتج من "البندول الباليستي" نتائج تجربة مايكلسون مورلي، وبالتالي تدعي في الواقع أن "جسيمات" الضوء لها زخم. أنت تتجاهل ازدواجية الضوء الموجي والجسيمي (أو تفترض أن الرصاصة التي تضرب كتلة البلاستيسين لها حركة موجية). هناك الكثير من الانتقادات، ولكن مهلا مهلا.
    حاولت في براءتي أن أبين أن هناك حلولا أخرى لأسئلتك، لكنك مشغول فقط بإظهار أمورك. إذا كنت ترغب في تعلم النموذج الخاص بي بعد عسى أن تنتهي من إرهاق أفكارك، أهلاً وسهلاً.

  677. اليوبيل.

    بادئ ذي بدء، سأقول رأيي: معظم النماذج المادية غير السائدة هي "موهمة" بدرجة أو بأخرى، إذا لم تكن مدعومة بصيغ أو تجارب لا لبس فيها.

    على الموقع:

    http://www.wbabin.net/

    وفي مواقع أخرى، يمكنك أن تجد مئات عديدة من النظريات والمقالات والأفكار، بما في ذلك نظرية أن أينشتاين الغبي لم يخترع النسبية على الإطلاق، بل نقلها عن زوجته الأولى.

    "نموذجي" ليس استثناءً، مما جعل صديقي نير يوصي بإرسالي إلى المستشفى قسريًا مرتين في الأسبوع على الأقل. من التواصل السابق معك، حصلت على انطباع بأنك تحب هذا النوع من الفكاهة السوداء (تذكر "الوحش مسموح من الإنسان؟")، لذلك سمحت لنفسي بالتعبير عن نفسي كما فعلت. ولكن للأسف لقد آذيتك، وأنا آسف لذلك. أنت تقوم بعمل مهم على الموقع، سواء من حيث أسلوب الكتابة السلس والممتع، أو في الشبكة السرية التي تنسجها بين المعلقين من خلال رسائل البريد الإلكتروني. تهنئة!

    فيما يتعلق بالنموذج الخاص بك:
    من الواضح أنني أجد صعوبة في الفهم، لأنني لم أتمكن من فهم كيف سنحصل على القصور الذاتي في المثال الذي قدمته لعجلة الدراجة النارية. ZA: لماذا يكون من الصعب هز العجلة أثناء دورانها، ولكن عندما لا تدور يكون الأمر سهلاً؟

    ولو أمكنك أنت أو أحد المعلقين أن تشرح لي هذه النقطة أكون شاكرا لك.

    فيما يتعلق بالباقي: من الممكن أن يكون نموذجك هو بالفعل وصفًا صحيحًا لتكوين الكون. ما أبحث عنه هو اختبار الدحض، وكما هو الحال دائمًا، الإجابة على السؤال: لماذا؟

    من الصعب بعض الشيء بالنسبة لي أن أتابع تطور النموذج، لأنني لم أره قط قطعة واحدة، ولكن فقط في أجزاء: في البريد الإلكتروني، في التعليقات، في الأسئلة. إذا كان بإمكانك إرساله إليّ بالكامل، أو ببساطة نشره على الموقع الإلكتروني أو في رابط، فسيكون ذلك سهلاً للغاية. عندما تفعل ذلك، يمكنني الرد بجدية.
    على سبيل المثال: فكرتك عن الحركة بالنسبة للمادة المظلمة فقط هي فكرة مثيرة للاهتمام، ولكن بما أنني لا أملك أي فكرة عما تقصده بالمادة المظلمة، فأنا عالق ومحبط.

    لذا سأنتظر بقية النموذج يا أخي.

    وفي هذه الأثناء، ماذا عن بعض الانتقادات لأفكاري؟ هذا هو مكان الحرث والجز..

  678. طالب.

    يبدو لي أن هناك نوعاً من التعليق في ردودكم، لأنها دائماً تظهر متأخرة، ومن الممكن أن تفوتها.

    لنكرر المثال:

    لوحة دوارة في الفضاء. هل هناك تغيير في إنتروبيا نظام اللوحة بين الدوران 20 والدوران 80؟

    الجواب القياسي هو لا. هذه العملية قابلة للعكس. وفقًا لـ "نموذجي"، فإن الكون يتكون من "جسيمات" و"تنتشر" مثل جزيئات الغاز في بالون قابل للنفخ - نعم. في كل دورة، ستعود اللوحة إلى موضع مختلف في النظام، حيث يوجد عدد أقل من الجزيئات لكل وحدة حجم.

    إذا طبقنا نموذج الانفجار الأعظم، فيبدو لي أن هناك شيئًا ما، لأنه من الناحية النظرية يمكن أن تكون سرعة الدوران الزاوي للوحة دورة واحدة لكل مليار سنة، لذلك ليس هناك شك في أن هناك زيادة في إنتروبيا الكون. النظام الذي يضم اللوحة والكون بعد كل دورة. وبما أن نظام الصفائح وحده يشمل جزءًا من الكون، فإن إنتروبيا هذا النظام تزداد أيضًا. لا؟

    ر.ح.

    أنت أعور. لقد طرحت مشكلة الفكرة، يجب أن نكررها حتى نتمكن من تحسينها:

    "بعد سرعة معينة، تتوقف الجزيئات ببساطة عن إحداث تأثير، وبالتالي تصبح "شفافة" من حيث المادة التي تصطدم بها (حتى الآن أفهم أن هذا هو ما تحاول القيام به باستخدام بندولك الباليستي، لا أفهم ذلك تمامًا) أوافق على ذلك لأن الرصاصة التي تمر عبر كتلة بسرعة عالية لن تحركها فعليًا ولكنها ستحدث ثقبًا وتدمر المادة وتطلق حرارة عالية لذا لا يمكنك القول أن الكتلة شفافة)

    هذه هي مشكلة الاحتكاك الثانية في نموذج ليساج - والتي أثارها اللورد كلفن إذا كنت أتذكر بشكل صحيح. مشكلة الديناميكا الحرارية. احتكاك الجزيئات مع المادة.

    أولاً، لاحظ أننا لا نزال نحصل على الجاذبية والقصور الذاتي، لكنها مشكلة. لكنني أعتقد أن هناك حلاً لهذه المشكلة، وهو الحل الذي لم يكن بإمكان اللورد كالفين وأسلافه التوصل إليه بالمعرفة التي كانت لديهم في ذلك الوقت.

    السؤال 1: هل يوجد مثل هذا النوع من الجسيمات، الذي يمر بالكتل دون أن يؤثر عليها إطلاقا، وهو قادر على اختراق ليس فقط جذع شجرة، بل الأرض كلها، بالإضافة إلى 100 شمس، دون أن يتأثر أو يؤثر على الكتل، دون أن يفقد السرعة من خلال الاحتكاك، جسيم حتى أن اسمه مشتق من مصطلحات "شفاف" "غير مبال" أو باختصار "محايد"؟
    .....

    فلنكمل:

    وهكذا، (وماذا إذن؟ كيف يترتب على ذلك ما كتبته من قبل؟ هل تقصد أن أجهزة الكشف لدينا تصبح شفافة من حيث الضوء فوق سرعة الضوء؟ إذا كان الأمر كذلك، فلماذا بالضبط بهذه السرعة؟ ولماذا لا نستطيع ذلك؟ صنع المزيد من الكاشفات "الصلبة" التي تقيس سرعات أعلى؟) كل موجة دعه يتقدم في الموقع النشط، سوف يتقدم بالفعل بسرعات عديدة، وربما حتى جميع السرعات، ولكن بالنسبة للمساح، أو المراقب، سيظهر دائمًا كما يلي: لها سرعة واحدة فقط (في حالتنا - سرعة الضوء)."

    السؤال الثاني: أليس هذا الجسيم ذاته (النيوترينو! النيوترينو!) مشتبها به في ميله إلى تجاوز سرعة الضوء في بعض الأحيان؟

    .....

    ولكن كيف يحدث هذا؟ كيف يمكن لجسيم بسرعة معينة أن يمر عبر كتلة كما لو كان شفافا بالنسبة لها؟

    فكر في وصول الإلكترون إلى منطقة سالبة الشحنة. الشحنة تطرد الإلكترون وتدفعه للخلف. لكن بعد تجاوز سرعة معينة للإلكترون سينجح في اختراق المنطقة، والشحنة التي قاومته سابقا، تدفعه الآن من الخلف، لتكون النتيجة الصافية أن الإلكترون يخترق الشحنة دون أي تأثير صافي على الشحنة. ، لا فقدان للطاقة، لا حفرة، لا احتكاك، لا تدمير، وبدون حمير وحشية. الإلكترون فوق سرعة معينة يكون ببساطة شفافًا من حيث الشحنة. لا؟

    ولماذا سرعة الضوء؟ يجب أن تكون هذه سرعة معينة ومحددة، كما هو الحال في أي نموذج هيدروديناميكي آخر. هذا هو ما خرج.

    وربما يكون من الممكن بناء المزيد من أجهزة الكشف الصلبة، التي من شأنها حجب الجسيمات أو الإشعاع، لكن الأمر معقد للغاية. تصنع الكاشفات من مادة، كما ذكرنا، شفافة للإشعاع، مثل الأرض شفافة للنيوترينوات.

    تتصرف الجسيمات - أو الإشعاع - في النموذج مثل الإلكترون، على الرغم من أنها ليست إلكترونات بالطبع. إنها أصغر بعدة مرات من أي شيء نعرفه. ويجب أن تكون كذلك للسماح بحركة متموجة للجسيمات. لكن هذه ليست النقطة الآن. هل ترى أنه ليس فقط من الممكن نظريًا أن يصبح الجسيم "شفافًا" عند سرعة معينة، بل أن مثل هذا الجسيم موجود أيضًا؟

    إذا كان الأمر كذلك، فيمكننا الانتقال إلى المرحلة التالية، وهي مرحلة رائعة جدًا، مرحلة الطريق السريع.

  679. آر إتش،

    ربما شعرت بالإهانة حقاً من إسرائيل، لكن الأهم من ذلك أنني تخليت عنها. أنا فقط لا أستطيع أن أشرح له.
    لقد فهمت الأمور بشكل صحيح، وكذلك تعليقك "(ما معناه)" في محله. إن حقيقة أن العدم (في مصطلحي "النفي") نشط، أتجاهلها من الواقع الذي نعرفه وأقبله حاليًا كبديهية.

    الانفجار الكبير الذي يتحدث عنه الفيزيائيون هو ظاهرة محتملة مستمدة من نموذجي، وأنا حقا أقدمها لاحقا (بعد عدة مراحل من التعريفات) على النحو التالي:
    في اللحظة المحددة، التي نبدأ فيها المسرحية، يكون الكون لا نهائيًا (أو ينمو بلا حدود إلى اللانهاية)، أحادي البعد لا رجعة فيه في الزمن، ومتعدد الأبعاد يمكن عكسه في المسافة، ومأهول بالكامل بـ "الوجود الأولي"* في حالة اللامبالاة*. اسم "نعم الأولي" واحد فقط نشط. ومن حوله يتأثرون به، وينشطون ويستمرون ويمررون هذه الميزة بلا حدود، ونحصل على "الانفجار الكبير".
    (* تم تعريف "الذات البدائية" و"الحالة اللامبالاة" بطريقة ما في فصل سابق كنتيجة للنفي).

    لقد سألت ما إذا كان "الكائن" الذي تم إنشاؤه عبارة عن عدد كبير من الجسيمات. الجواب هو "لا" أو "ليس بالضبط". وبما أن النفي ينفي نفسه دون انقطاع، تتشكل "سلاسل" من النفي تكون معًا "موجودًا" ولكن كل حلقة هي نفي. كلمة "سلاسل" موجودة بين علامتي تنصيص، لأنها ليست بالضرورة بنية مكانية. إذا أردت، يمكنك مقارنتها بالعناصر الأساسية لنظرية الأوتار.

    وعلاوة على ذلك، ليس كثيرا. وفي مرحلة ما وصلت إلى تعريف جسيمات المادة المظلمة ومن ذلك إلى "مستعمرات" البروتون وتعريف الجاذبية. ومن اتجاه مختلف تمامًا "مفاجئ" يأتي تعريف الفوتونات (والإلكترونات) المشتق أيضًا من هذا النموذج. يشرح النموذج بشكل جيد ازدواجية الضوء الموجي والجسيمي وكذلك التفاعل بين الإشعاع الكهرومغناطيسي وجزيئات المادة المظلمة والذي يتجلى، من بين أمور أخرى، في ظاهرة اضمحلال الجاذبية.

    إن الحركة النسبية بين الأجسام ليست في الواقع بين الأجسام والأشياء الأخرى، بل بينها وبين المادة المظلمة. ومن الواضح أن هناك حركة نسبية بين الأجسام، لكن تأثير الحركة على الأجسام يأتي من التفاعل مع المادة المظلمة وليس من التفاعل بين الأجسام. وبما أننا حتى اليوم تجاهلنا المادة المظلمة، فقد "أجبرنا" على أن ننسب الحركة إلى ما نعرفه، وكانت هذه أشياء أخرى - مما فتح الباب أمام الكثير من المفارقات. كما أن الجاذبية المرصودة بين الأجسام لا تنشأ منها، بل من تركيز المادة المظلمة حولها.

    هناك العديد من الأسئلة التي يجيب عليها النموذج، ولكن يكفي الآن.

  680. اليوبيل,

    منطق بدء النموذج الخاص بك واضح وصحح لي إذا كنت مخطئًا.
    1) نريد أن نعرف ماذا كان في البداية. ولهذا الغرض نفترض أنه كانت هناك بداية وأن الأشياء لم تكن موجودة منذ الأزل.
    2) بما أن كل ما يأتي من شخص ما لا يؤدي إلا إلى دفع البداية إلى الوراء، فيجب أن نفترض أن البداية كانت من لا شيء أو لا شيء أو صفر أو ما نسميه، الظلام فوق الهاوية يبدو وكأنه وصف جيد بالنسبة لي.
    3) أن العين كان عليه أن يخلق البداية لأن كل شيء سواه ومتضمنا له ليس شيئا وليس هناك شيء.
    4) ومن ثم عذب نفسه (مهما كان معناه) ليش (سنترك الأسئلة كيف ولماذا ونأخذها على أنها حقيقة).
    أليس كذلك؟
    الأسئلة من هنا حسب النموذج الخاص بك:
    1) هل الانفجار الكبير بحسب النظريات التقليدية (ولا تبدأ بإهانتي الآن كما في إسرائيل) هو عملية التأين التي تتحدث عنها؟ إذا لم يكن كذلك، ما هو الفرق؟ بعد كل شيء، تتحدث نظرية الانفجار الكبير أيضًا عن حالة بلا مكان وخلق الفضاء والمادة من مصدر.
    2) ما هي طبيعة ذلك المخلوق؟ وهل هو مكون من جزيئات كثيرة أم كان هناك جسيم في البداية خلقت منه الأشياء، وإذا كان الأمر كذلك فكيف؟
    3) ما هو التالي من هنا

  681. كذلك القول
    ننسى العجلة. اعتقدت أنك ستكون سعيدًا بالنسبة لي لاستخدام شيء أحضرته، لكنه ليس مثالًا جيدًا.
    لم أقل "مستعمرات الجسيمات التي تتبادل الجسيمات مع بعضها البعض". أنت لم تأخذ ما قلته على محمل الجد، لذلك أخطأت. إذا كان هناك تبادل للجسيمات بين المستعمرات، فهذا ليس ذا صلة في الوقت الحالي. يتم تدمير وبناء المستعمرات باستمرار، لكنها تحافظ على حجم ثابت، لأنها في حالة توازن ديناميكي مع بيئتها. عندما يتم تدمير مستعمرة في مكان واحد وبنائها في مكان مجاور، فقد تغير مكانها. لكن الجزيئات التي يتكون منها لم تتحرك معًا، بل تغيرت مع البيئة. ولذلك فإن الحركة متموجة.
    لا أعتقد أنك فهمت ولكني أحمل اللوم على نفسي لأنني لم أشرح جيدا.
    من فضلك أخبرني بما فهمته (وليس فقط "النسخ واللصق"، من فضلك) وسنرى ما إذا كان بإمكاننا الاستمرار.

  682. إسرائيل،

    هل يمكنك توضيح الجمل التالية التي كتبتها (أسئلتي وتعليقاتي بين قوسين):

    "بعد سرعة معينة، تتوقف الجزيئات ببساطة عن إحداث تأثير، وبالتالي تصبح "شفافة" من حيث المادة التي تصطدم بها (حتى الآن أفهم أن هذا هو ما تحاول القيام به باستخدام بندولك الباليستي، لا أفهم ذلك تمامًا) أوافق على ذلك لأن الرصاصة التي تمر عبر كتلة بسرعة عالية لن تحركها فعليًا ولكنها ستحدث ثقبًا وتدمر المادة وتطلق حرارة عالية لذا لا يمكنك القول أن الكتلة شفافة)

    وهكذا، (وماذا إذن؟ كيف يترتب على ذلك ما كتبته من قبل؟ هل تقصد أن أجهزة الكشف لدينا تصبح شفافة من حيث الضوء فوق سرعة الضوء؟ إذا كان الأمر كذلك، فلماذا بالضبط بهذه السرعة؟ ولماذا لا نستطيع ذلك؟ صنع المزيد من الكاشفات "الصلبة" التي تقيس سرعات أعلى؟) كل موجة دعه يتقدم في الموقع النشط، سوف يتقدم بالفعل بسرعات عديدة، وربما حتى جميع السرعات، ولكن بالنسبة للمساح، أو المراقب، سيظهر دائمًا كما يلي: لها سرعة واحدة فقط (في حالتنا - سرعة الضوء)."

  683. وهذا ما فهمته مصحوبا بالتعليقات:

    إليك رسم تخطيطي للنموذج، ولكي أجعلك سعيدًا سأبدأ بأوصافك: عجلة دراجة نارية تدور في الماء. شاهد العجلة نفسها مصنوعة من الماء. (حسنا حتى الآن). ولتمييزه عن الماء الذي نقع فيه، لنفترض للحظة أنه متجمد، فهو مصنوع من الجليد. (ربما للتوضيح.) وبما أن هذا هو الحال، فإنه يجمد الماء المحيط به. (ماذا؟ لماذا؟ حسنًا، دعونا نرى أين يجدف يوفيللي) ولكن التأثير متبادل وهناك أيضًا ماء يتم إذابته منه وإطلاقه في البيئة. (تذكر هذا من الفصل الأول، مستعمرات الجسيمات التي تتبادل الجسيمات مع بعضها البعض). فمن ناحية، تتحرك نسبة إلى الماء الذي تغمر فيه. (مقبول) لكن في المقابل فإن الحركة النسبية ليست بين العجلة والماء إطلاقا، وكل ما يتحرك هو حالة الركود. (يبدأ بالتوضيح) لذلك، في هذا التشبيه، يمكن اعتبار حركة الجسيم داخل المادة المظلمة ذات طبيعة موجية. (شو؟ من أين أتت الشخصية المتموجة؟) هل الأمر واضح حتى الآن؟ (لا! من الواضح أن هذا ما يدعيه يوفيللي، لكن ليس من الواضح السبب!)

    إذا أمكن، قدم توضيحات للتعليقات التي أدليت بها بين قوسين.

    حاول أن تفعل الشيء نفسه، مع المراجعات بين قوسين، لفكرتي. يتم تقديمه بالكامل تقريبًا في

    https://www.hayadan.org.il/astronomers-reach-new-frontiers-of-dark-matter-130112/#comment-324924

  684. يوفال، مئير
    لقد تحققت للتو من البريد الإلكتروني، وهناك بالفعل رسالة منك هناك! لذا سألقي نظرة عليه في عطلة نهاية الأسبوع. طاب مساؤك.

  685. اليوبيل.
    على الرغم من كلامك، أشعر أنني ربما بالغت في تقدير بيريز. أتمنى أن تعلم بالطبع أنني لا أقصد الإساءة - لقد كنت فقط أعبر عن الإحباط لأننا نتحرك ببطء شديد، حيث يقول الجميع بضع كلمات عن نموذجهم، ويتوقع من الجميع أن يفهموا بالضبط ما يعنيه من أجزاء الجمل .

    أنا جاد تمامًا بشأن أفكاري - على الرغم من أنني مقتنع تمامًا بأن لدي بعض الأخطاء الأساسية، لذلك أطلب دائمًا انتقادًا حادًا لها (وليس لي). أعتقد أنه إذا وحتى يتم إثبات وجود شيء ما فيها، فمن المؤكد أنها يمكن اعتبارها موهومة. وهذا أيضًا هو سبب إيقافي لرسائل البريد الإلكتروني - لأنني لم أتلق أي انتقاد منك، ولم تدعوني لأي انتقاد.

    لذلك دعونا نمضي قدما. إذا كنت مهتمًا بإرسال محتوى جنسي إلى عارضة الأزياء الخاصة بك - فاكتشف ما هي من الألف إلى الياء، حتى نتمكن من فهم ذلك. إذا لم يكن الأمر كذلك، فسوف أطلق سراحك كما طلبت، وسنواصل المزاح كالمعتاد.

    أنت وكل شخص آخر مرحب بك دائمًا لقص أم أي فكرة أتوصل إليها. ولكن على وجه الخصوص، المعرفة واليقظة.

    مئير.

    يبدو أنه موقع جميل، وصحيح أنه نفس الصيغة. لقد بحثت عنها دائمًا على الإنترنت دون جدوى. في الواقع، لقد قمت بقصها من أوراق عالم فيزياء يُدعى وودوارد. كنت قد تكون مهتمة في:

    http://physics.fullerton.edu/~jimw/general/inertia/index.htm

    كما وجهني إلى كتاب عظيم من تأليف دينيس شيما يربط بين موضوع القصور الذاتي والجاذبية في الصيغ.

    أود أن أسمع المزيد عن النموذج الخاص بك، وخاصة لرؤية الصيغ المرفقة.

    ليله سعيده للجميع.

  686. ليس بوحشية، يا إسرائيل، لا على الإطلاق. ولكن كيف أقول كفى ... ارحل. لماذا القتال؟ تكتب بشكل جميل. استمر
    النقطة التي تصر على عدم رؤيتها هي أنني أحاول شرح النموذج خطوة بخطوة هنا. إذا لم تفهم الخطوة الحالية، فلن تتمكن من الانتقال إلى الخطوة التالية. لقد قلت أنك قرأت فصلاً واحداً، وأنا أصدقك. ولكن لست متأكدا من أنك فهمت. أنت تكتب جملة عن أشيائي، ووصفتها بالوهمية، وتخبرنا عن المحادثات التي أجريتها مع كلبك. إن حقيقة قيامك بنشر كل شيء هنا علنًا بأسلوب شعبوي، بدلاً من الخوض بجدية والمراسلة عبر البريد الإلكتروني، على سبيل المثال، تجعلني أشك فيك باعتبارك شخصًا يحب كتاباته (وبحق، أنت أكتب جيدًا) ولكن ليس كشخص يحب القراءة. ربما أكون مخطئًا، لكن في الوقت الحالي لن أتحمل التحدي الذي ألقيته علي بكل لطف. لا أحتاج بشدة إلى تعزيزات منك أو من راكبي الأمواج الآخرين، فأنا لا أكتب مثلك، ولدي صبر. كل ما تبقى لي هو الاعتذار عن المتاعب التي سببتها لك وإضاعة وقتك الثمين. من فضلك اغفر وسامح وتوقف.

  687. إسرائيل،

    فيما يتعلق بـ GM=RC^2، أقوم أحيانًا بزيارة مدونة هذه السيدة
    http://riofriospacetime.blogspot.com/
    وبما أن R=tC (حيث t هو عمر الكون)، فإن الآنسة ريوفريو تدعي النظرية التي طورتها عندما كانت طالبة، والتي تتمحور حول نفس الصيغة التي أحضرتها بالضبط، والتي تشرح وفقًا لها الطاقة المظلمة.

    وفقًا لنموذجي، فإن كتلة الكون أقل بكثير من ربع الكتلة المقبولة اليوم، لذلك ليس لدي خيار سوى الشك في أهمية هذا الارتباط.

    لكنني أشكركم تمامًا على رعايتكم، وأشكركم أيضًا على هذه المناسبة الاحتفالية ليوفال.
    والشيء المثير للاهتمام هو أن الفيزيائيين وعشاق الفيزياء مشغولون جدًا بالبحث عن الطاقة المظلمة اليوم، لدرجة أنه لا أحد يلاحظ نوع الاكتشاف الذي كان من الممكن أن يسميه نيوتن "الفكرة السعيدة في حياتي".

    لذلك، إذا جاز لي استخدام كلماتك، فأنا فخور بيوفال وبك لأنه خلال المناقشة العاصفة هنا تمكنت من ملاحظة شيء قد يكون ذا أهمية غير عادية (لقد أضفت لك "ربما". باعتبارك شخصًا درس رهناً بالنحافة، فإن مستوى ثقتي بأهمية ذلك مطلق).

  688. اليوبيل.
    اترك كورتوف مورتوف. لقد ناقشت ما كتبته مع كلبي، فهو مستشاري للنماذج الوهمية. لقد توصلنا إلى أن الخفي أعظم من الظاهر.
    فلماذا لا تكتب كل شيء من البداية إلى النهاية كما تعظنا؟ وذلك حتى نتوقف عن التذمر طوال الوقت: "ولكن لماذا؟" أو "ماذا حدث؟" أو "يا رب الكون، ما الذي يتحدث عنه بالضبط؟"

    وللمرة الألف: أرجو توضيح مسألة اللا محلية! وليس عن طريق بديهية مثل: إذا تشابك جسيمان، فإن خصائص الجسيم A تنتقل إلى الجسيم B في زمن صفر وعلى أي مسافة.

    إذا كنت قاسيًا جدًا بالنسبة لأذن بريطانية يا أصدقائي الأعزاء، فأشيروا إلى ذلك وأحسنوا أسلوبي.

    مئير.

    أنا فخور جدًا بك. (لست فخوراً - فخوراً بك!).

    أنت تعطي وصفًا نوعيًا وكميًا. جميل.

    ما رأيك في البيانات التالية (يمكن العثور عليها على ويكيبيديا).

    ام:
    G = ثابت الجاذبية.
    ج = سرعة الضوء.
    M = الكتلة المقدرة للكون. (يمكن العثور عليها في ويكيبيديا).
    R = نصف القطر المقدر للكون. (نفس ما ورد أعلاه).

    إذن: GM=RC^2 تقريبًا.

    مذهل أليس كذلك؟ وما لا يقل إثارة للدهشة هو أننا إذا استخدمنا التحليل البعدي، بعد كل التخفيضات على جانبي المنارة، لم يبق لنا سوى:

    و = ماجستير

    قانون نيوتن الثاني، قانون القصور الذاتي.

  689. ر.ه.رفاعي.م
    أفكارك، بقدر ما هي جميلة ومثيرة للاهتمام، متقدمة جدًا بالنسبة لنموذجي. وهي مأخوذة من الفيزياء والرياضيات، في حين أن فكرتي لا تعرف هذه المجالات على الإطلاق لأنها بدائية تماما

  690. يوفال
    كانت فكرتك واضحة بالنسبة لي (وليس أنا فقط) حتى في المرة الأولى التي قدمتها فيها.
    بدلًا من الدوران والدوران، سأقدم لك هنا فكرة قد تغير شيئًا ما في هذه المناقشة العالقة:
    في الصورة الكبيرة: العدم الذي تقترحه يشبه المجال الذي يحدد غلاف الكون (ليس الكون نفسه - ولكن ضمن ماهية الكون). ما أدعيه (كإضافة إلى نموذجك) هو أن حقل الفراغ هو حقل ينشأ من الحقل (الذي يشكل على المستوى الرياضي مجموعة مغلقة) - ينشأ من حقل هو "لا شيء" الخاص بك. .
    أي أن المجال الذي اقترحته هو نوع من "مجال المعايرة" بين مجالك (لا شيء) ومجال الفراغ (وهو المجال الذي تبدأ فيه الفيزياء). وفي رأيي أن هذا المجال (أبعد من حقيقة وجوده) هو حقل من الطاقة المظلمة.

  691. إسرائيل شابيرا,
    إليك رسم تخطيطي للنموذج، ولكي أجعلك سعيدًا سأبدأ بأوصافك: عجلة دراجة نارية تدور في الماء. شاهد العجلة نفسها مصنوعة من الماء. ولتمييزه عن الماء الذي نقع فيه، لنفترض للحظة أنه متجمد، فهو مصنوع من الجليد. وبما أن الأمر كذلك، فإنه يتجمد الماء المحيط به. لكن التأثير متبادل وهناك أيضًا ماء يتم إذابته منه وإطلاقه في البيئة. فمن ناحية، يتحرك نسبة إلى الماء الذي يغطس فيه. لكن في المقابل، فإن الحركة النسبية ليست بين العجلة والماء على الإطلاق، وكل ما يتحرك هو حالة الركود. لذلك، في هذا التشبيه، يمكن اعتبار حركة الجسيم داخل المادة المظلمة ذات طبيعة موجية. حتى الآن هل الأمر واضح؟

  692. طالب،
    لقد سألتني إذا كنت "أدعي ببساطة أنني خلقت شيئًا من لا شيء". إنه واضح، وأنا سعيد لأنك توصلت إليه. لكن هذه ليست حجتي الرئيسية. وأشير إلى الآلية التي تسبب ذلك. لقد كتبت هذه الأشياء بالفعل هنا في "هيدان". يمكنكم رؤيتهم في الرابط التالي:
    https://www.hayadan.org.il/between-god-and-science-2807117/#comment-300326

  693. إسرائيل،

    يتكون التغير في إنتروبيا الكون من مجموع التغيرات في إنتروبيا العمليات في الكون. وبما أن معظم العمليات التي تحدث لا رجعة فيها (تسمى أحيانًا العمليات الطبيعية)، فإن الإنتروبيا في الكون تزداد. العملية التي وصفتها قابلة للعكس (إذا فهمت بشكل صحيح) وبالتالي لا تساهم في زيادة إنتروبيا الكون.
    أقترح عليك تحديد المثال مرة أخرى، لأنه يبدو لي أن هناك سوء فهم لما يدور حوله.

  694. إسرائيل،

    يتم الحصول على تسارع ثابت a عندما يكون التغير في سرعة الجسيم لكل وحدة زمنية ثابتًا.
    زمن دورة الجسيم ثابت.

    إذا أجبرت قوة خارجية الجسيم في كل دورة على التحرك مسافة ثابتة dX من مركز المجال الذي تم إنشاؤه في الدورة السابقة (والإجبار يعني إجبار الجسيم على التحرك من المكان الذي يوجه نفسه إليه تلقائيا دون تأثير القوة الخارجية نتيجة لكل ما حدث بالفعل في الدورات السابقة) فإن الجسيم سيتحرك بتسارع ثابت a، حجمه f * dx/dt حيث dt هو زمن الدورة و f هو عدد الدورات في الثانية

    التغير في delta G يتناسب مع الإزاحة، وبالتالي يتناسب مع قوة الانحراف (يظهر تطوير هذه الخطوة من قانون التربيع العكسي في الصفحة 5 من المقالة التي أتمنى أن تصلك عبر البريد الإلكتروني)

    ومن ثم فإن جسيمًا كتلته m سيتحرك بتسارع يتناسب مع القوة F المطبقة عليه.

  695. اليوبيل.
    أنت مليء بالتناقضات مثل رامون نفسه.

    تكتب أولاً "إن الحل الذي أقترحه للمشكلات التي أشرت إليها، والذي يوضح قانون ماخ بقوة، يتجسد في نموذج المادة المظلمة. هل سيكون من المفيد الخوض في الأمر؟"

    وعندما أحاول التعمق أكثر، وأطلب رؤية النموذج، تتدخل: "طالما أنها مجرد تكهنات في عينيك، فنحن نرحب بك لمواصلة الاستمتاع بوقتنا مع لآلئ لسانك."

    ونحن - ها نحن قادمون؟

    أطلب منك باستمرار أن تقوم بقص أفكاري، وتقديمها بوضوح، وعدم تلقي أي ردود. يبدو أنك تثير النقد، لكن تمتنع عن طرح الفكرة بشكل واضح.

    وأنا - آنا، هل أنا قادم؟

    مئير:

    ما أريد رؤيته هو كيف تحصل على F = ma

    طالب.

    ما أفهمه هو أن إنتروبيا الكون تتزايد بمرور الوقت. لذلك، إذا قسمنا هذا الوقت إلى وحدات صغيرة، فحتى في وقت قصير مثل مدة دوران اللوحة في الفضاء هناك زيادة معينة في إنتروبيا الكون، أليس كذلك؟

  696. يوفال، (https://www.hayadan.org.il/astronomers-reach-new-frontiers-of-dark-matter-130112/#comment-326768)

    - أنت ببساطة تدعي الخلق من العدم؟

    إسرائيل،

    "الإنتروبيا الكونية تتزايد طوال الوقت، أليس كذلك؟ لماذا لم ينمو أيضًا في العملية التي يمر فيها الوقت في نظام الألواح؟"
    – ليس من أجل عملية يمكن التراجع عنها، لقد أخبرتك بالفعل عدة مرات.

  697. إسرائيل،
    شرح مختصر يشير إلى الرسم التوضيحي الموجود في الرابط
    http://tinyurl.com/inertia-fig
    من أجل التوضيح، يشير الشكل إلى مقطع عبر قطر الجسيم بهندسة حلقة (كما نعلم، فإن "فتحة" الحلقة فارغة، وبالتالي فإن كتلة الجسيم منتشرة على المحيط) . يمكن تدوير الشكل في أي زاوية في الفضاء، وبالتالي فإن كل ما ينطبق على الحلقة ينطبق أيضًا على جسيم ذو هندسة غلاف كروي.
    يصف المنحنى الأخضر مجال الجاذبية الذاتية للجسيم. كما هو متوقع بالنسبة للجاذبية الكمية، يرتفع المجال في مركز الجسيم ويستقر على شكل نيوتوني عندما تبتعد عن المركز.
    تجد الجسيمات طريقها في مجال الجاذبية عن طريق دفعة متولدة في محيطها باتجاه المركز. كلما كان المجال أرق، كان التوجه أقوى نحو المركز.
    الجاذبية كما نعرفها هي عندما يتفاعل جسيم مع مجال جسيم آخر. في مثل هذه الحالة، يكون مركز مجال الجسيم الثاني خارج حدود الجسيم الأول، لذلك سيتم الحصول على "السقوط الحر" التقليدي لكل جسيم تجاه الآخر.
    وتختلف حالة الثبات الخاصة عن الحالة الأولى فقط في أن مركز المجال داخل الجسيم، فهو مجال ينتجه الجسيم نفسه.
    كما ترون في الجزء الأول من الرسم التوضيحي - الجسيم في حالة توازن مع المجال، وما هو جيد وما هو لطيف.
    في الجزء الثاني من الشكل، تؤدي قوة خارجية إلى انحراف الجسيم من مركز المجال إلى اليمين. لم يكن لدى الجسيم الوقت الكافي لإنشاء حقله في الموقع الجديد لأنه يخلق المجال في دورات، ولم تصل بعد دورة الجيل الجديد.
    ما وصل هو جزء الدورة الذي يتفاعل فيه الجسيم مع المجال.
    نظرًا لكسر تماثل الجسيم بالنسبة للمجال، ينشأ فرق في جهد الجاذبية مما يؤدي إلى سقوط الجسيم حرًا في الحالة المتماثلة.
    عندما تحاول القوة المتسارعة حرف الجسيم لمسافة أكبر من مركز المجال، فإنه يواجه فرق جهد أكبر، أي مقاومة أكبر لتغيير الحالة.
    الرسم يبالغ في التحويل. إذا قمنا بقياس الإزاحة بـ X، فإن الحالة المعتادة للسرعات الصغيرة تتجلى في الإزاحة الصغيرة dX.
    يمكن إثباته بسهولة تامة (باستخدام قانون التربيع العكسي لنيوتن) أن فرق جهد الجاذبية دلتا G يكون خطيًا إلى dx طالما أن التحول صغير (ولكنه يصبح أسيًا عندما يكون التحول كبيرًا). وبما أن قوة الجاذبية تتناسب مع g، فكلما حاولنا تسريع جسيم بتسارع أكبر (كنا نستمر في انحرافه عن مركز مجاله لمسافة أكبر في كل دورة) سنواجه مقاومة أكبر.

    أثبتنا هنا أن قوانين نيوتن الثلاثة للحركة مستمدة من قانون التربيع العكسي للجسيم الساكن.

    بالنسبة لجسيم يتحرك بسرعة ثابتة، كل شيء كما هو مذكور أعلاه، مع اختلاف أنه في كل دورة من دورات خلق الجاذبية، ينشأ المجال بواسطة جسيم متسارع إلى هذه السرعة، أي جسيم أجبرناه على توليد مجال جاذبيته في ظل ظروف فتح غير متماثلة، مما يديم عدم التماثل في المجال والسرعة.

  698. إسرائيل شابيرا !!!
    قبل لحظة من مغادرتك إلى مدينة القمار، تعاملت مع نموذجي بتحفظ على النحو التالي: "... إنها فكرتك، ويجب الإشارة إليها، حتى لا تربك القراء الذين قد يظنون أن هذا هو الرأي السائد". . بقدر ما أعرف، لا. أنا لا أقول إنها ليست فكرة مثيرة للاهتمام، ولكن حتى يتم إثباتها، يجب التعامل معها على أنها تكهنات".
    طالما أنها مجرد تكهنات في عينيك، فنحن نرحب بك لمواصلة الترفيه عن وقتنا بلآلئ لسانك.

  699. ليلة سعيدة يا إسرائيل
    سعيد أنه لا يزال لديك بنطال لمحررك.
    أتحدث هنا طوال الوقت فقط عن نموذج المادة المظلمة، بينما أنت تطحن التناقضات بين النماذج الفيزيائية التي تجاوزها الجليد.

  700. ينعم إسرائيل شابيرا بعودته قطعة واحدة.
    لقد سألت عن root i ووضحت لك الطريق. انت انتقائي في ردودك
    ليس هناك ما يمكن دحضه في المادة التي أرسلتها إليك. لديها فقط الإعدادات.
    الحل الذي أطرحه للمشكلات التي أشرت إليها، والذي يوضحه قانون ماخ بقوة، يتجسد في نموذج المادة المظلمة. هل سيكون من المفيد الخوض في الأمر؟

  701. R.H. Rafai.M،
    لم أكن أتحدث عن "مجموعة فارغة". هذا ما قلته انت. المجموعة الفارغة هي مصطلح رياضي يشير إلى كيان محايد (وليس "العدم"). "اللا شيء" الذي أتحدث عنه ليس محايدًا.
    ولم أتحدث عن الطاقة أو الفيزياء على الإطلاق. هذه هي "المواد" التي يتم إنشاؤها من العلاقة المتبادلة بين "اللا شيء" ونفسه.
    سيكون من العار أن تترك المناقشة، لأنك بأسئلتك وشكوكك تساعدني في تحسين الفكرة.

  702. لقد عدنا من فيجاس ببنطال جديد.

    طالب.
    لقد وصفت عملية عكسية (دورية) يكون فيها التغير في الإنتروبيا صفرًا. إن إنتروبيا الكون لا تزداد في مثل هذه العملية.

    إنتروبيا الكون تتزايد طوال الوقت، أليس كذلك؟ لماذا لم ينمو أيضًا في العملية التي يمر فيها الوقت في نظام الألواح؟

    مئير.
    ومن الواضح أن المقصود الجسيمات الأولية.
    قرأت ذات مرة مقالاً تناول فكرة مماثلة. ولكن كما في مثال KDA، الذي لا توجد جاذبية في مركزه على الإطلاق، كيف سيخلق القصور الذاتي؟
    إذا كان بإمكانك إرسال رابط أو تفاصيل فكرتك هنا، وخاصة التطور الجبري لقوانين نيوتن، سنكون سعداء.

    اليوبيل الافترائي.

    نظرياتك لا تصمد أمام اختبار الدحض.

    "تظهر المحلية (وغيابها) في الفصل السادس من نموذجي (الفصل الرابع في النموذج القديم الذي لم تكلف نفسك عناء قراءة الفصل الأول منه)." الشخص الذي خيب الآمال هنا هو أنت."

    قرأت قرأت أيضا. لقد ذكرت أنك أرسلت حتى أتمكن من تعلم كيفية بناء النماذج. لم تجب على أسئلتي المتكررة إذا كنت مهتمًا بالمراجعة. كيف يمكن دحضه؟
    من فضلك الفصل 6، غير محلية.

    "أنت لست جيدًا في الرياضيات وهناك احتمال أن تخسر كل ثروتك هناك في نيفادا. كن حذرا ورعاية!

    أنا معك تماما في هذا. المشكلة هي الكازينوهات البخيلة، التي منعتني لسنوات من الاقتراب من طاولات البلاك جاك (اللعبة الوحيدة في الكازينو حيث يمكنك التغلب على الكازينو). المصاصون
    وهنا أيضاً لا يمكن دحضه.

    "إذا كانت القسمة على صفر لا تمثل مشكلة، فأرجو أن تريني الاستخدامات الناجحة لها."

    أليس هذا إلى حد كبير ما نفعله عندما نشتق الدوال؟

    ولماذا لا يجيب أحد على سؤالي حول البندول الباليستي؟

    https://www.hayadan.org.il/astronomers-reach-new-frontiers-of-dark-matter-130112/#comment-326491

  703. يوفال
    سأكتب باختصار:
    وتزعمون أن العين لم تخلق من العدم لأنها لا شيء. وهذا مشابه لتعريف لا شيء كمصفوفة فارغة.
    هذه المجموعة الفارغة لن تكون صحيحة إلا من حيث النموذج الرياضي. ومن الناحية الفيزيائية - هذه المجموعة الفارغة لا يمكن أن توجد، لأن الحد الأدنى من حالة الطاقة التي يمكن أن يوجد فيها نظام فيزيائي - لنقل أنها طاقة فراغية - أكبر من الصفر. إنها لا تساوي حتى الصفر كما تحاول أن تدعي.
    وآمل أن يكون الأمر واضحا بما فيه الكفاية على الأقل، لأنه إذا لم يكن الأمر كذلك فلن نتمكن حتى من الاتفاق على تعريفات للأشياء، وفي مثل هذه الحالة ليس هناك أي معنى لمواصلة المناقشة.

  704. في الكتاب المقدس، الصفر يعني "لا شيء" واسم الله. ر. عاموس الفصل 6 الآية 10: ...وقال ليؤكد في الجزء الخلفي من البيت الباقي معك وقال صفر وقال هس لأنه لا يذكر بسم الله.

  705. اليوبيل,
    الطرح بدل الجمع، العين بدل نعم. هل ذهبت إلى الجانب المظلم أم ماذا؟ احرص على ألا تصبح دارث فيدر.
    وبالمناسبة لست متأكداً من أن إبسو تشير إلى المياه العذبة وخالق العالم السومري هو أصل كلمة إبس.
    Eps هو ببساطة مرادف لـ Ain في الكتاب المقدس

  706. ر.ه.رفاعي.م
    هذا ليس ادعائي. أنا لم أقل "لا يوجد". لا يوجد مكان للحرف "V" هناك.
    كل ما قلته هو أن بداية كل شيء هي "لا".
    لنفترض أن بداية كل شيء لم تكن "لا شيء" بل كانت "هناك". والسؤال الطبيعي الذي يجب طرحه هو "من أين أتى؟". إذا قلت أن هذا "هو" تم إنشاؤه من "هو" آخر، فسيظل هذا السؤال دون حل، لأننا ببساطة قمنا بحرف الإجابة. وهو مثل القول إن العالم خلقه خالق، لكننا لا نعرف من خلق هذا الخالق. ومن ناحية أخرى، لا تسأل من أين جاء "اللا شيء"، لأنه "لا شيء" وبالتالي لم يأت من أي مكان. ومن هنا أجد أن نقطة البداية الأكثر منطقية هي "لا". لا يوجد شيء سوى "لا شيء"، وبالتالي فهو "لا شيء" مطلق.
    والآن يطرح السؤال كيف يخلق "لا شيء" "نعم". حسنًا، بما أنه "لا شيء" مطلق، فهو يدمر كل شيء، وقبل كل شيء نفسه. ومما نعرفه في عالمنا أن نفي النفي هو أمر إيجابي وأنا استقراء من هذا إلى "العدم" الأولي. "لا شيء" يصنف نفسه إلى "نعم". من فضلك حاول التفكير في الأمر للحظة. انها ليست معقدة.

  707. بالإضافة إلى ذلك، حاول تطبيق نموذجك على العالم الحقيقي (المادي) وليس العالم الرياضي فقط.
    وإلا فإن النموذج الخاص بك لن يحمل الماء.

  708. يوفال

    فدعواك (لا يوجد ولا يوجد) يصلح لك أن تدعي أنه في البداية كانت هناك "مجموعة فارغة".
    ومع ذلك، أكرر مرة أخرى (وليس أنا فقط)، أنك في هذا بالفعل تطبق تعريفًا لشيء لا يمكن تعريفه في رأيك.

    اسمع، إذا كنا نتحدث بالعقيدة وليس باللغة العلمية، وإذا تمكنت من حل هذه المفارقة (تعريف شيء ما على أنه شيء "غير محدد") فسوف تفوز بعدد لا بأس به من الجوائز (هدايا النية. وليس الخميني) 🙂

  709. غادي الكسندروفيتش "،"
    في الواقع، في الأعداد الطبيعية، وليس فقط فيها، يتم تحديد نسبة الطلب. أسمائهم "طبيعية" و"كاملة" و"عقلانية" و"حقيقية" مستمدة أيضًا من الواقع المادي الذي تعكسه. إذا كان كل ما يهم في الرياضيات هو مدى ارتباطها بالواقع اليومي، فلن أجادلك.

    طالب،
    في معجمي البدائي، كلمة "اتصال" غير موجودة. وفي لغة أكثر تقدمًا، تعلمنا الاختصار ونقول "الإضافة" بدلاً من "الطرح الطرح". لكني مازلت في المرحلة الجنينية فقط.

  710. اليوبيل,
    أعترف أنني دخلت في منتصف المناقشة ولم ألاحظ أنك أشرت إلى نموذجك.
    على أية حال، لا يبدو لي أنه يجب أن يكون لديك أي سبب محدد لرغبتك في دمج كلمات الراحل ماخ في نموذجك. الرأي السائد هو أنه حتى اليوم لا يوجد دليل أو دحض لمبدأ ماخ، بحيث لا ترفع كلماته ولا تخفضها.

  711. اليوبيل,

    أنت تقول مرة أخرى أن الطرح هو الجمع. لقد قلت بالفعل أنه إذا اخترت تعريف أن الطرح هو إضافة - فأنا أفهم ذلك. في هذه الحالة، قمت بحفظ تعريف المقاوم ولكن أضفت هذا التعريف.

  712. ولا علاقة لها بنظرية الأعداد.

    إن تحديد نسبة الترتيب للأعداد الطبيعية ليس في الحقيقة أمرًا اعتباطيًا، وربما تكون نسبة الترتيب هذه (وتعميماتها على الأوامر) واحدة من أهم الأشياء في الرياضيات.

  713. طالب،
    نظرية الأعداد هي مجال أكثر تعقيدًا بكثير مما نقدمه هنا. تعريف نسبة الطلب تعسفي. ولأسباب التيسير، قررنا أنه من الممكن ترتيب الأرقام على نوع ما من المسطرة. لكن إذا فحصت المركبات مثلاً، سترى أن هذا القرار لا يصمد. كل رقم فريد من نوعه، ولكل منها ميزات مثيرة للاهتمام. أحد الأرقام، الذي نسميه "صفر"، له خاصية تجعله مفيدًا جدًا في حياتنا اليومية. تمامًا كما يمكنك القول أن "الرقم المقابل" للرقم a هو الذي جمعه مع a يعطي صفرًا، يمكنك القول أن طرح a من الرقم المحايد يعطي رقمًا جمعه (أي طرح طرحه) إلى الرقم a يعطي الصفر. يمكن للمرء أن يجادل، وهو محق في ذلك، بأن هذا مجرد دلالات. لكن هذا لا يغير شيئًا بالنسبة لحجتي لأن كل ما نفعله بعملية الجمع يمكن أن يتم بالطرح فقط.

  714. مئير،
    قلت "المادة المظلمة" وليس "الكتلة المظلمة". عندما لا تكون طبيعة المادة المظلمة واضحة لمعظمنا، فقد يبدو الأمر وكأنه قواعد نحوية سيئة. لكن المادة المظلمة التي كنت أتحدث عنها هي المكون الأساسي لكل المادة في الكون. كل الكتلة الموجودة في الكون عبارة عن مادة مظلمة، والتي تتبلور جزئيًا أيضًا إلى مادة "غير مظلمة". إن الحركة في الفضاء لجسم مكون من مادة "غير مظلمة" ما هي إلا حركة متموجة على سطح المادة المظلمة.
    ليس لدي أي شيء سيء في قلبي بشأن الراحل ماخ. لقد قمت للتو بتصحيح صياغته لتناسب نموذج المادة المظلمة الخاص بي. وكما أشارت إسرائيل، فإن هذا النموذج ليس في المجال العام (في الوقت الراهن).
    طلبت منا إسرائيل "عدم التهور" بينما يفقد سرواله في مدينة القمار. ولهذا لا أشير إلى كلامك الذي يستهدفه. لكني أحب أن أقرأ مقالتك.
    شكرا لكم مقدما

  715. اليوبيل,

    "وماذا في ذلك؟ وما علاقة هذا بادعائي بأن كل عملية جمع يمكن إجراؤها بالطرح؟
    - لا يوجد أي ارتباط بين مناقشة ادعائك بأن القسمة على صفر مشكلة ومناقشة الجمع والطرح.

    "وأنا لا أفهم لماذا جلبت المصفوفات هنا."
    – لأن ضرب المصفوفات غير ذات الترتيب الصحيح غير محدد. أردت أن أعرف ما إذا كانت ستكون هناك مشكلة حتى في الموقف الذي يحاولون فيه القيام بذلك على أي حال.

    "وليس من الواضح بالنسبة لي ما الذي تحاول إثباته من خلال السؤال عن مقدار قيمة Ai الأقل من Ai."
    - لا شيء. أحاول أن أفعل ما وصفته - أن أبني الحساب على الطرح بدلاً من الجمع، دون تعريف مصطلح مضاد (زعمت أنه من الممكن الحفظ) ودون تعريف أن الطرح هو إضافة. يمكنك أن تفترض ذلك، إذا كنت تعتقد أنني لم أفهمك في مرحلة ما، وينبغي أن يكون الأمر بسيطًا للغاية.

  716. R.H. Rafai.M،
    على الرغم من (أو بسبب) موقفك السلبي، فأنت حاليًا أقرب من جميع المعلقين الآخرين لفهم حجتي. أنت على حق تمامًا في أنني أبني بداية وجود الكون على "كذبة". في الواقع، أنا لا أتحدث عن الكذب فقط، بل عن "النفي" بشكل عام. يمكن أن يشمل هذا التعريف أي شيء له معنى "سلبي". على سبيل المثال، "الكذب" هو نفي "الحقيقة"، و"لا" هو نفي "نعم". "الفراغ" هو نفي للوجود، وبهذا المعنى فإن "الصفر" هو أيضًا نفي لأنه، من بين أمور أخرى، يشير إلى "الفراغ".
    سأضيف فقط، من أجل الدقة، أنني لم أقل "لا يوجد، وليس هناك خلق، هناك". يتم استخدام ربط الاتصال في العبارة التي أحضرتها كإجراء "وأكثر"، ولكن في نقطة التكوين التي أتحدث عنها، هذا الإجراء غير موجود. ما أقوله هو أن بداية كل شيء هو النفي، والنفي ينفي كل شيء لأن هذا هو جوهره. فهو ينفي نفسه، ونتاج نفي النفي نسميه "الإيجابية" أو "الموجود" أو "الحقيقة" أو "الموجود" أو "المستقر" أو "الممكن" أو أي شيء يمكن أن يوجد فيه طابع "إيجابي". .

  717. يوفال
    كل هذا صحيح، ولكن فقط في إطار الرياضيات. إذا حاولت تطبيق A في الفضاء المادي، فسيتعين عليك تعريف شيء مادي على أنه A. في الرياضيات، يمكنك اختراع شيء افتراضي، ولكن في الفيزياء - حتى الجسيمات الأولية التي لم يتم ملاحظتها مطلقًا، تم تعريفها بحيث يمكن إجراء التجارب والأبحاث. يمكن إجراء البحوث. وقد تم اشتقاق تعريفاتهم من المعادلات الرياضية. لكن تلك المعادلات الرياضية كانت مبنية على ظواهر فيزيائية محددة مسبقًا.
    عندما تقول "لا يوجد والعدم هو خلق نعم" - يجب عليك تحديد ما هو "لا" وما هو "هو".
    بمجرد أن تقول: في نموذجي وقت تكوين الوجود لا يوجد شيء آخر - فإنك تسقط نموذجك لأنه مبني على كذبة وليس على الحقيقة. وهذا شيء باطل، لأن هناك شيئًا ما في وقت تكوين "الكائن" وهو "اللا شيء". لكنك تدعي أن هذا غير محدد لأنه لا يوجد شيء موجود في تلك اللحظة (وفقًا لنموذجك).
    إنه مثل القول بأن 0 موجود، لكن 0 غير موجود بالفعل. هذا حشو لا يفسر شيئا.

  718. اليوبيل,
    ردا على
    https://www.hayadan.org.il/astronomers-reach-new-frontiers-of-dark-matter-130112/#comment-326447
    و. عندما تلاعب ماخ بدلو نيوتن وصاغ أفكاره، كانت المادة المظلمة لا تزال مجهولة.
    ب. الكتلة هي كتلة بغض النظر عما إذا كانت مظلمة أو غير مظلمة، والكتلة المظلمة لها نفس مجال الجاذبية مثل الكتلة غير المظلمة (في الواقع، كل ما نعرفه عن الكتلة المظلمة هو مجال جاذبيتها). ولذلك، فإن وجود (أو غياب) الكتلة المظلمة ليس شيئًا يمكن أن يغير مبدأ مبدأ ماخ.

    إسرائيل
    ردا على
    https://www.hayadan.org.il/astronomers-reach-new-frontiers-of-dark-matter-130112/#comment-326486

    لقد فهمت مطالبتي بشكل صحيح، ولكن عليك أن تكون حذرا في صياغتها. أنا أتحدث عن الجسيمات الأولية، وليس الأجسام. أي أنني لا أزعم أنه «من الصعب تحريك الأرض من مكانها» لأنها تميل إلى الوقوع في مجال جاذبيتها الخاصة. مثل هذا الادعاء غير صحيح لأنه وفقًا لنظرية الصدفة، فإن إجمالي قوى الجاذبية المؤثرة على نقطة كتلة داخل كرة مصنوعة من مادة موحدة تصبح أصغر كلما اقتربت من المركز (أي لا يمكن ادعاء ذلك لأن المسافة للأرض من مركزها تميل إلى الصفر فإن الجاذبية الذاتية المؤثرة عليها تميل إلى اللانهاية)

    ما أدعيه هو أنه بما أن كتلة أي جسم (بما في ذلك الأرض) هي مجموع كتل الجسيمات الأولية التي يتكون منها، وبما أن مسافة الغلاف الضخم لكل جسيم أولي من مركزه كبيرة جدًا بالقرب من الصفر، فإن تأثير قوة الجاذبية الذاتية للجسيم على الجسيم نفسه يجب أن يكون ملحوظًا، وبالفعل نعرفه جيدًا باسم "ظاهرة الثبات". وبما أن الجسيم الأولي ليس كتلة محددة (ولكنه كتلة في هندسة القشرة فقط)، فإن نظرية القشرة لا تنطبق عليه. باختصار، ثبات جسم مثل الأرض هو مجموع ثبات الجسيمات الأولية التي يتكون منها.

    بالمناسبة، في المقال الذي نشرته أثبت (جبريًا) أن قوانين نيوتن الثلاثة للحركة مستمدة بشكل طبيعي من هذا النموذج، مما يعني أن جميع قوانين نيوتن في الواقع مستمدة من قانون التربيع العكسي لنيوتن. ليس هذا فحسب، فمن الطبيعي أن قوانين الحركة في صياغتها النيوتونية من الجبر البسيط لهذا النموذج تكون صحيحة فقط للسرعات الصغيرة، بينما بالنسبة للسرعات العالية ("النسبية" باللغة العامية) يتم الحصول على زيادة أسية في الزخم. أي أن هناك أيضًا تفسيرًا نيوتونيًا لحقيقة عدم إمكانية تجاوز سرعة الضوء.

  719. ر.ه.رفاعي م.، مع الاعتذار عن الاختصار،
    هناك رقم نسميه Minus A، ولكن يمكننا بسهولة أن نسميه Moisha أو أي اسم نحبه. وسبب تسميته بهذا الاسم هو الارتباط المثير بينه وبين رقم آخر نسميه Ai (بدون السالب). العملية الحسابية التي تتم للانتقال من A إلى ناقص A هي طرح A من الرقم المحايد (والذي يسمى في اللغة العبرية نسبة إلى الإله السومري أفسو). ولا داعي لتعريفه بتعريف خاص، فهو رقم مثل سائر الأرقام.

  720. طالب،
    أنا حقا لا أفهم ما تحاول التعبير عنه. القسمة على الصفر غير محددة في الجبر، لأنها ليست ذات قيمة مفردة. وماذا في ذلك؟ ما علاقة هذا بادعائي بأن كل عملية جمع يمكن إجراؤها عن طريق الطرح؟ وأنا لا أفهم لماذا جلبت المصفوفات هنا. هل يدفعك نظام المعرفة للتعليق؟ أنا لا. من المحتمل أنك تحاول الآن البحث عن كيفية بدء النقاش حول القسمة على صفر وتجد أنني قلت ذلك بطريقة ما. صحيح جدا قلت. ولكن ليس كجزء مركزي أو أساسي من القصة البسيطة للغاية.
    وليس من الواضح بالنسبة لي ما الذي تحاول إثباته من خلال السؤال عن مقدار قيمة Ai الأقل من Ai. بعد كل شيء، يتعلمون هذا في المدرسة الابتدائية. هل تريد مني أن أعرف أنها اثنين من الدرجة الأولى؟

  721. لاجدي الكسندروفيتش
    من الواضح لي أن فعل الانضمام هو اتحاد مجموعتين، لكن بينما في الانضمام نقوم بذلك مع مجموعتين أجنبيتين، على سبيل المثال 5+3 يساوي 8، ففي توحيد المجموعات لا يوجد مثل هذا الالتزام بالنسبة للمجموعات الأجنبية.
    ولكن هذا شيء من ذكرياتي المدرسية الماضية وأعتذر مقدمًا إذا كنت مخطئًا.
    يوم جيد
    سابدارمش يهودا

  722. يهودا - يوجد نموذج للأعداد الطبيعية حيث 0 هي المجموعة الفارغة ويتم تعريف S (عملية التابع) على المجموعة A كما يلي

    S (أ) = أ \ كوب {أ}.

    وهذا يعني اتحاد A مع المجموعة التي يكون عضوها الوحيد هو A. وبهذه الطريقة، نحصل على عملية الاتصال من خلال اتحاد المجموعات.

  723. اعذروني على جهلي أو ارتباكي، لكن يبدو لي أن عملية التجميع يتم تعريفها بشكل مختلف، فهناك شيء أكثر أساسية من عملية التجميع، وهو توحيد المجموعات حسب نظرية المجموعة كانتور. على سبيل المثال المجموعة التي تحتوي على الأعضاء
    A، B، C، F، T، الاتحاد مع المجموعة التي تحتوي على الأعضاء A، E، L، M، هي المجموعة التي تحتوي على الأرقام A، B، C، E، F، T، L، M.
    حالة واحد وواحد هي حالة خاصة لربط عضوين متطابقين
    مساء الخير
    سابدارمش يهودا

  724. يوفال
    في الوقت الحالي يميل النقاش نحو الاتجاه الرياضي، لكن في البداية كان النقاش يميل نحو نظرية الكم وخاصة ميكانيكاها.
    في الرياضيات يمكنك اختراع أشياء افتراضية مثل المعادلات التي لا تتوافق مع الظواهر الفيزيائية الموجودة.
    عندما تحدد (أ-) فيجب أن يكون تعريفها واضحا ومقبولا.
    إن تطبيق التعريف على أي عضو يجب أن يعتمد على شيء مادي - إذا كنت تريد أن تكون النتائج مقبولة في العالم المادي (يمكنك أن تبنيها على ملاحظات الظواهر الفيزيائية وقياسها لتحديد قيمتها في العالم المادي).
    ربما يكون ما كتبته صحيحًا (أنا أتحدث عن المعادلة).
    لكن ما الذي تحدده بـ (أ-)؟ لماذا تطرح شيئًا من شيء لم تحدده في المقام الأول؟
    على سبيل المثال: كنت أتحدث عن الحقول في الفضاء الكمومي. لقد أخذت "نقطة الصفر" وافترضت أن أدنى نقطة لا تساوي الصفر (يمكن أن تكون أكبر أو أقل من الصفر - في رأيي أقل من الصفر).
    عندما قمت بهذه الفرضية، كنت أبنيها على نتيجة تأتي من نتائج تطبيق تعريف المجال (في الفضاء الكمي) كمجال محدد (مجال يعرف بالفراغ - الذي فيه كل ما هو فيزيائي معروف) تحدث العمليات، بما في ذلك ميكانيكا الجسيمات). وآمل أن أكون واضحا بما فيه الكفاية حتى الآن حتى نتمكن من مواصلة مناقشة موضوعية بيننا على الأقل.

    الأمر المؤكد هو أن الطالب وغادي ألكسندروفيتش يفهمان ويعرفان ما يتحدثان عنه. حتى لو كانوا لا يعرفون الكثير، أنصحك بفهم ما يقولونه (حتى لو كان الأمر يتعلق بجبر بسيط). على الأقل حتى الآن، ومع كل احترامي لهم وتقديري الكبير لمشاركتهم في المناقشة، فإنهم يشكلون بديلاً جديراً لمايكل روتشيلد. 🙂

  725. يعتمد تعريف المقال القياسي على حقيقة أنه إذا كنت مدينًا لعائلة إجرامية، على سبيل المثال، بـ 1000 شيكل و1000 شيكل أخرى، ولا تعرف ما يريدون منك، فسوف تموت.

  726. اليوبيل,

    القسمة على صفر ليست مشكلة لأنك لا تقسم على صفر. لا تقم بإجراء عملية غير محددة. هل تعتقد أن ضرب المصفوفات التي ليست بالترتيب الصحيح يمثل مشكلة أيضًا؟

    لقد قلت أنه يمكن تعريف عملية الجمع باستخدام عمليات الطرح والحسابات الأساسية بهذه الطريقة، لكنك لم تقم بالحساب. سأكرر السؤال الذي طرحته: ما قيمة التعبير أ-(-أ)؟

  727. غادي ألكسندروفيتش،
    لقد طلبت مني تحديد شيء ما، واعتقدت أنك كنت تستغل أشيائي. أعتقد أنك، كطالب في شانيون، لا تفهم أيضًا ما أتحدث عنه. ربما لأنك اعتقدت أنها كانت فلسفة معقدة. كل ما قصدته هو أنه باستخدام عمليات الطرح فقط يمكنك الوصول إلى كل ما يتم الوصول إليه عن طريق عمليات الجمع. ولم أقصد أي شيء أعمق من ذلك. أرجوك سامحني
    ونعم، أحب أن أقرأ مشاركاتك. شكرا.

  728. يوفال، السبب في أنه يكفي تعريف اتصال للمواد الطبيعية هو أنه بمجرد تعريف اتصال للمواد الطبيعية، تأتي بقية التعريفات بشكل طبيعي بناءً على هذه العملية (لدي سلسلة من المشاركات حول بناء أنظمة الأعداد إذا أنت مهتم).

    لا أفهم التعريف الذي قدمته للطرح. هل يمكنك أن تشرح لي ما هو أساس x-1؟

    في تعريف الاتصال الذي قدمته، يتم الحصول على x+1 على النحو التالي:
    x+1=x+S(0)=S(x+0)=S(x) – في الجناح الأيمن حصلنا على تعبير لا يتضمن +.

  729. طالب،
    إذا كانت القسمة على صفر لا تمثل مشكلة، فأرجو أن تريني الاستخدامات الناجحة لها.
    لقد قلت أنه يمكن تعريف الجمع باستخدام الطرح، وقد فهمت بشكل صحيح أنه من الممكن أيضًا إجراء العمليات الحسابية على الطرح بدلاً من الجمع.

    غادي ألكسندروفيتش،
    أنت تذهب فقط لتلك الطبيعية. هل هو عمداً للغضب؟ أنا أحب المركبات، ولكن لكي أسير نحوك سأعمل مع الكل. يعتمد تعريف الطرح (حسب رأيك) على دالة التابع S، ويتم تعريفه بشكل متكرر:
    x–0=x لكل عدد صحيح x، وهكذا
    (x–(0–S(y))=S(x–(0–y) لجميع الأعداد الصحيحة x,y.
    وأرجو عدم إرسالي إلى الروابط لأنني لا أستطيع فتحها كلها.

  730. يوفال، أنت تحاول تقديم تعريف غير قياسي؛ عادةً ما يكون من المعتاد في هذه المرحلة أيضًا إعطاء التعريف وعدم تركه كـ "تمرين".

    يعتمد تعريف الاتصال القياسي على وظيفة التابع، S، ويتم تعريفه بشكل متكرر:
    x+0=x لكل شيء طبيعي x و
    x+S(y)=S(x+y) لكل شيء طبيعي x,y.

    انظر على سبيل المثال هنا:

    http://he.wikipedia.org/wiki/%D7%9E%D7%A2%D7%A8%D7%9B%D7%AA_%D7%A4%D7%90%D7%A0%D7%95

  731. يوفال، أنت لم تجب علي. أسأل كيف تحدد الطرح، وبدلاً من الإجابة، تقول فقط إنه مشابه لشيء آخر (لا أعتقد أنه مشابه على الإطلاق، بالنسبة للتعريف الذي أعرفه عن الجمع). أنت تستخدم أيضًا مصطلحات لا أعرفها في هذا السياق على الإطلاق ("العلاقة").

    أطلب منك مرة أخرى، إن أمكن - تحديد "الطرح" بالنسبة لي. وهذا يعني إعطاء تعريف واضح. افترض أنني لا أعرف شيئًا تقريبًا عن الرياضيات.

  732. اليوبيل,

    إذا كان الأمر كذلك، فما هي قيمة التعبير أ-(-أ)؟

    قلت إن الجمع يمكن تعريفه باستخدام الطرح، وبقدر ما أفهم من الممكن أيضًا إجراء العمليات الحسابية على الطرح بدلاً من الجمع.

  733. غادي ألكسندروفيتش،
    تعريف الطرح يشبه إلى حد كبير تعريف الجمع. ونشترط أن يكون الجمع ملزما ومحققا لخصائص معينة، وهذا أيضا ما نطلبه من الطرح. نرحب بك، كتمرين، لتحديد كل ما تعرفه عن الاتصال دون استخدام علامة "+" ولكن فقط علامة "-". إنه أمر مرهق بعض الشيء، لكنه ممكن.

  734. اليوبيل,

    كيف تستنتج أن التعبير a-(-a) يساوي a+a بدون تعريف أن الطرح هو إضافة؟

    "أنت تقول "غير محدد" وأنا أقول "لديهم مشكلة""؛ هل نتجادل حول الدلالات مرة أخرى؟"
    – لا أعتقد أنها دلالات. إذا لم يتم تعريف شيء ما، فإنه ليس له معنى.

  735. طالب،
    أنا حقا لم أضيف تعريفا. لقد ذكرت للتو أنه، لأسباب تتعلق بالتوفير والراحة، فإننا نسمي عملية الطرح-الطرح عملية جمع.
    ولوغاريتم الصفر غير محدد في الجبر لنفس السبب الذي يجعل القسمة على الصفر غير محددة هناك. أنت تقول "غير محدد" وأنا أقول "لديهم مشكلة"؛ هل نتجادل حول الدلالات مرة أخرى؟

  736. اليوبيل,

    لا أعرف كيف ستبدو حياتي من الآن فصاعدا، مع العلم أنني لم أرق إلى مستوى توقعاتك.

    في واقع الأمر، ما زلت غير متأكد من أنني أفهم.

    أ-(0-ب) = أ-((سم مكعب)-ب) = أ+ب ؟

    أين يتم الانتقال من الطرح إلى الجمع؟ إذا فهمت بشكل صحيح، فقد "حفظت" تعريف المصطلح المضاد بالإضافة، ولكنك أضفت تعريفا بأن الطرح هو الجمع.

    "يواجه علماء الرياضيات مشكلة مع لوغاريتم الصفر الذي يعادل مشكلة القسمة على صفر."
    - ليس لديهم مشكلة، إنها فقط غير محددة. أنتم مدعوون لقراءة ما كتبه الدكتور ألكسندروفيتش.

  737. طالب،
    كنت أتوقع منك أن تفهم الأمور دون صعوبة. من العنوان الذي تحمله لنفسك هنا أعتقد أنك قمت بعلم الجبر في السنة الأولى.
    لقد سألت عن كيفية إجراء عملية الجمع فقط مع الطرح، حسنًا: (a-(0-b) يعطي ما يناسبنا أن نكتبه على شكل a+b. لاحظ أنني لا أحدد وجود مصطلح معاكس ولكني أستخدم الخاصية فقط الصفر كرقم محايد في عملية الطرح. وبذلك أضفت الكثير من الأقواس وفي المجمل استخدمت المزيد من العلامات، ولكنني احتفظت بتعريف واحد. لئلا تقول إنني من خلال القيام بذلك قمت بإدراج تعريف الرقم المقابل في الباب الخلفي، يمكنني إضافة عضو عشوائي، مثل c والكتابة (cc) بدلاً من 0. للإيجاز، من الملائم بالنسبة لنا أن نفتح الأقواس ونستبدل العلامات بعلامة سالب زائد، لكن راحتنا الشخصية ليست عاملاً يجب أخذه في الاعتبار.
    سأل ر.ه. الرفاعي م لماذا عبارة "ناقص ضرب ناقص يعطي علامة زائد". في كلامي "إنكار النفي يعطي الإيجابية". كما ذكرنا سابقًا، فإن الكلمات "زائد" أو "مدين" ليست ضرورية.
    نحن نعرّف القسمة على أنها ضرب، وهذا مرة أخرى للإيجاز. وبنفس الطريقة يمكننا تعريف الضرب على أنه قسمة. كان بإمكاننا، بشكل عام، استخدام نظام تعريف مختلف: فبدلاً من تعريفات الجمع والضرب، كان بإمكاننا استخدام الطرح واللوغاريتمات. لدى علماء الرياضيات مشكلة في لوغاريتم الصفر الذي يعادل مشكلة القسمة على صفر.

  738. إسرائيل شابيرا!
    أنت لست جيدًا في الرياضيات وهناك احتمال أن تخسر ثروتك بالكامل هناك في نيفادا. كن حذرا ورعاية!
    اخرج واحسب (1+i) أس 2. هل حصلت على اثنين؟ إذا كان الأمر كذلك، فأنت على الطريق الصحيح لحساب جذر i.
    أثار اهتمامي موشيه كلاين بعالم رياضيات إنجليزي يُدعى سبنسر براون، والذي وجد أنه مثلما يوجد رقم "خيالي" يمثل جذر سالب 1، هناك أيضًا قيمة حقيقة "خيالية" والتي عند تطبيقها على نفسها تخلق حالة نفي.
    تظهر المحلية (وعدمها) في الفصل السادس من نموذجي (الفصل الرابع في النموذج القديم الذي لم تكلف نفسك عناء قراءة الفصل الأول منه). الشخص الذي خيب الآمال هنا هو أنت.

  739. إسرائيل،

    "من الواضح، ولكن وفقًا لوصفي، قد يكون هناك عامل فيزيائي آخر بسبب أن التغير في الإنتروبيا ليس صفرًا. إنه توسع الكون. لذلك، في نظام الصفائح الدوارة، أثناء دورانه، تصبح كثافة الكون أصغر، وبالتالي تزداد الإنتروبيا."
    - لقد وصفت عملية عكسية (دورية) يكون فيها التغير في الإنتروبيا صفرًا. إنتروبيا الكون لا تزداد في مثل هذه العملية.

  740. ر.ح.

    أتمنى أن نكون قد انتهينا من هذا، بموافقة فاينمان، أنه في نموذج ليساج ستكون هناك جاذبية، ولكن ستكون هناك مشكلة احتكاك بين الكواكب والجزيئات.

    وخلصنا أيضًا، بتشجيع ماكسويل الحماسي، إلى أن نموذج الأثير يعد نموذجًا رائعًا إذا تمكن من استخلاص سرعة الضوء من ثوابت الكهرباء والمغناطيسية. المشكلة هي: سرعة الضوء بالنسبة إلى ماذا؟

    ورأينا أيضاً بحسب نصيحة ماخ أن كل كتلة في الكون تؤثر على كل كتلة أخرى، كما في حالة النجوم البعيدة. التأثير هو الجمود. السؤال هو: كيف؟

    نحن هنا عالقون. يوجد، أو على الأقل لا أعرفه، أي حل سائد يعطي إجابة مناسبة لهذه الأسئلة. إذا كان أي شخص يعرف مثل هذه الإجابة، يرجى الرد الآن.

    نحن في مرحلة حرجة من المناقشة، على الأقل بالنسبة لي. أنا على وشك تقديم فكرة يمكن أن تعطي التوجيه. أريد أن أصف تجربة فكرية، واستنتاجي حولها، والانتقادات منك ومن الجميع.

    يجب أن يكون الطالب على دراية بالسؤال التالي من دراسات السنة الأولى (مثال 3 في RESNICH HALLIDAY، PHYSICS، Chapter 10، COLLISIONS)

    البندول الباليستي.

    خذ جذعًا متصلًا بالسقف بالأسلاك وأطلق كرة ذات كتلة معروفة بداخله. باستخدام اعتبارات الزخم، يمكن حساب سرعة تأثير الكرة. كلما زادت السرعة، زاد انحراف الكرة في الخشب.

    سؤالي هو: من الواضح أنه عند سرعة معينة، VM، سوف تخترق الكرة وتنتقل إلى الجانب الآخر من الشجرة. ماذا لو أطلق الرصاص بسرعة أعلى بكثير من VM؟ استنتاجي هو أنه فوق سرعة معينة، ستصبح إزاحة السجل أصغر فأصغر، وعند سرعة معينة ستتوقف عن التأثير على السجل على الإطلاق وتمريره دون تحريكه على الإطلاق.

    لذلك، سنحصل على نوع من منحنى الجرس للسرعات التي تؤثر على السجل: السرعات المنخفضة والسريعة جدًا لا تحركه تقريبًا، والسرعات المتوسطة تحركه أكثر فأكثر، حتى سرعة معينة تنحرفه إلى الحد الأقصى، بينما السرعات التي تتجاوز هذه السرعة تنحرفها أقل فأقل حتى عند سرعة معينة تصبح الكرات محايدة فعليًا من حيث تأثيرها على الختم وهو 0.

    التحفظات؟

    سأذهب إلى فيغاس لبضعة أيام لذا لن أتمكن من التعليق. حاول أن ترى إذا كان لدي أي خطأ. إذا لم يكن الأمر كذلك، فأعتقد أنه من الممكن بناء نموذج، ذهني بالتأكيد، ولكن أيضًا عمليًا، في المختبر، من شأنه أن ينتج جاذبية دون احتكاك أو قصور ذاتي، والأهم من ذلك: موجة تنتشر بنفس السرعة في كل إطار مرجع.

    من فضلك لا تصاب بالجنون في غيابي، ولا تفعل أشياء لن أفعلها بنفسي!

  741. طالب علم

    "- لا أعرف عمل بوانكاريه، لكن بولتزمان على حق أيضًا. ما تصفه هو عملية عكسية، حيث يكون التغير في الإنتروبيا صفرًا. "

    من الواضح، ولكن وفقًا لوصفي، قد يكون هناك عامل فيزيائي آخر بسببه لا يكون التغير في الإنتروبيا 0. توسع الكون. لذلك، في نظام الصفائح الدوارة، أثناء دورانه، تصبح كثافة الكون أصغر، وبالتالي تزداد الإنتروبيا.

    مئير.
    ما تقوله مثير للاهتمام. أريد أن أتأكد من أنني فهمتك بشكل صحيح. لنأخذ جملتك الأخيرة:

    "الاستمرار هو السقوط الحر لجسيم أولي داخل مجال جاذبيته"

    والذي يعني في فهمي أن كل جسم يخلق جاذبية، فإذا حاولنا تحويل الجسم عن مركز الجذب الخاص به، فإنه سيقاومه كما يقاوم أي جسم آخر، ومن هنا نحصل على الإصرار. هل فهمت

    اليوبيلات
    أولاً، ينصح بعدم الخلط بين القراء بين الآراء المقبولة وآرائنا الخاصة. مبدأ ماخ، إليبا داماخ، هو كما قدمته. الإضافة: "إن حركة جميع الكتل في الكون مرتبطة بالمادة المظلمة". إنها فكرتك، ويجب الإشارة إليها، حتى لا تربك القراء الذين قد يعتقدون أنها الرأي السائد. بقدر ما أعرف، لا. أنا لا أقول إنها ليست فكرة مثيرة للاهتمام، ولكن حتى يتم إثباتها، يجب التعامل معها على أنها تكهنات.

    وفيما يخص 1- ماذا عن i، جذر 1-؟ هل لديه أيضا جذر؟

    وبشكل عام، ما هي المضامين الفلسفية المتعلقة بالإيمان بالألف الأول؟ هل يمكننا أيضًا أن نؤمن بالله ناقصًا واحدًا؟ وهذا منطقي جدًا في نظري، لأننا في الماضي كنا نؤمن بآلهة كثيرة، اختزلنا إلى إله واحد، والملحدون يؤمنون بالصفر، أليست هذه هي الخطوة المنطقية المطلوبة يا دوكي؟

    بالإضافة إلى ذلك، كذبة خيبت أملي، كذبة. ماذا طلبت في المجموع؟ هل يمكنك أن تشرح لي كيف تعمل اللامكانية في نموذجك؟ هل هذا كثير على لاطلبه؟ مليئة يهودا المختفي، الذي وعد "الاحتكاك علي!" واختفى. لكنك أيها السير جلاهاد الشهم تترك جرحى ينزفون على أرض الملعب؟

  742. اليوبيل,

    لقد كتبت: "إن إجراء العمليات الحسابية على الجمع بدلاً من الطرح يتطلب تعريف وجود رقم معاكس".
    سألت: كيف تعتمد الحساب على الطرح فقط؟ هل تستطيع أن تقول ما الذي لم يفهم في السؤال؟

    قلت: "ولكن يمكن إثبات أن البديهية الأخرى أكثر تعقيدًا".
    سألت: أي بديهية؟ مرة أخرى، هل يمكن أن تقول ما الذي لم يفهم في السؤال؟

    امر اضافي:
    "(والتقسيم. علماء الرياضيات حذرون من هذا الجانب منه)."
    - يتم تعريف القسمة باستخدام الضرب (الضرب بالرقم العكسي). 0 ليس له معاكس، وبالتالي فإن القسمة على 0 غير محددة (على الأقل ليس في الرياضيات "العادية"). يمكنك قراءة المزيد على مدونة غادي ألكساندروفيتش: http://www.gadial.net/?p=168

  743. ر.ه.رفاعي.م
    التشبيه الذي قدمته غير مقبول بالنسبة لي، ولو فقط لسبب أنه في الكون البدائي الذي أحاول بناءه، لا يوجد شيء وبالتأكيد لا توجد شحنات كهربائية. حتى التشبيه الذي أحمله، من الرياضيات (والمنطق)، مبالغ فيه للغاية، لكنه أصغر ما يمكن أن أجده.

  744. يوفال، صديقنا
    في رأيي لا شيء ولا شيء خلق الوجود (حسب تشبيهك) - في رأيي الوجود والوجود لم يخلقا شيئًا.
    ومن المعروف أن كل شحنة موجبة/سالبة تتنافر. القوة بين شحنتين موجبتين تخلق قوة تنافر.
    فكر في الأمر..

  745. اليوبيل,

    0 هو بالفعل عضو محايد - للاتصال. بالنسبة للضرب، حد الوحدة ("المحايد") هو 1.

    بالمناسبة لماذا لم تجيب على الأسئلة التي طرحتها؟ (أجبت بـ "لا أفهم ما لا تفهمه")

  746. ر.ه.، صديقي وعدوي،
    بمعنى ما، فإن "العدم" البدائي الذي أتحدث عنه هو صفر، لكنه ليس محايدًا. لاحظ أن الصفر أيضًا ليس عضوًا محايدًا ولكنه مدمر في الضرب (والقسمة. ويهتم علماء الرياضيات بهذا الجانب منه). إنني أنظر إلى كل جانب ممكن من جوانب "اللا شيء"، و"النفي"، و"الصفر"، و"الفارغ"، وما إلى ذلك، وأحاول أن أستنتج منها ما هو "اللا شيء" البدائي.
    ونعم، أنت على حق في حكمك. ومن بين أمور أخرى، أبني نموذجي أيضًا على دلالات وكلمات مستمدة من المنطق البشري دون أي أساس تجريبي. وبما أننا لا نعرفه فلا نستبعد أي احتمال وكل شيء يؤخذ بعين الاعتبار. هذه هي بداية كل شيء. في هذه المرحلة لا يوجد شيء، ولا حتى الرياضيات أو المنطق.
    وبالمناسبة، ربما تبدأ الأساطير السومرية بنموذج الصفر واللانهاية: في البداية كان هناك الزوج إبسو وتيهيمات، وهما الصفر والهاوية اللانهائية. فالاقتران بينهما (صفر في اللانهاية) خلق الكون واختفى صفره. ثم جاءت القصص

  747. يوفال صديقي
    على فكرة أخرى. المعادلان الرياضيان لـ "نعم" و"النفي" هما + و- على التوالي. العين هي 0 (وليست ناقصًا كما قدمتها). يمكنك الحصول على صفر من إضافة نعمين متساويين: زائد + ناقص ولكن من 0 فقط لا يمكنك الحصول على "نعم" لذا فهو عكس ما تقوله تمامًا.

  748. اليوبيل,
    هل تعتمد نموذجك على الدلالات والكلمات؟ كلمات تنشأ من المنطق البشري دون أي أساس تجريبي؟
    كتمرين فكري سأذهب معك. الجواب هو: "هناك عين". يبدو الأمر وكأنه تناقض لفظي، لكن جزيرتك ليست حقيقية أيضًا ولكنها تشكل نوعًا من "هناك" مثل مجموعة فارغة في الرياضيات. فبمجرد أن تقول "العين اللانهائية" أو "العين عين نفسها" بمعنى أن لها خصائص أو أنها قامت فعلا، ولو كان فعلا، فقد جعلته كائنا له خصائص وخصائص.
    هل الصفر هو "لا شيء" حسب تعريفك؟ لا أعتقد ذلك لأن 0 له خصائص محددة جدًا.
    بالمناسبة، إذا بدأت الأسطورة/النموذج بجملة "في البداية كان هناك 0" سترى أنه لا يمكنك الاستمرار لأنه لا توجد طريقة رياضية لإنشاء رقم من 0 فقط. عينك هي في الواقع رقم سالب وهو مرآة للإيجاب وليس في الحقيقة -0.

  749. بعد أن ذكرت إسرائيل مبدأ ماخ وسؤال أصل ظاهرة الثبات، أنا فقط أسأل.

    إذا كان مبدأ ماخ صحيحا، وجميع الكتل في الكون لديها خاصية مدهشة لمقاومة التغيير في حالة حركة جميع الكتل الأخرى في الكون، وإذا كان صحيحا أن جميع الكتل في الكون هي تقريبا مجموع كتل الجسيمات الأولية التي تتكون منها، ثم الاستنتاج بأن كل جسيم أولي له كتلة له خاصية مدهشة لمقاومة التغير في حالة حركة جميع الكتل الأخرى في الكون أمر لا مفر منه.

    إذا كان لجسيم أولي هذا التأثير المفاجئ على جميع الكتل الأخرى في الكون، أليس من المحتمل أن يكون له هذا التأثير المفاجئ على نفسه أيضًا، بل وأكثر من ذلك على الآخرين إذا أخذنا في الاعتبار أن بعده عن نفسه يميل إلى الصفر (وتذكر قانون التربيع العكسي لنيوتن)؟

    حسنًا، قد لا يكون من المعقول افتراض ذلك، إذا افترضنا أن مجال الجاذبية لجسيم أولي هو كيان لا ينفصل عن الجسيم. ولكن لماذا يجب أن نفترض ذلك؟

    هل يمكنك الإشارة إلى أي ملاحظة تتعارض مع أي من الافتراضات التالية:

    يتم توزيع كتلة الجسيم الأولي في شكل هندسي يشبه الكرة (الجسيم الأولي ليس نقطة "نقية").
    الجسيم الأولي هو كيان مادي منفصل عن مجال جاذبيته.
    يولد الجسيم الأولي مجال جاذبيته في دورات (الجاذبية الكمية).
    يميل الجسيم الأولي إلى الوقوع في مجال الجاذبية الذاتية الذي خلقه في دورة واحدة، إذا تم إزاحته قبل أن يخلق جاذبيته الخاصة في الدورة التالية.
    إن حركة الجسيم الأولي هي حركة مطلقة بالنسبة لمجموع مجالات الجاذبية المؤثرة عليه، وفي المقام الأول الجاذبية الذاتية.
    "الاستمرار هو السقوط الحر لجسيم أولي داخل مجال جاذبيته"
    ?

  750. R.H. Rafai.M،

    يرتبط مجال طاقة الفراغ والجسيمات الافتراضية بالكم على مستوى عالٍ جدًا - وأنا لا أعرف هذا المجال أكثر منك.

    اليوبيل,

    ليس من الواضح لماذا لم تفهم ما لم أفهمه - لقد طرحت أسئلة محددة.

  751. آر إتش،
    الأمثلة التي قدمتها ليست ناجحة. وفي مثال من الأساطير المصرية يظهر الإله الشرير في بداية القصة، وفي الأساطير الصينية يظهر "بان جو الذي كان أكثر حكمة من الأرض وأكثر إلهية من السماء".
    أنت تسأل عن المعقولية والأدلة المبنية على الملاحظات؛ ونحن نرى هذا طوال الوقت في خصائص النفي. محاولات قادرة على دحضه: محاولة خلق "ليس هناك" فقط من "هناك" (دون استخدام كلمات لها معنى النفي) أو، من ناحية أخرى، محاولة نفي النفي بطريقة لا تعطي لك تهمة. إذا كان لديك أي أفكار أيضا، يرجى مشاركتها.

  752. اليوبيل,
    تقول: "الفرق بين نموذجي ووصف الخلق في الأساطير المختلفة هو فقط في مسألة وجود خالق عاقل. لا يوجد شيء من هذا القبيل في "أساطيري".

    فيما يلي أساطير إضافية بدون خالق ذكي:
    http://www.planetnana.co.il/myth16/Egypt/egypt_creation.htm
    http://www.planetnana.co.il/myth169/chinese_creation.htm

    هل نموذجك مختلف عنهم؟ إذا كان الأمر كذلك فماذا؟ محتمل؟ أدلة مبنية على الملاحظات؟ في محاولات قادرة على دحض ذلك؟

  753. طالب علم
    لا أريد أن أزعجك أو أي شيء من هذا القبيل، لكنني لست كبيرًا في الفيزياء والرياضيات. المصطلحات مألوفة بالنسبة لي بالفعل، وأنا أتعلمها، ولكنني لا أزال أفتقر إلى قدر كبير من المعرفة حول هذه المواضيع، لذلك ربما لاحظت بالفعل أنني لا أستطيع أن أخبرك بالكثير.
    عندما تحدثت عن "المجال" كنت تقصد البنية الجبرية. كنت أفكر في اتجاه ميكانيكا الكم. لكن بعد ذلك أحضرت الرابط مع الأستاذ، وأدركت أنه ليس لدي ما أضيفه إلى كلامه. لقد قال بالفعل ما أردت أن أكتبه. على كل حال، هذه هي الأشياء في صياغتي: إذا أخذنا الحقل وعرفنا الحقل على أنه فراغ - فإن الفراغ تلقائيا يأخذ معنى مختلفا عن المعنى الأصلي. أي أن الحقل الأصلي - قبل أن يحصل الحقل على تعريف - هو حقل محايد. يمكنك أيضًا أن تقول حقلًا افتراضيًا، وليس له حجم مادي فعلي. من خلال التدخل الفعلي وتحديد المجال - يأخذ الحقل معنى مختلفًا. وهذا يعني أن الحقل يصبح حقلاً غير الحقل الأصلي. بالفعل هنا يتم إنشاء مفهومين لحقل واحد، وهما مختلفان في الغرض. وبما أن المجال الافتراضي لا يوجد بالفعل في الطبيعة، ولكن يمكن أن يوجد حقل محدد على أنه فراغ في الطبيعة، ومن هنا فإن الطاقة التي تخلق الفراغ تم إنشاؤها من طاقة جاءت من مجال آخر.
    وباختصار، إذا كانت طاقة الفراغ خلقت من طاقة أخرى، فلا يمكن أن تساوي الصفر. (إذا سألتني كيف توصلت إلى هذا الاستنتاج فسوف أجيبك لاحقاً لأنني سأكون مشغولاً جداً في المستقبل القريب).

  754. ومن المثير للاهتمام أن الجميع يقدم أدلة هنا على كيفية معاملة نيوتن أو أينشتاين أو بولتزمان...

    ؟؟؟؟

  755. اليوبيل,

    "تعريف الطرح على أنه إضافة "رقم معاكس" هو دلالات غير ضرورية."
    - ومن المؤكد أنه ليس من الضروري.

    "إن إجراء الحسابات على الجمع بدلا من الطرح يتطلب تعريف وجود عدد معاكس."
    – كيف تعتمد العملية الحسابية على الطرح فقط؟

    "ولماذا ناقص ناقص يعطي زائد؟ هذا هو بالضبط السؤال الذي سيبقى دون حل عندما يتم حل جميع الأسئلة الأخرى. في الوقت الحاضر نحن نقبل هذا كبديهية أو كعبارة تتبع من بديهية أخرى. ولكن يمكن إثبات أن البديهية الأخرى أكثر تعقيدًا."
    - هذا سؤال لم يتم حله إلا في عالم لا تقبل فيه الافتراضات الأساسية في الجبر.
    ما هي البديهية الأخرى الأكثر تعقيدا؟

    خلاصة القول، لا أفهم ما تحاول قوله وأين ترى مشكلة في الإعدادات الحالية.

  756. آر إتش،
    الفرق بين نموذجي ووصف الخلق في الأساطير المختلفة هو فقط في مسألة وجود خالق عاقل. في "أساطيري" لا يوجد شيء من هذا القبيل.
    إن الأسئلة التي طرحتها حول الطاقة، وعن الفيزياء بشكل عام، ليست ذات صلة بهذه المرحلة من النموذج، ولكن يجب تعريفها وشرحها بناءً عليها.

  757. علاوة على ذلك الطرح
    تعريف الطرح على أنه إضافة "رقم معاكس" هو دلالات غير ضرورية. إن بناء العمليات الحسابية على الجمع بدلا من الطرح يتطلب تعريف وجود عدد معاكس.
    ولماذا ناقص ناقص يعطي زائد؟ هذا هو بالضبط السؤال الذي سيبقى دون حل عندما يتم حل جميع الأسئلة الأخرى. في الوقت الحاضر نحن نقبل هذا كبديهية أو كعبارة تتبع من بديهية أخرى. ولكن يمكن إثبات أن البديهية الأخرى أكثر تعقيدًا.

  758. إسرائيل،

    "أنا أتحدث عن كل من الإنتروبيا والوقت."
    - لذلك من المهم معرفة التعاريف وفهمها.

    "لرؤية ذلك، فكر في لوحة تدور حول محور في الفضاء لسنوات. بالنسبة لبوانكاريه، في كل مرة تكمل فيها اللوحة دورة كاملة، فإنها تعود إلى نفس الوضع تمامًا كما كانت من قبل. إذا أهملنا التغيرات الصفرية التي تحدث في اللوحة نفسها، فإن بوانكاريه على حق. الوضع هو نفسه تماما في الجولة 542 كما في الجولة 37، رغم أنه لا شك أن 542 حدث في وقت لاحق".
    - لا أعرف عمل بوانكاريه، لكن بولتزمان على حق أيضًا. ما تصفه هو عملية عكسية، حيث يكون التغير في الإنتروبيا صفرًا. يمكن أن يتوازى مثالك مع نظام الغاز المثالي الموجود في حاوية مغلقة ومعزولة، ومع العبارة التي تقول إنه ليس لدينا طريقة لمعرفة ما إذا كان الجسيم رقم 1 عند النقطة رقم 1 كان في نقطة زمنية لاحقة من عندما كان الجسيم رقم 1 عند النقطة 2. حسنًا، هذا واضح، إنه تجميع قانوني دقيق، والإنتروبيا فيه ثابتة.

    "إن بوانكاريه على حق، بحسب ما كان يعرفه في نهاية القرن التاسع عشر. ولهذا السبب انتهى أيضًا من كلام بولتزمان، الذي ادعى أن الإنتروبيا لا يمكن عكسها في وقت محدد.
    - غير صحيح، على الأقل ليس بالنسبة للمثال الذي قدمته هنا. اقرأ مرة أخرى ما ادعى بولتزمان وما أثبته (حتى النسخة التي أحضرتها من ويكيبيديا).

    فيما يتعلق بالجسيمات الشبيهة بـ La Sage - لا أعرف، لذلك لن أخوض في الأمر.

    R.H. Rafai.M،

    "وبعبارة أخرى، لا يوجد شيء اسمه لا شيء، أو فراغ، وما إلى ذلك."
    - الفراغ كما نفهمه، لا شيء مطلق، ربما غير موجود. يذكر البروفيسور ديفيد جروس ذلك في محاضرته في التخنيون (الدقيقة ~25): http://www.youtube.com/user/Technion#p/c/4/iNmZqH01nX0 (المحاضرة الموصى بها بشكل عام)

  759. طالب علم
    https://www.hayadan.org.il/astronomers-reach-new-frontiers-of-dark-matter-130112/#comment-326233

    لقد أعطيتني فكرة، وسأشرحها هنا لاحقاً. في الأساس، أستطيع أن أخبرك الآن أنني توصلت إلى نتيجة مفادها أنه "لا يوجد شيء اسمه لا شيء" - أي أن "اللا شيء" هو أيضًا "شيء"، مجرد شيء آخر. أو بمعنى آخر لا يوجد شيء اسمه لا شيء، أو فراغ، الخ.. 🙂

  760. يهودا
    وراء كل تعريف من هذا القبيل في الفيزياء هناك فلسفة كاملة.
    ما هو الرقم السلبي؟
    الرقم غير السالب له تمثيل في العالم المادي (على سبيل المثال يمكن تمثيل "1" على شكل تفاحة واحدة).
    الرقم السالب ليس له قيمة فيزيائية حقيقية، ما له هو معنى القيمة.

  761. ر.ح.
    فيما يتعلق بجاليليو - واضح تماما، أليس كذلك؟ ما الفرق بين الراحة والحركة بسرعة ثابتة؟ الحركة - نسبة إلى ماذا؟
    النظام المطلق الوحيد الموجود هو نظام CMBR ومن الممكن بالفعل تعريف الحركة بالنسبة له. (سرعتنا حوالي 370 كم/ثانية باتجاه كوكبة الأسد، إذا كنت أتذكر بشكل صحيح). لكن بصرف النظر عن ذلك، وخاصة في زمن جاليليو وحتى أينشتاين، فإن جميع حركات القصور الذاتي تعادل الراحة، والحركة النسبية فقط هي التي تساوي أي شيء.
    وبالطبع التسارع. هذا مطلق.

  762. طالب.
    أنا أتحدث عن كل من الإنتروبيا والوقت.
    ولرؤية ذلك، فكر في لوحة تدور حول محور في الفضاء لسنوات. بالنسبة لبوانكاريه، في كل مرة تكمل فيها اللوحة دورة كاملة، فإنها تعود إلى نفس الوضع تمامًا كما كانت من قبل. إذا أهملنا التغيرات الصفرية التي تحدث في اللوحة نفسها، فإن بوانكاريه على حق. الوضع هو نفسه تمامًا في الجولة 542 كما في الجولة 37، على الرغم من أنه لا شك أن 542 حدث في وقت لاحق.

    إن بوانكاريه على حق، على حد علمه، في نهاية القرن التاسع عشر. ولهذا السبب انتهى أيضًا من كلام بولتزمان، الذي ادعى أن الإنتروبيا لا يمكن عكسها في وقت محدد.

    وأزعم أنه قد يكون هناك احتمال آخر: إذا قلنا أن سبب الجاذبية والقصور الذاتي هي، على سبيل المثال، جسيمات مشابهة لجسيمات لا سيج، ويتوسع النظام كما في نظرية الانفجار الأعظم، فمع كل ثورة تعود اللوحة إلى حالة مختلفة عن الثورة السابقة، وهي حالة يكون فيها عدد الجسيمات أقل لكل وحدة حجم.

    ولهذا السبب حصلنا على تعريف دقيق لكل حالة زمنية في تاريخ الكون: في كل مرحلة من الممكن معرفة الوقت المحدد في أي مكان في الكون، والذي يتم تعريفه على أنه عدد الجسيمات لكل وحدة حجم. درجة حرارة الكون هي دالة مباشرة لهذا الزمن، بحسب صيغة فريدمان، وهي مثل الزمن، لا تتكرر أبدا في نقطة معينة في الكون.

    سيكون من المثير للاهتمام محاولة معرفة ما إذا كان المشتق الثاني للمسافة بالنسبة إلى الزمن وفقًا لهذا التعريف، وهو التسارع، يعطينا بالفعل قانون نيوتن الثاني في النموذج الهيدروديناميكي. عندما يكون لدي المزيد من الوقت سألعب بهذا، يبدو وكأنه مقلاة.

    ر.ح.

    تعال..
    لذلك قمنا بقياس قوة الطرد المركزي داخل الصحون الطائرة، مع إغلاق نوافذها. لنفترض أن 10 منها تدور بالنسبة لبعضها البعض، وقد توصلنا إلى نتيجة مفادها أن الرقم 3 فقط هو في حالة سكون حقًا. كل الباقي يدور، بعضها في اتجاه عقارب الساعة وبعضها عكس اتجاه عقارب الساعة، وحتى أننا نعرف السرعة التي تدور بها، وكل هذا مجرد قياس بسيط داخل الألواح بمقياس قوة بسيط!

    ولكن السؤال الكبير هو - التناوب بالنسبة إلى ماذا؟

    لبعضهم البعض؟ ففي النهاية، إذا أزلنا 9 منها من النظام، فإن القوة العاشرة ستقيس نفس القوة تمامًا كما كانت من قبل، وهذا في مساحة فارغة تمامًا!

    المفاجأة الكبرى تنتظرنا عندما نفتح النوافذ.

    لأنه اتضح أنه فقط في المركبة الفضائية التي تقيس الطاقة 0، ستبدو النجوم البعيدة في الخارج وكأنها ثابتة. وفي جميع اللوحات الأخرى، ستبدو النجوم وكأنها تتحرك، وبنفس السرعة تمامًا ولكن في الاتجاه المعاكس للسرعة التي قمنا بقياسها في اللوحة.

    هذا هو مبدأ ماخ. القصور الذاتي سببه كل الكتل الموجودة في الكون.

    آينشتاين لديه وصف مختلف. ووفقا لطريقته، فإن الجاذبية والقصور الذاتي متساويان تماما. ولذلك فإن القوة التي يشعر بها الركاب في الصفائح الدوارة سيسميها أينشتاين قوة الجاذبية وليست القوة الطاردة المركزية. غريب بعض الشيء رغم ذلك في اتجاهاته، لكنه لا يزال يمثل قوة.

    عند التحرك في خط مستقيم، تكون مشكلة التسارع هي نفسها تمامًا: التسارع بالنسبة إلى ماذا؟ إذا كانت السيارة تتسارع بالنسبة إلى الطريق، فلماذا لا نقول إن الطريق هو الذي يتسارع بالنسبة إلى السيارة؟ هو من سيتمسك بالكرسي وليس نحن.

    وشيء آخر: هذه ليست قوة ضئيلة، ولكنها قوة هائلة: خذ منشارًا كهربائيًا دوارًا وانظر ما هي القوة التي ستؤثر عليك إذا حاولت إمالةها إلى الجانبين. إذا كان المنشار سريعًا وثقيلًا بدرجة كافية، فسوف يسقطك على الأرض إذا حاولت هزه.

    ما الذي يعارضنا بهذه القوة؟

    وفقًا لـ "النموذج" الخاص بي، فمن الممكن جدًا أن تشبه بعض الجزيئات جزيئات اللازاج. كما أوضحت، عند السرعة الثابتة يكون النظام في حالة توازن. أثناء التسارع، تؤثر قوة، مثل مقاومة الهواء للشراع الذي يتحرك عبره. لم أثبت ذلك، لكن إذا تمكنت من إثبات مفهوم الوقت ككثافة جسيمية رياضيًا، فيمكننا وصف التسارع باعتباره المشتق الثاني للمسافة بالنسبة للوقت، وسنرى ما سنحصل عليه.
    لكنني أترك ذلك لك الآن.

    يهوذا على حق. إن الشحنة "السالبة" للإلكترون هي ببساطة نتيجة لرمي عملة معدنية غير ناجحة. وكان من الأفضل أن نطلق عليها موجبة، لأن وجود فائض من الإلكترونات في الذرة من شأنه أن يجعلها أكثر إيجابية. لكن هذه مجرد مسألة وضع علامات.

  763. ر.ح.
    قلت: "تبين لنا الحقائق أن الشحنة الكهربائية للإلكترون محددة بالسالب. لماذا تم تحديده على أنه ناقص وليس زائد؟" نهاية الاقتباس.
    في رأيي، كل ما لدينا هنا هو جسيمان قررت أن تعطي أحدهما موجبًا والآخر سالبًا. لا توجد خاصية خاصة للإلكترون تتطلب منه إعطاء علامة الطرح. لاحظ أن كتلة البروتون الذي يحمل شحنة موجبة أقل من كتلة النيوترون الذي شحنته صفر، وربما كان من المنطقي أكثر أن نعطي البروتون علامة الطرح.
    مجرد غذاء للتفكير في بداية الأسبوع.
    يوم جيد
    سابدارمش يهودا

  764. إسرائيل،

    فيما يتعلق بالسؤال المتعلق بماهية القوة الطاردة المركزية، يرجى قراءة:

    http://he.wikipedia.org/wiki/מערכת_ייחוס

    وخاصة الفقرة التي تتحدث عن مبدأ النسبية لجاليليو ومحاكاة القوة واكتب رأيك.
    وقم بالتوسيع، والتوسع، ولا تتردد، لا تحتاج إلى السؤال في كل مرة، والحصول على موافقة تلقائية مقدمًا.

    اليوبيل,
    القول بعدم وجود سفر التكوين ليس نظرية بل صوفية. ما هي طبيعة العدم؟ هل هي موجودة اليوم؟ فهل من الممكن القيام بتجربة (ولو فكر) لاختبار طبيعتها؟ ماذا يعني المايا نفسه؟ من أي طاقة؟ إذا كان أيونًا حقيقيًا فلن يكون لديه طاقة، وإذا كان لديه طاقة فما نوع الأيون هو؟
    باختصار، أسئلة تلو أسئلة ليس لها أدنى دليل إلا قصة جميلة. ما هو الفرق الأساسي بين قصتك وسفر التكوين الذي خلق الله السماء... أو كل القصص الأخرى التي تظهر هنا: http://he.wikipedia.org/wiki/בריאת_העולם?
    هل حقيقة أنك وصفت نموذجك بـ "النموذج" وليس الأساطير تجعل الأمر مختلفًا؟

  765. إسرائيل،

    "من الواضح أن الإنتروبيا لا يمكن عكسها بمرور الوقت في أنظمة من رتبة رقم أفوجادرو أو أعلى."
    - حتى في الأنظمة التي تحتوي على عدد أقل بكثير من الجزيئات.

    "لكن لا تزال هناك مسألة قابلية عكس قوانين نيوتن في الزمن، كما أظهر بوانكاريه، مدفع رياضي.
    اقتراحي هو ربط كل لحظة محددة من الزمن بحالة محددة لعدد من الجسيمات لكل وحدة حجم، كما يمكن القيام به باستخدام بالون قابل للنفخ.
    - إذن أنت لا تتحدث عن الإنتروبيا، أنت تتحدث عن إمكانية عكس قوانين نيوتن في الوقت المناسب. يتم تعريف الإنتروبيا في الميكانيكا الإحصائية، وهي خاصية إحصائية للنظام.
    لم أفهم ما اقترحته وأين تحاول المضي قدمًا فيه.

  766. R.H. Rafai.M،

    "...السؤال "من أين أتت (أ-)؟"
    - من الميدان. العضو المقابل في الاتصال المميز بـ (-a) هو عضو في الحقل، إذا قمت بتوصيله بـ a فستحصل على العضو المحايد ("اللامبالاة الملتصقة"). علامة الطرح التي قبلها هي مجرد علامة، وكان بإمكاننا تمييزها بشكل مختلف. المصطلح المحايد هو أيضًا مصطلح في الحقل، إذا قمت بتوصيله بـ a فستحصل على. إن وجود مصطلح معاكس بالإضافة إلى ذلك يسمح لك بتعريف عملية الطرح بطريقة مألوفة لديك.

    وبعبارة أخرى، يجب أن تفترض "شيئًا" لتعريف "لا شيء". ومن ثم يصبح "اللا شيء" "شيئًا".
    - أعتقد ذلك. يجب عليك تقديم بعض الافتراضات لتحديد.

  767. طالب.
    مقالة جميلة، شكرا.
    من الواضح أن الإنتروبيا لا يمكن عكسها بمرور الوقت في الأنظمة ذات ترتيب رقم أفوجادرو أو أعلى. لقد أغلقنا ذلك منذ وقت طويل. ولكن لا تزال هناك مسألة قابلية عكس قوانين نيوتن في الزمن، كما أظهرها بوانكاريه، وهو مدفع رياضي.
    اقتراحي هو ربط كل لحظة محددة من الزمن بحالة محددة لعدد من الجسيمات لكل وحدة حجم، كما يمكن القيام به باستخدام بالون قابل للنفخ.

    ر.ح.

    لقد فهمت الأمر بشكل صحيح.
    لكن غليون التدخين لن يساعد. كل مركبة فضائية في نظامها تكون في حالة سكون، فما الاحتكاك؟ هذه هي بالضبط المسلمة الأولى للنسبية: من المستحيل أن نعرف بأي شكل من الأشكال من الذي يتحرك ومن هو في حالة سكون.

    لكن لاحظ أنه في مثال الصحون الطائرة فإن الوضع مختلف. من الممكن أن نحدد على وجه اليقين أي لوحة تستقر وأيها تدور وبأي سرعة وفي أي اتجاه. ومن هنا السؤال: التدوير بالنسبة إلى ماذا؟
    هذا هو دلو نيوتن الدوار، ومن هنا جاء مبدأ ماخ ومبدأ التكافؤ في النسبية العامة. إذا كنت تريد، سوف نقوم بالتوسع.

    ليس لدي أي ادعاءات لنظرية ميتا جديدة. على العكس من ذلك، فأنا شبه متأكد من أنني ارتكبت بعض الأخطاء الأساسية في فهم التناقض المفترض في النظريات القائمة. ومع ذلك، لم يتمكن أحد حتى الآن من إقناعي، والأكثر من ذلك، أن الجميع يقولون شيئًا مختلفًا، وفي كل نسخة هناك تناقضات أساسية متأصلة.

    كما هو الحال دائما، إذا كنت تريد، سوف نقوم بالتوسع.

    فقط لاحظ كم هو جميل أن فكرتي تحل مشكلة ثبات سرعة الضوء في جميع الأنظمة المرجعية، والاحتكاك في LS، وتمكن من القصور الذاتي وعدم المحلية.

    كما هو الحال دائما، إذا كنت تريد... وكأنه أنقذ العالم كله!
    اه لقد أخطأت في القول

    أشباح
    ضعيف.
    ليست لدي مشكلة مع الكساح، فقط مستوى الكتابة منخفض.
    ومن العار أن نضيع وقتنا في مشاحنات لا طائل من ورائها. أقترح عليك قراءة تعليقاتي وطرح الأسئلة. إذا لم تكن مهتمًا، أو غير قادر، هيا، قل الكلمة الأخيرة واتركها.

    اليوبيلات
    5.5
    وكنا نأمل أن نحصل على النظرية الكاملة، أو على الأقل مسألة اللا محلية. أين؟

  768. طالب علم
    أولا شكرا على الرابط. الموصى بها للجميع.
    ماشيل ر. أجب عن إجابة مشابهة لإجابتك، إلا أنه بعد ذلك ظهر السؤال "من أين جاءت (أ-)؟"
    أي أنه يجب عليك أن تفترض "شيئًا" لتعريف "لا شيء". ثم يصبح "اللا شيء" "شيئًا".
    آسف على الشرح المختصر، أتمنى أن تكون فهمت قصدي.

  769. يهودا
    لكن الحقائق توضح لنا أن الشحنة الكهربائية للإلكترون سالب. لماذا تم تعيينه على أنه ناقص وليس زائد؟

  770. ر.ح
    أليس الأصح أن نقول أن الإلكترون هو عكس البروتون. أعط أحدهما ناقصًا والآخر سيحصل على علامة زائد ولا يهم ماذا لمن. يمكننا أن نقرر أن إشارة الإلكترون موجب ثم يحصل البروتون على علامة السالب.
    لذا فإن السؤال الذي يجب طرحه هو: - ما هو جوهر الأضداد التي يحصل عليها الإلكترون والبروتون. أو ماذا يعني مصطلح "الشحنة الكهربائية"؟
    ويوفال
    عدم قبول إجراءات الاتصال الخاصة بك بشكل صحيح
    اسبوع جيد
    سابدارمش يهودا

  771. اليوبيل,

    "... وفي الواقع يمكن تعريف الجمع باستخدام الطرح (ولكن ليس العكس)."
    أنا أعرف بالفعل تعريف الطرح الذي ينتج عن وجود حد مضاد، أي أن الطرح يعرف بمساعدة الجمع. أين رأيت تعريف الجمع بالطرح ولماذا تزعم أن العكس لا يعطى؟

  772. بالإضافة إلى ذلك، في رأيي، الطريقة الصحيحة للإجابة على هذا السؤال هي من خلال فهم سبب كون الإلكترون سالبًا.

  773. يوفال
    دعونا نبدأ من البداية:

    لماذا - وأكثر - يساوي +؟
    (لقد سألت مايكل روتشيلد نفس السؤال منذ وقت ليس ببعيد).

  774. R.H. Rafai.M،
    مثال على فكرة فلسفية:
    بادئ ذي بدء - "اللا شيء"، النفي. لا يوجد شيء سوى "لا شيء"، وبالتالي ليس للنفي ما ينفيه سوى نفسه. النفي ينفي نفسه وينفي وينفي وينفي بلا حدود إلى ما لا نهاية، لأن هذا هو جوهره ومعناه الوحيد. وهكذا من "لا" نشأت "لا-لا"، والتي تسمى في لغتنا "نعم" للاختصار، و"لا-لا-لا"، والتي تسمى ببساطة "لا"، وما إلى ذلك. وهكذا من "اللا شيء" يُخلق "اللا شيء"، وهو ما يُسمى في لغتنا "نعم"، و"اللا شيء في العدم" الذي، في حدسنا، نفكر فيه على أنه "لا شيء" ونتخيل أنه هناك ولا فرق بين "لا شيء" المفرد و"لا شيء" مثلي أو خماسي أو مربع ونحو ذلك. استخدام الأرقام هنا مستعار من علم الحساب: من أجل الإيجاز نسمي "ناقص ناقص" الاسم "زائد" وبالتالي نحصل كما لو أنه بجانب عملية الطرح هناك أيضًا عملية الجمع. "الجمع" هو اختصار لـ "الطرح مرتين"، ونظرًا لعدم التنوع في العمليات الحسابية فهو أيضًا اختصار لـ "الطرح أربع مرات" و"الطرح ستة مرات" وهكذا، وفي الحقيقة يمكن تعريف الجمع باستخدام الطرح (ولكن ليس العكس). بهذه الطريقة، "نعم" هو اسم مختصر لأي مجموعة من "لا"، وفي الواقع يمكن تعريف كل "نعم" باستخدام "لا" وحده.

  775. بالمناسبة، يوفال

    "في البدء لم يكن هناك شيء، لا شيء؛ أليس نفيا فاعلا وقد عذب نفسه إلى "" - كيف؟
    (سأكون ممتنًا للحصول على إجابة مفصلة حتى تكون مفهومة ويكون هناك شيء للمناقشة)

  776. RH، لا يوجد خطر على سلامة قبعتك 🙂
    سابدارمش يهودا، من المستحيل أن أحصل على جائزة نوبل. يتم توزيعه على الأحياء فقط ويتم إعطاؤه بعد عدة عقود من الاكتشاف الذي أكسبهم. لنفترض أننا نشرنا أعمالنا في العام المقبل وحصلنا على قدر كبير من التقدير، وبحلول الوقت الذي يأتي فيه دورنا سنكون على الأقل 90 عامًا. ما هو احتمال أن نعيش حتى ذلك الحين؟ 🙁

  777. R.H. Rafai.M،
    نعم. لن نصل أبدا إلى الأفق، لكننا نسير على الطريق الصحيح إليه. إن المزيج الفائز من الآمال الوهمية والتفكير النقدي هو الذي أوصلنا إلى هذا الحد وسيستمر في إيصالنا إلى أبعد من ذلك.

  778. يوفال
    وصحيح أن المثقفين يحاولون الاقتراب من هذه النظرية. لكن في رأيي أن مثل هذه النظرية مجرد وهم.
    وهذا الوهم هو الذي "يدفع" المثقفين إلى السعي لتحقيقه.
    لولا الأمل (وقد يقول البعض الإيمان) لما وصلنا إلى ما وصلنا إليه.

  779. ر. ح. عزيزي
    دموع السعادة بلل عيني. هنا هنا. هناك من يؤمن بصلاح طريقنا إلى المكافأة (المكافأة هي المعرفة)
    وفيما يتعلق بالأمور الواردة في ردك:- أعرف النكتة من السهرة وأعجبتني كثيراً. سيتم سردها من وقت لآخر في محاضراتي.
    يجب أن يكون الأمر دقيقًا، وقد كان عالم الرياضيات على حق، لأن الأشياء المثبتة فقط هي التي يمكن قبولها على أنها صحيحة: - يوجد حقل واحد على الأقل في اسكتلندا به خروف واحد على الأقل على جانب واحد منه واحد على الأقل أسود.
    لقد أخطأ الفيزيائي وعالم الأحياء لأنه من الممكن أن يكون الخروف أسود من جانب واحد فقط.
    وفيما يتعلق باختلاف سرعة الضوء، تقبل صيغة التباين الخاصة بي كمساهمة في نكتتك الجميلة، حسنًا:-
    تتناسب سرعة الضوء مع جذر درجة حرارة الخلفية للكون.
    مجرد استنتاجات بسيطة عن الموقع الغازي.
    T.L.H.
    اسبوع جيد
    سابدارمش يهودا

  780. R.H. Rafai.M.، فيما يتعلق بمسألة ما إذا كانت نظرية كل شيء ممكنة على الإطلاق.
    أنا أميل إلى أتفق معك. ومع ذلك، فإن الطموح الفكري، على الأقل بالنسبة للبعض منا، هو الاقتراب من مثل هذه النظرية قدر الإمكان. وهذا هو الحال، على سبيل المثال، مع قصة الانفجار الكبير ونظرية الأوتار. تبدأ نظريتي (ببساطة شديدة) تقريبًا على النحو التالي: "في البدء لم يكن هناك شيء، لا شيء؛ العدم هو نفي نشط، وقد عذب نفسه حتى لا شيء." حتى لو توصلنا من هذا إلى نوع ما من النماذج التي تشمل كل الفيزياء، فإن أسئلة مثل "ما هو الشيء السلبي الذي يجعلها نشطة؟" ستظل مفتوحة. ومع ذلك، فإن هذه الأسئلة ليست كثيرة، ولهذا السبب تقترب مثل هذه النظرية كثيرًا من الهدف المنشود وهو "نظرية كل شيء".

  781. يهودا،
    أولا ليس لدي أهداف مظلمة. أنا هنا على مستوى الناقد السينمائي. على عكسك يا إسرائيل ويوفال كاتوناتي من حل مشكلة المشاكل في الفيزياء الحالية وإيجاد نظرية تشرح كل شيء بطريقة مختلفة. على الرغم من أنه يبدو لي أنكم الثلاثة مخطئون، إلا أنني أتمنى لكم النجاح حقًا وأعدكم بالحضور إلى ستوكهولم لحضور الحفل وأقول لمتلقي الجائزة: أحسنت!! أعترف أنني لم أصدق أنك ستصل إلى هذا الحد ومع ذلك فقد فعلت! ثم سأخلع قبعتي، وأقوم بانحناءة جميلة وأتناول قبعتي بكل سرور (بالمناسبة، القبعة جاهزة بالفعل، لذا كل ما تبقى لك هو كتابة المعادلات أو قياس سرعات الضوء المتغيرة أو أي شيء آخر) مما يقنعك بصحة كلامك).

    في واقع الأمر، أظهرت جميع الملاحظات مثل تلك التي أجراها مايكلسون مورلي وجميع التجارب المعملية أنه هنا في منطقة النظام الشمسي في هذا الوقت، قبل 15 مليار سنة من الانفجار الكبير، تبلغ سرعة الضوء في الفراغ 300 ألف كيلومتر في الثانية. الثانية دون الاعتماد النسبي. بالطبع يمكنك أن تقول نعم، ولكن قبل 100 عام كان الأمر مختلفًا أو نعم، في أندروميدا كان الأمر مختلفًا ولكن بعد ذلك سوف يتقلب أوكهام في قبره ونحن حقًا لا نريد ذلك، أليس كذلك؟

    وفي هذا الموضوع سأقتبس نكتة مع رجل عجوز:
    يسافر عالم أحياء وعالم فيزياء وعالم رياضيات إلى أيرلندا، وفي رحلة بالقطار فيها يرون خروفًا أسود في أحد الحقول.
    قال عالم الأحياء: "من المثير للاهتمام أن جميع الأغنام هنا سوداء."
    أجاب الفيزيائي: "ليس بالضبط، على الأقل بعض الأغنام هنا سوداء".
    ويجيب عالم الرياضيات: "يوجد في هذا البلد حقل واحد على الأقل، فيه خروف واحد على الأقل، يكون أحد جوانبه على الأقل أسود".

  782. شروليك (شابيرا)

    الذكاء الخاص بك يقطر منك فقط. كيف لا تتلاشى في الخلفية؟
    لقد طرحت عدة أسئلة. يطالب بالإجابة عليهم. يقول لديك فكرة عن الحلول. ولكن لا تظهر أي حل. هل أنت متشتت أنا أيضًا وعدد قليل من الآخرين هنا. إذا كنت لا تعرف كيف تشرح نفسك بوضوح، فهذه علامة على أنك نفسك لا تفهم ما تريد شرحه.
    ماذا حدث؟ لقد استيقظت في الصباح مع اختلافات في الضغط داخل الجمجمة، وشعرت بجاذبية قوية في معدتك وعندما نظرت إلى الجانب رأيت كلبي مع جورب على أذنه، وزجاجة فارغة من العلامة الزرقاء ملتصقة بمخلبه ولا يمكنك أن تقرر أي منكما يتجول؟

  783. يوفال
    لا أعتقد أن نظرية كل شيء ممكنة على الإطلاق. لقد كانت نيتي أن نظرية على مستوى نظرية كل شيء، أو على مستوى النظرية النسبية أو نظرية نيوتن مثلا، ستجلب معها أجوبة وأسئلة فلسفية جديدة. (على أية حال فإن رأيي مشابه تماماً لما كتبته في ردك لي)

  784. ر.ح
    لدي شعور بأنك "تطوي" الأمور حسب رأيك،
    الذي أقصده:-
    هل تفترض أن السرعة الطبيعية للضوء ثابتة؟
    هل تتذكر ما حدث منذ وقت ليس ببعيد أثناء تضخم الكون، بعد أجزاء قليلة من الثانية من الانفجار الأعظم؟
    قرر افتراض التوسع التضخمي للكون بدلاً من تغيير سرعة الضوء إلى سرعة أعلى. مثير للسخرية حقا!
    تحاول أيضًا الحفاظ على ثبات الضوء حتى لا تكون هناك أي مشاكل كمومية؟ هذا هو النهج الذي إذا لم أقدره لك سأسميه سخيفًا.
    وأخيرًا تشارك شفرة Ockham's Razor لتحقيق أهدافك المظلمة فيما يتعلق بثبات الضوء
    "شفرة أوكهام التي تقول أنه ليس لدينا حاليًا أي سبب لنفترض أن سرعة الضوء تختلف في أي مكان آخر." نهاية الاقتباس. ومن معرفتي الوثيقة بوليام الراهب الأوكامي الذي سمي الموسى باسمه، فإن المذكور كان بخيلاً وزاهداً على نحو فظيع. أنا متأكد أنه لو كان على قيد الحياة اليوم لحاول إبطاء سرعة الضوء.
    مساء الخير
    سابدارمش يهودا

  785. إسرائيل،
    أعتقد أنني حصلت عليه. في المثال الثاني، إذا كانت إحدى السفن الفضائية تدور والأخرى لا تدور، فإن الدوارة سيكون لديها قوة طرد مركزية سترمي الأشياء إلى الجانبين ولن تفعل الأخرى، هل أنا على حق؟

    فيما يتعلق بالمثال الأول، لنفترض أن كل مركبة فضائية ستضع غليون تدخين يخرج من المركبة الفضائية، أليس من الممكن معرفة من كان يتحرك فعلياً؟ هل سيستمر الدخان بالتحرك معها؟ أعتقد أنه يكفي أن يكون هناك احتكاك بسيط حتى يتمكن الدخان في النهاية من تحديد من يتحرك ومن يستريح فيما يتعلق بالمساحة. وإذا عدنا إلى الملاحة البحرية مرة أخرى، فهذه هي الطريقة التي تم بها قياس سرعة السفينة في الماضي بالنسبة للتيار المحيط بها، وذلك عن طريق رمي حبل به عقد في الماء وحساب السرعة التي تخرج بها العقد من السفينة (وبالتالي وحدة سرعة العقدة).

  786. يهودا،

    إن وجود "نظرية كل شيء" لم يثبت بالطبع، لكن هناك اعتبارات تدعمها. ومن بينها اعتبارات جمالية قد تبدو غير ذات صلة، ولكنها أثبتت نفسها حتى يومنا هذا، على سبيل المثال في نظريات الجسيمات: الجدول الدوري، والنموذج القياسي، والتناظرات في الطبيعة، وأكثر من ذلك بكثير.
    ثانيًا، الافتراض بأن الثوابت الفيزيائية تتغير بمرور الوقت أو في المكان هو أمر إشكالي للغاية.
    لنفترض أن الضوء يتصرف بشكل مختلف في الفضاء، أين الحد؟ هل الحد الكمي (أي سرعة الضوء تقفز من X إلى Y) عند نقطة معينة، وإذا كان الأمر كذلك، فما هو المميز في هذه النقطة؟ ربما يكون الانتقال تدريجيًا، فما الذي يحدد التدرج؟ وبالمثل بالنسبة لأي ثابت فيزيائي آخر. النقطة الأخيرة هي شفرة أوكهام التي تقول أنه ليس لدينا حاليًا أي سبب لنفترض أن سرعة الضوء مختلفة في أي مكان آخر.

  787. إسرائيل،

    مكتوب هناك (على الرغم من أنه يدعم ادعائي، فقد أخبرتك بالفعل أن ويكيبيديا ليست مصدرًا موثوقًا):
    "أثبت بولتزمان رياضيًا باستخدام الأساليب الفيزيائية أن الحالة الفيزيائية لنظام مغلق لا يمكن أن تكرر نفسها إلا إذا لم تتزايد الإنتروبيا. الإنتروبيا، وفقا لبرهانه، يجب أن تزيد أو تبقى ثابتة مع الزمن، ولكن لا يمكن أن تنخفض بشكل طبيعي. ستزداد الإنتروبيا في أي عملية لا رجعة فيها (والغالبية العظمى من العمليات الطبيعية تنطوي على إنتروبيا متزايدة). "

    كما قلت، هناك تفسير لزيادة الإنتروبيا مع مرور الوقت، وليس العكس. لقد أخبرتك بالفعل في مناقشة سابقة، في مثال بسيط للغاز المثالي، أنه من الناحية الرياضية هناك بالفعل إمكانية لتقليل الإنتروبيا، لكن مثل هذا الوضع ليس فيزيائيًا لأنه يتطلب وقتًا لا نهائيًا (لانهائي أكبر من العمر الكون).

    مقالة مثيرة للاهتمام حول هذا الموضوع:
    http://www.scholarpedia.org/article/Time%27s_arrow_and_Boltzmann%27s_entropy

  788. عزيزي يهودا سابدارمش أيضًا،
    ديفيد يوم، على ما تقول، هو تجريبي. وهو بالطبع على حق، بالمعنى الضيق الذي يتحدث عنه، وهناك بالفعل أماكن يتم فيها قبول طريقته (على سبيل المثال في المحاكم). لكن التجريبية، على الرغم من صحتها، لن تقدمنا ​​كثيرا.
    سبت سلام ومبارك

  789. عزيزي يوفال
    تأتي ماكينة Occam's Razor لاختيار الأبسط من حيث الراحة. قطعا لا ينبغي أن يتم ذلك من تحديد الصواب. تم اختيار نيوتن ليس لأنه أصح من أينشتاين - النسبية! لأنه من الملائم العمل معه حتى في المجرات.
    الصيغة أو النظرية تكون صحيحة فقط عندما يتم اختبارها. قال ديفيد هيوم، الفيلسوف الإنجليزي، وغيره كثيرون، إذا رأيت صحة ألف حالة فهذا يعني نتيجة واحدة فقط وهي أن الألف حالة التي راجعتها صحيحة. قد تكون الحالة التالية مختلفة. جميع أنواع "القواعد" مثل شفرة أوكام أو المبادئ الكونية المختلفة هي مجرد محاولات سيئة من قبل العلماء الذين يشعرون بأن قدراتهم محدودة.
    وبالمناسبة، فإن أخذ احتمال واحد فقط من اللانهاية واستخلاص استنتاجات منه على كل شيء هو أمر مبالغ فيه للغاية
    شاب شالوم
    سابدارمش يهودا

  790. إسرائيل الحلوة
    شيئًا فشيئًا تمكنت من إقناعي بأن علم الفيزياء هو في الواقع فرع من الشعر الغنائي.
    ما بعد النصي. كم عمر تومر اليوم؟

  791. سابدارميش يهودا،
    ينظر إسرائيل شابيرا إلى النماذج المادية الموجودة ويجد أنها تتناقض مع بعضها البعض. ولهذا السبب بالضبط نحتاج إلى نظرية واحدة لتفسير كل شيء.
    عندما قرر نيوتن أن كل جسيم من المادة في كل مكان في الكون يتصرف بنفس الطريقة، قام ببساطة بتطبيق شفرة أوكهام. ربما كان مخطئًا، كما اقترحت، ولكن إذا لم يفعل ذلك لكان قد افترض أنه في كل مكان وفي كل مكان توجد قوانين طبيعة فريدة بالنسبة له. وكما رأينا منذ أيام أينشتاين، فإن فيزياء نيوتن ليست دقيقة بالفعل، لكنها لم تكن مرفوضة، بل "موسعة". وبنفس الطريقة، قد تحتاج سرعة الضوء أيضًا إلى صيغ إضافية.

  792. R.H. Rafai.M،
    وبما أنك وجهت لي الأمور، فأنا أجيب بدلاً من ر.ح.:
    في الجامعة التي أدرس فيها، كانت مادة الفيزياء تسمى "فلسفة الطبيعة" حتى وقت قريب. ولعل كلمة فلسفة أخذت معنى غير محترم (ياني "الفلسف") أو أنه حدث ببساطة ما يحدث حيثما تبلورت الأفكار التي بدأت كفلسفة في صيغ أنيقة.
    عندما أطرح السؤال الذي لم يتلق بعد إجابة منظمة: "ما هو بداية الأشياء؟"، فإنه لا يزال من طبيعة الفلسفة. ولكن عندما يتم العثور على الجواب الذي سيتفق عليه المجتمع العلمي بأكمله، فإنه سيأخذ مكانه المشرف والمستحق على رف الفيزياء.

    والآن سأجيب على سابدارمش

  793. يوفلي اللطيف.

    بدأ كلبي اسمه بـ "راجازي" ولكن بسرعة كبيرة اختصرناه إلى كلب، واختصارًا إلى كلب، واختصارًا إلى كلب، واختصارًا إلى كلب، ومن ثم - إلى كلب.

    كما هو مكتوب: كلبي هو كلبي ابن كلب، ذو فرو بني وعيون بنية.

    وفيما يتعلق بميتزي، فقد ظهرت في أحد المقاطع في أغنية الطفل:

    مر الوقت، أطفالنا في روضة الأطفال،
    والمشاغب هو أيضا شقي جدا
    وقمنا بتجربة كلمات جديدة لنتعلمها
    والقليل لتوسيع آفاقه.
    الفأر يصدر صريرًا،
    الطير يغرد يغرد يغرد ،
    اسم القطة كيكي
    القطة تسمى ميتزي.

    فرفع الجلجال عينيه إلى فوق
    وأوضح، السيارات السيارات السيارات
    وأعطته سوليدا ماتيرنا حسب الرغبة
    وكلاهما غنى الشعار.

    سيارة سيارة سيارة
    ليس له ثاني ولا نظير
    والكثير من الأطفال المبتهجين
    في زاوية فينتورا في دي سوتو
    وناني سوليد، تتنهد جانبًا،
    كا بونيتو ​​مي بيرسيوسوتو.

  794. طالب علم

    http://he.wikipedia.org/wiki/%D7%9C%D7%95%D7%93%D7%95%D7%95%D7%99%D7%92_%D7%91%D7%95%D7%9C%D7%A6%D7%9E%D7%9F
    ر.ح.

    إذا كنت قد شاهدت فيديو Feynman على YouTube، فالعارضة L.S. من الغاز دون تصادمات يمكن أن يؤدي بالفعل إلى الجاذبية، على سبيل المثال بين كتلتين من البلاستيسين التي سوف تمتص بعض التصادمات، ويمكن إثبات أنها تتناسب عكسيا مع مربع المسافة.

    والسؤال هو: هل هذا ما يحدث بالفعل على أرض الواقع؟

    ويقال أن كتلتين من هذا القبيل تمثلان الشمس والأرض. فإذا تحركت الأرض حول الشمس فإنها سوف تصطدم بجزيئات مماسة لاتجاه دورانها، كما لو قمت بتدوير حجر مربوط بخيط فإنه سيصطدم بالهواء المحيط به ويحدث احتكاكاً. هذه هي مشكلة الاحتكاك مع L.S.

    يهودا، ر.ه. في مسألة الجمود والجيروسكوب.

    للتأكد من أننا نتحدث عن نفس الشيء، سأعطيك مثالاً يوميًا من الفضاء.

    يقال أن هناك سفينتين فضائيتين تتحركان بسرعة معينة وثابتة، دون تسارع، بالنسبة لبعضهما البعض. يقوم كلاهما بتصوير بعضهما البعض، ويتم إرسال مقاطع الفيديو إلى الأرض.

    على الأرض، يتم تحليل مقاطع الفيديو، والتي، كما هو متوقع، تظهر نفس الصورة ولكن في الاتجاه المعاكس: فيديو المركبة الفضائية A يُظهر المركبة الفضائية B تتحرك بعيدًا بسرعة V، والفيديو B يُظهر A يتحرك بعيدًا بسرعة V-. .

    الأسئلة: هل يمكن لإحدى المركبات الفضائية أن تدعي أنها ساكنة والأخرى تتحرك؟ هل من الممكن قياس بعض القوة داخل سفينة الفضاء التي ستحدد بشكل لا لبس فيه من الذي هبط بالفعل؟ فهل يمكن استنتاج هذه القوة من خلال النظر إلى مقاطع الفيديو فقط؟

    سنكرر التجربة الآن، ولكن بدلًا من السفن الفضائية سنستخدم الصحون الطائرة التي تدور حول محورها بسرعة زاوية ثابتة، واحدة فوق الأخرى على مسافة بضعة أمتار من بعضها البعض.

    يرسلون مقاطع الفيديو إلى الأرض، ويتم طرح نفس الأسئلة مرة أخرى: هل من الممكن معرفة من مقاطع الفيديو فقط ما إذا كان أحد قد هبط بالفعل، وهل من الممكن قياس بعض القوة في الصفائح التي ستحدد من يستريح ومن يدور ؟ وإذا كنت تتحرك نسبيا لماذا؟ وبأي سرعة زاوية بالدورات في الثانية؟

    حاول الإجابة على الأسئلة، حتى نتمكن من المزامنة.

    ر.ه.رفاعي.م

    أقترح أن نستمر في معاملة بعضنا البعض بنفس الطريقة التي تعاملنا بها مع بعضنا البعض حتى الأمس عندما ذهب البول إلى رأسك: ألا نعامل بعضنا البعض على الإطلاق. إذا نظرت أكثر قليلا من خلال الأرض، يمكننا أن نبدأ المناقشة. إذا هاجمت، فسوف تهاجم.

    ليس لدي القوة والوقت للبدء في الشرح لك منذ البداية كل المشاكل الموجودة في رأيي في النماذج الموجودة، وفكرتي. وهي مفصلة تماما في تعليقاتي. إذا كان لديك سؤال محدد، سأكون سعيدًا بالإجابة عليه.

  795. إلى يوفال ر. ه. وآخرون
    أنت ترتكب الخطأ الذي ارتكبه كل العظماء. متحمس ويسهل ربط (النظرية) بكل شيء!
    نيوتن، على سبيل المثال، اكتشف الجاذبية وقرر على الفور أنها موجودة لكل جسيم في الكون. لماذا؟ أثبت أنه موجود في كل نقطة في الكون؟ لا! ولكن ما الذي يهمه أن يقول مثل هذا؟ ومنذ ذلك الحين لا نقبل أن يكون الأمر خلاف ذلك.
    مثال آخر:- القول بأن سرعة الضوء هي واحدة في كل نقطة في الكون وفي كل زمان. هل قام أحد حقاً بقياس هذا؟ ففي نهاية المطاف، كان من المستحيل إثبات شيء كهذا في الحياة في وقت واحد عندما حدث ذلك في زاوية نائية من مجرة ​​درب التبانة، بعد سنوات من الانفجار الأعظم.
    لماذا أخبرك بهذا؟
    لأنك هنا تحاول إيجاد نظرية عن كل شيء! لماذا؟ هل يمكنك إثبات أن النظرية التي تكتشفها ستكون صحيحة في كل شيء؟ وسيبقى دائما فقط كجزء من إعلان رسمي
    دعونا نقلل من نظرياتنا البسيطة ومن وقت لآخر سوف نتعلم شيئًا من .... نظرية كل شيء.
    وبالمناسبة، لست متأكدًا من وجود دليل على وجود نظرية لكل شيء!. لقد كتبت ذات مرة مقالاً بعنوان "تطور النظريات" والذي خلصت منه إلى أن القول بأن هناك نظرية لكل شيء هو مثل القول بأن هناك إنتاجاً أكثر تطوراً ولا يحتاج إلى مزيد من التطور. يبدو الأمر غير معقول بعض الشيء بالنسبة لي.
    تمامًا مثل ذلك، تأملات حزينة في صباح يوم السبت.
    يوم جيد
    سابدارمش يهودا

  796. يوفال

    يبدو لي أن الشيخ ببساطة لم يفهم أن الحل الفيزيائي/الرياضي في شكل "نظرية كل شيء" سيأتي مع الحل على المستوى الفلسفي - كقطعة واحدة.

  797. R.H. على ردك على هذا https://www.hayadan.org.il/astronomers-reach-new-frontiers-of-dark-matter-130112/
    لقد قدمت الأساطير محاولة لحل اللغز، لكنها فشلت في تقديم حل أنيق لمسألة من خلق الخالق. حتى الفيزياء اليوم لا تدعي تقديم مثل هذا الحل ولكنها تحاول فقط تقديم حقيقة بدائية بسيطة قدر الإمكان. لا يتعين علي أن أتجاوز السؤال "كيف بدأ كل شيء؟" هدفي هو إظهار أن جميع ظواهر العالم المادي هي تطورات لكيان أساسي واحد فقط. ليس أنا فقط، بل الكثير منا هنا وفي مجالات العلوم الأخرى يبحثون عن نظرية واحدة لكل شيء. ربما ينبع هذا من الحدس وأجيال عديدة من الخبرة التي بموجبها يكون لكل ظاهرة معقدة سلف بسيط. ويصدق هذا ليس فقط على الفيزياء (والعلوم الطبيعية بشكل عام)، بل وأيضاً على العلوم الاجتماعية والإنسانية والعديد من المجالات الأخرى. السؤال "كيف بدأ كل شيء؟" ويسبقه السؤال "ما كل هذا على أي حال؟"، كما أشرت بشكل جيد، فهو وثيق الصلة بالموضوع.

  798. إسرائيل

    ما يعرفه العلماء اليوم - لن يتعلمه Le Sage مرة أخرى أبدًا. اتركه لوحده.
    ماذا تقترح؟
    ليس كحل، بل بشكل عام، ماذا تقترح؟ فكره؟ حسنًا، تريد مني أن أجيبك بأدب، حسنًا.
    لا توجهني إلى الروابط، أكتب في ردك لي ما هي فكرتك. وسوف أفكر في كيفية التعامل معك.

  799. لذلك عدنا إلى عيون الدراجات النارية، هل هذا هو المقصود؟
    حسنًا، السؤال ليس بهذه البساطة تمامًا، ولكل من يتعجل للتوصل إلى إجابات غامضة، اقترح إجراء التجربة التالية على مرحلتين، وهي تجربة ستساعد على الثبات في الأمر.
    وفيما يلي التجربة:-
    الخطوة الأولى:- إعطاء دراجة نارية. بارِع. منظمة العفو الدولية "الديك" تتحرك على الطريق من النقطة أ إلى النقطة ب. الاستنتاج: الدراجة النارية تصل إلى هدفها!
    الخطوة الثانية: - بالنظر إلى دراجة نارية من طراز Arley Davidson تتحرك مرة أخرى على الطريق من النقطة A إلى النقطة B مع وجود بركة من الزيت على الطريق (ربما تركها B.S.I.). الخلاصة - لن تصل.
    ماذا حدث هنا؟ ما الذي منع دراجة نارية هارلي ديفيدسون من الانهيار وعدم الوصول إلى هدفها؟
    لماذا جزيئاتك التي أصابتني؟ بارِع. جزر. هل سنفشل في هارلي؟
    أنت تدرك أن الجزيئات التي تطفو فوق الطريق ولا تتسخ بالزيت لن تعطي إجابة صحيحة هنا على طبق من فضة!
    في انتظار إجابتك الحكيمة
    شاب شالوم
    سابدارمش يهودا

  800. ليس بالون عيد ميلاد. اسطوانة غاز معدنية محكمة الغلق. لا يتوسع، لا يشع. ومع ذلك، منذ اللحظة التي تمتلك فيها الجزيئات طاقة أولية، فإنها ستظل في حالة حركة طوال الوقت دون مصدر خارجي للطاقة.
    ينبغي توجيه السؤال ب إلى ماكسويل.
    لدي إجابة على السؤال ج: لقد ذهب إلى كل الأرواح والشياطين.

  801. إسرائيل،
    ماذا عن الجيروسكوب؟ ماذا تريد من الجيروسكوب؟ وما هي مشكلة الاحتكاك الشهيرة التي تزعج LS كثيراً؟ هل سيكون للجماهير احتكاك مع الجزيئات؟

  802. إسرائيل،

    "إذا كنت تعرف الصراع الذي حدث في ذلك الوقت حول قابلية انعكاس الإنتروبيا (والذي بلغ ذروته بانتحار بولتزمان)، فإن نظريتي تقدم تفسيرًا رياضيًا لعدم رجعة الإنتروبيا مع الزمن."
    - إن تفسير سبب زيادة الإنتروبيا مع مرور الوقت موجود في إطار الديناميكا الحرارية الإحصائية. ربما لم أفهم وأنت تتحدث عن شيء آخر.

  803. اليوبيل,

    طلبك "كيف بدأ كل شيء؟" من الواضح أنه غير ذي صلة قبل أن تفهم ما هو "كل شيء". وعلى هذا السؤال - ما هو كل هذا، يحاول السادة إسرائيل وسفارداميش وعدد قليل من الشخصيات العلمية الأخرى الإجابة عليه. بعد أن نفهم في الله ما هو "كل شيء"، فربما نبدأ في محاولة فهم كيف بدأ أيضًا.
    إذا أصررت، يمكنك قراءة بعض الإجابات على السؤال "كيف بدأ كل شيء" باللغة البابلية/اليونانية/الهندية/الشمالية أو أي علامة تجارية أخرى. ما يجب فعله من وجهة نظر علمية ربما لا يكون هذا السؤال ذا صلة في الوقت الحالي وكذلك بما سبق الانفجار الأعظم.

    إسرائيل،
    1) فيما يتعلق بالبالون فمن الواضح أن البالون يتفاعل مع البيئة. تحدد البيئة حجمها بناءً على فرق الضغط الداخلي والخارجي. نفخ بالوناً ضع إشارة في حجرة مفرغة وسوف تتوسع وتتوسع وتنفجر أو إذا كان بالوناً معجزة غير قابل للانفجار فسوف يتوسع إلى حدود الغرفة. أخرجها من غواصة في قاع البحر وسوف تتقلص على الفور إلى نقطة صغيرة.
    وبالتالي فإن سرعة الجزيئات الموجودة في البالون يتم تحديدها بواسطة البيئة الخارجية.
    2) صحيح أنه في الهواء الراكد هناك حركة مستمرة للجزيئات، ولكن بما أنها عشوائية فإنها تقابلها وفي الماكرو يبدو أن لا شيء يتحرك. الرياح كما تقول هي ناقل آخر يتأكد من أن اتجاه واحد لديه حركة أكثر من الآخر. ومع ذلك، لكي يتم إنشاء المتجه المذكور أعلاه، يجب أن يكون هناك سبب ما، أي استثمار الطاقة. وفي الأرض كما ذكرنا هو دورانها وتغير درجات الحرارة والرطوبة وغيرها. ما الذي سيؤدي إلى تطوير جزيئات موقعك النشطة مثل هذا المتجه (الرياح) في الفضاء؟

    كاسبر ريف * ايم، ماذا حدث؟ هل حصلت على إجازة من المؤسسة؟ هل يعرفون أنك تعبث بالإنترنت مرة أخرى؟

  804. إلى إسرائيل
    حل زمن الاحتكاك لا يناسب الأشخاص الذين لا حياة في جزيئاتهم!
    اذهب إلى كل أرواح السماء، وارجع مقتنعًا بالروح القدس
    السبت شالوم كوديش
    سابدارمش يهودا

  805. هل لي أن أسألك سؤالا؟
    ما هي مشكلتك مع "الإصرار" الذي يبدو واضحًا بالنسبة لي ويزعجك كثيرًا؟
    شاب شالوم
    سابدارمش يهودا

  806. يهودا
    أحتاج إلى التحرك، لكن جزيئات La Sage ليست غازًا. لا توجد جاذبية في الغاز كما أشار R.H، ولكن نموذج LS نعم. الجاذبية. إشكالية، ولكن الجاذبية.
    مهما كان الأمر - في L.S. لا رياح
    وللمرة المائة: ماذا عن الاحتكاك؟؟

  807. طالب.

    إذا قرأت ما كتبته، فسيتم تقديم تفسير بديل للنسبية هناك. يكاد يكون من المؤكد أنه خطأ، لكني لا أجد فيه تناقضًا، وأريد أن يرشدني أحد إلى أين. فهو يحل المشكلة الموجودة في رأيي مع مسألة تمدد الزمن عند أينشتاين. فهو يسمح بالسرعة على أوريت، واختبار دحضه يكمن في التجربة التي أطلب القيام بها.
    إذا كنت على دراية بالصراع الذي احتدم في ذلك الوقت حول قابلية انعكاس الإنتروبيا (والذي بلغ ذروته بانتحار بولتزمان)، فإن نظريتي تقدم تفسيرًا رياضيًا لعدم رجعة الإنتروبيا مع الزمن.

  808. إسرائيل شابيرا
    إذن هذا كل شيء، لقد كان هاساج مخطئًا
    كان يتحدث فقط عن الجسيمات المتحركة التي تخلق الجاذبية. نقطة.
    אבל
    للمرة المئة
    إذا كانت هناك جسيمات تتحرك في الفضاء فهي غاز
    ومن هنا تأتي اختلافات الضغط والرياح
    وحقيقة أن هاساج تجاهلهم هي مشكلته وإغفاله.
    بالإضافة إلى ذلك، في عالمي البسيط، للغاز أيضًا خاصية أخرى يمتلكها كل غاز، وهي متوسط ​​المسار الحر لجزيئات الغاز.
    الاستنتاج الضروري من هذا هو أن الجاذبية تتلاشى في فضاء الكون بشكل أسرع من مربع المسافة
    ولذلك فإن الرياح فقط هي التي ستبقى نشطة على مسافات بعيدة.
    بهذه البساطة
    وكما قال الأميرال التركي وهو ينظر إلى المسافة:-
    نيوتن يوك!
    شاب شالوم
    سابدارمش يهودا

  809. ر.ح. عزيزي الشبح.

    "أحضر نموذجا واحدا يشمل الفيزياء كلها كقطعة واحدة ولا تختبئ وراء عبارة غامضة مثل "أنا لا أتحدث عن نموذج فلسفي". من فضلك، التالي. هذا لا يهمني.

    "ربما يفهم شخص ما ما تتحدث عنه، وقد أوضح لك بالفعل أين أنت مخطئ." ما الذي لا تفهمه؟ إذن ما الذي تتحدث عنه بالضبط؟

    "أنت تستمر في الحديث عن "عارضة أزياء"، لكن على الأقل بالنسبة لي شخصيًا، ليس من الواضح ما الذي تتحدث عنه على الإطلاق". "النموذج" هو ما يقوله يوفيلى. أحاول أن أتحدث عن "الأفكار". إذا لم تفهم، على الرغم من أن كل شيء مكتوب، اطرح الأسئلة بأدب، وسأكون سعيدًا بالإجابة.

    "وبدلاً من ذلك أجد رداً غبياً" لقد أخرجت الكلمات من فمي.

  810. يهودا، ر.ه.

    ألقيت نظرة ثانية على لاساجينو

    http://en.wikipedia.org/wiki/Le_Sage%27s_theory_of_gravitation

    الأمر لا يتعلق بالرياح.

    ويجب تمييز الرياح عن حركة الجزيئات، أو الجزيئات.
    في الهواء "الراكد"، تتحرك جزيئات الهواء بسرعة عالية. عندما تهب الرياح - على سبيل المثال 5 كم/ساعة - فإن سرعة الرياح هي ناقل آخر للحركة العشوائية للجزيئات. تستمر الجزيئات نفسها في التحرك بسرعات أعلى بكثير من 5 كم/ساعة.

    نموذج لو سيج يشبه الهواء الراكد تمامًا، حيث لا توجد رياح، ولكن بدون تصادمات بين الجزيئات، أو تصادمات على مسافات كبيرة جدًا.

    يوفلي ماكشين (هل أنت راضي عن تطور ألقابك؟)

    لماذا أريد أن أعرف من الذي خلق جزيئات الأثير النشطة؟ أو جزيئات LS؟ هل طرح ماكسويل هذا السؤال؟ لا ساز؟ من ماذا؟

    إسرائيل لا تطرح أسئلة وجودية. إن إسرائيل هي بيدق صغير في لعبة الحياة الكبرى. ترك حرفة صحية للقدير وهوكينج ويوبيل الفيلسوف.

    يذهب إلى أفوتا لفلفل الدماغ من أجل الدمامل. فقد كارلوس السيارة، وعليك أن ترسل راؤول فاين ليحضر سيارة خوسيه لكي تهرب.

  811. R.H. Rafai.M.، تشن تشن على بيرغون.
    إسرائيل وسفارداميش تفهمان: ليس لدي مشكلة مع موقع نشط أو مع شركة غازكوم. لا يهمني حقًا إذا كانوا موجودين أم لا. إن القول بوجود ظاهرة معينة أو ظاهرة أخرى تفسر شيئًا ما مع استنادها إلى ظاهرة فيزيائية أو حدس موجود لا يحل شيئًا.

  812. إسرائيل
    تعليق يوفال هاير الحكيم:
    "أحضر نموذجا واحدا يشمل الفيزياء كلها كقطعة واحدة ولا تختبئ وراء عبارة غامضة مثل "أنا لا أتحدث عن نموذج فلسفي".
    ربما يفهم شخص ما ما تتحدث عنه، وقد أوضح لك بالفعل أين أنت مخطئ.
    أنت تستمر في الحديث عن "النموذج"، لكن على الأقل بالنسبة لي شخصيًا، ليس من الواضح ما الذي تتحدث عنه على الإطلاق. (ولهذا السبب أحاول أيضًا عدم التدخل إلا في الحالات القليلة التي أدخل فيها هذا المقال لأقرأ تعليقًا قد أتعلم منه وبدلاً من ذلك أجد تعليقًا غبيًا)

  813. ولا يوجد مجال هنا لإسرائيل أيضًا. هو بدأ.
    في الواقع، أنا من دعوت كساح لأفكاري، وليس أنا شخصيا.

  814. إسرائيل،

    "كيف يمكنك استخدام النفق لإرسال إشارة أسرع من الضوء؟"
    - لا أعلم، ليس هذا ما فهمت أنك تريده. لكن على أية حال، إذا كنت تقصد المعلومات بكلمة فردية، فمن المحتمل أنك لا تستطيع ذلك. لا يمكن نقل المواد أو المعلومات بسرعة أكبر من سرعة الضوء.

    "ما هو التفسير لعدم رجعة الإنتروبيا غير الإحصائية؟"
    – يتم تعريف الإنتروبيا في الميكانيكا الإحصائية.

    "مثال البالون كان لـ R. H.، الذي سأل من أين تأتي الطاقة إلى النظام. من الناحية المثالية، يكون البالون معزولًا تمامًا وفي حالة توازن، ولا يحتاج إلى طاقة خارجية. وعلى الرغم من ذلك، لا يوجد انخفاض في سرعة الجزيئات داخل البالون، كما كان يخشى R. H.."
    - ربما لا أفهم المثال. أنت تصف بالونًا مغلقًا ومعزولًا تمامًا عن بيئته، أي أنه لا يتفاعل معه على الإطلاق، فما هو نوع البالون؟ وأكثر من ذلك، ليس من الواضح لماذا تقول "رغم هذا". هذا ليس على الرغم من أن هذا هو بالضبط السبب وراء عدم انخفاض سرعة الجزيئات الموجودة في البالون - لأن البالون لا يتفاعل مع بيئته.

  815. موقع نشط أو Gazcom، كله كلام وألعاب وأفعال. لنفترض أن هناك أثيرًا نشطًا، فمن أين يأتي؟ ولنفترض أن جزيئات الجاسكوم عبارة عن عضلات وموجودة، من الذي خلقها؟ هذه النماذج ليست أفضل من نظرية الانفجار الأعظم أو نظرية الأوتار، لأن أيا منها لا يحل السؤال الأساسي: "كيف بدأ كل شيء؟". أليس من الأفضل في هذه المرحلة البناء على الإصحاح الأول من سفر التكوين؟
    لقد طرحت إسرائيل سؤالاً جيداً حول المثابرة. يأخذ سابدارمش المثابرة كأمر طبيعي ويبني عليها جزيئات الغازكوم. لكن أخبرنا، من فضلك، هل تمنحنا جزيئات الغازكوم الآلية التي تخلق الثبات؟ والموقع النشط ما الذي يمنحه حركة أنا وآنا؟
    أرجو أن تعودوا إلى رشدكم وتخرجوا بنموذج واحد يشمل الفيزياء كلها كقطعة واحدة ولا تختبئوا وراء عبارة غامضة مثل "أنا لا أتحدث عن نموذج فلسفي".

  816. إسرائيل! لقد فتحت صندوق فيدورا وذهبت فوق Yaharg دون المرور بالرقم واحد: لا توجد افتتاحية لـ Jura على R.H. Refai.M! والباقي هو مسؤوليتك الكاملة.
    إذا كان لدي كلب سأسميه ميتزي. لكنني في الواقع أخطط لاقتناء قطة سيكون اسمها شيلبا في جيرسي. ما هو "الكلب"؟ أليس هذا عازف البيانو بالألمانية؟

  817. إسرائيل،
    شكرا على الإجابات. أحتاج إلى التفكير قليلاً في الأشياء التي كتبتها.

    هناك شيء واحد فقط يعني أنني أتفق مع يهودا في أنه "نعم، إذا قلت حكيم، فبالإضافة إلى الجاذبية الجزيئية، قلت أيضًا دفع الأرواح، وهما شيئان مختلفان مرتبطان ببعضهما البعض وينبعان من ظاهرة الجسيمات."

    وليس كما تقول." ريح. وبقدر ما أعرف، فهو غير موجود في لا سيج، بل في يهودا فقط".

    بعد كل شيء، ما هو فرق الضغط إن لم يكن إحصائيًا أن عدد الجزيئات التي تصطدم بأحد الجانبين أكثر من الجانب الآخر، ثم يحدث دفع أو ضغط نحو الجانب الذي يحتوي على عدد أقل من الجزيئات؟ باختصار الروح.

  818. إلى إسرائيل
    لماذا تقول :-
    . ريح. وهي غير موجودة في لا سيج على حد علمي، بل في يهودا فقط. مثير للاهتمام، ولكن من غير المرجح في رأيي. نهاية الاقتباس.
    بعد كل شيء، لا سيج وجزيئاته التي تتحرك من مكان إلى آخر هي غازات. هذا هو تعريف الغاز
    لذلك إذا قلت La Sage فقد قلت بالإضافة إلى جاذبية الجزيئات أيضًا دفع الأرواح
    شيئان مختلفان مرتبطان ببعضهما البعض وينبعان من ظاهرة الجسيمات.
    لا يمكن أن يكون غير ذلك!! يجب أن تكون مقتنعًا بهذا بالفعل لأنه لا يحتاج حتى إلى دليل.
    شاب شالوم
    سابدارمش يهودا

  819. طالب.
    شكرا على الروابط.
    1. كيف يمكنك استخدام النفق لإرسال إشارة أسرع من الضوء؟
    2. ما هو التفسير لعدم رجعة الإنتروبيا غير الإحصائية؟
    3. مثال البالون كان لـ R.H.، الذي سأل من أين تأتي الطاقة إلى النظام. من الناحية المثالية، يكون البالون معزولًا تمامًا وفي حالة توازن، ولا يحتاج إلى طاقة خارجية. وعلى الرغم من ذلك، لا يوجد انخفاض في سرعة الجزيئات داخل البالون، كما كان يخشى R.H.

    إذا كنت لا تؤمن بالجسيمات أو الإشعاع، فما الذي يثبت الجيروسكوب في الفضاء؟

  820. ر.ه.رفاعي.م

    أشكر لك حقا. كنت أفكر في نفسي: أتساءل ما الذي يفكر به ر.ح.رفاعي المجنون في نفسه؟ ما هو التنوير الذي يمكن أن نتلقاه من النجم العظيم في نظرية المنطق، الذي لم يأت قط بفكرة أصلية واحدة وشغله كله هو القتل؟

    بيننا، هل تفهم حتى موضوع النقاش هنا؟ هل قرأت أي شيء هل لديك أي تعليقات حول هذا الموضوع؟
    اذهب للمنزل يا فتى، توقف عن إزعاج الكبار.

  821. إسرائيل
    عندما تتحدث عن نموذجك فإنك تبدو مثل يهودا سابدارمش عندما يتحدث عن نموذجه.
    وعندما يتحدث يهودا عن نموذجه فإنه يبدو مثل حنين زعبي بعد قراءة تقرير غولدستون على متن سفينة مرمرة في يوم طوف.

    على أية حال، ما رأيك بإرسال نموذجك ليراجعه زملائك الذين يعيشون في مكان غير بعيد عنك في نفس القارة، اتركنا وحدنا لفترة من الوقت، اذهب للتنزه في الجبال مع كلبي وهايدي فتاة الجبل، و متى يحين الوقت لتنزل إلينا من الجبال ومعك إجابة من زملائك عن نموذجك؟

  822. "إذا أخذت بالونًا محكم الإغلاق ومملوءًا بالهواء إلى الفضاء، حيث تبلغ درجة الحرارة حوالي 300 كلفن، فسوف ترى أنه حتى بعد مرور عدة ساعات، ستنخفض درجة الحرارة قليلاً فقط، لأن سرعة جزيئات الهواء داخل البالون تقترب من 400 م/ث بالنسبة لنظام السكون وهو البالون. هذا هو التوازن الذي أتحدث عنه. إذا انفجر البالون الآن، فإن الجزيئات ستبتعد بسرعة عن بعضها البعض، ومعها سينخفض ​​الضغط".
    - لا بد من التمييز بين وصف التقلبات في درجات الحرارة في حالة S.M وبين الانخفاض الفعلي فيها. إذا انخفضت درجة الحرارة بمرور الوقت، فهذا يعني أن النظام لا يعمل. ويجب تحديد المشكلة - ما هي التفاعلات المحتملة بين نظام البالون والبيئة (الفضاء) وما هي التفاعلات بعد الانفجار؟ بمعنى، هل يستطيع البالون تبادل الطاقة و/أو الجسيمات مع الفضاء؟ هل يمكن أن يتقلص/يمتد؟

  823. إسرائيل،

    لست متأكدا من أنني فهمت. كيف يختلف عن نفق الجسيم الموجود في بئر محتمل ويتم وصفه بواسطة دالة موجية ثابتة؟

    فيما يتعلق بالانتروبيا - يوجد بالفعل تفسير لذلك.

  824. ر.ح. القنفذ

    معك آمل أن نتمكن من التسكع والكتابة. دعونا نأخذ نقاطك واحدة تلو الأخرى.

    1. "التوازن. ما هو التوازن في الهواء؟" إذا أخذت بالونًا محكم الغلق ومفضضًا مملوءًا بالهواء إلى الفضاء، حيث تبلغ درجة الحرارة حوالي 300 كلفن، فسوف ترى أنه حتى بعد مرور عدة ساعات، ستنخفض درجة الحرارة قليلاً فقط، لأن سرعة جزيئات الهواء داخل البالون تقترب من 400 كلفن. م/ث نسبة إلى نظام السكون الخاص بهم، وهو البالون. هذا هو التوازن الذي أتحدث عنه. إذا انفجر البالون الآن، فإن الجزيئات ستبتعد بسرعة عن بعضها البعض، ومعها سينخفض ​​الضغط.

    نموذج الانفجار الكبير يشبه إلى حد كبير البالون. أنا لا أسأل "من أين تأتي طاقة توسع الكون"؟ أنا فقط أقول أنه إذا كان الأمر كذلك، فإن نموذجي يحل مشكلة ثبات سرعة الضوء في جميع الأنظمة المرجعية، وكذلك مشكلة الاحتكاك في نموذج La Sage.

    سيكون اللون في زجاجك موحدًا، لكن هذا لا يعني أن جزيئات اللون توقفت عن الطيران بسرعة عالية في جميع الاتجاهات. إنهم يصطدمون ببعضهم البعض، لذلك ينتشرون ببطء في البداية. الطاقة - من جزيئات الماء الموجودة في الزجاج.

    ولكن لماذا تأتي لي مع الأسئلة؟ هذا هو نموذج ماكسويل. نفس المشاكل موجودة معه، ورغم هذا فهو نموذج ناجح إلى حد ما، إذا تمكن من استخراج المعادلة 136 منه، سرعة الضوء، أليس كذلك؟ لقد أضفت للتو التغيير اللازم في رأيي، وهو أنه لا يوجد نظام راحة للموقع باللون الأرجواني، ومن هنا جاء اسم الموقع النشط.

    2. لا سيج. لم تصطدم الكرات الخاصة بك ببعضها البعض لأنه في الهواء هناك تصادمات بين الجزيئات. في حالة Le Sage لا يوجد أي شيء، أو لا شيء تقريبًا، وفي حالة Yehuda يكون متوسط ​​المسافة التي تقطعها الجزيئات بين الاصطدامات كبيرًا جدًا لدرجة أن النموذج يسمح بالتجاذب. لا يوجد خلاف (على ما أعتقد) بين العلماء على أن الجذب سيكون موجودًا بالفعل في نموذج لا سيج - فالمشاكل لا تختلف، وأولها وقبل كل شيء مشكلة الاحتكاك.

    نموذجي يحل مشكلة الاحتكاك. إذا سألت، سأخبرك.

    3. غير محلية. لم يزعم نموذجي أبدًا أنه يحل المشكلة. إنه يتيح فقط جدوى الحل. سنناقش إذا كنت تصر.

    4. القياس. في الأساس، تجربة. إذا كان هناك أي شيء في هراءي، فهو أن الوقت مطلق. ويمكن قياسها في أي نقطة بعدد معين من الجزيئات لكل وحدة حجم. ليس من الممكن العودة بالزمن إلى الوراء، لأنه في كل نقطة نعود إليها في المستقبل سيكون هناك عدد أقل من الجسيمات لكل وحدة حجم مما كانت عليه في الماضي.
    لكن هذا يتعارض مع تمدد الزمن عند أينشتاين. لذلك - وإذا كنت تريد مني أن أشرح بالتفصيل - فإن اقتراحي هو أن الضوء يتحرك فعليًا بسرعات متعددة، ولكننا كمراقبين لا يمكننا قياس سوى واحدة: C. لا يمكننا قياس الفوتونات الأسرع، وهي شفافة بالنسبة لنا، أو محايدة، أو باختصار: النيوترينوات. ومع ذلك، إذا كان هناك في نفس النقطة التي نقيس فيها نظامًا آخر يتحرك بسرعة عالية بالنسبة إلينا، حتى أصغر بكثير من C، فسيكون قادرًا على قياس تلك الفوتونات الأسرع، حتى قبل إخوانها الأبطأ الذين يتحركون بسرعة ضوء فقط.

    ولذلك، فإن الحاجة إلى توقيت دقيق، ومن هنا الحاجة الملحة: نحن بحاجة للحصول على الجماعة في ساران.

    5. الهيكل الديناميكي الهوائي. لا، انها ليست ذات صلة. سيتم دفع أي كتلة. اقرأ مدخل Wiki الخاص بـ Le Sage، أو إذا كنت تريد، فسوف أشرح ذلك بالتفصيل.

    6. الروح. وهي غير موجودة في لا سيج على حد علمي، بل في يهودا فقط. مثير للاهتمام، ولكن من غير المرجح في رأيي.

    7. سبب الريح. أو للسحب حسب L.S. فرق الضغط طبعا كما ذكرنا، نحن لا نسأل من أين جاءت الطاقة البدائية، أو كما فعل كانط، مما تتكون الجسيمات التي تتكون منها. يقول LS فقط أنه إذا وضعت العديد من الجزيئات السريعة في الخلية التي لا تصطدم ببعضها البعض تقريبًا، وأضفت كتلتين كبيرتين من البلاستيسين إلى الخلية، مما سيحجب الجزيئات عن بعضها البعض، فسيتم إنشاء تجاذب بين البلاستيسين مع القوة معكوسة للمسافة المربعة ربما يكون على حق. ولا يعمل بوجود شراعين في الغرفة، لأن ضغط الهواء بين الأشرعة هو نفس الضغط الخارجي، نتيجة تصادم جزيئات الهواء. ستعمل إذا كنت تستخدم نفاثات المياه.

    واليوم بالمناسبة أخذت كلابي إلى البحيرة، لذلك طلبت منك أن تأخذ مهاراتي في الإبحار على محمل الجد.

  825. طالب.
    لم نصل حتى إلى نظريتي لأنه كان يتحدث معظم الوقت. ما أردت معرفته هو ما إذا كان من الممكن التسبب في انهيار الدالة الموجية للجسيمات السريعة في نقطة لا يفترض أن تكون فيها، بسبب حد سرعة الضوء، وذلك عن طريق اصطدامها بجسيمات سريعة أخرى. وادعى أنه لا يعتقد أن ذلك ممكن، لكنه لم يتمكن من تسمية تجربة محددة. ووعد بإرسال روابط لمقالات أو يوتيوب، لكني لم أتلقها حتى الآن.

    بالمناسبة، إذا تعمقت في نموذجي، فسوف ترى أنه يمكن أن يفسر بشكل جيد سبب عدم إمكانية عكس الإنتروبيا.

    يوفلي المتملق.

    من هذا؟ أستثمر وأعرق، وفي النهاية يحصد يهودا كل الثناء؟ انظر إلى نفسك وكأنك مدعو للنظام.
    هناك الكثير من الكتب في إسرائيل، بما في ذلك كتب الأطفال. أكتفي بقصص ما قبل النوم للأطفال، وأغاني لزوجتي في عيد ميلادها وذكرى زواجها. علاوة على ذلك، لا يبدو لي أن مستوى كتابتي يمكن أن ينفي لقمة العيش من مئير شالوف وألترمان.
    لكن من باب الصدق والغرور، فإن الأشياء التي كتبتها لم تجد سوى القليل من الضوء، حتى في "جاليليو"، عندما كانت لا تزال تصدر شهريًا. إذا كنت تريد، سأرسل لك الرابط.

    إلتواء. إذا بدأنا من فرضية أنه لا يوجد شيء أكثر فظاعة وفظاعة من اللامكانية، فأنا آمرك، كفارس، أن تشرح على الفور كيف يتم ذلك في نموذجك. على أية حال، سوف يسهل الأمر علينا جميعًا، الآن بعد أن استعدت المجلدات، وأن تقوم بنشر كل شيء وسننتهي من ذلك.

    شبل الأسد.
    أنام ​​بشكل أفضل في الليل الآن بعد أن قمت بحل مشكلة الاحتكاك في LS. ربما ستوافق على المشاركة معنا بعد كل شيء؟ لا تقلق، لن يسرق أحد فكرتك. يجب عليكم أيضًا، قبل نشر حل لمشكلة شغلت كل العلماء تقريبًا منذ نيوتن، قبل نشرها في مجلة Science and Nature، أن تعرضوها لمراجعة النظراء، أليس هذا صحيحًا أيها الإخوة؟

    ر.ح.
    سألعب بعض الشطرنج. انتظر، انتظر، عندما أعود سنقوم بتسوية الحساب معك.

  826. إسرائيل،

    ملحق للرسالة السابقة:
    وبما أنك مهتم بالعلم وترغب في تعلمه قبل إعادة اختراعه، اسمح لي أن أوصيك بموقع (أفترض أنك لست على دراية به) مركز المعرفة الهائلة باللغة العبرية حسب الدورات الأكاديمية: http://www.hapetek.co.il/?page_id=41
    يمكنك العثور على الكثير من المعلومات الموثوقة هناك حول الموضوعات التي أرى أنك مهتم بها - ما عليك سوى اختيار الدورة التدريبية المناسبة.

  827. يوفال المرتبك
    إذا كانت الأعمال الخيالية موجهة إليّ، فقد كان لي بالفعل شرف إنشاء الفيلم "البحثي" الذي تناول خالق العالم.
    وإلى جانب ذلك، هناك أعمال أخرى لندف
    لمعلومات المهتمين
    ليلة سعيدة للمرة الثالثة
    سابدارمش يهودا

  828. ر.ح.
    أنت قلت :
    "الرياح على الأرض شيء متغير وفوضوي. "رياح" جسيمات لازاج/سفاردميش ثابتة وموجهة وتؤثر على كل نظام شمسي وكل مجرة ​​وكل جسم على الأرض بنفس الطريقة. من المنطقي؟". نهاية الاقتباس.
    حسنًا، يا سيد ر.ه.، لماذا يجب أن تتحرك الرياح على الأرض في أي اتجاه وبأي قوة، بينما في الكون كله تجبر رياح الغازوم على التحرك بشكل مستقيم مثل المسطرة وبدون انحرافات؟
    لا معنى له على الإطلاق!
    كل التوفيق لها!
    باتجاه أندروميدا 600 كم في الثانية
    حول مجرة ​​درب التبانة 230 كم في الثانية
    إلخ
    مساء الخير
    والرياح الخفيفة الليلة
    سابدارمش يهودا

  829. لقد عدت للتو من الجمعية الفلكية
    صحبة لا تحتار، وأنت في حيرة من أمرك في تعريف الكون البسيط!
    هذا هو:
    هناك الجاذبية التي تنشأ عن حركة جزيئات لا ساز. بمعنى آخر، قياسًا على مثال الهواء، فإن حركة الجزيئات يجب أن تخلق الجاذبية. هذا يحتاج إلى إثبات ولدي دليل (للأفضل أو للأسوأ).
    وبصرف النظر عن ذلك هناك حركة الغاز نفسه، وهي الرياح، التي تحرك الأجسام الكبيرة مثل المجرات وعناقيد المجرات. لا تحتاج إلى جاذبية نيوتن أو لا سيج لذلك!
    وهذا يحتاج أيضا إلى إثبات.
    سيتم تجاهل حركة الاحتكاك.
    الاحتكاك علي!
    (إما نعم أو لا (:))
    مساء الخير
    سابدارمش يهودا

  830. آر إتش،
    يبدو لي أن إسرائيل، وكذلك يهوذا، ليس لديهم مشناه منظم (من أجل الإنصاف، سأعترف أن مشناه الخاص بي يفتقر أيضًا إلى الرابط). لكني متحمس جدًا لكتاباته. ليس من الواضح بالنسبة لي كيف أنه لم ينشر بعد رواية أو كتاب أطفال على الأقل 😉

  831. المزيد من النقاط عنه لـ Sage و Gazkom -
    1) لتتمكن الجسيمات من الدفع، تحتاج إلى هيكل ديناميكي هوائي كما ترى في الجناح أو الشراع أو حتى فيما يتعلق "بالرياح الشمسية" وخطط استخدامها للدفع. وبالتالي فإن الشكل ليس ديناميكيًا هوائيًا ولن يتحرك بكفاءة. في الجاذبية نتحدث عن كل كتلة فيما يتعلق بشكلها أو تركيبها. جاليليو؟

    2) الرياح على الأرض شيء متغير وفوضوي. "رياح" جسيمات لازاج/سفاردميش ثابتة وموجهة وتؤثر على كل نظام شمسي وكل مجرة ​​وكل جسم على الأرض بنفس الطريقة. من المنطقي؟

    3) ومرة ​​أخرى نفس السؤال، ما الذي يحرك "الروح" المذكورة أعلاه؟ لماذا يتم دفع كتلتين معًا إذا كان للرياح اتجاه واحد؟ حاول وضع شراعين في أي اتجاه مع الريح، فبدلاً من التحرك في نفس الاتجاه سوف يصطدمان ببعضهما البعض. لسبب ما، لا يبدو لي أن حكيم يهودا أو رجل الجبال مثلك قوي في البحار.

  832. إسرائيل،

    سوف تتفاجأ، لكنني نظرت إلى الرابط الموجود في المعادلة 136 والمناطق المحيطة بها. ونعم، أعرف إلى حد الإرهاق (سأعترف ولن أخجل أساسًا من العالم) نموذج ليساج والكون البسيط لعالمنا.

    فيما يتعلق بالنموذج الخاص بك:
    1) أنت تقول "التوازن". ما هو التوازن في الهواء؟". ويتحرك الهواء لأن الأرض تتحرك، ولأن الشمس تسخن وفي الليل يبرد الهواء، فتصعد الغازات من الأرض وهكذا. هل تقترح أن الكون بأكمله يدور وبالتالي فإن موقع الويب الخاص بك يتحرك؟ (بالمناسبة، أنت بحاجة إلى العثور على اسم عادي له لأنه يُسمى موقع ويب في كل مرة - مثل أي مكان وموقع ويب، مثل بعض مواقع الويب على الإنترنت). وإلا فإنه من المستحيل أن نفهم لماذا لا يوجد توازن وما الذي يدفع جزيئات الأثير إلى سرعات لا نهائية؟ خذ أي نظام مغلق، كوب من هذا الحبر بالتنقيط وبسرعة كبيرة سيكون اللون موحدًا في حالة توازن. ولا تقل لي أن هذه صفاتهم، إنها التلويح بالأيدي، من أين تأتي الطاقة؟؟؟
    2) فيما يتعلق بالكرات التي ستركض نحو بعضها البعض (وهذا ينطبق على ليساج وغازكوم)، هل ستركض كرتان إلى بعضهما البعض في غرفة بها غاز في حالة توازن، والهواء في هذا الصدد؟ لقد قمت للتو بالتجربة باستخدام كرتين مطاطيتين على مكتبي وأتساءل وأتساءل أنهما لم تتحركا.
    3) كيف يفسر نموذجك عدم وجود محلية؟ إن القول بأن هناك جسيمات تتحرك بسرعة لا نهائية هو بمثابة التلويح باليد. إنه مثل القول أن هناك أشياء لا نفهمها. ولا يزال السؤال قائما، حتى لو كانت هناك جسيمات سريعة لا متناهية، كيف "تعرف" كيف تنتقل من إلكترون متشابك تماما إلى آخر على مسافة سنة ضوئية؟ يمكنني أيضًا أن أزعم أن الفضاء مطوي بحيث يكون الإلكترونان المتشابكان في الواقع واحدًا في بُعد آخر. مثل رسم نقطتين على صفحة تبدو متباعدة وطيها في البعد الثالث بحيث تلامس بعضها البعض.

    4) أنا مندهش، كيف تعتقد أن القياس الدقيق جدًا سيؤكد نموذجك؟

    إذا كنت بحاجة إلى مزيد من الوقت في الجبال فلا تتردد

  833. إسرائيل،

    هل طلبت منه فكرة عن كيفية إجراء التجربة التي تريدها؟ عن نظريتك؟ إن الأفكار والانتقادات التي يمكنني تقديمها لك لا تقارن بما يمكن أن يفعله أحد كبار الفيزيائيين في مختبر الجسيمات. سيستغرق العثور على أخطائي أسبوعًا (إن وجد)، وسيجدها على الفور.
    على أية حال، لم أتعمق في المشاركات القليلة الماضية، لأنه ليس لدي الوقت للقراءة/الرد بشكل صحيح.

    بالمناسبة قصة كوميدية ممتعة: http://abstrusegoose.com/272

  834. شكرا. الآن أنا أفهم. في الواقع، نموذجي يشرح التشابك الكمي، لكنني لم ألاحظ ذلك. ليست هذه هي المستقطبات التي تؤثر على البيئة، بل البيئة هي التي يتم تنظيمها مسبقًا.
    أنا لست غير صبور لأن الوقت ضيق بين يدي اليوم. وسأعود قريبا إن شاء الله. وشكرا مرة أخرى.

  835. طالب.
    إنها ليست فكرة سيئة، إذا وصلت إلى المرحلة التي أستطيع أن أخبرهم فيها بالضبط بما هو مطلوب. التقيت قبل بضعة أسابيع مع أحد كبار السن في مختبر الجسيمات في جامعة كاليفورنيا، ولكن لم يأتِ الكثير مما يعجبني، والأكثر من ذلك، على الرغم من السرعة العالية للجسيمات في المسرعات، يبدو لي أنني سأدخل في هذا الموضوع. مشكلة بسرعة كبيرة مع التأثيرات الكمومية. من الأفضل بالنسبة لي قياس موجات الراديو بمؤقت دقيق. أيضا الكثير من الملعقة. لا تزال تبحث عن أفكار حول كيفية القيام بالتجربة.
    إذا صادف أنك قرأت ما كتبته، فهل وجدت أي خطأ جوهري في الحجة؟

    اليوبيل سريع الغضب.

    وسأحاول أن أكون مختصرا في أقسامي حتى لا يتبادر إلى ذهني القسم:

    الكتلة، الطاقة، إيه؟
    1. الجاذبية: "يبدو كما لو أنهم منجذبون لبعضهم البعض" لا يبدو الأمر كذلك. حتى أنهم يصطدمون في النهاية. هل هذا أيضًا مرئي فقط؟
    2. هناك طلب، هناك.
    3. ثم اشرح. ومع ذلك، هل تدعي أن الموقع لديه نظام معين للراحة؟ لماذا هذا بالضبط؟
    4. تحتوي كل مكتبة على كتب عن النسيج. هناك أيضًا محاضرات جامعة ستانفورد الممتازة:
    http://www.youtube.com/watch?v=0Eeuqh9QfNI
    انتبه للمحاضرة 5 دقيقة 27. جملة بيل. وما قد يهمك: 1:12 حيث يُذكر صراحة أن الدليل رياضي. لقد جاءت التجربة الجانبية فقط لإظهار ما كان واضحًا حتى بدونه.

    إذا كنت مهتمًا، فيمكنني أيضًا أن أحاول التوضيح، ولكن قد يستغرق الأمر نصف صفحة كاملة، لذا فكر فيما إذا كنت ستلتزم به.

    5. إذا كان بإمكانك نشر النموذج الخاص بك من البداية إلى النهاية وليس في أجزاء، فربما يمكننا معرفة ما هو عليه.

    بالمناسبة هل قرأت ما كتبته؟ صحيح، لقد تم الأمر كما طلب ر.إ.ه، ببطء وحذر. أي تحفظات؟ اي احد؟ يهودا؟ هل أنت لا تزال هناك؟
    أو ربما يكون النموذج مثاليًا وهذا كل شيء؟
    هيا يا كاسا! (ليس أنا، النموذج).

  836. من عجائب الثعلب الناري وموزيلا: قبل يومين أو ثلاثة أيام، وأنا أكتب السطور التالية، عطل الفيروس متصفحي. لقد قمت باستعادة الأشياء على جهاز كمبيوتر آخر، وكانت عملية الاستعادة ناجحة جدًا لدرجة أنه حتى الأشياء التي فقدتها أثناء كتابتها تم استعادتها على الرغم من عدم إرسالها. على الرغم من أن معظم الأشياء لم تعد ذات صلة لأنك تتقيأها من رأسك وترسلها إلى هنا، والمقدمة مُغرية وغير ذات صلة. لكن من باب الفضول ومن أجل الثناء والثناء والعلاقات العامة لموزيلا هم كذلك. وبالمناسبة، هذا لا يعني أنني عدت إلى النشاط الكامل. طريق التعافي لا يزال طويلاً، والحرب ضد الفيروس لا تزال طويلة (من كتبها؟):

    إسرائيل ضمانة نومك؟ أنا غارق في بحر الفيروسات وأنت كالعادة تكتب مخطوطات. أنت فقط تحب الكتابة، بشكل جميل وكثير، وفي الغالب تحب نفسك. هل أنت متأكد من أنك تمارس المهنة/الهواية المناسبة؟

    لا تريد أن تكتب نموذجًا لاهوتيًا أو فلسفيًا، ولكن هناك بعض الأسئلة التي يجب عليك طرحها قبل طرح أسئلتك. على سبيل المثال، ما هي الكتلة وما هي الطاقة؟ لدينا معادلات توضح الكتلة كتحول للطاقة والعكس، لكن هذه المعادلات لا تؤدي إلا إلى ادعاءات دائرية. ويجب أن تأتي هذه التعريفات من شيء خارجي عنها، ويفضل أن تكون بسيطة. النموذج الخاص بي يفعل.
    وبالنسبة لصعوباتك باختصار:

    1. الجاذبية - شرحت من قبلي. بدلاً من إرسالك إلى الرابط، يرجى قبول النسخ واللصق:
    الجاذبية ليست خاصية للجسيم ولكنها ظاهرة يتم ملاحظتها من مجموعة من الجسيمات. فكر في البروتون باعتباره "مستعمرة" من الجسيمات. تتركها الجزيئات بشكل عشوائي وتنضم إليها جزيئات أخرى. عدد الجسيمات التي تسكن البروتون في لحظة معينة هو دالة للكثافة الإقليمية الإجمالية. الآن فكر في بروتونين. تتركهم الجزيئات وينضم آخرون. يتم تدمير البروتونين وتراكمهما بشكل مستمر، لكن أثناء التدمير وإعادة البناء يقتربان من بعضهما البعض، لأن في المنطقة بينهما تركيز أكبر للجزيئات من المنطقة خارجهما. ويبدو الأمر وكأنهم ينجذبون لبعضهم البعض، وهذه الميزة نسميها "الجاذبية". י

    2. الجمود - وأوضح حتى أوضح. سيتم توفير وصف تفصيلي عند الطلب.

    3. ثبات سرعة الضوء في أي نظام مرجعي – وهذا خطأ ناتج عن سوء فهم نتائج تجربة مايكلسون مورلي. وهذا ما أوضحه لي أيضًا.

    4. غير محلية - ما زلت غير مقتنع بأن تجربة الأقطاب لم تؤثر على وسط التوصيل. أنت تقول "لقد تحدثنا عن ذلك بالفعل"، ولكن الذي تحدث عنه هو أنت فقط. وكما ذكرنا فإن صحة هذه الظاهرة تشير لنموذجي إلى أقل الطرق مباشرة، وحتى لو كان محاربتي لها ظالمة فهي مفهومة.

    5. الكتلة المظلمة والطاقة – نموذجي من البداية إلى النهاية هو من الجسيمات التي أعزوها حاليًا إلى الكتلة المظلمة. إن الإيمان بالطاقة المظلمة ينبع من سوء فهم لسبب وجود خطوط امتصاص الطيف الخاصة. وفقًا لنموذجي، تزداد سرعة الضوء كلما اقترب منا، وهذا يخلق تأثيرًا يحب دوبلر. بسبب الفهم الخاطئ بأن سرعة الضوء ثابتة في كل نظام مرجعي (مشكلتك 3)، يتم قبول التفسير القائل بأنه تأثير دوبلر فقط.

  837. حسنًا، لقد عدت من المشي مع زوجتي والكلب يا عزيزتي.

    في الطريق تحدثنا قليلاً عن الفيزياء. كانت المرأة مهتمة جدًا بنظرية ماكسويل، خاصة بالدوامات الموجودة على السطح، والتي ذكّرتها بالدوامات التي يصنعها الصبي في الحمام مع كلابي. كان لابي أكثر اهتمامًا بالكميات، وخاصة عدم اليقين وقطة شرودنغر، وأراد أن يعرف ما إذا كان مذاقها أفضل حيًا أم ميتًا.

    لقد قمت بتحليل التسلسل الهرمي للعائلة في بيت شابيرا وتوصلت إلى استنتاجات حزينة مفادها: في الأساس، أنا مهتم بشكل أساسي بالمرأة، التي في الواقع مهتمة بشكل أساسي بالطفل، والتي في الواقع مهتمة بشكل أساسي بأطفالي، والتي في الواقع مهتمة بشكل أساسي بأطفالي. الصبي.

    RH، إذا ألقيت نظرة على الرابط (لا بأس، وأنا أعلم أنه ليس كذلك)، فسترى أن سؤالك الأول ينطبق أيضًا على نموذج موقع Maxwell. ومع ذلك، هذا نموذج ممتاز. فهو لا يسأل من أين تأتي طاقة جزيئات الأثير، وما هي القوة التي تحركها، ومن أين أتت، وما هي وجهة نظرها السياسية. إنه يقول ببساطة أنه إذا كانت موجودة، ولها الخصائص التي وصفها، فيمكننا الحصول على موجات على الموقع بسرعة تعادل سرعة الضوء. في زمن ماكسويل، لم يكن أحد يعرف شيئًا عن الراديو، وشكك الكثيرون في هذه الفكرة التي بدت وكأنها وهم. فقط بعد تجربة هيرتز تم إثبات النظرية.

    لا أعرف ما إذا كانت سرعة جزيئات الأثير ستنخفض - فاللانهاية الأقل ثباتًا لا تزال لا نهاية - ولكن هناك نظريات حتى في الاتجاه السائد تدعي بشكل منطقي جدًا أن سرعة الضوء كانت أعلى في الماضي.

    توازن. ما هو التوازن في الهواء؟ وهناك أيضًا تطير الجزيئات في كل الاتجاهات ولا تسترخي. إذا كنت تقصد التوازن في أي نظام راحة، فأي نظام؟ لماذا هذا واحد؟ وماذا عن مبدأ التجانس في كل اتجاه؟

    استراحة. نعم، في النهاية سوف ترتاح. الانتروبيا القصوى كل نظرية تتحدث عنها.

    2. التشابك. (أفترض أنك تقصد غير المنطقة). لم أزعم أبدًا أن لدي تفسيرًا جيدًا لعدم المحلية. لقد زعمت أن النموذج أعلاه يسمح بإمكانية عدم المحلية، وهو يفعل ذلك. لدي فكرة، ولكن ذلك في مرحلة لاحقة.

    3. الجاذبية. هل تعرف نموذج لا سيج؟ تمت مناقشة المشكلة كثيرًا هنا على الموقع. يسمح النموذج بالجاذبية وفقًا لوصفي، لكنه يترك مشاكل مختلفة مفتوحة، وفي المقام الأول احتكاك الجسيمات مع الكواكب المتحركة.
    هذه المشكلة غير موجودة في نموذجي، لأن الكواكب لا تتحرك أبدًا، فهي دائمًا في حالة راحة بالنسبة إلى الموقع النشط. إذا كنت ترغب في ذلك، فسوف نقوم بتوسيع هذا لاحقًا من خلال تحليل مثال الطريق السريع، وتقليص الفكرة إلى بُعد واحد.

    وأيضاً، بحسب لو سيج، إذا وضعت كرتين في غرفة بها "غاز" كما تسميه، أو "جازكوم" كما يسميها يهودا، ولكن دون اصطدام كما هو الحال مع الغاز العادي، فسوف تركضان تجاه بعضهما البعض.
    سيكون يهودا أيضًا قادرًا على تفسير سبب كون متوسط ​​سرعة جزيئات الأثير أعلى بحوالي 40 بالمائة من سرعة الضوء.

    على أية حال، فإن مشكلة الجاذبية لا تهمني كثيرًا، ويعاني نموذج La Sage من مشاكل أخرى أيضًا. لقد أشرت للتو إلى أن حل مشكلة ثبات سرعة الضوء في جميع الأنظمة المرجعية يحل أيضًا مشكلة الاحتكاك في Le Sage. أنا لا أحاول بناء مجموعة طلعت مصطفى. أحاول أن أفعل شيئًا آخر، أكثر إلحاحًا وملموسًا: بناء جهاز قادر على إرسال إشارة أسرع من الضوء. هذا من واجبي. وظيفتك - مهمة الجميع حقًا - هي سحق والدة نموذجي، ووضع بعض المنطق في رأسي الغبي، وشرح لي ما هو الخطأ في أفكاري الوهمية، قبل أن أضيع الوقت والمال على أجهزة عديمة الفائدة.

  838. اليوبيل.
    فيما يتعلق بالفيروسات، اتصل بـ R.H. أجهزة الكمبيوتر لمايكل.
    على أية حال، في مسائل الخبث، اتصل بي.
    استمتع بالاستراحة، لكن لا تختفي مثل يهوذا، أليس كذلك؟

    ر.ح.

    حديقة إلزامية (باللون الأرجواني) مثل تشين إلزامية.

    وأذهب أيضًا في استراحة قصيرة (كاليفورنيا + يناير = 25 درجة، وبالتالي الزوجة + الكلب = المشي في الجبال).

    وفي هذه الأثناء، تحقق من الرابط

    http://en.wikisource.org/wiki/On_Physical_Lines_of_Force

    وانتقل لأسفل إلى المعادلة 136.

    سوف نتوسع عندما نعود.

  839. إسرائيل،

    على الرغم من ازدرائك لذكاء علماء الأحياء (رياض الأطفال؟؟) إلا أنني سعيد لأنك تمكنت أخيرًا من إعطاء بعض التوجيه لنموذجك. إنه نموذج مثير للاهتمام، ولكن حتى الشخص العادي الذي يدافع عن الواجب يرى بعض الثغرات غير المبررة. على سبيل المثال:
    1) أنت تدعي أن جزيئات موقعك النشطة تتحرك بسرعات من 0 إلى ما لا نهاية. ما هي القوة التي تحركهم؟ ولماذا لا يصلون إلى التوازن؟ فإذا كانت درجة حرارة الكون تمثل سرعتهم فإن سرعتهم تتضاءل. أنت تدعي أيضًا أن النظام سوف يتوسع في أي اتجاه محدد، صحيح، حتى يصل إلى حالة الراحة.
    2) في التشابك، حقيقة أنك تقول أن هناك جسيمات ذات سرعة لا نهائية لا تحل أي شيء. لماذا في انهيار الإلكترون المتشابك يتم إنشاء الجسيمات فجأة بسرعة لا نهائية؟ لماذا لا أقل؟ كيف "يعرفون" مكان وجود الإلكترون الثاني و"يطيرون إليه مباشرة؟"
    3) الجاذبية؟ هل تعتقد أن فرق الضغط بين جزيئات موقعك النشط هو المسؤول عن الجاذبية (أم هل أخطأت الفهم مرة أخرى؟). هل تعتقد أنك إذا وضعت كرتين في غرفة مملوءة بالغاز فسوف "تركضان" على الفور تجاه بعضهما البعض أم ستبدأان بالدوران حول بعضهما البعض بسبب "ضغط" جزيئات الغاز؟

  840. لقد استولى فيروس خبيث على جهاز الكمبيوتر الخاص بي. وإلى أن يتم ذلك، لا يمكنني الوصول إلى ملفاتي.
    1. لقد أحضرت لورينز فقط كمثال لتفسير محتمل. انه ليس الشخص الوحيد.
    2. المحادثة مع Free Shadow لم تبدأ بالرابط الذي قدمته لك. بالإضافة إلى ذلك، هناك أيضًا محادثة قصيرة هنا مع "يأتي ويذهب".
    3. حسنًا. نسميها عابرة. هل يوجد نموذج قياسي لتبادل الجرافتون؟ وإذا كان الأمر كذلك، فهل يعرض جسيمات المادة الباريونية كمستعمرات من الجرافيتونات؟ من فضلك الرابط.
    4. لقد فعلت ما يكفي. سوف أتحقق بعمق عندما تتاح لي الفرصة.
    لم أسمع عن كتاب اسمه النظرية النهائية. سأبحث عن الفرصة.
    والآن سأذهب في إجازة طويلة. شكرا لك على مثابرتك. مع السلامة

  841. 1. حسنًا. لورينز. تنقبض الأجسام في اتجاه الحركة. أعتقد أن حركة المرور مرتبطة بالموقع. ألا يتطلب هذا أن يكون لدى الموقع نوع من نظام الراحة المطلق؟ بعد كل شيء، هذا هو نفس نظام الراحة الذي كانت تجربة MM تحاول العثور عليه، أليس كذلك؟

    2. ما وجدته في الرابط الذي قدمته إلى لاتزل فيما يتعلق بالجاذبية هو "ما يشبه تأثير الجاذبية على حركة الضوء ("انحناء الفضاء")، ويرجع ذلك إلى حقيقة أن الضوء يتحرك بشكل أسرع في منطقة كثيفة (ولكن ليست كثيفة جدًا، وفي هذه الحالة يتم حظرها) وتكون ظاهرة التجاذب بين "مستعمرات" الجزيئات أقوى في المنطقة الكثيفة. بدلاً من استخدام مصطلح "انحناء الفضاء" أقول "تغير الكثافة".

    هل من المفترض أن أفهم من هذا كيف تعمل الجاذبية؟ هل فاتني شيء؟

    3. "الجسيمات تترك هذه المستعمرات بشكل عشوائي وتنضم الجسيمات" يبدو وكأنه بديل.

    4. وشرح ما ينطبق على المستقطبات موجود في كل كتاب عن النسيج. لقد سبق أن أعطيتك اسم الكتاب والصفحة، وقمت أيضًا بنسخ المقطع المعني لك. لا أستطيع أن أفعل أكثر من ذلك.

    هل سمعت عن كتاب اسمه النظرية النهائية؟ أعتقد أن هناك أفكارًا مماثلة إلى حد ما.

  842. لدي فيروس منتشر، وأجبرني على التبديل إلى جهاز كمبيوتر آخر بأحرف صغيرة. لهذا السبب "الكلاسيكية".
    على سبيل المثال، يمكن أن يكون الانكماش الذي اقترحه هندريك لورنتز تفسيرا جيدا.
    أنت لم تفهم مسألة المستعمرات. الفوضى الخاصة بك
    من تحدث عن تبادل الجسيمات؟
    لن أتمكن من الإجابة عليك بشأن المستقطبات حتى أفهم بالضبط ما حدث هناك

  843. 1. ما الخطأ في فهم M-M؟
    2. كيف يخلق بناء المستعمرة جاذبية؟ لماذا يتم تربيع القوة؟
    3. كيف يختلف تبادل الجسيمات عن النموذج القياسي لتبادل الجرافتون؟
    4. كيف يمكن للمستقطبين التواصل إذا تم تحديد حالتهم فقط بعد مغادرة الفوتونات للمصدر؟
    5. ما هي الكلاسيكيات البعيدة؟ (أول خطأ لك في اللغة العبرية حتى الآن).

  844. وفيما يتعلق بعدم المحلية في التشابك الكمي: لم أكن مقتنعا بأن التجربة لم تؤثر على الوسط. أنت تقول أننا تحدثنا بالفعل عن ذلك، ولكن في هذه الأثناء الشخص الوحيد الذي تحدث عنه بيننا هو أنت.
    وإذا تبين أن التجربة ليس لها تأثير، فسيظل من الممكن البحث عن تفسير داخل نموذجي، لكن من هذا الجسر لا فائدة من محاولتي القفز قبل أن أصل إليه.

  845. و. سوء فهم نتائج تجربة ميكلسون مورلي.
    ب. والافتراض المعقول هو أنه سيتم الحصول على نفس النتائج في كل مكان في الكون.

    وبالنسبة لمشكلتك الأخرى:
    وبما أنه تم الاتفاق على أن سرعة الضوء ثابتة، رغم أنها ليست كذلك، فإن الطريقة الوحيدة المقبولة لتفسير هروب خطوط طيف الامتصاص هي تأثير دوبلر، ومن هنا جاء الاعتقاد بوجود الطاقة المظلمة. في نموذجي، يتسارع الضوء القادم من المجرات البعيدة كلما اقترب منا، وهذا يخلق ظاهرة تشبه ظاهرة دوبلر.

    لقد تطرقت أيضًا إلى المادة المظلمة: يعتمد نموذجي بالكامل على الجسيمات التي أعزوها حاليًا إلى المادة المظلمة.

    لقد تم شرح الجاذبية بوضوح لي. من فضلك قم بإلقاء نظرة أخرى على كلامي إلى الظل الحر الذي أقدم فيه جزيئات مادة الباريون كمستعمرات من جزيئات المادة المظلمة. وتغادر الجزيئات هذه المستعمرات بشكل عشوائي وتنضم الجزيئات، وبالتالي يتم تدمير المستعمرات وبنائها بشكل مستمر. ويكون تركيز المادة المظلمة في الفضاء الموجود بين هاتين المستعمرتين أكبر من تركيزها في مناطق أخرى، وبالتالي تميل المستعمرات إلى إعادة البناء على المحور بينهما وتكون المسافة بينهما صغيرة. وتحت مسافة معينة، يتم تعزيز وجود القوى الكهربائية التي تمنع هذا النهج، ولكن هذه القوى تنشأ من سمة مختلفة لتركيزات الجسيمات. يتم شرح القصور الذاتي أيضًا بطريقة مماثلة، وفي مناسبة أخرى.

  846. حسنًا، من الصعب علي أن أكتب أيضًا. فيما بعد سأحاول تقديم الأفكار في الرقص.

    الان حان دورك. كيف يتم تفسير ثبات سرعة الضوء في أي نظام مرجعي؟

  847. إسرائيل البستاني. لقد انتهت أيام قراءتي، ولم تتوقف عن كتابة اللفائف.
    أعدك بكل صدق أن نموذجي يحل مشاكلك، لكن عليك الانتظار حتى يتم نشره بالطريقة المعتادة.

  848. جوفيلي، اترك النماذج الجسدية، ستبقى دائمًا نموذجًا لنا جميعًا في اللباقة والتشاتشا العبرية. ولهذا السبب وحده، كان الأمر يستحق الجهد الذي بذلناه لإرسالك إلى المملكة المتحدة. أعطيك الإذن بأن تأمرني بالنظام إذا بالغت وتصرفت بوحشية.

    لقد قرأت الرابط الخاص بك، وما زلت غير قادر على الاستنتاج منه بالضبط كيف يعمل العمل. والظاهر أن المخفي أكثر من الظاهر. مهما قلنا عن Le Sage، على الأقل معه، كل شيء واضح وبسيط ومنطقي، ومن هنا سحره. وهذه هي الطريقة الصحيحة لبناء النماذج في نظري، حتى لو كانت خاطئة أو ناقصة.

    لا أعرف طريقة علمية ليست بسيطة ومتماسكة ومنطقية. لا أتوصل إلى استنتاج مفاده أنه إذا كان النموذج (أ) لا يتفق مع النموذج (ب)، فهذا يعني أن النموذج غير صحيح. قد يكون من الممكن توسيعها حتى تستقر. اللورد كالفين (أعتقد أن صورته معلقة في مدرستك)، استبعد العمر الأقصى لكاداها من الاعتبارات الديناميكية الحرارية. بعد أينشتاين، تم توسيع كلا النموذجين ودمجهما بنجاح. وينطبق الشيء نفسه على الانجراف القاري.

    وأعتقد أيضًا أنه لا ينبغي بناء النماذج إلا إذا كان هناك سبب لذلك. وإلا ما الفائدة؟ محادثة على فنجان قهوة حول شؤون دوماس، حول ها ودا وخاصة حول ها؟ عن سيارة يوسي الجديدة؟ عن عمرها الذي قصرت تنورتها لأنها تحب المرشد جيورا؟ عن نظرية رامي الجديدة التي تجمع بين الجاذبية والكهرومغناطيسية مع ضريبة القيمة المضافة والمعكرونة وتفسير بابازو؟

    باختصار، لبناء نموذج يحتاج إلى تحسين النموذج الحالي.

    فيما يتعلق بفكرتي.

    أتناول عدة ظواهر أعتقد أن تفسيرها غير موجود أو غير كافٍ، وظاهرة واحدة أعتقد أن لها نموذجًا ممتازًا ولكن ليس مثاليًا. الظواهر التي لم يكتمل تفسيرها هي:

    1. الجاذبية

    2. الجمود.

    3. ثبات سرعة الضوء في أي نظام مرجعي.

    4. غير محلية.

    5. الكتلة والطاقة المظلمة.

    الظاهرة التي تفسيرها ممتاز ولكن غير مكتمل هي الكهرومغناطيسية.

    أحاول معرفة ما إذا كان من الممكن إدخال بعض التغيير في التفسيرات المقدمة لهذه الظواهر، لجعلها تعمل بشكل أكثر سلاسة، وإذا كان من الممكن التكامل مع بعضها البعض.

    1. الجاذبية. لماذا يجذب جسمان متباعدان بعضهما البعض؟

    2. الجمود. ما الذي يقاوم الجسم أثناء التسارع؟ ما الذي يعمل على استقرار الجيروسكوب أثناء الدوران؟

    3. ثبات سرعة الضوء في أي نظام مرجعي. فهل هناك تفسير مادي لهذه الظاهرة، أم يجب علينا، مثل أينشتاين، أن نتركها كمسلمة؟ وإذا كان هناك تفسير، فهل يحل كل غرابة النسبية؟

    4. غير محلية. أغرب ظاهرة على الإطلاق خصائص الجسيم أ تنتقل إلى الجسيم ب، على أي مسافة وفي زمن صفر، كما لو أن الجسيمين واحد. وعلى الرغم من ذلك، لا يمكن إرسال المعلومات عبر هذه الظاهرة.

    5. الكتلة والطاقة المظلمة. لماذا لا تتصرف المجرات بشكل صحيح حسب النماذج الموجودة؟

    الكهرومغناطيسية. كيف يعمل العمل عن بعد، والذي ينشأ من ظاهرتين مألوفتين: الكهرباء والمغناطيسية.

    لنبدأ بالكهرومغناطيسية.

    قدم ماكسويل تفسيرا ممتازا لهذه الظاهرة. وبحسب تفسيره فإن الكون مليء بـ "الأثير". الأثير عبارة عن جسيمات صغيرة ذات ثنائي قطب مغناطيسي، ويمكن رؤيتها كما لو أن سائلًا يتدفق من القطب "الشمالي" لثنائي القطب إلى القطب "الجنوبي". وباستخدام هذا النموذج، تمكن من حساب خصائص الدوامات الموجودة في الموقع بالوسائل الهيدروديناميكية والميكانيكية، ليبين أنها تخلق ظواهر الجذب والتنافر المعروفة لدينا من الكهرباء والمغناطيسية، وليبين أنها تتقدم على الموقع كموجات متعامدة مع بعضها البعض بسرعة الضوء.

    ولكن هنا تكمن المشكلة: سرعة الضوء بالنسبة إلى ماذا؟ ماكسويل، ومعظم معاصريه افترضوا أن ذلك بالنسبة للموقع بالطبع، مثل حركة الموجة الصوتية بالنسبة للهواء. كل ما تبقى هو العثور على الموقع فقط بالنسبة لما هو ثابت، وكل شيء يعمل.

    التكملة معروفة: فشلت تجربة ميشيلسون-مورلي في العثور على أي نظام راحة للموقع. حل أينشتاين المشكلة مع المسلمة 2 في العلاقات: سرعة الضوء هي نفسها في أي إطار مرجعي. ما يتغير هو الوقت والمسافة. لا يوجد موقع على الانترنت.

    سنأخذ استراحة قصيرة في القصة ونطرح بعض الأسئلة:

    1. إذا لم يكن هناك الأثير، فكيف تمكن ماكسويل من حساب سرعة الضوء بشكل جيد من خلال ثوابت الكهرباء والمغناطيسية؟ حدث؟

    2. في وقت تجربة مايكلسون مورلي، كان نموذج الكون عبارة عن كون لا نهائي ومتناحي الخواص في كل اتجاه. وقيل إن التجربة كانت ستنجح، وسيتبين أن نظام الراحة الخاص بالموقع هو نظام الراحة بربور 17. والسؤال هو: لماذا نظام الراحة هذا بالضبط؟ ما هو المميز عنها؟

    3. كما نعلم فإن حل أينشتاين يتضمن التخلي عن الزمن المطلق والمسافة. وهذا حل جذري وغير بديهي، وكما حاولت أن أبين في الماضي، فهو غير متوافق مع نظرية الانفجار الأعظم. ومن ناحية أخرى، ما هو الاحتمال الآخر الذي أمامنا والذي يتوافق مع حقيقة أن سرعة الضوء هي نفسها لكل جهاز قياس في كل نظام مرجعي؟

    ويجدر مقارنة هذه الظاهرة بظاهرة اللا محلية. اللامكانية هي ظاهرة غير مفهومة وغير بديهية، وأي تفسير آخر أفضل منها، كما حاولت يا يوفال في شرح القنوات بين القطبين. وعلى الرغم من أن الكثيرين حاولوا إظهار أن الظاهرة غير حقيقية (انظر كلام نيك هربرت في الرابط إلى RA) إلا أنه حتى الآن لم يتمكن أحد من إعطاء حل بديل، ونحن مضطرون إلى قبول حقيقة أن الواقع كمي. ، غير محلية وغير بديهية.

    ولكن ماذا عن المسلمة 2؟ فهل من الممكن أن نجد تفسيرا، مهما كان غريبا، بأن سرعة الضوء هي نفسها لكل جهاز قياس مع ترك الزمن المطلق سليما؟

    إجابتي هي نعم.

    الآن، قبل أن يبدأ الصراخ. كل ما سأقوله مجرد تكهنات، وعلى الأغلب ليس صحيحا في الواقع. بالطبع، أنا لا أدعي معارضة أينشتاين، ولكن فقط أطرح نظرية، لا أجد فيها أي عيب، وسأكون سعيدًا جدًا لو أظهر لي أحد المعلقين ما هو العيب فيها.

    لكن عليك أولاً الاتصال بـ Max.

    ماكس هو قائد أسطوري من الجيش. كل ما سنفعله كان يقول لنا: وماذا في ذلك، لقد قبضتم على T-T، إيه؟ وكل ما كنا نقوله هو وهي تقول: فماذا فتحت ت ت، إيه؟

    من تحليل نظرية ماكس، توصلت إلى استنتاج مفاده أن الطريقة المتبعة في الجيش هي الإمساك بـ T-T وفتحه.

    الأمر نفسه ينطبق على نظرية ماكسويل. تحتاج فقط إلى فتح نموذج الموقع.

    ماذا يعني ذالك. إذا أخذنا موجات في الهواء، يبدو أنها تتحرك دائمًا في نظام مغلق. وإذا كان في غرفة، فإن سرعة الصوت تتناسب مع الغرفة. إذا كان على متن الطائرة، ثم بالنسبة للطائرة. إذا كان في الهواء الطلق، فهذا يتعلق بهيئة الصحة بدبي. وإذا هبت الريح، فنسبة إلى الريح.

    ولكن ماذا يحدث إذا كان النظام مفتوحًا بالكامل؟ نسبة إلى ماذا ستتحرك الأمواج؟

    لفهم الفكرة، دعونا نفكر في بالون في الفضاء. ستكون سرعة الموجات داخل البالون نسبة إلى البالون. ويقال أن البالون يلتقي ببالون آخر، ويتحرك بسرعة مختلفة، ويندمج كلاهما في بالون واحد أكبر. ستكون سرعة الأمواج الآن مرتبطة بالبالون الجديد. سنضيف المزيد والمزيد من البالونات، وسنحصل على مركز سرعة جديد للموجات في كل مرة.

    والآن ماذا سيحدث إذا قمنا بتوحيد عدد لا نهائي من البالونات التي تتحرك بكل السرعات في كل الاتجاهات، وقمنا بتفجيرها فقط؟
    والنتيجة هي أنه لا يمكن أن يكون هناك مركز مشترك لسرعة الهواء المنطلق، لأنه حينها ستكون هناك أولوية لبالون معين لم يتغير مركز سرعته بالنسبة إليه، وهو ما يتعارض مع مبدأ التجانس. وإذا كان النظام بأكمله تحت الضغط، فسنحصل على "موقع نشط" حيث توجد في كل نقطة جزيئات تتحرك في كل الاتجاهات وبجميع السرعات من الصفر إلى اللانهاية. وسوف ينتشر النظام أيضًا، دون مركز محدد.

    و كما بينت من قبل:

    https://www.hayadan.org.il/particles-in-the-dark-2111111/#comment-319159

    وبعد تجاوز سرعة معينة، تتوقف الجزيئات ببساطة عن التأثير، وبالتالي تصبح "شفافة" من حيث المادة التي تصطدم بها.

    ولذلك فإن أي موجة تتقدم في الموقع النشط ستتقدم فعليًا بسرعات عديدة، وربما حتى جميع السرعات، لكن بالنسبة للمساح، أو الراصد، ستظهر دائمًا على أنها ذات سرعة واحدة فقط (في حالتنا - سرعة الضوء ).

    لاحظ أن صيغة الفعل المطلقة ظلت سليمة، مما أسعد أوكهام كثيرًا. إذا واصلنا تطوير النموذج، يبدو أنه من الممكن التوصل إلى وصف رياضي جيد لسؤال "ما هو الوقت؟"

    الآن، إذا رأينا الموقع النشط كمجموعة من المغناطيسات الصغيرة تتحرك بكل السرعات، كما وصفها ماكسويل بنجاح، فيبدو أنه بصرف النظر عن المادة المغناطيسية، يمكن أيضًا استخدام هذه المغناطيسات الطائرة كجسيمات La Sage، لكن هذه المرة نحن ليس لديك مشكلة الاحتكاك. السبب: بغض النظر عن مدى سرعة تحرك الجسم عبر الجسيمات، فهو ثابت، وهذا لنفس السبب الذي شرحناه سابقًا: لا يوجد مركز مشترك للسرعة، كما في وصف La Sage الكلاسيكي. في أحد الأبعاد، يمكنك التفكير في طريق سريع ذو اتجاهين، به ممرات لا نهائية حيث تتحرك السيارات بسرعة معينة في كل حارة. إذا رميت البلاستيسين في المنتصف، فلن يتحرك، لأن جميع السيارات تقابل بعضها البعض من جميع الاتجاهات، وبالتالي فإن البلاستيسين يكون دائمًا في حالة سكون، مهما كانت سرعته. ولن تواجه مقاومة إلا عندما تحاول زيادة سرعتها، وعندما تصل إلى سرعة ثابتة، ستتم إعادة ضبط القوة المؤثرة عليها مرة أخرى. وهذا ما يفسر الجمود.

    انتبه أيضًا إلى إمكانية عدم الموضعية، لأنه في الموقع النشط لدينا جسيمات تتحرك بجميع السرعات، بما في ذلك اللانهاية. لاحظ أيضاً التشابه مع الموجات الصوتية: فهي تتحرك بسرعة معينة - سرعة الصوت - ولكنها تتحرك في وسط وهو الهواء الذي يتكون من جزيئات يمكن أن تصل سرعتها إلى عدة أضعاف سرعة الصوت حسب نظرية ماكسويل-بولتزمان. توزيع. وتحمل هذه الجزيئات المعلومات معها، لكن لا يمكننا إرسال معلومات أعلى من سرعة الصوت.

    هذا كل شيء في الوقت الراهن. طاب مساؤك.

  849. إسرائيل،
    لن أجذبك بالكلمات. مما رأيته منك، لدي انطباع بأنك أحد القتلة. طريقتك بسيطة ومتسقة ومنطقية. أنت تأخذ نموذجين غير متوافقين مع بعضهما البعض وتخلص إلى أن أحدهما على الأقل غير صحيح. حتى أنك تقدم الحلول، وهو أمر مرحب به.
    يبدأ نموذجي من الصفر، لكنه لا يبقى هناك لأكثر من جزء من الثانية. ومن بين أمور أخرى، فهو يقدم أيضًا "الانفجار الكبير"، ويمكن لمن يريد أن يجد فيه نقاط المطابقة، لكن هذا أقل أهمية من الأشياء الأخرى التي يقدمها. أحد الأشياء ذات الصلة جدًا بعصرنا هو العلاقة التي وجدها بين الجاذبية وعائلة القوى الكهرومغناطيسية. يمكنك رؤية شيء من هذا في مراسلتي الأخيرة مع "Free Shadow" على هذا الرابط: https://www.hayadan.org.il/opera-confirms-and-submits-results-but-unease-remains-191111/#comment-324749
    أنا لا أصر على أن تبدأ بما يعادل "ليكن هناك ضوء"، ولكن إذا كان بإمكانك، على سبيل المثال، إظهار كيفية ارتباط الجاذبية والكهرومغناطيسية ببعضهما البعض، يرجى المشاركة.

  850. يوفلي تهامان.

    أنت تحاول أن تجعلني أشعل النار، هاه؟ لا لا. وظيفتي هي قطع النماذج. لك - لتربيةهم وذبحهم. نقرأ عن كل أنواع التسريبات الإعلامية عن موديلك والتي تم انتزاعها تحت التعذيب. أنت الكبير ويجب أن تبدأ. لقد سألتك من قبل إذا كنت تريد Pirgon أو Kesah ولم أحصل على إجابة. ولكن هناك شيء واحد يمكنني أن أؤكده لك بأمانة: إذا كان نموذجك يبدأ من "في البدء لم يكن شيء، ولم يكن شيء يتعذب"؟، هكذا دون توضيح السبب، فيبدو لي أنه من الأفضل أن تنشره في القسم العلمي في ياد نعمان.

    بالنسبة لنموذجي، وهو أقل تنظيمًا وانتظامًا من نموذجك بكثير، وفي الواقع حتى طرح موضوع النماذج لم أكن أعلم حتى أن لدي نموذجًا، لأنه من تجربتي السابقة القصيرة والمريرة، أفضل عدم المناقشة مع شخص ليس مهتمًا حقًا بالموضوع، أو لا يعرف المواضيع التالية:

    1. مبدأ ماخ.
    2. الجاذبية.
    3. الجمود.
    4. العلاقات، وقبل كل شيء إطالة الزمن.
    5. عدم المحلية في التشابك الكمي.

    هذا هو السبب الذي يجعلني أجتهد في توظيف أولئك الذين أبدوا اهتمامًا (حتى الآن، أنت وR.H. فقط)، وأرسلهم إلى الروابط. أولا، للتأكد من أنك تعرف المادة. ثانيا، أن نفهم أن المادة ليست سهلة حقا. ثالثا: التأكد من أن الموضوع مثير للاهتمام حقا للسائل، وإلا فسوف ينقطع بسرعة كبيرة عندما نفتح الموضوع.

    بالطبع أنا مستعد لتلبية معاييري الخاصة. إذا كان لديك أي روابط يجب أن أقرأها عن النموذج الخاص بك، أرسل لي إليهم.

    بالمناسبة، أنا غير مهتم بأي تعزيزات إيجابية. مجرد انتقاد على سبيل المزاح، ولكن المهنية وإلى هذه النقطة.

  851. "في الماضي اتهمتني (بشكل غير عادل في رأيي) بالتفلسف، وذكرت أيضًا أنه يجب دراسة الشكليات الرياضية. وأعتقد أن هذا هو بالضبط ما فعلته مع موضوع التشابك الكمي، بما في ذلك الشكلية الرياضية في الأعداد المركبة. "
    - لا أستطيع أن أعرف ما تعرفه أنت أو الآخرون حقًا، فقط احصل على انطباع بناءً على التعليقات. ربما أستطيع الحصول على انطباع خاطئ.

    "يبدو لي أن هذا أفضل من الإشارة إلى مقال جاليت الذي يتضمن نصف صفحة".
    - لا يوجد شك. ومن الأفضل أن تحصل على درجة الدكتوراه في مجال الكم.
    وبالمناسبة، المقال ليس لجاليت - غاليت هو من يسأل. يمكنك معرفة من هو المؤلف في نهاية المقال.

    "بالمناسبة، اقتباس من المقال: "في ميكانيكا الكم، بمجرد إجراء قياس على جسيم معين، الفوتون على سبيل المثال، نقول أن الدالة الموجية انهارت بمجرد إجراء القياس" الذي ربما يجيب على سؤالك "(بالمناسبة، أين انهيار الدالة الموجية المذكورة هناك؟)".
    - أنت مخطئ. لقد قمت بربط مقالتين: في الأولى لم أتمكن من العثور على مكان ذكر انهيار الدالة الموجية - وطرحت السؤال حول هذا الموضوع. وفي المقال الثاني ذكرت بالفعل، ولكني لم أسأل عنها.

    لم أقم بالربط بهذه المقالات فقط. الكتاب هناك (على حد علمي) هم باحثون في المعهد - يمكنك أن تتعلم منهم شيئًا أو اثنين.

    "الاختصار الذي تكتب عنه يشمل في حالتي كتابين كاملين في موضوع النسيج، ومحاضرات سسكيند العظيمة:"
    - يبدو أنك دفاعي، لا أعرف لماذا. لقد قلت سابقًا أنك تحب كل شيء، ماذا يمكنني أن أستنتج من ذلك؟
    على أية حال، كن قويا وشجاعا. يجب أن نتعلم من علماء الفيزياء في العالم وألا نتجاهل عملهم دون فهم ما يفعلونه على الإطلاق.

    "ليس هناك نقطة للمناقشة هنا. إما أن تعرف جهازًا يفعل ذلك أو لا تعرفه."
    - سألت إذا كان لدي فكرة، وليس إذا كنت أعرف جهازا يفعل ذلك.

    "لماذا لا يفهم الكثير من الناس ميكانيكا الكم؟ لماذا نبحث مثل أينشتاين عن حقيقة حتمية أعمق، ولماذا لا نقبل ببساطة الطبيعة الاحتمالية للواقع ونغلق الأمر؟
    – هذا سؤال فلسفي، ولكن أعتقد أن الإجابة قد تكون مختصرة: نظرية الكم لا تتفق مع الحدس.

    "حتى الآن. حظا سعيدا مع نموذج مضان.
    - شكرا. حظا سعيدا لك أيضا في الدراسات الكم الخاصة بك.

  852. إسرائيل شابيرا,
    من الواضح أن نموذجك لا يتناول أي جانب فلسفي أو بديهي وهو فيزيائي ورياضي بالكامل. هذا حقا ما ينبغي أن يكون. لكن عليه أن يبدأ من شيء ما. هل تمانع أن تخبرنا من ماذا؟

  853. يأتي ويذهب،
    أنا آسف لمرورك. في الوقت القصير الذي تحدثنا فيه، ساعدتني كثيرًا. من بين كل أولئك الذين يتواصلون معي، أنت وFree Shadow فقط هما الوحيدان اللذان يستمعان أيضًا. أما الباقون (وأنا منهم) فيبشرون فقط بالتوراة ولا يستمعون للآخرين.

  854. احصل على تصحيح - ما هو في أ وليس في ب، + ما في ب وليس في ج، - أكبر أو يساوي ما هو في أ وليس في ج.

  855. طالب.

    أولاً أريدك أن تعلم أنني أقدرك لعدة أسباب. الأول، أنك اخترت الذهاب إلى التخنيون وليس المحاماة، ليس لدي أدنى شك في أنه كان بإمكانك أن تكون ناجحًا جدًا هناك أيضًا. ثانياً، أن تستثمر وتبحث عن الحقيقة.

    حسنًا، الآن بعد أن أمتعنا أنفسنا، يمكننا العودة إلى العمل.

    1. لقد اتهمتني في الماضي (وهذا ليس صحيحًا في رأيي) بالتفلسف، وذكرت أيضًا أنك بحاجة إلى تعلم الشكليات الرياضية. وأعتقد أن هذا هو بالضبط ما فعلته مع موضوع التشابك الكمي، بما في ذلك الشكلية الرياضية في الأعداد المركبة. ويبدو لي أنه أفضل من الإشارة إلى مقالة غاليت التي تتضمن نصف صفحة. بالمناسبة، اقتباس من المقال: "في ميكانيكا الكم، بمجرد إجراء قياس على جسيم معين، الفوتون على سبيل المثال، نقول إن الدالة الموجية انهارت بمجرد إجراء القياس" والذي قد يكون أجب عن سؤالك "(بالمناسبة، أين انهيار الدالة الموجية المذكورة هناك؟)".

    2. بعد قراءة مقال ماريوس، لدي شعور بأن هناك بعض سوء الفهم الأساسي حول التشابك الكمي واللامكانية. ومن الممكن أن يكون سوء التفاهم معي بالفعل، ويمكن مناقشة ذلك.

    وفقا لفهمي، في التشابك الكمي هو أن:

    و. تنتقل معلومات الدوران أو الاستقطاب على الفور من الجسيم A إلى الجسيم B.

    ب. لا يمكننا إرسال أي معلومات عبر الرابط.

    متناقض؟ بالطبع لا. التفسير هو أن المعلومات تمر بلا شك، لكننا إذا حاولنا إرسال المعلومات، فسوف نفشل.

    3. في مفارقة EPR، أخطأ أينشتاين مرتين. الأول: عندما ادعى أن غير المحلة غير ممكنة. وفي الثانية، عندما ادعى أن انتقال معلومات السبين بسرعة تتجاوز سرعة الضوء يتناقض مع النسبية. لم تفعل.

    4. "لا أرى كيف يرتبط هذا حقًا بمناقشتنا هنا." - هذا لا. لا أفهم لماذا طرحت هذا الأمر". لقد أثرت هذه القضية في محاولة لمعرفة ما إذا كان الحل لمشكلة الاحتكاك في L.S. يتضمن برنامج يهودا أيضًا برامج غير محلية. وفقا لنموذجي، نعم، وأردت أن أرى ما إذا كنت أنا ويهودا نسير في نفس الاتجاه.

    5 "أحب أن أتعلم وأفهم من الألف إلى الياء حتى النتيجة. هذه هي الطريقة الصحيحة في رأيي، والاختصارات خطأ كبير في التعلم." الاختصار الذي تكتب عنه يشمل في حالتي كتابين كاملين في موضوع النسيج، ومحاضرات سسكيند العظيمة:

    http://www.youtube.com/watch?v=0Eeuqh9QfNI

    6. "هل لديك أي فكرة عن كيفية قياس وقت وصول إشارة الراديو بدقة نانو ثانية أو أكثر؟"

    ليس هناك مجال للمناقشة هنا. إما أنك تعرف جهازًا يقوم بذلك أو لا تعرفه.

    7. "أعتقد أيضًا أن الكثيرين يشعرون بنفس ما أشعر به، وعلى رأسهم آينشتاين، وإلا لما حاربها طوال حياته." - وتفشل، لا تنسى." لقد فشل - على حد علمنا. أقول دائمًا إنه لو عاش لربما خرج من الورطة التي لم تكن موجودة قبل محاكمة بيل. لكن السؤال لا يزال قائما: لماذا لا يفهم الكثير من الناس ميكانيكا الكم؟ لماذا نبحث مثل أينشتاين عن حقيقة حتمية أعمق، ولماذا لا نقبل ببساطة الطبيعة الاحتمالية للواقع ونغلق الأمر؟

    حتى الآن. حظا سعيدا مع نموذج مضان.

    اليوبيل.

    نموذجي لا يتعامل مع أي جانب فلسفي أو بديهي. إنه جسدي ورياضي بالكامل.

    ر.ح.

    "إلى تومي، اعتقدت أنك مهتم بالنشر والمناقشة" - فقط إذا كان هناك طلب، ومع أولئك الذين يعرفون المادة. (وليس مظلة!). إذا كنت مهتما، يجب عليك قراءة الروابط التي قدمتها. كما أنك لن تناقش اللقاح مع شخص سيخبرك أنه لا يفهم ما هي المشكلة مع جينات الفيروسات، وبشكل عام يجب إغلاق جميع رياض الأطفال العامة وإرسال جميع الفيروسات إلى رياض الأطفال الإلزامية.

    وأخيرًا، منذ تقاعد مايكل في هذه الأثناء، هناك لغز رياضي:

    نظرية عدم المساواة لبيل:

    1 + \اسم المشغل{C}(ب، ج) \geq |\اسم المشغل{C}(أ، ب) – \اسم المشغل{C}(أ، ج)|،

    يعني في الأساس أن:

    ما هو في أ وليس في ب، + ما هو في ب وليس في ج، - أكبر أو يساوي ما هو في ب وليس في ج.

    يرجى إظهار (ليس عليك إثبات ذلك، ولكنه أمر مرغوب فيه).

    تلميح: الحل الأبسط هو الحل الهندسي.

  856. طالب علم
    "كيف يتسبب انهيار الدالة الموجية لجسيم واحد (رأيناه كما سميته) في انهيار الدالة الموجية لجسيم آخر (الفوتون) (في أي إطار تصف التفاعل؟") - أشرت إلى إطار "الغاز الفريد شديد البرودة" كاحتمال لمعادلة موجية فريدة تعتمد على جزيئات النيوترينو والتفاعل مع جزيئات المادة المظلمة (على أساس ذلك الغاز شديد البرودة). لكن اترك الأمر، أعتقد أنني أخطأت للتو.

  857. يهودا،

    "إذا كان الأساس خاطئًا، فلن أضيع الوقت في التعرف على "الأشياء المظلمة"" - فمن الواضح بالنسبة لي أننا ربما نرى الطريقة التي يعمل بها العلم بطريقة مختلفة. ولا أستبعد النظريات المعقدة على هذا المستوى (والتي يدعمها المجتمع العلمي) دون فهمها - وعندما أقول أفهم أقصد التخصص. بروح كلمات البروفيسور شيختمان - ادرس موضوعًا وكن خبيرًا فيه.
    ومن المؤسف أن تراه بمثابة نزول إلى الخطوط الشخصية أو ازدراء (وهذا بالمناسبة شيء أراه فيك تجاه علماء الفيزياء في العالم).

  858. عزيزي الطالب

    لن تستطيع إقناعي بالكلام :-
    "يا يهودا، أنت لا تنسى "القانون" الذي تعلمته في السنة الأولى من المدرسة الثانوية، ولكن ما يحقق فيه آلاف العلماء ترفضه دون أن تفهمه. بالتأكيد الفطرة السليمة. نهاية الاقتباس.
    العلم ليس ديمقراطيا، ولا يتم قبول الأمور فيه بالتصويت، وآلاف العلماء، وحتى آلاف طلاب التخنيون، لن يغيروا ذلك.
    ولا مانع من أن أكون الوحيد الذي يقبل أبسط قانون في العلوم والذي يدرس بحق في السنة الأولى من المدرسة الثانوية والذي يقول:-

    إذا كانت النتائج في الحقل لا تتناسب مع الصيغة، فقم بإلقاء الصيغة جانبًا أو على الأقل قم بتصحيحها.
    لا تغير فترة البيانات!.

    وتلميحك بأنني لا أملك الحس السليم لأنني أدافع عن "القانون" لا يساعدك
    ومازلت تكتبه بين علامتي اقتباس،
    لن أروج لأية أفكار وظيفتها فهم الصيغ التي لا تتناسب مع البيانات الموجودة في هذا المجال
    إذا كانت الأساسيات خاطئة، فلن أضيع الوقت في التعرف على "الأشياء المظلمة"
    بالإضافة إلى ذلك، من العار أن تذهب إلى الأحاديث الشخصية وتقلل من شأن المعلقين
    وبالمناسبة، تخرجت بمرتبة الشرف من كلية شنكار في تخصص الإدارة الصناعية. لذلك درست "قليلاً" بعد السنة الأولى من المدرسة الثانوية.
    يوم جيد
    سابدارمش يهودا

    ن. ب. هل تعرف ميكال؟أنت أصبحت مثله أكثر فأكثر... أعتقد أنه درس أيضًا في التخنيون

  859. إسرائيل،
    ما هي هذه الألغاز؟ القسم الثاني مع الطالب؟ مثال؟ ما المثال؟
    اعتقدت أنك مهتم بالنشر والمناقشة، ولكن إذا كان الأمر سريًا مثل لقاح الأنفلونزا الخاص بي، فلماذا تثيره أصلاً؟ والمزيد من الألغاز؟

    إذا كنت تريد إطلاق النار، أطلق النار

  860. لليوبيل
    شكرا على الاجابة
    لم أفهم النموذج الخاص بك. لا يزال من غير الواضح بالنسبة لي كيف يعمل النموذج الخاص بك، لكنني متأكد من أن إسرائيل وشركائها سوف يضايقونك بأسئلة محددة وسأحاول أن أفهم.
    الآن حان الوقت للجزء الثاني من نكتي.

  861. إسرائيل،

    إذا كنت منزعجًا من ميزة الدوران - فلماذا لا تبحث عن تفسيرات؟

    "... وقد أخذت كل المتعة من الفيزياء." - لا اوافق. من خلال معرفتي القليلة بنظرية الكم، فهي أكثر إثارة للاهتمام من الفصول "الكلاسيكية".

    "أعتقد أيضًا أن الكثيرين يشعرون بما أشعر به، وآينشتاين في المقدمة، وإلا لما حاربها طوال حياته". – وتفشل، لا تنسى.

    "لا أرى كيف أن الأمر له علاقة بمناقشتنا هنا." - هذا لا. لا أعرف لماذا طرحته.

    "يبدو لي أنك تفضل الأمر برمته، وأنا كذلك في بعض الأحيان." - في الواقع لا، لا أفضله على الإطلاق. أحب أن أتعلم وأفهم من الألف إلى الياء حتى النتيجة. هذه هي الطريقة الصحيحة في رأيي، والاختصارات خطأ كبير في التعلم. أريد أن أصدق أنك تعلم أنك لا تبني فهمًا حقيقيًا بكلمة "تمامًا". أتذكر أن محاضري الكمي قال إنه في رأيه يجب عليك القيام بالدورة عدة مرات، لأنك في كل مرة تفهم عمقًا أكبر.
    أعتقد أن فهمك عبارة عن موجة لا يمكنها بشكل كلاسيكي أن تترك عوالم الحدس وتخترق الصندوق الأسود لميكانيكا الكم، ولكن إذا اقتربت من الحاجز المحتمل - فيمكنك أن تنفق هناك بعمر طويل بما يكفي لتعطيك صورة لما الجحيم يجري في هذا المربع.

    "هل هناك أي تفسير مادي لهذا بخلاف "الانهيار المتزامن للدالة الموجية"؟" - أقول مرة أخرى أنه عليك إعادة قراءة المقال الذي قمت بربطه سابقاً. (أين انهيار الدالة الموجية المذكورة فيه؟)

    رابط آخر حول الموضوع:
    http://davidson.weizmann.ac.il/online/askexpert/physics/%D7%91%D7%9E%D7%9B%D7%A0%D7%99%D7%A7%D7%AA-%D7%94%D7%A7%D7%95%D7%95%D7%90%D7%A0%D7%98%D7%99%D7%9D-%D7%97%D7%9C%D7%A7%D7%99%D7%A7%D7%99%D7%9D-%D7%A9%D7%96%D7%95%D7%A8%D7%99%D7%9D-%D7%9E%D7%95%D7%A9%D7%A4%D7%A2%D7%99%D7%9D-%D7%96%D7%94-%D7%9E%D7%96%D7%94-%D7%9C%D7%9C%D7%90-%D7%AA%D7%9C%D7%95%D7%AA-%D7%91%D7%9E%D7%A8%D7%97%D7%A7-%D7%9C%D7%9E%D7%94-%D7%90%D7%99%D7%9F-%D7%96%D7%94-%D7%A0%D7%97%D7%A9%D7%91-%D7%9E%D7%99%D7%93%D7%A2-%D7%A9%D7%A2%D7%95%D7%91%D7%A8-%D7%9E%D7%94

    "هل لديك أي فكرة عن كيفية قياس وقت وصول إشارة الراديو بدقة نانو ثانية وأكثر؟"
    "https://www.hayadan.org.il/astronomers-reach-new-frontiers-of-dark-matter-130112/#comment-324216" - آسف، ليس لدي الكثير من الوقت للدخول في مناقشة أخرى، أحتاج إلى البدء في تطوير نموذج متعلق بالتألق للبحث الذي أشارك فيه.

  862. يأتي ويذهب،
    كنت سعيدًا بالعثور على أخ بعيد. أنا أيضًا شخص عادي تمامًا.
    أخشى أنك لم تفهم نموذجي. ليس لدي جسيمتان بل جسيم واحد فقط. ومن الممكن الإشارة إلى "المساحة الفارغة" بين الجزيئات على أنها نوع من الجسيمات، لكن أبعادها تعتمد على الكثافة التي يتم ترتيب الجزيئات فيها. (علامات الاقتباس تشير إلى أن ما يسمى اليوم بالمساحة الفارغة ليس فارغًا على الإطلاق، ولكنه مأهول بجزيئات عالية الكثافة). ببساطة، نموذجي يقول أن الكون مأهول بعدد لا نهائي من الجسيمات دقيق ومساحة فارغة لا نهاية لها. "الفضاء الفارغ" هو المسؤول عن الظواهر الكهرومغناطيسية، في حين أن الجسيمات هي المسؤولة عن الجاذبية والمادة الباريونية. هناك علاقة متبادلة بين مجموعات الجسيمات والفضاء الفارغ، ويتم التعبير عنها في ظاهرة غبار الجاذبية والحجب الكامل للضوء في المناطق التي تكون فيها كثافة الجسيمات كبيرة لدرجة تكوين المادة الباريونية.
    وعن أسئلتك باختصار: 1) لا أعتقد أنك فهمت. 2) تتغير الكثافة بين المجرات اعتمادًا على النقطة التي يتم قياسها فيها وتسبب تأثيرًا يشبه تأثير دوبلر (مما يقود الفيزيائيين إلى استنتاج، ليس بالضرورة صحيحًا، وهو أن المجرات تتسارع).

  863. إسرائيل،
    لقد بدأت في التخلص من النموذج الخاص بي وواجهت نقصًا في الفهم. إنها طويلة ولا تتناسب مع "قاعدة الرعشة"* التي نشأت عليها.
    هل يبدأ نموذجك من "في البدء لم يكن شيء، ولم يكن شيء يتعذب"؟
    *قاعدة الرجفة: يجب ألا يزيد طول الرد عن رجفة واحدة، بحيث لا يتجاوز حدود الشاشة

  864. لليوبيل
    كشخص عادي في الفيزياء، قرأت باهتمام النموذج (الجزء المسرب منه) الذي تقترحه كبديل للطاقة المظلمة. بعد إذنك، سأحاول تلخيص النموذج الذي اقترحته بطريقة مبسطة (على حد فهمي المنقوص) وأشاركك المشكلة التي تزعجني في نموذجك.

    وفقا لنموذج شليف هناك نوعان من الجسيمات، أحدهما مسؤول عن خلق الجاذبية. الأول هو الجسيم الذي يخلق الكتلة المظلمة (بعد إذنك سوف نسميها الجرافيتون المظلم) والثاني هو المسؤول عن خلق جاذبية الكتلة "العادية" (والتي سنسميها الجرافيتون الخفيف). يتصرف الجرافيتون الداكن كما تتوقع من الجرافيتون وجميع الجرافيتونات الداكنة تنجذب إلى بعضها البعض بسعادة. علاوة على ذلك، فإن الجرافيتون الداكن ليس انتقائيًا بشكل مفرط بشأن "الجاذبية" كما أنه يجذب الجرافيتونات الخفيفة. التغيير الذي تقترحه موجود في خصائص الجرافيتون الضوئي. هذا الجرافيتون، خلافًا لاسمه، يحافظ في الواقع على علاقة تنافر مع الجرافيتونات المضيئة الأخرى. ولكن عندما تتفاعل مع الجرافيتونات المظلمة فإنها تغير ارتباطاتها ويتم استبدال علاقات التنافر بعلاقات الجذب. وفقًا لحجتك، في مثل هذا الكون داخل المجرات (حيث توجد المادة المظلمة) سنواجه الجاذبية الطبيعية، بينما في الفضاء بين المجرات ستهيمن علاقات التنافر، مما سيؤدي إلى توسع الكون.

    إذا (وهذا أمر كبير إذا) فهمت نموذجك بشكل صحيح فهذا هو سؤالي. وكما نعلم، فإن الكون لا يتسارع فحسب، بل إن معدل تسارعه يتزايد. لكي يشرح نموذجك هذا السلوك، يجب أن يزداد عدد الجرافيتونات المضيئة لكل وحدة مساحة في الفضاء بين المجرات طوال الوقت. ما هي الآلية الموجودة في كونك والتي تزيد من عدد الجرافيتونات المضيئة لكل وحدة مساحة؟ التفسير الوحيد الذي يمكنني التفكير فيه هو أن الجرافيتونات المضيئة يتم قذفها من النجوم إلى الفضاء بين المجرات وأن معدل الانبعاث يتجاوز معدل توسع الكون. المعنى هو أن الفضاء بين المجرات يجب أن يصبح على العكس من ذلك "أكثر سطوعًا" أكثر فأكثر. ولكن، إذا لم أكن مخطئًا (ومرة أخرى، إنها إجابة كبيرة)، فإن الكثافة في الفضاء بين المجرات يجب أن تنخفض على مر السنين.
    باختصار، أسئلتي هي: 1) هل فهمت نموذجك بشكل صحيح 2) هل هناك نتيجة تجريبية مفادها أن الكثافة في الفضاء بين المجرات آخذة في الازدياد؟

  865. طالب علم

    "ألا تزعجك ميزة الدوران؟" مستاء جدا خاصة في الليل، عندما تطن الإلكترونات بدورانها المزعج ولا تسمح لك بالنوم. لو كنت السلطة الفلسطينية لقصفت كل أنفاقهم، حتى يتوقفوا عن تهريب مراكز الكتلة ذات الطاقات المنفصلة.

    "لن تكون قادرًا على فهم ميكانيكا الكم" أعتقد أنني أدركت منذ وقت طويل أهم شيء بالنسبة لي فيما يتعلق بميكانيكا الكم: أنني لا أحبها، وأنها أخذت كل المتعة من الفيزياء. وأعتقد أيضًا أن الكثيرين يشعرون بنفس ما أشعر به، وآينشتاين في المقدمة، وإلا لما حاربها طوال حياته. إذا قرأت رابط مقالتي عن "قانون القانون الثاني للديناميكا الحرارية" فربما تتذكر أنه في كيبوتسنا كانت الموجة عبارة عن موجة والجسيم هو جسيم، ولم تجرؤ أي موجة على التصرف مثل الجسيم، والإلكترونات كانت تدور بسعادة حول النواة، في مسارات إهليلجية حادة ومقاسة.

    الآن، بالطبع، أود أن أناقش معك الهاملتونيين والنيجاراجيين، على الرغم من مرور بضع سنوات جيدة، وبينما كنت مشغولًا بالهاملتونيين، كنت بالتأكيد مشغولًا بالهايوالد. إذا قمت بطرح قضية، وسوف نناقش. لا أرى كيف يرتبط هذا حقًا بمناقشتنا هنا. يبدو لي أنك لا تفضل ذلك على الإطلاق، وأنا كذلك في بعض الأحيان. لكن حتى الآن لم أتلق أي إجابات منك على الأسئلة التي طرحتها، حتى في هذه المقالة، على الرغم من أنني كخريج من التخنيون ليس لدي أدنى شك في أنك ستتمكن من التعامل معها. سأكررها وأضيف:

    1. "يا طالب - ولكن أيضًا وفقًا للرابط الذي أرسلته، يمكن أن تصل معلومات استقطاب الفوتون إلى الفوتون الثاني في زمن الصفر على مسافة آلاف السنين الضوئية.
    هل هناك أي تفسير مادي لهذا غير "الانهيار المتزامن للدالة الموجية"؟

    2.

    https://www.hayadan.org.il/astronomers-reach-new-frontiers-of-dark-matter-130112/#comment-324216

    3. ما مدى معرفتك بموضوع تصادمات الجسيمات؟ تأثير كومبتون؟

    4. هل لديك أي فكرة عن كيفية قياس وقت وصول إشارة الراديو بدقة نانو ثانية وأكثر؟

    إذا أمكنك تخصيص الوقت والتفكير لهذه الأسئلة، سأكون ممتنًا لك.

    ر.ح.

    لا توجد وجبات مجانية. أنت أيضًا لن تبيعني لقاح السعال الديكي الذي طورته مجانًا. زحف يوفال وأجاب على الأسئلة قبل أن نبدأ في المراسلة. الدفع هو القسم 2 للطالب. علاوة على ذلك، سيكون من الصعب عليك فهم الموضوع إذا لم تقرأ المثال المرفق، والذي يمكنك من خلاله مواصلة التجربة.

    اليوبيل.

    لا يوجد شيء أكثر أصالة من مودي، لكنه بالطبع لا يستطيع أن يناقض الحقائق المعروفة. نظرًا لأن النموذج الخاص بك تم تحريره وجاهز للقراءة، فلماذا لا تطلب من الأمن الميداني مرة أخرى الحصول على إذن برفع السرية بسبب الطلب العام؟

  866. RH، شكرا على التذكير. سأحاول ألا أصرخ. في روضة الأطفال كانوا يطلقون عليّ اسم "يوفال مانوبل"، وقد ظل هذا الاسم قائمًا.
    وعندما تنتهي إسرائيل من الرد على بوعليم (من هو؟ مقاول؟) سأقول شيئا.
    وأفترض أن إسرائيل، مثلي، تبحث عن نموذج واحد لكل الفيزياء. لذلك، في رأيه، يجب أن تكون جميع ظواهر العالم المادي مرتبطة ببعضها البعض - على سبيل المثال، عدم المحلية في التشابك الكمي يرتبط بدرجة حرارة إشعاع الخلفية الكونية كمقياس لعمر الكون.
    ولكن هناك فرق معين بيننا. إسرائيل تأخذ النماذج الموجودة وتبحث عن العلاقة بينها، بينما أنا أقوم ببناء نموذج واحد من الصفر وأستخدمه لبناء الفيزياء. وعلى طول الطريق، من الواضح أنني أقوم أيضًا ببناء بعض النماذج المقبولة، لكنني لا أقبلها على أنها توراة سيناء.
    وتشير إسرائيل إلى التناقضات الموجودة بين النماذج المختلفة وتسعى إلى التوفيق بينها. عندما أرى تناقضًا، أرفض على الفور كل التناقضات والتناقضات وأخرج بنسخة جديدة خاصة بي. وهذا ما لا أفعله في الفيزياء فحسب، بل يمكنك أن تجد "بصماتي" المميزة أيضًا في الطريقة التي أشير بها، من بين أمور أخرى، إلى الكتب المقدسة لمختلف الديانات.

  867. يهودا،

    أنت لا تنسى "القانون" الذي تعلمته في السنة الأولى من المدرسة الثانوية، لكن ما يحققه آلاف العلماء ترفضه دون فهم. بالتأكيد الفطرة السليمة.

  868. إسرائيل،
    اشرح، سأمنع يوفال من الصراخ.
    فوق سرعة الضوء دون تشابك؟ هل هذا مرتبط بفكرتك عن الوقت مع ساعات درجة حرارة الكون؟

  869. يوفال
    وزنك ذهبا. أنا جديد نسبيًا على الموقع، ولم أقرأ مقال ماريوس وتعليقاته، فقط أتذكر أنني أحببته منذ زمن جاليليو المطبوع. سنذهب لإسعاد البواليم، وسوف نتصل عندما نعود.

  870. إسرائيل،

    ألا تزعجك ميزة الدوران؟ من تأثير النفق؟ من حقيقة أن الجزيء لا يمكنه الدوران حول مركز كتلته إلا بطاقة منفصلة؟
    لن تتمكن من فهم ميكانيكا الكم بطريقة كلاسيكية في التفكير. إذا كنت مستعدًا للابتعاد عن هذا التفكير، فاقرأ المقال مرة أخرى، وإذا لم يكن الأمر كذلك، حظًا سعيدًا. (بالمناسبة، أين انهيار الدالة الموجية المذكورة هناك؟)

  871. للطالب
    مع ذلك
    هناك قاعدة تعلمناها في السنة الأولى من المدرسة الثانوية وهي: - إذا كانت النظرية لا تناسب القياسات، فارمها، لا تصحح القياسات لتناسب النظرية. يبدو أن هناك الكثير ممن نسوا السنة الأولى من المدرسة الثانوية ويفضلون تغيير البيانات. أضف المزيد من الكتلة بحيث يتطابق HSA مع النتائج في مجال سرعة المجرة العالية جدًا. صحيح أنه لا ينبغي عليك التخلص من النظرية فورًا عندما تجد مشكلة، لكن عزيزي الطالب، أنت تبحث عن هذه المقالة "المظلمة" منذ ثمانين عامًا، وسوف تتفق معي على أنه دون نجاح كبير. على استعداد للمراهنة معك بشأن الآيس كريم بأنهم لن يكتشفوه حتى نهاية العام. أنا أحب الآيس كريم (مع الكريمة المخفوقة!)
    مساء الخير
    سابدارمش يهودا

  872. ر.ح.

    لا علاقة لها بالنسيج. أستطيع أن أشرح، ولكن قد يكون طويلا. سوف يصرخ يوفال.

    اليوبيل.

    لدي الصبر والوقت. يمكنك تمديد بقدر الحاجة. اسمحوا لي أن أعرف مقدما إذا كنت مهتما بالكرسي أو الإيماءات بالموافقة. سوط.

    وإذا كان عليك أن ترمي، فارمي!

  873. نعم، بالتأكيد.

    كيف بالضبط؟؟؟

    تذكر ذلك:
    1) بمجرد إلغاء مؤشر التشابك
    2) لا يمكنك التأثير على نتائج القياس، فهي مفروضة عليك وعلى من يحمل الجسيم الثاني المتشابك. لذا، إذا قمت بقياس 1 فإن الآخر سيحصل بالفعل على 0 أسرع من سرعة الضوء، لكن لا يمكنك جعل جسيمك يصبح 1 ولكن فقط قم بقياسه.
    إذن كيف ستنقل المعلومات؟
    .

  874. تحرير الكلبة المرأة ذات المقود وحاجز الفم منذ الزفاف.
    الطالب - ولكن أيضًا وفقًا للرابط الذي أرسلته، يمكن أن تصل معلومات استقطاب الفوتون إلى الفوتون الثاني في زمن الصفر على مسافة آلاف السنين الضوئية.
    هل هناك أي تفسير مادي لهذا غير "الانهيار المتزامن للدالة الموجية"؟

    ر.ح.
    أنت لم تقرأ عن التجربة لأنني لم أكتب عنها.
    لم أكتب لأنه لم يسألني أحد.

    ومنذ أن سألت -

    لإرسال إشارة تحتوي على معلومات بسرعة تفوق سرعة الضوء بالطبع.

  875. إسرائيل،

    "لم أفهم ما تقصده بقولك "لا توجد معلومات تمر"."
    "أعتقد أنني فهمت ما قصدته. أنه من غير الممكن إرسال المعلومات - وهذا صحيح - ولكن هذا لا يعني أن المعلومات لا تمر". - المعلومات "الحقيقية" لا تمر، ولا توجد طريقة "لاستخراج" المعلومات من القياسات.

    بالمناسبة رابط حول الموضوع:
    http://davidson.weizmann.ac.il/online/askexpert/physics/%D7%94%D7%A1%D7%91%D7%A8-%D7%A2%D7%9C-%D7%A0%D7%99%D7%A1%D7%95%D7%99-%D7%94-epr

    اليوبيل,

    "... النماذج التي تقدم بالفعل تفسيرًا للملاحظات ولكن لم يتم إثباتها بشكل قاطع." - لا يوجد شيء اسمه ثبت قطعيا. يمكن للنماذج أن تتلقى تأكيدات فقط، لأنها تمثل في مجملها وصفنا (القريب) للواقع. لا يوجد شيء اسمه دالة موجية أو إلكترون أو فوتون، هناك فقط معلومات نجمعها عن البيئة ونفهمها بطريقة أو بأخرى.

    "... لأن في كلامه تلميحاً بضرورة اتخاذ هذه النماذج كنقطة انطلاق..." - أليس من الواضح أننا نقف على أكتاف العمالقة؟ هل تعتقد حقًا أن العلم سوف يتقدم إذا كانت هناك مشكلة في النموذج، فسيتخلصون من كل شيء ويبدأون من الصفر؟

    "ولكن، بغض النظر، من الجيد أن تعرف ما يقوله الفيزيائيون "المحترفون"، لأنه يمكنك الحصول على أفكار منهم". - قد يكون مناسباً للأشخاص الذين يمتلكون الصفات التي كان يتمتع بها أينشتاين وشرودنجر وديراك. وبما أنه لا أحد من المعلقين يتمتع بهذه الصفات، وعلى الأرجح لا يتمتع بها القراء أيضًا، فإن الادعاء غير مناسب. الطريق إلى العلوم عالية المستوى (مثل النظريات في الفيزياء الفلكية التي رفضها يهودا لأنها لم تكن مقبولة لديه بناءً على ما تعلمه في السنة الأولى من المدرسة الثانوية)، بالنسبة للغالبية العظمى من الناس، يمر عبر التخصص بطريقة منظمة الطريقة - أي درجة البكالوريوس، والدرجة الثانية، والدرجة الثالثة، وما بعد الدكتوراه، وما إلى ذلك. لا توجد طرق مختصرة. ولا يخترع المرء نموذجا لوصف الكون قبل أن يتعلم أسس الفيزياء الحديثة. وأيضًا، لا يوجد فيزيائيون "محترفون" وفيزيائيون "غير محترفين" - يوجد فيزيائيون ولا يوجد فيزيائيون (أو علماء وغير علماء)، فالأولون يروجون للفيزياء (العلم) والآخرون لا.
    بالنسبة لي، تبدو هذه الجملة بمثابة محاولة للتقليل من قيمة المنهج العلمي، كما لو أن العمل الذي يقوم به الفيزيائيون "غير المحترفين" (من هم؟) يعادل العمل الذي يقوم به "الفيزيائيون المحترفون" (صححني إذا أسأت الفهم). من العار أن نقدم العلم بهذه الطريقة. ربما لا يزال المراهقون الذين يقرأون هنا يعتقدون ذلك ويعتقدون أنه من المشروع تخطي الدراسات العليا والانخراط في اختراع نظرياتهم الخاصة.

  876. إسرائيل
    "أنا آخذ الكلب والمرأة للنزهة في الجبال" - أخبرني، هل تسمح لهما بالذهاب إلى هناك أم فقط عند عودتك إلى المنزل؟

  877. إسرائيل؟ حتى زوجتي لا تشتكي بعد ذلك.

    والآن، بعد أن قرأنا جميعًا وفهمنا بيل أسبكت ونيك هربرت (حسنًا، على مستوى ما، على الأقل فهمنا أنه كان مثيرًا للاهتمام للغاية، أوه صحيح، لقد عرفنا ذلك بالفعل من قبل) وقرأنا عن العجائب السبع الأكثر غير المبررة في الكم علم الميكانيكا:
    http://www.newscientist.com/special/seven-wonders-of-the-quantum-world

    لذا ربما ستشرح مثل القنفذ (أي ببطء وحذر) ما هي التجربة التي تريد القيام بها وما الذي ستظهره؟
    لا تغضب، أعلم أنك كتبت هذا 100 مرة على الأقل ولكنه مدفون في مكان ما في جبال النص على الموقع العلمي، فلماذا لا تحاول مرة أخرى؟

  878. يهودا
    صحيح، لكن ماكسويل هو الذي طور نظرية فريدي حول مجالات القوة وليس العكس. لذلك هناك أيضًا مسألة مقدار العبقرية.
    باستثناء أن ماكسويل مات صغيرًا جدًا، بينما مات فريدي مشعرًا جدًا، لذا فإن كل شيء متوازن.

    اليوبيل.
    لقد ناقشنا هذه المسألة بالفعل. يتم تحديد حالة المستقطبات فقط بعد أن تقطع الفوتونات طريقها، وذلك لتجنب الدمج بين المستقطبات.

    ولكن كما قلت، وأعترف أنني تركت لكم الخوض في بابل، فقد تم إثبات الأمر رياضياً. وجاءت التجربة فقط لتأكيد النظرية.

  879. كما ذكرنا، أنا شخص عادي من بين الأشخاص العاديين وقررت قبول كلامك دون سؤال لأنه تم إثباته رياضيًا. ومع ذلك، فإن الفحص التجريبي غير موجود. نسخ لصقات هاريني من النسخة العبرية:
    تم إجراء معظم التجارب على الضوء المستقطب، وليس على دوران الإلكترونات كما هو مقترح في الأوراق الأصلية، مما يعني أن التجارب على الضوء المستقطب أسهل في التنفيذ. تتشابه خاصية استقطاب الضوء في كثير من النواحي مع خاصية دوران الإلكترونات، حيث تكون زوايا المستقطبات نصف الزوايا المقابلة لكاشف الدوران.
    توجد تجارب أحادية القناة حيث يوجد كاشف واحد على كل جانب. في مثل هذه التجربة، من المفترض أن كل فوتون أو إلكترون لا يلتقطه الكاشف له قيمة كمية معاكسة لتلك التي يلتقطها الكاشف. مثل هذا الافتراض يمكن أن يؤدي إلى عدم الدقة في التجربة.
    هناك خيار آخر أكثر تعقيدًا بعض الشيء، وهو تجربة ذات قناتين حيث يوجد كاشفان على كل جانب يقيس القيمتين الكميتين المحتملتين.
    المشكلة الرئيسية في هذه التجارب هي انخفاض كفاءة أجهزة الكشف، والتي، عند تضمينها في الحسابات الإحصائية، تقلل بشكل كبير من موثوقية النتائج.

    وأنا أسأل: كيف نعرف أن الأقطاب لا تؤثر على الفضاء المحيط بها؟

  880. إسرائيل
    لقد أعطيت مثالاً لفرادي وماكسويل
    ماكسويل المثالي - العبقري والمعرفة وفريدي الذي هو مجرد عبقري بدون معرفة
    مشتركهم عبقري مع العلم أنه ماكسويل.
    عندما تقول: - "كنت أقصد أنه يجب أن يكون هناك كل من مايكل باردي، العبقري البديهي الذي لم يتعلم الرياضيات، وماكسويل، الذي حصل على كليهما."، فإنك في الواقع تعني أنه يجب أن يكون هناك ماكسويل فقط.
    هذا غير مقبول بالنسبة لي!
    ترشيد
    31.12.2012 ناكل آيس كريم أتمنى على حسابك
    ولكن إذا كان ذلك على حسابي، فسوف أكون سعيدًا بعالم العلوم بأكمله!
    وبالمناسبة أنا أحبها مع الكريمة المخفوقة!
    يوم جيد
    سابدارمش يهودا

  881. إسرائيل شابيرا,
    معظمنا هنا من العلمانيين. "بقدر ما أفهم، تم إثبات الأمر رياضيًا" لا يعني الكثير بالنسبة لي إذا لم تقدم الدليل نفسه أو على الأقل رابطًا إليه.

  882. أعتقد أنني فهمت ما قصدته. أنه لا يمكن إرسال المعلومات - وهذا صحيح - ولكن هذا لا يعني أن المعلومات لا تمر.

  883. طالب علم
    لم أفهم ماذا تقصد بـ "لا توجد معلومات تمر".

    لدينا الإلكترون A عند النقطة A والإلكترون B عند النقطة B. عندما يتم قياس دوران الإلكترون A، فإنه يوجد في حالة أعلى أو أسفل. إذا لم تنتقل أي معلومات إلى الإلكترون B، فكيف يعرف أنه يختار الدوران المعاكس؟

    ولماذا اعتقد أينشتاين وناثان روزن وكل من يكتب في هذا الموضوع أن المعلومات تمر؟

    كما ذكرت، أنا فقط أسأل يهودا عن La Sage. وكان النسيج مجرد مثال. تم توجيه السؤال حول القصور الذاتي للعجلة إليك.

    أجاب العديد من الأساتذة على أسئلتي بالتفصيل والتفصيل. يشجع شيختمان على سؤالهم.

    مساء الخير.

  884. والحقيقة أن المشاغب سيكون هو المشاغب. هذا صحيح، ليس لدي أي صلة بالقطب في أندروميدا، وأنا أيضًا تساءلت كثيرًا عن ذلك. ومع ذلك، فإن ويكيبيديا ثابتة في رأيها: "ربما تم فصل الزوجين المتشابكين بمسافات كبيرة بشكل تعسفي". وأيضاً في كتابي "التشابك" مكتوب على ظهره: هل يمكن لجزئين أن يصبحا مرتبطين بشكل لا ينفصم...حتى لو كان الكون يفصل بينهما؟

    من المسلم به أنني لم أقم بإجراء تجارب خارج مجرة ​​درب التبانة.

    لكن حسب فهمي تم إثبات الأمر رياضيا.

  885. إسرائيل،

    أنا أتحدث عن مفارقة EPR، وهي مثال لما يسمى بالتشابك الكمي. اكتشف ناتان روزن هذه الظاهرة عندما كان صغيرا (22 عاما)، عندما قام بحساب مستويات الطاقة لجزيء الهيدروجين. واستنتج من حل معادلة شرودنغر أنه في ظل قيد تناظر معين (بسبب عدم القدرة على التمييز بين الإلكترونين)، نحصل على أنه حتى على مسافة لا نهائية عندما تكون الذرات منفصلة ويكون الجهد المتبادل صفراً، فإن الدالة الموجية يجب أن تحافظ على نفس التماثل. ومن المعروف من مبدأ التشتت أن الدالة الموجية للفريك هاملتونيان هي نتاج الدوال الموجية، ولكن في القيد أعلاه يتبين أنها مجموع.

    على أية حال، لم يتم تقديم أي معلومات في وصف هذه المفارقة وما شابهها (إنها ليست مفارقة حقًا، فقد تم الحفاظ على الاسم على حد علمي لأسباب تاريخية). ابحث عنه في الأدب.

    "هل من الواضح لكلينا أن العديد من الإلكترونات (أو الفوتونات) يمكن أن تتشابك، وتنتشر في الكون في نقاط مختلفة تفصل بينها ملايين السنين الضوئية، وأن انهيار أحدها سيؤدي إلى انهيار فوري في كل الباقي؟ ؟" - لا أعلم، أنا لا أجيد الفيزياء التجريبية.

    "أريد أن أرى ما إذا كان الحل نفسه يتضمن مشكلة اللا محلية. هذا كل شيء." - قال: إنك تستخدم مفاهيم غريبة عنه.

    "حدس. نعم أعتقد أنه ضروري. هل يمكنك أن تشرح لي ماذا يحدث في نموذج الدراجة النارية عندما تكون عجلة القيادة في الهواء؟ لماذا يكون من الصعب جدًا هزها جانبًا عندما تدور العجلة، على الرغم من أن سرعة الدراجة النارية 0 بالنسبة إلى الأرض؟" - لا أعرف في الوقت الحالي ما هو المثال الذي تتحدث عنه، لكنه غير ذي صلة. أنت تتوقع أن يكون لدى الأشخاص العاديين حدس في نظرية الكم - وهو أمر مثير للسخرية.

    R.H. Rafai.M،

    "الأمر تقريبًا هكذا (مع بعض البيانات المفقودة)، هل هذا منطقي بالنسبة لك؟ شيء في الاتجاه؟" - لا أفهم فيزياء الجسيمات، وعادةً لا أخوض في موضوعات لا أفهمها. لا يسعني إلا أن أشير إلى أنه ليس من الواضح ما هو المقصود بالطول الموجي 0، وكيف أن انهيار الدالة الموجية لجسيم واحد (كما رأيناه كما سميته) يتسبب في انهيار الدالة الموجية لجسيم آخر (فوتون) ) (في أي إطار تصف التفاعل؟)

    إسرائيل (الرد الثاني)،

    ليس لدي الوقت حاليًا لإلقاء نظرة هناك، لكن أنصحك بعدم تطوير التوقعات. الأساتذة وأعضاء هيئة التدريس أشخاص مشغولون. جدا.

    "يتم تحديد الدوران والاستقطاب فقط بالقياس، ويمران بأعجوبة إلى الإلكترون أو الفوتون المتشابك" - الخصائص لا تمر. ميزة مثل الدوران ليس لها معنى حتى تقوم بقياسها. لا توجد حقيقة موضوعية توجد فيها خاصية الدوران بغض النظر عن القياس.
    ومن المهم أيضًا الإشارة إلى أن القياسات التي تصف "المفارقة" يجب أن تكون على كميات ممثلة بعامل لا يتغير مع الطاقة، وإلا فيمكن وصف الظاهرة بشكل كلاسيكي.

  886. إسرائيل، استمع من فضلك (تجاوز تعريف المقلاة أو طنجرة الضغط)،
    يتابع الحريري تطورك، ويفرح بالعرض. الآن أنت تركز على موضوع واحد، وهذا شيء جيد. ما يذهلني الآن هو أنني لم أسمع في الأخبار أن أحدًا قد وضع كاشفًا للفوتون على أندروميدا. يرجى الارتباط، إذا لزم الأمر.
    و بجدية، المسافات التي أجريت فيها التجارب ليست كبيرة. على الرغم من أنه تم إجراء ترتيبات خلال التجارب بهدف منع إمكانية التأثير على الوسيط بين الكاشفين، لكن بما أننا لا نعرف بعد كيف يعمل الوسيط نفسه (أنت تتحدث مثلا عن "الأثير النشط")، فمن المستحيل لمعرفة ما إذا كانت الإجراءات الأمنية تقوم بتسليم البضائع بالفعل.

  887. ر.ح. - ها هو الرابط الالكتروني.

    لكن كعقاب، إذا تمكنت من الوصول إلى النهاية: لماذا لم يكتمل الدليل؟
    تلميح: لقد طرح يوفال هذا الأمر بالفعل في تعليق سابق.

    آريا - من الواضح أن الإشارة كانت إلى الإشعاع الكهرومغناطيسي الذي ينتشر كالصدفة. لا يمكن تركيز التشابك الكمي مثل شعاع الليزر. إنها حقيقة أنها تعمل في كل نقطة في الكون، على عكس شعاعنا الذي يمكن التركيز عليه والرقص معه.

  888. إسرائيل،

    شكرا على الشرح. سأبحث عن تفسير لمتباينة بيل وعن تجربة جانبية (؟) بلغة يستطيع حتى عالم الأحياء فهمها. اذا كان لديك رابط جميل ارسله

    الصلوات؟ بأي أسلوب؟

  889. ربما لا يكون الأمر مناسبًا للمناقشات المتعمقة هنا، لكن ليس من الصحيح القول إن الموجات الكهرومغناطيسية أصبحت أضعف. ونشهد ضعفها كدالة للمسافة، لأن الإشعاع يتبدد ومن ثم تقل كثافته لكل وحدة مساحة. على سبيل المثال، إذا كان هناك شعاع ليزر تم تركيزه بالكامل ولم يتباعد - فلن تتغير شدته مع المسافة (أو الوقت).

  890. ر.ح.

    إن حقيقة عودتك إلينا تظهر أن الصلاة تساعد أحيانًا.

    إلتواء. لقد تم طرح المعضلة التي وصفتها مرات عديدة، مثل معضلة الأستاذ المنتشرة. إذا رأيته بجورب أصفر، فاعلم على الفور أن الآخر برتقالي، والعكس صحيح.

    لذا دعونا أولًا نرى كيف يتم الأمر مع الفوتونات المتشابكة. هناك الاستقطاب هو نفسه - إذا كان للفوتون أ استقطاب معين، فإن أخيه لديه نفس الاستقطاب.

    كيف كيف؟ جاء أينشتاين بفكرة "المتغيرات الخفية" - كما هو الحال مع الأستاذ أو صناديقك، كانت البيانات موجودة بالفعل، والملاحظة كشفت عنها فقط. الشيء نفسه مع الفوتونات.

    لكن في الكم لا توجد متغيرات مخفية، بل فقط تراكب. يؤدي النظر إلى نفسه إلى انهيار الدالة الموجية، مثل عملة معدنية في الهواء متراكبة على شكل شجرة الوافل، وفقط عندما يتم الإمساك بها تختار حالة واحدة فقط. الشيء نفسه بالنسبة للقمة الدوارة، التي تهبط على حرف واحد فقط.

    في التشابك، مع عجلتين متشابكتين، إذا سقطت إحداهما على N، سقط شقيقها أيضًا.

    حتى ظهور بيل، كانت تفسيرات أينشتاين وبوهر تبدو مقبولة وغير حاسمة. أثبتت متباينة بيل، التي أعقبتها تجربة آسبكت، فشلها، وكان المحللون الكميون على حق وليس أينشتاين. يتم تحديد الدوران والاستقطاب فقط بالقياس، ويمران بأعجوبة إلى الإلكترون أو الفوتون المتشابك، في زمن صفر وعلى أي مسافة، ومن الممكن أيضًا تشابك العديد من الجسيمات الأولية. وإن شئت سأبحث عن الرابط الذي يشرح التجربة أو أنسخه لك. (طويل، R.H.، طويل).

    يهودا – إذا فسرت كلامي عن الحدس على أنه تخفيف للحاجة إلى دراسة الفيزياء المعروفة، لم يكن هذا هو القصد. قصدت أنه يجب أن يكون هناك كل من مايكل باردي، العبقري البديهي الذي لم يتلق تعليمًا رياضيًا، وماكسويل، الذي حصل على كليهما.

  891. إسرائيل،

    هناك شيء لم أفهمه أبدًا بشأن هذا التشابك. بقدر ما أفهم، في عملية إنشاء الإلكترونات المتشابكة، سيتم إنشاء أحدهما دائمًا "لأعلى" والآخر "لأسفل"، صحيح حتى الآن؟ إذن ما هو السحر الكبير؟
    لنفترض أنني وضعت كرتين، واحدة بيضاء والأخرى سوداء في صندوقين مغلقين. سأرسل لك واحدًا منهم في مجرة ​​المرأة المسلسلة. تفتح الصندوق وتكتشف أن الكرة بيضاء اللون، ولدهشتك تعرف أن الكرة المتبقية على الأرض سوداء! وبالفعل عندما نفتح الصندوق الذي بقي هنا نتعجب ونتعجب، هناك كرة سوداء بداخله. هل انتقلت المعلومات بسرعة لا نهائية؟ ما الفرق بين هذا وتجربة التشابك؟

  892. طالب علم
    ومن المثير للاهتمام أنك ذكرت انهيار الدالة الموجية لأنني كنت أفكر فيه فيما يتعلق بالتجربة التي أجريت منذ وقت ليس ببعيد على الناترينو.
    على العموم، توصلت إلى استنتاج (ربما لا يكون صحيحًا، ويبدو لي أنه يعمل بشكل جيد على الرغم من أن بعض المعلمات لا تزال مفقودة)
    ما لاحظناه هو نوع من الوسط الذي يتحرك عليه الفوتون، مثل "مجموعة من الموجات" الفريدة التي يتحرك عليها الفوتون. أي أن الدالة الموجية للنيوترينو تسبق الدالة الموجية للفوتون، لأن تراكب النيوترينو ينهار (أثناء قياس السرعة) - ويكون الفوتون على نفس مدار النيوترينو - وبالتالي فإن sp للفوتون أيضًا ينهار، تقريبًا في نفس الوقت ولكن على نفس المدار في الفضاء مثل المدار الذي لاحظناه، وتتغير الدالة الموجية للفوتون (الطول الموجي يساوي 0). باختصار، يُقرأ انهيار الدالة الموجية للفوتون أثناء القياس، ولكن فقط بعد أن نلاحظه في تلك المنطقة من الفضاء. يبدو الأمر كذلك تقريبًا (مع بعض البيانات المفقودة) هل هذا منطقي بالنسبة لك؟ شيء على طول الخطوط؟

  893. ترشيد
    ولن يجدوا في 2012 أو 3012 الجسيم المسؤول عن الكتلة المظلمة؟؟:,
    لقد استثمروا بالفعل المليارات في البحث عنها، يبحثون عنها منذ ثلاثينيات القرن الماضي، منذ أن وصل مفهوم الكتلة المظلمة إلى الوعي العام.
    لا أعتقد أنه موجود.
    على استعداد للمراهنة عليك على ذلك. (بوظة!)
    الى جانب ذلك، بيننا هل تؤمن حقا بمفهوم الطاقة الفراغية؟؟
    لا يمكن أن يكون هناك شيء؟؟
    ويتحدث يوفال وإسرائيل عن هذا الهروب للمنطق من الوعي العلمي.
    أنا متأكد من أن النظرية المنطقية للواقع المادي الكوني مخفية في مكان ما، وسوف يجدها العلم.
    سوف تتواجد الكتلة والطاقة المظلمة في مكان شرف بجوار إشعاع الفلوجستون والسعرات الحرارية والـ N!
    مساء الخير
    سابدارمش يهودا

  894. طالب علم

    لا أستطيع تجنب الشعور بأننا لا نتحدث عن نفس الموضوع.

    فقط للتأكد: هل من الواضح لكلينا أن هذا مثال لإلكترونين متشابكين، أحدهما في غرفة معزولة في تل أبيب، والآخر في غرفة مماثلة في مجرة ​​المرأة المسلسلة. طالما لم يتم قياس الدوران، فكلاهما في حالة تراكب الدوران "لأعلى" و"لأسفل". بمجرد قياس أحدهما، تجد أن دوران كليهما يكون دائمًا معاكسًا. التفسير الوحيد (وفقًا لبور وأينشتاين وبيل وأسبكت) هو أن معلومات الدوران مرت على الفور من الإلكترون A، المسافة بأكملها، واخترقت النجوم، ووصلت إلى الإلكترون B؟

    هل من الواضح لكلينا أنه من الممكن تشابك العديد من الإلكترونات (أو الفوتونات)، وتشتتها في الكون في نقاط مختلفة تفصل بينها ملايين السنين الضوئية، وأن انهيار أحدها سيؤدي إلى انهيار فوري في الكل؟ البقية؟

    يهودا. أنا لا أطلب منه شرحًا عن الكميات، ولكن فقط عن طفله، نظرية لا سيج. فأشرت: "أنا لا أطلب حل اللغز بالطبع. فقط برمجة الحل." يدعي يهودا أن لديه حلًا سريًا لمشكلة الاحتكاك في L.S. أريد أن أرى ما إذا كان الحل نفسه يشمل المشكلة غير المحلية. هذا كل شيء.

    حدس. نعم أعتقد أنه ضروري. هل يمكنك أن تشرح لي ماذا يحدث في نموذج الدراجة النارية عندما تكون عجلة القيادة في الهواء؟ لماذا يكون من الصعب جدًا هزها جانبًا عندما تدور العجلة، على الرغم من أن سرعة الدراجة النارية 0 بالنسبة إلى الأرض؟

    إذا كان شرحك يتضمن كلمات مثل "القصور الذاتي" و"الزخم الزاوي"، فهذا لا يكفي بالنسبة لي. شرح كامل: ما الذي يمسك عجلة القيادة؟ إذا كنت تعتقد أن لديك تفسيرًا بسيطًا، فأنت على خلاف مع نيوتن (الدلو الدوار)، وماخ، وأينشتاين.

    يوفال - لم تشعر بالإهانة لأنني وصفتك بالوغد، أليس كذلك؟ وذلك بسبب: "اسمع يا إسرائيل، أنت الرب إلهنا إله واحد!"

  895. إلى السيد صابردرميش،

    ومن المتوقع أن يتم العثور على الجسيم المسؤول عن "الكتلة المظلمة" هذا العام في عام 2012. كما يحاول "الطالب" أن يشرح: المشكلة أكثر إثارة للاهتمام من صيغة نيوتن للجاذبية. ربما يجب عليك أيضًا أن تنظر قليلًا إلى "طاقة الفراغ" وتأثير كازيمير الذي تم إثباته تجريبيًا مؤخرًا فقط.

    واليوبيل,

    لقد كنت أحاول مؤخرًا التعرف على الابتكارات المتعلقة باستكشاف الفضاء... هناك بعض الجهود والأفكار المثيرة للاهتمام في هذا المجال.

  896. سابدارميش يهودا،
    وأنا مكانك لن أتعجل في الرد على الطالب التخنيون. كان سؤاله استفزازيًا إلى حد ما، وقد تم تفجير عدد لا بأس به من الصمامات بالنسبة لك. وبالتالي، فإن إجابتك تحتوي على العديد من الاستفزازات المثيرة.
    وتحدث عن ضرورة معرفة أساسيات الفيزياء الحديثة، وأدرج في هذا النماذج التي رغم أنها تعطي تفسيرا للملاحظات، إلا أنها لم يتم إثباتها بشكل قاطع. وهنا أخشى أن يكون مخطئا، لأن هناك تلميحا في كلامه إلى أننا يجب أن نأخذ هذه النماذج كنقطة انطلاق، وبهذه الطريقة قد نفوت إمكانية إيجاد نماذج أخرى قد تكون أفضل من النماذج المقبولة.
    ولكن، بغض النظر، يجب أن تعرف ما يقوله الفيزيائيون "المحترفون"، لأنه يمكنك الحصول على أفكار منهم.
    أرى أنك تحارب المادة المظلمة منذ فترة طويلة ولكنك تؤمن بوجود أي جسيمات. من الممكن أن تكون جسيماتك والمادة المظلمة هما نفس الشيء مع تغيير الاسم.

  897. يهودا،

    "لا أعتقد أنني بحاجة إلى تعلم كل الأشياء غير المقبولة بالنسبة لي" - فكيف "تقرر" أنها غير مقبولة بالنسبة لك؟ هل تعتقد أنك تفهم النظريات الموجودة في علم الكونيات؟

    لديك مشكلة أساسية جدًا في فهم ماهية العلم بشكل عام وكيفية تقدمه. وسأبين أخطائك في القياسات التي قدمتها:
    "أنت، على سبيل المثال، ستدرس عشر سنوات من الدراسات الدينية قبل أن تقرر ما إذا كان هناك إله أم لا؟" - الفرق أن العلم لا يخضع للاعتقاد. سواء كنت تؤمن بالنسبية أو نظرية الكم أم لا، فهذا لا يغير من صحتها.

    "أفهم أنهم سيصوتون معك للكنيست في انتخابات حرة، نعم، ولكن فقط أولئك الذين لديهم دكتوراه في المواطنة، لأن الآخرين ليس لديهم المعرفة الكافية بالموضوع!" - التصويت للجميع وفقا للمبادئ الديمقراطية. في العلم أنت لا تشير إلى ما هو صواب. هناك نماذج، وهناك تجارب، وهناك تأكيدات.
    بالمناسبة، بعض الناس يعتقدون ذلك، وأن النظام الانتخابي الديمقراطي ليس جيدًا لسبب أنه ليس كل شخص "يستحق" التصويت. ليس لدي رأي في الموضوع، لكني سأذكر نسخة من مقولة تشرشل الشهيرة: الديمقراطية ليست أفضل أشكال الحكم، بل أقلها سوءا.

    "أنا لا أدعي إبداء الرأي في أشياء لا أعرفها ولا أجيب، على سبيل المثال، حول التشابك الكمي أو نظرية الأوتار." - وفي الفيزياء الفلكية هل تفهم؟ ففي النهاية، أنت لا تعبر عن رأي، أنت ببساطة تقوم بتزييف إحدى النظريات الأكثر تعقيدًا في الفيزياء اليوم وتحاول "تطوير" نظريتك الخاصة، دون معرفة ما تقوم بتزييفه على الإطلاق.

    "لقد تعلمت في السنة الأولى من المدرسة الثانوية أنه إذا كانت القياسات لا تتناسب مع الصيغة، فمن الضروري البحث عن صيغة أخرى." - من المحزن أن نسمع أن هذه هي الطريقة التي تعتقد أن العلم يعمل بها.

    "باختصار، كانت السنة الأولى من المدرسة الثانوية كافية بالنسبة لي، ولست بحاجة لدراسة مقال مظلم لمدة عشر سنوات." - إذن أنت متعجرف حقًا أو ساذج حقًا. هل تعتقد حقًا أن السنة الأولى من المدرسة الثانوية ستقودك إلى تطوير نظرية لوصف الكون؟

    "سوف تحصل على شرح لرأيي في الموضوع مدته 13 دقيقة، وليس عشر سنوات!" - لقد سمعت بالفعل جزءًا من المحاضرة التي نشرتها هنا مرة واحدة (التي تبلغ مدتها ساعة ونصف تقريبًا). يؤسفني أن أخبرك بذلك، لكن ما تعلمه هناك لا علاقة له بالعلم.

  898. للطالب
    تم تأكيد ردي من الساعة 2.16:XNUMX صباحًا. أحب أن أسمع رأيك في الأمور
    يوم جيد
    سابدارمش يهودا

  899. "كيف يؤثر انهيار الدالة الموجية عند النقطة A على انهيارها عند النقطة B، على مسافة كبيرة، في زمن الصفر." - تصف الدالة الموجية النظام وعند قياسه ينهار إلى قيمة ذاتية معينة للعامل الموافق للمتغير الذي قمت بقياسه. إذا قبلت فكرة انهيار الدالة الموجية، فأنا لا أفهم السؤال. قبل القياس هناك يقين في حالة الهوية بسبب الوصف الموجي وبعده هناك يقين مطلق بسبب الانتقال إلى الوصف الكلاسيكي.
    وبالمناسبة، هناك أعمال ومقالات في هذا المجال، ومن الممكن الحصول على تطابق بين الميكانيكا الكلاسيكية وميكانيكا الكم تحت تعديلات معينة، على سبيل المثال في الهاملتونية المعتمدة على الزمن.

    "الموجات الكهرومغناطيسية تضعف مع المسافة، وليس كذلك الدالة الموجية." - لا أفهم تمامًا ما تقصده بـ "نقاط الضعف" والقياس على الموجات الكهرومغناطيسية. خذ على سبيل المثال الدالة الموجية التي تقارب الإلكترونات الموجودة في ذرة الهيليوم - ماذا تقصد بنقاط الضعف التي تعادلها؟

    "إن انهيار إلكترون واحد يمكن أن يؤدي نظريًا إلى انهيار الإلكترونات في الكون بأكمله." - أنا لا أفهم هذا الادعاء.

    "إذا افترضنا أن الانهيار يستهلك طاقة قليلة، فأين قانون حفظ الطاقة؟" - في التفاعل من القياس؟ لا أعرف الموضوع على مستوى عالٍ بما يكفي للإجابة على السؤال، لكن من المحتمل أن تكون هناك مثل هذه التحليلات، على سبيل المثال، في إطار نظرية الاضطراب.

    "لذلك، حتى لا نبتعد عن الحدس إلى "المجالات المركبة" و"الفضاءات الفائقة متعددة الأبعاد"، لشرح ما يحدث للإلكترونات البسيطة المحبوبة والمستخدمة يوميًا، فإنني على الأقل أستمع دائمًا إلى أولئك الذين لديهم حدس فيزيائي بسيط" - في رأيي أنك لن تصل إلى أي مكان بهذه الطريقة. الإلكترونات البسيطة واليومية؟ لا يوجد حدس في الإلكترونات وسلوكها. على سبيل المثال، لا يمكنك تخيل خاصية مثل الدوران، أو فهم حقيقة أنه لا يوجد جسيم ولا موجة - بل هناك جسيم موجة، أو أن تفهم بشكل حدسي كيف يمكن للإلكترون أن يكون في كلا الطرفين دون المرور عبر المنتصف. ما هو الحدس الذي تريد استخلاصه من يهوذا بالضبط؟ حتى لو وصل الأساتذة في مرحلة ما من حياتهم الأكاديمية إلى مستوى من الفهم البديهي لنظرية الكم - وهذا بعد عقود من المعرفة العميقة بهذا المجال - فإن هذا ليس وضع يهودا.
    بالمناسبة، كان فاينمان هو الذي ادعى أنه لا أحد يفهم الكميات. أعلم أنه من الملائم الاعتقاد بأن هناك وصفًا بسيطًا ومنطقيًا وبديهيًا لكل شيء - لكن الأمر ببساطة ليس كذلك. يمكن أن يستمر يهودا في اختراع النظريات حتى نهاية الزمان، ولن يتساوى أبدًا مع نظرية علمية حقيقية مبنية على بحث ومعرفة حقيقيين.

  900. حول Into وItcia وحول الرياضيات بشكل عام
    وبما أن عمليات الفاتورة المتطابقة يمكن تطبيقها بنجاح متسق على أشياء مختلفة تمامًا (مثل الوقت والمسافة والوزن والمال في محل البقالة والمال في البنك وما إلى ذلك)، فيمكننا أن نستنتج أن كل هذه الأشياء هي في الواقع شيء واحد. ولكن بما أننا لم نتمكن من رؤية هذه الهوية بأعيننا، فقد صاغنا اسم "الرياضيات" ونفترض، دون مبرر - وبغض النظر عن التبرير التجريبي، أن نفس قوانين الرياضيات تنطبق على جميع الأشياء في العالم.
    ومع مرور الأجيال اكتشفنا أن الأمور أكثر تعقيدا وأن هناك أشياء لا يشبه سلوكها العددي الكمي سلوك الأشياء الأخرى تماما. وحتى لا نخترع قوانين رياضية جديدة، قررنا أن نقول إن الظواهر الجديدة ليست سوى ظواهر قديمة على مستوى أكبر من التعقيد. وهكذا، على سبيل المثال، فإن النظرية النسبية ليست فيزياء جديدة ولكنها امتداد للفيزياء النيوتونية (أو الفيزياء النيوتونية هي فيزياء أينشتاين مع الأعضاء المتحللة).
    الآن، عندما نأتي لمناقشة التشابك الكمي، نحن على مفترق طرق. فهل هذه ظاهرة جديدة تماما ولا تخضع لقواعد الرياضيات أم أنها في الواقع امتداد لظاهرة أخرى معروفة، وكل ما علينا فعله لفهمها هو مجرد إضافة معادلات تكون في مواقف أخرى منحلة.

  901. في الثانية صباحًا أرسلت ردًا للطالب. لماذا لا تزال تنتظر الموافقة؟
    سابدارمش يهودا

  902. طالب علم
    لا توجد مشكلة في التشابك - المشكلة في عدم المحلية. كيف يؤثر انهيار الدالة الموجية عند النقطة A على انهيارها عند النقطة B، على مسافة كبيرة، في زمن الصفر. تضعف الموجات الكهرومغناطيسية مع المسافة، وكذلك الدالة الموجية. ومن الناحية النظرية، يمكن أن يؤدي انهيار إلكترون واحد إلى انهيار الإلكترونات في الكون بأكمله. فإذا افترضنا أن الانهيار يستهلك طاقة قليلة، فأين قانون حفظ الطاقة؟

    من فضلك، لا تأخذ يهوذا وردود أفعاله باستخفاف. وبعيدًا عن الشبكاتي (انظر الابتهاج العام بعودته في هذا المقال)، فهو (على الأقل بالنسبة لي) اختبار للانحراف عن الحدس، الذي هو أساس البصيرة الجسدية وفقًا لفاينمان. إذا حاولت ضرب 9993 في 10007، فلديك عدة خيارات. إذا كان لديك حدس رياضي مثل مايكل، ستلاحظ أنه (7 +10,000) (7- 10,000)، وبما أن (A+B)(AB) يساوي A^2 – B^2 إذن نحصل على 100,000,000-49 وهو 99,999,951 . خلاص الله في طرفة عين. سيستخدم معاصروه الآلة الحاسبة، وسيستخدم معاصروني اللوغاريتمات. سيخبرك يوفال وفاينمان أنهما لا يفهمان ما هي المشكلة، خذ 10,007 براميل، واملأ كل منها بـ 9,993 جول، واخلط كل شيء ثم قم بالعد. بسيطة جدًا، ما القصة على أية حال؟ وبالطبع هم على حق. خلاصة القول، بعيدًا عن حيل الرياضيات، الضرب هو عملية مخطط.

    لذلك، لكي لا أبتعد عن الحدس إلى "المجالات المركبة" و"الفضاءات الفائقة متعددة الأبعاد"، لشرح ما يحدث للإلكترونات البسيطة والمحبوبة والمعيشية اليومية، فإنني على الأقل أستمع دائمًا إلى أولئك الذين لديهم حدس فيزيائي بسيط، كما في تجربة أمريكية ، هيئة المحلفين هي التي تقرر وليس قاضيًا خبيرًا كما هو الحال في إسرائيل.

  903. طالب التخنيون
    لا أعتقد أنني بحاجة إلى تعلم كل الأشياء غير المقبولة بالنسبة لي
    أنت، على سبيل المثال، هل ستدرس عشر سنوات من الدراسات الدينية قبل أن تقرر ما إذا كان هناك إله أم لا؟
    أفهم أنهم في بلدكم سيصوتون للكنيست في انتخابات حرة، نعم، ولكن فقط أولئك الذين لديهم دكتوراه في المواطنة، لأن الآخرين ليس لديهم المعرفة الكافية بالموضوع!
    نحن نعيش في عالم لا يمكننا فيه أن نعرف بشكل كامل سوى جزء صغير فقط، لذلك يجب علينا الاستفادة من هذا لتحديد موقف في دراسة المشاكل التي تهمنا.
    لا أدعي إبداء الرأي في أشياء لست على دراية بها ولا أجيب مثلا عن التشابك الكمي أو نظرية الأوتار.
    تعلمت في السنة الأولى من المدرسة الثانوية أنه إذا كانت القياسات غير متطابقة مع الصيغة، فمن الضروري البحث عن صيغة أخرى.
    لماذا لا نفعل هذا بخصوص الجاذبية؟، لماذا نحاول أن نثني القياسات بكل أنواع الإضافات المظلمة؟، خائفين من نيوتن؟
    باختصار، السنة الأولى من المدرسة الثانوية كانت كافية بالنسبة لي، لا أحتاج إلى دراسة مقال مظلم لمدة عشر سنوات.
    إذا كنت تريد، اذهب إلى http://www.youtube.com/watch?v=kAo5BQQpBqQ
    سوف تحصل على شرح لمدة 13 دقيقة لرأيي في الموضوع، وليس عشر سنوات!
    مساء الخير
    سابدارمش يهودا
    مساء الخير
    سابدارمش يهودا

  904. ولن أتسرع في القول إن "هذا هو اللغز الأكبر الذي يقف في طليعة الفيزياء". وهذه بالفعل "ظاهرة متناقضة" حيث يحدد القياس نفسه الحالة الكمومية. ويعتمد أحد تفسيرات هذه المفارقة على انهيار الدالة الموجية، والانتقال من الحالة الكمومية التي يوصف فيها النظام على أنه مزيج خطي لعدد من الدوال الموجية إلى حالة كلاسيكية حتمية. أثبتت التجارب التي تم إجراؤها صحة المفهوم الكمي (وتعارض مع متباينة بيل)، حيث لا توجد خصائص مثل الدوران في أي واقع موضوعي، والقياس هو الذي يعطي الهوية.
    إن انهيار الدالة الموجية هو بالفعل موضوع مفتوح في نظرية الكم، لكن لا يمكنك أن تتوقع من يهودا، الذي لا يفهم ما يدور حوله، أن يحل مثل هذه المشكلة أو يفكر في حل شرعي.

    وبالمناسبة، لا ينصح بالاعتماد على ما هو مكتوب في ويكيبيديا (وليست العبرية بالتأكيد) في القضايا على مستوى التشابك الكمي. المؤلفات العلمية والمقالات هي العنوان.

    يهودا،
    سؤال: أنت لا تعرف المفاهيم التي تمت دراستها في Quantum 1، ولكنك تحاول إيجاد بديل لواحدة من أكثر النظريات تعقيدًا في الفيزياء اليوم. الثقوب السوداء، المتفردات، الانفجار الكبير، الكتلة المظلمة، وأكثر من ذلك بكثير. ألا تظن مثلاً أنه من المستحسن تعلم أسس الفيزياء الحديثة قبل التوجه إلى حل واحدة من أعقد المسائل التي تتناولها؟

  905. أنا لست على دراية بكل تعقيدات نظرية الكم، لذا فإن المفاهيم التي تستخدمها غريبة بالنسبة لي.
    لكن، إذا قمت بتعريف المشكلة كما حددتها، فلا أعتقد أنه يمكن أن يكون هناك احتمال لتأثير متزامن على بعد ملايين السنين الضوئية. إذا كانت هناك تجربة توضح ذلك، فأنا أحب رؤيتها وإيجاد حل لها.
    لا يمكن أن تكون هناك أفكار إلا إذا تحرك جزء من الكون بسرعة أكبر من سرعة الضوء. على سبيل المثال، أصغرها هي أكثر من المانتراين. سوف يقومون ببناء نظام من القواعد التي تعمل بشكل أسرع من سرعة الضوء. إن النظرية المبنية على أساس الجسيمات الكبيرة لا يجب أن تكون صالحة بالنسبة للجسيمات الصغيرة. على سبيل المثال سرعة الضوء الثابتة.
    على سبيل المثال، أنا لا أقبل أن سرعة الضوء هي كمية ثابتة في الكون، وأعتقد أنها تتغير ببضعة ملليمترات في الثانية في السنة، وربما حتى سنتيمتر واحد في الثانية. لم تتمكن تجربة مايكلسون-مورلي من إثبات أن سرعة الضوء ثابتة كدالة للزمن بشكل كامل، لأنها قامت بقياس السرعة عند نقطة زمنية واحدة فقط. قياس في نقطتين زمنيتين والمقارنة. أعتقد أنه سيتم العثور على أن السرعة تتغير.
    لا أعتقد أنه من الممكن التوصل إلى استنتاجات متطرفة من خلال اختبار نظرية على نقاط غير متطرفة. على سبيل المثال، من سلوك صيغة نيوتن في النجوم إلى الوصول إلى استنتاجات في الثقوب السوداء. يتم تعريف النظرية فقط حيث تم اختبارها، وحتى هناك يتم تعريفها مع قدر معين من عدم اليقين الموجود دائمًا في القياسات.. وفي النقاط التي لم يتم قياسها فيها، لا يمكننا إلا أن نفترض أنها صالحة.
    يكفي حاليا
    مساء الخير
    سابدارمش يهودا

  906. نعم. أنا أفهم الحاجة إلى الأمن. مجرد سؤال صغير، لمعرفة ما إذا كنا نسير على نفس المسار: هل يسلط حلك الضوء على القسم 6، اللامكانية في التشابك الكمي؟ ففي النهاية، هذا هو اللغز الأعظم الذي يقف في طليعة الفيزياء: كيف يمكن أن يؤثر دوران الإلكترون A على دوران الإلكترون B على مسافة مليون سنة ضوئية، وذلك في زمن صفر.

    أنا لا أطلب حل اللغز بالطبع. فقط برمجة الحل، بدون كلمات مثل "27 بعدًا" أو "الوعي" أو علم الزبيريرولوجي الكوني التجاوزي. على أية حال، ماذا يحدث هناك؟

    إذا كان الأمر صعبًا عليك، اسأل داجاني، فهو يعيش في زاوية يهودا هاليفي.

  907. إسرائيل
    الحقيقة أنني لا أرغب في نشر حل مشكلة الاحتكاك (EA) إلا في المقالة.
    كما تعلم، غير مسموح لي هنا.
    سأحاول في مكان آخر ولكن الأمر سيستغرق بعض الوقت.
    ربما أنشره في النهاية على اليوتيوب ولكنه يتطلب التحضير.
    في هذه الأثناء، أنا مشغول، لذا في هذه الأثناء يمكن لـ Le Sage وRichard Feynman أن يستريحا في أسرتهما دون أن يشعرا بالحماس الشديد.
    سنقوم أيضًا بدراسة المبادئ من 1 إلى 15 ونرى ما يمكن أن تقدمه النظريات المختلفة.
    سيكون لدي عطلة نهاية أسبوع مزدحمة
    يوم جيد
    سابدارمش يهودا
    .

  908. يهودا.
    1. فأين هو حل الاحتكاك؟
    2. من كلامك: "أذكر أنه كان هناك شرح لعزم الدوران ومبدأ الجيروسكوب، ولكني كنت أراه دائما معجزة".
    فكيف فجأة "لا تفهم ما هي المشكلة"؟
    6-3 كما قلت، ليس لديك تفسير.
    7. هاه؟ هل لديك تفسير؟ شارك إسرائيل على الفور! وأينشتاين أيضا. إذا لم يكن هناك تفسير، وإلا فإنه لن يذكر ذلك كمسلمة، بل يشرحها بنفسه.

    14-8 لا يوجد كثيرًا في صنابير الكتلة المظلمة، فهو بالأحرى مجال من روافد البانثيون. لكن يبدو لي أنك إذا تمكنت من إثبات دفع الجاذبية، فإن كل القوانين ستتغير، وسيأخذ الكون البسيط مكانه الصحيح في الكون البسيط. مما يعيد 13 خطوة إلى 1: أين يوجد حل الاحتكاك؟
    15. هنا نقطة اتفاق.

    الطبقات هي لعبة. سوف نراك راكبًا في الحقول على حصان، دون سرج، واقفًا.

  909. 1 تعليقاتي على أقسام إسرائيل السبعة:
    1. الجاذبية. - إن دفع الجاذبية يفسر بشكل أساسي ما إذا كان لدي حل للاحتكاك
    2. الجمود. إذا كنت تقصد قانون نيوتن الأول فأنا أتقبله وعيني مغمضة، لا أفهم ما هي المشكلة
    3. المغناطيسية: لم أهتم
    4. الجذب والتنافر الكهربائي - كما ذكرنا سابقاً
    5. الموجات الكهرومغناطيسية - كما سبق
    6. اللا محلية في التشابك الكمي.- لا أعرف ما هو المقصود
    7. ثبات سرعة الضوء في أي نظام مرجعي، بدون مسلمة. - تفسير الاحتكاك في دفع الجاذبية قد يفسر السرعة المتطابقة في كل اتجاه.

    بالإضافة إلى ذلك، يجب شرح الظواهر التالية والتحقق من مدى ارتباط شفرة أوكهام بكل من التفسيرات المقترحة:
    8 الحركة الغريبة للمجرات
    9 الحركة الغريبة لمجموعات المجرات
    10 إيدوش
    11 التوسع المتسارع للكون
    12. تجنب مفردات الانفجار العظيم وغيرها أو قبولها
    13ـ طريقة توزيع الشرح في المجرة والكون عشوائية أو منطقية.
    14- خواص جسيمات الكتلة المظلمة مقابل جسيمات الجاذبية الدافعة
    15 احتمالا للتفنيد وفقا لبوبر

    ملحوظة: أوكهام هو اسم مكان في اسكتلندا وتم تسمية ماكينة الحلاقة باسم المكان. كان مبتكر ماكينة الحلاقة، إذا لم أكن مخطئا، يدعى ويليام، وهو راهب زاهد على نحو غير عادي حتى أن البابا في ذلك الوقت فرض عقوبات عليه

    مساء الخير
    سابدارمش يهودا

  910. في الواقع، هكذا هو نموذج. ولكن لكي لا يبقى في عالم الصفر أو لا شيء، فهو يحتاج إلى إحدى الاثنين: معادلات ساحقة لا لبس فيها، شيء على طريقة ماكسويل، أو تجربة ناجحة، شيء على طريقة هيرتز. صحيح أن نموذجك تمت صياغته بشكل صحيح ومن A، بينما أقول: خذ ماكسويل، أدخل التغيير الصغير والضروري الذي ذكرته، واتعجب وأتعجب، ها هي ثوابت سرعة الضوء في أي إطار مرجعي. ليست هناك حاجة لنسخ النظرية بأكملها، بمعادلاتها الـ 165.

    http://en.wikisource.org/wiki/On_Physical_Lines_of_Force

    (بالمناسبة، انتبه إلى المعادلات 130-140. نحن نتعامل معها).

    الشيء نفسه بالنسبة لماخ لا سيج. هل لاحظتم كيف يتم حل مشكلة الاحتكاك بطريقة أنيقة في Le Sage؟ لقد وعدنا يهودا بحل كوني بسيط. أين هو؟

    محاكمة. ألا ترى أنني أحاول بكل الطرق، بما في ذلك الكتابة في هذا الموقع، التخطيط للتجربة؟ ولكن ماذا تفعل ليس بالأمر السهل؟ ومع ذلك، نظرًا لأن المشكلة تتعلق بشكل أساسي بالتقنية/الميزانية، وليست نظرية، فيبدو لي أنه سيتم تنفيذ التجربة في النهاية، على الرغم من أنني كواقعي، أعطيها فرصة ضئيلة للنجاح.

    لاحظ أنني أطلب القوة دائمًا وليس الحنان. أحتاج إلى مراجعة النظراء، ولكن بشكل واقعي وفردي، وليس تبسيطيًا.

    طاب يومك.

  911. حتى صبري ينفد أحياناً.

    تتجلى المادة المظلمة، حتى يومنا هذا، في ظاهرتين. الأول، "الغراء" داخل المجرة الذي أشرت إليه في كلماتك.
    والثاني هو التبريد الجاذبي، الذي لم تتحدث عنه.
    إذا كنت تزعم أن تبريد الجاذبية أيضًا لا يسمح بالقياس المباشر، فلن أقول أنك مخطئ. لكن القياس باستخدامه يكون أكثر إلحاحًا من قياس المسار وحسابات السرعة.
    تقول "من الحسابات المخصصة لضبط الجاذبية، يتبين أن توزيع المادة المظلمة يقترب من جزء واحد من مربع المسافة" وهذا بالضبط ما دفعني إلى التساؤل (لم أعد أذكر في أي مقالة) ومتى) إذا كان الأمر مجرد صدفة أو إذا كان مجال الجاذبية ومجال المادة المظلمة متماثلين.
    إلى بيانك "يبدو لي أنك لم توجه الكلمات بعد لأنك لو فعلت ذلك لكان عليك أن تتوصل إلى نتيجة مفادها أن فرضيتك مخفية بالحقائق": إلى تومي اعتقدت أنني وجهت كلامك أكثر من مرة. إذا تجاهلت ذلك، لم يكن ذلك من باب محاولة التهرب. ذكرني فضلا.

    الرد في انتظار الموافقة.

  912. شيما إسرائيل!
    عادةً ما أترك مهمة القص للآخرين الذين يحبونها.
    على العموم، أنا أصر على نموذج يبدأ من الصفر، من لا شيء. وفي الطريق يمكنه زيارة الموقع النشط أو شرب القهوة التركية مع شركة غازكوم في هرتسليا أو حتى الانسجام معهما معًا. لقد صادف أنني قطعت شوطا طويلا معه دون التوقف عند الأصدقاء، لكنني أعتقد أنه لا ضرر من الالتقاء والمقارنة بين الحين والآخر.

  913. يهودا،
    شكرا على التوضيح. في ظل الظروف الحالية، أفضل الخير، ولكن في هذه الأيام المضطربة سأقبل أيضًا السيئ بالحب السيئ.
    هل تقبلون إجابة إسرائيل؟ يسأل عن نموذج يشرح سبعة أشياء. هل لديك واحدة لدي نموذج يشرح، في الوقت الحالي، ستة منها فقط، لأنني لم أتعمق في التشابك الكمي، ولكن من المحتمل أن أتغلب على هذه العقبة أيضًا.
    سوف ننهض وننجح

  914. يوفال
    سأكون سعيدًا بتوحيد الجهود إذا شرحت لي كيفية استيفاء الشروط التالية في المادة المظلمة أو الغازكوم:

    1. الجاذبية.
    2. الجمود.
    3. المغناطيسية.
    4. الجذب/التنافر الكهربائي.
    5. الموجات الكهرومغناطيسية.
    6. عدم المحلية في التشابك الكمي.
    7. ثبات سرعة الضوء في أي نظام مرجعي، بدون مسلمة.

    لأنه يبدو لي أنني شرحت لك بشكل كافٍ كيف يتم كل ذلك على الموقع النشط، وبطريقة بديهية. انتظرت كساح عبثا.

    باستثناء هذه الشروط، أنت على حق 100 في المئة. جميع النماذج الثلاثة متطابقة.

  915. إسرائيل ويهوذا,
    لماذا ذهبنا جولة وجولة؟ بعد كل شيء، مادتي المظلمة، والأثير النشط وغازك هم نفس الجزيئات تمامًا. نحن لا نتجادل حول الجوهر بل حول الطريقة. فلنوحد قوانا، ولنتكاتف جميعًا يدًا واحدة ونعيد حقوقنا كما كانت من قبل حتى تظهر عدالتنا وحكمتنا في الكثير، ولن نفكر في أي حرب أخرى.
    ومع شعار اتبعوا الشمس إلى هرتسليا والحصان إلى كريات عنافيم سنغزو عالم الفيزياء.
    بالمناسبة، كيف تنطق "الطبقات"؟ في المعطف أم في المعطف؟

  916. يوفال
    ما بال نيكيبا؟، بريسيا؟؟؟، كنا في الحي نسميها "الطبقات" وكنا نتنافس على أنجح الطبقات.
    إلى إسرائيل
    مثال الحصان ليس جيدًا جدًا، ولكن إذا حاولت تحويل الحصان فأنا متأكد من أنه سيقاوم.
    وهذا يثبت فعلياً أن الخيول لديها مقاومة للكتلة المظلمة!. ولكن هذا ليس هو الحال مع الدجاج الذي يقلبه عشية الغفران في ممارسة "إنها كفارة" وبالتالي يساهم في الجزء المظلم من الكون.
    أما بالنسبة للحمير فالأمر ليس كذلك لأنها لا تتحرك حتى وهي واقفة، وهي تتعارض تمامًا مع قانون نيوتن الثاني.
    مجرد معلومات شخصية.
    T.L.H.
    ولكن ما أجمل أن تشرق الشمس فوق هرتسليا!
    يوم جيد
    سابدارمش يهودا

  917. ليس مجرد ضربة. أعتقد أن يهودا يبحث عن شيء أكثر بديهية. إذا أخذت BSA ووضعته مع عجلة القيادة في الهواء الحر دون دوران، سترى أنه من السهل جدًا إمالته من جانب إلى آخر. ضع البنزين ووصل إلى سرعة عداد السرعة 200 كم/ساعة، وسترى أنه سيكون من الصعب جدًا عليك تحريكها جانبًا، على الرغم من أن سرعة الدراجة النارية 0 بالنسبة إلى الأرض. سيحدث حتى بدون هواء. ما الذي يمسك عجلة الغزل بهذه القوة القوية ويمنعها من الميل؟ بعد كل شيء، ما هو القصور الذاتي والتمعدن؟ لماذا يحدث هذا بقوة أكبر مع الماء؟ ألا يثير الشك وجود جزيئات صغيرة من جميع الاتجاهات مثل جزيئات الهواء أو الماء؟

    ولم يكن لدينا في المزرعة دراجة نارية، ولكن كان لدينا حصان مع ما لا يقل عن 3.5 من قوات جيش صرب البوسنة.

  918. إسرائيل،
    أنت لم تخبرنا القصة كاملة عندما تحاول إمالة محور العجلة أثناء دورانها، فإن الميل "يبتعد" بزاوية قائمة. وهذا ما يسمح، على سبيل المثال، بتوجيه الدراجة النارية عن طريق إمالة الجسم. لها اسم. هل تتذكر ما هو؟
    (الجواب رأسا على عقب)

    هيستسارب زيلفو هيبكين
    (http://en.wikipedia.org/wiki/Precession)

  919. خذ عجلة ذات مسامير (مسامير) وضعها في الماء. طالما أن العجلة لا تدور، يمكنك إمالةها بسهولة. بمجرد أن تبدأ في الدوران، ستحتاج إلى الكثير من القوة لإمالتها. ما الذي تغير؟ ما الذي يؤثر على قوة الاستقرار على العجلة؟ ما هو القياس؟

  920. إسرائيل يا صديقي
    لكن الدراجة النارية تكون مستقرة فقط أثناء القيادة وكما تعلم، حتى في حالة السكون تصطدم الجزيئات بالدراجة النارية باهظة الثمن ولكنها لن تكون قادرة على الوقوف بثبات وكل طموحها هو الدوران والتحرك. لذا فإن تفسير الجسيمات ليس هو الحل هنا ولا تساعد جزيئات La Sage.
    إذا كان الأمر كذلك، فيجب علينا أن نحاول التفسير باستخدام النظرية المنافسة - الظلام.
    لنفترض أنه أثناء الحركة، يتحول جزء من الحركة الدورانية إلى كتلة داكنة تغلف الدراجة النارية بالمودة ولا تتركها تسقط. يبدو لي أن هذا يمكن أن يكون الجواب. وحتى هناك دليل على ذلك:-
    عندما لا تتحرك الدراجة النارية، تختفي الكتلة المظلمة وتسقط. دليل أم لا؟
    مساء الخير
    سابدارمش يهودا

  921. إسرائيل:
    لقد عدت شخصيا للتعليق، وذلك ببساطة لأنه كان لدي بعض وقت الفراغ.
    لقد كنت مشغولًا جدًا مؤخرًا وفي الوقت المتاح لي أجيب على التعليقات الموجهة إلي في المقالات السابقة ثم أتصفح المقالات الجديدة واحدًا تلو الآخر، وأقرأ التعليقات عليها وأتحقق مما إذا كان لدي أي شيء لأقوله.
    على الأقل بالنسبة لي - لا علاقة له بالكتلة المظلمة.

  922. يا أخي، يا لها من متعة، يا لها من حنين!

    كل الرفقة القديمة والجيدة منذ فترة طويلة في مقال عن علم الكونيات. يودا على حق، لا يوجد شيء مثل الكتلة والطاقة المظلمة لجذب ردود الفعل وجمع القلوب معًا. من المفقود؟ ر.ح. مسار! كالعادة في تشارلز. دعونا نسميه، النبوش! أنا على استعداد للقفز رأسا على عقب! وعلى الفور نقرأ، وبعدكم جميعا نردد:

    ر.ح. يعود!

    يهودا.

    ولا عجب في ذلك، إذا فكرت في جزيئات تتحرك من جميع الاتجاهات وتصطدم بالدراجة النارية. إذا كنت في شك، تخيل جسيمات أثقل وأسرع، وسترى أنه يمكنك الاستقرار عند سرعة أقل مما هو ممكن بالفعل.

    في الواقع التكافؤ في أفضل حالاته!

  923. متى
    كانت هناك أوقات!
    وما زلت يسيل لعابي عندما أجد هارلي ديفيدسون.
    وفي جفعتايم منذ حوالي شهرين رأيت واحدة جميلة كهذه في شارع رمبام.
    وإسرائيل فيما يتعلق بسؤالك
    السبب الوحيد الذي يجعلني لا أسقط من على دراجة نارية هو أن زوجتي لن تسمح لي بركوب واحدة.
    أتذكر أنه كان هناك شرح لعزم الدوران ومبدأ الجيروسكوب، لكنني كنت أعتبره دائمًا معجزة.
    على الأقل يمكن رؤية هذه الأعجوبة، على عكس عجائب الكتلة المظلمة والطاقة.
    (:))
    مساء الخير
    سابدارمش يهودا

  924. اليوبيل:
    وهذا ليس مرجعا بل تجاهلا.
    إنهم لا يقيسون المادة المظلمة بشكل مباشر لأنهم لا يعرفون كيفية القيام بذلك.
    يتم قياس الجاذبية.
    وتلاحظ جاذبية الأرض وهي تسير على جزء واحد من مربع المسافة.
    كما يتم ملاحظة جاذبية المجرة وبما أنها لا تتوافق مع قانون أحد أجزاء مربع المسافة، فمن المفترض أن هناك مادة مظلمة.
    ومن الحسابات المصممة لضبط الجاذبية، يتبين أن توزيع المادة المظلمة يقترب من جزء واحد من مربع المسافة.
    يبدو لي أنك لم تتناول الكلمات بعد لأنك لو فعلت ذلك لا بد أن تتوصل إلى نتيجة مفادها أن فرضيتك مخفية بالحقائق

  925. مايكل،
    معظم "الجدل" بيننا يدور حول الدلالات. في جوهر الأمر، ليس لدي أي جدال معك ومن العار أن نضيع الوقت في لا شيء.
    وأما الادعاء الذي لم أجب عليه "فأن كل جسم يخلق مجال جاذبية بينما التوزيع الذي تمت مناقشته للكتلة المظلمة موجود فقط حول مركز المجرة":
    مع أنني لم أشر إليها بشكل مباشر، لكن بشكل غير مباشر نعم. وأكرر وأدعي أن الأجهزة التي لدينا اليوم ليست حساسة بما يكفي لقياس وجود الكتلة المظلمة بتركيزات صغيرة.

  926. يهودا! مرحباً بعودتك، اشتقت لك!!!
    جيش صرب البوسنة هاه؟ شيء كلاسيكي
    أعيش الآن في ألمانيا واشتريت نورتون قديمًا للعبة بالطبع
    وكان عندي Royal Enfield 62 الذي قمت بتجديده

  927. يوفال:
    وكما قلت، فإن تركيز المادة المظلمة لا يُستنتج من التأكيد على عدم وجود مادة مظلمة في مجرتنا.
    على العكس من ذلك، إذا قرأت الرابط الذي قدمته بخصوص مجرة ​​درب التبانة، فسوف ترى أنهم يعتقدون أن هناك الكثير من المادة المظلمة في مجرتنا.
    وهذا أيضًا يجيب على ادعاءك بشأن اشتقاق النسبية مما يحدث في مجرتنا. هذا ادعاء كاذب تمامًا، رغم أنه كان صحيحًا - عندها فقط سيكون هناك أساس لمحاولتك مقارنة مجرتنا بمجرات أخرى.
    وما يحدث في مجرتنا يتناقض أيضًا مع النظرية النسبية إذا لم نفترض وجود الكتلة المظلمة فيها.
    بالإضافة إلى ذلك - لا يوجد فرق كبير بين "لا توجد كتلة مظلمة" و"هناك القليل جدًا بحيث يستحيل اكتشافه".
    ولو كانت الكتلة المظلمة هي سبب وجود الجاذبية، لتفككت المجرة حتى لو كانت الكتلة المظلمة "قليلة جدًا".
    أفترض أنه من الواضح لك أنك لم تجب على حجتي فيما يتعلق بحقيقة أن كل جسم يخلق مجال جاذبية بينما التوزيع الذي تمت مناقشته للكتلة المظلمة موجود فقط حول مركز المجرة - وهذا وحده يشكل تناقضًا ساحقًا مع فرضيتك وفي الواقع لم تكن هناك حاجة إلى حجة أخرى، ولكن، كما رأيت، هناك العديد من الحجج الأخرى.

    إذا كنت تريد حجة أخرى فإليك حجة أخرى:
    الجاذبية تتناسب عكسيا مع مربع المسافة فقط .......
    .......
    ......
    في غياب الكتلة المظلمة.

    وفي الواقع تم إضافة الكتلة المظلمة لأنه بحسب الملاحظات فإن الكون لا يتصرف وفق هذا القانون!

    لم أوجه كلامي حول الحجة ضد "طريقة تحطيم الجسيمات" إليك. وجهتها إلى المطالبة نفسها.
    انها ليست شخصية. كنت أعرف أن ياعيل قد عبرت بذلك لأول مرة، لكن في رأيي هو ادعاء استهزاء لا مكان له وشرحت السبب.

    יעל:
    قرأت الكثير من كلمات إيلام جروس.
    لدينا أيضًا أصدقاء مشتركون.
    لا شيء من هذا يقودني إلى استنتاج أن الطريقة الوحيدة التي نعرفها لاكتشاف الجسيمات الأولية الموجودة هي طريقة متخلفة.
    إذا اخترعت طريقة أخرى لجذب الجسيمات إلى خارج الهياكل الأكثر تعقيدًا، فقد يكون هناك بعض الأساس لادعائك، ولكن حتى في هذه الحالة سيكون أساسًا هشًا.
    وكما تظهر التجارب حتى الآن - هناك جسيمات أساسية ذات عمر افتراضي قصير جدًا - وهي ليست على الإطلاق أحد مكونات أي جسيم يمكنك محاولة تفكيكه.
    بشكل عام - ما هي المشكلة وما هو الشيء السيئ؟
    وهذا ليس أمراً مطلقاً، بل نتيجة المقارنة بين الأشياء.
    على سبيل المثال - لا يقال عن الشخص السليم أنه مريض رغم أنه لا يستطيع الطيران.

  928. السيد مايكل روتشيلد،

    إن "التحطيم" هو أسلوب سيئ للغاية، بل إنه تقريبًا الأسلوب الوحيد المتاح اليوم. هل تعلم لماذا يصعب العثور على "هيغز"؟ هل تفهم أنهم يحاولون بالفعل إنشائه وليس استخراجه؟ هل تفهم لماذا احتمال العثور عليه منخفض جدًا؟ بعد كل شيء، يجب أن تنتج التجربة نفس النتائج طوال الوقت حتى تكون موثوقة... لماذا لا يحدث هذا في أبحاث الجسيمات؟

    أنصحك بقراءة القليل من كتابات البروفيسور إيلام جروس (على الرغم من أنك قد قرأت بالفعل)، هناك مشكلة تكنولوجية كبيرة في هذا البحث الأساسي.

  929. إسرائيل
    من المسلم به أنني لا أعرف المقطع أعلاه الذي كتبه كوبولا (ثغرة في التعليم)، ولكنه يذكرني بالدراجة النارية الإنجليزية التي كنت أملكها في أوائل السبعينيات، وهي BSA ("الديك")، بقوة 3.5 حصان، والتي كانت تتسابق في الظلام الحقول، مع صديقتي (زوجتي المستقبلية) الجميلة.
    لا كتلة مظلمة، وتركت الطاقات المظلمة للآخرين.
    ولكن، كانت هناك أوقات!
    يوم جيد
    سابدارمش يهودا

  930. يودا

    يا رجل، اعتقدت أنك غادرت إلى الأبد

    (ألعاب صدئة لصبي الدراجات النارية، كوبولا 1983).

  931. شكرا مايكل.

    كنت أتحدث عن تركيز المادة المظلمة الذي يختلف من مجرة ​​إلى أخرى. كان هذا رداً على قولك أن هناك مجرات لا تحتوي على مادة مظلمة، وأقصد أن أقول أن هناك مجرات يكون تركيز المادة المظلمة فيها صغيراً جداً بحيث لا نستطيع اكتشافها. إن أدوات الرصد التي لدينا ليست حساسة بما يكفي لتزويدنا ببيانات دقيقة لتحديد كمية المادة المظلمة وتركيزها، ولهذا السبب أشك في مقولة أن هناك مجرات لا تحتوي على مادة مظلمة. لاحقاً قدمت عرضاً متطرفاً، مفاده أنه ليس لدينا مادة مظلمة، لأبين سبباً افتراضياً لوجود مجرات تبدو لنا خالية من المادة المظلمة بسبب سلوكها (أي سلوك المادة المظلمة فيها) هو مثل لنا.
    وُلدت النظرية النسبية في مجرتنا ولم يتم اختبارها عمليًا إلا في نظام أحد نجومها. صيغها صحيحة هنا فقط (وهذا أيضًا بضمان محدود). ولذلك، فمن الواضح أن النظرية النسبية لكل مجرة ​​تتضمن صيغًا وثوابت فريدة خاصة بها.

    قبل ساعات قليلة طرحت "سؤالًا كاملاً" هنا فيما يتعلق بالخصائص البصرية لعدسات الجاذبية واستخداماتها المحتملة. وحتى الآن لم يقم أحد برفع القفاز.

    وكلمة "ازدراء" هي اقتباس من كلام ياعيل. لقد أحببته بسبب الشعرية الموجودة فيه (وأيضًا لأنه أعطاني فرصة للقليل من التفاخر). يرجى توجيه رأيك لها.

  932. اليوبيل (https://www.hayadan.org.il/astronomers-reach-new-frontiers-of-dark-matter-130112/#comment-323446):
    1. كتبت في ردي السابق أن كمية المادة المظلمة تختلف من مجرة ​​إلى أخرى، لذا من الواضح أنني أريدك أن تخبرني عنها.
    2. لماذا يجب أن ننظر إلى مجرتنا على أنها لا تحتوي على مادة مظلمة؟! وخاصة - لماذا نفعل ذلك عندما تظهر النتائج أنه يحتوي على مادة مظلمة؟! http://en.wikipedia.org/wiki/Milky_Way
    3. ما هي "المجرة الأخرى التي تتصرف مثلها"؟!
    4. يتم الاستدلال على وجود المادة المظلمة في المجرات (وكذلك كميتها وتوزيعها) من قياسات المجرات نفسها وليس من المقارنة مع مجرتنا. المقارنة تكون فقط بين النتيجة المقاسة وتوقعات النسبية.

    أنا شخصياً لست متحمساً على الإطلاق للعبارة المهينة "الأبحاث التي تُجرى اليوم حول الجسيمات الأولية هي أبحاث خام وخام للغاية".
    عندما تصنف شيئًا ما على أنه "خام وخشن" فيجب أن يكون ذلك مرتبطًا بشيء أفضل - وهو ما لا نفعله وفقًا للمقارن.
    ليس من المؤكد على الإطلاق أننا سنحصل على طريقة أفضل، وبهذه الطريقة اكتشفنا الكثير بالفعل (ولكن في الحقيقة الكثير!)
    بالمناسبة - هذا لا يختلف جوهريا عن الطرق التي تعتبر الأكثر تقدما لاختبار بنية المادة (على سبيل المثال، باستخدام المجهر الإلكتروني). وأتساءل من يريد تصنيف مثل هذا المجهر على أنه "خشن وخام".
    كقاعدة عامة - كل هيكل اكتشفناه على الإطلاق تم اكتشافه عن طريق تقسيمه إلى أجزائه.
    وفيما يتعلق بالجسيمات الأولية على وجه التحديد، فإن هذا ليس هو الحال تمامًا لأن فعل "التحطم" لا يتحطم على الإطلاق. إنه يأخذ الطاقة الإجمالية للاصطدام ويخلق مجموعة متنوعة من الجسيمات التي في كثير من الحالات لا تكون جزءًا من الجسيمات المتصادمة.

  933. هل لاحظت أن الكتلة المظلمة والطاقة لها القدرة على جذب العديد من المستجيبين؟
    علاوة على ذلك، لقد كنت مشغولاً بشؤوني الخاصة وأرى أنك بخير بدوني.
    لذا…
    تحية وبعد
    وداعا
    سابدارمش يهودا

  934. سؤال تام حول تصريف الجاذبية:
    وبما أن المجرة هي في الواقع عدسة بصرية، فإنها تمتلك خصائص واحدة، على سبيل المثال معامل الانكسار والبعد البؤري. يمكن الافتراض باحتمال كبير أن المجرات المماثلة تشكل عدسات مماثلة. هل من الممكن الاعتماد على هذا الافتراض لطرق حساب مسافات الأجرام السماوية - العدسات المجرية نفسها والأشياء التي تشوه صورتها بها؟

  935. مرحباً السيد يوفال تشيكين،

    وهل من الممكن أن تفعل شيئا حيال ذلك؟ "السنونو" التي رأيتها كانت باللونين الأبيض والأسود، وكانت مصنوعة من الهيدروجين. والافتراض هو أن المستعرات الأعظمية من النوع La تبعث شدة ضوء ثابتة حسب الشدة التي تصل إلينا وبالتالي يمكن معرفة بعدها عنا. وبالتالي فإن تتبع هذه المستعرات الأعظمية يشير إلى سرعة حركتها بالإضافة إلى تسارعها.

  936. السيد يائيل (فيما يتعلق بالسيد [ح] زير، إذا كان هناك واحد هناك 🙂 )
    أعجبني كلامك: "البحث اليوم عن الجسيمات الأولية هو بحث خشن وخام للغاية، وتحطيم الجسيمات لدراستها هو أسلوب مشكوك فيه للغاية. ومن ناحية أخرى، فإن "التلسكوبات" المعتمدة على النيوترينوات أو موجات الجاذبية موجودة على جدول الأعمال في مراحل التطوير"، ومن أجل الفضول سأضيف أنني كل يوم تقريبًا أمر بالمختبر حيث يتم تركيب أجهزة للكشف عن موجات الجاذبية. . إنه موجود في مبنى كلفن (الذي سمي باسمه مقياس درجة الحرارة) في جامعة جلاسكو.
    ومن ناحية أخرى، فإن قولك "خلال الـ 300,000 ألف سنة الأولى بعد الانفجار الكبير، كان الكون منيعًا أمام انبعاث الإشعاع الكهرومغناطيسي، وليس الأمر كذلك بالنسبة للنيوترينوات والجاذبية" يُظهر التزامًا بارعًا ومحافظًا إلى حد ما بنموذج مقبول. لا يعني ذلك أنني أنكره، ولكني أيضًا لا أقبل تأكيده في المسامير.
    الاستمرار في المراسلة على انفراد؟
    ivrit.yuval00@googlemail.com

  937. مايكل،
    أولا، شكرا لاهتمامكم. وهنا فهمان خاطئان حول كلامك:
    1) أظهرت الملاحظات أن تركيز المادة المظلمة يختلف من مجرة ​​إلى أخرى. إذا نظرنا إلى مجرتنا على أنها خالية من المادة المظلمة، فإن أي مجرة ​​أخرى تتصرف مثلها ستعتبر فارغة في أعيننا أيضًا.
    2) تختلف نسبة المادة الباريونية التي يمكننا رؤيتها أيضًا من مجرة ​​إلى أخرى.
    ولهذا السبب يصعب علي قبول استنتاجاتك على أنها مطلقة.

  938. نعم يائيل. أعتقد أنك على حق الآن. تُظهر "توقيعات" المستعرات الأعظم انزياحًا أحمرًا متزايدًا مع زيادة المسافة بين مصدرها وبيننا. لكي لا يبدو الأمر غامضًا، هناك شرح أكثر تفصيلاً: في التوقعات (الطيف)، يتم الحصول على نطاقات الامتصاص في أنماط مميزة تقع في مناطق ذات ألوان معينة. في الضوء القادم من المجرات البعيدة، تظهر نفس أنماط نطاق الامتصاص في المناطق الملونة الأخرى. ومع زيادة المسافة التي يأتي منها الضوء، تزداد أيضًا نطاقات الامتصاص وتبتعد عن موضعها الأصلي. وكنت الحق تعليقاتكم وتصحيحاتكم من فضلكم.

  939. اليوبيل (https://www.hayadan.org.il/astronomers-reach-new-frontiers-of-dark-matter-130112/#comment-323353):
    لقد أجبتك بالفعل عن ذلك!
    لا يمكن أن يكون ذلك لأن هناك مجرات بدون مادة مظلمة وهي مجرات بسبب الجاذبية (وإلا فإنها تتشتت).
    كما تختلف كمية المادة المظلمة من مجرة ​​إلى أخرى بغض النظر عن كمية المادة الباريونية.
    باختصار - إنه لا يعمل على الإطلاق! هذا ليس سؤالاً ولكنه تشخيص واقعي يناقض فرضيتك.

    التشخيص الواقعي الآخر الذي يناقض فرضيتك هو أن توزيع الكتلة المظلمة يكون بحسب أحد أجزاء مربع المسافة فقط بالنسبة إلى مركز المجرة بينما الجاذبية تعمل وفق هذه العلاقة حول كل جزء من الأجزاء. الأجسام المختلفة في المجرة.

  940. السيد يوفال تشيكين،

    لقد أخطأت بالفعل عدة مرات، ولكن ليس هذه المرة. أظهرت ملاحظات "توقيعات" المستعرات الأعظم أن انتشار المادة في الكون يتسارع. وبما أن تسارع الكتلة يحتاج إلى قوة والقوة تحتاج إلى طاقة... اليوم لا يوجد تفسير لماذا هذه الطاقة هي التي تسرع سرعة المجرات التي تبتعد عن أحد مراعيها - هذه هي "الطاقة المظلمة" التي تقصدها.

  941. إسرائيل شابيرا,
    "الموقع النشط" هو المصطلح الخاص بك. على الرغم من وجود شيء "نشط" في النموذج الذي أقوم بإعداده، إلا أنني أحرص على عدم استخدام اسم "موقع" لأنه يستخدم للإشارة إلى نموذج مفرط في التبسيط.

  942. السيد يوفال تشيكين،

    ما كتبته عن الطاقة المظلمة ليس دقيقًا - الانزياح نحو الأحمر يرجع إلى اختلاف السرعة النسبية للأجسام المختلفة، ولا علاقة له بالطاقة المظلمة!

    نشأ الاعتقاد بوجود الطاقة المظلمة من دراسة المستعرات الأعظم من النوع 1A (أو "La"). نتجت الكتلة المظلمة عن "نقص" المادة المرئية لتفسير حركة الجاذبية على مراتب المجرة من حيث الحجم وكذلك من ضغط الجاذبية.

    يجب أن نتذكر أن البحث اليوم عن الجسيمات الأولية هو بحث خشن وخام للغاية، وأن تحطيم الجسيمات لدراستها يعد طريقة مشكوك فيها للغاية. ومن ناحية أخرى، فإن "التلسكوبات" المعتمدة على النيوترينوات أو موجات الجاذبية موجودة على جدول الأعمال في مراحل التطوير. يجب أن نتذكر أنه خلال الـ 300,000 ألف سنة الأولى من الانفجار الأعظم، كان الكون منيعًا أمام انبعاث الضوء والإشعاع الكهرومغناطيسي، لكن الأمر ليس كذلك بالنسبة للنيوترينوات والجاذبية.

  943. R.H. Rafai.M،
    إن القياس المباشر للطاقة المظلمة سيكون دليلا قاطعا على وجودها. ولهذا السبب أنا من وجهة نظرك أنه ينبغي بناء جهاز قياس مناسب.
    "شيء مثل الأثير" هو الوسط الذي يوصل الإشعاع الكهرومغناطيسي. يقوم الوسيط "الخاص بي" بإجراء جميع الظواهر الفيزيائية. نظرًا لأنه، من بين أمور أخرى، ينقل الإشعاع الكهرومغناطيسي والجاذبية، فيجب أن يكون ضوءه أكثر تعقيدًا مما يمكن اكتشافه.

    نقطة،
    اضحك، اضحك، ولكن في يوم من الأيام، سيتبين أنك كنت على حق.

  944. وفقًا لنماذجي في عالم الكتلة المظلمة، يوجد بالفعل مئات من الأرض المظلمة حيث حدثت الكثير من التطورات وتم إنشاء الكثير من الكائنات المظلمة الذكية التي تحب بشكل أساسي رواية النكات السوداء عن بعضها البعض.
    لا بد أنهم يتساءلون أيضًا عن الكتلة المفقودة البالغة 5% والتي لا تناسبهم في الحسابات التي يقومون بها على حافة الظلام.
    وحسب نماذجي نفسها، فإن الشمس بالنسبة لهم مظلمة وتسبب الليل، والثقوب السوداء فقط هي التي تضيء نهارهم الخافت.
    معهم يكون الجذع العلوي أسود أيضًا ويستخدمون مواد تشوه الغسيل حتى يخرج الغسيل نظيفًا.
    بعد تأمل طويل هناك قد تخرج مغشيا عليك ثم يأتي الجميع ليسمعوا ما ستقوله لتسود أعينهم.
    ومن الممكن أن يكون الكون الذي يرونه أكبر بكثير لأنه ربما تكون سرعة الظلام أعلى بكثير من سرعة الضوء ومن ثم يكونون قادرين على عدم الرؤية لمسافة أكبر بكثير مما نستطيع رؤيته.
    في أعياد الميلاد، ربما يطفئون الشموع ثم يضيئونها لتحقيق أمنية.

    ...

  945. يوفال
    أفترض أنك أخذت في الاعتبار شيئًا مثل "الأثير" كبديل للطاقة المظلمة 🙂، هل كنت على حق؟
    على أية حال، في رأيي أن الطاقة المظلمة هي في الحقيقة طاقة (إشعاع). وأنا أتفق أيضًا مع جميع المعادلات ذات الصلة، مثل تلك التي يستنتج منها وجود بعض الطاقة غير العادية، بالإضافة إلى جميع الأنواع الموجودة. أعتقد أننا بحاجة إلى إيجاد طريقة لقياس هذه الطاقة.

  946. مايكل،

    في هذه الأثناء، لم أقم بصياغة نظرية، بل أشرت فقط إلى ظاهرتين متشابهتين كميًا ومحليًا. بعد إذنك، سأعيد كتابة الأشياء مع إصلاح تجميلي بسيط.
    وتحيط المادة المظلمة بالمادة الباريونية، وتتناسب كثافتها عكسيا مع مربع المسافة من المركز.
    كما أن قوة مجال الجاذبية تتناسب عكسيا مع مربع المسافة من المركز.
    يشير هذا المزيج غير العرضي إلى احتمال أن تكون المادة المظلمة ومجال الجاذبية كيانًا واحدًا.

    مناقشة الفرضية (ليست نظرية. مجرد فرضية)، أن المادة الباريونية ليست سوى مادة مظلمة موجودة بتركيز أعلى من مستوى عتبة معين، أنا أتجمد في الوقت الحالي.

  947. ر.ه.رفاعي.م
    لم يتم قياس الطاقة المظلمة على الإطلاق. ولهذا السبب يطلق عليه "الظلام". ويفترض أن وجودها يرجع إلى الانزياح الأحمر للضوء القادم من المجرات البعيدة، وهو التحول الذي يزداد مع زيادة المسافة. يعتمد التفسير المقبول لهذه الظاهرة على افتراضين: الأول، أنه تأثير دوبلر؛ والثاني، أن سرعة الضوء ثابتة في كل مكان (فارغ) في الكون. ويترتب على ذلك أن المجرات لا تتحرك بعيدًا فحسب، بل تتسارع أيضًا. تتطلب الحركة المتسارعة طاقة، لكن النماذج الفيزيائية الموجودة لا تفسر وجودها.
    ورغم أنه لم يتم قياسه، إلا أنه يمكن حسابه.
    وكما ذكرنا فإن وجودها هو نتيجة تترتب على الفرضيتين السابقتين. إذا تم العثور على تفسير آخر، فمن الممكن أن يسقط افتراض الطاقة المظلمة أيضًا.

  948. وهناك سؤال آخر مثل: ماذا قبل ماذا؟ - المادة المظلمة إلى مادة باريونية أم العكس؟ أم أن كلا الأمرين منتج لنفس الطاقة؟ المادة المظلمة لا تتفاعل مع المادة العادية أو الإشعاع المعروف في الفيزياء. ومن ثم، فمن المحتمل أن تكون المادة المظلمة على "مقياس" مختلف عن المقياس الذي يتم قياس جميع أنواع الإشعاع المعروفة عليه.
    الطريقة الوحيدة لاختبار ذلك هي بناء جهاز يعرف كيفية اكتشاف الإشعاع - من ذلك النوع المحدد، وهو غير معروف حاليًا للفيزياء. هذا يعني أنه عليك أولاً العثور على الطاقة نظرًا لوجود علاقة بينها وبين المادة المظلمة. ربما هي الطاقة المظلمة؟ هل يعرف أحد كيفية قياس الطاقة المظلمة؟ 🙂

  949. اليوبيل:
    هذه أسئلة يجب على من يقدم عرضًا مثل عرضك أن يجيب عليها.
    وطالما لم تتم الإجابة على هذه الأسئلة (وقد لا تتم الإجابة على بعضها على الإطلاق!) فلا يوجد ما يدعو لطرح النظرية لأن عدد المشكلات التي تخلقها أكبر بكثير من عدد المشكلات التي تحلها.

  950. هذه في الواقع أسئلة مثيرة للاهتمام للغاية.
    طالما أننا لا نعرف شيئًا عن الكتلة المظلمة (بخلاف الاعتراف بوجودها)، فلا يمكننا الإجابة على هذه الأسئلة.
    والإغراء كبير للبحث عن حل لها من خلال بناء نموذج (على سبيل المثال، الجسيمات) للكتلة المظلمة.
    عند بناء النماذج، يمكنك أن تنطلق وتبتكر العديد من الميزات التي تريدها، بشرط أن يكون المنتج النهائي مشابهًا قدر الإمكان لما ينعكس في الواقع. ليس في السماء ولا وراء البحر. يُسمح لأي شخص بالمحاولة. وطالما أن مثل هذا النموذج غير موجود، فليس هناك ما يمكن استبعاده.

  951. اليوبيل:
    لقد تحدثنا بالفعل عن مسألة المربع وأخبرتك أنه أثار بعض الأفكار في داخلي حول العلاقة بين الكتلة المظلمة والطاقة المظلمة (فكرة شرحتها هنا بشكل عام ولكن لاحقًا عملت أكثر على الصيغ ذات الصلة وتوصلت إلى إلى الاستنتاج الذي يبدو أنه غير صحيح).
    إن فكرة أن الكتلة المظلمة تخلق مجال الجاذبية هي فكرة خاطئة من الأساس.
    كل كتلة لها مجال جاذبية، وهناك مجرات لا تحتوي على مادة مظلمة على الإطلاق.
    فكرة أن كتلة الفتوة هي "كتلة داكنة مركزة" تبدو خاطئة أيضًا.
    كيف يمكن للكتلة المظلمة أن تتركز عندما يكون التفاعل فيما بينها ضعيفًا جدًا؟
    بالإضافة إلى ذلك - كيف يمكن ألا تتناثر "شظايا الكتلة المظلمة" من الاصطدامات في مسرعات الجسيمات؟
    علاوة على ذلك، كيف تصبح الطاقة كتلة إذا لم تكن الكتلة عبارة عن تركيز للطاقة بل كتلة مظلمة.

    وكما قلت - فإن فكرة أن الكتلة المظلمة عنصر مهم في تكوين المجرات هي النظرية المقبولة، ولكن لا ينبغي دفعها إلى أبعد من ما هو ضروري

  952. مايكل،
    ومن الواضح أن هذه ليست مصادفة. لقد كان سؤال بلاغي.
    وتحيط المادة المظلمة بالمادة الباريونية، وتتناسب كثافتها عكسيا مع مربع المسافة من المركز.
    كما أن قوة مجال الجاذبية تتناسب عكسيا مع مربع المسافة من المسافة.
    المزيج ليس من قبيل الصدفة ويشير هذا إلى احتمال أن تكون المادة المظلمة هي التي تخلق مجال الجاذبية.
    علاوة على ذلك، بما أن المادة الباريونية تقع في مركز مجال الجاذبية، حيث يكون تركيز المادة المظلمة في أعلى مستوياته، فيبدو لي على الأرجح أن نفترض أن المادة الباريونية ليست سوى مادة مظلمة بتركيز أعلى من عتبة معينة مستوى.

  953. اليوبيل:
    هذا ليس مصادفة.
    وهذا نتيجة للجاذبية التي تميل إلى خلق تركيزات جماعية ولا تميز بين أنواع الكتلة.
    يعتقد التفكير المقبول حاليًا في العلوم أن المادة المظلمة لعبت دورًا مركزيًا (حرفيًا) في تكوين مجرات المادة الباريونية.

  954. الذات الآخرين:
    لم تقيس هذه الملاحظة التغيرات في توزيع المادة المظلمة، بل توزيعها الحالي.
    كما أنها لم تقيس المادة المظلمة في الكون بأكمله، ولكن فقط في النطاق بين موقعنا الحالي و6 مليارات سنة ضوئية منا (النوع الذي يمكن أن يخلق سحابة جاذبية على ضوء النجوم غادرت قبل 6 مليارات سنة وهي تصل إلينا الآن). .
    في الواقع، الأمر أكثر تعقيدًا من ذلك لأن توسع الكون لا بد أنه قد حرك بالفعل المادة المظلمة التي نرى آثارها المتحللة اليوم.

    فيما يتعلق بديناميكيات توزيع المادة المظلمة، لا يمكننا الحصول إلا على نظريات اليوم لأنه ليس لدينا طريقة لمراقبة المادة المظلمة نفسها.
    جميع المعلومات التي لدينا عن الكون تأتي من خلال الإشعاع الكهرومغناطيسي ويتم استخلاص جزء كبير من هذه المعلومات من الاكتشافات المختلفة لتأثير دوبلر (لأن المسافات كبيرة جدًا بحيث لا يمكن اكتشاف الحركة مباشرة).
    تحتفظ المادة المظلمة فقط بتفاعل الجاذبية مع الإشعاع الكهرومغناطيسي، وبالتالي، من بين أمور أخرى، ليس من الممكن اكتشاف حركتها بناءً على تأثير دوبلر.

    أشباح:
    لا أعرف ما هي المعدات التي تتوقعها.
    ووفقا لتوقعات النظريات العلمية المعاصرة - لا يمكننا أبدا أن نتوقع مسافة أكبر من المسار الذي يمكن أن يقطعه الضوء خلال حياة الكون.

  955. مايكل،

    شكرا على التوضيح، الآن يعمل.

    ماذا عن السؤال الثاني؟
    وسؤال آخر - بحسب المقال، فإن المادة المظلمة/الطاقة المظلمة (احذف ما لا لزوم له، لم أستطع أن أفهم) تم ترتيبها على شكل شبكة في جميع أنحاء الكون منذ 6 مليارات سنة. وهل هناك تغيير في طريقة ترتيبها مع تقدم الزمن؟

  956. ماشيل
    عندما تحاول "اختصار" شرح مثل هذا الموضوع، على الأقل بالنسبة لي، تحدث أخطاء 🙂
    صحيح أنه بمساعدة التلسكوبات يمكنك المراقبة على مسافة تصل إلى 13.7 مليار سنة ضوئية.
    أي أن التكنولوجيا تسمح حاليًا بالمشاهدة لمسافة تصل إلى 13.7 مليار سنة ضوئية فقط. وباعتبار أن الكون يتوسع بمعدل متسارع، فإن ما نراه اليوم على مسافة 13.7 م سنة ضوئية، هو في الواقع 46 م سنة ضوئية. أي أن المسافة الحقيقية (الفعلية) لتلك المنطقة وقت الرصد هي 46 م سنة ضوئية.
    وهذا يعني أنه من الناحية النظرية يمكن ملاحظة حجم X من الكون
    (4.1X10^32 سنة ضوئية مكعبة - حسب ويكيبيديا) لكن من هذا المقدار نرى فقط نسبة معينة (13.7 م سنة ضوئية).
    وفيما يتعلق بالسؤال ما هو سبب التوسع المتسارع للكون؟ لنفترض الطاقة المظلمة.
    ولا يمكن قياس هذه الطاقة، على الأقل حتى الآن. ومن الناحية العملية، يمكن ملاحظة كمية معينة من الكون بأكمله، وهذه الكمية هي X. وبمجرد اكتشاف جسيم من الطاقة المظلمة وبناء الأدوات المناسبة، سيكون من الممكن ملاحظة ما هو أبعد مما يعتبر اليوم "عصر الكون"، سيكون من الممكن، في الواقع، ملاحظة حدود الكون نفسه. كما يبدو متناقضا.
    بعد كل شيء، من الشائع الاعتقاد بأن الطاقة المظلمة موزعة بالتساوي في جميع أنحاء الكون. وإذا كان الأمر كذلك، فسوف تنشأ حالة تكون فيها حدود الكون موجودة حتى بين المسافات الكمومية. إنها مفارقة، أليس كذلك؟
    وهذا يجعلني أعتقد أن عدم اكتشاف جسيم الطاقة المظلمة يحل المفارقة. 🙂

  957. إذن، منذ حوالي 6 مليارات سنة، قام علماء الفلك بقياس تركيز المادة المظلمة وما زالوا لم يتوصلوا إلى نتيجة؟
    ربما يحتاجون إلى 6 مليارات سنة أخرى؟
    تفكير عميق

  958. اصدقاء:
    وكانت أبعد المجرات التي نراها على بعد 13.7 مليار سنة ضوئية عنا في اللحظة التي يخرج منها الضوء الذي ندركه اليوم.
    وهذا ليس مكان وجود هذه المجرات (المجرات الباقية) اليوم.
    الكون في توسع متسارع واستمرت هذه المجرات في الابتعاد عنا.
    كل هذا واضح وسهل حقًا، ولكن الأصعب هو فهم كيف تمكنوا خلال 13.7 مليار سنة من اجتياز مسافة 32.3 (الفرق بين 46 و 13.7) مليار سنة ضوئية؟ ففي نهاية المطاف، فإن السرعة القصوى التي يمكنك التحرك بها لا تتجاوز سرعة الضوء، لذلك يبدو أنه خلال 13.7 مليار سنة من المستحيل السفر مسافة تتجاوز 13.7 مليار سنة ضوئية!
    وهنا يأتي دور توسع الكون نفسه (والذي يتسارع أيضًا، لكن هذا ليس مهمًا لأغراضنا في الوقت الحالي).
    وهذا التوسع (الفضاء نفسه) يمكن أن يتسبب في زيادة المسافة بيننا وبين أي جسم في الفضاء بمعدل يتجاوز سرعة الضوء.

    شبح:
    لا ترى تلسكوباتنا على بعد 46 مليار سنة ضوئية، بل ترى 13.7 مليار سنة ضوئية فقط.
    تم التوصل إلى 46 مليارًا من خلال حسابات معقدة حقًا (ولكنها حقًا!) وتم إجراء كل هذه الحسابات وفقًا لافتراضات النموذج القياسي لعلم الكونيات الذي يميل العلماء إلى الاعتقاد بأنه صحيح ولكن - كما نعلم - لا يوجد "يقين" في العلم. .

    كل هذا بالطبع لا ينتمي إلى المقال الذي يظهر هنا.
    بالإضافة إلى ذلك، على الرغم من أنه يشير إلى الاقتباس الذي جلبته يائيل من ويكيبيديا، إلا أنه لا يشير أيضًا إلى سؤال ي.س. لقد عرف بالفعل أن عمر الكون هو 13.7 مليار سنة (وكتب هذا في متن السؤال)، لذا فإن إجابة يائيل لا تجيب على سؤاله.

  959. المقال غير واضح جداً... ومن التعليقات ارتبكت تماماً، آسف على الأسئلة الغبية:

    هل هناك من يرغب في شرح مقارنة النظرية الرائدة لعمر الكون بقطره وكيف يتطابق الرقمان؟

    وسؤال آخر - بحسب المقال، فإن المادة المظلمة/الطاقة المظلمة (احذف ما لا لزوم له، لم أستطع أن أفهم) تم ترتيبها على شكل شبكة في جميع أنحاء الكون منذ 6 مليارات سنة. وهل هناك تغيير في طريقة ترتيبها مع تقدم الزمن؟

    شكرا.

  960. أبعد النقاط التي يمكن ملاحظتها (بمساعدة التلسكوب) تبعد حوالي 46 مليار سنة ضوئية. في كل اتجاه ننظر. أي أن الاستنتاج الذي تم الحصول عليه من الملاحظات يشير إلى أن الكون على الأرجح "كرة" (شكل "كرة" أي "دائري"). وإذا تم قياس المسافة (الملاحظة) من الأرض، فستعتبر الأرض مركز الدائرة. ومن هذا نستنتج أن نصف قطر الدائرة يبلغ حوالي 46 مليار سنة ضوئية. وبعملية حسابية بسيطة يمكن التوصل إلى استنتاج مفاده أن طول قطر الكون المرئي (الذي يمكن ملاحظته - ولا يمكن ملاحظة مروره) يبلغ حجمه حوالي 93 مليار سنة ضوئية.

  961. جاء في المقال "يتكون الكون من شبكة كونية معقدة من المادة المظلمة والمجرات التي تمتد إلى مسافة تزيد عن مليار سنة ضوئية".

    إنه بالتأكيد ليس خطأً لأن 93 مليارًا هو في الواقع أكثر من مليار، لكنه أيضًا ليس دقيقًا حقًا.

  962. AR:
    أنت على حق.
    وهذا خطأ في الترجمة ويقول المصدر أنه سيتم استخدام المعلومات لدراسة الطاقة المظلمة وليس المادة المظلمة.

    يوجد:
    لا توجد معلومات متضاربة هنا حول عمر الكون.
    لا أعرف ماذا تقصد بالضبط، لكن الأقرب إلى عدم الدقة هو الجمع بين الادعاء بأن الضوء بدأ طريقه قبل 6 مليارات سنة مع الادعاء بأن عمر الكون كان آنذاك 6 مليارات سنة.
    وهذا في الحقيقة غير دقيق إلى حد ما لأنه يؤدي إلى استنتاج مفاده أن عمر الكون هو 12 مليار سنة وليس 13.7 مليار سنة، ولكن من الواضح أن هذا تعبير عن "حول" ولا يمكن اعتبار الاختلاف تناقضا.

  963. هل المادة المظلمة هي سبب تسارع الكون؟
    من المفترض أن تكون المادة المظلمة هي سبب خلق المجرات وتركيزها (وخلق المادة نتيجة لذلك)
    الطاقة المظلمة هي المسؤولة وحكم الكون... وهذان شيئان منفصلان... على الأقل هكذا فهمت الأمر.

ترك الرد

لن يتم نشر البريد الإلكتروني. الحقول الإلزامية مشار إليها *

يستخدم هذا الموقع Akismat لمنع الرسائل غير المرغوب فيها. انقر هنا لمعرفة كيفية معالجة بيانات الرد الخاصة بك.